Está en la página 1de 642

00_FRONT MATTER.

indd 2 3/7/09 11:59:21 AM


MICRO-
ECONOMÍA
INtERMEdIA

00_FRONT MATTER.indd 1 3/7/09 11:59:20 AM


00_FRONT MATTER.indd 2 3/7/09 11:59:21 AM
MICRO-
ECONOMÍA
INtERMEdIA
ANálIsIs ECONóMICO
y COMpORtAMIENtO
Séptima edición

ROBERT H. FRANK
Cornell University

Revisión técnica:
Carlos Blanco Huitrón
Instituto Tecnológico y de Estudios Superiores de Monterrey,
Campus Ciudad de México

Rocío Román Collado Manuel Luis Pazos Casado


Departamento de Teoría económica Departamento de Teoría económica
y Economía política, y Economía política,
Universidad de Sevilla, España Universidad de Sevilla, España

MÉXICO • BOGOTÁ • BUENOS AIRES • CARACAS • GUATEMALA • MADRID


NUEVA YORK • SAN JUAN • SANTIAGO • SÃO PAULO • AUCKLAND
LONDRES • MILÁN • MONTREAL • NUEVA DELHI • SAN FRANCISCO
SINGAPUR • SAN LUIS • SIDNEY • TORONTO

00_FRONT MATTER.indd 3 3/7/09 11:59:22 AM


Director Higher Education: Miguel Ángel Toledo Castellanos
Director editorial: Ricardo Alejandro del Bosque Alayón
Editor sponsor: Jesús Mares Chacón
Coordinadora editorial: Marcela I. Rocha Martínez
Editor de desarrollo: Edmundo Carlos Zúñiga Gutiérrez
Supervisor de producción: Zeferino García García
Traductores: Martha Elsa Mauri Hernández, María del Carmen Enriqueta Hano Roa, Raúl Arrioja Juárez

MiCRoEConoMía inTERMEDia.
anáLiSiS EConóMiCo y CoMPoRTaMiEnTo
Séptima edición

Prohibida la reproducción total o parcial de esta obra,


por cualquier medio, sin la autorización escrita del editor.

DERECHOS RESERVADOS © 2009 respecto a la séptima edición en español por


McGRAW-HILL/INTERAMERICANA EDITORES, S.A. DE C.V.
A Subsidiary of The McGraw-Hill Companies, Inc.
Corporativo Punta Santa Fe
Prolongación Paseo de la Reforma 1015, Torre A
Piso 17, Colonia Desarrollo Santa Fe,
Delegación Álvaro Obregón
C.P. 01376, México, D. F.
Miembro de la Cámara Nacional de la Industria Editorial Mexicana, Reg. Núm. 736

iSBn: 978-970-10-7274-5

Traducido de la séptima edición de: MICROECONOMICS AND BEHAVIOR,


McGraw-Hill/Irwin. Copyright © MMVIII by The McGraw-Hill Companies Inc.
All rights reserved. Previous editions 2006, 2003, 2000, 1997, 1994 y 1991.

ISBN 0-07-337573-X

0123456789 08765432109

Impreso en México Printed in Mexico

00_FRONT MATTER.indd 4 3/7/09 11:59:24 AM


Para Dana

00_FRONT MATTER.indd 5 3/7/09 11:59:24 AM


00_FRONT MATTER.indd 6 3/7/09 11:59:24 AM
ACERCA dEl AutOR

Robert H. Frank es profesor de administración y de economía en la


Escuela Johnson de Graduados en Administración ( Johnson Gra-
duate School of Management) de la Universidad de Cornell, donde
también ha sido docente de principios de microeconomía en el Co-
llege of Arts and Sciences. Su columna “Economic Scene” se publica
mensualmente en The New York Times. Tras haber obtenido el grado
de Bachiller en Ciencias (B.S.) en el Georgia Tech, estuvo durante
dos años como voluntario de los Cuerpos de Paz en el Nepal rural
enseñando matemáticas y ciencias. Después de obtener el grado de
maestría (M.A.) en estadística y de doctor (Ph.D.) en economía en
la Universidad de Berkeley, California, inició su carrera docente de
la Universidad de Cornell. Durante sus ausencias con licencia de la
Universidad de Cornell, ha sido economista jefe del Civil Aeronautics Board, Fellow del Center for
Advanced Study in the Behavioral Sciences y profesor de civilización americana en l’École des Hau-
tes Études en Sciences Sociales de París. Sus investigaciones se han centrado en los temas de rivalidad
y cooperación en la economía y comportamiento social. Algunos de sus libros acerca de estos temas
son: Choosing the right pond, What price the moral high ground?, Passions within reason y Falling behind,
que han sido traducidos a 11 idiomas. Otros de sus libros son The Economic Naturalist y Principles of
Economics, en los que comparte la autoría con Ben Bernanke. The Winner-Take-All Society, escrito en
colaboración con Philip Cook, obtuvo el premio Critics’ Choice Award y fue elegido como Libro
Notable del Año por The New York Times e incluido en la lista de los 10 mejores libros de Business Week
de 1995. Su Luxury Fever fue elegido para figurar en la lista de los mejores libros de Knight-Ridder de
1999. Es ex presidente de la Asociación Económica del Este. Compartió el Premio Leontief al Avance
de las Fronteras del Pensamiento Económico y ha sido distinguido con el Merrill Scholars Program
Outstanding Educator Citation. En la Johnson School fue distinguido con el Russell Distinguished
Teaching Award en 2004 y con el Apple Distinguished Teaching Award en 2005.

vii

00_FRONT MATTER.indd 7 3/7/09 11:59:24 AM


00_FRONT MATTER.indd 8 3/7/09 11:59:25 AM
pREfACIO

c
uando escribí Microeconomía intermedia, mi objetivo fue ofrecer un libro intelec-
tualmente interesante y que al mismo tiempo fuera accesible y atractivo para
los estudiantes. Lo más común en este mercado ha sido hacer hincapié en una
u otra de estas dimensiones. Por ejemplo, algunos libros han obtenido buenos resultados
sacrificando el rigor en favor de hacer un libro amigable para el usuario. Pero aunque a
veces tales libros resultan del agrado de los estudiantes, no logran prepararlos para cursos
más avanzados. Otros libros han tenido éxito sacrificando la accesibilidad en favor del
rigor, donde rigor, con demasiada frecuencia, significa algo más que densidad matemá-
tica. Estos cursos abruman a muchos estudiantes de licenciatura, y aun aquellos pocos
que se vuelven expertos en la solución de problemas de optimización matemática bien
planteados suelen quedar desconcertados ante cuestiones tomadas de contextos cotidia-
nos. Siempre he creído que un libro debe ser al mismo tiempo riguroso y amigable para
el usuario. Y, a juzgar por la amplia aceptación de Microeconomía intermedia, parece que
muchos de ustedes están de acuerdo.
Escribí este libro con la convicción de que enseñar a utilizar la intuición y manejar
las herramientas técnicas son cosas complementarias, no sustitutas. Los estudiantes que
únicamente aprenden a utilizar las herramientas técnicas rara vez logran desarrollar un
verdadero gusto por nuestra disciplina; y todavía más raro es que adquieran esa especial
manera de pensar a la que llamamos “pensar como un economista”. En cambio, los es-
tudiantes que desarrollan una intuición para la economía se ven estimulados para pensar
más profundamente acerca de las herramientas técnicas que estudian y a hallar maneras
más interesantes de aplicarlas. Y lo que es más importante, estos estudiantes terminan
teniendo gusto por la economía.
Microeconomía intermedia desarrolla, con paciencia y atención a los detalles, las he-
rramientas analíticas fundamentales. Al mismo tiempo, se insertan estas herramientas
en una variedad de ejemplos y aplicaciones para ilustrar el poder y la versatilidad de la
manera económica de pensar.

NATuRAlismO EcONómicO
A lo largo de más de 35 años de enseñanza no he encontrado nada más efectivo para desa-
rrollar la intuición que entrenar a los estudiantes para que se conviertan en “naturalistas
económicos”.
Estudiar biología hace que las personas aprendan a observar y maravillarse con
muchos detalles de la vida que, de otra manera, les hubieran pasado desapercibidos. De
manera similar, estudiar microeconomía permite a los estudiantes ver con otros ojos de-
talles habituales de la vida cotidiana. A lo largo de todo el texto he tratado de desarrollar
la intuición mediante ejemplos y aplicaciones tomadas de la vida diaria. Microeconomía
intermedia enseña al estudiante a ver cada aspecto del paisaje creado por el hombre como
reflejo de un cálculo, implícito o explícito, costo-beneficio.
Para ilustrarlo, un naturalista económico es alguien que se pregunta por qué el admi-
nistrador del departamento de economía quedó encantado cuando empecé a poner mis
apuntes de clase en la intranet de la universidad, mientras que esto mismo incomodó al
secretario de la escuela de administración, donde también soy docente. Aproximadamen-
te una semana después de haber empezado el trimestre, recibí una carta urgente de este

ix

00_FRONT MATTER.indd 9 3/7/09 11:59:25 AM


x prefacio

secretario diciéndome que a partir de hoy debía hacer copias de mis notas de clase y distri-
buirlas gratuitamente a los estudiantes. Ninguna instrucción similar recibí del administrador
del departamento de economía. Cuando solicité una aclaración, el secretario de la escuela de
administración me dijo que los alumnos habían estado bajando mis notas e imprimiéndolas
en los laboratorios de computación de la escuela a un costo de 5 centavos por página, que
era mucho más de los 1.25 centavos que cobraba el centro de copiado de la escuela. Bueno.
Pero entonces, ¿por qué al administrador del departamento de economía no le preocupaba
este mismo problema? (Cuando le pregunté si quería que distribuyera copias de mis notas,
respondió: “¡No se le ocurra!”)
Pronto puede descubrir que estos tan diversos puntos de vista no tenían nada que ver
con las diferencias entre las culturas de estas dos unidades. Se debían a una pequeña pero
importante diferencia de origen económico: en la escuela de administración, el mismo admi-
nistrador paga las impresiones tanto en los laboratorios de computación como en el centro de
copiado. Por otro lado, el administrador del departamento de economía únicamente paga las
impresiones de la copiadora del departamento. Cuando los estudiantes de economía impri-
men mis notas de clase bajadas de la red en los laboratorios de computación del campus, en el
departamento de artes, la cuenta va directamente a la universidad. Desde el punto de vista del
departamento de economía, estas copias son gratis.
Año con año, las tareas más valiosas de mis cursos son los dos pequeños ensayos en los
que les pido a los estudiantes que hablen de sus esfuerzos por convertirse en naturalistas eco-
nómicos. Su tarea consiste en usar los principios de la microeconomía para responder una pre-
gunta que haya surgido de una observación personal. En los últimos trimestres los estudiantes
han tratado de resolver preguntas como: ¿Por qué los teclados de los cajeros automáticos tie-
nen puntos Braille? ¿Por qué las modelos femeninas ganan más que los modelos masculinos?
¿Por qué las novias gastan tanto dinero en el vestido de novia mientras que los novios rentar
un smoking (a pesar de que los novios tienen probabilidad de poder usar sus smokings en
muchas otras ocasiones y las novias nunca más usarán su vestido de novia)? ¿Por qué en los
automóviles se exigen asientos de seguridad para los niños pero en los viajes en avión no? ¿Por
qué las aerolíneas venden más caros los boletos de avión a los pasajeros de último minuto,
mientras que en los teatros de Broadway ocurre exactamente lo contrario?
Lo bonito de estas tareas no es solamente que los estudiantes disfrutan escribiendo es-
tos ensayos, sino que además pocos logran terminar su ensayo sin convertirse en naturalistas
económicos para el resto de su vida. Para aquellos que deseen saber más acerca de estas tareas
pueden visitar Authors@google series here: www.youtube.com/watch?v=QalNVxeIKEE.

ATENcióN EspEciAl A lA sOlucióN


dE pROBlEmAs
La mayoría de los economistas estarán de acuerdo en que un paso muy importante en el
aprendizaje de la teoría del precio es la solución de problemas. Microeconomía intermedia, más
que cualesquiera de los demás libros actualmente en el mercado, prepara a los estudiantes
para la solución de los problemas al final de cada capítulo, conduciéndolos por una serie de
ejemplos y ejercicios cuidadosamente elaborados. Debido a que la mayoría de los ejemplos
y de los ejercicios han sido tomados de contextos familiares, y a que los estudiantes se inte-
resan más fácilmente por cosas concretas que por cosas abstractas, este método ha resultado
eficiente. Cuando no se cuenta con este trabajo preliminar, muchos estudiantes llegan a los
problemas al final del capítulo con poca o ninguna idea de cómo proceder.

00_FRONT MATTER.indd 10 3/7/09 11:59:25 AM


prefacio xi

TRATAmiENTO ópTimO dE lOs TEmAs


En la evolución de Microeconomía intermedia, un principio guía ha sido hacer hincapié sobre los temas
en proporción directa, tanto de su importancia como de la dificultad de los estudiantes para domi-
narlos. Debido a que el modelo básico de la elección racional es la piedra angular para gran parte de
lo que ve más adelante en el curso, he dedicado a su desarrollo considerablemente más atención de
la que se da a otros libros semejantes. También he dejado espacio adicional para la elasticidad y sus
aplicaciones a la teoría de la demanda, así como para la distinción marginal media en la teoría de la
producción.
Para descubrir cuáles son los temas más difíciles de dominar he empleado la investigación en
economía conductual que identifica las desviaciones sistemáticas de lo que prescribe el modelo de la
elección racional. Por ejemplo, mientras este modelo dice que las personas racionales ignorarán los
costos ocultos/no recuperables/hundidos, muchas personas los toman muy en consideración. (Al-
guien que recibe de regalo un par de zapatos caros pero dolorosamente apretados, es mucho menos
probable que los use que alguien que gastó $400 de su bolsillo para adquirirlos.) Especialmente en
los capítulos sobre comportamiento del consumidor, trato de atraer la atención de los estudiantes
hacia situaciones en las que probablemente ellos mismos hagan una elección irracional. Dado que
los estudiantes disponen de recursos limitados, es útil centrarse precisamente en problemas en los
que el conocimiento de la teoría del precio pueda ser de utilidad.
Es natural preguntarse si analizar ejemplos de elección irracional no confunde a los estudiantes
cuando lo que están tratando de dominar son los detalles del modelo de elección racional. Aunque
es una pregunta razonable, mi experiencia ha sido exactamente lo contrario. Estos ejemplos en rea-
lidad subrayan el mensaje normativo de la teoría tradicional. Los estudiantes que se encuentran con
estos ejemplos invariablemente obtienen una comprensión más profunda de los principios teóricos
en cuestión. Incluso, con frecuencia parecieran estar secretamente orgullosos de poder ver a través
de los errores de juicio que hacen muchos de los consumidores. Aquellos docentes interesados en co-
nocer de manera detallada el efecto de las limitaciones cognitivas sobre la conducta del consumidor
encontrarán todo un capítulo dedicado a este tema. Cuando apareció en 1990 la primera edición de
Microeconomía intermedia, en la profesión de la economía muchos se mostraban escépticos respecto
al emergente campo de la economía conductual. Pero como ha sido demostrado por el economista
Matthew Rabin de U.C. Berkeley, quien recibió el John Bates Clark Award (otorgado cada dos años
por la American Economics Association al más destacado economista estadounidense menor de 40
años) en 2001, y por Daniel Kahneman, quien obtuvo el Premio Nobel de Economía en 2002, el
enfoque conductual es actualmente parte de las principales corrientes microeconómicas.

uNA cONcEpcióN más AmpliA


dEl iNTERés pERsONAl
Otro de mis objetivos ha sido incorporar un concepto más amplio de las preferencias en el modelo
de la elección individual. La mayoría de los libros mencionan desde el principio que el modelo de
la elección racional considera los gustos de la gente como dados, ya sean altruistas o masoquistas,
o que se preocupen exclusivamente por sus limitados intereses materiales. Es fácil ver por qué la
investigación económica ha conseguido sus mayores triunfos basándose en esta interpretación de
la motivación humana. Por ejemplo, esta interpretación explica por qué Ford no dio continuidad a la
producción de su SUV Excursion de 7 500 libras ante el aumento del precio de la gasolina; y por qué
en los departamentos que tienen medidores propios de consumo (de agua, luz, etc.), los termostatos
suelen colocarse más abajo.

00_FRONT MATTER.indd 11 3/7/09 11:59:26 AM


xii prefacio

Y luego, cuando los estudiantes están sumamente atentos, nuestra caricatura del homo
economicus aparece, en clara discrepancia con mucho de lo que conocemos de la conducta
humana. La gente vota en elecciones presidenciales. Da donativos anónimos a las estaciones
de televisión pública y a la beneficencia privada. Dona médula ósea a desconocidos que pa-
decen leucemia. Asume problemas y gastos con objeto de que se haga justicia, aun cuando
el daño originado no pueda ser reparado. Poniendo en riesgo sus vidas, rescata personas de
edificios en llamas o salta a ríos congelados para salvar la vida de alguien a punto de ahogarse.
Los soldados se arrojan desde lo alto para salvar a sus camaradas. Visto a través del lente de
la teoría del interés personal, sobre la que se hace hincapié en la mayoría de los libros, tales
comportamientos son el equivalente humano a que los planetas tuvieran órbitas cuadradas.
En realidad, muchos estudiantes se ven sumamente desconcertados por nuestro modelo del
interés personal, al que encuentran limitado y miserable.
Microeconomía intermedia reconoce, por supuesto, la importancia que la motivación del
interés personal tiene en muchos contextos. Pero dedica todo un capítulo al papel que de-
sempeñan los desinteresados en las transacciones sociales y económicas. Empleando teoría
elemental de juegos, este capítulo identifica circunstancias en las que las personas con estas
motivaciones tienen una ventaja competitiva sobre los oportunistas puros. Se muestra, por
ejemplo, que las personas que tienen predisposición a la cooperación suelen resolver el pro-
blema del prisionero y otros problemas de cooperación de una manera que las personas que
sólo se preocupan por los intereses personales no pueden hacerlo.
Los modelos teóricos sobre la naturaleza humana son importantes no porque moldean
nuestras expectativas sobre el comportamiento de los demás. La economía es la ciencia social
que más fuertemente se identifica con el modelo del interés personal de la conducta humana.
¿No tiñe este modelo nuestras expectativas sobre los demás e incluso nuestro propio compor-
tamiento? Cuando los psicólogos de la Universidad de Cornell, Tom Gilovich, Dennis Regan,
y yo, investigamos esto, encontramos numerosos indicios de que los economistas muestran
una mayor tendencia a comportarse de manera oportunista ante los dilemas sociales.1 Por
ejemplo, la posibilidad de que un académico economista reporte que no da dinero a ninguna
organización de beneficencia privada es más del doble en comparación con los miembros de
otras disciplinas investigadas. Además, en un experimento encontramos que la posibilidad
de que un estudiante con especialidad en economía fuera delator en un breve juego del dile-
ma de los prisioneros, era más del doble que la de un estudiante cuya especialidad no fuera
economía. Esta diferencia no era meramente el reflejo del hecho de que las personas que
eligen especializarse en economía sean, desde un principio, más oportunistas. Encontramos,
por ejemplo, que la diferencia en la tasa de delatores aumentaba a medida que los estudiantes
tenían más tiempo estudiando economía. Respuestas a cuestionarios también indicaron que
los estudiantes de primer año, que cursar su primer curso de microeconomía, era más proba-
ble que esperaran una conducta oportunista de los demás al final del trimestre que al principio
de éste.
Por lo tanto, existen razones para pensar que, al hacer tanto hincapié en la limitada mo-
tivación del interés personal, los economistas hayan dañado la inclinación de los estudiantes
al comportamiento cooperativo. Lo irónico es que, como trato de mostrar en el capítulo 7, la
lógica interna del modelo económico nunca predijo tal conducta rigurosamente interesada en
el provecho personal.

1
Ver R. H. Frank, T. D. Gilovich y D. T. Regan, “Does Studing Economics Inhibits Cooperation?”, Journal of Economic
Perspectives, primavera de 1993.

00_FRONT MATTER.indd 12 3/7/09 11:59:26 AM


prefacio xiii

OTRAs cARAcTERísTicAs pEdAgógicAs


A diferencia de la mayoría de los textos de nivel intermedio, Microeconomía intermedia no
contiene recuadros con aplicaciones que distraen la atención del estudiante del tema que se
está tratando. Las aplicaciones y los ejemplos están totalmente integrados al texto. Además,
muchos de ellos tienen la ventaja de ser tomados de experiencias con las que los estudiantes
pueden estar directamente relacionados. En cada capítulo, la introducción y el resumen son
otra característica innovadora de Microeconomía intermedia. La mayor parte de los capítulos
empiezan con una anécdota que plantea un problema o una pregunta a la que los estudian-
tes podrán responder con el material que se desarrolla en el capítulo. Estas introducciones
han resultado de gran ayuda, especialmente para aquellos estudiantes para los que el comien-
zo es el paso más difícil. En la mayoría de los textos, los resúmenes de los capítulos consisten
en breves listas de los temas vistos. En cambio, en Microeconomía intermedia estos resúmenes
están escritos en forma narrativa, sintetizando de manera cuidadosa el material visto en el
capítulo.
Todos los capítulos concluyen con una selección de problemas que van desde problemas
rutinarios hasta problemas muy exigentes. Estos problemas han sido probados en clase para
comprobar su precisión y eficacia como ayuda para el dominio de los conceptos más impor-
tantes del capítulo.
Las respuestas a todos los ejercicios que aparecen dentro de un capítulo se encuentran al
final de ese capítulo. Los problemas al final del capítulo presentan variaciones y extensiones
de estos ejercicios, lo que permite al estudiante abordar estos problemas con confianza. En el
manual para el docente se presentan respuestas detalladas a todos los problemas al final del
capítulo.

cAmBiOs EN lA sépTimA EdicióN


Los extraordinarios dibujos que aparecen en los ejemplos del Naturalista económico fueron
hechos por el conocido caricaturista neoyorkino Mick Stevens, en tinta negra. Así es como se
ven mejor, y así es como aparecen en el libro.
Muchos textos universitarios son demasiado largos. La solución más sencilla sería publi-
car libros más cortos. El problema de un libro más corto es que puede que no contenga los
temas favoritos de muchos de los potenciales lectores. La mayoría de los libros de éxito son
grandes desde un principio, y en cada nueva edición se vuelven invariablemente más grandes.
Después de todo, es necesario incluir los nuevos avances y es casi imposible eliminar el mate-
rial ya existente, que los lectores están acostumbrados a usar.
Sin embargo, los editores de McGraw-Hill y yo coincidimos en que sería un error de-
jarnos paralizar totalmente por este poderoso sesgo hacia el statu quo. De esta manera, en
esta edición he continuado con el proceso, iniciado en la sexta edición, de eliminar bloques
considerables de material. Así, desaparecieron, por ejemplo, los análisis sobre la evolución
de los métodos para secarse las manos en los baños públicos, la elasticidad de arco, el marco
hedonista y los mercados contestables.
De esta edición fue eliminada también la aplicación del modelo de Hotelling sobre la
competencia espacial por las posiciones tomadas por los candidatos a la presidencia de los dos
principales partidos estadounidenses. Ese análisis, que predice que los candidatos se acercarán
paulatinamente al centro político, ya no parece describir el actual escenario altamente parti-
dista. La principal eliminación del libro es una que en realidad no es una eliminación sino una
reubicación. En amplias encuestas a los lectores durante las ediciones anteriores, se encontró
que la porción de lectores de la teoría general del equilibrio (capítulo 16 de la sexta edición)
era cada vez más escasa. Si incluir este capítulo en la edición impresa del libro no hubiera te-

00_FRONT MATTER.indd 13 3/7/09 11:59:26 AM


xiv prefacio

nido ningún costo, se habría conservado aquí. Pero dado que los costos de producción crecen
continuamente, conservar ese material hubiera significado dejar fuera otro material más va-
lioso para la mayoría de los docentes. Así, el capítulo sobre equilibrio general no aparece en la
versión impresa de esta edición. Pero para quienes deseen continuar empleando este capítulo
pueden bajar una versión actualizada en la siguiente dirección: www.mhhe.com/frank7e.
Hay algunas cosas que fueron agregadas. En el capítulo 4 la lista de las estimaciones em-
píricas de la elasticidad del precio de la demanda fue ampliada; en el capítulo 5 también se
amplió la explicación del efecto que tiene el impuesto a la gasolina sobre el uso del automóvil,
cuya renta es devuelta a los consumidores mediante reducciones en otros impuestos. En el
capítulo 6 se presenta un nuevo ejemplo que ilustra cómo un abogado de un demandante,
insensible al riesgo, fija el precio de sus servicios para maximizar su atractivo ante potenciales
clientes conservadores.
El capítulo 8 contiene ahora un análisis del efecto de errores en el pronóstico de las pro-
pias preferencias futuras sobre las decisiones de gasto del consumidor. Siguiendo la sugerencia
del profesor Fred Moseley de Mount Holyoke College, en el capítulo 9 ahora trato los produc-
tos intermedios de manera más parecida al resto de los factores de producción, conforme el
uso adoptado por la mayoría de los autores de otros libros.
Mediante una cuidadosa lectura a esta edición, las personas familiarizadas con ediciones
anteriores notarán también que continúo con mi intento de simplificar el texto y hacerlo más
conciso. Para esto ha sido de gran ayuda la experiencia de ajustarme a un número estricto de
palabras al escribir mi columna para el New York Times durante estos años. Pero lo más breve
no es siempre lo mejor. Así, por ejemplo, en el capítulo 13 de esta edición he ampliado nota-
blemente el análisis del equilibrio de Nash, un concepto desafiante para muchos estudiantes.
Aquellos familiarizados con ediciones anteriores notarán también que en esta edición
he tratado de atraer más la atención hacia los ejemplos del naturalista económico aumen-
tando el tamaño de las caricaturas de Mick Stevens que acompañan a algunos de ellos. ¿Por
qué incurrir en estos gastos adicionales? De nuevo, estoy convencido de que el servicio más
importante que podemos prestar a nuestros estudiantes de microeconomía es inculcarles la
inclinación a ver el mundo a su alrededor en términos económicos. Aprender economía es
como aprender a hablar otro idioma. Estudiar la gramática ayuda, pero la única manera de
hablarlo es hablar mucho. El enfoque del naturalista económico es el recurso más efectivo
que he encontrado para hacer que los estudiantes “hablen economía”. Como parece que las
caricaturas ayudan a que los estudiantes recuerden los ejemplos, los animan a contar historias
económicas. Muchos de mis estudiantes han contado de viajes a casa a mitad del semestre
en los que los ejemplos del naturalista económico vistos en clase se han convertido en los
temas principales de conversación en las comidas familiares. Una vez que los estudiantes se
dan cuenta de que son capaces de plantear y responder, ellos solos, interesantes preguntas
económicas, quedan entusiasmados. Empieza una trayectoria de por vida en la que el domi-
nio de los principios de la economía no sólo no disminuirá con los años una vez terminado el
curso, sino que se fortalecerá a medida que continúan afinando su fortaleza.
Esta séptima edición es, en mi opinión, la mejor a la fecha de Microeconomía intermedia.
Esta edición conserva los aspectos esenciales de las ediciones anteriores que han atraído a un
muy leal grupo de usuarios. Pero se ve mejor y es mejor en innumerables aspectos.

REcONOcimiENTOs
Quiero expresar mi sincero agradecimiento tanto a la administración como a mis editores
de McGraw-Hill por su excelente disposición para emprender acciones que van en contra
de las tendencias del mercado. Por supuesto, los mayores riesgos fueron corridos por Scott
Stratford, el editor de la primera edición. (Espero que esté satisfecho de que los avances sub-

00_FRONT MATTER.indd 14 3/7/09 11:59:27 AM


prefacio xv

siguientes dentro de la economía parezcan haber ratificado lo acertado de aquellas primeras


decisiones.) Pero también en esta edición, los sucesores de Scott, Douglas Reiner y Angela
Cimarolli, estuvieron dispuestos a innovar. Estoy muy agradecido por su entusiasta apoyo.
También quiero expresar mi agradecimiento a los muchos revisores que participaron en
este proyecto, tanto en esta edición como en las anteriores. Sus ideas y sus críticas han condu-
cido a mejoras demasiado numerosas para ser enumeradas. Espero que estén tan satisfechos
como yo con su contribución al producto final.

Ronald Adams Satyajit Ghosh


California State University, Chico University of Scranton
Susan L. Averett Adam Grossberg
Lafayette College Trinity College
Amitrajeet A. Batabyal Philip J. Grossman
Rochester Institute of Technology Saint Cloud State University
Jeff Bauer Susan He
University of Cincinnati–Clermont Washington State University
David Bernotas Steve Holland
University of Georgia Luther College
Tibor Besedes E. James Jennings
Louisiana State University Purdue University Calumet
Pinaki Bose Robert Jerome
University of Memphis James Madison University
James Cardon Sumit Joshi
Brigham Young University George Washington University
Phoebe Chan Woodrow W. Hughes, Jr.
University of Michigan–Flint Converse College
Ron Cheung Mark Kazarosian
Florida State University Stonehill College
Joy L. Clark Jonathan Lanning
Auburn University Montgomery Albion College
Ardeshir Dalal Marc Law
Northern Illinois University University of Vermont
Eric Dodge Christopher McHugh
Hanover College Tufts University
Robert R. Ebert Stanley McMillen
Baldwin-Wallace College Trinity College
Robert Fairlie Gyan Pradhan
University of California Westminster College
Fred Foldvary Farahmand Rezvani
Santa Clara University Montclair State University
David Garman Stephen Shmanske
Tufts University California State University, East Bay

00_FRONT MATTER.indd 15 3/7/09 11:59:27 AM


xvi prefacio

Mark Steckbeck Michael Visser


Hillsdale College Sonoma State University
Kurtis J. Swope Mark D. White
U.S. Naval Academy College of Staten Island/CUNY

Como siempre, cualquier comentario o sugerencia es bienvenido.


Robert H. Frank

00_FRONT MATTER.indd 16 3/7/09 11:59:27 AM


CONtENIdO bREvE

Parte 1 Introduccción
1 Pensar como un economista 3
2 La oferta y la demanda 25

Parte 2 teoría del comportamiento


del consumidor
3 La elección racional del consumidor 61
4 Demanda individual y del mercado 75
5 Aplicaciones de las teorías de la elección racional
y de la demanda 139
6 Economía de la información y elección
bajo incertidumbre 169
7 Explicación de los gustos: importancia del altruismo
y de otros comportamientos no egoístas 211
8 limitaciones cognitivas y comportamiento
del consumidor 237

Parte 3 La teoría de la empresa y la


estructura de mercado
9 Producción 263
10 Costos 297
11 Competencia perfecta 333
12 Monopolio 371
13 Competencia imperfecta: enfoque
de la teoría de juegos 413

Parte 4 Mercados de factores


14 Mano de obra 459
15 Capital 505

Parte 5 externalidades, bienes


públicos y bienestar social
16 Externalidades, derechos de propiedad
y el teorema de coase 535
17 Gobierno 567

Índice temático 597

00_FRONT MATTER.indd 17 3/7/09 11:59:28 AM


00_FRONT MATTER.indd 18 3/7/09 11:59:28 AM
CONtENIdO

Parte 1 Introduccción 1
Capítulo 1 Pensar como un economista 3
El método costo-beneficio en la toma de decisiones 4
Ejemplo 1.1 5
El papel de la teoría económica 5
Errores comunes en la toma de decisiones 6
Ejemplo 1.2 7
Ejemplo 1.3 7
Ejemplo 1.4 8
Ejemplo 1.5 9
Ejemplo 1.6 10
Ejemplo 1.7a 11
Ejemplo 1.7b 11
Ejemplo 1.8 12
Ejemplo 1.9 13
Gráfica de beneficio marginal y costo marginal 14
Ejemplo 1.10 15
La mano invisible 15
Ejemplo 1.11 16
¿Querrán los padres que su hija o hijo se case con un homo economicus? 17
El naturalista económico 17
el naturalista económico 1.1: ¿por qué es tan mala la comida
de las aerolíneas? 18
el naturalista económico 1.2: ¿por qué las transmisiones manuales
tienen cinco velocidades y las automáticas sólo cuatro? 19
Cuestiones positivas y cuestiones normativas 19
Microeconomía y macroeconomía 19
Resumen 20
Preguntas de repaso 20
Problemas 20
Respuestas a los ejercicios del capítulo 23

Capítulo 2 La oferta y la demanda 25


Visión preliminar del capítulo 25
Curvas de oferta y demanda 26
Equilibrio cantidad y precio 29
ajuste al equilibrio 30
algunas propiedades de bienestar del equilibrio desde
el punto de vista del bienestar 31
Los mercados libres y los pobres 32
Ejemplo 2.1 32
Ejemplo 2.2 34

00_FRONT MATTER.indd 19 3/7/09 11:59:28 AM


xx contenido

apoyos a los precios 36


Funciones de distribución y de racionamiento de los precios 37
Determinantes de la oferta y la demanda 37
Predicción y explicación de variaciones en el precio y la cantidad 40
el naturalista económico 2.1: ¿por qué los precios de algunos bienes, como
las manzanas, bajan en los meses de mayor consumo mientras que otros, como
las casas de campo a la orilla del mar, suben? 40
Ejemplo 2.3 41
álgebra de la oferta y la demanda 41
Resumen 42
Preguntas de repaso 43
Problemas 44
Respuestas a los ejercicios del capítulo 45
Apéndice 2 ¿Cómo afectan los impuestos las cantidades y los precios de equilibrio? 47

Parte 2 teoría del comportamiento del consumidor 53


Capítulo 3 La elección racional del consumidor 55
Visión preliminar del capítulo 56
El conjunto de oportunidad o restricción de presupuesto 56
Ejemplo 3.1 61
Ejemplo 3.2 62
Preferencias del consumidor 63
El paquete más asequible 69
Ejemplo 3.3 72
Una aplicación del modelo de elección racional 73
Ejemplo 3.4 73
el naturalista económico 3.1: ¿por qué la gente suele dar regalos en especie
en lugar de dar dinero en efectivo? 75
Resumen 76
Preguntas de repaso 77
Problemas 77
Respuestas a los ejercicios del capítulo 80
Apéndice 3 Aplicación de la función de utilidad al problema del presupuesto del consumidor 83

Capítulo 4 Demanda individual y del mercado 95


Vista preliminar del capítulo 95
Efectos de la variación del precio 96
Efectos de la variación del ingreso 98
Efectos de una variación de precio sobre el ingreso y la sustitución 100
Ejemplo 4.1 104
Ejemplo 4.2 105
Sensibilidad del consumidor ante las variaciones de precio 106
Ejemplo 4.3 108
Demanda del mercado: agregación de las curvas de demanda
individuales 109
Ejemplo 4.4 109
Ejemplo 4.5 111
Elasticidad precio de la demanda 111
Ejemplo 4.6 118

00_FRONT MATTER.indd 20 3/7/09 11:59:29 AM


contenido xxi

Dependencia de la demanda del mercado con respecto al ingreso 120


Ejemplo 4.7 120
el naturalista ecónomico 4.1: ¿por qué en las últimas décadas ha habido un
cambio tan dramático en los aparatos para cocinar en el exterior? 123
aplicaciones: pronóstico de las tendencias económicas 124
Elasticidades precio cruzadas de la demanda 125
Resumen 126
Preguntas de repaso 127
Problemas 128
Respuestas a los ejercicios del capítulo 130
Apéndice 4 Temas adicionales sobre la teoría de la demanda 133

Capítulo 5 Aplicaciones de las teorías de la elección racional


y de la demanda 139
Visión preliminar del capítulo 140
Uso del modelo de la elección racional para responder
a cuestiones políticas 140
Excedente del consumidor 144
Ejemplo 5.1 145
el naturalista económico 5.1: ¿por qué algunos clubes de tenis cobran una
membresía anual además de la renta por hora de la cancha? 146
el naturalista económico 5.2: ¿por qué algunos parques de atracciones
cobran sólo una tarifa de entrada y ninguna cantidad adicional incluso por
atracciones para las que hay largas colas? 147
Comparaciones generales del bienestar económico 147
Ejemplo 5.2 148
Uso de la elasticidad-precio de la demanda 153
aplicación: la elasticidad-precio de la demanda del alcohol 154
El modelo de elección intertemporal 155
Ejemplo 5.3 159
Resumen 162
Preguntas de repaso 163
Problemas 163
Respuestas a los ejercicios del capítulo 165

Capítulo 6 Economía de la información y elección bajo incertidumbre 169


Visión preliminar del capítulo 170
Economía de la información 170
el naturalista económico 6.1: ¿por qué los automóviles usados
“seminuevos” se venden mucho más baratos que los nuevos? 175
el naturalista económico 6.2: ¿por qué la timidez es una cualidad
atractiva? 176
el naturalista económico 6.3: ¿por qué los habitantes de pueblos pequeños
gastan menos en su vestuario profesional que los de ciudades grandes? 178
Elección con incertidumbre 179
Ejemplo 6.1 181
Ejemplo 6.2 183
Ejemplo 6.3 184
Ejemplo 6.4 185
Ejemplo 6.5 186

00_FRONT MATTER.indd 21 3/7/09 11:59:29 AM


xxii contenido

Seguro contra malos resultados 187


Resumen 193
Preguntas de repaso 193
Problemas 194
Respuestas a los ejercicios del capítulo 196
Apéndice 6 Teoría de la búsqueda y la maldición del ganador 199

Capítulo 7 Explicación de los gustos: importancia del altruismo y de


otros comportamientos no egoístas 211
Visión preliminar del capítulo 212
Una aplicación del estándar del objetivo presente: las preferencias
altruistas 213
Ejemplo 7.1 214
Papel estratégico de las preferencias 214
El problema del compromiso 219
Ejemplo: el problema del engaño 220
Un sencillo experimento de reflexión 226
Los gustos no sólo pueden, sino que deben ser diferentes 228
el naturalista económico 7.1: ¿por qué vota la gente en las elecciones
presidenciales? 228
Ejemplo 7.2 231
La importancia de los gustos 232
Resumen 233
Preguntas de repaso 233
Problemas 233
Respuestas a los ejercicios del capítulo 234

Capítulo 8 Limitaciones cognitivas y comportamiento del consumidor 237


Visión preliminar del capítulo 238
Racionalidad limitada 238
La función asimétrica del valor 239
Costos irrecuperables 242
Costo desembolsado contra costos de oportunidad 242
Errores afectivos de predicción 243
Elección con incertidumbre 245
Heurísticas críticas y sesgos 247
el naturalista económico 8.1: ¿por qué el novato del año en el beisbol
suele tener una segunda temporada mediocre? 250
La psicofísica de la percepción 251
La dificultad real de decidir 252
el naturalista económico 8.2: ¿por qué los agentes inmobiliarios muestran
a los clientes dos casas casi idénticas, aun cuando una sea menos cara y esté
en mejores condiciones que la otra? 254
El escollo del autocontrol 255
Resumen 256
Preguntas de repaso 257
Problemas 257
Respuestas a los ejercicios del capítulo 259

00_FRONT MATTER.indd 22 3/7/09 11:59:29 AM


contenido xxiii

Parte 3 La teoría de la empresa y la estructura


de mercado 261
Capítulo 9 Producción 263
Visión preliminar del capítulo 263
La relación insumo-producto, o la función de producción 264
Producción a corto plazo 266
el naturalista económico 9.1: ¿por qué la gente de todo el mundo no
puede ser alimentada a partir de la cantidad de grano cultivada en una sola
maceta? 268
Ejemplo 9.1 273
Ejemplo 9.2 274
Ejemplo 9.3 275
Producción a largo plazo 275
Rendimientos de escala 278
el naturalista económico 9.2: ¿por qué los constructores utilizan marcos
prefabricados para techos pero no para muros? 279
Resumen 281
Preguntas de repaso 282
Problemas 282
Respuestas a los ejercicios del capítulo 284
Apéndice 9 Extensiones matemáticas de la teoría de la producción 287

Capítulo 10 Costos 297


Visión preliminar del capítulo 298
Costos a corto plazo 298
Ejemplo 10.1 301
Ejemplo 10.2 305
Ejemplo 10.3: 307
asignación de la producción entre dos procesos 308
Ejemplo 10.4 309
La relación entre MP, aP, MC y aVC 310
Costos a largo plazo 311
el naturalista económico 10.1: ¿por qué la grava se fabrica a mano en
nepal pero con máquina en estados Unidos? 314
el naturalista económico 10.2: ¿por qué los sindicatos apoyan las leyes del
salario mínimo con tal fuerza? 315
el naturalista económico 10.3: ¿por qué un fabricante de equipos de baño
pondría la imagen de una mosca en el centro de sus mingitorios de
cerámica? 316
Los costos a largo plazo y la estructura de la industria 319
La relación entre las curvas de costo a largo plazo y a corto plazo 321
Resumen 322
Preguntas de repaso 323
Problemas 323
Respuestas a los ejercicios del capítulo 325
Apéndice 10 Extensiones matemáticas de la teoría de costos 327

00_FRONT MATTER.indd 23 3/7/09 11:59:30 AM


xxiv contenido

Capítulo 11 Competencia perfecta 333


Visión preliminar del capítulo 333
La meta de la maximización de las utilidades 334
Ejemplo 11.1 335
Las cuatro condiciones de la competencia perfecta 337
La condición de corto plazo de la maximización de utilidades 338
oferta de la industria competitiva a corto plazo 343
Ejemplo 11.2 344
Equilibrio competitivo a corto plazo 344
Eficiencia del equilibrio competitivo a corto plazo 346
Excedente del productor 347
Ejemplo 11.3 349
ajustes a largo plazo 350
La mano invisible 353
aplicación: el costo de insumos extraordinarios 354
Curva de oferta a largo plazo en industrias competitivas 356
el naturalista ecónomico 11.1: ¿por qué las fotos a color tienen un precio
menor que las fotos en blanco y negro? 359
Elasticidad de la oferta 360
aplicación del modelo competitivo 361
el naturalista ecónomico 11.2: ¿por qué los camiones de 18 ruedas empezaron
a usar perfiles aerodinámicos a mediados de la década de 1970? 365
Resumen 366
Preguntas de repaso 367
Problemas 367
Respuestas a los ejercicios del capítulo 369

Capítulo 12 Monopolio 371


Visión preliminar del capítulo 371
Definición de monopolio 372
Cinco causas de los monopolios 373
El monopolista que maximiza utilidades 377
Ejemplo 12.1 383
Ejemplo 12.2 385
Un monopolista no tiene curva de oferta 388
ajustes a largo plazo 388
Discriminación de precios 389
Ejemplo 12.3 390
el naturalista ecónomico 12.1: ¿por qué algunos médicos y abogados hacen
descuentos a personas de bajos ingresos? 392
el naturalista ecónomico 12.2: ¿por qué los dueños de los cines ofrecen
descuento a los estudiantes en el boleto de entrada pero no en las
palomitas? 392
El monopolista con discriminación perfecta 393
Pérdida de eficiencia del monopolio 397
Política pública hacia un monopolio natural 398
Ejemplo 12.4 406
Resumen 407
Preguntas de repaso 408
Problemas 408
Respuestas a los ejercicios del capítulo 410

00_FRONT MATTER.indd 24 3/7/09 11:59:30 AM


contenido xxv

Capítulo 13 Competencia imperfecta: enfoque


de la teoría de juegos 413
Visión preliminar del capítulo 414
introducción a la teoría de juegos 414
el naturalista ecónomico 13.1: ¿por qué las tabacaleras se anuncian
“demasiado”? 417
el naturalista ecónomico 13.2: ¿por qué una compañía hace una inversión
que sabe que no usará nunca? 424
el naturalista ecónomico 13.3: ¿para qué una empresa construye una
fábrica con más capacidad de la que podría necesitar? 426
Modelos específicos de oligopolios 426
Ejemplo 13.1 429
Ejemplo 13.2 430
Ejemplo 13.3 431
Competencia cuando hay rendimientos crecientes de escala 434
Competencia monopolística 435
interpretación espacial de la competencia monopolística 440
el naturalista ecónomico 13.4: ¿por qué en la mayoría de las ciudades
ahora hay menos tiendas de abarrotes que en 1930? ¿por qué los barrios
residenciales de nueva York tienen más tiendas de abarrotes que los
barrios residenciales de Los Ángeles? 445
nota histórica: los vendedores de hot dogs de Hotelling 449
Preferencias de los consumidores y publicidad 450
Resumen 452
Preguntas de repaso 452
Problemas 453
Respuestas a los ejercicios del capítulo 454

Parte 4 Mercados de factores 457

Capítulo 14 Mano de obra 459


Visión preliminar del capítulo 460
Demanda de mano de obra a corto plazo de la empresa
perfectamente competitiva 460
La demanda de mano de obra a largo plazo de la empresa
perfectamente competitiva 462
Curva de demanda del mercado para la mano de obra 462
Demanda de mano de obra de un competidor imperfecto 463
oferta de mano de obra 464
Ejemplo 14.1 466
el naturalista ecónomico 14.1: ¿por qué es tan difícil encontrar un taxi
cuando llueve? 467
Ejemplo 14.2 467
¿Es el tiempo libre un bien Giffen? 469
Reacción del que no es economista ante el modelo de la oferta de
mano de obra 469
Curva de oferta del mercado 470
Ejemplo 14.3 470
Monopsonio 472
Legislación del salario mínimo 476

00_FRONT MATTER.indd 25 3/7/09 11:59:31 AM


xxvi contenido

Sindicatos 478
Discriminación en el mercado laboral 480
Discriminación estadística 482
Estructura salarial interna 485
Mercados en que el ganador se queda con todo 488
Resumen 489
Preguntas de repaso 490
Problemas 490
Respuestas a los ejercicios del capítulo 493
Problemas 503
Apéndice 14 La economía de la seguridad laboral 497

Capítulo 15 Capital 505


Visión preliminar del capítulo 506
Capital financiero y capital real 506
Demanda de capital real 506
Relación entre tasa de la renta y tasa de interés 507
Criterio para comprar un bien de capital 508
Determinación de la tasa de interés 508
Tasas de interés nominales y reales 510
Mercado de acciones y bonos 510
el naturalista ecónomico 13.4: ¿por qué tener una acción de un
monopolio no es mejor que tener una acción de una empresa perfectamente
competitiva? 514
La anomalía del boletín de inversiones 514
Política fiscal y mercado de capital 516
Renta económica 517
Precios de consumo de carga máxima 518
Recursos no renovables como insumos de la producción 519
Resumen 521
Preguntas de repaso 522
Problemas 522
Respuestas a los ejercicios del capítulo 523
Apéndice 15 Examen más detallado de la asignación de recursos no renovables 525

Parte 5 externalidades, bienes públicos y bienestar


social 533

Capítulo 16 Externalidades, derechos de propiedad y el teorema


de Coase 535
Visión preliminar del capítulo 536
naturaleza recíproca de las externalidades 536
Ejemplo 16.1 537
Ejemplo 16.2 538
Ejemplo 16.3 538
Ejemplo 16.4 539
Ejemplo 16.5 540
Ejemplo 16.6 541

00_FRONT MATTER.indd 26 3/7/09 11:59:31 AM


contenido xxvii

aplicación: efectos externos de las plantas de energía nuclear 542


Derechos de propiedad 543
el naturalista ecónomico 16.1: ¿por qué la ley permite que las líneas aéreas
vuelen sobre propiedad privada sin autorización? 543
el naturalista ecónomico 16.2: ¿por qué la ley de allanamiento no se
aplica a la propiedad costera? 544
el naturalista ecónomico 16.3: ¿por qué las leyes de propiedad se
suspenden durante las tormentas? 545
el naturalista ecónomico 16.4: ¿por qué los límites de altura
para las edificaciones varían según la ciudad? 545
Ejemplo 16.7 546
Externalidades, eficiencia y libertad de expresión 550
Reglas para fumar, públicas y privadas 550
Ejemplo 16.8 551
Externalidades positivas 552
Externalidades por posición 552
El gravamen de las externalidades 555
Ejemplo 16.9 557
Resumen 560
Preguntas de repaso 560
Problemas 560
Respuestas a los ejercicios del capítulo 566

Capítulo 17 Gobierno 567


Visión preliminar del capítulo 568
Bienes públicos 568
Ejemplo 17.1 573
Elección pública 576
Ejemplo 17.2 580
Ejemplo 17.3 582
Distribución del ingreso 582
Resumen 591
Preguntas de repaso 592
Problemas 593
Respuestas a los ejercicios del capítulo 595

Índice temático 597

00_FRONT MATTER.indd 27 3/7/09 11:59:32 AM


00_FRONT MATTER.indd 28 3/7/09 11:59:32 AM
Pa rt e

L A O F E INTRODuccIóN
RTA Y L A D E M A N DA
1
en estos dos primeros capítulos se repasa la materia esencial de un curso de
introducción a la microeconomía. en el capítulo 1 se aplican los principios del
análisis costo-beneficio* a diversas decisiones (elecciones) de la vida cotidiana.
Su objetivo es que el autor percibe de manera intuitiva lo que significa “pensar
como un economista”.
en el capítulo 2 se desarrolla el análisis básico de la oferta y la demanda, el
instrumento analítico para explicar los precios y las cantidades de bienes que
se intercambian en los mercados. Se verá que aunque los mercados sin regula-
ción puedan producir consecuencias no deseadas, a menudo generan los mejo-
res resultados posibles bajo ciertas circunstancias. en contraste, los esfuerzos
del gobierno para ayudar a los pobres mediante la regulación de los precios y
las cantidades suelen producir efectos colaterales indeseables. Como también
se verá, una mejor manera de ayudar a los pobres es mediante programas que
incrementen sus ingresos.**

* En otros países de habla hispana, se denomina coste.


** En algunos países se les conoce como rentas.

01_CHAPTER 1.indd 1 3/6/09 6:40:45 PM


01_CHAPTER 1.indd 2 3/6/09 6:40:46 PM
CaPítulo

1
Pensar como un
economista

G
ran parte de la microeconomía consiste en el estudio de cómo elige la gente en condicio-
nes de escasez. Muchas personas reaccionan ante esta descripción diciendo que es un tema
de poca relevancia en los países desarrollados, donde la escasez material es, en gran parte,
cosa del pasado.
Sin embargo, esta visión de la escasez es muy limitada porque siempre hay recursos esenciales
que no están disponibles en cantidad suficiente. Cuando Aristóteles Onassis murió tenía una fortuna
de varios miles de millones de dólares. Tenía más dinero del que podía gastar y lo utilizaba en adqui-
rir cosas como, por ejemplo, reposapiés de marfil de ballena finamente trabajados para los bancos del
bar de su yate. Onassis era víctima de miastenia gravis, un padecimiento neurológico degenerativo
y progresivo. Para él, la escasez no era de dinero sino de tiempo, de energía y de la capacidad física
necesaria para realizar las actividades cotidianas.
El tiempo es un recurso escaso para todo el mundo, no sólo para los enfermos terminales. Por
ejemplo, al decidir qué película ver, es el tiempo y no el precio de la entrada lo determinante para la
mayoría. Cuando se tienen sólo unas cuantas noches libres al mes, ver una película implica dejar de
ver otra o no cenar con los amigos.
El tiempo y el dinero no son los únicos recursos esenciales escasos. Considere la elección eco-
nómica a la que se enfrenta cuando un amigo lo invita a un almuerzo tipo buffet. Tendrá que decidir
qué servirse en su plato. Aun cuando usted no sea rico, el dinero no será el problema, ya que podrá
comer todo lo que quiera sin tener que pagar ni un centavo. El tiempo tampoco será un obstáculo
porque tendrá toda la tarde para disfrutarla en compañía de su amigo. Aquí el recurso escaso es la
capacidad de su estómago. Delante de usted están sus platos favoritos y debe decidir qué comer y en
qué cantidad. Comer otro waffle implicará comer menos huevos revueltos. El hecho de que aquí el
dinero no cambie de dueño no hace que su elección deje de tener un carácter económico.


01_CHAPTER 1.indd 3 3/6/09 6:40:46 PM


 CaPítulo 1 PenSar Como un eConomiSta

© the new Yorker Collection 1976 Donald reilly from cartoonbank.com. all rights reserved.
“Qué bien, todo es perfecto, pero estar en un lugar en el que no hay presupuesto alguno
es una sensación un poco rara.”

En toda elección interviene el problema de la escasez. Algunas veces la restricción más importan-
te tendrá que ver con el dinero, pero no siempre. Afrontar la escasez es una característica inherente
a la condición humana. En efecto, si no fuera por ella la vida quedaría despojada de gran parte de su
intensidad. Alguien que tuviera una vida y unos recursos materiales ilimitados, difícilmente tendría
que tomar una decisión de importancia.
En este capítulo se examinan algunos de los principios básicos de la teoría microeconómica y
se ve cómo un economista puede aplicarlos a una gran variedad de elecciones relacionadas con la
escasez. En capítulos posteriores se desarrolla con más formalidad esta teoría. Por ahora, el único
objetivo es tener una impresión intuitiva de esa mentalidad particular conocida como “pensar como
economista”. Y la mejor manera de lograrlo es resolviendo una serie de problemas de la vida coti-
diana.

EL MÉTODO COSTO-BENEFICIO
EN LA TOMA DE DECISIONES
Muchas de las elecciones que estudian los economistas pueden plantearse con la pregunta siguiente:
¿Debo hacer la actividad x?
En el caso de la elección a la que se enfrenta un cinéfilo, “¿…hacer la actividad x?”, podría ser
“¿…veré la película Casablanca esta noche?” En el caso de la persona que va al almuerzo tipo buffet,
“¿…me comeré otro waffle?” Los economistas responden a tales preguntas comparando los costos y
los beneficios de realizar la actividad en cuestión. La regla de decisión que se utiliza es muy sencilla.
Si C(x) denota el costo de hacer x y B(x) el beneficio, entonces:
Si B(x) > C(x), se hace x; de lo contrario, no se hace.
Para aplicar esta regla es necesario definir y medir los costos y los beneficios. Los valores mo-
netarios son un denominador común útil para este propósito, aun cuando la actividad no tenga de

01_CHAPTER 1.indd 4 3/6/09 6:40:48 PM


el PaPel De la teoría eConómiCa 

manera directa nada que ver con el dinero. B(x) se define como la máxima cantidad que está usted
dispuesto a pagar por hacer x. Con frecuencia B(x) será una magnitud hipotética, la cantidad que
usted estaría dispuesto a pagar si tuviera que hacerlo, incluso aunque el dinero no cambie de manos.
C(x), por otro lado, es el valor de todos los recursos a los que tiene que renunciar por hacer x. En este
caso C(x) tampoco implica una transferencia explícita de dinero.
En la mayoría de las decisiones no es posible valorar en términos monetarios algunos de los
costos o los beneficios. Para ver cómo se procede en tales casos, considere la siguiente sencilla de-
cisión.

¿Debo bajar el volumen de mi equipo de música? EJEMPLO .


Se acaba de instalar en un cómodo sillón y está escuchando su equipo de música cuando se da cuenta
de que las dos siguientes canciones no le gustan. Si hubiese tenido un equipo programable, habría
predeterminado que no las tocara. Pero como no lo tiene, debe decidir si busca el control remoto y
salta las dos pistas, o si con toda paciencia espera hasta que pasen.
El beneficio de buscar el control remoto es no tener que oír las canciones que no le gustan.
Por otro lado, el costo es la molestia de tener que levantarse de su sillón y buscar el control. Si está
sentado muy a gusto y la música no le molesta demasiado, es probable que siga sentado. Pero si no
ha estado sentado mucho tiempo o si la música le resulta en verdad molesta, lo más probable es que
se levante.
Aun en decisiones sencillas como ésta, es posible valorar los costos y los beneficios relevantes en
términos monetarios. Considere primero el costo de levantarse de su sillón. Si alguien le ofreciera 1
centavo por levantarse de su confortable sillón y no hubiera ninguna otra razón más que ese centavo
para hacerlo, ¿aceptaría la oferta? La mayoría de las personas no lo harían. Pero si alguien le ofreciera
1 000 dólares estaría de pie en un instante. En algún punto entre 1 centavo y 1 000 dólares está su
precio de reserva, que es la cantidad mínima necesaria para que se levante del sillón. precio de reserva de la
Para determinar esta frontera imagínese una subasta mental con usted mismo en la que eleva actividad x precio al que
de manera continua la oferta en incrementos pequeños desde 1 centavo hasta llegar al punto en el a una persona le daría igual
que apenas valga la pena levantarse. Dónde se encuentre este punto depende, obviamente, de las hacer x o no hacerlo.
circunstancias. Si usted es rico la cantidad será sensiblemente superior a la que aceptaría si es pobre,
porque una determinada cantidad de dinero será relativamente menos importante para usted; si
se siente con mucha energía, la cantidad sería menor que si está cansado, etc. Para los fines de este
análisis, suponga que su precio de reserva por levantarse del sillón resulta ser de 1 dólar. Puede rea-
lizar una subasta mental similar para determinar la cantidad máxima que estaría dispuesto a darle a
alguien por buscar el control remoto y saltar las canciones. Este precio de reserva mide las ventajas
de no escuchar la música no deseada; suponga que resulta ser de 75 centavos.
En términos de esta regla de decisión formal, se tiene entonces que x = “buscar el control remoto y
saltar las canciones”, con B(x) = $0.75 < C(x) = $1, lo que significa que deberá quedarse en su sillón. Escu-
char las dos siguientes melodías será desagradable, pero menos que tener que levantarse. Si se invirtie-
ran las cifras de costo y beneficio, esto implicaría levantarse y bajar el volumen de la música. Si B(x) y
C(x) fueran iguales, a usted le daría lo mismo cualquiera de las dos alternativas.

EL PAPEL DE LA TEORÍA ECONÓMICA


La idea de que cualquiera podría calcular los costos y los beneficios de buscar el control remoto y sal-
tar las canciones puede sonar absurda. A los economistas se nos ha criticado por hacer suposiciones
poco realistas acerca de la conducta de la gente y los ajenos a nuestra disciplina se preguntarán qué
objeto tiene imaginar a una persona tratando de decidir cuánto pagará por no tener que levantarse
de su sillón.
Hay dos respuestas para estas críticas. La primera es que los economistas no suponen que las
personas hagan tales cálculos de manera explícita; en cambio, lo que muchos de ellos argumentan
es que suponer que la gente actúa como si hiciera tales cálculos permite hacer predicciones úti-
les. Esto fue expresado de manera convincente por Milton Friedman Premio Nobel, ilustrando

01_CHAPTER 1.indd 5 3/6/09 6:40:49 PM


 CaPítulo 1 PenSar Como un eConomiSta

su argumento mediante la observación de la técnica


empleada por los jugadores de billar experimentados.1
Friedman argumenta que los tiros que eligen, y la mane-
ra específica en que tratan de realizarlos, pueden prede-
cirse muy bien si se supone que los jugadores toman en
cuenta cuidadosamente las leyes relevantes de la física
newtoniana. Por supuesto que habrá muy pocos juga-
dores de billar experimentados que hayan tenido alguna
formación en física, y es difícil que alguno pueda recitar
leyes tales como “el ángulo de incidencia es igual al án-
gulo de reflexión”. Tampoco es probable que conozcan
las leyes de las “colisiones elásticas” y el “momento
angular”. Aun así, dice Friedman, en principio jamás
hubieran llegado a ser buenos jugadores de no haber te-

© Jay Directo/Getty images


nido en cuenta las leyes de la física. Nuestra teoría del
comportamiento del jugador de billar supone, de ma-
nera poco realista, que éste conoce las leyes de la física.
Friedman pide que esta teoría se juzgue no de acuerdo
con la exactitud de su supuesto central, sino con base en
su capacidad de predecir una conducta. Y en este sentido
es posible que el campeón de billar Corey
Deuel no conozca las leyes formales de
en realidad funciona muy bien.
la física de newton, pero la calidad de su Como los jugadores de billar, tenemos que desa-
juego hace pensar que tiene una profunda rrollar habilidades para enfrentarnos a nuestro entorno.
comprensión de ellas. Muchos economistas, Friedman entre ellos, consideran
que entenderemos mejor nuestra conducta si supone-
mos que actuamos regidos por las reglas de la toma racional de decisiones. Mediante prueba y error
adoptamos estas reglas, de la misma manera que los jugadores de billar asimilan las leyes de la física.
La segunda respuesta a la acusación de que los economistas hacen suposiciones poco realistas
es reconocer que, con frecuencia, las conductas difieren de lo que predicen los modelos económicos.
Por ello, como dice el economista Richard Thaler, a menudo nos comportamos más bien como ju-
gadores de billar novatos que ignoran las leyes de la física y que no tienen idea de cómo golpear a la
bola blanca para posicionarla para el tiro siguiente. Se verán importantes evidencias que sustentan
este segundo punto de vista.
Pero incluso en los casos en que los modelos económicos fracasan en los aspectos descriptivos,
estas teorías suelen ser una orientación útil para la toma de decisiones. Es decir, aun cuando no
siempre predigan cómo nos comportamos realmente, proporcionan ideas útiles sobre cómo alcanzar
objetivos de una manera más eficiente. Si los jugadores novatos de billar todavía no han aprendido las
leyes relevantes de la física, pueden, no obstante, consultarlas como guía para saber cómo mejorar.
En las decisiones de consumo y de negocios, los modelos económicos suelen desempeñar un papel
similar. Sin duda, este papel constituye por sí solo una razón convincente para estudiar economía.

ERRORES COMUNES EN LA TOMA DE DECISIONES


Algunos economistas se sonrojan cuando alguien ajeno a esta disciplina señala que gran parte de lo
que hacen se reduce a la aplicación del principio de que cualquier acción se debe realizar si y sólo si
sus beneficios son superiores a sus costos. ¡Esto no parece suficiente para que una persona con doc-
torado en economía se mantenga ocupada todo el día! Sin embargo, hay más de lo que a simple vista
se observa. La gente que estudia economía pronto descubre que medir los costos y los beneficios es
tanto un arte como una ciencia. Algunos costos parecen estar ocultos casi a propósito; otros pueden
parecer relevantes pero, examinados más de cerca, resulta que no lo son.

1
Milton Friedman, “The Methodology of Positive Economics”, en Essays in Positive Economics, Chicago: University of Chicago
Press, 1953.

01_CHAPTER 1.indd 6 3/6/09 6:40:51 PM


erroreS ComuneS en la toma De DeCiSioneS 

La economía enseña a identificar los costos y los beneficios que son en realidad importantes.
Uno de los objetivos principales de este libro es enseñar al lector a tomar mejores decisiones. Un
buen punto de partida es examinar algunos errores comunes que se presentan en la toma de deci-
siones. Los principios económicos relevantes son sencillos y de sentido común, pero mucha gente
los ignora.

ERROR . IGNORAR LOS COSTOS IMPLÍCITOS


Es erróneo ignorar los costos que no son explícitos. Si hacer la actividad x implica no poder hacer
la actividad y, entonces para usted el valor de hacer y (si lo hubiera hecho) es el costo de oportuni- costo de oportunidad de una
dad de hacer x. Muchas personas toman malas decisiones porque tienden a ignorar el valor de las actividad el valor de todo lo
oportunidades de las que se privan. Esto sugiere que casi siempre será útil traducir preguntas como que se debe sacrificar para
“¿debo hacer x?” en preguntas como “¿debo hacer x o y?”. En esta última pregunta y es simplemente hacer x.
la alternativa más valiosa para hacer x. El ejemplo siguiente ayuda a entender con más claridad esta
importante idea.

¿Debo ir hoy a esquiar o a trabajar como asistente de investigación? EJEMPLO .2


Cerca de su universidad hay una pista de esquí. Por experiencia sabe que pasar un día esquiando tiene
un costo de 60 dólares. El costo por día en la pista de esquí tiene un precio de 40 dólares (incluye
el transporte en autobús, el boleto para subir al funicular y el alquiler del equipo). Pero éste no es el
único costo de ir a esquiar, también debe tomar en cuenta el valor de la alternativa más atractiva de la
cual se privará por ir a la pista de esquí. Suponga que su mejor opción es su nuevo trabajo como asis-
tente de investigación de un profesor. En este trabajo gana 45 dólares por día y le gusta lo suficiente
como para estar dispuesto a hacerlo gratis. La pregunta a la que se enfrenta es “¿debo ir a esquiar o a
trabajar como asistente de investigación?”.
En este caso el costo de ir a esquiar no es sólo el precio explícito del paquete para esquiar ($40),
sino también el costo de oportunidad de los ingresos perdidos ($45). Por lo tanto, los costos totales
son de 85 dólares, cantidad superior a los beneficios, 60 dólares. Como C(x) > B(x), deberá quedarse
en la universidad y trabajar para su profesor. Alguien que ignore el costo de oportunidad de las ga-
nancias que deja de obtener decidirá, de manera incorrecta, ir a esquiar.

El hecho de que el trabajo de investigación le guste lo suficiente como para estar dispuesto a
hacerlo gratis es una manera de decir que no tiene costos psíquicos asociados. Esto es importante
porque significa que al no ir a trabajar no está evitando algo desagradable. Claro que no todos los
trabajos están dentro de esta categoría. Suponga ahora que su trabajo no sea el anterior, sino lavar
platos en el comedor de la universidad por el mismo sueldo, 45 dólares al día, y que esta labor es tan
desagradable que no estaría dispuesto a hacerla por menos de 30 dólares al día. Suponiendo que su
jefe en el comedor universitario le permite tomarse un día libre siempre que lo desee, reconsidere
su elección entre ir a esquiar o no.

¿Debo ir hoy a esquiar o a trabajar lavando platos? EJEMPLO .


Hay dos maneras equivalentes de ver esta decisión. Una es decir que la ventaja de ir a esquiar es no
tener que lavar platos. Como usted no estaría dispuesto a lavar platos por menos de 30 dólares al
día, no tener que hacer esa tarea tiene para usted ese valor. Así que ir a esquiar conlleva la ventaja,
indirecta, de no lavar platos. Si agrega este beneficio indirecto a los 60 dólares del beneficio directo de
ir a esquiar, se tiene que B(x) = $90. Viendo el problema de esta manera C(x) tiene el mismo valor que
antes: 40 dólares que cuesta esquiar más 45 dólares del costo de oportunidad de las ganancias sacrifi-
cadas, es decir 85 dólares. De manera que ahora B(x) > C(x), lo que significa que debe ir a esquiar.
Otra alternativa es ver lo desagradable del trabajo de lavar platos como una compensación in-
cluida en su salario. Si se emplea este método habrá que restar 30 dólares por día a su salario de
45 dólares por día y decir que el costo de oportunidad de no trabajar es de sólo 15 dólares por día.
Entonces C(x) = $40 + $15 = $55 < B(x) = $60, y la conclusión es la misma: debe ir a esquiar.

01_CHAPTER 1.indd 7 3/6/09 6:40:51 PM


 CaPítulo 1 PenSar Como un eConomiSta

No importa de cuál de las dos maneras evalúe lo desagradable de lavar platos. Lo importante es
que haga la valoración, únicamente de una manera o de otra. ¡Tenga cuidado de no hacerlo dos veces!

Como se muestra en el ejemplo 1.3, los costos y los beneficios son recíprocos. No incurrir en un
costo es lo mismo que obtener un beneficio. De la misma manera, no obtener un beneficio supone
incurrir en un costo.
Tan obvio como parece, se pasa por alto con frecuencia. Un caso concreto fue el de un estudian-
te extranjero que hace unos años terminó sus estudios y pensaba regresar a su país. Las regulaciones
comerciales nacionales permitían a las personas que regresaban del extranjero introducir al país un
automóvil nuevo sin tener que pagar el impuesto normal de 50 por ciento. El suegro del estudiante
le pidió que le llevara un Chevrolet de 20 000 dólares y le envió un cheque por esa cantidad exacta.
Esto colocó al estudiante ante un dilema, pues había planeado llevar un Chevrolet para venderlo en
su país. Dado que, como ya se dijo, los automóviles nuevos tenían que pagar un impuesto de 50 por
ciento, un automóvil así se podía vender a un concesionario de automóviles en 30 000 dólares. El
estudiante estimaba que con facilidad podría venderle el automóvil a un comprador particular en
28 000 dólares, de lo que obtendría una ganancia neta de 8 000 dólares. Por lo tanto, el costo de opor-
tunidad de dejarle el automóvil a su suegro en 20 000 dólares iba a ser de ¡8 000 dólares! No obtener
este enorme beneficio iba a suponer para él un gran costo. Al final el estudiante decidió asumirlo,
pues para él valía más conservar la tranquilidad en la familia. Como muestra el principio del costo-
beneficio, la mejor decisión no siempre es la que deja más dinero.

EJEMPLO . ¿Debo trabajar primero o ir a la universidad?


Los costos de ir a la universidad no se limitan a la
inscripción, la colegiatura, el alojamiento, la ali-
mentación, los libros, etc.; también comprenden el
costo de oportunidad de las ganancias no obtenidas
mientras se estudia. Los ingresos aumentan con la
experiencia; por lo tanto, cuanta más experiencia se
tenga, serán mayores los ingresos perdidos por asis-
tir a la universidad. Este costo de oportunidad, por
lo tanto, será el más bajo posible inmediatamente
después de terminar el bachillerato.
Por el lado de los beneficios, una enorme
ventaja de poseer una educación universitaria es
obtener unos ingresos bastante más altos. Entre
más pronto obtenga su título más tiempo tendrá
para aprovechar este beneficio. Otra ventaja es lo
agradable de estudiar la universidad en lugar de
tener que trabajar. En general, mientras más edu-
cación se tiene, el tipo de trabajo que se obtiene
tiende a ser menos desagradable (o más agradable).
Si va a la universidad inmediatamente después
del bachillerato, evita realizar los trabajos menos
agradables. Así, para la mayoría de las personas lo
razonable es ir primero a la universidad y trabajar
después. Consecuentemente, es más prudente asis-
tir a la universidad a los 20 años que a los 50.
Una excepción común es la de las personas que
al salir del bachillerato son demasiado inmaduras
para aprovechar los beneficios del aprendizaje uni-
¿Por qué la mayoría de los estudiantes ingresan a la universidad inmediatamente versitario; para ellas es más recomendable trabajar
después de terminar el bachillerato?
1 o 2 años antes de ir a la universidad.

01_CHAPTER 1.indd 8 3/6/09 6:40:53 PM


erroreS ComuneS en la toma De DeCiSioneS 

El ejemplo de la universidad es una ilustración perfecta del argumento de Friedman acerca de


cómo evaluar una teoría. Los jóvenes que terminan el bachillerato no deciden cuándo ingresar a la
universidad basándose en cálculos complicados en los que intervengan los costos de oportunidad.
Al contrario, la mayor parte ingresa en la universidad inmediatamente después del bachillerato sola-
mente porque eso es lo que hace la mayoría de sus compañeros. Es lo que se debe hacer.
Pero esto nos lleva a la pregunta de cómo se sabe qué es lo que se debe hacer. Las costumbres no
surgen de la nada. Una enorme cantidad de sociedades diversas han tenido que experimentar durante
siglos para llegar a ciertas decisiones. Si hubiera una manera significativamente mejor de organizar
los periodos de aprendizaje y trabajo durante la vida, hace tiempo que se hubiera descubierto en
alguna sociedad. Las costumbres actuales han sobrevivido porque son eficaces. Puede ser que la gen-
te no calcule de manera explícita los costos de oportunidad de las ganancias de que se priva, pero a
menudo se comporta como si lo hiciera.2
Este concepto tan simple del costo de oportunidad es uno de los más importantes de la mi-
croeconomía. El arte de aplicarlo en forma correcta consiste en ser capaz de reconocer la alternativa
más valiosa que se sacrifica por hacer una determinada actividad.

ERROR 2. COSTOS IRRECUPERABLES


Un costo de oportunidad puede no parecer relevante aunque en realidad lo sea. Por otro lado, a veces
un gasto puede parecer relevante cuando en realidad no lo es. Éste suele ser el caso de los costos hun-
didos o costos irrecuperables, que no se pueden recuperar una vez tomada la decisión. A diferencia de
los costos de oportunidad, los costos irrecuperables deben ignorarse; no hacerlo implica un segundo
error a la hora de tomar una decisión. El principio de ignorar los costos irrecuperables se ve con
claridad en el ejemplo siguiente.

¿Debo ir a Boston en mi automóvil o en autobús? EJEMPLO .


Usted planea hacer un viaje de 250 millas a Boston. Salvo por los costos, le da lo mismo ir en su
automóvil o tomar un autobús. El pasaje tiene un precio de 100 dólares. Usted no sabe cuánto tendrá
que gastar por ir en su automóvil, de manera que llama a una agencia de renta de vehículos para
tener una idea. En ella le informan que para su marca de automóvil los gastos promedio anuales para
10 000 millas/año son los siguientes:

Seguro $1 000
intereses 2 000
Gasolina y aceite 1 000
mantenimiento 1000
total $5 000

Suponga que con estas cantidades usted calcula que la milla sale a 50 centavos y que, por lo
tanto, el viaje de 250 millas en su automóvil ascenderá a 125 dólares. Como esto es más que los 100
dólares del pasaje en autobús, decide tomar el autobús.
Si decide de esta manera, está siendo víctima del error de los gastos irrecuperables. Los pagos
del seguro y de los intereses no varían con la cantidad de millas que recorra en su automóvil cada
año. Ambos son costos irrecuperables y no se modifican si usted va a Boston en su automóvil o no.
De los costos mencionados, los de gasolina, el aceite y el mantenimiento son los únicos que varían
con las millas recorridas. Éstos suman 2 000 dólares por cada 10 000 millas recorridas, es decir, 20
centavos por milla. A 20 centavos por milla ir a Boston en su automóvil le cuesta sólo 50 dólares, y
como esto es menos que el costo del pasaje en autobús, deberá ir en su automóvil.

2
Esto no significa forzosamente que todas las costumbres promuevan la eficiencia. Por ejemplo, las circunstancias pueden haber
cambiado de manera que una costumbre que en el pasado promovía la eficiencia, no lo haga más. Con el tiempo las costumbres
pueden cambiar. Sin embargo, muchos hábitos y costumbres, una vez arraigados, son difíciles de modificar.

01_CHAPTER 1.indd 9 3/6/09 6:40:53 PM


0 CaPítulo 1 PenSar Como un eConomiSta

En el ejemplo 1.5, observe el papel del supuesto según el cual, salvo los costos, a usted le da lo
mismo cualquiera de los tipos de transporte. Si usted hubiera preferido uno u otro, se tendría que ha-
ber ponderado también esa preferencia. Por ejemplo, si hubiera estado dispuesto a pagar 60 dólares
por evitarse la molestia de conducir, el costo real de ir en su automóvil sería de 110 dólares y no de 50
dólares, y en ese caso debería ir en autobús.
Ejercicios como el siguiente se encuentran a lo largo del libro como ayuda para confirmar que
se comprenden los conceptos analíticos importantes. Se dominará mejor la microeconomía si se
resuelven a medida que se avanza en la lectura.

EJERCICIO .
¿Cambiaría su respuesta a la pregunta del ejercicio . si el valor de evitarse la molestia de ir
en su automóvil fuera 20 dólares y además calculara 2 dólares por infracciones de tránsito, en
promedio, por cada 200 millas de recorrido?

A manera de comprobación, al final de cada capítulo se encuentran las respuestas a los ejercicios
contenidos en él. Por supuesto, los ejercicios serán mucho más útiles si trabaja en ellos antes de
consultar las respuestas.

EJEMPLO . El experimento de las pizzas


Una pizzería ofrece un almuerzo con el lema “coma todo lo que quiera por sólo 5 dólares”. Se paga
a la entrada y el mesero le sirve todas las porciones de pizza que quiera. Un ex colega realizó este
experimento: pidió a uno de sus asistentes que fungiera como mesero de un grupo de mesas.3 El
“mesero” eligió en forma aleatoria la mitad de las mesas y a cada una de las personas de esas mesas
les devolvió sus 5 dólares antes de tomarles su pedido. Los comensales de las mesas restantes no
obtuvieron ningún reembolso. Después el “mesero” llevó con detalle la cuenta de la cantidad de
porciones de pizza que comió cada persona. ¿Pronosticaría usted alguna diferencia entre las canti-
dades consumidas por los dos grupos?
Los comensales de los dos grupos se enfrentan a la pregunta: “¿debo comer una porción más de
pizza?”. Aquí la actividad x consiste en comerse una porción más de pizza. Para los dos grupos, C(x)
es cero: incluso los miembros del grupo que no obtuvo ningún reembolso podrían comer tantas por-
ciones como deseasen sin tener que pagar nada más. Como el grupo al que se le hizo el reembolso
fue elegido en forma aleatoria, no hay razón para suponer que a sus miembros no les guste la pizza
o que les agrade menos que a los otros. Para todos, la regla de decisión es continuar comiendo hasta
que ingerir una porción más deje de ser agradable. Por lo tanto, B(x) debe de ser igual para ambos
grupos y todas las personas deben continuar comiendo hasta que B(x) llegue a cero.
De acuerdo con este razonamiento, los dos grupos deberían comer en promedio la misma can-
tidad de pizza. Los 5 dólares pagados a la entrada son un costo irrecuperable y no deben influir sobre
la cantidad de pizza que coma una persona. Sin embargo, el grupo que no obtuvo el reembolso consumió
mucha más pizza.

Aunque en este experimento la regla de decisión costo-beneficio no pasó la prueba de la pre-


dicción, su mensaje para quien toma decisiones racionales permanece incuestionable. Por lógica los
dos grupos deberían haberse comportado igual. Después de todo, la única diferencia entre ellos es
que los componentes del grupo que recibieron el reembolso tienen de por vida ingresos que son 5
dólares superiores a los de los otros. Una diferencia tan trivial no debería tener ningún efecto sobre
el consumo de pizza. Parece que los miembros del grupo que no recibió el reembolso quisieron

3
Vea Richard Thaler, “Toward a Positive Theory of Consumer Choice”, en Journal of Economic Behavior and Organization 1,
1980.

01_CHAPTER 1.indd 10 3/6/09 6:40:54 PM


erroreS ComuneS en la toma De DeCiSioneS 

asegurarse de “aprovechar bien su dinero”. De hecho, es casi seguro que sólo este motivo los llevó a
comer en exceso.4
¿Qué tiene de malo estar motivado para “aprovechar bien nuestro dinero”? Nada, siempre y
cuando la fuerza de este motivo opere antes de que se inicie una negociación. Es del todo razonable
que dicho motivo lleve a alguien a preferir un restaurante en lugar de otro idéntico pero más caro.
Sin embargo, una vez que ha determinado el precio de su comida, el impulso de “aprovechar bien
nuestro dinero” debe olvidarse. La satisfacción que obtenga por comer otra porción dependerá en-
tonces sólo del apetito que tenga y de qué tanto le guste la pizza, no de cuánto pagó. Sin embargo,
con frecuencia la gente no parece comportarse de esta manera. La dificultad puede ser que no somos
seres completamente flexibles. Quizás haya motivos que tengan sentido en un contexto y no sean
fáciles de abandonar en otro.

EJERCICIO .2
Jim obtiene de una emisora de radio una entrada gratis para un concierto de jazz al aire libre.
Mike paga  dólares por una entrada para el mismo concierto. La noche del concierto cae una
tremenda tormenta eléctrica. Si Jim y Mike tienen los mismos gustos, ¿quién de ellos es más
probable que asista al concierto, suponiendo que cada uno decide basándose en una compara-
ción estándar costo-beneficio?

ERROR . MEDIR COSTOS Y BENEFICIOS


COMO PROPORCIONES Y NO COMO
CANTIDADES ABSOLUTAS
Cuando un niño le pregunta a su mamá “¿ya casi llegamos?”, ¿cuál será la respuesta si se encuentran a
10 millas de su destino? Sin saber algo más acerca del contexto del viaje, no se puede decir nada. Si se
encuentran casi al final de un viaje de 300 millas, es casi seguro que la respuesta será sí. Pero si acaban
de embarcarse en un viaje de 12 millas, la mamá dirá que no.
El contexto es importante en diversos juicios comunes. Considerar una distancia como un por-
centaje de la cantidad total que se recorrerá es natural e informativo. Mucha gente también encuen-
tra natural pensar en términos de porcentaje cuando compara los costos y los beneficios pero, como
se ilustra en el siguiente par de ejemplos, esto suele causar problemas.

¿Debe ir en su automóvil a Wal-Mart para ahorrar 0 dólares en un radio-reloj de 20 dólares? EJEMPLO .a
Usted decide comprar un radio-reloj en una tienda cercana a un precio de 20 dólares cuando un ami-
go le dice que el mismo aparato está en oferta en Walt-Mart a sólo 10 dólares. Si el almacén está a 15
minutos en automóvil, ¿donde deberá comprar el radio? (Si tiene garantía y se descompone, tendrá
que mandarlo a reparar con el fabricante, sin importar dónde lo haya comprado.)

¿Debe ir en su automóvil al centro para ahorrar 0 dólares en un televisor de  000 dólares? EJEMPLO .b
Usted decide por comprar un televisor en una tienda cercana a un precio de 1 010 dólares cuando un
amigo le dice que el mismo aparato está en oferta en Walt-Mart a sólo 1 000 dólares. Si la tienda de
autoservicio está a 15 minutos en automóvil, ¿donde deberá comprar el televisor?

4
Además del argumento de “aprovechar bien nuestro dinero”, otra explicación puede ser que 5 dólares es parte considerable de
la cantidad de dinero con que, a corto plazo, cuentan muchos comensales. De manera que las personas del grupo que recibió el
reembolso puede ser que se hayan limitado con el propósito de comer el postre, el cual ahora pueden darse el lujo de comprar. Para
probar esta explicación, a las personas del grupo que no recibió el reembolso se les podría haber dado antes, durante el día, 5 dólares
en efectivo de regalo y ver si aún se encontraba diferencia entre las cantidades de pizza consumidas por los dos grupos.

01_CHAPTER 1.indd 11 3/6/09 6:40:54 PM


2 CaPítulo 1 PenSar Como un eConomiSta

(De nuevo, emplear la garantía para una reparación del televisor implica enviarlo al fabricante.)
Para ninguna de estas dos preguntas hay una única respuesta correcta, en ambas se cues-
tiona si el beneficio (ahorro en la compra) de ir en automóvil a Wal-Mart compensa el costo del
desplazamiento. La mayoría de las personas dirán en definitiva que vale la pena en el caso del radio,
pero no en el del televisor. Cuando se les pide que expliquen esto, dicen que en el caso del radio, al
ir en automóvil a la tienda de autoservicio se obtiene 50% de ahorro, pero en el caso del televisor
se obtiene menos de 1 % de ahorro.
Sin embargo, estos porcentajes son irrelevantes. En ambos casos el beneficio de ir en automóvil
a Walt-Mart es de 10 dólares de ahorro por una compra a un precio más bajo. ¿Cuál es el costo de ir
en automóvil a la tienda de autoservicio? Puede ser que algunas personas estén dispuestas a hacer el
viaje por sólo 5 dólares, mientras que otras no están dispuestas a hacerlo por menos de 50 dólares.
Pero, sea cual sea esta cantidad, será la misma en los dos casos. De manera que las respuestas a las
preguntas planteadas deben ser las mismas. Si usted está dispuesto a hacer el viaje por, digamos, 8 dó-
lares, entonces deberá comprar tanto el radio-reloj como el televisor en Wal-Mart. Pero si su precio
de reserva por hacer el viaje es, digamos, 12 dólares, entonces deberá comprar los dos aparatos en la
tienda cercana.

Al usar la prueba costo-beneficio, los costos y los beneficios deben expresarse en cantidades
absolutas. Comparar porcentajes no es una manera adecuada de pensar con respecto a decisiones
como la anterior.

EJERCICIO .
Usted tiene un cupón que lo hace acreedor a un descuento en uno de dos viajes que tiene progra-
mado hacer el mes próximo. Puede obtener 00 dólares de descuento en la tarifa aérea normal
de 200 dólares a la ciudad de NuevaYork, o 20 dólares de descuento en la tarifa aérea normal de
2 00 dólares a Nueva Delhi. ¿Para cuál de los dos vuelos usará su cupón?

ERROR . NO ENTENDER LA DIFERENCIA


PROMEDIO-MARGINAL
Hasta ahora se han considerado decisiones con respecto a si realizar o no una determinada actividad.
Sin embargo, a menudo lo que hay que decidir no es si realizar o no una actividad, sino en qué me-
costo marginal incremento en dida. En este caso más complicado, el principio del costo-beneficio se puede aplicar reformulando la
el costo total que resulta de cuestión. En lugar de preguntar “¿debo hacer la actividad x?”, debe plantearse una y otra vez “¿debo
realizar una unidad más de una
aumentar la medida en la que estoy participando en la actividad x?”
actividad.
Para responder esta pregunta se deben comparar el beneficio y el costo de una unidad adicional
beneficio marginal de esa actividad. Al costo de una unidad más de una actividad se le llama costo marginal de la activi-
incremento en el beneficio dad y al beneficio de una unidad más de una actividad se le llama beneficio marginal.
total como resultado de La regla del costo-beneficio dice que se continúe aumentando el nivel de una actividad en tanto
realizar una unidad más de una su beneficio marginal supere a su costo marginal. Pero, como ilustra el ejemplo siguiente, a menudo
actividad. la gente no aplica esta regla de manera correcta.

EJEMPLO . ¿Tom debe botar otro barco?


Tom administra una pequeña flota pesquera de tres embarcaciones. El costo diario de operación,
que comprende el alquilar de las embarcaciones y los salarios de los pescadores, es de 300 dólares,
es decir, en promedio 100 dólares por embarcación botada. El ingreso total diario, o beneficio, que
se obtiene por la venta del pescado es de 600 dólares, es decir, en promedio, 200 dólares por em-
barcación botada. Tom decide que como el costo por embarcación es menor que el ingreso por
embarcación, deberá botar otra. ¿Es sensata esta decisión?

01_CHAPTER 1.indd 12 3/6/09 6:40:54 PM


erroreS ComuneS en la toma De DeCiSioneS 

Para responder a esta pregunta se debe comparar el costo marginal de botar una embarcación
con su beneficio marginal. Pero la información sólo da el costo promedio y el beneficio promedio costo promedio el costo
de lanzar una embarcación; que son, respectivamente, un tercio del costo total de las tres embar- promedio de realizar n
caciones y un tercio del ingreso total de las tres embarcaciones. Con estos dos datos no es posible unidades de una actividad es
el costo total de esa actividad
decidir si botar una embarcación más es económicamente razonable. Aunque el beneficio promedio
dividido entre n.
de las tres embarcaciones que se emplean hasta ahora podría ser igual al beneficio marginal de botar
otra embarcación, también puede ser mayor o menor. Lo mismo ocurre con los costos promedio y beneficio promedio el
marginal. beneficio promedio de
Para ilustrar esto, suponga que el costo marginal de botar una embarcación con su tripulación realizar n unidades de una
es constante, 100 dólares por embarcación por día. Entonces Tom deberá botar la cuarta embarca- actividad es el beneficio
ción únicamente si al hacerlo el ingreso total diario de la pesca aumenta por lo menos 100 dólares. El total de la actividad dividido
solo hecho de que el ingreso promedio actual sea 200 dólares por embarcación no dice cuál será el entre n.
beneficio de botar la cuarta embarcación.
Suponga, por ejemplo, que la relación entre la cantidad de embarcaciones botadas y el ingreso
total diario es como se describe en la tabla 1.1. Si son tres embarcaciones por día, el beneficio prome-
dio por embarcación será entonces 200 dólares, como se indicó antes. Si Tom bota la cuarta embarca-
ción, el ingreso promedio diario bajará a 160 dólares por embarcación, lo que sigue siendo más que el
costo marginal supuesto de 100 dólares. Sin embargo, observe que en la segunda columna el ingreso
total correspondiente a cuatro embarcaciones es de sólo 40 dólares más por día que el ingreso total
de tres embarcaciones. Y como esto es menos que su costo marginal (100 dólares), no tiene sentido
botar la cuarta embarcación.

TABLA .
Variación del costo total de acuerdo con el número de embarcaciones botadas

Cantidad de Beneficio promedio diario


embarcaciones Beneficio total diario ($) ($/embarcación)
0 0 0
1 300 300
2 480 240
3 600 200
4 640 160

El ejemplo siguiente ilustra cómo aplicar de manera correcta el principio del costo-beneficio en
este caso.

¿Cuántas embarcaciones deberá botar Tom? EJEMPLO .


El costo marginal de botar una embarcación con tripulación sigue siendo constante de 100 dólares
por día. Si el ingreso total diario de la pesca varía de acuerdo con la cantidad de embarcaciones bota-
das, como se muestra en la tabla 1.1, ¿cuántas deberá botar Tom?
Tom debe seguir botando embarcaciones en tanto el beneficio marginal de hacerlo sea por lo
menos igual al costo marginal. Si tiene un costo marginal constante de 100 dólares por botadura,
Tom debe seguir botando embarcaciones en tanto el beneficio marginal sea por lo menos de 100
dólares.
Al aplicar la definición de beneficio marginal a las entradas de beneficio total de la segunda
columna de la tabla 1.1 se obtienen los valores de beneficio marginal de la tercera columna de la tabla

01_CHAPTER 1.indd 13 3/6/09 6:40:55 PM


 CaPítulo 1 PenSar Como un eConomiSta

1.2. (Como el beneficio marginal es la variación que hay en el beneficio total por un aumento de uno
en la cantidad de embarcaciones, las entradas del beneficio marginal se colocan a la mitad entre los
renglones correspondientes a las entradas de beneficio total.) Por ejemplo, el beneficio marginal de
aumentar la cantidad de embarcaciones de una a dos es de 180 dólares, la diferencia entre el rendi-
miento total de 480 dólares de dos embarcaciones y los 300 dólares de una.

TABLA .2
Variación del beneficio marginal de acuerdo con la cantidad de embarcaciones
botadas

Cantidad de Beneficio total Beneficio marginal diario


embarcaciones diario ($) ($/embarcación)
0 0
300
1 300
180
2 480
120
3 600
40
4 640

Si se compara el costo marginal de 100 dólares por embarcación con las entradas correspondien-
tes al beneficio marginal en la tercera columna de la tabla 1.2, se ve que las primeras tres botaduras
satisfacen la prueba costo-beneficio, pero la cuarta no. Por lo tanto, Tom deberá botar tres embarca-
ciones.

EJERCICIO .
Si el costo marginal de la botadura de cada embarcación no hubiera sido de 00 dóalres sino de
0 dólares, ¿cuántas embarcaciones debería haber botado Tom?

El principio costo-beneficio dice que los costos y los beneficios marginales —medidas que corres-
ponden al aumento de la actividad en consideración— son los relevantes para decidir hasta qué punto
se sigue realizando una actividad. Sin embargo, para tomar tales decisiones, muchas personas com-
paran el costo y el beneficio promedio de la actividad. Pero, como debe haber quedado claro con el
ejemplo 1.8, aun cuando el beneficio promedio de una actividad en el nivel presente sea por mucho
mayor que su costo promedio, aumentar la medida en que se realiza puede no estar justificado.

GRÁFICA DE BENEFICIO MARGINAL


Y COSTO MARGINAL
Los ejemplos que se acaban de analizar implican decisiones respecto de una actividad que puede
tener lugar sólo en niveles específicos (cero embarcaciones, una embarcación, dos embarcaciones,
etc.). Sin embargo, los niveles de otras múltiples actividades varían de manera continua. Por ejem-
plo, se puede comprar gasolina en la cantidad que se desee. Cuando se trate de actividades que varían
de manera continua, suele ser conveniente representar en una gráfica la comparación entre beneficio
y costo marginal.

01_CHAPTER 1.indd 14 3/6/09 6:40:55 PM


la mano inviSible 

¿Cuánto deberá hablar Susan con Hal cada mes? EJEMPLO .0
Susan tiene una tarifa telefónica con la que paga 4 centavos por minuto en las llamadas de larga
distancia a su novio Hal. (Las fracciones de minuto se cobran a la misma tarifa, de manera que 30 se-
gundos le significarán 2 centavos.) La curva BM de la figura 1.1 muestra el valor, medido en términos
de lo que Susan está dispuesta a pagar, que tiene para ella un minuto más de conversación con Hal.
¿Cuántos minutos deberá hablar por teléfono Susan con Hal cada mes?

FIGURA .
Tarifa de
Cantidad óptima de
larga distancia
conversación.
es la cantidad en la que el
(centavos por minuto)
beneficio marginal de la
conversación es igual a su
Valor de costo marginal.
BM un minuto Costo de
adicional un minuto
8 adicional

Precio = 4 CM

2
1
Minutos
200 400 600 800 por mes

La pendiente negativa de la curva BM refleja el hecho de que el valor de un minuto adicional


disminuye con la cantidad total de conversación mantenida hasta entonces. (Como se verá en el
capítulo 3, una ley generalmente admitida indica que mientras más se tenga de un bien, menos valor
se le da a una unidad adicional del mismo.) La curva CM del diagrama mide el costo de cada minuto
adicional, que en este caso se ha considerado constante, 40 centavos. La cantidad óptima de conver-
sación se encuentra en el punto en que estas dos curvas se cruzan, es decir, 400 minutos por mes. Si
Susan habla con Hal menos de ese tiempo, el beneficio marginal de un minuto más será superior al
costo marginal, por lo tanto Susan debe hablar más tiempo. Pero si hablan más de 400 minutos por
mes, la cantidad que Susan ahorraría por hablar menos excederá el beneficio que sacrificaría, lo que
significa que deben hablar menos.

EJERCICIO .
Si la curva del beneficio marginal sigue siendo la misma que en la figura ., ¿cuántos minutos
por mes debe hablar Susan con Hal si la tarifa de larga distancia disminuye a dos centavos por
minuto?

LA MANO INVISIBLE
Una de las ideas más importantes en el análisis económico es que la búsqueda de los intereses per-
sonales de los individuos no sólo suele ser consistente con los más amplios objetivos sociales, sino
incluso es necesaria para ellos. Ajenos a los efectos de sus acciones, los consumidores preocupados
por sus propios intereses suelen actuar como si estuvieran dirigidos por lo que Adam Smith llamó la

01_CHAPTER 1.indd 15 3/6/09 6:40:56 PM


 CaPítulo 1 PenSar Como un eConomiSta

mano invisible a través de la cual se produce un mayor beneficio social. En el que quizá sea el párrafo
más citado de La riqueza de las naciones, Smith escribe:

No es de la benevolencia de los carniceros, de los cerveceros o de los panaderos que espe-


ramos obtener nuestros alimentos, sino de su preocupación por sus propios intereses. No
apelamos a su humanidad, sino a su amor a sí mismos, y nunca les hablamos de nuestras
necesidades, sino de su provecho.

Smith observó que la competencia entre los vendedores fomenta que se hagan intentos para
desarrollar productos mejores y métodos más baratos para producirlos. El primero que tiene éxito
en dichos intentos disfruta de ganancias mayores que sus rivales, pero sólo durante un tiempo. Una
vez que otros copian los productos y métodos nuevos, sus ofertas, de manera inevitable, presionarán
hacia abajo los precios. La idea de Smith, en pocas palabras, era que aunque los vendedores sólo
estén buscando promover su propio beneficio, al final los favorecidos serán los consumidores.
Los economistas modernos a veces pierden de vista el hecho de que Smith no creía que úni-
camente los motivos egoístas fuesen importantes. En su tratado anterior, Teoría de los sentimientos
morales, por ejemplo, escribe de manera conmovedora acerca de la compasión que se siente por los
demás:

A pesar de cuán egoísta pueda suponerse que sea el hombre, es evidente que existen al-
gunos principios naturales en él que lo hacen interesarse por la fortuna (bienestar) de los
demás, y que transforman su felicidad en la suya, a pesar de que él no obtenga nada por
ello, salvo el placer de verlo. A esta clase pertenece la pena o la compasión, la emoción
que sentimos por el sufrimiento de otros, cuando lo presenciamos o se nos hace percibirlo
de manera muy vívida. Que con frecuencia sintamos pena por la desgracia de otros es
un hecho demasiado obvio para requerir de ejemplo alguno para probarlo; ya que este
sentimiento, como todas las demás pasiones propias de la naturaleza humana, no es, de
manera alguna, exclusivo de los virtuosos y humanitarios, aunque quizás ellos lo sientan
con la sensibilidad más exquisita. El peor rufián, el más empedernido violador de las leyes
de la sociedad no está del todo desprovisto de dicho sentimiento.

Smith estaba consciente, además, de que el resultado de la búsqueda desenfrenada del interés
personal está algunas veces muy lejos de ser favorable a la sociedad. Como se ilustra con el ejemplo
siguiente, el mecanismo de la mano invisible se viene abajo cuando considerables costos o beneficios
quedan en manos de personas distintas de las que toman las decisiones.

EJEMPLO . ¿Debo quemar mis hojas o llevarlas al bosque?


Suponga que el costo de transportar las hojas es de 20 dólares y el de quemarlas es de 1 dólar. Si el
dueño de la casa se preocupa sólo por los costos que recaen de manera directa en él, quemará las
costo externo de una hojas. La dificultad estriba en que la quema de las hojas implica un costo externo importante que
actividad un costo que recae en personas que no participan directamente en la decisión. Dicho costo externo es el daño
recae en personas que no causado por el humo del fuego, el cual recae no sobre el dueño de la casa que toma la decisión de
están relacionadas en forma quemar las hojas, sino sobre las personas que viven en la dirección del viento. Suponga que el daño
directa con la actividad. causado por el humo asciende a 25 dólares. El bienestar de la comunidad requiere que las hojas sean
transportadas, no quemadas. Desde la perspectiva de los intereses del dueño de la casa, sin embargo,
es mejor quemarlas.5

Los costos y los beneficios externos suelen motivar leyes que limitan el criterio personal. (Los
costos y los beneficios externos serán el tema principal del capítulo 17.) Por ejemplo, en la mayoría
de las comunidades hay leyes que prohíben quemar las hojas dentro de los límites de las ciudades.

5
Por supuesto, si el dueño de la casa con frecuencia interacciona con las personas que viven en la dirección del viento, el interés
personal le dictará transportar las hojas con el fin de conservar la buena relación para futuras interacciones. Pero si las personas que
viven en la dirección del viento son ajenas y anónimas, este motivo operará con menos fuerza.

01_CHAPTER 1.indd 16 3/6/09 6:40:56 PM


el naturaliSta eConómiCo 

Estas leyes pueden verse como una manera de hacer que los costos y los beneficios que ven los
individuos tengan una mejor coincidencia con los que experimenta la comunidad como un todo. Si
hay una ley que prohíbe quemar las hojas, las personas que podrían hacerlo ponderarán las conse-
cuencias de quebrantar la ley contra el costo de transportarlas. La mayoría de las personas concluye
que sale más barato transportar las hojas.

¿QUERRÁN LOS PADRES QUE SU HIJA O HIJO


SE CASE CON UN HOMO ECONOMICUS?
Muchos economistas y otros científicos de la conducta se mantienen escépticos acerca de la impor-
tancia del deber y de otros motivos desinteresados. Creen que mientras mayor sea el dominio que,
en una primera aproximación, la recompensa asociada con la conducta egoísta ejerza sobre otros
motivos, con toda tranquilidad pueden ignorarse los motivos desinteresados.
Con esto en mente, al estereotipo de la persona que toma decisiones de acuerdo con el modelo
del interés personal se le suele colocar la etiqueta de Homo economicus u “hombre económico”. El
Homo economicus no experimenta el tipo de sentimientos que motivan a las personas a votar o a de-
volverle a su dueño, con el dinero intacto, una billetera extraviada. Por el contrario, lo único que le
preocupa son los costos y los beneficios materiales de carácter personal. No contribuye de manera
voluntaria con la beneficencia privada o con las entidades públicas de carácter humanitario, cumple
sus promesas sólo en el caso de que valga la pena hacerlo, y si las leyes contra la contaminación
no son de observancia estricta, desconecta el convertidor catalítico de su automóvil para ahorrar
combustible, etcétera.
Por supuesto, muchas personas no encajan en la caricatura del “yo primero” del modelo del
interés personal. Ellas donan médula ósea a extraños que padecen leucemia. Soportan problemas y
gastos para hacer que se haga justicia, aun cuando esto no repare el daño original. Ponen en riesgo
sus propias vidas para liberar a personas atrapadas en edificios en llamas y se arrojan a ríos helados
para rescatar a alguien a punto de ahogarse. Si son soldados se lanzan sobre granadas activadas para
salvar a sus camaradas.
Sin duda, los motivos egoístas son importantes. Cuando un detective investiga un crimen, por
ejemplo, su primera pregunta es: “¿quién se beneficia con la muerte de la víctima?”. Cuando un eco-
nomista estudia una normativa estatal, quiere saber el ingreso de quién se incrementa. Cuando un
senador propone un nuevo proyecto de gasto, el politólogo trata de descubrir cuáles de sus electores
serán sus principales beneficiarios.
El objetivo de gran parte de este libro es entender los tipos de comportamiento a los que los
motivos egoístas dan lugar en determinadas situaciones. A lo largo de este proceso es necesario re-
cordar que el modelo del interés personal no debe entenderse como una receta de cómo conducir los
asuntos propios. Al contrario, en capítulos posteriores se verá que el Homo economicus está contrain-
dicado respecto a las diversas demandas sociales que se plantean. Es probable que todos conozcamos
personas que corresponden más o menos a la caricatura del Homo economicus. Y la mayoría de las
veces lo primero que hacemos es huir de ellas.
Lo irónico aquí es que ser una persona interesada sólo en el propio beneficio conlleva un grado
de aislamiento social que no sólo es malo para el alma, sino también dañino para el bolsillo. Para
tener éxito en la vida, incluso sólo en términos materiales, las personas necesitan formar alianzas y
relaciones personales. Pero, ¿qué persona sensata está dispuesta a confiar en el Homo economicus? En
capítulos posteriores se presentan ejemplos concretos de cómo los motivos desinteresados propor-
cionan recompensas materiales a quienes los albergan. Por ahora, sin embargo, es importante tener
en mente que el modelo del interés personal sólo pretende captar una parte de la conducta humana,
aunque una muy importante.

EL NATURALISTA ECONÓMICO
Estudiar biología permite a las personas observar y maravillarse con muchos detalles de la vida que
de otra manera les pasarían inadvertidos. Para el naturalista, un paseo por un tranquilo bosque se
convierte en una aventura. De manera muy parecida, estudiar microeconomía permite convertirse

01_CHAPTER 1.indd 17 3/6/09 6:40:57 PM


 CaPítulo 1 PenSar Como un eConomiSta

en un “naturalista económico”, una persona que ve con nuevos ojos los detalles ordinarios de la vida
diaria. Cada aspecto del paisaje hecho por el hombre deja de ser una masa amorfa para convertirse en
el resultado de un cálculo implícito costo-beneficio. A continuación se presentan algunos ejemplos
del naturalismo económico.

¿Por qué es tan mala la comida de las aerolíneas?


Todo el mundo se queja de la comida de las aerolíneas. En efecto, si un restaurante serio se atreviera
a servir esa comida, quebraría. Estas quejas parecen dar por sentado que los alimentos en las aerolí-
EL NATURALISTA neas deben ser tan buenos como los que se comen en un restaurante. Pero, ¿por qué habrían de serlo?
La perspectiva costo-beneficio dice que las aerolíneas deben mejorar la calidad de sus alimentos si y
ECONÓMICO sólo si el beneficio es mayor que el
1.1 costo. Es probable que el beneficio
de una mejor comida esté bien
¿Ternera empanizada, chuleta medido por lo que los pasajeros
o rollitos vegetarianos? están dispuestos a pagar por ella
en la forma de precios más altos de
los boletos de avión. Si se pudieran
tener alimentos de la calidad de
un restaurante por un aumento
en el precio de los boletos de, por
ejemplo, 10 dólares, es probable
que la mayor parte de las personas
estarían encantadas de pagarlos.
Sin embargo, el problema es que
cuesta mucho más que eso prepa-
rar alimentos significativamente
mejores a 39 000 pies de altura en
una diminuta cocina de avión con
casi nada de tiempo. Se podría ha-
cer, por supuesto. Las aerolíneas
AEROLINEAS: EL RESTAURANT podrían quitar 20 asientos de las
aeronaves, instalar una cocina mo-
© the new Yorker Collection 2001 mick Stevens from cartoonbank.com. all rights reserved derna, bien equipada, contratar
personal especializado, gastar más
en los ingredientes, etc. Pero estos
costos adicionales serían de 100 dólares y no de 10 dólares por pasajero. A pesar de todas las quejas
acerca de la mala calidad de los alimentos en las aerolíneas, pocos estarían dispuestos a soportar este
aumento. El triste resultado es que los alimentos de las aerolíneas están destinados a seguir siendo
incomibles.

Muchos aceptamos con gusto la máxima “cualquier cosa que valga la pena hacerse, vale la
pena hacerla bien”. Después de todo, esto fomenta un cierto orgullo del trabajo que tristemen-
te falta con frecuencia. El naturalista económico 1.1 aclara, sin embargo, que si esta máxima se
interpreta de manera literal, no tiene ningún sentido. Hacer algo bien requiere tiempo, dinero y
esfuerzo; pero estos recursos son escasos. Dedicarlos a una actividad hace que no estén disponibles
para otra. Mejorar la calidad de una de las cosas que se hacen significa reducir la calidad de otras;
otra aplicación del concepto de costo de oportunidad. Toda decisión inteligente debe ser consciente
de estas disyuntivas.
En la vida todo es resultado de este tipo de negociaciones. A María Sharapova jugar tenis profe-
sional le impide convertirse en concertista de piano. Pero esto, por supuesto, no quiere decir que no
tenga tiempo para tocar el piano. Sólo significa que no obtendrá el nivel que tiene en el tenis.

01_CHAPTER 1.indd 18 3/6/09 6:40:58 PM


miCroeConomía Y maCroeConomía 

¿Por qué las transmisiones manuales tienen cinco velocidades y las automáticas sólo cuatro?
Mientras más velocidades tenga la transmisión de un automóvil, mayor será el ahorro de combus-
tible. Cada velocidad adicional funciona como “la marcha directa” de los automóviles de la década
de 1940, ahorra combustible permitiendo que a las velocidades de las autopistas el automóvil viaje EL NATURALISTA
a un ritmo constante con una menor velocidad del motor. La mayoría de los automóviles de pro- ECONÓMICO
ducción actual ofrecen cinco velocidades si son de transmisión manual y sólo tres o cuatro si son de
transmisión automática. Ya que el ahorro de combustible es sin duda una ventaja, ¿por qué limitar la
1.2
cantidad de velocidades en las transmisiones automáticas?
La razón es que el ahorro de combustible no es un objetivo único. También se quiere mantener
el precio de los automóviles dentro de ciertos límites. Las transmisiones automáticas son más com-
plejas que las manuales y, por lo tanto, el costo de agregar una velocidad es mucho mayor. Pero, las
ventajas de agregar una velocidad son las mismas en ambos casos. Si los fabricantes de automóviles
siguen la regla “agregar una velocidad si su beneficio supera a su costo”, entonces los vehículos auto-
máticos tendrán menos velocidades que los manuales.

El razonamiento que se presenta en El naturalista económico 1.2 también ayuda a entender por
qué las transmisiones manuales tienen ahora cinco velocidades cuando hace 50 años sólo tenían tres
(y muchas transmisiones automáticas sólo dos). El beneficio de agregar una velocidad extra se debe,
otra vez, a que aumenta el ahorro de combustible. El valor de este beneficio, en dinero, depende de
manera directa del precio del combustible. El precio de la gasolina en relación con otros bienes es
mucho más alto que hace 50 años, lo que ayuda a explicar por qué ahora las transmisiones tienen
más velocidades que antes.

CUESTIONES POSITIVAS Y CUESTIONES


NORMATIVAS
En la región al noroeste del Pacífico (de Estados Unidos) para proveer a los contratistas con made-
ra para construir casas las empresas taladoras están derrumbando las pocas secuoyas vírgenes que
quedan en pie. Muchos de estos árboles tienen más de 2 000 años, un tesoro nacional que jamás se
podrá reemplazar. Sin embargo, para las empresas taladoras estos árboles valen más como madera
que como monumentos del pasado. Si se deben o no proteger las reservas restantes de secuoyas
vírgenes es al final una cuestión normativa en la que están involucrados nuestros valores. Una cues- cuestión normativa una
tión normativa es una cuestión concerniente a lo que debe ser o debería ser. El análisis económico no cuestión acerca de qué
políticas específicas o acuerdos
puede por sí mismo responder a tales cuestiones. Una sociedad en la que se preserve la naturaleza
institucionales llevan a los
y lo ancestral bien puede darle a las secuoyas un destino distinto al de una sociedad que tiene otros
mejores resultados.
valores, aun cuando los miembros de ambas sociedades coincidan por completo con respecto a to-
dos los hechos y teorías económicas importantes. El análisis económico está en su campo cuando
cuestión positiva una cuestión
se trata de responder a cuestiones positivas, asuntos acerca de cuáles serán las consecuencias de
acerca de las consecuencias
determinadas políticas o disposiciones institucionales. Si se prohíbe la tala de secuoyas vírgenes, ¿qué
de determinadas políticas o
pasará con el precio de la madera? ¿Qué material de construcción puede usarse como sustituto y a acuerdos institucionales.
qué costo? ¿Cómo se afectará el empleo en las industrias taladora y de la construcción? Todas estas
son cuestiones económicas positivas y sus respuestas son relevantes para nuestra manera de pensar
acerca de las cuestiones normativas subyacentes.

MICROECONOMÍA Y MACROECONOMÍA
El interés principal en este capítulo es el aspecto de la toma de decisiones individuales. A medida que
se avance, también se considerarán modelos económicos para grupos de individuos; por ejemplo,
para el grupo de todos los compradores o de todos los vendedores en un mercado. El estudio de las
elecciones individuales y del comportamiento de un grupo en mercados individuales pertenecen,
ambos, a la microeconomía. La macroeconomía, en cambio, es el estudio de un agregado más exten-
so de mercados. Por ejemplo, la macroeconomía trata de explicar la tasa de desempleo nacional, el
nivel general de los precios y el valor total de la producción nacional.

01_CHAPTER 1.indd 19 3/6/09 6:40:59 PM


20 CaPítulo 1 PenSar Como un eConomiSta

Los economistas son mucho mejores para predecir y explicar lo que ocurre en mercados indi-
viduales que en la economía como un todo. Cuando economistas prominentes, en la prensa o en
la televisión, están en desacuerdo, lo más probable es que el tema trate de macroeconomía y no de
microeconomía. Pero aun cuando los economistas tengan problema con las cuestiones macroeconó-
micas, el análisis macroeconómico tiene una importancia innegable. Después de todo, las recesiones
y las inflaciones crean problemas en millones de vidas.
Los economistas creen cada vez más que la clave para el avance de la macroeconomía está en un
análisis más cuidadoso de los mercados individuales, que son constituyentes de agregados más exten-
sos. Esto ha ocasionado que la diferencia entre micro y macroeconomía se haya vuelto menos clara
en los últimos años. La formación de los economistas, micro y macro, está cada vez más enfocada en
el análisis microeconómico.

RESuMEN
• La microeconomía supone el estudio de la elección en condi- importantes. Los beneficios también suelen ser difíciles de
ciones de escasez, la cual está siempre presente aun cuando los medir. La experiencia enseña que estar atento a los errores
recursos materiales sean abundantes. Siempre existen limita- más comunes ayuda a convertirse en personas que toman me-
ciones importantes de tiempo, energía y otras cosas necesarias jores decisiones.
para lograr nuestros objetivos. • Cuando la pregunta no es si realizar una actividad sino en qué
• Gran parte de la tarea de los economistas es tratar de respon- medida hacerlo, el análisis marginal atrae nuestra atención
der a preguntas de la forma: “¿debo realizar la actividad x?”. El hacia la importancia de los beneficios y los costos marginales.
método para responderlas es muy simple. Se debe hacer x si y Siempre que los beneficios marginales de una actividad sean
sólo si sus costos son menores que sus beneficios. No incurrir superiores a sus costos marginales, debe aumentarse la canti-
en ningún costo es lo mismo que obtener un beneficio. dad de esa actividad.
• El modelo costo-beneficio falla algunas veces al predecir el • Los principios de la elección racional no están de ninguna ma-
comportamiento de la gente cuando se enfrenta a las eleccio- nera limitados a los mercados formales de bienes y servicios.
nes cotidianas. El arte del análisis costo-beneficio está en ser En realidad, detrás de casi cualquier acción, objeto o conducta
capaz de especificar y medir los costos y los beneficios rele- humana está alguna forma de cálculo costo-beneficio explíci-
vantes, habilidad de la que muchas personas carecen al tomar to o implícito. El conocimiento de los principios subyacentes
una decisión. Algunos costos, como los irrecuperables, suelen arroja sobre nuestro mundo una nueva y penetrante luz, no
parecer importantes pero resultan no serlo. Otros, como los siempre halagadora, pero fuente continua de estimulante ins-
costos implícitos, a veces son ignorados, incluso cuando son piración.

pREguNTAS DE REpASO
1. ¿Cuál es su costo de oportunidad de leer una novela esta no- 4. ¿Por qué el costo de oportunidad de ir a la universidad es más
che? elevado para una persona de 50 años que para una de 20?
2. Su compañero de cuarto está pensando dejar la escuela este 5. ¿Por qué los costos irrecuperables deben ser irrelevantes para
semestre. Si la colegiatura del semestre no es reembolsable, las decisiones presentes?
¿deberá tener esto en cuenta al tomar la decisión? 6. ¿Cómo puede ser útil el modelo costo-beneficio al estudiar el
3. Dé tres ejemplos de actividades junto con sus costos y benefi- comportamiento de las personas que no piensan de manera
cios externos. explícita en términos de costos y beneficios?

pRObLEMAS
1. Jamal tiene un trabajo de verano flexible. Puede trabajar todos los días y se le permite tomar un día
libre cuando quiera. Su amigo Don sugiere que el jueves vayan al parque de atracciones. La entrada
tiene un precio de 15 dólares por persona y además cada uno tendrá que gastar 5 dólares en gasolina
y estacionamiento. A Jamal le encanta el parque de atracciones y para él un día en el parque vale 45
dólares. Pero a Jamal también le gusta tanto su trabajo que estaría dispuesto a pagar 10 dólares por
día por hacer ese trabajo.

01_CHAPTER 1.indd 20 3/6/09 6:40:59 PM


ProblemaS 2

a) Si Jamal gana 10 dólares si trabaja, ¿debe ir al parque de atracciones?


b) ¿Si Jamal gana 15 dólares…?
c) ¿Si Jamal gana 20 dólares…?
2. Tom cultiva champiñones. Invierte todos sus ahorros en más champiñones que cultiva en un terreno
detrás de su garaje, que no tiene ninguna otra utilidad. En el primer año los champiñones duplican
su tamaño y pasado este tiempo se cosechan y se venden a un determinado precio por libra. Dick, un
amigo de Tom, le pide que le preste 200 dólares y le promete devolvérselos en un año. ¿Cuánto de-
berá pagarle Dick a Tom por los intereses, para que este último no esté en peor situación económica
por haberle hecho el préstamo?
3. El plan de comedor de la universidad A permite a los estudiantes comer todo lo que quieran por una
cuota de 500 dólares por semestre. El estudiante promedio de esta universidad consume 250 libras de
alimentos por semestre. La universidad B vende a los estudiantes un talonario con boletos de come-
dor que les permite consumir 250 libras de alimentos durante el semestre. Si el estudiante consume
más de 250 libras, tiene que pagar una cantidad extra; si consume menos, se le da un reembolso. Si
los estudiantes son razonables, ¿en cuál de las universidades será más caro el consumo de alimentos?
4. Usted planea hacer un viaje de 1 000 millas a Florida. Excepto por los costos, le da lo mismo ir en su
automóvil o en autobús. El pasaje asciende a 260 dólares. Los costos por el uso de su automóvil con
un promedio de 10 000 millas/año son los siguientes:

Seguro $1 000
intereses 2 000
Combustible y aceite 1 200
neumáticos 200
Permiso y registro 50
mantenimiento 1 100
total $5 550

¿Deberá ir en su automóvil o tomar el autobús?


5. Al y Jane alquilaron un salón de banquetes para celebrar su aniversario de bodas. Tienen 50 invitados
confirmados. La empresa del servicio de banquetes cobra por los alimentos 5 dólares por persona y
por las bebidas 2 dólares por persona. El grupo musical cobra 300 dólares por noche y el salón cuesta
200 dólares. Ahora Al y Jane quieren invitar a otras 10 personas. ¿Cuánto aumentarán los costos de la
fiesta con estos invitados extra?
6. Le presta 1 000 dólares a un amigo y al cabo de un año su amigo le extiende un cheque por 1 000
dólares para pagarle el préstamo. Si la tasa de interés anual en su cuenta de ahorros es de 6 por ciento,
¿cuál fue su costo de oportunidad por hacer el préstamo?
7. Bill y Joe viven en Ithaca, Nueva York. A las 2 de la tarde, Bill va al Ticketmaster y compra un boleto
de 30 dólares para el partido de baloncesto que se jugará esa noche en Syracuse (a 50 millas al norte).
Joe también planea asistir a ese partido, pero no compra sus entradas con anticipación porque sabe
por experiencia que siempre es posible conseguir lugares igual de buenos en el estadio. A las 4 de
la tarde empieza una inesperada tormenta de nieve que hace a la idea de ir a Syracuse en el propio
automóvil un poco menos atractiva que antes. Si tanto Bill como Joe tienen los mismos gustos y son
razonables, ¿la probabilidad de asistir al partido es mayor para alguno de ellos? Si es así, ¿para quién y
por qué? Si no es así, explique por qué no.
8. Para los aviones de pasajeros existen dos tipos de dispositivos para detección del clima por radar:
los que emplean la tecnología más moderna y otros que son mucho menos costosos, pero también
menos eficientes. La Federal Aviation Administration (FAA) lo contrata a usted para que los asesore
sobre si es necesario que todos los aviones de pasajeros empleen el dispositivo con la última tecnolo-
gía. Después de un cuidadoso estudio, su recomendación es emplear los dispositivos más caros sólo
en los aviones con más de 200 asientos. ¿Cómo justifica usted esta recomendación ante los miembros
de la FAA que se quejan diciendo que todos los pasajeros tienen derecho al mejor radar detector que
existe?

01_CHAPTER 1.indd 21 3/6/09 6:41:00 PM


22 CaPítulo 1 PenSar Como un eConomiSta

9. Un grupo alquila un autobús para ir a la ciudad de Nueva York. El conductor cobra 100 dólares, el
alquiler del autobús cuesta 500 dólares y el peaje 75 dólares. El pago del conductor no es reembol-
sable, pero el alquiler del autobús puede cancelarse con una semana de anticipación pagando sólo
50 dólares. A 18 dólares por boleto, ¿cuántas personas deben comprar un boleto para que el viaje no
tenga que ser cancelado?
10. En su ciudad se recolecta la basura por una cuota semanal de 6 dólares sin importar la cantidad de
botes de basura. La cantidad promedio de botes de basura por hogar es de tres botes cada semana.
Ahora suponga que se cambia el sistema por uno de “etiquetas”. Cada bote de basura debe llevar
pegada una etiqueta, cada una de las cuales cuesta 2 dólares. ¿Qué efecto tendrá este sistema sobre la
cantidad de basura recolectada?
11. Suponga que el costo de agregar memoria RAM a su computadora es de 100 dólares por gigabyte.
Considere también que el valor para usted, medido en términos de su disposición a pagar, de un
gigabyte más de memoria es de 800 dólares para el primero, y que este valor baja después a la mitad
con cada gigabyte extra. Trace una gráfica de costo marginal y beneficio marginal. ¿Cuántos gigab-
ytes de memoria deberá comprar?
12. Suponga que en el problema 11 el costo de la memoria RAM baja a 50 dólares por gigabyte. ¿Cuántos
gigabytes de memoria deberá comprar ahora? Suponga además que su beneficio por un gigabyte
más de memoria aumenta a 1 600 dólares por el primero y que también baja a la mitad con cada giga-
byte extra. Ahora, teniendo tanto un precio más bajo como un beneficio mayor, ¿cuántos gigabytes
de memoria deberá comprar?
*13. Dana compró una entrada de 40 dólares para un concierto de rock. El día del concierto la invitan a
una fiesta de bienvenida para un amigo suyo que regresa del extranjero. No puede asistir a la fiesta y
al concierto. Si hubiera sabido de la fiesta antes de comprar la entrada, hubiera preferido ir a la fiesta
en lugar de al concierto. Verdadero o falso: se concluye que, si es razonable, de cualquier manera irá a
la fiesta. Explique.
*14. Ayer, de manera inesperada, le regalaron una entrada para un concierto de Dave Matthews que será
el 1 de abril. El precio normal de esta entrada es de 75 dólares, pero podría venderlo a un máximo
de 50 dólares. Hoy se entera de que ese mismo día habrá un concierto de Ani DiFranco. Todavía hay
entradas para el concierto de ella a 75 dólares. Si ayer, antes de que le regalaran la entrada para Dave
Matthews hubiera sabido que iba a haber un concierto de Ani DiFranco, hubiera comprado una en-
trada para verla a ella, no a Dave Matthews. Verdadero o falso: de acuerdo con lo que se sabe acerca de
sus preferencias se concluye que si usted maximiza su utilidad en forma racional, asistirá al concierto
de Ani DiFranco. Explique.
*15. Hace poco el señor Smith tuvo que decidir entre trabajar como a) profesor de economía, ganando
60 000 dólares al año, o b) guía de safaris, ganando 50 000 dólares al año. Después de una cuidadosa
reflexión, Smith se decidió por el trabajo de guía de safari, pero fue sólo por un pelo. “Por un dólar
más”, dijo, “hubiera elegido el otro”.
Ahora se acerca su cuñado con una propuesta de negocio. Los términos son los siguientes:
• Smith tendrá que renunciar a su trabajo como guía de safaris para trabajar de tiempo completo en
el negocio de su cuñado.
• Smith debe hacerle a su cuñado un préstamo libre de intereses por 100 000 dólares, el cual éste le
devolverá a Smith cuando se salga del negocio, en caso de que esto llegue a ocurrir. (Smith tiene
mucho más que 100 000 dólares en el banco.)
• El negocio le pagará a Smith un sueldo de 70 000 dólares al año. No recibirá ningún otro pago.
La tasa de interés es de 10 por ciento anual. Fuera de las consideraciones sobre el salario, Smith
siente que trabajar en ese negocio será tan agradable como ser profesor de economía. Para simplifi-
car las cosas, suponga que no existe ninguna incertidumbre con respecto al salario ni a la seguridad
de su inversión monetaria en el negocio. ¿Deberá Smith trabajar con su cuñado? En caso de acep-
tar, ¿cuál sueldo más bajo de Smith en el negocio para que no valiera la pena trabajar en él? Si rechaza
la propuesta, ¿cuál sería el sueldo más alto de Smith en el negocio para que valiera la pena trabajar
en él?

* Los problemas marcados con asterisco (*) tienen mayor grado de dificultad.

01_CHAPTER 1.indd 22 3/6/09 6:41:00 PM


reSPueStaS a loS eJerCiCioS Del CaPítulo 2

*16. Usted acaba de comprar un Ford Taurus nuevo en 20 000 dólares, pero el mayor precio al que lo
puede vender a un particular es de 15 000 dólares. Ahora se entera de que Toyota está ofreciendo su
modelo Camry, cuyo precio normal es 25 000 dólares a un precio especial de 20 000 dólares. Si antes
de comprar el Taurus hubiera sabido que podía comprar un Camry a ese mismo precio, hubiera
preferido este último. Verdadero o falso: de acuerdo con lo que se sabe de sus preferencias, se concluye
que si usted maximiza su utilidad de manera racional no venderá al Taurus para comprar un Camry.
Explique.

RESpuESTAS A LOS EjERcIcIOS DEL cApíTuLO


1.1 Una persona que por cada 200 millas de recorrido tiene infracciones de tránsito por 28 dólares, paga-
rá, en promedio, 35 dólares de multas por cada 250 millas de recorrido. Sumando estas cifras a los 20
dólares del costo de la molestia de ir en su automóvil y agregando después los 50 dólares de los costos
de combustible, aceite y mantenimiento, se tienen 105 dólares. Esto es más que los 100 dólares del
pasaje en autobús, lo que significa que lo mejor es tomar el autobús.
1.2 Los 18 dólares que pagó Mike por la entrada al concierto son un costo hundido o irrecuperable en el
momento de decidir si asistir o no al concierto. Por consiguiente, tanto para Jim como para Mike, los
costos y beneficios deben ser los mismos. Si el beneficio de asistir al concierto sobrepasa al costo de
estar bajo la lluvia, irán al concierto. Si no es así se quedarán en casa.
1.3 Deberá usar su cupón para el vuelo a Nueva Delhi, porque vale más ahorrar 120 dólares que 100
dólares.
1.4 Dos embarcaciones. Observe en la tabla 1.2 que si el costo marginal es 150 dólares, entonces, ahora
vale la pena botar la segunda embarcación (beneficio marginal = 180 dólares) pero no la tercera.
1.5 A 2 centavos por minuto, Susan deberá hablar con Hal 600 minutos por mes.

Tarifa de
larga distancia
(centavos por minuto)

Valor de
BM un minuto Costo de
adicional un minuto
8 adicional

Precio = 4 CM

2
1
Minutos
200 400 600 800 por mes

01_CHAPTER 1.indd 23 3/6/09 6:41:00 PM


01_CHAPTER 1.indd 24 3/6/09 6:41:00 PM
Capítulo

2
LA OFERTA Y LA DEMANDA

n 1979 trabajaba para el Gobierno federal y vivía en Washington, D.C. Frente a mi departa-
E mento había una gasolinera que contaba con 16 bombas y era más grande que la mayoría,
pero por lo demás era una típica moderna estación de autoservicio urbana.
En abril de ese año se interrumpió el abastecimiento de petróleo del Medio Oriente, lo
cual hizo subir enormemente los precios de la gasolina. Para evitar que los precios subieran aún más,
la administración Carter puso en marcha un complejo sistema de distribución y un estricto control
de precios. Uno de los resultados de lo anterior fue que muchos mercados urbanos recibían mucha
menos gasolina de la que los automovilistas deseaban comprar al precio regulado. En la gasolinera
que está frente a mi casa se formó una fila de automóviles que se extendía varias manzanas.
Las discusiones por los lugares en las filas eran frecuentes entre los automovilistas y en ocasio-
nes hasta llegaban a las manos. Incluso murió un individuo a causa de un disparo por colarse en la
fila. La tensión continuó hasta que las filas disminuyeron conforme pasaban los meses de vacaciones
de verano.
El sistema del gobierno para controlar la distribución y el precio pretendía realizar una tarea que
usualmente desempeña el mercado. Esta experiencia en Washington es similar a otras intervenciones
en diversos momentos y lugares. Por lo regular estos programas provocan confusión y conflicto. Por
supuesto, los mercados sin restricciones también pueden dar resultados que no sean agradables, pero
es raro que no logren distribuir los recursos existentes de una manera eficiente y sin problemas.

VISIÓN PRELIMINAR DEL CAPÍTULO


En este capítulo se analizará por qué la mayoría de las veces los mercados funcionan tan bien y por
qué con tanta frecuencia los intentos por dirigir la distribución resultan problemáticos. En la primera
parte del capítulo se verán los aspectos fundamentales del análisis de la oferta y la demanda. Primero
se repasarán las características descriptivas del análisis de la oferta y la demanda que ya se han estu-
diado en los cursos introductorios. Después se verá que de acuerdo con los atributos de compradores
25

02_CHAPTER 2.indd 25 3/6/09 6:33:30 PM


26 Capítulo 2 la oferta y la demanda

Parece que su hijo se quedó sin

autorización de Universal Press Syndicate. Derechos reservados.


Es una solicitud gasolina en una autopista de

Doonesbury copyright 1975 G. B.Trudeau. Reimpreso con la


por cinco galones Ohio. Él ha tenido que vivir en ¡Mi anillo
¡De
de gasolina, su un restaurante en la población Oh, ¡Dios lo y cera para
¿Qué es inmediato,
señoría, de esta de Hot Shoppe desde finales de La petición bendiga, señor! lacrar!
esto? señor!
mujer. noviembre. es justa. La
aprobaré. Sniff

y vendedores, los mercados competitivos no regulados producen el mejor resultado alcanzable, en el


sentido de que cualquier otra combinación de precio y cantidad será peor, por lo menos para algunos
de los vendedores o compradores.
A pesar de esto, con frecuencia los resultados del mercado no cuentan con la aprobación de la
sociedad. La preocupación por la gente pobre ha motivado a gobiernos de todas las sociedades occi-
dentales a intervenir de distintas maneras; por ejemplo, adoptando leyes que fijan precios superiores
o inferiores a su nivel de equilibrio. Dichas leyes, como se verá, casi siempre generan consecuencias
perjudiciales, aun sin proponérselo.
Una solución más eficiente al problema de los pobres es aumentar sus ingresos directamente. La
ley de la oferta y la demanda no puede ser abolida por los cuerpos legislativos. Pero éstos sí pueden
alterar las fuerzas que rigen la forma y la posición de las curvas de oferta y de demanda.
Por último, se examinará el análisis de la oferta y la demanda como un instrumento útil para
entender el efecto que tienen los impuestos sobre las cantidades y los precios de equilibrio. En par-
ticular, esto ayudará a refutar el mito de que un impuesto es pagado básicamente por el segmento
de población sujeto a la imposición; más bien, la carga del impuesto recae con más fuerza en el
segmento del mercado con menor capacidad para evadirlo.

CURVAS DE OFERTA Y DEMANDA


El instrumento básico para analizar los resultados del mercado es el análisis de la oferta y la deman-
da, ya conocido desde los cursos introductorios. Para empezar se da la siguiente definición, cómo la
más aceptada, de mercado.

Definición: un mercado consta de los compradores y vendedores de un bien o servicio.

Algunos mercados están confinados a un lugar y tiempo determinados. Por ejemplo, para una
subasta de antigüedades, todos los participantes, compradores y vendedores (o por lo menos sus
representantes) se congregan en un lugar. Otros mercados se expanden en un vasto territorio geo-
gráfico y los participantes jamás se encuentran, ni siquiera se ven unos a otros. La Bolsa de Valores de
Nueva York es un ejemplo de mercado de este tipo. También Internet proporciona acceso a merca-
dos de este tipo para una gran variedad de bienes.
La definición de mercado que se elige depende, algunas veces, de los intereses del observador.
En los casos antimonopolio, por ejemplo, la política actual prohíbe las fusiones entre empresas cuya
participación conjunta en el mercado exceda un determinado límite. Por consiguiente, los fiscales
que se oponen a la fusión tratarán de dar una definición de mercado lo más estrecha posible, ha-
ciendo con esto que la participación de los mercados combinados sea más grande. Las empresas
que se fusionan, en cambio, tienden a ver sus mercados en términos mucho más amplios, lo que,
naturalmente, hace parecer más pequeña su participación en los mercados combinados. Cuando
Stouffer´s Corporation se quería fusionar con Nestlé afirmó que las dos empresas estaban en el ramo

02_CHAPTER 2.indd 26 3/6/09 6:33:32 PM


CurvaS de oferta y demanda 27

FIGURA 2.1
Curva de demanda de
Precio langostas en Hyannis,
($/langosta) Mass., 20 de julio de
D 2009
20 la curva de demanda
indica las cantidades que
los compradores desean
16 adquirir a los diversos
precios. Su característica
12 principal es su pendiente
negativa; a medida que
disminuyen los precios
8 aumenta la cantidad
demandada. a esta
4 propiedad se le llama ley de
D la demanda.

Cantidad (miles de langostas/día)


0 1 2 3 4 5

de “alimentos congelados”. El Departamento de Justicia (de Estados Unidos) argumentó que las dos
empresas estaban en el negocio de la venta de “platos exóticos de lujo”. En general, como en este
caso particular, la mejor definición de mercado dependerá del propósito que se tenga.
Los economistas han observado a lo largo de los años que para algunos consumidores incluso las
diferencias más sutiles en los productos son de gran importancia y la tendencia es ir hacia definicio-
nes cada vez más estrechas de los bienes y de los mercados. Dos productos, por lo demás idénticos,
suelen clasificarse como distintos con sólo diferir en tiempo o lugar en que pueden obtenerse. Por
ejemplo, un paraguas en un día soleado es, en este sentido, un producto muy diferente a un paraguas
a la hora de un aguacero. Y los mercados de estos dos productos se comportan de manera verdadera-
mente diferente. (Mi editor me contó que en Manhattan un paraguas de no muy buena calidad en un
día lluvioso se vende en 10 dólares, mientras que en un día soleado vale tan sólo 5 dólares.)
Para ser un poco más concretos, a continuación se considerará el funcionamiento de un mer-
cado específico; por ejemplo, el mercado de 1½ libras de langosta en Hyannis, Massachusetts, el 20
de julio de 2009. La tarea es explicar tanto el precio de las langostas como la cantidad que se vende
en este mercado. Se empezará con la curva de demanda, una relación matemática sencilla que dice
cuántas langostas desean adquirir los compradores a los diferentes precios posibles (si todo lo demás
permanece constante). La curva DD en la figura 2.1, por ejemplo, dice que al precio de 8 dólares cada
una, la cantidad demandada es de 4 000 langostas, al precio de 20 dólares es de 1 000, etcétera.
Si a un visitante de Marte se le dijera nada más que una langosta tiene un precio de 8 dólares, no
tendría manera de saber si eso es caro o barato. En 1900, una langosta de ese precio hubiera estado
fuera del alcance de todos los consumidores, salvo de los más adinerados. En 2009, en cambio, a ese
precio podría ser considerada como una ganga. A menos que se diga otra cosa, el precio en el eje
vertical en la gráfica de la curva de demanda será el precio real del bien, lo que significa su precio en precio real de un producto
relación con el de los demás bienes y servicios. Así, por ejemplo, en el eje vertical de la figura 2.1 se es su precio con respecto
representan los precios de las langostas el 20 de julio de 2009 y el contexto en el que los compradores al precio de otros bienes y
servicios.
los interpretan es el conjunto de precios de todos los demás bienes en esa misma fecha.
En el análisis anterior se describe la curva de demanda como un cuadro que indica cuánto de
un producto desean adquirir los consumidores a los distintos precios. A esto se le conoce como in-
terpretación horizontal de la curva de demanda; en ella se parte del precio en el eje vertical y en el eje
horizontal se lee la correspondiente cantidad demandada. Por ejemplo, a 20 dólares por langosta, la
curva de demanda de la figura 2.1 indica que la cantidad requerida será de 1 000 por día.
Otra interpretación para la curva de demanda es partir de la cantidad en el eje horizontal y
luego leer en el eje vertical el precio de reserva marginal del consumidor (el precio que está dispuesto
a pagar). Así, si la cantidad vendida de langostas es 4 000 por día, en la curva de demanda de la figura

02_CHAPTER 2.indd 27 3/6/09 6:33:33 PM


28 Capítulo 2 la oferta y la demanda

2.1 se encuentra que el precio de reserva marginal de los compradores es 8 dólares por langosta. A
esta manera de leer la curva de demanda se le llama interpretación vertical.
La curva de demanda que se muestra en la figura 2.1 es lineal, pero en general las curvas de de-
manda no necesitan serlo. La característica esencial de estas curvas es que su pendiente es negativa:
la cantidad demandada aumenta a medida que el precio del producto baja. A esta propiedad se le
ley de la demanda conoce como ley de la demanda. Aunque, como se verá en el capítulo 4, en teoría es posible que
observación empírica de una curva de demanda tenga pendiente positiva, en la práctica esta situación nunca se encuentra. Sin
que cuando el precio de duda, la pendiente negativa de la curva de demanda coincide con nuestra idea intuitiva acerca de la
un producto baja, la gente manera en que la gente responde al aumento de los precios.
demanda mayor cantidad de él. Como se verá en el capítulo 4, normalmente hay dos razones independientes por las cuales la
cantidad demandada disminuye a medida que aumenta el precio. Una es que mucha gente busca un
sustituto cercano. Así, si la langosta se encarece, algunos consumidores la cambiarán por camarones,
otros por carne o por pollo. La otra razón es que la gente ya no puede comprar la misma cantidad que
antes. Después de todo, el ingreso no puede estirarse. Si aumenta el precio, ya no es posible comprar
tanto como antes, a menos que se compre menos de otra cosa.
La curva de demanda de un bien, como se verá con más detalle en el capítulo siguiente, es un
resumen de los diversos cálculos costo-beneficio que realizan los compradores respecto de ese bien.
La pregunta que se plantea cada persona es, “¿deberé comprar este producto?” (y por lo general,
“si es así, ¿cuánto?”). En estos cálculos, la parte del costo es simplemente el precio del producto (y
de manera implícita, los otros bienes o servicios que podrían comprarse con ese mismo dinero).
La parte del beneficio es la satisfacción que proporciona el producto. La pendiente negativa de la
demanda indica que a medida que aumenta el precio de un producto, el criterio costo-beneficio será
satisfecho por cada vez menos compradores potenciales.
En el lado de los vendedores del mercado, el instrumento analítico correspondiente es la curva
de la oferta. Una curva hipotética para el mercado de langostas es la recta SS en la figura 2.2. Aquí
también, la forma lineal de esta curva particular no es en general un rasgo característico de ella.
Lo que tienen en común estas curvas es su pendiente positiva: la cantidad que se ofrece aumenta a
ley de la oferta observación medida que crece el precio del producto. A esta propiedad se le llama ley de la oferta. Para que un
empírica de que cuando sube oferente esté dispuesto a vender un producto, su precio debe saldar el costo marginal de producirlo o
el precio de un producto, las de adquirirlo. Como se verá en el capítulo 9, el costo de producir unidades adicionales tiende a subir
empresas ponen a la venta más a medida que se producen más unidades, en especial en el corto plazo. Cuando éste es el caso, una
de ese producto.
producción mayor es rentable sólo a un precio más alto.
En nuestro mercado de langostas, las razones para que ocurra lo anterior son claras. Primero el
productor pesca las langostas cerca de la costa y para atrapar más tiene que alejarse de la costa. Cuan-
tas más langostas trate de pescar, más millas tendrá que recorrer, y más aumentará el costo total.

FIGURA 2.2
Curva de oferta de Precio
langostas en Hyannis, ($/langosta)
Mass., 20 de julio de
S
2006
la pendiente positiva 20
de la curva de oferta
refleja el hecho de que 16
los costos tienden a
aumentar a medida que los
productores aumentan la 12
producción a corto plazo.
8

4
S
Cantidad (miles de langostas/día)
0 1 2 3 4 5

02_CHAPTER 2.indd 28 3/6/09 6:33:35 PM


equilibrio Cantidad y preCio 29

Otro factor que contribuye a la pendiente positiva es la sustitución por parte del pescador. A
medida que el precio de la langosta aumenta, más pescadores producen langosta en lugar de seguir
pescando bacalao, por ejemplo.
Como en el caso de las curvas de demanda, las curvas de oferta pueden interpretarse horizontal
o verticalmente. En la interpretación horizontal se parte del precio, desde donde se va a la curva de
oferta para después leer en el eje horizontal la cantidad que el vendedor desea vender a ese precio.
Por ejemplo, en la figura 2.2 se ve que al precio de 8 dólares por langosta, los vendedores desean
vender 2 000 langostas por día.
En la interpretación vertical se parte de una cantidad, y después se asciende hasta la curva de de-
manda para leer en el eje vertical el costo marginal correspondiente. Por ejemplo, si los vendedores
en la figura 2.2 están ofreciendo 5 000 langostas por día, el costo de oportunidad de la última langosta
ofrecida por el vendedor marginal sería de 20 dólares. En otras palabras, la curva de la oferta dice que
el costo marginal de ofrecer la langosta número 5 000 es de 20 dólares. Si alguien puede vender la
langosta número 5 001 a menos de 20 dolares habrá tenido un incentivo para hacerlo, en cuyo caso,
para empezar, la cantidad de langostas vendidas al precio de 20 dólares no habrá sido 5 000 por día.
Si se sigue un razonamiento similar se encuentra que, si la cantidad ofrecida es de 2 000 langostas por
día, el costo marginal de una langosta más deberá ser mayor de 8 dólares.
Otra manera de describir la curva de oferta es viéndola como el conjunto de pares precio-canti-
dad con los que el proveedor está satisfecho. Aquí, el término “satisfecho” tiene un significado técni-
co, cada punto de la curva de la oferta representa la cantidad que el productor desea vender, dado el
precio al que se enfrenta. Es claro que los vendedores estarían más contentos si obtuvieran precios más
altos para sus ofertas. Pero se considerarían en una peor situación si a cada precio dado se vieran obli-
gados a vender más o menos de la cantidad que indica la curva de oferta. Si, por ejemplo, en la figura
2.2 el precio de las langostas es de 8 dólares, los proveedores no estarían satisfechos si vendieran más
o menos de 2 000 langostas por día.
Para la curva de demanda puede darse una descripción similar. Es el conjunto de pares precio-
cantidad con los que los compradores estarán satisfechos precisamente en el mismo sentido. A cada
precio dado, se considerarían en peor situación si se vieran obligados a comprar una cantidad mayor
o menor que la correspondiente en la curva de demanda.

EQUILIBRIO CANTIDAD Y PRECIO


Observando las curvas de oferta y demanda, se pueden obtener la cantidad y el precio de equilibrio de
las langostas. Es el par precio-cantidad al cual tanto compradores como vendedores quedarán satis-
fechos. Dicho de otra manera, es el par precio-cantidad en el que las curvas de oferta y de demanda
se intersecan. En la figura 2.3 el equilibrio en nuestro mercado de langostas es el punto en el que se
venden 3 000 langostas al precio de 12 dólares cada una.

Precio FIGURA 2.3


($/langosta) Equilibrio en el
S mercado de langostas
D la intersección de las
20
curvas de oferta y de
demanda representa
16 el par precio-cantidad
con el que todos los
participantes del mercado
12
quedan “satisfechos”:
los compradores están
8 adquiriendo la cantidad
que desean a ese precio
y los vendedores están
4
vendiendo la cantidad que
S D quieren.
Cantidad (miles de langostas/día)
0 1 2 3 4 5

02_CHAPTER 2.indd 29 3/6/09 6:33:36 PM


30 Capítulo 2 la oferta y la demanda

FIGURA 2.4
Exceso de oferta y Precio
exceso de demanda ($/langosta)
Si el precio es superior
Exceso S
al precio de equilibrio, D
20 de oferta
hay un exceso de oferta
o excedente. Si el precio
es inferior al precio de 16
equilibrio, hay un exceso de
demanda o déficit.
12

4 Exceso
de demanda
S D
Cantidad (miles de langostas/día)
0 1 2 3 4 5

Con cualquier otro par distinto al de la figura 2.3, los compradores o los vendedores, o ambos,
quedarán insatisfechos en el sentido que se describió antes. Si por alguna razón resulta que los precios
son superiores al nivel de equilibrio, 12 dólares, los vendedores quedarán frustrados. A 16 dólares, por
ejemplo, los compradores desearán adquirir sólo 2 000 langostas, mientras que los vendedores ofrece-
rán 4 000. (Vea la figura 2.4.) Los compradores quedarán satisfechos al precio de 16 dólares, pero los
vendedores no. A la situación en la que el precio es superior a su valor de equilibrio se le conoce como
exceso de oferta monto en situación de exceso de oferta, o excedente. A 16 dólares hay un exceso de oferta de 2 000 langostas.
el que la cantidad ofrecida Si, en cambio, sucede que el precio es inferior al precio de equilibrio, 12 dólares, los compradores
es superior a la cantidad serán los insatisfechos. Si, por ejemplo, el precio es de 8 dólares, los compradores querrán comprar
demandada. 4 000 langostas, mientras que los oferentes estarán dispuestos a vender sólo 2 000. A una situación en
exceso de demanda la que el precio es inferior al precio de equilibrio se le conoce como de exceso de demanda o déficit. Si
monto en el que la cantidad el precio en este mercado de langostas es de 8 dólares, habrá un exceso de demanda de 2 000 langos-
demandada es superior a la tas. Al precio de equilibrio del mercado, 12 dólares, tanto el exceso de demanda como el exceso de
cantidad ofrecida. oferta son exactamente cero.

EjERCICIO 2.1
A un precio de 4 dólares en este mercado hipotético de langostas, ¿de cuánto será el exceso de
demanda? ¿De cuánto será el exceso de oferta a un precio de 20 dólares?

AjUSTE AL EQUILIBRIO
Cuando el precio difiere del precio de equilibrio, la compraventa en el mercado se verá restringida
—por la conducta de los compradores si el precio es superior al de equilibrio, por la conducta de los
vendedores si es inferior—. A cualquier precio distinto al de equilibrio, una u otra parte del mercado
quedará insatisfecha. A precios superiores al precio de equilibrio, por ejemplo los vendedores no
venderán tanto como desean. Un vendedor insatisfecho tenderá a reducir el precio. Después de todo,
la regla en el mercado de los mariscos es “véndelo antes de que huela”. A un precio de 16 dólares
cada una, se venderán 2 000 langostas, pero otras 2 000 se quedarán. Cada vendedor pensará, acer-
tadamente, que si baja un poco su precio, mientras los demás siguen vendiendo a 16 dólares, podrá
vender toda la langosta que le queda. Los compradores abandonarán a los vendedores que cobran
16 dólares en favor de los que sólo cobran 15.95. Pero entonces los vendedores abandonados tendrán

02_CHAPTER 2.indd 30 3/6/09 6:33:37 PM


algunaS propiedadeS del equilibrio deSde el punto de viSta del bieneStar 31

motivo para bajar sus precios. Y si todos los vendedores bajan el precio a 15.95, otra vez cada uno
tendrá una gran cantidad de langostas sin vender. Esta presión hacia abajo sobre los precios persistirá
en tanto haya un vendedor insatisfecho; es decir, hasta que el precio baje hasta su valor de equilibrio.
Cuando el precio es inferior a 12 dólares, los compradores están insatisfechos. En estas condi-
ciones los vendedores se darán cuenta de que pueden aumentar sus precios y a pesar de eso vender
tanto como deseen. Esta presión hacia arriba sobre el precio persistirá hasta que llegue a su valor de
equilibrio. Dicho de otra manera, los consumidores comenzarán a competir, unos con otros, con la
esperanza de ver satisfecha su demanda.
Una propiedad extraordinaria de este proceso hacia el equilibrio es que nadie lo planea o lo di-
rige de manera consciente. Los pasos que los consumidores y los productores tienen que dar para ir
hacia el equilibrio suelen ser excesivamente complejos. Por ejemplo, cuando los productores tratan
de expandir sus operaciones tienen que elegir entre un enorme y desconcertante menú de opciones.
Los compradores, por su parte, se encuentran con millones de alternativas para gastar su dinero.
Sin embargo, el ajuste hacia el equilibrio se da de manera más o menos automática a partir de las
reacciones naturales del interés personal de los individuos al enfrentarse a un excedente o al déficit.

ALGUNAS PROPIEDADES DEL EQUILIBRIO


DESDE EL PUNTO DE VISTA DEL BIENESTAR
Dadas las características —gustos, capacidades, conocimientos, ingresos, etc.— de compradores y
de vendedores, el equilibrio posee algunas propiedades interesantes. En concreto, puede decirse que
no hay ninguna redistribución que pueda mejorar la situación de alguna persona sin perjudicar la
posición de por lo menos algunas otros. Sin embargo, si precio y cantidad toman valores distintos a los
de equilibrio, siempre será posible elegir otra combinación de manera que beneficien por lo menos a algunas
personas sin dañar a otras.
Continuando con el ejemplo de las langostas, suponga que el precio es 8 dólares, por lo que los
oferentes únicamente venden 2 000 langostas. Como se indica en la figura 2.5, la interpretación ver-
tical de la curva de demanda dice que cuando nada más hay 2 000 langostas disponibles, los compra-
dores están dispuestos a pagar 16 dólares. De manera similar, la interpretación vertical de la curva de
oferta dice que cuando se suministran 2 000 langostas por día, el costo marginal de suministrar una
langosta más es de sólo 8 dólares. Cuando para el comprador, el valor de la última langosta pescada
(16 dólares) es más alto que el costo de pescarla, es posible negociar.

FIGURA 2.5
Una oportunidad de
Precio mejora en el mercado
($/langosta) de langostas
S
Cuando la cantidad que se
D comercializa en el mercado
20
es menor (o mayor) que
la cantidad de equilibrio,
16 siempre es posible
redistribuir recursos de
12
manera que se beneficie a
algunas personas sin dañar
a otras. en este caso, un
8 comprador insatisfecho
puede pagar al vendedor
10 dólares por una langosta
4
más, beneficiando a las dos
S D partes.
Cantidad (miles de langostas/día)
0 1 2 3 4 5

02_CHAPTER 2.indd 31 3/6/09 6:33:38 PM


32 Capítulo 2 la oferta y la demanda

Suponga, por ejemplo, que un comprador insatisfecho ofrece a un proveedor 10 dólares por una
langosta. A este precio el proveedor venderá con gusto una langosta más (ya que después de 2 000
langostas, pescar una langosta más sólo cuesta 8 dólares). Esta transacción mejorará la posición del
comprador en 6 dólares (la diferencia entre 16 dólares, el valor que le atribuye a la langosta y los 10
que paga por ella). La posición del vendedor también mejorará en 2 dólares (la diferencia entre los
10 dólares que obtuvo y los 8 que cuesta pescar una langosta más). Con esta transacción nadie sufre
ningún daño (¡excepto la langosta extra!) y los participantes obtienen de ella un beneficio adicional
de 8 dólares (6 del comprador y 2 del vendedor). Un argumento similar puede elaborarse respecto de
cualquier precio inferior al valor de equilibrio. Dado cualquier precio de este tipo, siempre se puede
beneficiar a alguna persona sin dañar a nadie.
¿Y qué pasa si se parte de un precio más alto que el de equilibrio? Suponga que el precio es de
16 dólares, con lo que la demanda de los compradores se limita a 2 000 langostas. (De nuevo, vea la
figura 2.5.) Ahora un vendedor insatisfecho puede proponer una transacción que beneficie tanto al
vendedor como a los compradores. Suponga, por ejemplo, que un vendedor ofrece una langosta más
a la venta por 14 dólares. Como los compradores valoran una langosta adicional en 16 dólares, quien
la compre se verá beneficiado con 2. Y como pescar las langostas sólo cuesta 8 dólares, el beneficio
para el vendedor será de 6. Otra vez, con esta operación no se daña a nadie, y de nuevo las dos partes
ganan 8 dólares en total.
Por lo tanto, no importa si al comienzo un precio es superior o inferior a su valor de equilibrio,
siempre será posible una transacción mutuamente favorable. En capítulos posteriores se examinarán
con más detalle las propiedades del sistema de mercado respecto al bienestar. Pero, por ahora es
suficiente decir que el precio y la cantidad de equilibrio constituyen el mejor resultado que puede
obtenerse dadas las características y dotaciones iniciales de vendedores y compradores.

LOS MERCADOS LIBRES Y LA POBREZA


El hecho de que el equilibrio de mercado sea eficiente en el sentido que se acaba de describir no
significa que necesariamente sea deseable en sentido absoluto. Puede ocurrir, por ejemplo, que todos
los mercados estén en perfecto equilibrio y, sin embargo, mucha gente carezca del ingreso suficiente
para comprar incluso lo estrictamente necesario para la vida. Decir que el equilibrio de mercado es
eficiente no cuestiona que ser pobre sea difícil, con frecuencia incluso doloroso. Esta eficiencia sólo
dice que, dado el bajo ingreso de los pobres, el libre intercambio les permite vivir lo mejor que pueden.
Aun si se está de acuerdo con esto es posible creer que es deseable proporcionar asistencia pública a
los pobres.
La preocupación por el bienestar de los pobres motiva a la mayoría de las sociedades a tratar
de intervenir, como en el ejemplo del control del precio de la gasolina que se mencionó antes. El
problema, como en dicho ejemplo, es que las intervenciones suelen producir, sin proponérselo, con-
secuencias perjudiciales. En efecto, muchas ocasionan más daño que beneficio.
Se verá que una mejor comprensión del funcionamiento del mecanismo del mercado ayudará a
evitar muchas de las peores consecuencias del enfoque actual.

EjEMPLO 2.1 Indemnización por negarse a abordar


¿Qué consecuencias sobre la distribución y la eficiencia tiene manejar el exceso de demanda en vue-
los con exceso de reservaciones mediante la política “el que llega primero es atendido antes”, en
comparación con el mecanismo de subasta?
Las aerolíneas comerciales suelen emitir más reservaciones que la cantidad de asientos de que
disponen en un vuelo. Sin embargo, esto rara vez ocasiona problemas debido a que muchas personas
que tienen reservación no la utilizan. Pero hay ocasiones en que llegan 160 personas con reservación
a un vuelo en el que sólo hay 150 asientos disponibles. Hasta antes de finalizar la década de los años
setenta, las aerolíneas solucionaban este problema empleando el principio “el que llega primero es
atendido antes”.
Esta solución no resultaba adecuada para los pasajeros que no podían retrasarse en llegar a su
destino final. Con este problema en mente, el Civil Aeronautics Board (CAB), la dependencia del

02_CHAPTER 2.indd 32 3/6/09 6:33:38 PM


loS merCadoS libreS y loS pobreS 33

gobierno de Estados Unidos encargada de reglamentar la in-


dustria de la aviación comercial, propuso un reglamento sen-
cillo. Se pidió a las aerolíneas que cuando hubiera un exceso
de pasajeros con reservación buscaran voluntarios dispuestos
a dejar sus lugares a cambio de un pago en efectivo o en espe-
cie, por ejemplo, un vuelo gratis de avión. Y se exigió que las
aerolíneas incrementaran su oferta hasta tener los voluntarios
suficientes.
La ventaja de la propuesta del CAB es que permite a los
pasajeros decidir de acuerdo con la urgencia de sus actividades.
Las personas con compromisos importantes simplemente no
se ofrecen como voluntarios. Otros pueden aceptar esperar
unas cuantas horas a cambio de varios cientos de dólares o de
un vuelo gratis a Hawai. En comparación con la solución “el
que llega primero es atendido antes”, la propuesta del CAB
prometía mejores resultados para todos los pasajeros.
O, en cualquier caso, así lo parecía. Un grupo de acción
en favor de los consumidores objetó la propuesta del CAB di-
ciendo que era injusta para los pasajeros de bajos ingresos. Lo
que argumentaba este grupo era que el método de la subasta
ocasionaba que siempre los pasajeros más pobres fueran los ¿por qué en un vuelo con exceso de reservaciones es mejor una
que esperaran hasta el siguiente vuelo. subasta que la política “el que llega primero es atendido antes”,
Ahora, claro que es más probable que una persona pobre para distribuir los lugares?
considere que un pago en efectivo es una razón convincente
para ofrecerse como voluntario. Al hacerlo, la persona piensa
que por el pago en efectivo bien vale la pena esperar. En realidad el mundo sería un mejor lugar si
la gente pobre tuviera ingresos más altos y no cayera en la tentación, a causa de su pobreza, de
renunciar a sus lugares en los vuelos. Pero este grupo de acción a favor de los consumidores no
proponía dar a los pobres un ingreso más alto. Lo que quería era que la industria siguiera haciendo
esperar el vuelo siguiente a los pasajeros de aviones con exceso de reservaciones sin tener en cuenta
la importancia que para ellos tuviera permanecer en dicho vuelo.
Es difícil entender de qué manera se ayudaría a los pobres evitando que ganen un dinero extra
por ofrecerse como voluntarios para esperar el vuelo siguiente. Finalmente, para beneficio de los
viajeros de todos los niveles de ingresos, el CAB optó por su propuesta de indemnización por negarse
a abordar.

Muchas de las críticas al sistema de mercado argumentan que no es justo racionar los bienes
y servicios de acuerdo con cuánto está dispuesta la gente a pagar por ellos. Este criterio, dicen, no
toma en cuenta las necesidades de los pobres. Pero como lo ilustra claramente el ejemplo 2.1, el
resto de los sistemas de distribución están plagados de serias contradicciones. Considere de nuevo
el mercado hipotético de langostas. Suponga que existe preocupación porque el precio de equilibrio,
12 dólares, no permitirá a muchas persona pobres experimentar el placer de una merecida cena de
langosta. Y suponga que, con esto en mente, se adopta un sistema en el que periódicamente se re-
galen langostas a los pobres. ¿Representaría este sistema una clara mejoría a los ojos de alguien que
siente compasión por los pobres?
La respuesta, como en el ejemplo 2.1, es que por el mismo precio se puede hacer algo mejor. Si
una persona pobre, o incluso una persona rica, no compra langostas porque son demasiado caras, lo
que en realidad está diciendo es que prefiere gastar su dinero en otras cosas. Si se le regala una lan-
gosta, ¿qué haría con ella? En un mundo ideal, de inmediato la vendería a alguien dispuesto a pagar
por ella el precio de equilibrio de 12 dólares. Se sabe que tales personas deben existir porque algunas
de las langostas que deberían haber sido vendidas a 12 dólares se regalaron a los pobres. Si la persona
pobre vende la langosta a una de las personas dispuestas a pagar, será benéfico para ambas —para
el comprador porque de otra manera no hubiera podido comprar la langosta y para el vendedor
porque para él la langosta vale menos que 12 dólares—.

02_CHAPTER 2.indd 33 3/6/09 6:33:40 PM


34 Capítulo 2 la oferta y la demanda

La dificultad práctica, como se verá en capítulos posteriores, es que este pobre hipotético nece-
sitará tiempo y esfuerzo para hallar un comprador de la langosta. A la larga, es probable que se coma
la langosta, y; no cabe duda que la disfrutará. Pero de acuerdo con sus cálculos, él hubiera preferido
los 12 dólares.
El problema es el mismo con el control de los precios de la gasolina. Esos controles se pusieron
en funcionamiento creyendo en verdad que eran necesarios para proteger a los pobres de los precios
de la gasolina claramente altos. Sin embargo, el resultado fue que indujeron un montón de conduc-
tas que no ayudaron a nadie, ni a los ricos ni a los pobres.
A pesar de afirmaciones contrarias de los críticos del sistema de mercado, las personas son muy
sensibles a los precios de la energía al decidir cómo gastar su ingreso. Si la gasolina se vende a 3
dólares por galón, por ejemplo, muchas personas se organizarán para viajar al trabajo en un solo
automóvil o comprarán vehículos que ahorren combustible, cosa que no harían si el precio de la
gasolina fuera de 1.50 por galón. Al considerar si vale o no la pena realizar un largo viaje tomarán en
cuenta el precio de la gasolina.
Haya o no escasez de gasolina, es del interés de todos —ricos o pobres— usarla para actividades
que sean de más valor para la gente. El costo de una política que no haga lo anterior es en especial
alto cuando hay escasez. Vender gasolina a un precio inferior al de equilibrio es precisamente una de
estas políticas porque favorece que las personas la desperdicien.

CONTROL DE LOS ALQUILERES


Se ha dicho que la manera más segura de destruir una ciudad, poco antes de que caiga en ella una
bomba atómica, es aprobar una ley de control sobre el alquiler. Estas leyes, como muchas otras,
están motivadas por una sincera preocupación por el bienestar de los ciudadanos de ingresos bajos.
Pero sus dramáticas consecuencias no dejan de ser perjudiciales por no ser intencionadas.
Una vez más, lo único que se necesita para ver con claridad
el carácter de estos problemas es el análisis básico de la oferta y
la demanda. En la figura 2.6 se representan las curvas de oferta
y de demanda de un mercado hipotético de vivienda. El precio
de alquiler de equilibrio en este mercado es de 600 dólares al
mes, y a este precio habrá 60 000 viviendas alquiladas. Pero el
ayuntamiento aprueba una ley que fija el alquiler en Rc = 400
dólares al mes, es decir, 200 más abajo que el valor de equilibrio.
En este caso, Rc es un precio techo para el alquiler porque no
se permite que suban por encima de este valor. A 400 dólares
al mes, los arrendatarios desearían alquilar 80 000 viviendas, pe-
ro los propietarios sólo están dispuestos a proporcionar 40 000.
Hay un exceso de demanda de 40 000. Y si este precio regulado
del alquiler permanece fijo en 400 dólares al mes, el exceso de
demanda aumentará a medida que la población crezca y la infla-
ción reduzca el valor del dinero.
En un mercado no regulado la respuesta inmediata sería el
aumento continuo del alquiler. Pero en este caso la ley impide
subirlo más allá de Rc. Sin embargo, existen otras maneras en
las que la presión del exceso de demanda puede hacerse sentir.
Una de ellas es que los propietarios gasten menos en el mante-
nimiento de las unidades alquiladas. Aunque haya dos arrenda-
tarios llamando a la puerta de cada vivienda vacía, los drenajes
¿por qué el mantenimiento de los edificios de viviendas con alquiler
controlado no es tan bueno como el de los que no lo tienen?
tapados, las paredes sin pintar o los termostatos descompuestos
no recibirán una pronta atención.
Pero éstos no son los problemas más serios. Como se ve en
precio techo nivel por encima la figura 2.6, ante una oferta de sólo 40 000 viviendas por mes, los arrendatarios están dispuestos a
del cual no se permite que pagar hasta 800 dólares al mes por una vivienda (otra vez la interpretación vertical de la curva de de-
suba el precio de un bien. manda). Esta presión casi siempre encuentra una vía de escape, sea legal o ilegalmente. En la ciudad

02_CHAPTER 2.indd 34 3/6/09 6:33:42 PM


loS merCadoS libreS y loS pobreS 35

FIGURA 2.6
Control del alquiler
Renta ($ al mes) Si el alquiler regulado se fija
S en 400 dólares mensuales,
D el exceso de demanda será
de 40 000 viviendas al mes.
800

600

Rc = 400

200 Exceso de
demanda D
S
Cantidad (miles de viviendas/mes)
0 20 40 60 80 100

de Nueva York, por ejemplo, no es poco usual ver “comisiones por corretaje” o “fianzas” hasta de
varios miles de dólares. Los propietarios que no pueden cobrar el precio de alquiler de equilibrio
del mercado por una vivienda tienen la opción de convertirlo en un condominio, lo que les permite
vender su propiedad a un precio más cercano a su verdadero valor económico.
Aun cuando los propietarios de viviendas con alquiler regulado no puedan incrementar sus pre-
cios mediante alguna de estas maneras, se producen serios problemas de distribución. Viudas cuyos
hijos ya se hayan ido de casa se aferran a sus viviendas de siete habitaciones porque son más baratas
que una posibilidad de vivienda no protegida por el control de renta. Para todos sería mejor si deja-
ran ese espacio a familias más grandes. Pero mientras haya control de la renta no tienen incentivo
alguno para hacerlo.

Suponga que disminuye el precio regulado del alquiler a 200 dólares al mes. ¿De cuánto es el EjEMPLO 2.2
exceso de demanda y de cuánto es el exceso de demanda en comparación con los alquileres
limitados (con menos rigor) a 400 dólares al mes?
A 200 dólares al mes, los compradores desearán alquilar 100 000 viviendas, pero los oferentes sólo
desearán poner en el mercado 20 000. Por lo tanto el exceso de demanda será de 80 000 viviendas. El
exceso de demanda es mayor que cuando el precio regulado de alquiler es de 400 dólares al mes, que
en este caso es de 40 000 viviendas.

EjERCICIO 2.2
En el mercado de viviendas descrito en la figura 2.6, ¿qué pasaría si el precio regulado de alqui-
ler fuera de 625 dólares al mes?

En respuesta a los tipos de problemas que se describieron antes, algunos programas de control
de alquiler se han modificado permitiendo al propietario incrementar el alquiler cuando se va un
inquilino. Estos cambios reducen pero no eliminan la mala distribución e, incluso, suelen crear otros
problemas. Por ejemplo, un propietario de una vivienda que sabe que si su inquilino se va podrá
aumentar el alquiler hará todo lo legalmente posible para que la vida de su inquilino sea menos
placentera.
Para ayudar a la gente pobre existen maneras mucho más efectivas que darles gasolina barata,
viviendas con el alquiler regulado o langostas gratis. Una de ellas es darles un ingreso adicional y
dejarles decidir cómo gastarlo.

02_CHAPTER 2.indd 35 3/6/09 6:33:43 PM


36 Capítulo 2 la oferta y la demanda

En el capítulo 18 se examinan algunos de los problemas prácticos que involucra transferir un


mayor poder de compra a las manos de los pobres. En pocas palabras, el principal problema es que
el dinero vaya a los que realmente lo necesitan sin atraer a otros que pueden valerse por sí mismos.
Pero el razonamiento económico, como se verá, sugiere también maneras prácticas de superar este
problema. No existen soluciones sencillas o fáciles. Pero dadas las enormes pérdidas ocasionadas por
las políticas que mantienen los precios debajo de su nivel de equilibrio, este tema merece la más seria
atención.

APOYOS A LOS PRECIOS


El control de alquiler es un ejemplo de precio techo que impide que el precio suba a su nivel de
equilibrio. Para muchos productos agrícolas, las políticas públicas han impuesto no precios techo
precio suelo precio mínimo sino precios de apoyo o precios suelo, con los cuales se mantiene el precio por encima de su nivel de
de un bien, establecido por equilibrio. A diferencia de los precios techo, en los cuales sólo se requiere fijar un valor más allá del
ley, y mantenido mediante el cual los precios no pueden elevarse, los precios suelo requieren que el gobierno se convierta en un
ofrecimiento del gobierno de comprador activo en el mercado.
comprar ese bien a ese precio. En la figura 2.7, por ejemplo, se representa un precio suelo, PS, en el mercado del frijol de soya.
Como PS es superior al precio de equilibrio, hay un exceso de oferta de 200 000 toneladas/año. Para
mantener el precio en PS = 400 dólares/tonelada, el gobierno tiene que comprar 200 000 toneladas
de frijol de soya cada año. De otra manera los agricultores tendrían un fuerte incentivo para dismi-
nuir sus precios.
Un objetivo importante de la regulación de los precios agrícolas es asegurar que sean lo sufi-
cientemente altos para proporcionar un ingreso adecuado a las familias de los agricultores. En la
práctica, sin embargo, las regulaciones de precios han demostrado ser un instrumento ineficiente y
costoso. Uno de los problemas es el empleo del excedente que compra el gobierno. Para producir
este excedente se requieren trabajo, capital, fertilizantes y otros valiosos insumos. Sin embargo, el
excedente simplemente no se aprovecha y se deja deteriorar en los almacenes del gobierno. Otro
problema es que gran parte del excedente es producido por grandes explotaciones agrícolas, cuyos
propietarios no tienen necesidad de ayuda. Por cada dólar que las regulaciones de precios ponen en
las manos de una familia de agricultores necesitada, varios dólares más van a manos de prósperos
empresarios agrícolas. La regulación de precios incrementa también el costo de los alimentos de
todas las familias y, con frecuencia, el de los bienes no regulados. (Vea más adelante en este capítulo
el ejemplo 2.3.) Si la sociedad desea subsidiar a las familias de agricultores pequeños, existen medios
más eficientes y directos que las regulaciones de los precios agrícolas.

FIGURA 2.7
Regulación del precio Precio ($/tonelada)
en el mercado del frijol
de soya 600 S
D
para que una regulación Exceso
de precios tenga impacto, de oferta
500
debe fijarse más alto que
el precio de equilibrio. Su
Ps = 400
efecto es crear exceso
de oferta, la cual compra
después el gobierno. 300

200

100
S D
Cantidad (miles de toneladas/año)
100 200 300 400 500 600

02_CHAPTER 2.indd 36 3/6/09 6:33:44 PM


determinanteS de la oferta y la demanda 37

FUNCIONES DE DISTRIBUCIÓN
Y DE RACIONAMIENTO DE LOS PRECIOS
Los precios tienen dos funciones importantes. Primero, racionan la oferta de bienes existentes. La es-
casez es la característica universal de la vida económica. La gente quiere más de prácticamente todo
lo que puede ofrecerse a un precio de cero. Los precios de equilibrio restringen estos comportamien-
tos excesivos mediante el racionamiento de las provisiones escasas a los usuarios que les dan mayor
valor. Ésta es la función de racionamiento del precio. Es una función a corto plazo en el sentido de función de racionamiento
que se enfoca en la distribución del producto existente. del precio proceso a través
La segunda función del precio es la de servir de señal para repartir los recursos productivos del cual el precio dirige la
entre los diversos sectores de la economía. En industrias en las que hay exceso de demanda, las oferta de un producto hacia
empresas pueden cobrar más de lo necesario para pagar sus costos de producción. Las ganancias los usuarios que más lo
valoran.
resultantes actúan como una zanahoria que atrae más recursos hacia esa industria. El otro lado de
la moneda es que las pérdidas actúan como la palanca que expulsa recursos de industrias en las que
existe un exceso de oferta. Ésta es la llamada función distributiva del precio, la fuerza impulsora función distributiva del
que actúa detrás de la mano invisible de Adam Smith. Se trata de una función a largo plazo en el precio proceso mediante el
sentido de que dirige los recursos para que emigren de industrias con exceso de oferta a aquellas cual el precio actúa como una
con exceso de demanda. señal que aleja los recursos de
El control de alquileres trastorna estas dos funciones del mecanismo del precio. La función de la producción de bienes cuyos
precios se encuentran por
racionamiento se distorsiona por los mecanismos alternativos que distribuyen la vivienda sin tomar
debajo del costo y los dirigen
en consideración el valor que la gente le da a la misma. Las necesidades de los arrendadores son
a la producción de bienes
relegadas a segundo plano. Las rentas artificialmente bajas distorsiona la función distributiva del cuyos precios son superiores
precio mandando una señal falsa a los inversionistas acerca de la necesidad de más viviendas. Con al costo.
el control de los alquileres los constructores de viviendas ganan menos de lo que podrían obtener
si invirtieran su dinero en otra cosa. La cruel ironía es que la apremiante necesidad de muchas
comunidades en las que hay control de alquileres es de más viviendas para bajos ingresos, no de
menos —que es precisamente lo que el mercado produciría por sí solo si a los pobres se les diera
más dinero—.

DETERMINANTES DE LA OFERTA
Y LA DEMANDA
El análisis de la oferta y la demanda es útil no sólo en relación con la idea normativa que ofrece sobre
cuestiones de políticas públicas sino también en relación con una gran variedad de propósitos des-
criptivos. Lo más importante es que predice cómo responderán los precios y las cantidades de equili-
brio a los cambios de las fuerzas del mercado. Como las curvas de oferta y demanda se intersecan en
determinados precios y cantidades de equilibrio, cualquier cosa que desplace estas curvas alterará,
de manera predecible, los valores de equilibrio. En los siguientes capítulos se investigarán con detalle
las fuerzas que determinan la forma y posición de las curvas de demanda del mercado. Por ahora se
discutirán sólo algunas de estas fuerzas cuyo papel es intuitivamente claro.

DETERMINANTES DE LA DEMANDA

Ingreso
Con la mayoría de los bienes ocurre que la cantidad demandada a cualquier precio aumenta con el
ingreso. Los bienes que tienen esta propiedad se llaman bienes normales. Los bienes inferiores (como
carne picada con alto contenido de grasa) son la excepción; la cantidad demandada de estos bienes
disminuye con el ingreso. La idea es que los consumidores dejan de comprar esos bienes en favor
de sustitutos de mayor calidad (como, por ejemplo, carne magra en el caso de la carne molida) tan
pronto como sus ingresos les permiten hacerlo.

02_CHAPTER 2.indd 37 3/6/09 6:33:44 PM


38 Capítulo 2 la oferta y la demanda

Gustos
Los gustos varían con las personas y con el tiempo. En las sociedades occidentales la cultura inculca
el gusto por sentarse en muebles acojinados, mientras que en muchas sociedades orientales la gente
está acostumbrada a sentarse en el suelo con las piernas cruzadas. Por lo tanto, la demanda de sillas
tiende a ser mayor en Occidente que en Oriente. Asimismo, la demanda de faldas cortas tiende a
variar de una década a otra.

Precios de sustitutos y complementos


El tocino y los huevos desempeñan un papel complementario en la dieta de algunas personas. Para
ellas un fuerte aumento del precio del tocino no sólo lleva a reducir la demanda de éste sino también
la de huevos. Tales bienes se consideran complementarios: el aumento del precio de uno disminuye la
demanda del otro. En el caso de los sustitutos cercanos, como por ejemplo, café y té, el aumento del
precio de uno de ellos tiende a aumentar la demanda del otro.

Expectativas
Las expectativas respecto del ingreso y de los precios futuros también afectan las decisiones presen-
tes de compra. Por ejemplo, una persona que espera que su ingreso futuro aumente quizá gaste más
hoy que otra persona en idénticas circunstancias pero que espera que su ingreso futuro disminuya.
(Después de todo, cuando se espera que el ingreso futuro se incremente, se tiene menos necesidad
de ahorrar.) También, las personas acelerarán la compra en el presente de bienes cuyos precios se
espera que aumenten en los meses venideros.

Población
En general, la cantidad de gente que compra un producto aumenta a medida que la cantidad de
compradores potenciales se incrementa. Así, en ciudades en las que la población crece, la demanda
de viviendas aumenta año con año, mientras que ésta tiende a disminuir en las ciudades donde la
población disminuye.
En la figura 2.8 se muestran gráficamente algunos factores que desplazan las curvas de deman-
da. Éstos volverán a verse con más detalle en los capítulos 4 y 10.

FIGURA 2.8
Factores que desplazan P P P P
las curvas de demanda
los precios de los
sustitutos y de los
complementos, el ingreso, la
población, las expectativas D1 D0 D1 D0
de variación de precios
y salarios y los gustos
D0 D1 D0 D1
influyen sobre la posición
de la curva de demanda de Q Q Q Q
un producto. El precio del comple- El precio del sustituto El ingreso aumenta, El ingreso aumenta,
mento disminuye disminuye bien normal bien inferior

P P P P

D1 D1 D0 D1

D0 D0 D1 D0
Q Q Q Q
La población Se espera un Se espera una Cambio favorable
aumenta incremento en el disminución del en los gustos
precio ingreso

02_CHAPTER 2.indd 38 3/6/09 6:33:45 PM


determinanteS de la oferta y la demanda 39

P P P P FIGURA 2.9
S0 S1 S0 S1 Factores que desplazan
la curva de oferta
la tecnología, los precios
S1 S0 S1 S0 de los insumos, la
cantidad de empresas, las
expectativas acerca de los
Q Q Q Q precios futuros y el clima
Avances en la Salarios más altos Tasas de interés Precios más altos para son factores que afectan
tecnología menores las materias primas la posición de la curva de
oferta de un determinado
P P P P
producto.
S0 S1 S0 S1

S1 S0 S1 S0

Q Q Q Q
Aumento de la Expectativa de Buen clima Mal clima
cantidad de empresas aumento de precios

DETERMINANTES DE LA OFERTA
Tecnología
La cantidad que los proveedores están dispuestos a ofrecer a un determinado precio depende sobre
todo de sus costos de producción. Éstos, a su vez, están estrechamente vinculados con la tecnología.
Por ejemplo, si se descubre una manera más eficiente de atrapar langostas, disminuirá el costo de
pescarlas, lo que da como resultado que la curva de oferta se desplace hacia la derecha.

Precios de los factores


Los costos de un productor dependen también de lo que tiene que pagar por los factores de producción:
trabajo, capital, etc. Si el precio de las embarcaciones para pescar langostas aumenta o si se incrementa
el salario de los pescadores, la curva de oferta de las langostas se desplazará hacia la izquierda.

Cantidad de productores
Cuantas más empresas puedan ofrecer un bien, mayor será la cantidad ofrecida a cada precio dado.
La curva de la oferta de las computadoras personales se ha desplazado claramente hacia la derecha a
medida que la cantidad de empresas que las producen ha aumentado.

Expectativas
En sus decisiones de producción, los proveedores también toman en cuenta los cambios que se espe-
ran en los precios. Por ejemplo, si los ganaderos esperan que los precios de la carne de res aumenten
en el futuro a causa de una epidemia que afecta al ganado vacuno joven, es posible que detengan el
suministro actual para sacar ventaja de los precios futuros.1

Clima
En el caso de algunos productos, en especial en el caso de los agrícolas, la naturaleza ejerce un efecto
importante sobre las curvas de oferta. En años de sequía, por ejemplo, la curva de oferta de muchos
comestibles se desplaza hacia la izquierda.
En la figura 2.9 se muestran los efectos de algunos factores que desplazan las curvas de oferta.
No se pretende que ninguna de las listas anteriores de oferta ni de demanda sean exhaustivas.

1
Observe que la oferta es la cantidad que se ofrece a la venta a distintos precios, no necesariamente la producción presente (cuando
los productores pueden almacenar la producción obtenida). Por lo tanto, los ganaderos reducen la venta de ganado en el periodo
presente, ya que pueden venderlo después, cuando los precios sean más altos.

02_CHAPTER 2.indd 39 3/6/09 6:33:47 PM


40 Capítulo 2 la oferta y la demanda

CAMBIOS DE LA DEMANDA FRENTE A CAMBIOS


DE LA CANTIDAD DEMANDADA
Cuando los economistas emplean la expresión cambio de la demanda, se refieren a un desplazamiento
de toda la curva de demanda. Así, si el ingreso promedio de los compradores cambia, y la curva de
demanda se desplaza, hay un cambio de la demanda. Cuando se habla de un cambio de la cantidad
demandada, se quiere decir que hay un movimiento a lo largo de la curva de demanda. Por ejemplo, si
el precio de un bien baja, el resultado es un aumento de la cantidad demandada, no un aumento de
la demanda.
A las expresiones cambio de la oferta y cambio de la cantidad ofrecida les corresponden interpretacio-
nes análogas. Estas distinciones terminológicas son importantes para una comunicación clara tanto
en los análisis en el salón de clase como en los exámenes. Y si la experiencia de las anteriores genera-
ciones de estudiantes puede servir de guía, es necesario un esfuerzo para mantener esto claro.

PREDICCIÓN Y EXPLICACIÓN DE
VARIACIONES EN EL PRECIO Y LA CANTIDAD
Para predecir o explicar los cambios de los precios y de las cantidades de equilibrio es necesario
pronosticar o explicar los desplazamientos más relevantes de las curvas de oferta y de demanda. Si
dichas curvas tienen las pendientes convencionales, las proposiciones siguientes acerca de los precios
y las cantidades de equilibrio son verdaderas:
n El aumento de la demanda llevará al incremento tanto del precio como de la cantidad de equili-
brio.
n La disminución de la demanda llevará a la disminución tanto del precio como de la cantidad de
equilibrio.
n El aumento de la oferta llevará a la disminución del precio de equilibrio y al aumento de la
cantidad de equilibrio.
n La disminución de la oferta llevará al aumento del precio de equilibrio y a la disminución de la
cantidad de equilibrio.
Memorizar esta lista no tiene caso, ya que cada proposición puede deducirse desplazando la
curva relevante en el diagrama estándar de oferta y demanda.
Estas simples proposiciones darán respuestas a una serie de preguntas.

¿Por qué los precios de algunos bienes, como las manzanas, bajan en los meses de mayor
consumo mientras que otros, como las casas de campo a la orilla del mar, suben?
La respuesta es que el aumento del consumo estacional, en el caso de las manzanas, es resultado
del aumento en la oferta, y en el caso de las casas de campo, es el resultado del aumento en la de-
EL NATURALISTA manda. Como lo muestra la figura 2.10, estos desplazamientos ocasionan las relaciones estacionales
ECONÓMICO observadas entre precios y cantidades de equilibrio. (Los subíndices w y s que aparecen en la figura
2.1 2.10 se utilizan para denotar valores de verano y de invierno, respectivamente.) Cuando aumenta la
demanda (como en el caso de las casas de campo), el aumento de la cantidad de equilibrio se presenta
de manera concurrente con un aumento del precio de equilibrio. Cuando aumenta la oferta (como
en el caso de las manzanas), el aumento de la cantidad de equilibrio se presenta conjuntamente con
una disminución en el precio de equilibrio.

EjERCICIO 2.3
¿Qué pasará con el precio y con la cantidad de equilibrio en el mercado de mariscos si se
presenta cada uno de los eventos siguientes: 1) se publica un informe científico en el que
se afirma que el pescado contiene mercurio, el cual es tóxico para los humanos y 2) el precio
del combustible diesel (que se emplea en las embarcaciones pesqueras) disminuye de manera
significativa?

02_CHAPTER 2.indd 40 3/6/09 6:33:47 PM


álgebra de la oferta y la demanda 41

FIGURA 2.10
Dos fuentes de variación
estacional
en los meses de verano
P P
Sw Ds aumenta el consumo tanto de
D manzanas como de casas
Ss de campo a la orilla del mar.
Pw Dw S a) en los meses de verano
el precio de las manzanas
Ps es el más bajo porque el
Ps
Sw aumento de la cantidad es
resultado de un incremento
Pw
Ds de la oferta. (los subíndices
D
Ss w y s denotan valores
S Dw
en invierno y en verano,
Q Q respectivamente.) b) en los
Qw Qs Qw Qs
meses de verano el precio
Manzanas Casas de campo a la orilla del mar de las casas de campo a la
a) b) orilla del mar es el más alto
debido a que el aumento
de la cantidad se debe a un
incremento de la demanda.

Si el frijol de soya es uno de los ingredientes en el pienso para ganado, ¿qué efecto tendrá sobre el EjEMPLO 2.3
precio y la cantidad de equilibrio de la carne de res un programa de regulación del precio del frijol
de soya en el mercado?
El programa de regulación del precio eleva el precio del pienso para ganado vacuno, lo que ocasiona
un desplazamiento hacia la izquierda de la curva de oferta de la carne de res. (Vea la figura 2.11.)
Esto, a su vez, provoca un aumento en el precio de equilibrio y una disminución en la cantidad de
equilibrio de la carne de res.

FIGURA 2.11
Precio ($/lb) Efecto de la regulación
S' del precio del frijol de
6 S soya sobre el precio y la
D
cantidad de equilibrio
5 de la carne de res.
al aumentar el precio
4 del frijol de soya, un
insumo que se usa en la
producción de carne de
3 res, la regulación del precio
ocasiona un desplazamiento
S'
2 hacia la izquierda de la
curva de oferta. esto tiene
1
como consecuencia un
S aumento del precio de
D
equilibrio y una disminución
Cantidad (1 000 000 lb/año) en la cantidad de equilibrio.
100 200 300 400 500 600

ÁLGEBRA DE LA OFERTA Y LA DEMANDA


En los ejemplos que se han visto hasta ahora se ha empleado una interpretación geométrica del
equilibrio del mercado. Este método es bueno para ilustrar los principios básicos de esta teoría. Pero
para calcular valores numéricos es más conveniente calcular de manera algebraica los precios y las

02_CHAPTER 2.indd 41 3/6/09 6:33:49 PM


42 Capítulo 2 la oferta y la demanda

Figura 2.12
Gráficas de las Precio S
ecuaciones 2.1 y 2.2 12
los métodos algebraico
y geométrico conducen
10 D
exactamente a los mismos
precios de equilibrio y
cantidades de equilibrio. 8
la ventaja del método
algebraico es que es más 6
fácil llegar a soluciones
numéricas exactas. el
método geométrico es 4
útil debido a que da una
descripción intuitivamente 2
más clara de las curvas de S
D
la oferta y la demanda.
Cantidad
0 2 4 6 8 10 12

cantidades de equilibrio. Suponga, por ejemplo, que la curva de oferta de un determinado producto
está dada por

P = 2 + 3Q S , (2.1)

y que la curva de demanda está dada por P = 2 + 3Q S ,


P = 10 − Q d ,
P = 10 −S Q d , (2.2)
P = 2 + 3Q ,
2 + 3Q * = 10
donde P es el precio del producto y QS y Qd representan la −cantidad
Q *, ofrecida y la cantidad demandada,
+S3=
respectivamente. Se sabe que en el equilibrio2 Q Q *Q=
d
. 10 − Q *,
Representando este valor común como Q*, se
= 10ecuaciones
pueden igualar los miembros derechos deP las − Qd , 2.1 y 2.2 y resolver:

2 + 3Q * = 10 − Q *, (2.3)

que da Q* = 2. Si se sustituye Q* = 2 en la ecuación de oferta o en la ecuación de demanda se obtiene


el precio de equilibrio, P* = 8.
No es necesario decir que para llegar a este mismo resultado se pudieron haber representado
gráficamente las ecuaciones 2.1 y 2.2 (vea la figura 2.12). La ventaja del método algebraico es que es
mucho menos laborioso que trazar las curvas exactas de la oferta y la demanda.

EjERCICIO 2.4
Encuentre el precio y la cantidad de equilibrio en un mercado cuyas curvas de oferta y de
demanda están dadas por P = 4QS y P = 12 – 2Qd, respectivamente.

RESUMEN
• La curva de oferta generalmente es una línea con pendiente el precio y la cantidad de equilibrio están determinados por la
positiva que indica la cantidad que ofrecen los vendedores intersección de estas dos curvas.
a cada precio dado. La curva de demanda es una línea con
pendiente negativa que indica la cantidad que demandan los • Si un precio está por encima de su equilibrio habrá vendedo-
compradores a cada precio dado. En un mercado no regulado, res insatisfechos, es decir, habrá exceso de oferta. Esta situa-

02_CHAPTER 2.indd 42 3/6/09 6:33:51 PM


preguntaS de repaSo 43

ción hace que los vendedores bajen sus precios. En cambio, mientras hacen poco por aliviar la apremiante situación de las
si un precio está por debajo de su equilibrio, habrá compra- familias pequeñas de agricultores. En casi cualquiera de los
dores insatisfechos, es decir, habrá exceso de demanda. Esta casos es posible diseñar otra intervención que sea mejor en
situación hará que los vendedores eleven los precios. El único todos los aspectos.
resultado estable es aquel en que el exceso de demanda y el
exceso de oferta son exactamente cero. • Si el problema es que los pobres tienen muy poco dinero, la so-
lución es descubrir maneras de incrementar directamente su
• Dados los atributos de los compradores y de los vendedores, ingreso. Los cuerpos legislativos no pueden ignorar la ley de
el precio y la cantidad de equilibrio representan el mejor re- la oferta y la demanda pero sí tienen la capacidad de modificar
sultado posible, en el sentido de que cualquier otro par precio- las fuerzas que gobiernan la forma y posición de las curvas de
cantidad será peor por lo menos para algunos compradores o oferta y de demanda.
vendedores.
• El análisis de la oferta y la demanda es el instrumento básico
• El hecho de que los resultados del mercado sean eficientes con que cuenta el economista para predecir cómo cambiarán
en este sentido no significa que necesariamente cuenten con los precios y las cantidades de equilibrio en respuesta a los
la aprobación de la sociedad. Al contrario, con frecuencia se cambios en las fuerzas del mercado. Cuatro proposiciones
lamenta el hecho de que muchos compradores entren en el sencillas sirven de guía en esta tarea: 1) el aumento de la de-
mercado con un ingreso muy pequeño. La preocupación por manda lleva al incremento tanto del precio como de la can-
el bienestar de los pobres ha hecho que los gobiernos de casi tidad de equilibrio; 2) la disminución de la demanda lleva a la
todas las sociedades intervengan de diversas maneras para al- reducción tanto del precio como de la cantidad de equilibrio;
terar los resultados de las fuerzas del mercado. 3) el aumento de la oferta lleva a la disminución del precio de
equilibrio y al incremento de la cantidad de equilibrio, y 4) la
• Algunas veces estas intervenciones toman la forma de leyes disminución de la oferta lleva al aumento del precio de equili-
que fijan precios superiores o inferiores a los de equilibrio. Es- brio y a la reducción de la cantidad de equilibrio.
tas leyes, si bien de manera no intencionada, suelen generar
consecuencias perjudiciales. El control de los alquileres, por • Los ingresos, los gustos, los precios de sustitutos y comple-
ejemplo, distorsionan las funciones de racionamiento y de mentos, las expectativas y la población son algunos de los
distribución de recursos que desempeñan los precios. Estas factores que desplazan las curvas de demanda. Las curvas de
intervenciones conducen a mercados negros y a un rápido oferta, a su vez, dependen de factores como la tecnología, el
deterioro de las existencias de viviendas en alquiler. Del mis- precio de los insumos, la cantidad de proveedores, las expecta-
mo modo, en la agricultura las leyes de regulación del precio tivas y, en el caso de los productos agrícolas, el clima.
tienden a enriquecer a las grandes explotaciones agrícolas

pREgUNtaS dE REpaSo
1. ¿Cuál es la diferencia entre “escasez” y “déficit”? c) El consumo de uvas aumenta debido a una buena cosecha.
2. ¿Cómo sería la curva de la oferta de un bien que no es esca- d) El consumo de uvas aumenta debido a un cambio en los
so? Si se supone que el bien es útil, ¿cómo será su curva de gustos.
demanda? Explique por qué un precio positivo para un bien 6. Cuando existe un exceso de oferta, ¿por qué cualquier vende-
de consumo implica que es escaso. dor puede vender todo lo que desee con sólo ofrecer un precio
3. Dé dos ejemplos de acciones realizadas por la administración ligeramente inferior al de mercado?
de su universidad que impidan que determinados mercados 7. Dé un ejemplo de un mercado en el que la función distributiva
alcancen el equilibrio. ¿Qué evidencias de exceso de oferta o del precio no sea muy importante.
de demanda puede dar en estos ejemplos? 8. Suponga que usted es un funcionario público y necesita recau-
4. ¿Cuál es la diferencia entre “una disminución de la oferta” y dar más ingresos gravando un producto. Por razones políticas
“una disminución de la cantidad ofrecida”? desea que la carga del impuesto recaiga principalmente sobre
5. En cada uno de los incisos diga si se trata de 1) un cambio de la los consumidores, no sobre las empresas (que contribuyeron
demanda o 2) un cambio de la cantidad demandada. de manera especial a los fondos para su campaña). ¿En qué se
a) El consumo de uvas disminuye debido a un boicoteo de los fijaría para encontrar un producto qué gravar?
consumidores. 9. ¿Qué preferiría una persona racional pobre y por qué?
b) El consumo de uvas disminuye debido a un impuesto a los a) Un Mercedes de 50 000 dólares (con valor inmediato de
productores de uvas. reventa = 30 000 dólares).
b) 35000 dólares en efectivo.

02_CHAPTER 2.indd 43 3/6/09 6:33:51 PM


44 Capítulo 2 la oferta y la demanda

pRoblEMaS
1. Suponga que el té y los limones son complementos y el café y el té son sustitutos.
a) Diga si la imposición de un precio techo al té afectaría, y de qué manera, el precio de los limones.
Explique.
b) Diga si la imposición de un precio techo al té afectaría, y de qué manera, el precio del café. Explique.
2. Las curvas de oferta y de demanda del mercado de los DVD están dadas por P’ = 2QS y P = 42 – Qd,
respectivamente.
a) ¿Cuántas unidades serán comercializadas al precio de 35 dólares? ¿Y al precio de 14? ¿Cuál de los
participantes en el mercado quedará insatisfecho con estos precios?
b) En el equilibrio, ¿cuál es la cantidad de DVD que se comercializará y a qué precio?
c) ¿Cuál es el ingreso total de la venta de DVD?
3. El hardware y el software para computación son complementos. Analice los efectos sobre la cantidad
y el precio de equilibrio.
a) En el mercado del software de cómputo cuando el precio del software para computación baja.
b) En el mercado del hardware de cómputo cuando el precio del software baja.
4. Suponga que un estudio recientemente publicado indica que los juguetes con baterías perjudican el
desarrollo de los niños y recomiendan a los padres tener cuidado con dichos juguetes. Use gráficas
para indicar el efecto sobre el precio y la cantidad de equilibrio en cada uno de los mercados siguien-
tes:
a) El mercado de juguetes con batería.
b) El mercado de baterías tamaño D.
c) El mercado de yo-yos (que no requieren baterías).
5. Emplee gráficas para mostrar los cambios que se esperan, tanto en el precio como en la cantidad, en
los mercados siguientes con las condiciones que se indican:
a) Petróleo crudo: a medida que disminuyen las reservas de petróleo es más difícil hallar y obtener
petróleo crudo.
b) Viajes en avión: la preocupación por la seguridad aérea hace que las personas eviten viajar en
avión.
c) Viajar en tren: la preocupación por la seguridad hace que las personas eviten viajar en tren.
d) Habitaciones de hotel en Hawai: la preocupación por la seguridad hace que las personas eviten viajar
en avión.
e) Leche: una hormona obtenida por ingeniería genética permite a los grandes productores de leche
reducir los costos de producción.
6. En cada una de las situaciones del problema 5 diga si el efecto es un cambio en la demanda o sólo un
cambio en la cantidad demandada.
7. Suponga que la demanda de entradas a los partidos de fútbol es P = 1900 – (1/50)Q y que la oferta
está fija en Q = 90000 asientos.
a) Halle el precio y la cantidad de equilibrio para las entradas a los partidos de fútbol (use álgebra y
una gráfica).
b) Suponga que el gobierno prohíbe la reventa de entradas (vender las entradas a un precio superior
a su valor) y que el valor de una entrada es de 50 dólares (esta política establece un precio techo
de 50 dólares). ¿Cuántos consumidores quedarán insatisfechos? ¿cuál es la cuantía del exceso de
demanda?
c) Suponga que el partido siguiente es muy importante, de manera que la demanda pasa a P = 2100
– (1/50)Q. ¿Cuántos consumidores quedarán insatisfechos?
4) ¿En qué difiere la distorsión de este precio techo del caso más típico de oferta con pendiente posi-
tiva?
8. La demanda de viviendas es P = 1200 – Q, mientras que la oferta es P = Q unidades. El gobierno
impone un control de alquiler en P = 300 dólares al mes. Suponga que en el mercado la demanda
crece a P = 1 400 – Q.
a) ¿Cómo afecta al exceso de demanda el aumento de la demanda de viviendas?
b) ¿En qué precio debe establecer el gobierno el control de alquiler para mantener el exceso de de-
manda en el nivel que tenía antes de que aumentara la demanda?

02_CHAPTER 2.indd 44 3/6/09 6:33:52 PM


reSpueStaS a loS ejerCiCioS del Capítulo 45

9. Suponga que en el mercado del frijol de soya la demanda es P = 600 – Q y la oferta es P = Q, donde Q
representa toneladas de frijol de soya por año. El gobierno establece un precio regalado en P = 500
dólares/tonelada y compra cualquier exceso de oferta a este precio. En respuesta a lo anterior, como
un ajuste a largo plazo, los agricultores cambian el cultivo de trigo por el de frijol de soya, con lo que
la oferta se incrementa a P = (1/2)Q.
a) ¿Cómo es el exceso de oferta con la llegada de nuevos proveedores comparado con el exceso de
oferta antes de que los agricultores cambiaran el cultivo de trigo por el de frijol de soya?
b) ¿Cuánto más tendrá que gastar el gobierno para comprar el exceso de oferta?
10. En el mercado que se representa abajo, ¿cómo se modificarían el precio y la cantidad de equilibrio si
para cada productor el costo marginal aumentara 2 dólares/libra? (Sugerencia: recuerde la interpreta-
ción vertical de la curva de oferta que se vio en el capítulo 1.)

Precio ($/lb)

8
S

D
Cantidad (lb/día)
0 1 2 3 4

RESpUEStaS a loS EjERcicioS dEl capítUlo


2.1 A un precio de 4 dólares/langosta, la cantidad demandada es de 5 000 langostas/día y la cantidad
ofrecida es de 1 000 langostas/día, con lo que el exceso de demanda es igual a 4 000 langostas/día. A
20 dólares/langosta, el exceso de oferta es de 4 000 langostas/día.
2.2 Si el alquiler controlado se fija en un valor mayor al precio de equilibrio no tendrá ningún efecto. El
alquiler se establecerá en su valor de equilibrio de 600 dólares al mes.
2.3 La caída en el precio del combustible diésel hace que la curva de oferta se desplace a la derecha. La
información sobre el mercurio hace que la curva de demanda se desplace a la izquierda. Como se
muestra en los diagramas siguientes, el precio de equilibrio bajará (en ambas gráficas, pero la canti-
dad de equilibrio puede subir (grafica b) o bajar (grafica a).
P P S
D D
S S' S'

D'
D'

S D D
S' S'
D'
D'
Q Q
a) b)

2.4 4Q* = 12 – 2Q*, de donde se obtiene Q* = 2 y P* = 4Q* = 8.

02_CHAPTER 2.indd 45 3/6/09 6:33:54 PM


02_CHAPTER 2.indd 46 3/6/09 6:33:54 PM
a p é N d i c E

2
¿CóMO AFECTAN LOs iMpuEsTOs
LAs CANTiDADEs Y LOs pRECiOs
DE EquiLibRiO?

E
l análisis de la oferta y la demanda también es un instrumento útil para analizar los efectos
de diversos impuestos. En esta sección se considera un impuesto constante por unidad
de producción. ¿Cómo afecta al precio y a la cantidad de equilibrio de un producto que
cada unidad que se venda se grave con un impuesto T = 10? Para responder esta pregunta hay dos
caminos equivalentes. El primero es suponer que este impuesto se recauda de los vendedores. En la
figura A.2.1 la recta SS representa la curva original de la oferta. Al precio de P0 = 25, los vendedores
están dispuestos a suministrar Q0 unidades del producto. Cuando un impuesto de T = 10 se carga a
los vendedores, el precio del mercado tendrá que ser P0 + 10 = 35 para que obtengan el mismo pago
neto que tenían cuando el precio era P0 = 25. Al precio de 35 los vendedores ofrecerán la misma
cantidad de producto que al precio de 25. La curva de oferta que se obtiene, una vez considerados los
impuestos, es la curva de oferta original desplazada hacia arriba en T = 10.
En la figura A.2.2, la línea DD representa la curva de demanda con la que se encuentran los
vendedores a los que se grava con T = 10 por unidad de producción. El efecto de este impuesto es la
causa de que la cantidad de equilibrio disminuya de Q* a Q1*. El precio pagado por los consumidores
aumenta de P* a P1* y el precio, neto sin el impuesto, que recibe el vendedor baja a P1* – 10.
Observe en la figura A.2.2 que aun cuando el vendedor paga un impuesto de T, la cantidad total
que recibe se sitúa a menos de T por debajo del precio de equilibrio inicial. Observe también que aun
cuando el impuesto se recauda del vendedor, el efecto que tiene es incrementar el precio que paga el
consumidor. De esta manera, la carga del impuesto se divide entre el vendedor y el consumidor.
47

02_CHAPTER 2.indd 47 3/6/09 6:33:55 PM


48 Capítulo 2 apéndiCe 2: ¿CÓmo afeCtan loS impueStoS laS CantidadeS y loS preCioS de equilibrio?

FIGURA A.2.1
Un impuesto de T = 10
Precio
que se recauda de los S'
vendedores desplaza
la curva de oferta T
unidades hacia arriba
T = 10 S
la curva de oferta original 35 = P0 + T
indica los precios que
tienen que cobrar los
vendedores para poder
cubrir sus costos en 25 = P0
cada uno de los niveles
de producción. desde la S'
perspectiva del vendedor,
un impuesto de T = 10
unidades es lo mismo que
un aumento de 10 unidades S
en el costo unitario. de Cantidad
manera que la nueva curva Q0
de oferta se encuentra 10
unidades por encima de la
original.

Algebraicamente, la participación del vendedor en el impuesto, que se representa por ts, es la


reducción en el precio que recibe dividida entre el impuesto:
P * – ( P1 * –T )
ts = . (A.2.1)
T
De manera similar, la participación del consumidor en el impuesto, que se representa por tb es el
aumento en el precio (incluyendo el impuesto) dividido entre el impuesto;
P1 * –  P *
tb = . (A.2.2)
T

EjERCICIO A.2.1
Verifique que ts + tb = 1.

En general, tb y ts dependen de la forma de las gráficas de oferta y demanda. Si, por ejemplo, la oferta
es muy insensible a los cambios de precio, tb será cercano a cero y ts será cercano a 1.

FIGURA A.2.2 Precio


Precios y cantidades S'
de equilibrio cuando D
se recauda de los
vendedores un S
T = 10
impuesto T = 10
este impuesto ocasiona una P* 1
disminución de la cantidad
de equilibrio de Q* a Q*1 . P*
el nuevo precio que pagan
los consumidores aumenta P*1 – 10
de P* a P*1 . el nuevo precio S'
que recibe el oferente
baja de P * a P*1 – 10. D
S
Cantidad
Q*1 Q*

02_CHAPTER 2.indd 48 3/6/09 6:33:59 PM


¿CÓmo afeCtan loS impueStoS laS CantidadeS y loS preCioS de equilibrio? 49

FIGURA A.2.3
Precio Efecto de un impuesto
de T = 10 que se carga a
D
los consumidores
P1 + 10 antes del impuesto, los
D' consumidores compran
Q1 unidades al precio P1.
después del impuesto, el
P1 precio P1 se convierte en
P1 + 10, lo que significa que
los consumidores podrán
comprar únicamente la
cantidad Q2. el efecto
T = 10 que tiene este impuesto
D es desplazar la curva de
demanda 10 unidades hacia
D' abajo.
Cantidad
Q2 Q1

En cambio, si la demanda es muy insensible al precio, tb será cercano a 1 y ts será cercano a cero.
Esto equivale a decir que el impuesto tiende a recaer más fuertemente sobre el lado del mercado que
tiene menos posibilidad de escapar de él. Si los consumidores no tienen productos sustitutos a los
que estén preparados a recurrir, los productores les transferirán la mayor parte del impuesto. Pero
si los proveedores no tienen más remedio que seguir suministrando el producto, la mayor parte de
la carga del impuesto recaerá sobre ellos. En tanto que la curva de oferta tenga pendiente positiva
y la curva de demanda tenga pendiente negativa, tanto tb como ts serán positivas.
La segunda manera de analizar el efecto de un impuesto T = 10 por unidad de producto es
pensar que este impuesto se recauda directamente de los consumidores. ¿Cómo afecta esto a la
curva de demanda del producto? En la figura A.2.3, DD es la curva de demanda antes de la im-
posición del impuesto. Al precio P1, los consumidores demandan la cantidad Q1. Después de la impo-
sición del impuesto, si el precio del producto es P1, la cantidad que tendrán que pagar los consumido-
res será P1 + 10. Por lo consiguiente la cantidad demandada bajará de Q1 a Q2. De la misma manera
se pueden calcular las cantidades demandadas a cualquier otro precio después de la imposición del
impuesto. En la figura A.2.3, D'D' es la curva de demanda que se obtiene después de establecer el
impuesto. Ésta es la curva original de demanda desplazada 10 unidades hacia abajo.
En la figura A.2.4, si SS es la curva de la oferta para este mercado, no es difícil determinar
el efecto del impuesto sobre la cantidad y el precio de equilibrio.

FIGURA A.2.4
Precio Precio y cantidad de
D
equilibrio después de un
impuesto T = 10 pagado
S por los consumidores
D' el impuesto ocasiona una
disminución de la cantidad
P*2 + 10
de equilibrio de Q* a Q2*.
el nuevo precio pagado
P*
por los consumidores
P*2 aumenta de P* a P2* + 10.
el precio que reciben los
proveedores disminuye de
T = 10 P* a P2*.
D
S
D'
Cantidad
Q*2 Q*

02_CHAPTER 2.indd 49 3/6/09 6:34:01 PM


50 Capítulo 2 apéndiCe 2: ¿CÓmo afeCtan loS impueStoS laS CantidadeS y loS preCioS de equilibrio?

FIGURA A.2.5
Un impuesto al
consumidor conduce al Precio Precio
mismo resultado que un
impuesto al vendedor 10 D 10 D
el precio que recibe
el vendedor (neto sin S'
S 8 D' S
el impuesto), el precio
que paga el consumidor T=2
6 6
(incluyendo el impuesto)
y la cantidad de equilibrio 5 5
serán iguales sin importar 4 4
que el impuesto se recaude
del oferente (gráfica a) o del 2 S' T=2
consumidor (gráfica b).
S D S D' D
Cantidad Cantidad
4 5 10 4 5 8 10
a) b)

La cantidad de equilibrio baja de Q* a Q2* y el precio de equilibrio antes del impuesto baja de P* a
P2*. El precio total que paga el consumidor después de sumar el impuesto sube a P2* + 10.
¿Hay alguna diferencia entre el efecto que tiene un impuesto al productor y un impuesto al
consumidor? En absoluto. Para ilustrar esto, suponga que las curvas de oferta y de demanda de un
mercado están dadas por P = Q* y P = 10 – Qd, respectivamente, y considere primero el efecto de
un impuesto aplicado a los proveedores de 2 por unidad de producción. La figura A.2.5a muestra las
curvas originales de oferta y de demanda y la nueva curva de oferta S'S' después del impuesto. El
precio y la cantidad de equilibrio originales son ambos iguales a 5. El nuevo precio de equilibrio para
el consumidor (incluyendo el impuesto) es 6 y la nueva cantidad de equilibrio es 4. El precio obtenido
por el oferente, neto sin impuesto, es 4.
Ahora considere un impuesto aplicado al consumidor de 2 por unidad. En la figura A.2.5b se
muestran las curvas originales de oferta y de demanda y la nueva curva de demanda D'D' después del
impuesto. Observe que los efectos sobre el precio y la cantidad son exactamente los mismos que en
el caso del impuesto al proveedor que se muestran en la gráfica a.

EjERCICIO A.2.2
Considere un mercado cuyas curvas de oferta y de demanda están dadas por P = 4Qx y P = 12
– 2Qd, respectivamente. ¿Cómo afectará al precio y a la cantidad de equilibrio un impuesto de
6 u. m. por unidad de producción recaudado del vendedor? ¿Qué pasa si el impuesto se recauda
del consumidor?

Cuando es necesario elevar los ingresos fiscales a algunos líderes políticos les parece adecuado
proponer a las empresas un impuesto a las ventas porque “para ellos es más fácil pagarlo”. Pero un
análisis cuidadoso de los efectos de un impuesto a las ventas muestra que su carga será la misma si
se impone a los consumidores o a los oferentes. La incidencia legal del impuesto (que se aplique a los
consumidores o a los productores) no tiene efecto sobre su incidencia económica (las respectivas partes
de la carga fiscal que recaen en los consumidores y en los proveedores). Económicamente hablando,
la entidad de la que se recaude el impuesto es, por lo tanto, del todo indiferente.
Una advertencia: cuando se dice que la carga económica del impuesto no depende de la parte de
la cual se recaude directamente no significa que los consumidores y los oferentes compartan siem-
pre la carga de los impuestos por igual. Las participaciones respectivas, como ya se hizo notar, pue-
den ser muy desiguales. La independencia de las incidencias legal y económica sólo significa que la
carga se compartirá de la misma manera sin importar a quién se le aplique el impuesto.

02_CHAPTER 2.indd 50 3/6/09 6:34:02 PM


problemaS 51

EjERCICIO A.2.3
¿Verdadero o falso? Entre más inclinada sea la curva de demanda de un bien en relación con
su curva de oferta mayor será la proporción del impuesto de ese bien que recaerá sobre el
consumidor. Explique.

pRoblEMaS
1. El gobierno, temeroso de que el déficit de titanio pueda poner en peligro la seguridad nacional, gra-
vó con un impuesto de 2 dólares/oz el precio de venta al público de este metal raro. Este impuesto se
recauda de los vendedores de titanio. En el diagrama siguiente se presentan las curvas originales de
oferta y demanda del titanio. Muestre en este mismo diagrama cómo afectará este impuesto al precio
y a la cantidad de equilibrio en el corto plazo. Indique en el diagrama todos los puntos importantes.

Precio ($/oz)
6
S
5

1
D
Cantidad (toneladas/año)
0 1 2 3 4 5 6

2. Suponga que en el mercado de titanio (sin impuesto), descrito en el problema 1, un precio suelo de 4
dólares/oz, da como resultado ventas de sólo 2 toneladas/año (sin impuesto). Describa una transac-
ción que beneficie a algunos consumidores y productores sin dañar a otros.
3. Suponga que en el mercado de titanio del problema 1, con un impuesto de 2 dólares/oz, la demanda
de titanio crece debido a aplicaciones médicas recién halladas. La nueva curva de demanda es P = 8
– Q. Halle la variación en los ingresos fiscales por el aumento de la demanda de titanio.
4. Suponga ahora que en el mercado de titanio del problema 2, sin impuestos pero con un precio suelo
de 4 dólares/oz, se presenta una reducción de la oferta debido a la disminución de las reservas de
titanio. La nueva curva de oferta es P = 2 + Q. ¿Cómo varía el exceso de oferta a causa de la disminu-
ción de la oferta? ¿Sigue siendo vinculante el precio suelo (hace que el precio se eleve de su nivel de
equilibrio)?
5. Suponga que el gobierno del estado grava con un impuesto de 9 dólares cada DVD que se venda,
recaudándolo de los vendedores.
a) ¿Qué cantidad de DVD se venderá en el equilibrio?
b) ¿Cuál será el precio que paguen los consumidores?
c) ¿Cuánto, en total, gastan ahora los consumidores?
d) ¿Cuánto dinero va al gobierno?
e) Muestre gráficamente los resultados anteriores.
6. Dado el impuesto que se describe en el ejercicio 5,
a) ¿Qué fracción del impuesto recae sobre el vendedor?
b) ¿Qué fracción del impuesto recae sobre el consumidor?
7. Para los automóviles japoneses que se vendieron en Estados Unidos al comienzo de los años de 1980,
el presidente Reagan negoció una cuota de importación “voluntaria”. Algunos de sus asesores le
habían recomendado que en lugar de eso aplicara un impuesto de importación (arancel) más alto.

02_CHAPTER 2.indd 51 3/6/09 6:34:04 PM


52 Capítulo 2 apéndiCe 2: ¿CÓmo afeCtan loS impueStoS laS CantidadeS y loS preCioS de equilibrio?

Si se supone que el arancel tenía la forma de un impuesto unitario de tipo constante T por cada
automóvil japonés vendido en Estados Unidos y que T hubiera sido escogido para producir la misma
reducción de la cantidad que la cuota de importación, compare los precios pagados por los consumi-
dores estadounidenses por los automóviles japoneses de acuerdo con las dos políticas.
8. Estudios realizados en ratas y ratones han demostrado que la carne asada al carbón ocasiona cáncer.
Dado que el gobierno no puede reglamentar los métodos para cocinar que se emplean en los hoga-
res, se propuso un método para evitar el consumo de carne asada; se propuso poner un impuesto de
ciento por ciento a la venta de carbón para asar. Si se supone que la demanda diaria de carbón era
P = 120 – 2Q y la oferta P = 30 + Q, donde P está en dólares por bolsa y Q es la cantidad de bolsas de
20 lb que se venden por semana.
a) ¿Cuál es el precio del carbón antes y después del impuesto?
b) ¿Cuál es la cantidad de carbón vendida antes y después del impuesto?
c) ¿Cómo se divide el impuesto entre consumidores y vendedores?
9. La oferta es P = 4Q y la demanda es P = 20, donde P es el precio en dólares por unidad y Q son unida-
des de producción por semana.
a) Encuentre el precio y la cantidad de equilibrio (algebraica y gráficamente).
b) Si el impuesto que tiene que pagar el vendedor es 4 dólares/unidad, ¿qué pasa con la cantidad co-
mercializada, con el precio que paga el consumidor y con el precio que cobra (neto sin impuesto)
el vendedor?
c) ¿Cómo se distribuye la carga del impuesto entre consumidores y vendedores y por qué?
10. Repita el problema 9, pero suponga que el impuesto es pagado por el consumidor, que la demanda es
P = 28 – Q y que la oferta es P = 20.

RESpUEStaS a loS EjERcicioS dEl apéNdicE


A.2.1 t s + tb = (P * – P 1* + T ) + ( P 1* – P * )  /T = T /T = 1 .
A.2.2 El precio y la cantidad originales son P* = 8 y Q* = 2, respectivamente. La curva de oferta con el
impuesto está dada por P = 6 + 4QS. Si P´ y Q' representan el nuevo precio y cantidad de equilibrio,
respectivamente, se tiene ahora 6 + 4Q´ = 12 – 2Q´, lo cual da Q´ = 1 y P´ = 10, donde P´ es el
precio que pagan los consumidores. P´ – 6 = 4 es el precio que reciben los vendedores. De manera
alternativa, la curva de demanda con el impuesto de 6 a los consumidores está dada por P = 6 – 2Qd
y se tiene 4Q´ = 6 – 2Q´, lo que nuevamente da Q´ = 1. P˝ = 4, donde P˝ es el precio que obtiene el
vendedor. P˝ + T = P˝ + 6 = 10 es el precio que paga el consumidor.
A.2.3 Verdadero.
S+T
P

S
a
b

P*
a–T
b–T
D
D'

Participación de los consumidores en el impuesto con D' = (a – P*)/T


Participación de los consumidores en el impuesto con D' = (b – P*)/T

02_CHAPTER 2.indd 52 3/6/09 6:34:06 PM


Pa rt e

LTEORíA
2
DEL cOMpORTAMiENTO
A O F E RTA Y L A D E M A N DA
DEL cONsuMiDOR
En los siguientes seis capítulos se desarrollará la teoría del comportamiento
del consumidor. El capítulo 3 tiene una especial importancia porque expone la
teoría económica de cómo las personas con recursos limitados eligen entre
alternativas en competencia. Los métodos y herramientas que se desarrollan
en este capítulo aparecen de manera recurrente en el resto del libro y, en
realidad, en toda la economía. En el capítulo 4 se muestra cómo usar la teoría
de la elección racional individual para obtener curvas de demanda individuales
y de mercado. En el capítulo 5 se exploran numerosas aplicaciones de la elec-
ción racional y de las teorías de demanda, incluida la teoría de decisiones que
implica consecuencias futuras.
El capítulo 6 muestra cómo extender el modelo de elección racional para
incluir opciones en las que interviene la incertidumbre o la información in-
completa. El capítulo 7 examina el papel que tienen los motivos altruistas en el
comportamiento económico y social y muestra por qué las personas honradas
suelen tener una ventaja económica sobre las que no lo son. Por último, en el
capítulo 8 se analizan diversas circunstancias en las que las personas comunes
y corrientes tienden a hacer elecciones irracionales. La experiencia demuestra
que ser conscientes de esta tendencia ayuda a las personas a tomar mejores
decisiones.

53

03_CHAPTER 3.indd 53 3/6/09 7:04:01 PM


03_CHAPTER 3.indd 54 3/6/09 7:04:02 PM
CaPítulo

3
LA ELECCIÓN RACIONAL
DEL CONSUMIDOR

U
sted acaba de cambiar el cheque de su pago mensual y se dirige a una tienda de música
para comprar un CD de Eric Clapton que ha estado esperando. El precio del disco es de 10
dólares. Situación 1: de camino a la tienda pierde 10 dólares. Situación 2: compra el disco y
al salir de la tienda se tropieza y se cae; el disco cae en la acera y se hace pedazos. Imagine su estado
de ánimo en cada situación.
a) En la situación 1, ¿compraría el disco?
b) En la situación 2, ¿volvería a comprar el disco?
Estas preguntas1 se hicieron a un gran grupo de estudiantes universitarios que no habían recibido
nunca un curso de economía. A la primera pregunta 54 por ciento respondió con un sí, es decir, com-
prarían el disco después de haber perdido un billete de 10 dólares. Pero sólo 32 por ciento respondió
con un sí a la segunda pregunta —68 por ciento dijeron que no volverían a comprar el disco después
de haber roto el primero—. Por supuesto que para ninguna de estas preguntas hay una respuesta
que sea “correcta”. Los sucesos descritos tendrán mayor impacto para un consumidor pobre que
para uno rico. Pero un momento de reflexión permite ver que, por lógica, su conducta en una y otra
situación debe ser exactamente la misma. Después de todo, en los dos escenarios el único cambio
económico importante es que ahora tiene usted 10 dólares menos para gastar. Esto bien puede sig-
nificar que tenga que renunciar a tener el disco, que podrá ahorrar menos o que tendrá que dejar
de comprar algún otro bien. Pero a su decisión no le debería afectar la manera particular en la que

1
Estas preguntas se plantearon con base en otras similares propuestas por los especialistas en teoría de la decisión: Daniel
Kahneman y Amos Tversky (vea el capítulo 8).

55

03_CHAPTER 3.indd 55 3/6/09 7:04:02 PM


56 CaPítulo 3 la eleCCión raCional del Consumidor

se haya vuelto 10 dólares más pobre. En ambos casos, el precio del disco es 10 dólares y el beneficio
que tendrá al escucharlo es también el mismo. En ambas situaciones usted debería comprar el disco
o no comprarlo. Sin embargo, como ya se indicó, mucha gente actúa de forma diferente en las dos
situaciones.

VISIÓN PRELIMINAR DEL CAPÍTULO


El objetivo en este capítulo es ampliar el modelo económico básico para responder a preguntas como
las que se plantearon antes. Este modelo se conoce como teoría de la elección racional del consumidor
y subyace en todas las decisiones de compra de los individuos, las cuales, a su vez, contribuyen a las
curvas de demanda con las que se trabajó en el capítulo anterior.
La teoría de la elección racional parte del supuesto de que los consumidores llegan al mercado
con preferencias bien definidas. Si los precios se consideran fijos, su tarea es distribuir su ingreso de
manera que satisfaga de la mejor manera posible sus preferencias. Para realizar esta tarea se requie-
ren dos pasos. El primero es describir las diversas combinaciones de bienes que el consumidor puede
adquirir. Estas combinaciones dependen tanto de su ingreso como de los precios de los bienes. El
segundo es, de estas combinaciones posibles, elegir la que prefiera. El análisis del paso 2 requiere de
algún medio para describir estas preferencias, en particular, un orden del grado en que se desean todas
las combinaciones posibles. El desarrollo formal de estos dos elementos de la teoría será el punto de
atención en este capítulo. Como el primer elemento —describir el conjunto de posibilidades— es
mucho menos abstracto que el segundo, se empezará por ahí.

EL CONJUNTO DE OPORTUNIDAD
O RESTRICCIÓN DE PRESUPUESTO
Para simplificar el análisis se comenzará considerando un mundo en el que sólo hay dos bienes:2
paquete una combinación de vivienda y alimentación. Un paquete de bienes es el término que se emplea para describir una de-
dos o más bienes. terminada combinación de vivienda medida en yardas cuadradas por semana y alimento medido
en libras por semana. De esta manera, puede haber un paquete que consista en 5 yd2/semana de
vivienda y 7 lb/semana de alimento (paquete A), mientras que otro (paquete B) puede consistir en 3
yd2/semana de vivienda y 8 lb/semana de alimento, como se muestra en la figura 3.1. Para abreviar,
se usará (5, 7) para representar el paquete A y (3, 8) denota el paquete B. En general, (S0, F0) represen-
tará el paquete que tiene S0 yd2/semana de hospedaje y F0 lb/semana de alimento. Por convención, el
primer número del par representa el bien que se mide en el eje horizontal.

FIGURA 3.1
Dos paquetes de bienes Alimentos (lb/semana)
un paquete es una
combinación específica de
bienes. el paquete A tiene B
5 unidades de vivienda y 7 8 A
unidades de alimentos. el 7
paquete B tiene 3 unidades
de vivienda y 8 unidades de
alimentos.

Hospedaje (yd2/semana)
3 5

2
Cuando un economista usa el término “bien” puede estarse refiriendo ya sea a un producto o a un servicio.

03_CHAPTER 3.indd 56 3/6/09 7:04:02 PM


el Conjunto de oPortunidad o restriCCión de PresuPuesto 57

Observe que en ambos ejes las unidades son flujos, lo que significa cantidades físicas por unidad
de tiempo —libras por semana, yardas cuadradas por semana—. El consumo se mide siempre en for-
ma de flujo. Es importante tener en cuenta la dimensión del tiempo porque sin ella no hay manera
de evaluar si una cantidad de consumo dada es grande o pequeña. (Suponga que todo lo que sabe
es que su consumo de alimento es de 4 lb. Si eso es lo que come en un día, es mucho. Pero si eso es
lo que come en un mes, es posible que no sobreviva mucho tiempo.)3
Suponga que el ingreso de una persona es M = 100 dólares/semana y que todo lo gasta en algu-
na combinación de vivienda y alimentación. (Observe que también el ingreso es un flujo.) Suponga,
además, que los precios de la vivienda y del alimento son PS = 5 dólares/yd2 y PF = 10 dólares/lb,
respectivamente. Si el consumidor gasta todo su ingreso en vivienda, podrá comprar M/PS = (100
dólares/semana) ÷ (5 dólares/yd2) = 20 yd2/semana. Es decir, el consumidor podrá comprar el
paquete que consta de 20 yd2/semana de vivienda y 0 lb/semana de alimentos, que se representa
(20, 0). O también se puede suponer que el consumidor gaste todo su ingreso en alimentos. En ese
caso obtendrá el paquete que consiste en M/PF = (100 dólares/semana) ÷ (10 dólares/lb), que es 10
lb/semana de alimentos y 0 yd2 de vivienda, que se representa (0, 10).
Hay que observar que las unidades en las que se miden los bienes de consumo están sujetas a
las reglas comunes de la aritmética. Por ejemplo, al simplificar la expresión en el lado derecho de la
ecuación M/PS = (100 dólares/semana) ÷ (5 dólares/yd2), en esencia se está dividiendo una fracción
entre otra, de manera que se sigue la regla de invertir la fracción del denominador y multiplicarla por
la fracción del numerador (yd2/5 dólares) × (100 dólares/semana) = (100 dólares × yd2)/(5 dólares ×
semana). Después de dividir tanto numerador como denominador de la fracción en el lado derecho
de esta última ecuación entre 5 dólares, se obtiene 20 yd2/semana, que es la cantidad máxima de vi-
vienda que puede comprar el consumidor con un ingreso de 100 dólares/semana. De manera similar
M/PF = (100 dólares/semana) ÷ (10 dólares/lb), que es 10 dólares lb/semana, la cantidad máxima de
alimento que puede comprar el consumidor con un ingreso de 100 dólares/semana.
En la figura 3.2 estos casos extremos se han indicado como K y L, respectivamente. El consu-
midor también puede comprar cualquiera de las otras combinaciones que se encuentran en la línea
Restricción presupuestaria
recta que une los puntos K y L. [Verifique, por ejemplo, que el paquete (12, 4) se encuentra también
conjunto de todos los
sobre esta misma recta.] Esta recta se llama restricción presupuestaria y en el diagrama se ha indi- paquetes que pueden
cado con la letra B. adquirirse empleando todo el
Recuerde que la pendiente de una línea recta es la “variación en y” respecto de “la variación en ingreso de un consumidor a los
x” (el cambio en el eje vertical dividido entre el cambio en el eje horizontal). Observe en este caso precios dados. también se le
que la pendiente de la restricción de presupuesto es la intersección con el eje vertical (la variación llama línea de presupuesto.

FIGURA 3.2
Restricción presupuestal
o línea de presupuesto
Alimentos (lb/semana) la recta B describe el
conjunto de todos los
B paquetes que puede comprar
L el consumidor dados los
M/PF = 10
valores de ingreso y los
8 E
precios. su pendiente es el
Pendiente = – –12 = – (precio de la vivienda) ÷ (precio del alimento) valor negativo de la vivienda
D dividido entre el precio del
4 alimento. en valor absoluto,
esta pendiente es el costo
K de oportunidad de una
Vivienda (yd2/semana)
0 5 12 20 = M/PS unidad más de vivienda, es
decir, la cantidad de unidades
de alimentos que hay que
sacrificar para comprar una
unidad más de vivienda al
precio del mercado.

3
Que el consumo sea un flujo resuelve el problema de que los bienes no sean divisibles. Si se consumen 1.5 lb/mes, entonces se consu-
men 18 lb/año, que es un número entero.

03_CHAPTER 3.indd 57 3/6/09 7:04:03 PM


58 CaPítulo 3 la eleCCión raCional del Consumidor

en y) dividida entre la intersección con el eje horizontal (la variación correspondiente en x): –(10
lb/semana)/(20 yd2/semana) = – _21 lb/yd2. (Observe de nuevo que las unidades obedecen las reglas
de la aritmética.) El signo menos significa que la línea de presupuesto disminuye a medida que se
desplaza hacia la derecha, es decir, que tiene pendiente negativa. En general, si M representa el ingre-
so semanal del consumidor y PS y PF representan los precios de vivienda y comida, respectivamente,
las intersecciones con el eje vertical y con el eje horizontal estarán dadas por (M/PS) y (M/PF), res-
pectivamente. Por lo tanto, la fórmula general para la pendiente de la restricción de presupuesto
está dada por –(M/PF)/(M/PS) = –PS/PF, lo que sencillamente es el valor negativo del cociente de los
precios de los dos bienes. Dados sus respectivos precios, la restricción de presupuesto es la tasa a la
cual el alimento puede ser intercambiado por vivienda. Entonces, en la figura 3.2, 1 lb de comida se
puede intercambiar por 2 yd2 de vivienda. En términos del costo de oportunidad del capítulo 1 se
dirá que el costo de oportunidad de 1 yd2 más de vivienda es PS/PF = 2_1 lb de alimentos.
Además de poder comprar cualquiera de los paquetes que se encuentran a lo largo de la restric-
ción de presupuesto, el consumidor también puede comprar cualquiera que se encuentre dentro del
triángulo de presupuesto limitado por la restricción de presupuesto y los dos ejes. En la figura 3.2, D es
uno de esos paquetes, el cual tiene un precio de 65 dólares/semana, que es bastante menos que el
ingreso del consumidor de 100 dólares/semana. A los paquetes situados sobre el triángulo de presu-
conjunto asequible puesto o dentro de él se les conoce también como conjunto factible o conjunto asequible. Paquetes
paquetes en la restricción como E, que se encuentran fuera del triángulo de presupuesto se dice que son no factibles o inasequibles.
de presupuesto o debajo de Como el precio de E es 140 dólares/semana, queda simplemente fuera del alcance del consumidor.
ella para los cuales el gasto Si S y F representan cantidades de vivienda y alimentos, respectivamente, la restricción de pre-
requerido a los precios dados supuesto debe satisfacer la ecuación siguiente:
es menor o igual al ingreso
disponible. PS S + PF F = M , (3.1)
la cual dice, simplemente, que el gasto semanal del consumidor en vivienda (PSS) más el gasto se-
manal en alimento (PFF) debe ser igual a su ingreso semanal (M). Para expresar la restricción de
presupuesto en la manera convencional que se usa para representar una línea recta, de la ecuación
3.1 se despeja F, con lo que se obtiene
M P
F= – SS (3.2)
PF PF
La ecuación 3.2 es otra manera de ver que la intersección de la restricción de presupuesto con
el eje vertical está dada por M/PF y que su pendiente viene dada por –(PS/PF). En la figura 3.2 la
ecuación de la restricción de presupuesto es F = 10 – _21 S.

DESPLAZAMIENTOS DE LA RESTRICCION DE PRESUPUESTO


DEBIDOS A VARIACIONES EN EL PRECIO O EN EL INGRESO
Variaciones en el precio
La pendiente y la posición de la restricción de presupuesto quedan totalmente determinadas por el
ingreso del consumidor y los precios de los bienes respectivos. Si cualquiera de estos factores cambia

FIGURA 3.3
Efecto de un aumento Alimentos (lb/semana)
en el precio de la
vivenda
Cuando el precio de M/PF = 10
la vivienda aumenta, la
intersección vertical de la B1
restricción de presupuesto B2
Pendiente = –1 Pendiente = – –12
permanece igual. la
restricción de presupuesto
original rota hacia
adentro en torno a esta Vivienda (yd 2/semana)
intersección. 0 M/PS2 = 10 M/PS1 = 20

03_CHAPTER 3.indd 58 3/6/09 7:04:03 PM


el Conjunto de oPortunidad o restriCCión de PresuPuesto 59

se tiene otra restricción de presupuesto. En la figura 3.3 se muestra el efecto del aumento en el precio
de la vivienda de PS1 = 5 dólares/yd2 a PS2 = 10 dólares. Como el ingreso semanal y el precio de los
alimentos no cambian, la intersección de la restricción de presupuesto del consumidor con el eje ver-
tical permanece igual. El aumento en el precio de la vivienda hace que la restricción de presupuesto
rote hacia adentro en torno a esta intersección, como se muestra en el diagrama.
Observe en la figura 3.3 que aun cuando el precio del alimento no ha cambiado, la nueva res-
tricción de presupuesto, B2, hace que disminuya no sólo la cantidad de vivienda que el consumidor
puede comprar sino también la cantidad de alimento.4

EJERCICIO 3.1
Muestre en la figura 3.3 el efecto que tiene sobre la restricción de presupuesto B1 una disminu-
ción del precio de la vivienda de 5 dólares/yd2 a 4 dólares/yd2.
En el ejercicio 3.1 se observó que la disminución del precio de la vivienda no modifica la inter-
sección de la restricción de presupuesto con el eje vertical. Esta vez la restricción de presupuesto rota
hacia afuera. Observó también que aunque el precio de los alimentos permanece igual, la nueva
restricción de presupuesto permite al consumidor comprar paquetes que contienen no sólo más
vivienda, sino también más alimento que los que podía adquirir con la restricción de presupuesto
original.
El ejercicio siguiente muestra cómo la variación del precio del bien en el eje vertical afecta la
restricción de presupuesto.

EJERCICIO 3.2
Muestre en la figura 3.3 el efecto que ejerce sobre la restricción de presupuesto B1 un aumento
en el precio de los alimentos de 10 dólares/lb a 20 dólares/lb.
Cuando varía el precio de un solo bien, necesariamente se modifica la pendiente de la restricción
de presupuesto, –(PS/PF). Lo mismo ocurre si varían ambos precios en proporciones diferentes. Pero
como se muestra en el ejercicio 3.3, la variación de ambos precios en exactamente la misma propor-
ción da lugar a una nueva restricción de presupuesto con la misma pendiente que antes.

EJERCICIO 3.3
Muestre en la figura 3.3 el efecto que ejerce sobre la restricción de presupuesto B3 un aumento
del precio de los alimentos de 10 dólares/lb a 20 dólares/lb y un aumento del precio de la vivien-
da de 5 dólares/yd2 a 10 dólares/yd2.
En el ejercicio 3.3 observe que el efecto de duplicar los precios del alimento y de la vivienda es
que la restricción de presupuesto se desplaza hacia adentro y paralela a la restricción de presupuesto
original. Este ejercicio enseña que la pendiente de una restricción de presupuesto sólo suministra
información de los precios relativos, pero no de los precios en términos absolutos. Cuando los precios
de alimentos y vivienda varían en la misma proporción, el costo de oportunidad de la vivienda, en
términos de los de alimentos, permanece igual que antes.
Variaciones en el ingreso
El efecto de una variación en el ingreso se parece mucho al efecto de una variación de los precios en
una misma proporción. Suponga, por ejemplo, que el ingreso del consumidor hipotético disminuye
a la mitad, de 100 dólares/semana a 50 dólares/semana. La intersección horizontal de la restricción
de presupuesto del consumidor disminuye entonces de 20 yd2/semana a 10 yd2/semana, como se
muestra en la figura 3.4. Por lo tanto, el nuevo presupuesto, B2, es paralelo al anterior, B1, y cada
uno tiene pendiente – _21 . En términos del efecto sobre lo que puede comprar el consumidor, no hay
diferencia entre disminuir el ingreso a la mitad y duplicar cada uno de los precios. Ambos cambios
dan como resultado la misma restricción de presupuesto.

4
La única excepción a esto es la intersección vertical (0, 10), que se encuentra tanto sobre la restricción de presupuesto original
como sobre la nueva restricción de presupuesto.

03_CHAPTER 3.indd 59 3/6/09 7:04:04 PM


60 CaPítulo 3 la eleCCión raCional del Consumidor

FIGURA 3.4
Alimentos (lb/semana)
Efecto de reducir el
ingreso a la mitad
tanto la intersección
vertical como la horizontal M/PF = 10
B1
disminuyen la mitad. la
nueva restricción de
Pendiente = – 12–
presupuesto tiene la misma 0.5M/PF = 5 B2
pendiente que la anterior, Pendiente = – 12–
pero está más cerca del
origen.
Vivienda (yd 2/semana)
0 0.5M/PS = 10 M/PS = 20

EJERCICIO 3.4
Muestre en la figura 3.4 el efecto que tiene sobre la restricción de presupuesto B1 un aumento
del ingreso de 100 dólares/semana a 120 dólares/semana.
El ejercicio 3.4 muestra que un incremento en el ingreso desplaza la restricción de presupuesto
de manera paralela hacia afuera. Como ocurre en el caso de una reducción del ingreso, la pendien-
te de la restricción de presupuesto permanece constante.

PRESUPUESTOS EN LOS QUE HAY DOS O MÁS BIENES


En los ejemplos vistos hasta ahora, el consumidor sólo puede comprar dos bienes. Ningún consumi-
dor se encuentra ante opciones tan estrechas. En su forma más general, el problema del presupues-
to del consumidor puede plantearse como una elección no entre dos, sino entre N bienes diferen-
tes, donde N puede ser un número indefinidamente grande. Si hay sólo dos bienes (N = 2), la
restricción de presupuesto es, como se acaba de ver, una línea recta. Si hay tres bienes (N = 3) es un
plano. Cuando se tienen más de tres bienes, la restricción de presupuesto se transforma en lo que los
matemáticos llaman un hiperplano o un plano multidimensional. Este caso multidimensional es difícil
de representar en forma geométrica. No es posible visualizar superficies de más de tres dimensiones.
El economista del siglo xix, Alfred Marshall, propuso una solución extraordinariamente sencilla
para este problema. Consiste en considerar la elección del consumidor como una decisión entre un
bien particular —llámesele X— y un conjunto de otros bienes, que se representan como Y. A este
bien compuesto en una conjunto se le llama bien compuesto. Por convención, las unidades del bien compuesto están defi-
elección entre un bien X nidas de manera que su precio sea 1 dólar por unidad. Dicha convención permite considerar el bien
y otros numerosos bienes, compuesto como la cantidad de ingreso que le ha sobrado al consumidor después de comprar el
cantidad de dinero que gasta bien X. De manera equivalente, un bien compuesto es la cantidad que el consumidor gasta en los
el consumidor en esos otros demás bienes distintos a X. Por el momento, en todos los ejemplos que se consideren se supondrá
bienes.
que el consumidor gasta todo su ingreso. En el capítulo 5 se usará el modelo de elección racional
para analizar la decisión de ahorrar.
Para mostrar cómo se usa el concepto de bien compuesto, suponga que el ingreso del consumi-
dor es M dólares/semana y que el precio de X es PX. La restricción de presupuesto del consumidor
puede representarse entonces mediante una línea recta en el plano X, Y, como se muestra en la figura
3.5. Como el precio de una unidad del bien compuesto es de 1 dólar, el consumidor que dedica todo
su ingreso a este bien compuesto podrá comprar M unidades. Todo esto significa que si no compra
nada de X dispondrá de M dólares para gastarlo en otros bienes. O, si gasta todo su ingreso en X,
podrá comprar el paquete (M/PX, 0). Como se supone que el precio de Y es 1 dólar/unidad, la pen-
diente de la restricción de presupuesto es simplemente –Px.
Como antes, la restricción de presupuesto resume las diferentes combinaciones de paquetes que
agotan el ingreso del consumidor. Así, en la figura 3.5, el consumidor puede tener X1 unidades de X
y Y1 unidades del bien compuesto o X2 y Y2 o cualquier otra combinación que se encuentre sobre la
restricción del presupuesto.

03_CHAPTER 3.indd 60 3/6/09 7:04:04 PM


el Conjunto de oPortunidad o restriCCión de PresuPuesto 61

FIGURA 3.5
Y (unidades/semana) Restricción de
presupuesto con el bien
M compuesto
en el eje vertical se mide
Y1 la cantidad de dinero que
se gasta semanalmente en
Y2 todos los bienes distintos
Pendiente = –PX a X.

X (unidades/semana)
0 X1 X2 M/PX

RESTRICCIONES PRESUPUESTARIAS QUEBRADAS


Las restricciones de presupuesto vistas hasta ahora han sido líneas rectas. Cuando los precios relati-
vos son constantes, el costo de oportunidad de un bien en términos de cualquier otro es el mismo,
sin importar el paquete de bienes que ya se tenga. Sin embargo, algunas veces las restricciones de
presupuesto son líneas quebradas. Para mostrar esto, considere el ejemplo siguiente de descuento
por volumen.

La empresa Gigawatt Power cobra 0.10 de dólar por kilowatt-hora (kWh) por los primeros EJEMPLO 3.1
1 000 kWh mensuales de energía consumida por un cliente residencial, pero por los siguien-
tes kWh cobra solamente 0.05 de dólar/kWh. Represente gráficamente la restricción de pre-
supuesto de la energía eléctrica y el bien compuesto de un cliente residencial cuyo ingreso
mensual es de 400 dólares.
Si el cliente no compra energía eléctrica, tendrá 400 dólares/mes para gastarlos en otros bienes. De
manera que la intersección vertical de esta restricción de presupuesto es (0, 400). Como se muestra
en la figura 3.6, por cada uno de los primeros 1 000 kWh que compre, tendrá que renunciar a 0.10
1
de dólar, lo que significa que la pendiente de su restricción de presupuesto comienza en – __
10 . A los
1 000 kWh/mes, el precio baja a 0.05 de dólar/kWh, lo que significa que a partir de ese punto, ha-
1
cia la derecha, la pendiente de la restricción de presupuesto es – __
20 .

FIGURA 3.6
Y($/mes) Un descuento por
volumen da lugar a
400 una restricción de
presupuesto no lineal
300 una vez que el consumo
de electricidad llega a
1 000 kWh/mes, el costo
200 de oportunidad de energía
eléctrica adicional baja de
0.10 de dólar/kWh a 0.05
100 de dólar/kWh.

Cantidad (miles de kWh/mes)


0 1 2 3 4 5 6 7

Observe que a lo largo de la restricción de presupuesto que se observa en la figura 3.6, el costo
de oportunidad de la electricidad depende de cuánto haya comprado ya el consumidor. Considere un
consumidor que hasta este momento haya usado 1 020 kWh por mes y esté decidiendo si dejar en-
cendida durante la noche la luz de la puerta de su casa, lo que ocasionará 20 kWh más de consumo.
Dejar la luz encendida le significará 1 dólar/mes adicional. Sin embargo, si su consumo normal fuera
980 kWh/mes, el precio de dejar encendida la luz de la puerta de su casa subirá a 2 dólares/mes.
De acuerdo con esta diferencia puede pronosticarse que es más probable que sean las personas que

03_CHAPTER 3.indd 61 3/6/09 7:04:05 PM


62 CaPítulo 3 la eleCCión raCional del Consumidor

ya hayan consumido una cantidad grande de electricidad (más de 1 000 kWh/mes) quienes dejen
encendida la luz de la puerta de entrada de su casa en comparación con las demás personas.

EJERCICIO 3.5
Suponga ahora que la empresa Gigawatt Power cobra 0.05 de dólar/kWh por los primeros
1 000 kWh de energía que consuma cada mes un cliente residencial y 0.10 de dólar/kWh por
los kilowatt-hora restantes. Grafique la restricción de presupuesto para la electricidad y el
bien compuesto de un consumidor residencial cuyo ingreso mensual sea de 400 dólares. ¿Qué
ocurre si la tarifa cambia a 0.10 de dólar/kWh, para todos los kilowatt hora, en caso de que el
consumo mensual sea mayor a 1 000 kWh (en cuyo caso la nueva tarifa se aplica a todos los
kilowatt-hora, no sólo a los adicionales)?

SI LA RESTRICCIÓN DE PRESUPUESTO ES LA MISMA,


LA DECISIÓN DEBE SER LA MISMA
Aun sin saber nada acerca de las preferencias de un consumidor, la información presupuestaria puede
emplearse para hacer ciertas inferencias acerca de cómo se comportará un consumidor racional.
Suponga, por ejemplo, que los gustos del consumidor no cambian con el tiempo y que en dos situa-
ciones diferentes se enfrenta a exactamente la misma restricción de presupuesto. Si el consumidor es
racional, hará la misma elección en ambas situaciones. Después de todo, si la restricción de presu-
puesto es igual que antes, el consumidor tendrá el mismo menú de paquetes disponibles y dado que
no existe ninguna razón para pensar que la jerarquía de deseabilidad de éstos haya cambiado, el más
deseable seguirá siendo el mismo. Sin embargo, como muestra el ejemplo siguiente, puede que no
siempre sea claro que las restricciones de presupuesto sean en realidad las mismas.

EJEMPLO 3.2 En una ocasión Gowdy llena el tanque de gasolina de su automóvil la noche anterior a su
salida a un día de pesca. Cuando despierta descubre que un ladrón le ha sacado toda la ga-
solina del tanque de 21 galones de capacidad dejándole nada más un galón. En otra ocasión
decide cargar combustible a la mañana siguiente, antes de irse de pesca. Cuando despierta
se da cuenta de que ha perdido 60 dólares de su billetera. Si la gasolina tiene un precio de 3
dólares/galón y en el viaje redondo para ir de pesca se consumen cinco galones, ¿será diferen-
te, en estos dos casos, la decisión de Gowdy respecto a ir de pesca? (Suponga que, fuera de
los costos monetarios, el inconveniente de tener que volver a llenar el tanque de gasolina es
insignificante.)
Suponga que el ingreso de Gowdy es M dólares/mes. Antes de la pérdida su restricción de presu-
puesto es B1, como se muestra en la figura 3.7. En las dos situaciones descritas, su restricción de
presupuesto en el momento en que descubre la pérdida se desplaza hacia dentro, a B2.

FIGURA 3.7
Restricciones de Y($/mes)
presupuesto tras un
robo de gasolina o una M
pérdida de efectivo M – 60
un robo de 60 dólares
en gasolina tiene B1
exactamente el mismo
efecto sobre la restricción
de presupuesto que la B2
pérdida de 60 dólares en
efectivo. el paquete elegido Gasolina (galones/mes)
deberá ser el mismo,
independientemente de la
0
( M –2 60 ) M2
causa de la pérdida.

03_CHAPTER 3.indd 62 3/6/09 7:04:05 PM


PreferenCias del Consumidor 63

En ambos casos, si no va a pescar contará con M – 60 dólares para gastar en otros bienes. Y si va a
pescar, en ambos casos, tendrá que comprar la gasolina necesaria a 3 dólares/galón. No importa cuál
sea la causa de la pérdida, las oportunidades restantes son exactamente las mismas. Si el presupuesto
de Gowdy es muy limitado, es probable que decida no ir de pesca. Si no es así, puede que vaya a
pescar a pesar de la pérdida. Pero como su restricción de presupuesto y sus gustos son los mismos
tanto en el caso de la pérdida en efectivo como en el caso del robo de gasolina, no sería razonable que
en un caso decidiera ir de pesca y en el otro no.

Observe que la situación planteada en el ejemplo 3.2 tiene la misma estructura que la descrita
en el ejemplo del disco que se rompe, con el cual inició este capítulo. Ése también es un ejemplo en
el que la decisión debería ser la misma en ambos casos puesto que la restricción de presupuesto y los
gustos son los mismos en ambos.
Aunque el modelo de la elección racional dice que la decisión deberá ser la misma si la restricción
de presupuesto y las preferencias son las mismas, en ocasiones las personas reaccionan de manera
diferente. El problema es que la manera en la que se desarrollan las distintas situaciones suele hacer
que la gente no vea las semejanzas esenciales entre ellas. Por ejemplo, en el ejemplo 3.2 mucha gente
concluye de manera errónea que en el caso de la gasolina robada el costo de ir de pesca es mayor
que en el caso del dinero perdido y por lo tanto es menos probable que vayan de pesca en la primera
situación. De manera similar, muchas personas se vieron menos inclinadas a comprar el disco des-
pués de haber roto el primero, que después de haber perdido 10 dólares, porque pensaron de forma
incorrecta que en la situación del disco roto, éste resultaba más caro. Sin embargo, como se ha visto,
la cantidad que se ahorra al no comprar el disco o al no ir de pesca es exactamente la misma en las
dos situaciones.
Para recapitular, la restricción de presupuesto o línea de presupuesto resume las combinaciones
de paquetes que puede comprar el consumidor. La posición de esta línea está determinada por el
ingreso y por los precios. Dado el conjunto de paquetes factibles, la tarea del consumidor es elegir
el que más le guste. Para identificar este paquete se necesitan ciertos medios que resumen las pre-
ferencias del consumidor respecto de todos los paquetes que puede consumir. A continuación se
hablará de esta tarea.

PREFERENCIAS DEL CONSUMIDOR


Para simplificar se empezará considerando un mundo en el que sólo hay dos bienes: vivienda y ali-
mento. Un orden de preferencia permite jerarquizar los distintos paquetes de bienes en términos de orden de preferencia una
su deseabilidad, es decir, su orden de preferencia. Considere dos paquetes A y B. En concreto, supon- jerarquización de todos los
ga que A contiene 4 yd2/semana de vivienda y 2 lb/semana de alimento y que B tiene 3 yd2/semana posibles paquetes de consumo
de vivienda y 3 lb/semana de alimento. Si no se sabe nada acerca de las preferencias del consumidor, en orden de preferencia.
no se podrá decir cuál de los paquetes preferirá. A tiene más vivienda pero menos alimento que B. Es
probable que una persona a la que le guste estar en casa prefiera A, mientras una persona que tenga
un metabolismo muy rápido prefiera B.
Se supone que, en general, dados dos paquetes, el consumidor tiene las siguientes tres posi-
bilidades: 1) A se prefiere en lugar de B, 2) B se prefiere en lugar de A y 3) A y B resultan igualmente
atractivos. El orden de preferencia permite al consumidor jerarquizar los distintos paquetes pero
no dar de una manera cuantitativa más precisa su deseabilidad relativa. Es decir, el consumidor se-
rá capaz de decir que prefiere A en lugar de B, pero no que A le proporciona el doble de satisfacción
que B.
El orden de preferencia suele diferir ampliamente entre los consumidores. A una persona le pue-
de gustar Rachmaninoff y a otra la banda Red Hot Chili Peppers. Sin embargo, a pesar de estas di-
ferencias la mayor parte de los órdenes de preferencia comparten varias características importantes.
Los economistas suponen, por lo general, que los órdenes de preferencia tienen cuatro propiedades
sencillas. Éstas permiten construir la representación analítica concisa de las preferencias necesaria
para el problema de la asignación de presupuesto.

03_CHAPTER 3.indd 63 3/6/09 7:04:06 PM


64 CaPítulo 3 la eleCCión raCional del Consumidor

1. Completitud
Un orden de preferencia está completo si permite al consumidor jerarquizar todas las combinaciones
posibles de bienes y servicios. Tomado en sentido literal, el supuesto de completitud nunca se satisfa-
ce porque hay demasiados bienes de los que se conoce muy poco para poder evaluarlos. No obstante,
es un supuesto de simplificación útil para el análisis de las elecciones entre paquetes de bienes con
los que los consumidores están familiarizados. Su objetivo real es eliminar toda posibilidad de que
ocurran casos como el que se representa en la fábula del asno de Buridán. El hambriento animal no
puede decidir entre dos montones de heno que tiene enfrente y muere de inanición.
2. Más es mejor
La propiedad “más es mejor” significa que, si todo lo demás permanece igual, es preferible más de un
bien que menos. Por supuesto que existen ejemplos de que más de algo dañe en lugar de beneficiar
(por ejemplo, alguien que coma en exceso). Pero estos ejemplos, por lo general, contemplan algún
tipo de problema práctico, por ejemplo, un problema de autocontrol o de no ser capaz de guardar un
bien para uso futuro. Mientras las personas puedan almacenar o disponer con libertad de bienes que
no desean, tener más de algo no puede hacerles daño.
Como ejemplo de la aplicación del supuesto “más es mejor”, considere dos paquetes: A, que tie-
ne 12 yd2/semana de vivienda y 10 lb/semana de alimentos y B, que tiene 12 yd2/semana de vivienda
y 11 lb/semana de alimentos. Este supuesto dice que B se prefiere en lugar de A porque tiene más
alimento pero no menos vivienda.
3. Transitividad
Si a usted le gustan más los bisteces que las hamburguesas y las hamburguesas más que los hot dogs,
es probable que le gusten más los bisteces que los hot dogs. Decir que el orden de preferencia del
consumidor es transitivo significa que, dados tres paquetes A, B y C cualesquiera, si el consumidor
prefiere A en lugar de B y B en lugar de C, entonces preferirá siempre A en lugar de C. Por ejemplo, su-
ponga que A es (4, 2), B es (3, 3) y C es (2, 4). Si usted prefiere (4, 2) en vez de (3, 3) y (3, 3) en lugar de
(2, 4), entonces preferirá (4, 2) más que (2, 4). De manera que se supone que la relación de preferencia
es como la relación que se usa para comparar la estatura de las personas. Si O’Neal es más alto que
Nowitzki y Nowitzki es más alto que Bryant, entonces se sabe que O’Neal es más alto que Bryant.
No todas las relaciones de comparación son transitivas. La relación “medio hermano”, por ejem-
plo, no es transitiva. Yo tengo una media hermana que, por su parte, tiene tres medias hermanas.
Pero sus medias hermanas no son mis medias hermanas. Otra relación que no es transitiva es la rela-
ción “derrotar en el fútbol”. En una temporada, Ohio State derrota a Michigan y Michigan derrota a
Michigan State, pero esto no quiere decir que necesariamente Ohio State derrote a Michigan State.
La transitividad es una propiedad de consistencia simple y se aplica tanto a la relación “igual
de atractivo que” como a cualquier combinación de ella y la relación “se prefiere en lugar de”. Por
ejemplo, si A es tan atractivo como B y B es tan atractivo como C, se concluye que A es tan atractivo
como C. De manera similar, si A se prefiere en lugar de B y B se prefiere en lugar de C, se concluye
que A se prefiere en lugar de C.
El supuesto de transitividad puede justificarse porque elimina la posibilidad del problema de la
“bomba de dinero”. Para ilustrar esto, suponga que usted prefiere A en lugar de B y B en lugar de C,
pero que también prefiere C en vez de A, de manera que sus preferencias son intransitivas. Si empieza
con C, usted intercambiará C por B, B por A y después A por C. Este ciclo puede continuar eterna-
mente. Si en cada etapa se le cobrara una cantidad muy pequeña por cada intercambio, acabaría
transfiriendo todo su dinero al otro negociante. Es claro que tales preferencias son problemáticas.
A pesar de que la propiedad de transitividad suena tan razonable, en capítulos posteriores se
verán ejemplos de comportamientos que parecen inconsistentes con ella. Pero en la mayoría de los
casos la transitividad es una descripción exacta de las preferencias. A menos que se diga otra cosa
aquí se adoptará la transitividad.
4. Convexidad
Se prefieren las mezclas de bienes a los extremos. Si a usted le son indiferentes dos paquetes A y B,
sus preferencias son convexas si prefiere un paquete que contenga la mitad de A y la mitad de B (o
cualquier otra mezcla) en vez de los paquetes originales. Por ejemplo, suponga que a usted le son

03_CHAPTER 3.indd 64 3/6/09 7:04:06 PM


PreferenCias del Consumidor 65

indiferentes los paquetes A = (4, 0) y B (0, 4). Si sus preferencias son convexas, preferirá el paquete (2,
2) en vez de cualquiera de los paquetes extremos. Esta propiedad expresa la idea de que nos gusta que
haya balance en las mezclas de bienes de consumo.

CURVAS DE INDIFERENCIA
Considere ahora algunas implicaciones de estos supuestos acerca de los órdenes de preferencia. Lo
más importante es que permiten generar una descripción gráfica de las preferencias del consumi-
dor. Para ver cómo, considere primero el paquete A en la figura 3.8, el cual tiene 12 yd2/semana de
vivienda y 10 lb/semana de alimentos. El supuesto “más es mejor” dice que todos los paquetes al
noreste de A se prefieren en lugar de A, y que A a su vez se prefiere a todos los paquetes al suroeste de
A. Por lo tanto, el supuesto “más es mejor” indicará que Z, que tiene 28 yd2/semana de vivienda y 12
lb/semana de alimentos, se prefiere en lugar de A y que, a su vez, A se prefiere en vez de W, que tiene
sólo 6 yd2/semana de vivienda y 4 lb/semana de alimentos.

FIGURA 3.8
Alimento (lb/semana) Generación de paquetes
con igual preferencia
Z se prefiere en lugar de A
12 Z porque tiene más de cada
Mejor
A que A bien que A. Por la misma
10
razón, A se prefiere en
Peor B lugar de W. se concluye
8 que A
7 C que sobre la recta que
6 une W con Z debe haber
un paquete B que sea tan
W
4 atractivo como A. de la
misma manera se puede
2 hallar un paquete C que sea
tan atractivo como B.
Vivienda (yd 2/semana)
0 6 12 17 20 28

Considere ahora el conjunto de paquetes que se encuentran sobre la recta que une W y Z. Como
Z se prefiere en lugar de A y A se prefiere en lugar de W, se decide que a medida que se avanza de Z a
W se tendrá que encontrar un paquete que sea igualmente atractivo que A. (La idea intuitiva detrás
de esto es la misma que dice que si se escala una montaña desde un punto a 1 000 pies sobre el nivel
del mar hasta otro a 2 000 pies sobre el nivel del mar, a lo largo del camino se tendrá que pasar por
todas las altitudes intermedias.) Sea B el paquete que es igual de atractivo que A, y suponga que con-
tiene 17 yd2/semana de vivienda y 8 lb/semana de alimentos. (Las cantidades exactas de cada bien
en B dependerán, por supuesto, del consumidor de cuyas preferencias se esté hablando.) El supuesto
“más es mejor” también dice que en la recta entre W y Z habrá sólo un paquete con estas característi-
cas. En dicha recta todos los puntos al noreste de B son mejores que B; aquellos que se encuentran al
suroeste de B son peores que B.
De la misma manera se puede hallar otro punto —llámele C— que sea tan atractivo como B. C
aparece como el paquete (20, 7), en el que las cantidades específicas dependen de nuevo de las prefe-
rencias del consumidor en consideración. Por el segmento de transitividad se sabe que C es también
tan atractivo como A (ya que C es tan atractivo como B, el cual es tan atractivo como A).
Este proceso se puede repetir tantas veces como se desee, el resultado final es una curva de curva de indiferencia un
indiferencia, un conjunto de paquetes en el que todos son igualmente atractivos que el original A y, conjunto de paquetes que le
por lo tanto, también igual de atractivos entre sí. En la figura 3.9 este conjunto aparece como la curva son indiferentes al consumidor.
I. A ésta se le llama curva de indiferencia porque al consumidor le son indiferentes todos los paquetes
que se encuentran a lo largo de ella.
Una curva de indiferencia también permite comparar la satisfacción implícita en paquetes que
se encuentran a lo largo de ella con los que se encuentran arriba o debajo de ella. Permite, por ejem-
plo, comparar el paquete C (20, 7) con el K (23, 4), que tiene menos alimento y más vivienda que C. Se

03_CHAPTER 3.indd 65 3/6/09 7:04:07 PM


66 CaPítulo 3 la eleCCión raCional del Consumidor

FIGURA 3.9
Una curva de Alimento (lb/semana)
indiferencia 12 L
es un conjunto de paquetes
que el consumidor 10 A
considera igualmente
atractivos. Cualquier B
8
paquete, como el L, que C
7 D
se encuentre arriba de
6
una curva de indiferencia I0
se prefiere en lugar de K
4
cualquier paquete sobre
la curva de indiferencia. a
2
su vez, cualquier paquete
que se encuentre sobre
Vivienda (yd 2/semana)
la curva de indiferencia 5 12 17 20 25
se prefiere en vez de 11 23
cualquier otro, como el
K, que se encuentre por
debajo de ella.

sabe que C es tan atractivo como D (25, 6) porque ambos se encuentran a lo largo de la misma curva
de indiferencia. D, a su vez, se prefiere en lugar de K debido a la suposición “más es mejor”: D tiene
2 yd2/semana más vivienda y 2 lb/semana más alimento que K. Por último, la transitividad dice que
como C es tan atractivo como D y D se prefiere en lugar de K, C debe preferirse en vez de K.
Mediante un razonamiento análogo, se puede decir que el paquete L se prefiere en lugar de A.
En general, los paquetes que se encuentran por arriba de una curva de indiferencia se prefieren en vez de los que
se encuentran en estas curvas. De manera similar, todos los que se encuentran en una curva de indiferencia se
prefieren en lugar de los que se encuentran debajo de ella.
La propiedad de completitud de las preferencias implica que existe una curva de indiferencia que
pasa a través de todo paquete posible. Si esto es así, las preferencias de un consumidor pueden que
mapa de indiferencia una representarse mediante un mapa de indiferencia, un ejemplo del cual se muestra en la figura 3.10.
muestra representativa Éste muestra nada más cuatro de las infinitas curvas de indiferencia que, juntas, dan una descripción
del conjunto de curvas de completa de las preferencias del consumidor.
indiferencia de un consumidor, En la figura 3.10, los números I1,…, I4 son valores índice que se usan para representar el orden
se usa como un resumen de preferencia correspondiente a cada una de las curvas de indiferencia. Los números índice deben
gráfico de su orden de
satisfacer la propiedad I1 < I2 < I3 < I4. Al representar las preferencias del consumidor, lo que realmen-
preferencia.
te cuenta es la jerarquía de las curvas de indiferencia, no los valores numéricos particulares que se les
asignen.5

FIGURA 3.10
Parte de un mapa de Alimentos (lb/semana)
indiferencia
al conjunto completo de
las curvas de indiferencia Satisfacción creciente
de un consumidor se le
conoce como mapa de
indiferencia del consumidor.
los paquetes en una
curva de indiferencia se I4
prefieren menos que los
que se hallan en una curva I3
de indiferencia superior y I2
más que los que están en I1
una curva de indiferencia
inferior. Vivienda (yd 2/semana)

5
Para una discusión más completa de este tema, vea las páginas 87-89 en el apéndice de este capítulo.

03_CHAPTER 3.indd 66 3/6/09 7:04:07 PM


PreferenCias del Consumidor 67

FIGURA 3.11
Alimento (lb/semana) ¿Por qué dos curvas
de indiferencia no se
cruzan?
si las curvas de indiferencia
se cruzaran, violarían
por lo menos una de las
propiedades de los órdenes
de preferencia.

D
F

E
Vivienda (yd 2/semana)

Las cuatro propiedades de los órdenes de preferencia implican cuatro propiedades importantes
de las curvas y los mapas de indiferencia:
1. Las curvas de indiferencias son ubicuas. Cada paquete tiene una curva de indiferencia que pasa
a través de él. Esta propiedad está garantizada por la propiedad de completitud de las preferen-
cias.
2. Las curvas de indiferencia tienen pendiente negativa. Una curva de indiferencia con pendiente
positiva violaría la propiedad “más es mejor”, pues diría que un paquete con más de ambos
bienes es equivalente a un paquete con menos de ambos bienes.
3. Las curvas de indiferencia (de un mismo mapa de indiferencia) no pueden cruzarse. Para ver
por qué, suponga que, como se muestra en la figura 3.11, dos curvas de indiferencia se cruzan.
Entonces las siguientes afirmaciones tendrían que ser verdaderas:
E es tan atractivo como D (porque los dos se encuentran en la misma curva de indiferencia).
D es tan atractivo como F (porque los dos se encuentran en la misma curva de indiferencia).
E es tan atractivo como F (por el supuesto de transitividad).
Pero se sabe también que: F se prefiere en vez de E (porque “más es mejor”).
Como no es posible que las afirmaciones E es tan atractivo como F y F se prefiere más que E sean
verdaderas simultáneamente, el supuesto de que dos curvas de indiferencia se crucen implica
una contradicción. La conclusión es que la proposición original debe ser verdadera, es decir, dos
curvas de indiferencia no se pueden cruzar.
4. Las curvas de indiferencia se vuelven menos inclinadas a medida que se avanza a lo largo de ellas
hacia abajo y hacia la derecha. Como se vio antes, esta propiedad es resultado de la propiedad de
convexidad de las preferencias.

ALTERNANCIA ENTRE BIENES


Una propiedad importante de las preferencias del consumidor es la tasa a la cual está dispuesto a
intercambiar, o a “sustituir”, una cosa por otra. Esta tasa está representada en cualquier punto de una
curva de indiferencia por la tasa marginal de sustitución (TMS), la cual está definida como el valor tasa marginal de sustitución
absoluto de la pendiente de la curva de indiferencia en ese punto. En el lado izquierdo de la figura (TMS) en cualquier punto de
3.12, por ejemplo, la tasa marginal de sustitución en el punto A está dada por el valor absoluto de la la curva de indiferencia, la tasa
pendiente de la tangente a la curva de indiferencia en A, la cual es el cociente ∆FA/∆SA.6 (La notación a la cual el consumidor está
∆FA significa “un pequeño cambio en alimento a partir de la cantidad inicial en el punto A”.) Si al dispuesto a intercambiar el
bien medido en el eje vertical
consumidor se le quita ∆FA unidades de alimento en el punto A, habrá que darle ∆SA unidades más de
por el bien medido en el eje
horizontal; es igual al valor
6
De manera más formal, la curva de indiferencia puede expresarse como una función Y = Y(X) y la TMS en el punto A se define
absoluto de la pendiente de la
como el valor absoluto de la derivada de la curva de indiferencia en ese punto: MRS = |dY(X)/dX|. curva de indiferencia.

03_CHAPTER 3.indd 67 3/6/09 7:04:07 PM


68 CaPítulo 3 la eleCCión raCional del Consumidor

FIGURA 3.12
Tasa marginal de
Área ampliada
sustitución Alimento (lb/semana)
la tms en cualquier punto
a lo largo de la curva de TMSA = |ΔFA /ΔSA|
indiferencia está definida
como el valor absoluto A A
ΔFA 2
de la pendiente de la
curva de indiferencia en ΔSA
ese punto. es la cantidad
I0 1
de alimento que se debe
dar al consumidor para Vivienda(yd 2/semana)
compensar la pérdida de
una unidad de vivienda.

vivienda para dejarlo en la misma buena situación en que estaba. En la figura, la gráfica de la derecha
muestra una ampliación de la región que rodea al paquete A. Si la tasa marginal de sustitución en A
es 2, esto significa que al consumidor habrá que darle 2 lb/semana de alimento para compensar la
pérdida de 1 yd2/semana de vivienda.
Mientras la pendiente de la restricción de presupuesto indica la tasa a la que se puede sustituir
alimento por vivienda sin modificar el gasto total, la TMS representa la tasa a la que se puede susti-
tuir alimento por vivienda sin modificar la satisfacción total. Dicho de otra manera, la pendiente de
la restricción de presupuesto es el costo marginal de la vivienda en términos del alimento y la TMS es
el beneficio marginal del hospedaje en términos del alimento.
La propiedad de convexidad de las preferencias indica que a lo largo de cualquier curva de indi-
ferencia, cuenta más cantidad de un bien tenga el consumidor, más habrá que darle del mismo para
que esté dispuesto a renunciar a una unidad de otro bien. Dicho de otra manera, la TMS disminuye a
medida que se recorre la curva de indiferencia hacia abajo y hacia la derecha. Las curvas de indiferen-
cia con tasas de sustitución marginal decrecientes son convexas —o arqueadas hacia fuera— vistas
desde el origen. Las curvas de indiferencia que se muestran en las figuras 3.9, 3.10 y 3.12 tienen esta
propiedad, así como la curva que muestra la figura 3.13.
Observe en la figura 3.13 que en el paquete A el alimento es relativamente abundante y el con-
sumidor estará dispuesto a sacrificar 3 lb/semana para obtener una yarda cuadrada más de vivienda.
Su TMS en A es tres. En C, las cantidades de alimento y hospedaje están más balanceadas y ahí el
consumidor está dispuesto a renunciar nada más a 1 lb/semana para obtener una yarda cuadrada
más de vivienda. Su TMS en C es 1. Por último, observe que el alimento en D es relativamente escaso

FIGURA 3.13
Tasa marginal de Alimento (lb/semana)
sustitución decreciente
Cuanto más alimento
tiene el consumidor,
17 3 A (3, 17)
más dispuesto está a
sacrificar para obtener una
1
unidad más de vivienda.
las tasas marginales de
sustitución en los paquetes
A, C y D son 3, 1 y 1/4, 9 1 C (7, 9)
respectivamente. 1

D (16, 3)
3 1
4 1 I0
Vivienda (yd 2/semana)
0 3 7 16

03_CHAPTER 3.indd 68 3/6/09 7:04:08 PM


el Paquete más asequible 69

y estará dispuesto a renunciar a sólo ¼ lb/semana de alimento para obtener una unidad más de
vivienda. La TMS en D es –14 .
De manera intuitiva, la TMS decreciente significa que al consumidor le gusta la variedad. Las
personas solemos estar dispuestas a renunciar a bienes de los que ya tenemos mucho para obtener
más de los que sólo tenemos poco.

USO DE CURVAS DE INDIFERENCIA PARA DESCRIBIR


PREFERENCIAS
Para tener una idea de cómo describen los mapas de indiferencia las preferencias del consumidor,
se verá su uso para reflejar las distintas preferencias entre dos consumidores. Suponga, por ejem-
plo, que tanto a Tex como a Mohan les gustan las papas, pero que a Mohan el arroz le gusta mucho
más que a Tex. Esta diferencia de gustos se capta en las distintas pendientes de sus curvas de indife-
rencia, que se muestran en la figura 3.14. En la figura 3.14a, que muestra el mapa de indiferencia de
Tex observe que en el paquete A Tex está dispuesto a cambiar 1 lb de papas por 1 lb de arroz. Pero
en el paquete correspondiente de la figura 3.14b, que muestra el mapa de indiferencia de Mohan, se
observa que Mohan está dispuesto a cambiar 2 lb de papas por 1 lb de arroz. La discrepancia en sus
preferencias se muestra con claridad en estas diferencias de sus tasas de sustitución marginales de
papas por arroz.

FIGURA 3.14
Papas (lb/semana) Papas(lb/semana) Personas con gustos
diferentes
Mapa de Hablando en términos
indiferencia relativos, tex es un amante
de Tex de las papas; mohan lo es
Mapa de del arroz. esta diferencia se
A A indiferencia hace patente en el hecho
5 5 de Mohan de que para cualquier
I3
1 paquete dado la tasa
4 2
1 marginal de sustitución de
I2
I3 tex de papas por arroz es
3
I1 1 menor que la de mohan.
I2
I1
Arroz Arroz
3 4 (lb/semana) 3 4 (lb/semana)
a) b)

EL PAQUETE MÁS ASEQUIBLE


Ahora ya se tienen las herramientas necesarias para determinar cómo distribuirá el consumidor su
ingreso entre dos bienes. El mapa de indiferencia indica cómo están jerarquizados los diferentes pa-
quetes en orden de preferencia. La restricción de presupuesto, a su vez, indica qué paquetes son ase-
quibles. La tarea del consumidor es juntar los dos y elegir el paquete que prefiera o el paquete más paquete más asequible el
asequible. (Recuerde del capítulo 1 que no es necesario suponer que los consumidores piensan de paquete más preferido entre
manera explícita en las restricciones de presupuesto y en los mapas de indiferencia cuando deciden los paquetes asequibles.
qué comprar. Es suficiente suponer que las personas toman decisiones como si estuvieran pensando
en estos términos, de la misma manera que el jugador de billar experto elige los tiros como si cono-
ciera todas las leyes relevantes de la física newtoniana.)
Considere de nuevo la elección entre alimento y vivienda a la que se enfrenta un consumidor
cuyo ingreso es M = 100/semana si los precios son PF = 10 dólares/lb y PS = 5 dólares/yd2. La figura
3.15 muestra esta restricción de presupuesto del consumidor y parte de su mapa de indiferencia. De
los cinco paquetes, identificados A, D, E, F y G, del diagrama, G es el preferido porque se encuentra en
la curva de indiferencia más alta. Sin embargo, no es asequible como tampoco lo son otros paquetes
que se encuentran más allá de la restricción de presupuesto. El supuesto “más es mejor” implica que

03_CHAPTER 3.indd 69 3/6/09 7:04:08 PM


70 CaPítulo 3 la eleCCión raCional del Consumidor

FIGURA 3.15
El paquete más Alimento (lb/semana)
asequible
lo mejor que puede
hacer el consumidor es
elegir el paquete de la 10 A Paquete más asequible
9 G
restricción de presupuesto 8
que se encuentre en la F
curva de indiferencia 6 I3
más alta posible. en este 5
caso, ese paquete es el F, D I2
que se encuentra en la E
2 I1
tangencia entre la curva de
indiferencia y la restricción Vivienda (yd 2/semana)
0 2 6 8 15 16 20
de presupuesto.

el paquete más asequible debe estar sobre la restricción de presupuesto, no dentro de ella. (Cualquier
paquete dentro de la restricción de presupuesto se preferirá menos que uno que esté ligeramente al
noreste, el cual también puede ser asequible.)
Con exactitud, ¿dónde, a lo largo de la restricción de presupuesto, se encuentra el mejor paquete
asequible?
Se sabe que no puede estar en una curva de indiferencia que se encuentre de manera parcial
dentro de la restricción de presupuesto. En la curva de indiferencia I1, por ejemplo, los únicos puntos
que podrían ser candidatos a ser el mejor paquete asequible son los dos que se encuentran en la
restricción de presupuesto, es decir, los puntos A y E. Pero A no puede ser el mejor paquete asequible
porque es tan atractivo como D, que a su vez es menos deseable que F debido al supuesto “más es
mejor”. De manera que, por transitividad, A es menos deseable que F. Por la misma razón, E tampo-
co puede ser el paquete más asequible.
Como el paquete más asequible no puede estar sobre una curva de indiferencia que se encuen-
tre parcialmente dentro de la restricción de presupuesto y como debe encontrarse en la restricción
de presupuesto, se sabe que debe ubicarse en una curva de indiferencia que sea tangente a la restric-
ción de presupuesto. En la figura 3.15, esta curva de indiferencia es la que se identifica como I2 y
el paquete más asequible es F, que se encuentra en el punto de tangencia entre I2 y la restricción
de presupuesto. Si se tiene un ingreso de 100 dólares/semana y si el precio de la vivienda es de 5
dólares/yd2 y el del alimento 10 dólares/lb, lo mejor que puede hacer este consumidor es comprar 6
lb/semana de alimento y 8 yd2/semana de vivienda.
De manera intuitiva elegir el paquete F parece perfectamente razonable. El objetivo del con-
sumidor es, después de todo, dada su restricción de presupuesto, alcanzar la curva de indiferen-
cia más alta posible. Su estrategia es pasar a curvas de indiferencia cada vez más altas hasta llegar a la
más alta posible. En mapas de indiferencia en los que existe un punto de tangencia, como en la figura
3.15, el mejor paquete se encuentra siempre en el punto de tangencia.
Observe en la figura 3.15 que la tasa marginal de sustitución en F es igual al valor absoluto de
la pendiente de la restricción de presupuesto. Esto será así siempre que el paquete más asequible se
presente en un punto de tangencia. Por lo tanto, la condición que en tales casos debe satisfacerse es
P
TMS = S , (3.3)
PF
El lado derecho de la ecuación 3.3 representa el costo de oportunidad de vivienda en términos
del alimento. Por lo tanto, si PS = 5 dólares/yd2 y PF = 10 dólares/lb, el costo de oportunidad de una
yarda cuadrada más de vivienda es –12 lb de alimento. El lado izquierdo de la ecuación 3.3 es |∆F/∆S|,
el valor absoluto de la pendiente de la curva de indiferencia en el punto de tangencia. Es la cantidad
adicional de alimento que hay que dar al consumidor para compensarlo completamente por la pér-
dida de una de yarda cuadrada de vivienda. En el lenguaje del análisis costo-beneficio que se discutió
en el capítulo 1, la pendiente de la restricción de presupuesto representa el costo de oportunidad de
la vivienda en términos del alimento, mientras que la pendiente de la curva de indiferencia represen-

03_CHAPTER 3.indd 70 3/6/09 7:04:09 PM


el Paquete más asequible 71

ta el beneficio de consumir vivienda comparado con consumir alimento. Como en este ejemplo la
pendiente de la restricción de presupuesto es – –12 la condición de tangencia indica que se necesitará
–12 lb de alimento para compensar los beneficios a que se renuncia con la pérdida de una yarda cuadra-
da de vivienda.
Si el consumidor se encontrara en algún paquete sobre la línea de presupuesto para el cual las
dos pendientes no fueran iguales, entonces siempre podría comprar un paquete mejor. Para ver por
qué, suponga que el consumidor está en un punto en el que la pendiente de la curva de indiferencia
(en valor absoluto) es menor que la pendiente de la restricción de presupuesto (también en valor
absoluto), como en el punto E de la figura 3.15. Suponga, por ejemplo, que la TMS en E es de sólo
–14 . Esto indica que el consumidor puede ser compensado por la pérdida de una yarda cuadrada de
vivienda dándole –14 lb más de alimento. Pero la pendiente de la restricción de presupuesto indica que
al renunciar a una yarda cuadrada de vivienda, el consumidor puede comprar –12 lb más de alimento.
Como esto es –14 de libra más de lo que necesita para quedar igualmente satisfecho, es claro que estará
en mejor posición si compra más alimento y menos vivienda que en el punto E. El costo de oportuni-
dad de una libra más de alimento es menor que el beneficio que confiere.

EJERCICIO 3.6
Suponga que la tasa marginal de sustitución en el punto A de la figura 3.15 es 1.0. Demuestre
que esto significa que el consumidor se verá más beneficiado si compra menos alimento y más
vivienda que en A.

SOLUCIONES DE ESQUINA
El paquete más asequible no necesariamente se encontrará siempre en un punto de tangencia. Hay
casos en los que puede no haber punto de tangencia —puede ser que en todas partes la TMS sea
mayor o menor que la pendiente de la restricción de presupuesto—. En este caso se obtiene una
solución de esquina, como la que se muestra en la figura 3.16, donde M, PF y PS son de nuevo 100 solución de esquina en una
dólares/semana, 10 dólares/lb y 5 dólares/yd2, respectivamente. El paquete más asequible es el in- elección entre dos bienes, un
dicado con A y se encuentra en el extremo superior de la restricción de presupuesto. En A la TMS caso en el cual el consumidor
es menor que el valor absoluto de la pendiente de la restricción de presupuesto. Para ilustrar esto, no consume uno de ellos.
suponga que la TMS en A = 0.25, lo cual significa que el consumidor está dispuesto a renunciar a
0.25 lb de alimento para obtener una yarda cuadrada más de vivienda. Pero al precio de mercado el
costo de oportunidad de una yarda cuadrada más de vivienda es 0.5 lb de alimento. El consumidor
incrementa su satisfacción renunciando continuamente a vivienda por más alimento hasta que ya no
es posible hacerlo. Aun cuando considera la vivienda como un artículo deseable, lo mejor que puede
hacer es gastar todo su ingreso en alimento. Los precios de mercado son tales que el consumidor
tendrá que renunciar a demasiado alimento para que valga la pena comprar una sola unidad de
vivienda.

FIGURA 3.16
Alimento (lb/semana) Una solución de esquina
Paquete más asequible Cuando la tms de alimento
por vivienda es siempre
A menor que la pendiente
10 de la restricción de
I3 presupuesto, lo mejor que
puede hacer el consumidor
I2 es gastar todo su ingreso
en alimento.
I1

Vivienda (yd 2/semana)


20

03_CHAPTER 3.indd 71 3/6/09 7:04:09 PM


72 CaPítulo 3 la eleCCión raCional del Consumidor

El mapa de indiferencia de la figura 3.16 satisface la propiedad de la tasa marginal de sustitución


decreciente —al moverse hacia la derecha sobre cualquiera de las curvas de indiferencia, la pendiente
se hace cada vez menor en términos absolutos—. Pero como en este caso desde un principio las pen-
dientes de las curvas de indiferencia son menores que la pendiente de la restricción de presupuesto,
nunca llegan a ser iguales.
Las curvas de indiferencia que no son fuertemente convexas son características de los bienes
sustituibles. Las soluciones de esquina se presentan con más frecuencia cuando se tienen tales bienes
y es casi seguro que ocurran cuando los bienes son sustitutos perfectos. (Vea el ejemplo 3.3.) Para
tales bienes la TMS no disminuye en absoluto, más bien es constante en todos los puntos. Cuando se
tienen sustitutos perfectos, las curvas de indiferencia son líneas rectas. Si éstas son más inclinadas que
la restricción de presupuesto, se obtiene una solución de esquina en el eje horizontal; si son menos
inclinadas, se obtiene una solución de esquina en el eje vertical.

EJEMPLO 3.3 A Mattingly le encantan las bebidas de cola con cafeína y gasta todo su presupuesto para
refrescos en Coca-Cola y Jolt cola; lo único que le preocupa es el contenido total de cafeína.
Si Jolt tiene el doble de cafeína que la Coca y si Jolt tiene un precio de 1 dólar/lata y la Coca
de 0.75 de dólar/lata, ¿cómo gastará Mattingly su presupuesto para refrescos, que es de 15
dólares/semana?
Para Mattingly Jolt y Coca-Cola son sustitutos perfectos, lo que significa que su curva de indiferencia
es lineal. En la figura 3.17 la línea superior es el conjunto de todas las combinaciones Coca-Jolt que
proporcionan la misma satisfacción que el paquete que consiste en cero latas de Jolt por día y 30
latas de Coca por día. Como cada lata de Jolt tiene el doble de cafeína que una lata de Coca, todos los
paquetes a lo largo de esta línea contienen la misma cantidad de cafeína. La primera línea gris, hacia
abajo, es la curva de indiferencia de los paquetes equivalentes al paquete (0, 20), y la segunda línea
gris, hacia abajo, es la curva de indiferencia que corresponde a (0, 10). A lo largo de estas curvas de
indiferencia, la tasa marginal de sustitución de Coca por Jolt es siempre 2/1, es decir, 2 latas de Coca
por cada lata de Jolt.

FIGURA 3.17
Equilibrio cuando Coca-Cola (latas/semana)
se tienen sustitutos
perfectos 30
aquí, la tms de Coca-Cola
por jolt es 2 en cualquier
punto. siempre que el
cociente de los precios 20 B
P J / P C sea menor que 2 el
resultado es una solución
de esquina en la que
el consumidor compra 10
únicamente jolt. en la
restricción de presupuesto
B, lo mejor que puede A
Jolt (latas/semana)
hacer el consumidor es 5 10 15
comprar el paquete A.

En este mismo diagrama, la restricción de presupuesto de Mattingly se indica como B. La pen-


diente de sus curvas de indiferencia es –2; la de su restricción de presupuesto es – _34. El paquete más
asequible es el que se indica como A, una solución de esquina en la que él gasta todo su presupuesto
en Jolt. Esto tiene sentido intuitivo a la luz de las preferencias particulares de Mattingly: a él sólo le
preocupa el contenido total de cafeína y Jolt contiene el doble de cafeína por dólar que Coca-Cola.
Si el cociente de los precios Jolt-Coca, PJ/PC hubiera sido _13 (o cualquier otra cantidad mayor que _12 ),
Mattingly hubiera gastado todo su ingreso en Coca-cola. Es decir, otra vez se hubiera tenido una
solución de esquina, sólo que esta vez en el eje vertical. Sólo si el cociente de los precios hubiera sido
exactamente _12, Mattingly hubiera gastado una parte de su presupuesto en cada bien. En ese caso,
cualquier combinación de Coca-Cola y Jolt en su restricción de presupuesto hubiera sido igual de
buena.

03_CHAPTER 3.indd 72 3/6/09 7:04:10 PM


una aPliCaCión del modelo de eleCCión raCional 73

La mayoría de las veces se encontrarán problemas que tienen soluciones interiores —es decir, pro-
blemas en los que el paquete más asequible se encontrará en un punto de tangencia—. Una solución
interior, una vez más, es una en la que la TMS es exactamente igual a la pendiente de la restricción
de presupuesto.

EJERCICIO 3.7
Suponga que Albert siempre coloca dos porciones de mantequilla en cada pan tostado. Si el
pan tostado se vende en 0.10 de dólar/rebanada y la mantequilla en 0.20 de dólar/porción,
encuentre el paquete más asequible para Albert si tiene 12 dólares/mes para gastar en pan
tostado y mantequilla. Suponga que Albert empieza a preocuparse por el colesterol y modifica
su preferencia y ahora usa exactamente una porción de mantequilla en cada rebanada de pan
tostado. ¿Cuánto pan tostado y cuánta mantequilla consumirá entonces Albert cada mes?

CURVAS DE INDIFERENCIA CUANDO HAY MÁS DE DOS BIENES


En los ejemplos vistos hasta ahora el consumidor tiene que elegir sólo entre dos bienes. Cuando hay
más de dos se pueden construir curvas de indiferencia empleando el mismo recurso que se usó antes
para representar restricciones de presupuesto con bienes múltiples. Simplemente se ve la elección
del consumidor como una elección entre un bien determinado X y un conjunto de otros bienes Y, a
la que una vez más se le llama el bien compuesto. Como antes, el bien compuesto es la cantidad de
ingreso que le queda al consumidor después de comprar el bien X.
En el caso de bienes múltiples, puede continuarse la representación de las preferencias del con-
sumidor mediante un mapa de indiferencia en el plano XY. En este caso, la curva de indiferencia
indica la tasa a la cual el consumidor intercambiará el bien compuesto por X. Como en el caso de
dos bienes, el equilibrio se presenta cuando el consumidor alcanza la curva de indiferencia más alta
posible sobre su restricción de presupuesto.

UNA APLICACIÓN DEL MODELO


DE ELECCIÓN RACIONAL
Como se muestra en el ejemplo siguiente, el bien compuesto permite ocuparse de cuestiones más
generales que las que permite el caso de sólo dos bienes.

¿Es preferible dar a las personas pobres dinero en efectivo o vales para alimentos? EJEMPLO 3.4
Un objetivo del programa de vales para alimentos es aliviar el hambre. De acuerdo con los términos
del programa, las personas cuyos ingresos son menores a una cierta cantidad tienen derecho a recibir
una determinada cantidad de vales. Por ejemplo, una persona que tenga un ingreso de 400 dóla-
res/mes tiene derecho a 100 dólares/mes en vales. Éstos pueden usarse entonces para comprar 100
dólares/mes de alimentos. Cualquier compra de alimentos que exceda esta cantidad deberá pagarse
en efectivo. Los vales no pueden usarse para comprar cigarros, alcohol u otros diversos artículos. El
gobierno paga en efectivo los vales que reciben las tiendas.
El costo que tiene el consumidor para el gobierno en el ejemplo que se dio es de 100 dólares
—cantidad que tiene que reembolsarle a la tienda por los vales—. ¿Se verá más favorecido el consu-
midor si se le dan los 100 dólares en efectivo?
Esta pregunta se puede responder si se analiza cuál será la alternativa que llevará al consumidor
a una curva de indiferencia más alta. Suponga que Y representa el bien compuesto y X denota los
alimentos. Si el ingreso del consumidor es 400 dólares/mes y PX es el precio de los alimentos, su
equilibrio inicial es el paquete J de la figura 3.18. El resultado del programa de los vales es incremen-
tar la cantidad total de alimento que puede comprar el consumidor al mes de 400 dólares/PX a 500

03_CHAPTER 3.indd 73 3/6/09 7:04:10 PM


74 CaPítulo 3 la eleCCión raCional del Consumidor

dólares/PX. En términos de la cantidad máxima de alimento que puede comprar, el programa de


vales es igual que un subsidio de 100 dólares en efectivo.
Donde estas dos alternativas difieren es en términos de la cantidad máxima de otros bienes
que puede comprar el consumidor. Con un subsidio en efectivo de 100 dólares, su ingreso mensual
total es 500 dólares y, por supuesto, ésta es la cantidad máxima de bienes que no sean alimentos (el
bien compuesto) que puede comprar. En este caso, su restricción de presupuesto es la línea marcada
como AE en la figura 3.18.

FIGURA 3.18
Programa de cupones Y ($/mes)
para alimento vs. E
$500
programa de subsidio
en efectivo
en comparación con la F D
$400
restricción de presupuesto
con un subsidio en efectivo
(AE), la restricción de
presupuesto en el caso K
Y*2
de los vales (ADF) limita
la cantidad que puede Y*1 I2
J
gastarse en bienes que no
sean alimentos. Pero en I1
el caso de un consumidor
cuyo mapa de indiferencia
sea el que se muestra en H A
X (unidades/mes)
la gráfica, los paquetes de X*1 X*2 $400/PX $500/PX
equilibrio son los mismos
con ambos programas.

En cambio, con el programa de vales, el consumidor no puede comprar 500 dólares/mes en


otros bienes porque sólo puede gastar sus 100 dólares en vales en alimentos. La cantidad máxima
que puede comprar de bienes que no sean alimentos es de 400 dólares. En la figura 3.18 la restricción
de presupuesto, en el caso del programa de vales, está indicada como ADF. Por lo tanto, para valo-
res de Y menores que 400 dólares, la restricción de presupuesto es exactamente igual a la que se tiene
en el caso del programa de un subsidio en efectivo. Sin embargo, para valores de Y mayores que 400
dólares la restricción de presupuesto con el programa de vales es completamente horizontal.
Observe que el consumidor cuyas curvas de indiferencia se presentan en la figura 3.18 compra
con cualquiera de los dos programas el mismo paquete: el K. Aquí el resultado del programa de vales
para alimentos es exactamente igual que el efecto del subsidio en efectivo. En general, esto ocurrirá
cada vez que el consumidor que reciba un subsidio en efectivo gaste en alimentos un valor mayor
que el de los vales que recibe.
En la figura 3.19 se representa un consumidor que no se encuentra en este supuesto. Con un
subsidio en efectivo, él habría elegido el paquete L, que lo colocaría en una curva de indiferencia
más alta que la que puede obtener con el programa de vales, el cual lo lleva a comprar el paquete D.
Observe que el paquete D contiene exactamente 100 dólares en alimentos, la cantidad que recibe en
vales. El paquete L, en cambio, contiene un equivalente en alimentos menor que 100 dólares. Aquí el
resultado del programa de vales es que el destinatario gasta más en alimentos de lo que gastaría si se
le hubiera dado dinero en efectivo.
El valor nominal de los vales que reciben la mayor parte de los participantes es menor que lo
que gastan en alimentos. Para estas personas el programa de vales para alimentos conduce, como se
hizo notar, a la misma conducta que tendrían con un programa de subsidio en efectivo.

El análisis del ejemplo 3.4 plantea la pregunta de por qué el gobierno no da a la gente pobre sólo
subsidios en efectivo. Al parecer, la razón es que el gobierno desea ayudar a la gente pobre a comprar
alimentos, no artículos de lujo o incluso cigarros y alcohol. Sin embargo, si la mayor parte de los par-

03_CHAPTER 3.indd 74 3/6/09 7:04:10 PM


una aPliCaCión del modelo de eleCCión raCional 75

FIGURA 3.19
Área ampliada Cuando los vales
para alimentos y los
subsidios en efectivo
dan resultados
diferentes
Y ($/mes) L Para el consumidor cuyo
E mapa de indiferencia se
$500 muestra en esta gráfica, un
subsidio en efectivo hubiera
L D I3
F sido preferible a los vales,
$400 I2 que lo obligan a gastar en
D I3
I2 alimentos más de lo que él
J hubiera deseado.
I1

H A
X (unidades/mes)
$100/PX $400/PX $500/PX

ticipantes en el programa de vales hubieran gastado en alimentos por lo menos tanto como recibe
en vales, no poder usar los vales para comprar otras cosas es una restricción sin sentido. Por ejemplo,
si una persona, en cualquier caso, hubiera gastado 150 dólares en alimentos, obtener 100 en vales
simplemente le permite gastar parte del dinero que hubiera empleado en alimentos en cualquier
otra cosa que ella decida.
De acuerdo con razones únicamente económicas, existe una fuerte razón para sustituir el pro-
grama de vales para alimentos por uno mucho más sencillo de subsidios en efectivo para los pobres.
Esto, por lo menos, eliminaría el engorroso paso de hacer que las tiendas canjeen sus vales por dinero
en efectivo. Sin embargo, desde el punto de vista político, es fácil ver por qué el gobierno hace así las
cosas. A muchos de los contribuyentes les dolería ver que sus impuestos se usan para comprar sus-
tancias ilícitas. Si bien el programa de vales para alimentos sólo evita que una muy pequeña minoría
de los participantes gaste más en tales bienes, sí ahorra muchos problemas políticos.
El ejemplo 3.4 llama la atención hacia un problema que no sólo tiene que ver con el programa
de estampillas para alimentos, sino también con todas las demás formas de transferencias en especie:
Aunque las dos formas de transferencia son algunas veces equivalentes, los donativos en efectivo
parecen ser claramente mejores en aquellas ocasiones en las que las dos difieren.

¿Por qué la gente suele dar regalos en especie en lugar de dar dinero en efectivo?
Ocasionalmente alguien recibirá un regalo similar a lo que hubiera comprado si se le hubiera dado
la cantidad equivalente de dinero. Sin embargo, todos conocen demasiado bien los obsequios poco EL NATURALISTA
acertados. ¿Quién no ha recibido una prenda de vestir que se avergonzaría de usar? La lógica del ECONÓMICO
modelo de la elección racional parece establecer de manera inequívoca que es posible ahorrarse el
problema de dar regalos inútiles si se emplea el sencillo recurso de dar dinero en efectivo; y sin em- 1.2
bargo, todas las sociedades continúan apegadas al ritual de dar obsequios.
El hecho de que esta costumbre persista no debe entenderse como una evidencia de que la gente
sea tonta. Más bien, sugiere que el modelo de la elección racional no logra captar algo importante
acerca de dar regalos. Uno de los propósitos de obsequiar es expresar afecto por el destinatario. Un
regalo bien elegido logra esto de una manera que el dinero en efectivo no puede. O es posible que al-
gunas personas tengan problema para darse pequeños lujos y se sientan obligadas a gastar en artícu-

03_CHAPTER 3.indd 75 3/6/09 7:04:11 PM


76 CaPítulo 3 la eleCCión raCional del Consumidor

© The New Yorker Collection 1992. Ed Franscino de cartoonbank.com.Todos los derechos reservados.
“Cariño, ¿no crees que este año no deberíamos hacer intercambio de regalos?”

los prácticos un regalo que se les ha dado en efectivo. Para estas personas, un regalo es una manera
de disfrutar de un pequeño lujo sin tener que sentirse culpables.7 Esta interpretación está respalda-
da por la observación de que rara vez se dan regalos puramente prácticos, como ropa interior cien-
to por ciento de algodón o detergente para lavar.
Sea cual sea la razón que tienen las personas para dar obsequios en especie y no en efectivo,
es acertado suponer que si no lo hacen no es porque no se les ocurra regalar dinero en efectivo. Al
contrario, hay ocasiones en las que se da un regalo en efectivo, en especial a parientes jóvenes con
bajos ingresos. Pero aun cuando tiene algunas ventajas regalar dinero en efectivo, la gente parece
rehusarse a abandonar la práctica de dar obsequios en especie.

En el apéndice de este capítulo se desarrolla la aplicación de la función utilitaria al problema de


elaborar el presupuesto del consumidor. Los temas que ahí se tratan son utilidad cardinal versus utili-
dad ordinal, la construcción algebraica de curvas de indiferencia y el uso del cálculo para maximizar
la utilidad.

REsuMEN
• El objetivo en este capítulo fue ampliar el modelo básico de un ingreso dado de antemano y que se encuentra ante precios
la elección racional del consumidor. En todas sus variantes, fijos. Los precios y el ingreso definen la restricción de presu-
este modelo da por sentadas las preferencias del consumidor puesto del consumidor, la cual es una recta con pendiente ne-
y supone que éste tratará de satisfacerlas de la manera más gativa. El valor absoluto de su pendiente es el cociente de los
eficiente posible. dos precios. Esta restricción de presupuesto es el conjunto de
• El primer paso para resolver el problema de la elaboración del todos los paquetes que el consumidor puede comprar si gasta
presupuesto es identificar el conjunto de paquetes de bienes todo su ingreso.
que el consumidor puede comprar. Se supone que éste tiene

7
Un análisis de esta interpretación se encuentra en R. Thaler, “Mental Accounting and Consumer Choice”, en Marketing Science, 4,
verano de 1985.

03_CHAPTER 3.indd 76 3/6/09 7:04:12 PM


Problemas 77

• El segundo paso para resolver el problema de la elaboración más tenga de un bien un consumidor, más habrá que darle
del presupuesto del consumidor es resumir las preferencias del para inducirlo a desprenderse de una unidad de otro bien. A
consumidor. En este caso se parte de un orden de preferencia la propiedad decreciente de la TMS se debe la forma convexa
mediante el cual el consumidor puede jerarquizar todos los característica de las curvas de indiferencia.
posibles paquetes de bienes. Se supone que esta clasificación • La restricción de presupuesto indica cuáles son las combinacio-
es completa y transitiva y que presenta la propiedad “más es nes de bienes que puede comprar el consumidor. Para resumir
mejor”. Órdenes de preferencia que satisfacen estas restric- las preferencias del consumidor entre diversos paquetes se usa
ciones dan lugar a los mapas de indiferencia o colecciones de un mapa de indiferencia en la mayoría de los casos, el paquete
curvas de indiferencia, cada una de las cuales representa com- más asequible se alcanza en un punto de tangencia entre una
binaciones de bienes que son indistintas para el consumidor. curva de indiferencia y una restricción de presupuesto. En ese
Se supone que los órdenes de preferencia presentan también punto la tasa marginal de sustitución es exactamente igual a la
una tasa marginal de sustitución decreciente, lo cual signifi- tasa a la que los productos pueden ser intercambiados uno por
ca que, a lo largo de cualquier curva de indiferencia, cuanto otro a los precios del mercado.

pREguNTAs DE REpAsO
1. Si los precios de todos los productos aumentan 20 por cien- 5. Explique con sus propias palabras cómo la pendiente de una
to por año y su jefe le da un aumento de 20 por ciento en el curva de indiferencia proporciona información acerca del por-
salario, ¿está usted económicamente mejor, peor o igual en centaje en que un consumidor prefiere un bien en lugar de otro.
comparación con su situación un año antes? 6. Explique por qué ocurre a menudo que un consumidor com-
2. Verdadero o falso: si se conoce la pendiente de la restricción de pra un paquete de bienes aun cuando prefiera otro.
presupuesto (con dos bienes), se conocen los precios de los 7. Verdadero o falso: si el mapa de las curvas de indiferencia es
dos bienes. Explique. cóncavo respecto al origen, entonces la canasta óptima debe
3. Verdadero o falso: que las curvas de indiferencia tengan pen- presentarse en un equilibrio de esquina, salvo, quizás, en el
diente negativa es consecuencia de que la tasa marginal de caso en el que haya descuentos por volumen.
sustitución TMS sea decreciente. 8. Si se le dan 10 dólares a Ralph, no gastará nada de este dinero
4. Elabore un ejemplo de un orden de preferencia para Coca- en atún. Pero cuando se le pregunta, dice que le da lo mismo
Cola, Diet Coke y Diet-Pepsi que viole el supuesto de transiti- recibir el equivalente a 10 dólares en atún que recibir un billete
vidad. de 10 dólares. ¿Cómo puede ser esto?

pRObLEMAs
1. La empresa Acme Seed ofrece las primeras 10 lb de semillas de caléndula que se compren por semana
a 2 dólares/lb y cada libra subsecuente a 1 dólar/lb. Si su ingreso es de 100 dólares/semana dibuje la
restricción de presupuesto para el bien compuesto y las semillas de caléndula.
2. Igual que el problema 1, excepto que ahora el precio de cada libra adicional después de 10 lb/semana
es de 4 dólares/lb.
3. A Smith le gustan más las nueces de la India que las almendras, y las almendras le gustan más que
las nueces. Le gustan igual las nueces pecanas y las nueces de macadamia, y prefiere las nueces de
macadamia a las almendras. Si se supone que sus preferencias son transitivas, ¿qué prefiere?:
a) ¿Las pecanas o las nueces?
b) ¿Las nueces de macadamia o las nueces de la India?
4. Al inicio PX es 120 dólares y PY es 80 dólares. Verdadero o falso: si PX aumenta a 18 dólares y PY a 12, la
nueva restricción de presupuesto se desplazará hacia adentro y paralela a la anterior línea de balance.
Explique.

03_CHAPTER 3.indd 77 3/6/09 7:04:12 PM


78 CaPítulo 3 la eleCCión raCional del Consumidor

5. Martha tiene cada semana 150 dólares para gastar y no puede pedir dinero prestado. Compra bolitas
de leche malteada y el bien compuesto. Suponga que el precio de las bolitas de leche malteada es de
2.50 dólares la bolsa y el del bien compuesto es de 1 dólar por unidad.
a) Represente gráficamente la restricción de presupuesto de Martha.
b) ¿Cuál es el costo de oportunidad, en términos de bolsas de bolitas de leche malteada, de una
unidad más del bien compuesto?
6. En el problema 5 suponga que en un periodo inflacionario el precio del bien compuesto aumenta a
1.50 dólares por unidad, pero el precio de las bolitas de leche malteada permanece igual.
a) Trace la nueva restricción de presupuesto.
b) ¿Cuál es el costo de oportunidad de una unidad más del bien compuesto?
7. En el problema 6 suponga que para combatir la inflación, Martha solicita un aumento de sueldo. Su
jefe le aumenta el sueldo a 225 dólares/semana.
a) Trace la nueva restricción de presupuesto.
b) ¿Cuál es el costo de oportunidad de una unidad más del bien compuesto?
8. Picabo, una agresiva esquiadora, gasta todo su ingreso en esquíes y fijadores de esquí. Gasta un par
de esquíes por cada par de fijadores que usa.
a) Grafique las curvas de indiferencia de los esquíes y de los fijadores.
b) Ahora trace las curvas de indiferencia dado el supuesto de que Picabo es una esquiadora tan agre-
siva que gasta dos pares de esquíes por cada par de fijadores.
9. En el problema 8 suponga que Picabo tiene 3 600 dólares por año para gastar en esquíes y fijadores.
Encuentre su paquete más asequible de esquíes y fijadores de acuerdo con las dos preferencias que se
describieron en el problema anterior. Los esquíes se venden en 480 dólares/par y los fijadores en 240
dólares/par.
10. Para Alexi, el café y el té son sustitutos perfectos: una taza de café equivale a una taza de té. Suponga
que Alexi tiene 90 dólares/mes para gastar en estas bebidas y que el café tiene un precio de 0.90 de
dólar/taza mientras que el té tiene un precio de 1.20 dólares/taza. Encuentre el paquete de té y café
más asequible para Alexi. ¿Cuánto puede subir el precio de una taza de café sin que se dañe el nivel de
vida de Alexi?
11. A Eva le gustan las manzanas pero las peras le son indiferentes. Si los dos únicos bienes de los que
puede disponer son peras y manzanas, trace sus curvas de indiferencia.
12. A Koop le gusta la comida pero le disgusta el humo de cigarro. Cuanto mayor es la cantidad de comida
que tiene, más dispuesto está a renunciar a comida para lograr una reducción en el humo del cigarro.
Si los únicos dos bienes son alimentos y humo de cigarro, trace las curvas de indiferencia de Koop.
13. Si usted fuera presidente de un grupo de protección del medio ambiente, ¿qué estructura tarifaria
preferiría que usara la empresa Gigawatt Power: la descrita en el ejemplo 3.1 o una en la que toda la
energía costará 0.08 de dólar/kWh? (Suponga que todas las estructuras tarifarias deben compensar
exactamente los gastos de la empresa.)
14. Paula ha sido artista y gasta todo su ingreso en el teatro y en el cine; el teatro le gusta tres veces más
que el cine.
a) Trace su mapa de indiferencia.
b) Paula gana 120 dólares/semana. Si cada boleto para el teatro se vende en 12 dólares, y cada boleto
para el cine en 4, trace su línea de presupuesto y su curva de indiferencia más alta posible. ¿Cuán-
tas obras de teatro verá Paula?
c) Si el precio de cada boleto de teatro es de 12 dólares y el de cada boleto de cine es de 5, ¿cuántas
obras de teatro verá Paula?
15. Para cada uno de los casos siguientes grafique:
a) Las curvas de indiferencia para una persona típica entre basura y el bien compuesto.
b) Las curvas de indiferencia entre los dos mismos bienes para Óscar, el gruñón de Plaza Sésamo, al
que le encanta la basura y no usa para nada el bien compuesto.
16. Boris tiene un presupuesto de 9 dólares/semana para su café con leche matutino. Pero le gusta prepa-
rado con cuatro partes de café y una de leche. El café tiene un precio de 1 dólar/onza y la leche 0.50
de dólar/onza. ¿Cuánto café y cuánta leche compra Boris por semana? ¿Cuáles serían sus respuestas
si el precio del café aumenta a 3.25 dólares/onza? Muestre sus respuestas gráficamente.
17. El gobierno federal quiere dar apoyo económico a la educación pero no a la religión. Con este fin,
otorga a la Universidad de Notre Dame 2 millones de dólares, estipulando que el dinero debe usar-

03_CHAPTER 3.indd 78 3/6/09 7:04:12 PM


Problemas 79

se sólo para propósitos seculares. En la siguiente gráfica se muestra la restricción de presupuesto y


la curva de indiferencia más alta posible entre gastos seculares y no seculares de la Universidad de
Notre Dame antes de la donación federal. ¿Cómo se modificaría el bienestar de la Universidad si la
donación no tuviera esta restricción?
Actividades no seculares ($M)
16

14

12

10

Actividades seculares ($M)


0 2 4 6 8 10 12 14 16 18

18. Continental Long Distance Telephone Service ofrece un paquete opcional para llamadas intraes-
tatales, en el que cada mes los primeros 50 minutos de llamadas dentro del estado son gratis, los
siguientes 100 minutos tienen un precio de 0.25 de dólar/min y cualquier tiempo adicional se cobra
a la tarifa normal de 0.50 de dólar/min. Trace la restricción de presupuesto para llamadas telefónicas
intraestatales y el bien compuesto de un suscriptor cuyo ingreso es de 400 dólares/mes.
19. Para el suscriptor de Continental Long Distance del problema 18, ¿cuál es el costo de oportunidad de
20 minutos más de llamadas si ha hecho
a) 40 minutos de llamadas cada mes?
b) 140 minutos de llamadas cada mes?
20. Usted tiene la oportunidad de rentar un automóvil a una tarifa de 40 dólares/día o a una tarifa de
200 dólares/semana. Trace su restricción de presupuesto correspondiente a un presupuesto de 360
dólares/viaje.
a) Encuentre su paquete más asequible si sus preferencias de viaje son tales que necesita exactamen-
te el equivalente a 140 dólares de otros bienes por cada día que rente el automóvil.
b) O bien, suponga que usted considera un día de renta del automóvil como un sustituto perfecto del
equivalente a 35 dólares de otros bienes.
21. Howard dijo que le daba lo mismo consumir cuatro rebanadas de pizza y una cerveza o consumir
tres rebanadas de pizza y dos cervezas. Dijo también que prefería un paquete que consistiera de una
rebanada de pizza y tres cervezas a cualquiera de los primeros dos paquetes. ¿Muestran las preferen-
cias de Howard tasas de sustitución marginal decrecientes?
22. Su empresa telefónica le ofrece los siguientes planes de pago:
Plan A: pagar 0.05 de dólar por llamada.
Plan B: pagar una tarifa inicial de 2 dólares/semana, que permite hacer hasta 30 llamadas por semana
sin cargo extra. Cualquier llamada adicional tiene un precio de 0.05 de dólar por llamada.
Si su ingreso es de 12 dólares/semana y el precio del bien compuesto es de 1, grafique su restricción
de presupuesto para el bien compuesto y las llamadas con cada uno de los planes.
*23. En el día de campo para recaudar fondos que organiza su escuela, los refrescos se pagan con boletos
comprados de antemano —un boleto por cada botella de refresco—. Los boletos se venden en pa-
quetes de tres tipos:
Pequeño: 3 dólares por 3 boletos
Mediano: 4 dólares por 5 boletos
Grande: 5 dólares por 8 boletos
Si usted cuenta con 12 dólares para gastar y no se pueden comprar fracciones de paquete, grafique su
restricción de presupuesto para los refrescos y el bien compuesto.

* Los problemas marcados con asterisco (*) son más difíciles.

03_CHAPTER 3.indd 79 3/6/09 7:04:13 PM


80 CaPítulo 3 la eleCCión raCional del Consumidor

*24. Considere dos restaurantes italianos localizados en pueblos idénticos a 200 millas de distancia uno de
otro. Ambos son iguales en todos los aspectos excepto en su política de propinas. En un restaurante
se cobra sólo 15 dólares por el servicio, pero no hay ningún otro tipo de propina. En el otro, a la
cuenta se le agrega 15 por ciento para propinas. En el primer restaurante la cuenta promedio, sin
incluir el pago por el servicio, es de 100 dólares. ¿Piensa usted que el consumo de alimentos en estos
dos restaurantes sea diferente? ¿Cómo piensa que sea esta diferencia?
*25. El señor R. Plane, un administrador universitario retirado, consume únicamente uvas y el bien com-
puesto Y (PY = 1 dólar). Sus ingresos consisten en 10 000 dólares/año de seguridad social, más las
ganancias de lo que vende de los 2 000 bushels de uvas que cosecha anualmente en su viñedo. El año
pasado el precio de las uvas fue de 2 dólares/bushel y el señor Plane consumió los 2 000 bushels de
uvas además de 10 000 unidades de Y. Este año el precio de las uvas es de 3 dólares/bushel; mientras
que PY sigue siendo 1 dólar. Si sus curvas de indiferencia tienen la forma convencional, ¿este año el
consumo de uvas será igual, menor o mayor que el del año pasado?

REspuEsTAs A LOs EjERciciOs DEL cApíTuLO


3.1 Alimentos (lb/semana)
Alimentos (lb/semana)

10

B1 B2
Pendiente = – –12

Pendiente = – –25

Vivienda(yd 2/semana)
0 20 25

3.2 Alimentos (lb/semana)


Alimentos (lb/semana)

10

B1

5 B2 Pendiente = – 14–

Pendiente = – –12

Vivienda (yd 2/semana)


0 20

3.3 Alimentos (lb/semana)

Alimentos (lb/semana)

10

B1
Pendiente = – 12–
5 B2
Pendiente = – –12

Vivienda (yd 2/semana)


0 10 20

* Los problemas marcados con asterisco (*) son más difíciles.

03_CHAPTER 3.indd 80 3/6/09 7:04:13 PM


resPuestas a los ejerCiCios del CaPítulo 81

3.4 Alimentos (lb/semana)

Alimentos (lb/semana)

12
10
B2
B1
Pendiente = – –12

Pendiente = – –12

Vivienda (yd 2/semana)


0 20 24

3.5 La restricción de presupuesto de un consumidor, cliente residencial de la empresa Gigawatt Power,


será quebrada hacia fuera, ya que la tarifa de los primeros 1 000 kWh/mes es más baja. Para un
consumo X de energía, hasta 1 000 kWh/mes, la restricción de presupuesto tiene la pendiente de la
tarifa más baja de 0.05 de dólar/kWh.
Y = 400 – 0.05X 0 ≤ X ≤ 1 000 kWh/mes
Para un consumo de energía X, superior a 1 000 kWh/mes, la restricción de presupuesto tiene la
pendiente de la tarifa más alta 0.10 de dólar/kWh.
Y = 450 – 0.10X X > 1 000 kWh/mes
La inflexión se presenta cuando X = 1 000 kWh/mes, en cuyo caso el nivel de consumo de otros
bienes es Y = 400 – 0.05 X = 400 – 50 = 350 o lo que es equivalente, Y = 450 – 0.10 X = 450 – 100 = 350.
En cambio, si la tarifa fuera de 0.10 de dólar kWh/mes para todos los kWh/mes más allá de los 1 000
kWh/mes, entonces la restricción de presupuesto para X > 1 000 kWh/mes sería
Y = 400 – 0.10X X > 1 000 kWh/mes
y tendría un salto discreto de Y = 350 a Y = 300 en X = 1 000 kWh/mes.

Y ($/mes)
450
400 Y = 400 – 0.05X
350
300

Y = 450 – 0.10X

Y = 400 – 0.10X

Electricidad (kWh/mes)
0 1 000 4 000 4 500

3.6 En el paquete A, el consumidor está dispuesto a renunciar a 1 lb de alimento para obtener una yarda
cuadrada más de vivienda. Pero a los precios del mercado sólo es necesario renunciar a –12 lb de ali-
mento para comprar una yarda cuadrada más de vivienda. Se concluye que el consumidor se verá
más beneficiado que con el paquete A si compra 1 lb menos de alimento y 2 yardas cuadradas más de
vivienda.

03_CHAPTER 3.indd 81 3/6/09 7:04:14 PM


82 CaPítulo 3 la eleCCión raCional del Consumidor

3.7 La restricción de presupuesto de Albert es T = 120 – 2B. Al inicio las preferencias de Albert son dos
porciones de mantequilla por cada rebanada de pan tostado B = 2T. Sustituyendo esta ecuación en su
restricción de presupuesto se obtiene T = 120 – 4T o 5T = 120, de donde se obtiene T = 24 rebanadas
de pan tostado y, por lo tanto, B = 48 porciones de mantequilla por mes. Las nuevas preferencias de
Albert son una porción de mantequilla por cada rebanada de pan tostado B = T. Al sustituir esta ecua-
ción en la restricción de presupuesto se obtiene T = 120 – 2T o 3T = 120, lo que da T = 40 rebanadas
de pan tostado y por lo tanto B = 40 porciones de mantequilla por mes. ¡No sólo redujo la grasa sino
que también consumirá más fibra!

Pan tostado (rebanadas/mes)


120
T = 120 – 2B

T=B

40
T = (1/2)B
24

Mantequilla (porciones/mes)
0 40 48 60

03_CHAPTER 3.indd 82 3/6/09 7:04:14 PM


A p é N D i c E

3
ApLICACIÓN DE LA fUNCIÓN
DE UtILIDAD AL pRObLEMA DEL
pRESUpUEStO DEL CONSUMIDOR

APLICACIÓN DE LA FUNCIÓN DE UTILIDAD


A LA ELECCIÓN DEL CONSUMIDOR
Hallar la curva de indiferencia más alta posible con la restricción de presupuesto es sólo una manera
que los economistas han encontrado para analizar el problema de la elección del consumidor. Para
muchas aplicaciones existe otro enfoque que también es útil. En éste se representan las preferencias
del consumidor no mediante un mapa de indiferencia sino mediante una función de utilidad.
Una función de utilidad asigna a cada paquete de bienes un número que representa la cantidad
de satisfacción que proporciona. Por ejemplo, suponga que Tom únicamente consume alimento y
vivienda y que su función de utilidad está dada por U(F, S) = FS, donde F denota la cantidad de lb
de alimento, S la cantidad de yardas cuadradas de vivienda que consume por semana y U su satis-
facción, medida en “útiles” por semana.1 Si F = 4 lb/semana y S = 3 yd2/semana, Tom obtendrá 12

1
El término “útiles”, o “útiles de satisfacción”, representa una unidad arbitraria. Como se verá, lo que importa en la elección del
consumidor no es la cantidad de útiles que proporcionen los diferentes paquetes, sino su jerarquía de acuerdo con las utilidades que
se les asignen.
83

03_CHAPTER 3.indd 83 3/6/09 7:04:14 PM


84 CaPítulo 3 aPéndiCe 3: aPliCaCión de la funCión de utilidad

útiles/semana de utilidad, que es lo mismo que obtendrá si consume 3 lb/semana de alimento y 4


yd2/semana de vivienda; en cambio si consume 8 lb/semana de alimento y 6 yd2/semana de vivienda
obtendrá 48 útiles/semana.
La función de utilidad se parece al mapa de indiferencia en que ambos proporcionan una des-
cripción completa de las preferencias del consumidor. En el marco de las curvas de indiferencia, dos
paquetes pueden jerarquizarse viendo cuál se encuentra en una curva de indiferencia más alta. En el
marco de las funciones de utilidad, dos paquetes pueden compararse viendo cuál da una mayor can-
tidad de útiles. En realidad, como ilustra el ejemplo siguiente, es sencillo usar la función de utilidad
para construir un mapa de indiferencia.

EJEMPLO A.3.1 Si la función de utilidad de Tom está dada por U(F, S) = FS grafique las curvas de indiferencia
que corresponden a 1, 2, 3 y 4 útiles, respectivamente.

En el lenguaje de las funciones de utilidad, una curva de indiferencia consiste en todas las combi-
naciones de F y S que proporcionan el mismo nivel de utilidad —el mismo número de útiles—. Su-
ponga que se observa la curva de indiferencia que corresponde a una unidad de utilidad; es decir, las
combinaciones de paquetes en las que FS = 1. Al despejar S de esta ecuación se tiene
1 (A.3.1)
S=
F′
que es la curva de indiferencia marcada en la figura A.3.1 como U = 1. La curva de indiferencia que
corresponde a 2 unidades de utilidad se genera despejando S de la ecuación FS = 2, con lo que se
obtiene S = 2/F, que se representa por la curva indicada como U = 2 en la figura A.3.1. De manera
similar, se generan las curvas de indiferencia correspondientes a U = 3 y U = 4, que se ven en el
diagrama. En general, la curva de indiferencia correspondiente a un nivel de utilidad U0 se encuentra
despejando S de la ecuación FS = U01/F con lo que se obtiene S = U0/F.

FIGURA A.3.1
Curvas de indiferencia S
para la función de
utilidad U = FS 4
Para obtener la curva
de indiferencia que
corresponde a todos los
paquetes que proporcionan 3
un nivel de utilidad U0 se
plantea FS = U0 y se despeja
S, con lo que se obtiene 2
S = U0/F.

1
U=4
U=3
U=2
U=1
F
1 2 3 4

En el marco de las curvas de indiferencia, el mejor paquete alcanzable es el que se ubica en la


restricción de presupuesto que se encuentra en la curva de indiferencia más alta. De manera aná-
loga, en el marco de las funciones de utilidad el mejor paquete alcanzable es el que se ubica en la
restricción de presupuesto que proporciona el nivel de utilidad más alto. En el marco de las curvas
de indiferencia, el mejor paquete alcanzable se encuentra en un punto de tangencia entre una cur-
va de indiferencia y la restricción de presupuesto. En el paquete óptimo, la pendiente de la curva

03_CHAPTER 3.indd 84 3/6/09 7:04:15 PM


aPliCaCión de la funCión de utilidad a la eleCCión del Consumidor 85

FIGURA A.3.2
S A lo largo de una
curva de indiferencia
MUS ΔS = MUF ΔF la utilidad permanece
K constante
ΔS L al pasar de K a L, la pérdida
ΔF de utilidad por tener
U = U0 menos vivienda, MUs∆S, es
compensada exactamente
por la ganancia de utilidad
F por tener más alimento,
MUF ∆F.

de indiferencia, es decir la TMS, es igual a la pendiente de la restricción de presupuesto. Suponga


que alimento y vivienda son otra vez los únicos bienes y que PF y PS son sus respectivos precios. Si
∆S/∆F denota la pendiente de la curva de indiferencia más alta alcanzable con el paquete óptimo,
la condición de tangencia dice que ∆S/∆F = PF/PS. ¿Cuál es la condición análoga en el marco de las
funciones de utilidad?
Para responder a esta pregunta es necesario introducir el concepto de utilidad marginal (la utili-
dad marginal de un bien es la tasa a la cual varía la utilidad total con el consumo de dicho bien), que
es la tasa a la cual varía la utilidad total a medida que varían las cantidades de alimento y vivienda.
Más concretamente, sean MUF la cantidad de útiles adicionales que se obtienen por cada unidad adi-
cional de alimento y MUS la cantidad de útiles adicionales que se obtienen por cada unidad adicional
de vivienda. Observe en la figura A.3.2 que el paquete K tiene ∆F unidades menos de alimento y ∆S
unidades más de vivienda que el paquete L. Por lo tanto, si se pasa del paquete K al paquete L, se ga-
nan MUF ∆F útiles por tener más alimento, pero se pierden MUS∆S útiles por tener menos vivienda.
Dado que tanto K como L se encuentran en la misma curva de indiferencia, se sabe que los
dos paquetes proporcionan el mismo nivel de utilidad. De manera que la utilidad que se pierde por
tener menos vivienda deberá ser compensada exactamente por la utilidad que se gana por tener más
alimento. Esto indica que
MUF ∆F = MUS ∆S (A.3.2)
Al multiplicar en forma cruzada los términos de la ecuación A.3.2 se obtiene
MUF ∆S
= (A.3.3)
MUS ∆F

Suponga que el paquete óptimo se encuentra entre K y L, las cuales se encuentran muy cerca una de
otra, de manera que tanto ∆F como ∆S son muy pequeñas. A medida que K y L se acercan al paquete
óptimo, el cociente ∆S/∆F se irá haciendo igual a la pendiente de la curva de indiferencia de ese
paquete, lo cual —de acuerdo con la ecuación A.3.3— indica que es igual al cociente de las utilidades
marginales de los dos bienes. Y como en el paquete óptimo la pendiente de la curva de indiferencia
es igual a la pendiente de la restricción de presupuesto, para el paquete óptimo debe satisfacerse
también la condición siguiente:
MUF P
= F (A.3.4)
MUS PS
La ecuación A.3.4 es, en el marco de las funciones de utilidad, la condición análoga a la condi-
ción TMS = PF/PS en el marco de las curvas de indiferencia.
Si se multiplican en forma cruzada los términos de la ecuación A.3.4 se obtiene una condición
equivalente que tiene una interpretación intuitiva muy sencilla
MUF MUS
= (A.3.5)
PF PS

03_CHAPTER 3.indd 85 3/6/09 7:04:15 PM


86 CaPítulo 3 aPéndiCe 3: aPliCaCión de la funCión de utilidad

Expresada en palabras, la ecuación A.3.5 dice que en el paquete óptimo el cociente de la utilidad
marginal entre el precio debe ser el mismo para todos los bienes. El ejemplo siguiente muestra por
qué debe satisfacerse esta condición cuando el consumidor ha distribuido su presupuesto de manera
óptima.

EJEMPLO A.3.2 Suponga que la utilidad marginal del último dólar que John gasta en alimentos es mayor que
la utilidad marginal del último dólar que gasta en vivienda. Por ejemplo, suponga que los
precios del alimento y de la vivienda son 1 dólar/lb y 2 dólares/yd2, respectivamente, y que las
utilidades marginales correspondientes son 6 y 4. Demuestre que es posible que John no está
maximizando su utilidad.

Si John compró 1 yd2 menos de hospedaje, habrá ahorrado 2 dólares/semana y habrá perdido 4
útiles. Pero esto le permitirá comprar 2 lb/semana más de comida, que le darán 12 útiles, lo cual
será una ganancia neta de 8 útiles.

Si se hace una abstracción del caso especial de las soluciones de esquina, una condición necesa-
ria para una distribución óptima del presupuesto es que el último dólar que se gaste en cada bien dé
el mismo incremento de utilidad.

EJEMPLO A.3.3 Mary recibe semanalmente 10 dólares y gasta todo en periódicos (N) y revistas (M), cuyos
precios respectivos son 1 y 2 dólares. La utilidad que obtiene de estas compras está dada
por U(N) + V(M). Si los valores de U(N) y V(M) son los que se muestran en la tabla, ¿Mary
está maximizando su utilidad si cada semana compra 4 revistas y 2 periódicos? Si no lo está
haciendo, ¿cómo debe distribuir su gasto semanal?

N U(N) M V(M)
0 0 0 0
1 12 1 20
2 20 2 32
3 26 3 40
4 30 4 44
5 32 5 46

Para que Mary maximice su utilidad, la utilidad extra por dólar debe ser la misma tanto para el últi-
mo periódico comprado como para la última revista adquirida. Pero como el segundo periódico pro-
porciona 8 útiles más por dólar gastado, que es el cuádruple de las 2 útiles por dólar que obtiene de la
cuarta revista (4 utilidades más a un precio de 2 dólares), Mary no está maximizando su utilidad.

N U(N) MU(N) MU(N)/PN M U(M) MU(M) MU(M)/PM


0 0 0 0
12 12 20 10
1 12 1 20
8 8 12 6
2 20 2 32
6 6 8 4
3 26 3 40
4 4 4 2
4 30 4 44
2 2 2 1
5 32 5 46

03_CHAPTER 3.indd 86 3/6/09 7:04:16 PM


utilidad Cardinal Contra utilidad ordinal 87

Para ver con claridad cómo Mary debe reasignar sus compras, se escribe de nuevo la tabla in-
cluyendo la información relevante sobre las utilidades marginales. En esta tabla se ve que hay varios
paquetes en los que MU(N)/PN = MU(M)/PM —a saber, 3 periódicos y 2 revistas, o 4 periódicos y 3
revistas, o 5 periódicos y 4 revistas—. El último paquete es el que da la utilidad total más alta, pero
tiene un precio de 13 dólares y por lo tanto queda fuera de la restricción de presupuesto de Mary. El
primero, que le significa sólo 7 dólares, sí es asequible, pero también el segundo, cuyo precio es de 10
dólares y proporciona una utilidad total mayor que el primero. Si compra 4 periódicos y 3 revistas,
Mary obtiene 4 útiles por dólar de su última compra en cada categoría. Su utilidad total es 70 útiles,
que supera por 6 a lo que obtiene con su paquete original.

Observe en el ejemplo A.3.3 que si todos los valores de utilidad de Mary se duplicaran o se redu-
jeran a la mitad, de todas maneras lo mejor que podría hacer es comprar 4 periódicos y 3 revistas por
semana. Esto ilustra que la elección del consumidor no depende del valor absoluto de las utilidades
asociadas a los distintos paquetes, sino de la jerarquía ordinal de la utilidad que se le asigne a cada
uno. Si se duplican todos los útiles que se asocian a los diferentes paquetes o se reducen a la mitad, la
jerarquía ordinal de los paquetes se preserva, con lo cual el paquete óptimo sigue siendo el mismo.
Esto sigue siendo así si se saca el logaritmo de la función de utilidad, su raíz cuadrada o se le suma 5 o
se le hace cualquier otra transformación que preserve la jerarquía ordinal de los paquetes.

UTILIDAD CARDINAL
CONTRA UTILIDAD ORDINAL
En el estudio acerca de cómo representar las preferencias del consumidor se supuso que la gente
es capaz de dar una jerarquía a cada uno de los paquetes en orden de preferencia. A esto se le llama
enfoque de la utilidad ordinal al problema de la elaboración del presupuesto del consumidor. Esto no
requiere que la gente diga de manera cuantitativa qué tanto le gustan los diversos paquetes. Lo que
se supone es que el consumidor siempre es capaz de decir si prefiere A o B, aunque puede que no sea
capaz de expresar algo así como “A es 6.43 veces mejor que B”.
En el siglo xix los economistas suponían que las personas eran capaces de hacer tales afirmacio-
nes. Hoy a esto se le llama el enfoque de la utilidad cardinal al problema de la elección del consumi-
dor. En el caso de dos bienes se supone que a la satisfacción proporcionada por cualquier paquete se
le puede asignar un valor numérico, o cardinal, mediante una función de utilidad de la forma

U = U(X, Y) (A.3.6)

donde X y Y son los dos bienes.


En tres dimensiones, la gráfica de una de estas funciones de utilidad se verá más o menos como
la que se muestra en la figura A.3.3. La figura recuerda a una montaña, pero debido al supuesto “más
es mejor”, se trata de una montaña sin cumbre. El valor en el eje U mide la altura de la montaña, la
cual sigue aumentando entre más se tiene de X o de Y.
Suponga que en la figura A.3.3 se fijara la utilidad en una determinada cantidad, por ejemplo, U0.
Es decir, que la montaña de utilidad se corta con un plano paralelo al eje XY, U0 unidades sobre él. La
línea indicada como JK en la figura A.3.3 representa la intersección de ese plano con la superficie de la
montaña de utilidad. Todos los paquetes de bienes que se encuentran en JK proporcionan utilidad U0.
Si la línea JK se proyecta hacia abajo, sobre el plano XY, se obtiene la curva del nivel de indiferencia U0
que se muestra en la figura A.3.4.
Suponga que después se interseca la montaña de utilidad con otro plano, esta vez U1 unidades
sobre el plano XY. En la figura A.3.3 este segundo plano interseca la montaña de utilidad a lo largo de
la línea etiquetada LN. Ésta representa el conjunto de todos los paquetes que confieren una utilidad
U1. Si se proyecta LN hacia abajo sobre el plano XY, se obtiene la curva de indiferencia indicada como

03_CHAPTER 3.indd 87 3/6/09 7:04:16 PM


88 CaPítulo 3 aPéndiCe 3: aPliCaCión de la funCión de utilidad

FIGURA A.3.3
Superficie U
tridimensional
de utilidad N
L

U1 K
J

X
U0

FIGURA A.3.4
Curvas de indiferencia
como proyecciones Y Proyección de JK

Proyección de LN

U = U1
U = U0
X

U1 en la figura A.3.4. De la misma manera, se puede generar todo un mapa de indiferencia correspon-
diente a la función de utilidad cardinal U(X, Y).
Por lo tanto, se ve que es posible partir de cualquier función de utilidad cardinal y terminar con
un mapa de indiferencia único. ¡Pero no es posible ir en la dirección contraria! Es decir, no es factible partir
de un mapa de indiferencia y llegar de manera retrospectiva a una única función de utilidad cardinal.
La razón es que existe una cantidad infinita de funciones de utilidad que dan lugar a exactamente el
mismo mapa de indiferencia.
Para ver por qué, imagine que se toma la función de utilidad de la ecuación A.3.4 y se duplica,
de manera que ahora la utilidad está dada por V = 2U(X, Y). Al graficar V como función de X y Y, la
forma de la montaña de utilidad que se obtiene será muy parecida a la de antes. Si se hace pasar un
plano a 2U0 unidades sobre el plano XY, intersecará la montaña de utilidad exactamente de la misma
manera que lo hizo el plano original a U0 unidades de altura. Si ahora la intersección obtenida se
proyecta hacia abajo sobre el plano XY, la proyección coincidirá de manera perfecta con la curva de
indiferencia original U0.

03_CHAPTER 3.indd 88 3/6/09 7:04:16 PM


generaCión de Curvas de indiferenCia en forma algebraiCa 89

Lo único que se hace cuando se multiplica (se divide, se le suma o se le resta a) una función de
utilidad cardinal es renombrar las curvas de indiferencia a las que da lugar. Es más, se puede hacer
una afirmación aún más general: si U(X, Y) es una función de utilidad marginal y V es cualquier fun-
ción creciente, entonces U = U(X, Y) y V = V[U(X, Y)] darán lugar a exactamente los mismos mapas
de indiferencia. La propiedad especial que tiene una función creciente es que preserva el orden jerár-
quico de los valores de la función original. Es decir, si U(X1, Y1) > U(X2, Y2), el hecho de que V sea una
función creciente asegura que V[U(X1, Y1)] será mayor que V[U(X2, Y2)]. Y mientras esta condición se
satisfaga, las dos funciones darán lugar a exactamente las mismas curvas de indiferencia.
El primero que analizó el concepto de mapa de indiferencia fue Francis Edgeworth, quien lo
obtuvo de una función de utilidad marginal de la manera que se describió antes. Fueron necesarias
las ideas de Vilfredo Pareto, Irving Fisher y John Hicks para establecer que el aparato de Edgeworth
no dependía de manera única de una función de utilidad cardinal de soporte. Como se ha visto, el
único aspecto que importa de las preferencias del consumidor en el problema estándar de asignación
del presupuesto es la forma y la ubicación de sus curvas de indiferencia. La elección del consumidor
resulta ser del todo independiente de las etiquetas que se les pongan a estas curvas de indiferencia, la
única condición es que curvas más altas correspondan a niveles más elevados de utilidad.
Los economistas modernos prefieren el enfoque ordinal porque se apoya en supuestos mucho
más débiles que el enfoque cardinal. Resulta mucho más razonable imaginar que la gente puede
asignar jerarquías a los diferentes paquetes que suponer que hace evaluaciones cuantitativas precisas
acerca de cuánta satisfacción le proporciona cada uno.

GENERACIÓN DE CURVAS DE INDIFERENCIA


EN FORMA ALGEBRAICA
Aun cuando se suponga que los consumidores sólo tienen jerarquías de preferencia ordinales, con
frecuencia es conveniente representarlas con un índice de utilidad marginal. La ventaja de este pro-
cedimiento es que proporciona una manera algebraica compacta de resumir la información implícita
en la representación gráfica de las preferencias, como se vio en el ejemplo A.3.1.
Considere otra ilustración, esta vez se trata de una función de utilidad que genera curvas de
indiferencia que son líneas rectas: U(X, Y) = ( –23 )X + 2Y. Los paquetes de X y Y que dan un nivel
de utilidad U0 se encuentran despejando Y de U(X, Y) = U0. Esta vez se obtiene Y = (U0/2) – ( –13 )X. En
la figura A.3.5 se muestran las curvas de indiferencia correspondientes a U = 1, U = 2 y U = 3. Obser-
ve que todas son lineales, lo que indica que esta particular función de utilidad describe un orden de
preferencia en el que X y Y son sustitutos perfectos.

FIGURA A.3.5
Curvas de indiferencia
Y
para la función de
2 2
utilidad U(X, Y) = ( – )X
+ 2Y 3
la curva de indiferencia que
1 corresponde a todos los
paquetes que dan un nivel
U=1 U=2 U=3 de utilidad de u0 está dada
X por Y = (U0/2) – ( 1– )X.
1 2 3 4 3

03_CHAPTER 3.indd 89 3/6/09 7:04:17 PM


90 CaPítulo 3 aPéndiCe 3: aPliCaCión de la funCión de utilidad

USO DEL CÁLCULO PARA MAXIMIZAR LA UTILIDAD


Los estudiantes que hayan tomado un curso de cálculo son capaces de resolver el problema de asig-
nación de presupuesto del consumidor sin recurrir a la geometría de los mapas de indiferencia. Sea
U(X, Y) una función de utilidad del consumidor y suponga que M, PX y PY denotan ingreso, el precio
de X y el precio de Y, respectivamente. El problema de asignación del consumidor puede expresarse
de manera formal como sigue:

Maximizar U ( X , Y ) sujeto a PX X + PY Y = M . (A.3.7)


X, Y

La aparición de los términos X y Y debajo de la expresión “maximizar” indica que éstas son las varia-
bles cuyos valores tiene que escoger el consumidor. Los valores de precio e ingreso de la restricción
de presupuesto están dados de antemano.

EL MÉTODO DE LOS MULTIPLICADORES DE LAGRANGE


Como se dijo antes, la función U(X, Y) no tiene ningún máximo; simplemente crece y crece a medida
que X o Y aumentan. El problema de maximización definido en la ecuación A.3.7 se conoce como
problema de maximización restringido, lo cual significa que se desea hallar los valores de X y Y con los
que se obtiene el mayor valor de U sujeto a la restricción de que el consumidor sólo gaste tanto como su
ingreso. Se examinarán dos enfoques de este problema.
Una manera de asegurar que la restricción de presupuesto se satisface es emplear el método de
los multiplicadores de Lagrange. En este método se empieza por transformar el problema de maximiza-
ción restringida de la ecuación A.3.7 en el siguiente problema de maximización no restringida:

Maximizar £ = U ( X , Y ) −λ( PX X + PY Y − M ). (A.3.8)


X, Y , λ

Al término λ se le llama multiplicador de Lagrange y su papel es asegurar que se satisfaga la restricción


de presupuesto. (Cómo lo hace quedará claro en un momento.) Las condiciones de primer orden
para hallar un máximo de £ se obtienen encontrando la primera derivada parcial de £ con respecto a
X, Y y λ e igualándolas a cero:

∂ £ ∂U
= −λPX = 0 (A.3.9)
∂X ∂X
∂ £ ∂U
= −λPY = 0 (A.3.10)
∂Y ∂Y

y
∂£
= M − PX X − PY Y = 0 (A.3.11)
∂λ

El siguiente paso es encontrar X, Y y λ resolviendo las ecuaciones A.3.9 a A.3.11. Encontrar X y Y es


lo único que en realidad interesa. El papel del valor de equilibrio de λ es garantizar que la restricción
de presupuesto se satisfaga. Observe en la ecuación A.3.11 que igualando a cero la primera derivada
parcial de £ con respecto a λ garantiza que se tenga ese resultado.
Para hallar los valores de X y Y que maximizan la función de utilidad se requiere una forma
funcional específica para la función de utilidad. En un momento se presentará un ejemplo que ilustra

03_CHAPTER 3.indd 90 3/6/09 7:04:17 PM


generaCión de Curvas de indiferenCia en forma algebraiCa 91

esto. Pero primero observe que se puede obtener una característica interesante de los valores ópti-
mos de X y Y si se divide la ecuación A.3.9 entre la ecuación A.3.10 para obtener

∂U ∂X λPX PX
= = . (A.3.12)
∂U ∂Y λPY PY

La ecuación A.3.12 es la función de utilidad análoga a la ecuación 3.3 del capítulo 3, lo que indica
que los valores óptimos de X y Y deben satisfacer TMS = PX/PY. En la ecuación A.3.12, a los términos
∂U/∂Y y ∂U/∂Y se les conoce como utilidad marginal de X y utilidad marginal de Y, respectivamente.
Expresado en palabras, la utilidad marginal de un bien es la utilidad adicional que se obtiene de
consumir una unidad adicional del bien. La ecuación A.3.12 dice que el cociente de estas utilidades
marginales es simplemente la tasa marginal de sustitución de Y por X.
Si se reordena la ecuación A.3.12 en la forma

∂U ∂X ∂U ∂Y (A.3.13)
= ,
PX PY

surge otra propiedad interesante de los valores óptimos de X y Y. Expresado en palabras, el lado
izquierdo de la ecuación A.3.13 puede interpretarse como la utilidad adicional obtenida del último
dólar gastado en X. La ecuación A.3.13 es entonces la deducción obtenida mediante el cálculo del
resultado que ya se demostró antes en la ecuación A.3.5.

Un ejemplo
Para ilustrar el método de Langrange, suponga que U(X, Y) = XY y que M = 40, PX = 4 y PY = 2.
Entonces, este problema de maximización no restringida se escribirá como

Maximizar £ = XY −λ( 4 X + 2Y − 40) (A.3.14)


X, Y , λ

Las condiciones de primer orden para hallar un máximo de £ están dadas por

∂ £ ∂( XY )
= − 4λ = Y − 4λ = 0 (A.3.15)
∂X ∂X
∂ £ ∂( XY )
= − 2λ = X − 2λ = 0 (A.3.16)
∂X ∂X

∂£
= 40 − 4 X − 2Y = 0 (A.3.17)
∂λ

Al dividir la ecuación A.3.15 entre la ecuación A.3.16 y despejando Y se obtiene Y = 2X; si se sustituye
este resultado en la ecuación A.3.17 y se despeja X se obtiene X = 5, lo que a su vez da Y = 2X = 10.
De manera que (5, 10) es el paquete que maximiza la utilidad.2

2
Se supone que también se satisfacen las condiciones de segundo orden para un máximo local.

03_CHAPTER 3.indd 91 3/6/09 7:04:18 PM


92 CaPítulo 3 aPéndiCe 3: aPliCaCión de la funCión de utilidad

UN MÉTODO ALTERNATIVO
Hay otra manera de garantizar que la restricción del presupuesto se satisfaga en la que interviene
una notación menos complicada que la del método de Lagrange. En este otro método simplemente
se despeja Y en la restricción del presupuesto y se sustituye este resultado siempre que aparezca Y en
la función de utilidad. Entonces la utilidad se convierte en una función sólo de X y se puede maximi-
zar si se obtiene su primera derivada respecto de X y se iguala a cero.3 El valor de X que resuelve esta
ecuación es el valor óptimo de X, el cual puede sustituirse en la restricción de presupuesto para hallar
el valor de Y.
Para ilustrar esto, suponga de nuevo que U(X, Y) = XY con M = 40, PX = 4 y PY = 2. Entonces la
restricción de presupuesto es 4X + 2Y = 40, en la que se despeja Y y se obtiene Y = 20 – 2X. Al sustituir
esta expresión en la función de utilidad se obtiene U(XY) = X(20 – 2X) = 20X – 2X2. Se calcula la
primera derivada de U respecto de X y se iguala el resultado a cero
dU
= 20 − 4 X = 0, (A.3.18)
dX
de donde X = 5. Si se sustituye ahora este valor de X en la restricción de presupuesto se encuentra
que el valor óptimo de Y es 10. De manera que otra vez el paquete óptimo es (5, 10), lo mismo que
se encontró empleando el método del Lagrange. Con estos valores óptimos de X y Y, el consumidor
obtendrá (5)(10) = 50 unidades de utilidad.
Los dos enfoques algebraicos del problema de asignación del presupuesto dan el mismo resulta-
do que el enfoque gráfico antes descrito. Observe en la figura A.3.6 que la curva de indiferencia U =
50 es tangente a la restricción de presupuesto en el paquete (5, 10).

FIGURA A.3.6
Paquete óptimo cuando
U = XY, Px = 4, Py = 2 y Y
M = 40.
20

15

10

B U = 50
X
5 10 15 20

UNA TÉCNICA DE SIMPLIFICACIÓN


Suponga que el problema de maximización restringida sea de la forma

Maximizar U ( X , Y ) sujeto a PX X + PY Y = M , (A.3.19)


X, Y

3
Aquí la condición de segundo orden para un máximo local es que d2U/dX2 < 0.

03_CHAPTER 3.indd 92 3/6/09 7:04:18 PM


Problemas 93

Si (X*, Y*) es el paquete óptimo de este problema de maximización, entonces, como se sabe, será el
paquete óptimo para la función de utilidad V[U(X, Y)], donde V es cualquier función creciente.4 Esta
propiedad suele permitir transformar un problema de maximización con cálculos complicados en
uno más simple. A manera de ilustración considere el ejemplo siguiente:

Maximizar X 1 3Y 2 3 ) sujeto a 4 X + 2Y = 24 (A.3.20)


X, Y
Observe primero lo que ocurre cuando se procede con la función de utilidad no transformada dada
en la ecuación A.3.20. Al despejar Y = 12 – 2X de la restricción de presupuesto y sustituir su valor en
la función de utilidad, se tiene U = X1/3(12 – 2X)2/3. En este caso, calcular dU/dX es un poco tedioso,
pero si se realiza con cuidado cada paso, se obtiene la siguiente condición de primer orden:

dU  1  −2 3  2
=   X (12 − 2 X ) 2 3 + X 1 3  (12 − 2 X )−1 3 (−2) = 0 (A.3.21)
dX  3   3 
de donde, luego de un también tedioso reordenamiento, se obtiene X = 2. Y de la restricción de
presupuesto se obtiene Y = 8.
Ahora suponga que la función de utilidad se transforma tomando su logaritmo:

1  2
V = ln U (X , Y ) = ln(X 1/3Y 2/3 ) =   lnX +   lnY . (A.3.22)
 3   3 
Como el logaritmo es una función creciente, cuando se maximiza V sujeta a la restricción de pre-
supuesto, se obtiene el mismo resultado que al emplear U. Aquí, la ventaja de la transformación
logarítmica es que la derivada de V es mucho más fácil de calcular que la derivada de U. De nuevo, de
la restricción de presupuesto se obtiene Y = 12 – 2X y se sustituye este resultado en V para obtener
() ( )
V = 13 lnX + 23 ln(12 − 2 x ) . Esta vez la condición de primer orden sigue casi sin ningún esfuerzo:
 2
1 2 
dV 3  3 
= − = 0, (A.3.23)
dX X 12 − 2 X

de donde con facilidad se encuentra que X = 2. Al sustituir X = 2 en la restricción de presupuesto, se


obtiene otra vez Y = 8.
La mejor transformación que se pueda hacer dependerá naturalmente de la función de utilidad
de que se parta. La transformación logarítmica simplifica en gran medida el problema en el ejemplo
anterior, pero puede no ser de utilidad con otras formas de U.

pRObLEMAs
1. Tom gasta todo su ingreso semanal, que es de 100 dólares, en dos bienes, X y Y. Su función de utilidad
está dada por U(X, Y) = XY. Si PX = 4 y PY = 10, ¿cuánto debe comprar de cada bien?
2. Igual que el problema 1, sólo que ahora la función de utilidad de Tom está dada por U(X, Y) =
X1/2Y1/2.
3. Observe la relación entre sus respuestas al problema 1 y al problema 2. ¿Qué explica esto?
4. Sue consume sólo dos bienes, alimento y vestido. La utilidad marginal del último dólar que gasta en
alimento es 12 y la utilidad marginal del último dólar que gasta en vestido es 9. El precio del alimento
es 1.20 dólares/unidad y el precio del vestido es 0.90 de dólar/unidad. ¿Está maximizando Sue su
utilidad?

4
De nuevo, una función creciente es una para la cual V(X1) > V(X2) siempre que X1 > X2.

03_CHAPTER 3.indd 93 3/6/09 7:04:19 PM


94 CaPítulo 3 aPéndiCe 3: aPliCaCión de la funCión de utilidad

5. Albert tiene un ingreso semanal de 17 dólares, todo lo cual gasta en CD usados (C) y en renta de pe-
lículas (M), cuyos precios respectivos son 4 y 3 dólares. La utilidad que obtiene de estas compras está
dada por U(C) + V(M). Si los valores de U(C) y V(M) son los que se muestran en la tabla, ¿maximiza
Albert su utilidad si cada semana compra 2 CD y renta 3 películas? Si no es así, ¿cómo debe reasignar
su presupuesto?

C U(C) M V(M)
0 0 0 0
1 12 1 21
2 20 2 33
3 24 3 39
4 28 4 42

03_CHAPTER 3.indd 94 3/6/09 7:04:19 PM


Capítulo

4
DEMANDA INDIVIDUAL
Y DEL MERCADO

U
na libra de sal se vende en 30 centavos de dólar en la tienda de abarrotes. Mi familia y yo
consumimos la misma cantidad de sal a este precio y lo haríamos aunque su precio fuera
de 5 centavos/libra o incluso 10 dólares/libra. Además, yo ingiero la misma cantidad de sal
que cuando terminé mis estudios, y entonces mi ingreso económico era menos de una décima parte
de lo que es ahora.
La sal es un caso inusual. Las cantidades que se compran de otros numerosos artículos son mu-
cho más sensibles a los precios y los ingresos. Algunas veces, por ejemplo, mi familia y yo pensamos
en irnos a pasar un año sabático en la ciudad de Nueva York, donde los precios de la vivienda son
más del cuádruple de lo que se pagan en Ithaca. Si algún día vamos allá, tendremos que vivir en un
departamento que tenga menos de la mitad de superficie que la casa que tenemos ahora.

VISTA PRELIMINAR DEL CAPÍTULO


Visto dentro del marco del modelo de elección racional, mi conducta con respecto a la compra de sal
y de vivienda es perfectamente entendible. En este capítulo se usarán las herramientas del capítulo
3 para entender mejor por qué, exactamente, en diversas decisiones de compra las respuestas a los
cambios en el ingreso y en el precio difieren tanto. En el capítulo 3 se vio cómo se ve afectada la res-
tricción de presupuesto por los cambios en los precios y los ingresos. Aquí se verá cómo los cambios
en la restricción de presupuesto afectan las decisiones de compra. En concreto, se usará el modelo de

95

04_CHAPTER 4.indd 95 3/6/09 8:22:19 PM


96 Capítulo 4 demanda individual y del merCado

la elección racional para generar una curva de demanda del consumidor individual de un producto y
para construir una relación que resuma de qué manera varía la demanda individual de acuerdo con
el ingreso.
Se verá cómo se puede descomponer el efecto total de la variación de precio en dos efectos
separados: 1) el de la sustitución, que muestra el cambio de la cantidad demandada debido a que una
variación de precio modifica el atractivo de los bienes sustitutos, y 2) el del ingreso, que muestra el
cambio de la cantidad demandada que resulta de la alteración en el poder de compra ocasionada a su
vez por la modificación de los precios.
Después se mostrará cómo se agregan las curvas de demanda de los individuos para obtener
una curva de demanda del mercado como un todo. Un concepto analítico central que se desarrollará
en este capítulo es la elasticidad-precio de la demanda, una medida de la sensibilidad de las decisiones
de compra ante pequeñas variaciones en el precio. Se considerará también la elasticidad-ingreso de
la demanda, una medida de la sensibilidad de las decisiones de compra ante pequeñas variaciones en
el ingreso. Y se verá que, en el caso de algunos bienes, la distribución del ingreso, y no sólo su valor
promedio, es un determinante importante de la demanda del mercado.
Un último concepto de elasticidad en este capítulo es la elasticidad-precio cruzada de la de-
manda, que es una medida de la sensibilidad de la cantidad demandada de un bien ante pequeñas
variaciones del precio de otro bien. La elasticidad-precio cruzada es el criterio mediante el cual se
clasifican pares de bienes como sustitutos o como complementos.
Estos términos analíticos proporcionan una comprensión más profunda de diversos comporta-
mientos de mercado, así como un poderoso fundamento de las decisiones racionales y de la política
económica.

EFECTOS DE LA VARIACIÓN
DEL PRECIO
LA CURVA PRECIO-CONSUMO
Recuerde que en el capítulo 2 se vio que la curva de demanda del mercado indica la cantidad de un
bien que el mercado como un todo desea comprar a los diferentes precios. Suponga que se desea
obtener la curva de demanda de un bien —por ejemplo, vivienda— no para el mercado, sino para
un solo consumidor. Si se mantienen constantes el ingreso, las preferencias y los precios de todos
los demás bienes, ¿cómo afectará la variación en el precio de la vivienda a la cantidad de ésta que
compra el consumidor? Para responder esta pregunta se parte del mapa de indiferencia del consu-
midor, representando la vivienda en el eje horizontal y el bien compuesto Y en el eje vertical. Su-
ponga que el ingreso del consumidor es 120 dólares/semana y el precio del bien compuesto es, de
nuevo, 1 dólar por unidad. La intersección vertical de su restricción de presupuesto será entonces
120. La intersección horizontal será 120/PS, donde PS representa el precio de la vivienda. La figura
4.1 muestra cuatro restricciones de presupuesto que corresponden a cuatro precios distintos de la
vivienda en dólares: 24/yd2, 12/yd2, 6/yd2 y 4/yd2. Los correspondientes paquetes más asequibles
contienen 2.5, 7, 15 y 20 yd2/semana de vivienda, respectivamente. Si se repitiera este procedi-
miento con una cantidad infinita de precios, los puntos de tangencia que se obtuvieran describirían
curva precio-consumo la línea que se indica como CPC en la figura 4.1. Esta línea es la curva precio-consumo o CPC.
(CPC) si se mantiene En el caso del consumidor cuyo mapa de indiferencia se muestra en la figura 4.1, observe que
constante el ingreso y el precio cada vez que baja el precio de la vivienda, la restricción de presupuesto rota hacia afuera, permitien-
de Y, la CpC de un bien X es el do que el consumidor compre no sólo más vivienda, sino también más del bien compuesto. Y cada
conjunto de paquetes óptimos
vez que baja el precio de la vivienda, el consumidor opta por un paquete que contiene más vivienda
en un mapa de indiferencia
trazado a medida que varía el
que el paquete elegido previamente. Sin embargo, observe que cuando el precio de la vivienda baja,
precio de X. la cantidad de dinero que gasta en el bien compuesto puede aumentar o disminuir. Así, la cantidad
que gasta en otros bienes disminuye cuando el precio de la vivienda baja de 24/yd2 a 12/yd2, pero
aumenta cuando el precio de la vivienda baja de 6/yd2 a 4/yd2. Más adelante se verá por qué éste es
un patrón de compra relativamente común.

04_CHAPTER 4.indd 96 3/6/09 8:22:19 PM


efeCtos de la variaCión del preCio 97

Figura 4.1
Y ($/semana) Curva precio-consumo
si se mantienen constantes
120 el ingreso y el precio
de Y varía el precio del
100 hospedaje. el conjunto de
paquetes óptimos definidos
80 por las diferentes líneas de
presupuesto es la curva
60
precio-consumo o CpC.
CPC
36 40 I4
30
20 I3
I1 I2 Vivienda
(yd2/semana)
2.5 5 7 10 15 20 25 30

CURVA DE DEMANDA INDIVIDUAL DEL CONSUMIDOR


La curva de demanda individual del consumidor se parece a la curva de demanda del mercado en la
cual se indican las cantidades que el consumidor desea comprar a los diferentes precios. Toda la in-
formación que se necesita para construir la curva de demanda individual se encuentra en la curva
precio-consumo. El primer paso para pasar de la curva precio-consumo a la de demanda individual
es anotar, como en la tabla 3.1, las combinaciones relevantes precio-cantidad que aparecen en la CPC
de la figura 4.1. (Recuerde que en el capítulo 3 se vio que el precio de la vivienda a lo largo de alguna
restricción de presupuesto está dado por el ingreso dividido entre la intersección horizontal de di-
cha restricción de presupuesto.)

TABLA 4.1
Tabla de demanda
para obtener la
curva de demanda
Precio de la vivienda Cantidad de vivienda demandada
individual para la
($/yd2) (yd2/semana)
vivienda a partir
24 2.5 de la CpC de
12 7 la figura 4.1 se
empieza por anotar
6 15 las cantidades
4 20 de vivienda que
corresponden a
los precios de la
vivienda en cada
restricción de
presupuesto.

El paso siguiente es graficar los pares precio-cantidad de la tabla 4.1, con el precio de la vivienda
en el eje vertical y la cantidad de vivienda en el eje horizontal. Con una cantidad suficiente de pares
precio-cantidad, se genera la curva de demanda individual que se indica como DD en la figura 4.2.
Observe con cuidado que, al moverse de la CPC a la curva de demanda individual, se pasa de una
gráfica en la cual ambos ejes miden cantidades a una en la cual se representa el precio frente a la
cantidad.

04_CHAPTER 4.indd 97 3/6/09 8:22:20 PM


98 Capítulo 4 demanda individual y del merCado

FIGURA 4.2 Vivienda (yd2/semana)


Una curva de
D
demanda individual del
consumidor 24
Como la curva de demanda
del mercado, la curva de
demanda individual es 20
una relación que indica
cuánto desea comprar el
consumidor a los diferentes 15
precios.
12
10

6
5
4 D

Precio($/yd2)
2.5 5 7 10 15 20

EFECTOS DE LA VARIACIÓN DEL INGRESO


CURVA CONSUMO-INGRESO
La CPC y la tabla de demanda individual son dos maneras de resumir cómo responden las decisiones
de compra del consumidor a las variaciones de los precios. Para resumir las respuestas a las varia-
curva consumo-ingreso (CCI) ciones del ingreso existen mecanismos análogos. El análogo del ingreso a la CPC es la curva consu-
si se mantienen constantes mo-ingreso, o CCI. Para generar CPC de la vivienda se mantuvieron constantes las preferencias, el
los precios de X y Y, la CCi ingreso y el precio del bien compuesto y se identificaron los efectos de la variación del precio de la
de un bien X es el conjunto vivienda. En el caso de la CCI, las preferencias y los precios relativos se mantienen constantes y se
de paquetes óptimos trazados identifican los efectos de la variación del ingreso.
en un mapa de indiferencia a
En la figura 4.3, por ejemplo, se mantienen constantes el precio del bien compuesto, en 1 dólar
medida que varía el ingreso.
por unidad y el precio de la vivienda en 10 dólares/yd2, y se examina lo que ocurre cuando el ingreso
toma los valores en dólares de 40/semana, 60/semana, 100/semana y 120/semana. Recuerde que en
el capítulo 3 se vio que una variación del ingreso hace que la restricción de presupuesto se desplace
paralela a sí misma. Como antes, a cada presupuesto le corresponde un paquete más asequible. En

FIGURA 4.3
Una curva consumo- El bien compuesto ($/semana)
ingreso
a medida que aumenta
el ingreso, la restricción 120 CCI
de presupuesto se
100
desplaza hacia afuera. si
se mantienen constantes
las preferencias y los 80
precios relativos, la CCi
describe cómo afectan al 60
consumo estas variaciones 50
del ingreso. la CCi es el 40
conjunto de las tangencias 30
a medida que la línea de 20
presupuesto se desplaza
hacia afuera. Vivienda (yd2/semana)
0 2 3 4 5 6 8 10 12

04_CHAPTER 4.indd 98 3/6/09 8:22:21 PM


efeCtos de la variaCión del ingreso 99

la figura 4.3 el conjunto de paquetes más asequibles se denota como CCI. En el caso del consumidor
cuyo mapa de indiferencia se muestra, la CCI resulta ser una línea recta, pero éste no tiene que ser
siempre el caso.

CURVA DE ENGEL
El análogo de la curva de demanda individual en el dominio del ingreso es la curva de Engel indivi- curva de Engel grafica la
dual. Ésta toma de la CCI las cantidades de vivienda demandadas y las grafica contra los valores de relación entre la cantidad que
ingreso correspondientes. La tabla 4.2 muestra los pares ingreso-hospedaje de las cuatro restriccio- se consume de X y el ingreso.
nes de presupuesto de la figura 4.3. Si se graficara una cantidad infinita de pares ingreso-consumo
correspondientes al consumidor de la figura 4.3, se obtendría la línea EE que aparece en la figura 4.4.
La curva de Engel que aparece en la figura 4.4 es lineal, pero en general no es necesario que sea así.

TABLA 4.2
Ingreso y cantidad de vivienda demandada

Ingreso Cantidad de vivienda demandada


($/semana) (yd2/semana)
40 2
60 3
100 5
120 6

FIGURA 4.4
Ingreso ($/semana) Curva de Engel Una curva de Engel del
E consumidor individual
120 si se mantienen constantes
las preferencias y los
100 precios relativos, la curva
de engel dice cuánta
80 vivienda comprará el
consumidor en diversos
60 niveles de ingreso.

40

20
E
Vivienda (yd2/semana)
0 2 3 4 5 6 8 10 12

Observe con cuidado la diferencia entre lo que se mide en el eje vertical de la CCI y lo que se
mide en el eje vertical de la curva de Engel. En el eje vertical de la CCI se mide la cantidad que el
consumidor gasta cada semana en todos los demás bienes distintos al hospedaje. En cambio, en el eje
vertical de la curva de Engel se mide el ingreso semanal total del consumidor.
Observe también que, como ocurrió con la CPC y con las curvas de demanda individuales, la
CCI y las curvas de Engel contienen en esencia la misma información. La ventaja de la curva de
Engel es que permite observar de un vistazo cómo varía la cantidad demandada con el ingreso.

04_CHAPTER 4.indd 99 3/6/09 8:22:22 PM


100 Capítulo 4 demanda individual y del merCado

FIGURA 4.5
Curvas de Engel para
bienes inferiores y
M($/semana) M($/semana)
normales E E
a) Ésta es una curva de engel
para un bien normal. la
cantidad demandada aumenta
cuando se incrementa el
ingreso. b) esta curva de
engel para hamburguesa
tiene la pendiente negativa E
característica de los bienes E
inferiores. a medida que Filete Hamburguesa
el ingreso del consumidor
aumenta, el consumidor cambia a) b)
las hamburguesas por filetes
más apetitosos.

BIENES NORMALES E INFERIORES


Observe que la curva de Engel de la figura 4.5a tiene pendiente positiva, lo cual significa que cuanto
mayor sea el ingreso de un consumidor, comprará más filetes por semana. La mayoría de las cosas que
bien normal un bien cuya se compran tienen esta propiedad, que es la característica que define a un bien normal. Los bienes
cantidad demandada aumenta que no tienen esta propiedad se llaman bienes inferiores. En el caso de éstos, el aumento del ingreso
a medida que se incrementa el lleva a una disminución de la cantidad demandada. La figura 4.5b es un ejemplo de una curva de
ingreso. Engel para un bien inferior. Cuanto mayor sea el ingreso de una persona, comprará menos hambur-
guesas cada semana.
¿Por qué algunas personas compran menos de un bien cuando aumenta su ingreso? El prototipo
bien inferior un bien cuya de un bien inferior es aquel que tiene varios sustitutos fuertemente preferidos pero más caros. Por
cantidad demandada disminuye ejemplo, los supermercados tienen varios tipos de carne molida, que van desde la hamburguesa, con
a medida que aumenta el un alto contenido de grasa, hasta el solomillo molido, que tiene contenido de grasa más bajo. Un
ingreso. consumidor que trata de restringir la ingesta de grasa en su dieta cambiará en cuanto pueda a una
carne más magra. Para este consumidor la hamburguesa es un bien inferior.
Para cualquier consumidor que gasta todo su ingreso, es una cuestión de simple aritmética el
que no todos los bienes pueden ser inferiores. Después de todo, cuando aumenta el ingreso, es mate-
máticamente imposible gastar menos en todos los bienes al mismo tiempo. Se concluye que cuanto
más amplia sea la definición de un bien, menos probable es que sea inferior. Así, mientras las ham-
burguesas son un bien inferior para muchos consumidores, la “carne” probablemente sea un bien
inferior para unas cuantas personas, y todavía para menos será el “alimento” un bien inferior.1

EFECTOS DE UNA VARIACIÓN DE PRECIO


SOBRE EL INGRESO Y LA SUSTITUCIÓN
En el capítulo 2 se vio que la variación del precio de un bien afecta las decisiones de compra por dos
razones. Considere los efectos de un aumento de precio. (Los efectos de una reducción del precio
irán en dirección contraria.) Cuando aumenta el precio de un bien, los sustitutos cercanos se vuelven
más atractivos que antes. Por ejemplo, si aumenta el precio del arroz, el trigo se vuelve más atractivo.
efecto sustitución aquel Éste es el llamado efecto sustitución de un aumento de precio.
componente del efecto total El segundo efecto de un aumento de precio es que reduce el poder de compra del consumidor. En
provocado por la variación del el caso de un bien normal, este efecto reducirá además la cantidad comprada. Pero en el caso de bie-
precio que resulta del cambio
en el precio relativo de otros
bienes. 1
Otra manera de distribuir el conjunto de los bienes de consumo es entre indispensables y de lujo. Un bien se define como un lujo
para una persona si, cuando aumenta su ingreso, gasta una gran parte de éste en ese bien. Un bien indispensable, en cambio, es uno
en el que gasta una pequeña cantidad cuando aumenta su ingreso. (Más adelante se profundiza en esta distinción.)

04_CHAPTER 4.indd 100 3/6/09 8:22:24 PM


efeCtos de una variaCión de preCio sobre el ingreso y sobre la sustituCión 101

nes inferiores, este efecto es precisamente el contrario. La pérdida de poder de compra, por sí misma,
incrementa la cantidad comprada de un bien inferior. A la variación de la cantidad comprada atribui-
ble al cambio del poder de compra se le conoce como efecto de ingreso de la variación de precio. Efecto ingreso aquel
El efecto total de un aumento de precio es la suma de los efectos sustitución e ingreso. El efecto componente del efecto total
sustitución siempre ocasiona que la cantidad comprada se desplace en dirección opuesta a la varia- provocado por una variación
ción de precio —cuando el precio sube, la cantidad demandada baja y viceversa—. La dirección del de precio que resulta del
efecto ingreso depende de si se trata de un bien normal o inferior. En el caso de bienes normales, cambio en el poder real de
compra.
el efecto ingreso va en la misma dirección que el efecto sustitución —cuando el precio sube [baja], la
caída [aumento] del poder de compra hace que la cantidad demandada baje [suba]—. En cambio, en
el caso de bienes inferiores, los efectos ingreso y sustitución van en direcciones opuestas.
Los efectos sustitución, ingreso y total de un aumento de precio pueden verse con más clari-
dad cuando se representan gráficamente. Para empezar se muestra el efecto total. En la figura 4.6
el ingreso inicial del consumidor es de 120 dólares/semana y el precio inicial de la vivienda es 6
dólares/yd2. Esto da lugar a una restricción de presupuesto que se indica como B0, a lo largo de la
cual el paquete óptimo es A, que contiene 10 yd2/semana de vivienda. Ahora el precio de la vivienda
aumenta de 6 dólares/yd2 a 24 dólares/yd2 y pasa a la restricción de presupuesto indicado como B1.
El nuevo paquete óptimo es D, que contiene 2 yd2/semana de vivienda. El cambio de A a D se llama
efecto total del aumento de precio. Naturalmente, el aumento de precio ocasiona que el consumidor
termine en una curva de indiferencia más baja (I1) que la que era capaz de alcanzar con su presupues-
to original (I0).

FIGURA 4.6
Y($/semana) Efecto total de un
aumento de precio
si se tiene un ingreso 120
120 dólares/semana y el precio
de la vivienda es de 6
dólares/yd2, el consumidor
elige el paquete A de la
D restricción de presupuesto
72 A
60 B0. Cuando el precio de
la vivienda aumenta a 24
I0 dólares/yd2 y el ingreso
B1 B0 permanece constante en
I1 120 dólares/semana, el
Vivienda (yd2/semana) paquete más asequible es
0 2 5 10 15 20 D. al cambio de 10 a 2
Efecto yd2/semana de vivienda se
total le llama efecto total del
aumento de precio.

Para descomponer el efecto total en los efectos ingreso y sustitución, se empieza por hacer la
pregunta siguiente: ¿cuánto ingreso necesitará el consumidor para alcanzar su curva de indiferen-
cia original (I0) después del incremento en el precio de la vivienda? Observe en la figura 4.7 que la
respuesta es 240 dólares/semana. Si al consumidor se le diera esta cantidad como ingreso, se le re-
sarciría del daño causado por la pérdida del poder de compra como resultado del aumento de precio
de la vivienda. La restricción de presupuesto indicada como B' es hipotética, un recurso construido
para el presente propósito. Ésta tiene la misma pendiente que la nueva restricción de presupuesto
(B1) —es decir, –24 — y se encuentra lo suficientemente hacia afuera para ser tangente a la curva de
indiferencia original, I0. Con la restricción de presupuesto B', el paquete óptimo es C, que contiene 6
yd2/semana de vivienda. El paso de A a C da lugar al efecto sustitución de la variación de precio —en
este caso, una disminución de 4 yd2/semana de vivienda y un aumento de 36 unidades/semana del
bien compuesto—.

04_CHAPTER 4.indd 101 3/6/09 8:22:25 PM


102 Capítulo 4 demanda individual y del merCado

FIGURA 4.7
Efectos sustitución
e ingreso de una
variación de precio Y($/semana)
para obtener el efecto
sustitución, el nuevo 240
presupuesto B1 se desplaza
hacia afuera paralelamente
a sí mismo hasta ser
tangente a la curva de B'
indiferencia original, I0. el
movimiento de A a C da
lugar al efecto sustitución,
120
la reducción de la vivienda
debida sólo al hecho de 96 C
que la vivienda es ahora
72 D A
más cara en relación 60
con los demás bienes. el I0
movimiento de C a D da
lugar al efecto ingreso, B1 B0
que es la disminución de I1
Vivienda(yd2/semana)
la vivienda resultante de 0 2 56 10 15 20
la pérdida de poder de Efecto
compra implícita en el total
aumento de precio.
Efecto Efecto
ingreso sustitución

La restricción hipotética de presupuesto B' indica que aun cuando el consumidor tuviera in-
greso suficiente para alcanzar la misma curva de indiferencia que antes, el aumento del precio de
la vivienda lo haría reducir su consumo en favor de otros bienes y servicios. Para los consumidores
cuya curva de indiferencia tenga la forma convexa convencional, el efecto sustitución de un aumento de precio
reducirá siempre el consumo del bien cuyo precio se incrementa.
El efecto ingreso se origina por el paso de C a D. El bien particular que se muestra en la figura
4.7 es casualmente un bien normal. El cambio hipotético del ingreso del consumidor de 240 dólares/
semana a 120/semana acentúa la reducción de su consumo de vivienda, ocasionando que baje de 6
yd2/semana a 2 yd2/semana.
Mientras el efecto ingreso refuerza el efecto sustitución tratándose de bienes normales, en el
caso de bienes inferiores los dos efectos tienden a contrarrestarse uno al otro. En la figura 4.8, B0 re-
presenta la restricción de presupuesto de un consumidor que tiene un ingreso de 24 dólares/semana
ante un precio de la hamburguesa de 1 dólar/lb. En B0 el paquete más asequible es A, que contiene
12 lb/semana. Cuando el precio de la hamburguesa sube a 2 dólares/lb, la restricción de presupuesto
que resulta es B1 y el paquete más asequible ahora es D, que contiene 9 lb/semana de hamburguesa.
El efecto total del aumento de precio es, entonces, reducir el consumo de hamburguesa en 3 lb/se-
mana. La restricción de presupuesto B' es, de nuevo, la restricción hipotética de presupuesto que
le permite al consumidor alcanzar la curva de indiferencia original a la nueva relación de precio.
Observe que el efecto sustitución (la variación del consumo de hamburguesa relacionada con el paso
de A a C en la figura 4.8) es reducir la cantidad de hamburguesa que se consume en 4 lb/semana —es
decir, disminuirla en una cantidad mayor al valor del efecto total—. El efecto ingreso por sí mismo
(la variación del consumo de hamburguesa relacionado con el paso de C a D) en realidad aumenta
el consumo de hamburguesa en lb/semana. El efecto ingreso, por lo tanto, funciona en dirección
opuesta al efecto sustitución en el caso de bienes inferiores como la hamburguesa.

04_CHAPTER 4.indd 102 3/6/09 8:22:26 PM


efeCtos de una variaCión de preCio sobre el ingreso y sobre la sustituCión 103

FIGURA 4.8
Y($/semana) Efectos ingreso y
sustitución para un bien
inferior
34 a diferencia del caso
de un bien normal, el
B’ efecto ingreso actúa
contrarrestando el efecto
sustitución para un bien
24
inferior.
C
18
B1

12 A

6 D I0
B0

I1
Hamburguesa (lb/semana)
0 8 9 12 15 18 24
Efecto 17
total
Efecto ingreso
Efecto sustitución

BIENES GIFFEN
Un bien Giffen es aquel para el cual el efecto total de un aumento de precio es incrementar, no Bien Giffen un bien para el
reducir, la cantidad comprada. Como el efecto sustitución de un incremento del precio es redu- que la cantidad demandada
cir siempre la cantidad comprada, el bien Giffen debe ser uno cuyo efecto ingreso contrarreste el aumenta a medida
efecto sustitución. Es decir, debe ser un bien inferior —de hecho, tan fuertemente inferior, que el efecto que aumenta el precio
ingreso es mayor que el efecto sustitución—.
Un ejemplo de bien Giffen muy citado es el de la papa durante la hambruna irlandesa del siglo
xix. La idea es que las papas constituían una parte tan importante de la dieta de la gente pobre, que
cualquier aumento en su precio tenía un severo efecto adverso sobre el poder adquisitivo. Al tener
un ingreso real menor, muchas familias disminuían el consumo de carne y de otros alimentos más
caros, y compraban aún más papas. (Vea la figura 4.9.) O más o menos así va la historia.
Los historiadores modernos discuten si la papa ha sido realmente un bien Giffen. Sea cual sea
el resultado de la controversia, la historia de la papa ilustra las características que, por lógica, debe

FIGURA 4.9
Curva de demanda para
Precio de las papas un bien Giffen
($/lb) si un bien es tan
fuertemente inferior que
D el efecto ingreso de un
aumento de precio domina
sobre el efecto sustitución,
su curva de demanda
tendrá pendiente positiva.
D los bienes giffen son una
Papas ($/semana) posibilidad teórica, pero
son tan raros, que pocas
veces se llegan a observar
en la práctica.

04_CHAPTER 4.indd 103 3/6/09 8:22:27 PM


104 Capítulo 4 demanda individual y del merCado

tener un bien Giffen. Primero, no sólo tiene que ser inferior, sino también tener una participación
grande en el presupuesto del consumidor. De otra manera, un aumento de su precio no constituirá
una reducción significativa del poder de compra real. (Que se duplique el precio de los llaveros, por
ejemplo, no hace que nadie se vuelva más pobre.) La segunda característica requerida en un bien de
Giffen es que tenga un efecto sustitución relativamente pequeño, lo suficiente para ser superado por
el efecto ingreso.
En la práctica es muy poco probable que un bien satisfaga ambos requerimientos. Después de
todo, la mayoría de los bienes sólo tiene una participación muy pequeña en el gasto total del consu-
midor. Además, como ya se dijo, cuanto más amplia es la definición de un bien, es menos probable
que éste sea inferior. Por último, por su misma naturaleza, los bienes inferiores suelen tener sustitu-
tos cercanos. La tendencia del consumidor a sustituir la hamburguesa por carne molida de solomillo,
por ejemplo, es lo que hace a la hamburguesa un bien inferior.
Los bienes Giffen son una anomalía desconcertante cuyo principal uso es probar que los es-
tudiantes comprenden las sutilezas de los efectos ingreso y sustitución. A menos que se diga otra
cosa, se supondrá que todas las curvas de demanda que se emplean en el resto de este libro tienen la
pendiente negativa convencional.

EJEMPLO 4.1 Efectos ingreso y sustitución en el caso de complementos perfectos. Suponga que los esquíes
y los fijadores de esquíes son complementos perfectos y que Paula gasta todo su presupuesto
para equipo, que es de 1 200 dólares/año, en estos dos bienes. Los esquíes y los fijadores se
venden, cada uno, en 200 dólares. ¿Cuál será el efecto ingreso y el efecto sustitución de un
aumento del precio de los fijadores de esquíes a 400 dólares el par?
Como en este caso el objetivo es examinar el efecto sobre dos bienes específicos (esquíes y fijadores)
se dedica un eje a cada uno y se prescinde del bien compuesto. En la figura 4.10, el paquete óptimo
sobre la restricción de presupuesto original B0, se representa A. Paula compra al año tres pares de
esquíes y tres de fijadores. Si el precio de estos últimos aumenta de 200 a 400 dólares el par se obtiene
una nueva restricción de presupuesto, B1, y ahora el paquete óptimo es D, que tiene dos pares de
esquíes por año y dos pares de fijadores. Con el nuevo precio, el presupuesto para equipo que el
consumidor necesitaría para alcanzar la misma curva de indiferencia que tenía originalmente (I0)
es de 1 800 dólares/año. (Para obtener esta cifra se desliza B1 hacia afuera hasta ser tangente a I0, y
después se calcula el costo de comprar el paquete en la intersección vertical; en este caso, nueve pares
de esquíes por año a 200 dólares por par.) Observe que como los complementos perfectos tienen
curvas de indiferencia en ángulo recto, el presupuesto B' da como resultado el paquete óptimo C que
es idéntico que el paquete original A. En el caso de los complementos perfectos, el efecto sustitución

FIGURA 4.10
Efectos ingreso y Esquíes (pares/año)
sustitución en el caso 9
de complementos
perfectos 8
en el caso de B’
complementos perfectos, 7
el efecto sustitución del
aumento de precio de los 6
fijadores (el paso de A a
5
C) es igual a cero. el efecto C
ingreso (el paso de A a D) B1
4
y el efecto total son una A
misma cosa. 3 I0

2 I1
D
1 B0

Fijadores (pares/año)
0 1 2 3 4 5 6 7 8

04_CHAPTER 4.indd 104 3/6/09 8:22:28 PM


efeCtos de una variaCión de preCio sobre el ingreso y sobre la sustituCión 105

es cero. Así que en este caso el efecto total del aumento de precio es exactamente el mismo que el
efecto ingreso del aumento de precio.

El ejemplo 4.1 indica que si el precio de los fijadores de esquí aumenta en relación con el precio de
los esquíes, los consumidores no alterarán la proporción de esquíes y fijadores que compra. Pero debi-
do a que el aumento de precio disminuye su poder real de compra (es decir, limita las cantidades que
puede comprar de ambos bienes), adquirirán menos unidades de equipos para esquí. El efecto ingreso
ocasiona que disminuya el consumo tanto de esquíes como de fijadores en la misma proporción.

EJERCICIO 4.1
Repita el ejemplo 4.1 pero ahora suponga que los pares de esquíes y los fijadores son comple-
mentos perfectos dos a uno. (Esto es, suponga que Paula gasta dos pares de esquíes por cada
par de fijadores.)

Efectos ingreso y sustitución para sustitutos perfectos. Suponga que Pam considera al té y al EJEMPLO 4.2
café sustitutos perfectos uno a uno y que en las dos bebidas gasta 12 dólares/semana. El café
tiene un precio de 1 dólar/taza y el té de 1.20/taza. ¿Cómo serán los efectos ingreso y sustitu-
ción de un aumento del precio del café a 1.50/taza?
Al inicio la demanda de Pam será de 12 tazas de café por semana y ninguna taza de té (punto A en la
figura 4.11), dado que los dos bienes contribuyen igual a su utilidad pero el té es más caro. Cuando
el precio del café sube, Pam cambia y sólo consume té, compra 10 tazas por semana y no adquiere
ninguna taza de café (punto D). Pam necesitaría un presupuesto de 14.40 dólares/semana para poder
comprar 12 tazas de té (punto C) que le gustan tanto como las 12 tazas de café que consumía origi-
nalmente. El efecto sustitución va de (12, 0) a (0, 12) y el efecto ingreso de (0, 12) a (0, 10), con lo
cual el efecto total va de (12, 0) a (10, 0). Con los sustitutos perfectos el efecto sustitución puede
ser muy grande: para pequeñas variaciones de precio (cerca de la tasa marginal de sustitución), los
consumidores pueden pasar de consumir sólo de un bien a consumir sólo del otro.

FIGURA 4.11
Efectos ingreso y
Té (tazas/semana) sustitución en el caso de
sustitutos perfectos
12 C en el caso de sustitutos
perfectos, el efecto
10 D sustitución de un aumento
del precio del café (el paso
I0 de A a C) puede ser muy
grande.
I1

B1 B’ B0 A
Café (tazas/semana)
0 8 9 10 12

EJERCICIO 4.2
Partiendo del precio original del ejemplo 4.2, ¿cuáles serían los efectos ingreso y sustitución
que resultarían de un aumento del precio del té a 1.50 de dólar/taza?

04_CHAPTER 4.indd 105 3/6/09 8:22:29 PM


106 Capítulo 4 demanda individual y del merCado

SENSIBILIDAD DEL CONSUMIDOR


ANTE LAS VARIACIONES DE PRECIO
Este capítulo empezó con la observación de que para algunos bienes, como la sal, el consumo es muy
insensible a las variaciones de precio, mientras para otros, como la vivienda, el consumo es mucho
más sensible. La principal razón para estudiar los efectos ingreso y sustitución es que ayudan a enten-
der tales diferencias.
Considere primero el caso de la sal. Cuando se analizan los efectos ingreso y sustitución respec-
to de la sal hay dos características sobresalientes. Primero, que para la mayoría de los consumidores
la sal no tiene un sustituto cercano. Si a alguna persona se le prohíbe que le agregue sal a la carne,
reaccionará poniéndole más pimienta o un poco de jugo de limón. Pero para la mayor parte de las
personas estas alternativas no serán lo mismo que la sal. La segunda característica importante de
la sal es que representa una parte muy pequeña del total de los gastos. Una persona que usara una
cantidad excesiva de sal, consumiría una libra por mes. Si el ingreso de dicha persona es de 1 200
dólares/mes y el precio de la sal se duplica —por ejemplo, de 0.30/lb a 0.60/lb— la porción de su in-
greso que emplea en la sal aumentaría de 0.00025 a 0.0005. Para todo fin práctico, el efecto de ingreso
que tiene la sal es despreciable.
En la figura 4.12 se representa el hecho de que la sal no tenga sustitutos cercanos mediante
curvas de indiferencia casi en ángulo recto. La insignificante participación que tiene la sal en el pre-
supuesto es captada por el hecho de que los vértices de las curvas de indiferencia se presentan en
cantidades extremadamente pequeñas de sal.
Suponga que, como se muestra en la figura 4.12, inicialmente el precio de la sal es 0.30 de dólar/
lb, dando lugar al paquete de equilibrio A que se muestra en la región amplificada, el cual contiene
1.0002 lb/mes de sal. Si el precio aumenta a 0.60 de dólar/lb, se obtiene un nuevo paquete de equili-
brio, D, que tiene 1 lb/mes de sal. Los efectos ingreso y sustitución se miden en términos del paquete
intermedio C. Desde el punto de vista geométrico, el efecto ingreso es pequeño porque la tangencia
original se presenta muy cerca de la intersección vertical de la restricción de presupuesto. Muy cerca
del punto de giro de la restricción de presupuesto, incluso una rotación muy grande produce un
cambio muy pequeño. El efecto sustitución también es pequeño debido a la forma casi en ángulo
recto de las curvas de indiferencia.

FIGURA 4.12
Efectos ingreso y
sustitución de un Y($/mes)
aumento del precio de
la sal C
el efecto total de una A
variación de precio será Y($/mes)
muy pequeño cuando 1) 1 200
el paquete de equilibrio I0
original se encuentre Área amplificada
cerca de la intersección D I1
vertical de la restricción de
presupuesto y 2) la forma B0
1 200 I0
de la curva de indiferencia
I1 B’
sea casi en ángulo recto. el
B1
primer factor hace que el
efecto ingreso (la reducción
del consumo de sal Sal(lb/mes)
relacionada con el cambio B0 1 1.0002
B1 1.0001
de C a D) sea pequeño; el
segundo factor ocasiona
que el efecto sustitución
(la reducción del consumo
de sal relacionada con Sal(lb/mes)
el cambio de A a C) sea 0 2 000 4 000
pequeño.

04_CHAPTER 4.indd 106 3/6/09 8:22:31 PM


sensibilidad del Consumidor ante las variaCiones de preCio 107

Compárese ahora sal con vivienda. Las dos características más sobresalientes con respecto a la
vivienda son que 1) forma parte importante del total de los gastos (para muchas personas, más del
30 por ciento), y 2) la mayor parte de las personas tiene libertad para sustituir la vivienda por otras
cosas. Por ejemplo, muchas de las personas que viven en Manhattan podrían pagar un departamento
más amplio que el que ocupan, pero prefieren gastar lo que ahorran en renta en restaurantes, teatros
y cosas por el estilo. Otra posibilidad de sustitución es ocupar viviendas con peor ubicación. Una
persona que trabaja en Manhattan puede vivir cerca de su trabajo y pagar una renta muy alta; pero
tiene la alternativa de vivir en Nueva Jersey o en Long Island y pagar bastante menos. También
puede buscar un departamento en una zona menos elegante o no muy cerca de una estación del tren
subterráneo. La cuestión es que tratándose de vivienda hay muchas opciones y la elección que se
haga de entre éstas depende fuertemente del ingreso y los precios relativos.
En la figura 4.13, el ingreso del consumidor es de 120 dólares/semana y el precio inicial de
la vivienda es 0.60 de dólar/yd2. La restricción de presupuesto resultante es B0 y el mejor paquete
asequible en esta restricción de presupuesto es A, el cual contiene 100 yd2/semana de vivienda. Un
aumento del precio de la vivienda a 2.40 de dólar/yd2 hace que la cantidad demandada baje a 20
yd2/semana de vivienda. La forma ligeramente convexa de las curvas de indiferencia representa el
alto grado de posibilidades de sustitución entre vivienda y otros bienes y explica el efecto sustitución
relativamente grande (la disminución del consumo de vivienda relacionada con el cambio de A a C).
Observe también que el paquete de equilibrio original, A, se encuentra lejos del punto de giro verti-
cal de la restricción de presupuesto. A diferencia del caso de la sal, aquí la rotación de la restricción de
presupuesto ocasionada por el aumento de precio produce una diferencia grande en la ubicación del
segmento relevante de la nueva restricción de presupuesto. En consecuencia, el efecto ingreso sobre
la vivienda (la caída del consumo de vivienda relacionada con el cambio de C a D) es mucho más
grande que en el caso de la sal. Al tener tanto un efecto ingreso grande como un efecto sustitución
grande que actúan juntos, el efecto total de un aumento de precio de la vivienda (la caída del consu-
mo de vivienda relacionada con el cambio de A a D) es muy grande.

FIGURA 4.13
Y($/mes)
Efectos ingreso y
sustitución para un bien
240
sensible al precio
Como la vivienda requiere
una porción grande del
presupuesto, su efecto
ingreso tiende a ser
B’ grande.y dado que es fácil
de reemplazar, el efecto
sustitución también tiende
a ser grande. las cantidades
demandadas de bienes
120 con efectos grandes, tanto
C sustitución como ingreso,
108
son muy sensibles a las
variaciones de precio.
72 A
D
60

I0
B1 B0
I1
Vivienda (yd2/semana)
0 20 50 100 150 200
55
Efecto total

Efecto Efecto
ingreso sustitución

04_CHAPTER 4.indd 107 3/6/09 8:22:32 PM


108 Capítulo 4 demanda individual y del merCado

EJEMPLO 4.3 Obtención de la curva de demanda individual para complementos perfectos. James conside-
ra el lavado del automóvil y la gasolina como complementos perfectos en una relación 1 a 10,
es decir, necesita un lavado de automóvil por cada 10 galones de gasolina. La gasolina tiene
un precio de 3 dólares/galón y James cuenta con 144 dólares/mes para gastar en gasolina y
lavar el automóvil. (Vea la figura 4.14.) Trace la curva de demanda de lavadas de automóvil
considerando sus cantidades demandadas a varios precios (por ejemplo, 6, 18 y 24 dólares;
vea la figura 4.15).

FIGURA 4.14
Aumento del precio de Gasolina (gal/mes)
lavado de automóvil 48 G = 10L
Con 144 dólares/mes, James A
lava 4 veces al mes el automóvil 40 B
si el precio es 6 dólares/lavado C
(restricción de presupuesto B), 30
3 lavados/mes si el precio es D
18 dólares/lavado (restricción 20 B’
de presupuesto B´) y 2
lavados/mes si el precio es 42
dólares/lavado (restricción de B”
presupuesto B″). Lavados de automóvil/semana
0 2 3 4 6

FIGURA 4.15
Demanda de lavados de Precio ($/lavado)
automóvil de James 42
D
las cantidades de lavados de
automóvil demandadas por
James a los diferentes precios
forman su curva de demanda.
18

6
Lavados de automóvil/mes
0 2 3 4 6

Las preferencias de James muestran que su paquete óptimo debe satisfacer G = 10L (G de gasolina,
L de lavados de automóvil), ya que su curva de indiferencia tiene forma de L. La restricción de pre-
supuesto de James es 3G + PW W = 144 , es decir, G = 48 − Pw3 W . Al sustituir
G = 10W , su restric-
ción de presupuesto es 30W + PW W = 144 , lo cual implica que ( 30 + PW )W = 144 . En PW = 6, W
= 4; en PW = 18, W = 3; en PW = 42, W = 2, como se resume en la tabla 4.3.

TABLA 4.3
Tabla de demanda de lavados de auto

Precio de un lavado de automóvil Cantidad demandada de lavados de


($/lavada) automóvil (lavados/mes)
6 4
18 3
42 2
114 1

04_CHAPTER 4.indd 108 3/6/09 8:22:37 PM


demanda del merCado: agregaCión de las Curvas de demanda individuales 109

DEMANDA DEL MERCADO: AGREGACIÓN


DE LAS CURVAS DE DEMANDA INDIVIDUALES
Una vez que se ha visto de dónde provienen las curvas de demanda individuales, se está en condicio-
nes de ver cómo pueden agregarse para formar una curva de demanda del mercado. Considere un
mercado de demanda de un bien —en la búsqueda de la concreción, una vez más la vivienda— en
el que sólo hay dos consumidores potenciales. Dadas las curvas de demanda de estos consumido-
res, ¿cómo se obtiene la curva de demanda del mercado? En la figura 4.16, D1 y D2 representan las
curvas de demanda individuales de los consumidores 1 y 2, respectivamente. Para obtener la curva
de demanda del mercado se empieza por elegir un precio —por ejemplo, $4/yd2— y sumar las can-
tidades demandadas por cada consumidor a ese precio. Esta suma, 6yd2/semana + 2yd2/semana =
8yd2/semana es la cantidad total de vivienda demandada al precio de 4 dólares/yd2. A continuación
se grafica el punto (4,8) como uno de los pares precio-cantidad de la curva de demanda D de la
gráfica de la derecha en la figura 4.16. Para generar más puntos de la curva de demanda del mercado
se repite este proceso con otros precios. Así el precio 8 dólares/yd2 corresponde a la cantidad 4 + 0
yd2/semana de la curva de demanda del mercado para la vivienda. Si se procede de la misma manera
con otros precios, se traza toda la curva de demanda del mercado. Observe que a precios superiores
a 8 dólares/yd2, el consumidor 2 no demanda vivienda y, de esta manera, la curva de demanda del
mercado para precios superiores a 8 dólares es idéntica a la curva de demanda del consumidor 1.

FIGURA 4.16
Generación de la
demanda del mercado
a partir de demandas
Precio ($/yd2) Precio ($/yd2) Precio ($/yd2) individuales
16 16 16 la curva de demanda del
mercado (D en la gráfica
14 14 14
de la derecha) es la suma
12 12 12 horizontal de las curvas de
10 10 10 demanda individuales D1
8
+ 8
= 8 (gráfica de la izquierda) y
D2 (gráfica del centro).
6 6 6
4 4 4
2 D1 2 D2 2 D

0 2 4 6 8 10 0 2 4 6 8 0 2 4 6 8 10 12
Cantidad (yd2/semana) Cantidad (yd2/semana) Cantidad (yd2/semana)

Al procedimiento de señalar un precio y sumar las cantidades individuales demandadas se le


llama suma horizontal. Ésta se lleva a cabo de la misma manera tanto si en el mercado hay sólo dos
consumidores o varios millones. En los mercados grandes y pequeños, la curva de demanda del
mercado es la suma horizontal de las curvas de demanda individuales.
En el capítulo 2 se vio que suele ser más fácil obtener soluciones numéricas cuando las curvas de
oferta y de demanda están expresadas en forma algebraica y no gráfica. Al usar el método algebraico,
un error común es sumar las curvas de demanda individuales en sentido vertical en lugar de horizon-
tal. Un ejemplo sencillo aclara este peligro.

En un pequeño pueblo de Vermont, Smith y Jones son los únicos consumidores en el mercado EJEMPLO 4.4
de árboles de haya jóvenes. Sus curvas de demanda están dadas por P = 30 – 2QJ y P = 30

04_CHAPTER 4.indd 109 3/6/09 8:22:38 PM


110 Capítulo 4 demanda individual y del merCado

– 3QS donde QJ y QS son las cantidades demandadas por Jones y Smith, respectivamente.
¿Cuál es, en ese pueblo, la curva de demanda del mercado de hayas jóvenes?
Al sumar las curvas de demanda horizontalmente se adicionan cantidades, no precios. De manera
que primero es necesario despejar las cantidades respectivas de las ecuaciones de demanda en térmi-
nos del precio. Esto da Qj = 15 – (P/2) para Jones y Qs = 10 – (P/3) para Smith. Si la cantidad deman-
dada en el mercado se representa con Q, se tiene, Q = DJ – QS = 15 – (P/2) + 10 – (P/3 = 25 – (5P/6).
Despejando ahora P se obtiene la ecuación para la curva de demanda del mercado: P = 30 – (6Q/5).
Se puede verificar que ésta es la curva de demanda del mercado correcta al sumar gráficamente las
curvas individuales como en la figura 4.17.

FIGURA 4.17
Curva de demanda Precio ($/árbol joven) Precio ($/árbol joven) Precio ($/árbol joven)
del mercado de hayas
jóvenes 30 30 30
Cuando se suman
algebraicamente las curvas + =
de demanda, antes de
sumar hay que despejar la D1 D2 D
demanda. QJ QS Q
0 15 0 10 0 25

Cantidad Cantidad Cantidad


(árboles jóvenes/semana) (árboles jóvenes/semana) (árboles jóvenes/semana)

El error que suele cometerse es sumar las funciones de demanda como están dadas original-
mente y despejar después P en términos de Q. En este caso, esto daría P = 30 – (5Q/2) que, como se
ve, no es la curva de demanda que se busca.

EJERCICIO 4.3
Escriba en forma algebraica las curvas de demanda para vivienda de la figura 4.16, después
súmelas algebraicamente para generar la curva de demanda para vivienda del mercado. (Ad-
vertencia: observe que la fórmula de la cantidad a lo largo de D2 sólo es válida para precios
entre 0 y 8.)

La suma horizontal de las demandas de los consumidores individuales para obtener la demanda
del mercado tiene una forma sencilla cuando todos los consumidores del mercado son idénticos.
Suponga n consumidores, cada uno con la curva de demanda P = a – bQi. Para sumar las cantidades
de los n consumidores y obtener una demanda de mercado, se reordena la curva de demanda del
consumidor P = a – bQi para dejar la cantidad sola en un lado Qi = a/b – (1/b) P. Entonces la deman-
da del mercado es la suma de las cantidades demandadas Qi por cada uno de los n consumidores.

 a 1  na n
Q = nQi = n − P  = − P .
 b b  b b

Después la demanda del mercado Q = na/b – n(P/b) puede reordenarse para volver a la forma
en la que se deja el precio solo en un lado P = a – (b/n) Q. De manera intuitiva se ve que de cada
unidad demandada por el mercado, 1/n unidades son para cada consumidor. Estos cálculos sugieren
una regla general para construir la curva de demanda del mercado cuando los consumidores son
idénticos. Si se tienen n curvas de demanda P = a – bQi de consumidores individuales, entonces la
curva de demanda del mercado es P = a – (b/n)Q.

04_CHAPTER 4.indd 110 3/6/09 8:22:40 PM


elastiCidad preCio de la demanda 111

Suponga que un mercado tiene 10 consumidores, la curva de demanda de cada uno de los EJEMPLO 4.5
cuales es P = 10 – 5Qi, donde P es el precio en dólares por unidad y Qi es la cantidad de uni-
dades demandadas por semana por el consumidor i-ésimo (figura 4.18). Obtenga la curva de
demanda del mercado.

FIGURA 4.18
Precio ($/unidad) Demanda del mercado
10 con consumidores
idénticos
si 10 consumidores tienen
la misma curva de demanda,
p = 10 – 5Q, la curva de
demanda del mercado
es la suma horizontal
P = 10 − ( 12 )Q para cada
uno de los precios y con
1 .
pendiente — 10
Di D

Cantidad (unidades/semana)
0 2 20

Primero, se necesita reordenar la curva de demanda del consumidor representativa P = 10 – 5Qi,


de manera que la cantidad se tenga sola en un lado:

Qi = 2 − 51 P .

Después se multiplica por el número de consumidores, n = 10:

Q = nQi = 10Qi = ( 2 − 15 P ) = 20 − 2P .

Por último se reordena la curva de demanda del mercado Q = 20 – 2P, de manera que se tenga el
precio solo en un lado, P = 10 − ( 12 )Q, para volver a la forma pendiente-ordenada al origen.

EJERCICIO 4.4
Suponga un mercado que tiene 30 consumidores y que cada uno tiene la curva de demanda P =
120 - 60Qi, donde P es el precio en dólares por unidad y Qi es la cantidad de unidades demanda-
das por semana por el consumidor i-ésimo. Obtenga la curva de demanda del mercado.

ELASTICIDAD PRECIO DE LA DEMANDA


Una herramienta analítica de importancia central es la elasticidad precio de la demanda. Es una elasticidad precio de la
medida cuantitativa de la sensibilidad de las decisiones de compra ante variaciones de precio y, como demanda cambio porcentual
se verá en éste y en posteriores capítulos, es útil en diversos problemas prácticos. La elasticidad precio en la cantidad demandada
de la demanda se define como el cambio porcentual en la cantidad demandada de un bien ante un cambio de un bien ante un cambio
porcentual en el precio. Por ejemplo, si el precio de la vivienda aumenta uno por ciento ocasiona que la porcentual en su precio.
cantidad de vivienda demandada disminuya 2 por ciento, entonces la elasticidad precio de la deman-
da de vivienda es –2. La elasticidad precio de la demanda siempre será negativa (o cero) porque los
cambios de precio siempre van en dirección opuesta a los cambios en la cantidad demandada.
Se dice que la demanda de un bien es elástica con respecto al precio si su elasticidad precio es
menor que –1. El bien vivenda, mencionado en los párrafos anteriores, tiene una demanda que es
elástica con respecto al precio. La demanda de un bien es inelástica con respecto al precio si su elas-

04_CHAPTER 4.indd 111 3/6/09 8:22:43 PM


112 Capítulo 4 demanda individual y del merCado

FIGURA 4.19
Tres categorías de
elasticidad precio
Con respecto al precio, Elástica unitaria
la demanda de un bien es
elástica si su elasticidad Elástica Inelástica
precio es menor que –1,
inelástica si su elasticidad
precio es mayor que –3 –2 –1 0
–1 y elástica unitaria si su
elasticidad precio es igual
a –1.

ticidad precio es mayor que –1 y es elástica unitaria con respecto al precio si su elasticidad precio es
igual a –1. Estas definiciones se representan de manera gráfica en la figura 4.19.
Al interpretar datos reales de demanda suele ser útil tener una definición más general de la
elasticidad precio que pueda adaptarse a casos en los que el cambio observado en el precio no sea
precisamente de uno por ciento. Sea P el precio real de un bien y Q la cantidad demandada a ese
precio. Sea ∆Q el cambio en la cantidad demandada que ocurre como respuesta a un cambio muy
pequeño en el precio, ∆P. La elasticidad precio de la demanda al precio y cantidad actuales estará
dada entonces por
∆Q P
ε= . (4.1)
∆P P
El numerador en el lado derecho de la ecuación 4.1 es el cambio proporcional en la cantidad. El deno-
minador es el cambio proporcional en el precio. La ecuación 4.1 es idéntica a la definición dada antes
cuando ∆P era un uno por ciento de cambio en el precio real. La ventaja de la definición más general es
que también funciona cuando ∆P representa un pequeño cambio porcentual sobre el precio inicial.

INTERPRETACIÓN GEOMÉTRICA DE LA ELASTICIDAD PRECIO


Otra manera de interpretar la ecuación 4.1 es reescribirla como

∆Q P
ε= .
∆P Q (4.2)
La ecuación 4.2 sugiere una interpretación sencilla en términos de la geometría de la curva de
demanda. Cuando ∆P es pequeño, el cociente ∆P/∆Q es la pendiente de la curva de demanda, lo que
significa que el cociente ∆Q/ ∆P es la inversa de la pendiente. Por lo tanto, la elasticidad precio de la
demanda puede interpretarse como el cociente entre precio y cantidad multiplicado por el recíproco
de la pendiente de la curva de demanda:2

P 1
ε= .
Q pendiente (4.3)
La ecuación 4.3 se llama método punto-pendiente para calcular la elasticidad precio de la demanda.
A manera de ilustración, considere la curva de demanda de vivienda que se muestra en la figura 4.20.
Como dicha curva es lineal, su pendiente es la misma en cualquier punto, a saber, –2. La inversa de
ε A = ( 122es
esta pendiente – 12 .) =
)(− Por−lo
3 tanto, la elasticidad precio de la demanda en el punto A está dado por
el cociente entre el precio y la cantidad en A (12/2) multiplicado por la inversa de la pendiente en
ε A = ( 122A)((–
− 12 ) =
de−manera
3 que se tiene ε A = ( 122 )(− 12 ) = −3 .

2
En términos del cálculo diferencial la elasticidad precio se define como ε = ( P / Q )[ dQ( P )/ dP ].

04_CHAPTER 4.indd 112 3/6/09 8:22:48 PM


elastiCidad preCio de la demanda 113

FIGURA 4.20
Precio ($/yd2) El método punto-
pendiente
16
la elasticidad precio de
14 la demanda en cualquier
A punto es el producto del
PA = 12 cociente precio-cantidad en
εA = (PA/QA)(1/pendiente) = (122)(– 12) = –3 ese punto, multiplicado por
10 la inversa de la pendiente
8 de la curva de demanda en
ese mismo punto. por lo
6 tanto, la elasticidad precio
en A es (12 )(− 1 ) = −3 .
4 D Pendiente = ΔP/ΔQ = –2 2 2

2
Cantidad (yd2/semana)
0 4 6 8 10 12
2 = QA

Cuando la curva de demanda del mercado es lineal, como en la figura 4.20, pueden observar-
se rápidamente varias propiedades de la elasticidad precio a partir de esta interpretación. La prime-
ra es que a lo largo de la curva de demanda la elasticidad precio es diferente en cada punto. De
manera más concreta, se sabe que la pendiente de una curva de demanda lineal es constante, lo cual
significa que la inversa de la pendiente también lo es. En cambio, el cociente precio entre cantidad
toma diferentes valores a lo largo de la curva de demanda. Al acercarse a la intersección vertical,
dicho cociente se aproxima a infinito y disminuye en forma constante a medida que se desciende por
la curva de demanda, alcanzando al final el valor cero en la intersección horizontal.
La segunda propiedad de la elasticidad de la demanda es que nunca es positiva. Como se indicó
antes, dado que la pendiente de la curva de demanda siempre es negativa, su inversa debe ser tam-
bién negativo, y como el cociente P/Q siempre es positivo, se concluye que la elasticidad precio de
la demanda —que es el producto de estos dos— será siempre un número negativo (excepto en la in-
tersección horizontal de la curva de demanda, donde P/Q, y por lo tanto, la elasticidad, es cero). Sin
embargo, por conveniencia, los economistas suelen ignorar el signo negativo de la elasticidad precio y
referirse simplemente a su valor absoluto. Cuando se dice que un bien tiene una elasticidad precio de la
demanda “elevada”, significa que su elasticidad precio es grande en valor absoluto, lo cual indica que
la cantidad demandada es muy sensible a los cambios en el precio. De manera similar, si la elasticidad
precio de un bien es “baja”, el valor absoluto de su elasticidad es pequeño, lo que indica que la canti-
dad demandada es relativamente insensible a los cambios en el precio.
La tercera propiedad de la elasticidad precio en cualquier punto a lo largo de una curva de de-
manda en línea recta es que está relacionada de manera inversa con la pendiente de la curva de
demanda. Cuanto más inclinada es la curva de demanda, menos elástica es la demanda en cualquier
punto de ella. Esto es consecuencia de que la inversa de la pendiente de la curva de demanda es uno
de los factores que se emplean para calcular la elasticidad precio.

EJERCICIO 4.5
Use el método punto-pendiente (ecuación 4.3) para determinar la elasticidad de la curva de
demanda P = 32 – Q en el punto donde P = 24.

En la figura 4.21 se muestran dos casos extremos de la elasticidad de la demanda. En la figura


4.21a, la curva de demanda horizontal, con pendiente cero, tiene una elasticidad precio infinitamente
alta en todos los puntos. Tales curvas de demanda suelen llamarse perfectamente elásticas y, como se
verá, son muy importantes en el estudio del comportamiento de las empresas competitivas. En la

04_CHAPTER 4.indd 113 3/6/09 8:22:50 PM


114 Capítulo 4 demanda individual y del merCado

FIGURA 4.21 P P
Dos casos extremos
importantes
a) la elasticidad precio de la Demanda perfectamente
curva de demanda es igual a elástica Demanda perfectamente
–∞ en todos los puntos. se (ε = – ) inelástica
dice que tales curvas de (ε = 0)
demanda son perfectamente
elásticas. b) la elasticidad
precio de la curva de demanda
es igual a 0 en todos los Q Q
puntos. se dice que tales
curvas de demanda son a) b)
perfectamente inelásticas.

figura 4.21b, la curva de demanda vertical tiene una elasticidad precio igual a cero en todas partes.
Tales curvas se llaman perfectamente inelásticas.
Por razones prácticas, es imposible que una curva de demanda sea perfectamente inelástica para
todos los precios. Más allá de algún precio suficientemente alto, los efectos del ingreso deben reducir
el consumo, aun cuando se trate de bienes al parecer esenciales que no tengan sustitutos, como
operaciones de tumores malignos. De igual manera, la curva de demanda de muchos de esos bienes
y servicios será perfectamente inelástica en un intervalo muy amplio de precios (recuerde el ejemplo
de la sal que se analizó antes en este mismo capítulo).

PROPIEDADES DE LA ELASTICIDAD
DE SER INDEPENDIENTE DE LAS UNIDADES
Otra manera de medir la sensibilidad a los cambios de precio es usar la pendiente de la curva de
demanda. Por ejemplo, si todo lo demás permanece constante, se sabe que la cantidad demandada
de un bien con una curva de demanda muy inclinada será menos sensible a los cambios en el precio
que otro con una curva de demanda menos inclinada.
Como la pendiente de la curva de demanda es mucho más fácil de calcular que su elasticidad,
parece razonable preguntar, “¿qué necesidad hay de ocuparse de la elasticidad?”. Una razón es que
la pendiente de la curva de demanda es sensible a las unidades que se usen para medir precio y canti-
dad, mientras que la elasticidad no lo es. Para ilustrar esto, observe en la figura 4.22a que si el precio
de la gasolina se mide en dls/gal, la pendiente de la curva de demanda en el punto C es –0.02. En
cambio, en la figura 4.22b, donde el precio se mide en dls/onza, la pendiente en C es –0.00015625. Sin
embargo, observe que en ambos casos la elasticidad precio de la demanda es –3. Éste será el valor de

FIGURA 4.22
La elasticidad es P($/gal) P($/oz)
independiente de las
4 A Pendiente = –0.02 0.03125 A Pendiente = –0.00015625
unidades
la pendiente de la curva de ε = –3 ε = –3
demanda en cualquier punto
depende de las unidades 3 0.0234375
C C
en las que se mida precio y
cantidad. la pendiente en el
punto C donde el precio de
la gasolina se mide en dólares
por galón a) es mucho más
grande que cuando el precio
se mide en dólares por onza Q Q
b). en cambio, la elasticidad 50 (gal/día) 50 (gal/día)
precio en cualquier punto es
completamente independiente a) b)
de las unidades de medida.

04_CHAPTER 4.indd 114 3/6/09 8:22:51 PM


elastiCidad preCio de la demanda 115

la elasticidad sin importar cómo se midan precio y cantidad. Y para la mayoría de las personas es más
informativo saber que una disminución de 1% en el precio lleva a un aumento de 3% en la cantidad
demandada que saber que la pendiente de la curva de demanda es –0.00015625.

ALGUNAS ESTIMACIONES REPRESENTATIVAS


DE LA ELASTICIDAD
Como muestra la tabla 4.4, la elasticidad precio de la demanda de diferentes productos suele diferir
de manera sustancial. Es probable que la baja elasticidad del teatro y de la ópera refleje el hecho
de que quienes compran en este mercado tienen un ingreso muy superior al promedio, de manera
que es posible que los efectos del ingreso de las variaciones de precio sean pequeños. Es posible
que los efectos del ingreso en los chícharos también sean pequeños incluso para consumidores de
bajo ingreso; sin embargo, la elasticidad precio de la demanda de chícharos es más de 14 veces más
grande que la del teatro o de la ópera. La diferencia se debe a que hay muchos más sustitutos para los
chícharos que para el teatro o la ópera. Posteriormente en este capítulo se investigarán con más detalle
los factores que afectan la elasticidad precio de la demanda de un producto.

TABLA 4.4
Elasticidad precio de algunos productos*

Bien o servicio Elasticidad precio


Chícharos –2.8
viajes en avión (vacaciones) –1.9
pollos para freír –1.8
Cerveza –1.2
marihuana –1.0
Cine –0.9
viajes en avión (no vacaciones) –0.8
Zapatos –0.7
Cigarros –0.3
teatro, ópera –0.2
llamadas telefónicas locales –0.1
*
algunas de estas estimaciones de la elasticidad a corto plazo representan el punto medio
del correspondiente intervalo de estimaciones. fuentes: fred nordhauser y paul l. farris, “an
estimate of the short-run price elasticity of demand for fryers”, en Journal of Farm Economics,
noviembre de 1959; H. s. Houthakker y lester taylor, Consumer Demand in the United States:
Analyses and Projections, 2a. ed., Cambridge, ma: Harvard university press, 1970; Charles t. nisbet
y firouz vakil, “some estimates of price and expenditure elasticities of demand for marijuana
among uCla students”, en Review of Economics and Statistics, noviembre de 1972; l. taylor, “the
demand for electricity: a survey”, en Bell Journal of Economics, primavera de 1975: K. elzinga, “the
beer industry”, en Walter adams (ed.), The Structure of American Industry, nueva york: mcmillan,
1977; rolla edward park, bruce m. Wetzel y bridger mitchell, Charging for Local Telephone Calls:
Price Elasticity Estimates from the GTE Illinois Experiment, santa mónica, Ca: rand Corporation,
1983; tae H. oum, W. g. Waters ii y Jong say yong, “a survey of recent estimates of price
elasticities of demand for transport”, en World bank infraestructure and urban development
department Working paper 359, enero de 1990; m. C. farrelly y J. W. bray, “response to
increases in Cigarette prices by race/ethnicity, income, and age groups—united states, 1976-
1993”, Journal of the American Medical Association, 280, 1998.

ELASTICIDAD Y GASTO TOTAL


Suponga que usted es el administrador encargado de fijar el peaje del puente Golden Gate, que en-
laza San Francisco con Marin County y que cuando la tarifa es de 3 dólares/trayecto, hay 100 000

04_CHAPTER 4.indd 115 3/6/09 8:22:52 PM


116 Capítulo 4 demanda individual y del merCado

FIGURA 4.23
Efecto sobre el gasto Precio ($/yd2)
total de una reducción
del precio 16
si el precio baja, la gente 14
gasta menos en las unidades Reducción del gasto por la
existentes (E). pero también 12 venta a precio más bajo
E
compra más unidades (G). 10
aquí, G es mayor que E, lo Aumento del gasto debido
que significa que el gasto 8
a ventas adicionales
total aumenta. 6
F G
4
2
Cantidad (yd2/semana)
0 2 4 6 8 10 12 14 16

viajes por hora a través del puente. Si la elasticidad precio de la demanda es –2.0, ¿qué pasará con la
cantidad de viajes que se realizan por hora si el peaje se incrementa 10%? Si la elasticidad es –2.0, un
aumento de 10% en el precio producirá una reducción de 20% en la cantidad. Así, que el número de
viajes bajará a 80 000/h. Para la tarifa más alta, el gasto total será (80 000/h)(3.30 dólares/trayecto)
= 264 000 dólares/h. Observe que esta cantidad es menor que el gasto total de 300 000 dólares/h
asociado a un peaje es de 3 dólares.
Ahora suponga que la elasticidad precio no es –2.0 sino –0.5. ¿Cómo afectará un incremento de
10% en el peaje a la cantidad de viajes y al gasto total? Esta vez el número de viajes disminuirá 5%
a (95 000 viajes/h)(3.30 dólares/viaje) = 313 500 dólares/h. Si su objetivo como administrador es
incrementar el ingreso total obtenido del peaje del puente, necesitará saber algo acerca de la elastici-
dad precio de la demanda antes de decidir si aumenta o baja el peaje.
Este ejemplo ilustra la importante relación entre elasticidad precio y gasto total. Las preguntas
que se desea responder son: “si cambia el precio de un producto, ¿cómo se verá afectado el gasto
total en el producto?” y “¿habrá más gasto si se venden más unidades a un precio más bajo o menos
unidades a un precio más alto?”. Por ejemplo, en la figura 4.23, se desea saber cómo afecta a los
gastos totales en vivienda que el precio baje de 12 dólares/yd2 a 10 dólares/yd2.
El gasto total, R, correspondiente a cualquier par cantidad-precio (Q, P) está dado por el producto

R = PQ. (4.4)

En la figura 4.23, el gasto total correspondiente al par original cantidad-precio es entonces (12 dóla-
res/yd2)(4 yd2/semana) = 48 dólares/semana. Geométricamente, esto es la suma de las áreas som-
breadas E y F. Después de reducir el precio, el nuevo total de gastos es (10 dólares yd2)(6 yd2/semana)
= 60 dólares/semana, que es la suma de las áreas sombreadas F y G. Estas cantidades tienen en
común el área sombreada F. El cambio en el gasto total es entonces la diferencia entre las dos áreas
sombreadas E y G. El área E, que es (2 dólares/yd2)(4 yd2/semana) = 8 dólares/semana, puede inter-
pretarse como la reducción en el gasto causada por vender las 4 yd2/semana iniciales al nuevo precio
más bajo. Por otro lado, G es el aumento en el gasto debido a la venta de 2 yd2/semana más. Esta área
es (10 dólares/yd2)(2 yd2/semana) = 20 dólares/semana. Que el gasto total aumente o disminuya se
reduce entonces a si la ganancia por las ventas adicionales es superior a la pérdida debida a la reduc-
ción del precio. En este caso la ganancia es 12 dólares superior a la pérdida, de manera que el gasto
total aumenta en esta cantidad después de la reducción de precio.
Si la variación de precio es pequeña, se puede predecir cómo cambiará el gasto total si se conoce
la elasticidad precio inicial de la demanda. Recuerde que una manera de expresar la elasticidad precio
es el cambio porcentual en la cantidad dividido entre el correspondiente cambio porcentual en el
precio. Si el valor absoluto de este cociente es mayor que 1, se sabe que el cambio porcentual en

04_CHAPTER 4.indd 116 3/6/09 8:22:53 PM


elastiCidad preCio de la demanda 117

la cantidad es mayor que el cambio porcentual en el precio; y cuando eso pasa, el aumento en el gasto
debido a las ventas adicionales siempre será mayor que la reducción debida a la venta de las unidades
existentes a un precio más bajo. Observe en la figura 4.23 que la elasticidad en el precio inicial de 12
dólares es 3.0, lo que confirma la primera observación de que la reducción de precio lleva a un incre-
mento del gasto total. Suponga ahora que la elasticidad precio es menor que la unidad. Entonces el
cambio porcentual en la cantidad será menor que el correspondiente cambio porcentual en el precio
y las ventas adicionales no compensarán la reducción en el gasto debida a las ventas a un precio más
bajo. En este caso la reducción de precio llevará a una reducción en el gasto total.

EJERCICIO 4.6
Dada la curva de demanda de la figura 4.23, ¿cuál es la elasticidad precio de la demanda cuando
P = 4 dólares/yd2? ¿Qué pasará con el gasto total en vivienda cuando el precio baja de 4 dóla-
res/yd2 a 3 dólares/yd2?

La regla general para una reducción de precio pequeña es, entonces, la siguiente: una reducción
de precio incrementará el ingreso total si y sólo si el valor absoluto de la elasticidad precio de la demanda es
mayor que 1. Con un razonamiento paralelo se obtiene la regla análoga para un pequeño aumento del
precio: un aumento del precio incrementará el ingreso total si y sólo si el valor absoluto de la elasticidad precio
es menor que 1. Estas reglas quedan resumidas en la gráfica superior de la figura 4.24, en la cual M es el
punto medio de la curva de demanda.

FIGURA 4.24
P Demanda y gasto total
ε una reducción del precio incrementa el gasto Cuando la demanda es
8 total; un incremento del precio lo reduce elástica, el gasto total
cambia en dirección
ε el gasto total está en un máximo contraria a la variación
6
en el precio. Cuando la
ε una reducción del precio reduce el gasto total demanda es inelástica el
4 un incremento del precio lo incrementa
M gasto total y el precio
cambian en la misma
2 dirección. en el punto
medio de la curva de
Q demanda (M), el gasto
0 2 4 6 8 total se encuentra en un
Gasto total máximo.
16

12

Q
0 2 4 6 8

La relación entre elasticidad y gasto total se explica con gran detalle a partir de la relación entre
las gráficas superior e inferior de la figura 4.24. En la gráfica superior aparece una curva de demanda
lineal. En la gráfica inferior se muestra el gasto total correspondiente a cada cantidad. Como se indica
en la gráfica inferior, el gasto total comienza en el origen cuando Q es cero y aumenta hasta alcanzar
su valor máximo en la cantidad correspondiente al punto medio de la curva de demanda (punto M
en la gráfica superior). En esa cantidad la elasticidad precio es uno. A partir de esa cantidad, el gasto
total disminuye, llegando a cero en la intersección horizontal de la curva de demanda.

04_CHAPTER 4.indd 117 3/6/09 8:22:54 PM


118 Capítulo 4 demanda individual y del merCado

EJEMPLO 4.6 La curva de demanda del mercado correspondiente a los viajes en autobús en una pequeña
comunidad está dada por P = 100 – (Q/10) donde P es la tarifa por viaje en centavos y Q es
la cantidad de viajes diarios. Si el precio es 50 centavos/viaje, ¿cuál será el ingreso diario que
tendrá la empresa de autobuses? ¿Cuál es la elasticidad precio de la demanda de viajes? Si la
empresa necesita mayores ingresos, ¿debe aumentar o bajar el precio? ¿Cuál sería su respues-
ta en el caso de que el precio inicial fuera no de 50 centavos/viaje sino de 75?
El ingreso total de la empresa de autobuses es igual al gasto total, que es el producto PQ. Pri-
mero se despeja Q de la curva de demanda y se obtiene Q = 1 000 – 10P. Cuando P es 50
centavos/viaje, Q será 500 viajes/día y el ingreso total será $250/día. Para calcular la elasti-
cidad precio de la demanda se puede usar la fórmula e = (P/Q)(1/pendiente). Aquí la pendiente
es – 101 , de modo que 1/pendiente = –10 (vea la nota a pie de página 3). P/Q toma el valor 50/500 =
1
10
. La elasticidad precio es entonces el producto (− 101 )(10) = −1. Con una elasticidad precio igual
a la unidad, el ingreso total alcanza su valor máximo. Si la empresa de autobuses aumenta o baja sus
precios, obtendrá menos de lo que gana al precio actual.
Al precio de 50 centavos la empresa está operando en el punto medio de su curva de demanda.
Si el precio hubiera sido de 75 y no de 50 centavos estaría operando arriba de su punto medio. Con
más precisión, estaría a la mitad entre el punto medio y la intersección vertical (punto K en la figura
4.25). La cantidad sería únicamente de 250 viajes por día y la elasticidad precio sería –3 (obtenido, por
ejemplo, al multiplicar el cociente precio-cantidad en K, 103 , por la inversa de la pendiente de la curva
de demanda, – 101 ). Si opera en un punto elástico sobre su curva de demanda, la empresa puede
incrementar su ingreso total reduciendo su precio.

FIGURA 4.25
Demanda de viajes en Precio (centavos/viaje)
autobús
al precio de 50 centavos/ 100 A
viaje, la empresa de
autobuses maximiza su K
75
ingreso total. al precio
de 75 centavos/viaje,
la demanda es elástica M
50
con respecto al precio y
entonces la empresa puede
incrementar sus ingresos 25
totales si reduce su precio.
E
Cantidad (viajes/día)
0 250 500 750 1 000

DETERMINANTES DE LA ELASTICIDAD
PRECIO DE LA DEMANDA
¿Qué factores influyen en la elasticidad precio de la demanda de un producto? El análisis anterior so-
bre los efectos sustitución e ingreso sugiere que los factores siguientes tienen un papel primordial:
n Posibilidades de sustitución. El efecto sustitución de un cambio de precio tiende a ser pe-
queño cuando se trata de bienes que no tienen sustitutos cercanos. Considere, por ejemplo, la
vacuna contra la rabia. Las personas que han sido mordidas por un animal con rabia no tienen
sustitutos para la vacuna, de manera que su demanda es fuertemente inelástica. Como se vio,
lo mismo ocurre con bienes como la sal. A pesar de la publicidad de los productores de sal, una
marca es un sustituto más o menos perfecto de cualquier otra.

3
En este caso la pendiente proviene de la fórmula P = 100 – (Q/10).

04_CHAPTER 4.indd 118 3/6/09 8:22:57 PM


elastiCidad preCio de la demanda 119

Dado que el efecto sustitución entre las marcas es grande, el aumento de precio de una marca
reducirá notablemente la cantidad demandada de ella. En general, el valor absoluto de la elasti-
cidad precio aumentará con la posibilidad de tener sustitutos atractivos.
n Participación en el presupuesto. Cuanto mayor sea la participación de un producto en los
gastos totales, mayor importancia tendrá el efecto ingreso de una variación de precio. Bienes
como la sal, las ligas, las envolturas de celofán y otros muchos, tienen una participación tan
pequeña en el total de los gastos que los efectos ingreso de una variación de precio son despre-
ciables. En cambio, cuando se trata de bienes como la vivienda y la educación superior, el efecto
ingreso de un aumento de precio es grande. En general, cuanto menor sea la participación de un
bien en el total de los gastos, menor será la elasticidad de la demanda.
n Dirección del efecto ingreso. Un factor estrechamente relacionado con la participación en
el presupuesto es la dirección —positiva o negativa— de su efecto ingreso. Mientras la parti-
cipación en el presupuesto indica si el efecto ingreso de una variación de precio será grande o
pequeña, la dirección del efecto ingreso indica si este efecto contrarrestará o reforzará el efecto
sustitución. Así, un bien normal tendrá una elasticidad precio mayor que un bien inferior, si todo
lo demás permanece constante, debido a que el efecto ingreso reforzará el efecto sustitución del
bien normal pero lo compensará cuando se trate de un bien inferior.
n Tiempo. El análisis hecho aquí de la demanda individual no se centra de manera explícita en
el papel del tiempo, pero también tiene un efecto importante sobre las respuestas a las variacio-
nes de precio. Considere el aumento del precio de la gasolina en los últimos años; una posible
reacción a esto es simplemente usar menos el automóvil. Pero el empleo del automóvil, con
frecuencia, no puede abandonarse, ni siquiera alterarse en un plazo corto. Por ejemplo, una per-
sona no puede dejar de ir a su trabajo. Puede reducir el uso diario de su automóvil reuniéndose
con otras personas para ir y venir al trabajo o comprar una casa más cerca del lugar de trabajo.
También puede reducir el consumo de gasolina cambiando su automóvil por uno más eficiente.
Pero todas estas medidas necesitan tiempo, lo que da como resultado que la demanda de gasoli-
na sea mucho más elástica a largo que a corto plazos.

En la figura 4.26 se comparan los efectos a corto y largo plazos de un desplazamiento de la oferta
en el mercado de la gasolina. El equilibrio inicial en A es perturbado por una disminución de la ofer-
ta de S a S'. A corto plazo, el efecto es que suba el precio a PCP = $2.80/gal y que la cantidad dismi-
nuya a QCP = 5 millones de galones/día. La curva de demanda a largo plazo es más elástica que la
curva de demanda a corto plazo. A medida que los consumidores tienen más tiempo para ajustarse,
los efectos del precio tienden a moderarse mientras los de la cantidad tienden a hacerse más pronun-

FIGURA 4.26
P($/galón)
La elasticidad precio es
S’ mayor a largo plazo que
a corto plazo
S Cuanto más tiempo tiene
la gente, es más fácil
PCP = 2.8 que pueda cambiarse a
PLP = 2.6 productos sustitutos. los
A
efectos en el precio de una
2.4 variación en la oferta son
por lo tanto siempre más
S’ extremos en el corto plazo
DLP que en el largo.
S DCP
Cantidad (millones de galones/día)
4 5 6
QLP QCP

04_CHAPTER 4.indd 119 3/6/09 8:22:58 PM


120 Capítulo 4 demanda individual y del merCado

ciados. De manera que en la figura 4.26 el nuevo equilibrio a largo plazo se presenta en el precio PLP
= 2.60 dólares/galón y en la cantidad QLP = 4 millones de galones/día.
Un caso extremo de la diferencia entre los valores de elasticidad precio a corto y a largo plazos es
la del gas natural que se emplea en los hogares. La elasticidad precio de este producto a corto plazo
es sólo de –0.1, pero cae de manera estruendosa a –10.7 a largo plazo.4 Esta diferencia refleja el hecho
de que una vez que los consumidores han elegido un aparato doméstico para cocinar, están prácti-
camente atados a corto plazo. La gente no puede cocinar el arroz sólo 10 minutos debido a que el
precio del gas natural ha subido. Sin embargo, a largo plazo, el consumidor puede emplear, y lo hace,
otros combustibles cuando hay variaciones significativas en los precios relativos.

DEPENDENCIA DE LA DEMANDA DEL MERCADO


CON RESPECTO AL INGRESO
Como se ha visto, la cantidad que una persona demanda de un bien no sólo depende de su precio, sino
también del ingreso de la persona. Como la curva de demanda del mercado es la suma horizontal de
las curvas de demanda individuales, también depende del ingreso de los consumidores. En algunos
casos el efecto del ingreso sobre la demanda de mercado puede explicarse por completo si se conoce
el nivel de ingreso promedio en el mercado. Éste sería el caso, por ejemplo, si todos los consumidores
del mercado fueran parecidos en términos de preferencias y todos tuvieran los mismos ingresos.
Sin embargo, en la práctica, un determinado nivel de ingreso promedio en un mercado algunas
veces dará lugar a diferentes demandas dependiendo de cómo esté distribuido el ingreso. Un ejemplo
sencillo ayuda a aclarar esto.

EJEMPLO 4.7 Dos consumidores, A y B, se encuentran en un mercado de alimento. Sus gustos son idén-
ticos y los dos tienen el mismo ingreso inicial, 120 dólares/semana. Si sus curvas de Engel
para alimento son como la EE de la figura 4.27, ¿cómo se verá afectada la curva de demanda
de alimento del mercado si el ingreso de A se reduce 50 por ciento mientras el ingreso de B
aumenta 50 por ciento?

FIGURA 4.27 Ingreso ($/semana) E


Curvas de Engel para
alimento de A y de B 180
Cuando las curvas de
engel individuales toman
la forma no lineal que se
muestra, el incremento 120
en el consumo de
alimento que resulta de un
determinado aumento en el
ingreso será menor que la
reducción del consumo de 60
alimento resultante de una
disminución del ingreso de E
la misma cantidad.
Alimento (lb/semana)
0 2 4 6 8 10 12

La forma no lineal de la curva de Engel que se muestra en la figura 4.27, es plausible si se considera
que un consumidor sólo puede comer una determinada cantidad de alimento. A partir de ese mo-
mento, el aumento en el ingreso no tiene un efecto apreciable sobre la cantidad de alimento con-
sumida. El resultado es que el nuevo ingreso de B (180 dólares/semana) producirá un aumento en su

4
H. S. Houthakker y Lester Taylor, Consumer Demand in the United States: Analyses and Projections, 2a. ed., Cambridge, MA: Harvard
University Press, 1970.

04_CHAPTER 4.indd 120 3/6/09 8:22:58 PM


dependenCia de la demanda del merCado Con respeCto al ingreso 121

consumo (2 lb/semana) menor que la reducción en el consumo de A (4 lb/semana) debido al nuevo


ingreso de A (60 dólares/semana).
¿Qué dice todo esto acerca de las correspondientes curvas de demanda de alimento, individual
y de mercado? Ingresos y gustos idénticos dan lugar a curvas de demanda individuales idénticas,
denominadas DA y DB en la figura 4.28. Al sumar DA y DB de manera horizontal, se obtiene la curva de
demanda de mercado inicial, denominada D. El carácter de las curvas de Engel individuales indica
que el aumento de la demanda de B será menor que la reducción de la demanda de A después del
desplazamiento de la distribución del ingreso. Por lo tanto, cuando se suman las nuevas curvas de
demanda individuales (D'A y D'B) se obtiene una curva de demanda de alimento del mercado (D') que
se encuentra a la izquierda de la curva de demanda original.

FIGURA 4.28
Algunas veces la
Precio ($/lb) Precio ($/lb) Precio ($/lb)
demanda del mercado
depende de la
El ingreso de El ingreso de La demanda del distribución del ingreso
A disminuye B aumenta mercado disminuye un determinado aumento
del ingreso produce un
pequeño aumento en la
demanda de B, b); una
D’ D reducción del ingreso de la
D’B misma magnitud produce
DA DB
D’A una mayor reducción de
la demanda de A, a). la
Alimento de A (lb/semana) Alimento de B (lb/semana) Alimento (lb/semana) redistribución de A a B
no modifica el ingreso
a) b) c) promedio pero reduce la
demanda del mercado, c).

Es importante que, al considerar las políticas de redistribución del ingreso, el gobierno tenga en
mente que la demanda del mercado depende de la distribución del ingreso. Una política que reparte
el ingreso de los ricos entre los pobres, por ejemplo, es posible que incremente la demanda de bienes
como alimento y reduzca la de artículos de lujo, como joyas y viajes al extranjero.
La demanda en muchos otros mercados es relativamente insensible a las variaciones en la dis-
tribución del ingreso. En particular, es poco probable que la distribución del ingreso tenga mucha
importancia en mercados en los que las demandas de los individuos apenas tienden a cambiar en
proporción con las variaciones en el ingreso.
Las curvas de Engel en el ámbito del mercado relacionan la cantidad demandada con el ingreso
promedio. La existencia de una relación estable entre ingreso promedio y cantidad demandada no
puede asegurarse para ningún producto determinado debido a la complicación de distribución que
acaba de analizarse. En particular, observe que a nivel del mercado, no se puede construir una curva
de Engel mediante la simple suma horizontal de curvas de Engel individuales. La suma horizontal
funciona para generar curvas de demanda del mercado a partir de curvas de demanda individuales
porque en el mercado todos los consumidores encuentran un mismo precio para el producto. Pero
si el ingreso difiere de un consumidor a otro no tiene sentido mantener el ingreso constante y sumar
cantidades entre los consumidores.
Sin embargo, en la práctica pueden existir relaciones razonablemente estables entre diversas
medidas globales del ingreso y cantidades demandadas en el mercado. Suponga que existe una rela-
ción así para el bien X y que es como la que describe la línea EE en la figura 4.29, donde Y representa elasticidad ingreso de la
el ingreso promedio de los consumidores en el mercado del bien X y Q representa la cantidad de X. demanda cambio porcentual
Esta gráfica es la análoga en el ámbito del mercado de las curvas individuales de Engel que se vieron en la cantidad demandada de
antes. un bien como resultado de
Si un bien muestra una curva de Engel estable, se puede definir su elasticidad ingreso de la de- un cambio porcentual en el
manda, como una medida formal de la sensibilidad de las decisiones de compra ante las variaciones ingreso.

04_CHAPTER 4.indd 121 3/6/09 8:23:00 PM


122 Capítulo 4 demanda individual y del merCado

FIGURA 4.29
Curva de Engel en el Ingreso promedio
ámbito del mercado ($/semana)
la curva de engel E
del mercado indica
qué cantidades serán
demandadas a diversos
ingresos promedio.

E
Q

en el ingreso promedio en el mercado. Representada por η, está dada por una fórmula análoga a la
de la elasticidad precio:5
∆Q Q
η= , (4.5)
∆Y Y
donde Y representa el ingreso promedio en el mercado y ∆Y es un cambio pequeño en él.
Bienes como el alimento, en los cuales una variación del ingreso produce una variación me-
nos que proporcional en la cantidad demandada a cualquier precio, tienen una elasticidad ingreso
menor a 1. Tales bienes se llaman necesarios y su elasticidad ingreso debe encontrarse en el interva-
lo 0 < η < 1. El alimento es un ejemplo que se cita con frecuencia. Lujos son aquellos bienes para
los que η > 1. Ejemplos comunes de estos bienes son las joyas caras y los viajes al extranjero. Los
bienes inferiores son aquellos para los que η < 0. Los bienes para los que η = 1 tienen curvas de
Engel que son líneas rectas que pasan a través del origen, como la línea EE de la figura 4.30a.

FIGURA 4.30
Curvas de Engel para
diversos tipos de bienes
a) el bien cuya curva de
engel se muestra tiene
una elasticidad ingreso
igual a 1. en el caso de
estos bienes una variación Ingreso promedio Ingreso promedio
proporcional en el ingreso ($/semana) ($/semana)
Bienes necesarios (η < 1)
produce la misma variación
proporcional en la cantidad E
demandada. así, cuando 2M0
el ingreso promedio se
duplica, de M0 a 2M0,
la cantidad demandada Bienes de lujo (η > 1)
M0
también se duplica de
Q0 a 2Q0. b) las curvas E Bienes inferiores (η > 0)
de engel muestran que Q Q
el consumo aumenta de Q0 2Q0
manera proporcional más
a) b)
que el ingreso cuando
se trata de artículos de
lujo y proporcionalmente
menos que el ingreso
cuando se trata de artículos
necesarios, y disminuye con
el ingreso cuando se trata
de bienes inferiores.

5
En términos del cálculo, la fórmula correspondiente es η = (Y/Q)[dQ(Y)/dY].

04_CHAPTER 4.indd 122 3/6/09 8:23:02 PM


dependenCia de la demanda del merCado Con respeCto al ingreso 123

En la figura 4.30b se presentan las curvas de mercado de Engel para artículos de lujo, necesarios
e inferiores, donde éstos existen y son estables.
La fórmula elasticidad ingreso en la ecuación 4.5 es más fácil de interpretar geométricamente si
se reescribe como
Y ∆Q
η= , (4.6)
Q ∆Y
El primer factor en el lado derecho de la ecuación 4.6 es simplemente el cociente ingreso entre can-
tidad en un punto a lo largo de la curva de Engel. Este cociente es la pendiente de la línea desde el
origen hasta ese punto (un rayo). El segundo factor es la inversa de la pendiente de la curva de Engel
en ese punto. Si la pendiente del rayo es mayor que la de la curva de Engel, el producto de los dos
factores debe ser mayor que uno (caso de un bien de lujo). Si el rayo es menos inclinado, η será me-
nor que uno pero todavía positivo, siempre que la pendiente de la curva de Engel sea positiva (caso
de un bien necesario). Por lo tanto, para distinguir entre curvas de Engel de artículos necesarios y de
artículos de lujo, lo que cuenta no es la pendiente de las curvas de Engel mismas, sino dicha pendien-
te comparada con la pendiente del rayo correspondiente. Por último, si la pendiente de la curva de
Engel es negativa, η será menor que cero (caso de un bien inferior).

¿Por qué en las últimas décadas ha habido un cambio tan dramático en los aparatos para
cocinar en el exterior?
La parrilla de propano que compré al final de los años ochenta se fue desgastando durante varios
años. Lo primero que se echó a perder fue el botón de ignición, el generador mecánico de la chispa EL NATURALISTA
que normalmente enciende el gas. De repente encender la parrilla se volvió una operación delicada. ECONÓMICO
Tenía que abrir la válvula del gas, esperar unos segundos y lanzar en su interior un cerillo. Si lo lan-
zaba demasiado pronto se apagaba antes de encender el gas. Pero si esperaba demasiado ocasionaba
4.1
una pequeña explosión. Otro problema era que el centro del deflactor metálico que va encima de
los quemadores se había oxidado por completo. Esto hacía que sobre una pequeña área, cerca del
centro de la superficie para cocinar, se concentrara una enorme cantidad de calor pero muy poca en
el resto. Sin embargo, se podía seguir cocinando bien pollo o pequeños bisteces cambiando las piezas
continuamente hacia dentro y hacia fuera de la zona de calor. Pero era imposible asar un filete de
pescado grande.
Probablemente, las diversas deficiencias de mi asador tenían reparación, pero yo no tenía ni
idea de quién podía hacerlo. Y aunque la hubiera tenido, es casi seguro que el precio de la reparación
hubiera sido mayor a los 89.95 dólares que pagué originalmente por el asador. Y a pesar de no estar
convencido, fui al mercado a comprar uno nuevo.
Cortesía de Robert H. Frank

asador profesional, perfectamente


equipado (similar al viking que se cita
asador sunbeam de 1989, 90 dólares en el texto)

04_CHAPTER 4.indd 123 3/6/09 8:23:03 PM


124 Capítulo 4 demanda individual y del merCado

Quedé impresionado con la manera en que había cambiado la enorme variedad de opciones
durante los años que habían pasado. Vagamente recuerdo que algunos modelos de finales de los años
ochenta tenían anaqueles y extensiones en ambos lados. Pero aun con estos aditamentos, lo más que
se podía gastar eran unos cuantos cientos de dólares. No había nada —absolutamente nada— pareci-
do al asador profesional Viking.
Este asador puede funcionar con gas natural o gas propano, viene con un asador infrarrojo que
puede asar lentamente y a la perfección dos pavos de 20 lb mientras se cocinan hamburguesas para
40 personas en su parrilla de 828 pulgadas cuadradas. Tiene un sistema de ahumado que “utiliza su
propio quemador de 5 000 BTU y un recipiente hermético con astillas de madera para sazonar los
alimentos con un delicioso sabor a bosque”. Junto a la parrilla hay dos quemadores auxiliares. A
diferencia de los de tipo estándar de una estufa de cocina, que generan 7 500 BTU, éstos producen
15 000 BTU, una capacidad que se usa sobre todo para un salteado instantáneo, necesario en algunas
recetas de cocina étnicas y para hacer hervir más rápido calderos grandes de agua. Si alguna vez ha
deseado preparar un cerdo a la Szechuán (platillo chino) en el patio trasero de su casa o teme que se
le haga tarde cuando sus invitados están a punto de llegar y le falta cocinar 40 elotes, el Viking tiene
la potencia extra que usted necesita. Toda la unidad está construida en flamante acero inoxidable,
con detalles en esmalte y bronce, y su superficie de trabajo, completamente expandible, mide más de
siete pies.
El precio del Viking en el catálogo Frontgate, sin incluir costos de envío, es de 5 000 dólares.
Existen también otros aparatos para cocinar en el exterior cuyo precio es más de diez veces esa can-
tidad.
¿Qué generó este dramático auge en el mercado estadounidense de asadores de lujo para ex-
terior? Una respuesta rápida es que la mayor parte del reciente crecimiento del ingreso en Estados
Unidos se dio entre aquellos que perciben los salarios más altos de la nación. Por ejemplo, aunque
en el periodo 1979 a 2003, el ingreso familiar mediano después de impuestos creció sólo 12.6 por
ciento, el crecimiento correspondiente en el uno por ciento de nivel más alto fue de más de 200
por ciento.6 Así, los ejecutivos de las corporaciones más grandes de Estados Unidos, que en 1980
ganaban 42 veces lo que un trabajador promedio, en 2000 ganaban 531 veces lo que un trabajador
promedio.7 El rápido crecimiento del ingreso entre aquellos que ya contaban con ingresos altos ge-
neró una demanda mayor, no sólo de costosos dispositivos para cocinar en el exterior, sino también
de un amplio espectro de otros bienes de lujo.

APLICACIONES: PRONÓSTICO
DE LAS TENDENCIAS ECONÓMICAS
Si la elasticidad ingreso de la demanda de todos los bienes y servicios fuera 1, la composición del
producto interno bruto sería completamente estable a lo largo del tiempo (suponiendo que la tecno-
logía y los precios relativos permanecieran constantes). Cada año la proporción del gasto total que se
dedica a alimento, viajes, vestido y a cualquier otra categoría de consumo permanecería constante.
Sin embargo, como se observa en las entradas de la tabla 4.5, la elasticidad ingreso de las dife-
rentes categorías de consumo difiere notablemente. Y allí se encuentra una de las aplicaciones más
importantes del concepto de elasticidad ingreso, que es el pronóstico de la composición de los pa-
trones futuros de compra. Desde la revolución industrial en Occidente, el poder de compra real per
cápita ha crecido aproximadamente 2 por ciento cada año. El conocimiento de las diferencias de la
elasticidad ingreso permite predecir cómo diferirán en el futuro dos patrones de consumo de los que
se ven ahora.
Así una creciente proporción del presupuesto del consumidor estará dedicada a bienes como
comidas en restaurantes y automóviles, mientras que una proporción cada vez menor se dedicará a

6
http://www.census.gov/hhes/income/histinc/hφ3ar.html y Center on Budget and Policy Priorities, “The New Definitive CBO
Data on Income and Tax Trends”, 23 de septiembre de 2003.
7
Business Week, informes anuales de compensación ejecutiva. Vea www.inequality.org

04_CHAPTER 4.indd 124 3/6/09 8:23:03 PM


elastiCidades preCio CruZadas de la demanda 125

TABLA 4.5
Elasticidades ingreso de la demanda de algunos productos*

Bienes o servicios Elasticidad ingreso


automóviles 2.46
muebles 1.48
Comidas en restaurante 1.40
agua 1.02
tabaco 0.64
gasolina y aceite 0.48
electricidad 0.20
margarina –0.20
productos de cerdo –0.20
transporte público –0.36

*estas estimaciones se tomaron de H. s. Houthakker y lester taylor, Consumer Demand in the United States: Analyses
and Projections, 2a. ed., Cambridge, ma: Harvard university press, 1970; l. taylor y r. Halvorsen, “energy sustitution
in u. s. manufacturing”, en Review of Economics and Statistics, noviembre de 1977; H. Wold y l. Jureen, Demand Analysis:
nueva york, Wiley, 1953.

tabaco, combustible y electricidad. Y si las estimaciones de la elasticidad son correctas, las cantidades
absolutas gastadas por persona en margarina, productos de cerdo y transporte público serán conside-
rablemente menores en el futuro comparadas con lo que son hoy.

ELASTICIDADES PRECIO CRUZADAS


DE LA DEMANDA
La cantidad que se compra de un producto en el mercado depende no sólo de su precio y de los
ingresos del consumidor sino también de los precios de los bienes relacionados con él. La elasticidad elasticidad precio cruzada
precio cruzada de la demanda es la variación porcentual en la cantidad demandada de un bien de la demanda cambio
debido a un cambio de uno por ciento en el precio del otro bien. De manera más general, dados dos porcentual en la cantidad
productos cualesquiera, X y Z, la elasticidad precio cruzada de la demanda puede definirse como demandada de un bien que es
sigue:8 resultado de una variación de
uno por ciento en el precio de
(4.7) otro bien.
∆QX QX
ε XZ = ,
∆PZ PZ

donde ∆QX es un cambio pequeño en QX, la cantidad de X y ∆PZ es un cambio pequeño en PZ, el precio
de Z. eXZ mide cómo responde la cantidad demandada de X a un pequeño cambio en el precio de Z.
A diferencia de la elasticidad de la demanda con respecto al precio propio de un bien (elasticidad
del precio propio), que nunca es mayor que cero, la elasticidad precio cruzada puede ser positiva o
negativa. X y Z se definen como complementarios si eXZ < 0. Si eXZ > 0, los bienes son sustitutos. Así,
si aumenta el precio del jamón no sólo disminuirá la cantidad que se demanda de él, sino también
la cantidad demandada de huevos, ya que ambos productos son complementarios. En cambio, si
aumenta el precio del café, tenderá a aumentar la demanda de té. En la tabla 4.6 se muestran estima-
ciones de la elasticidad precio cruzada de la demanda de algunos pares de productos.

8
En términos del cálculo, la expresión correspondiente está dada por eXZ = (PZ/QX)(dQX/dPZ).

04_CHAPTER 4.indd 125 3/6/09 8:23:04 PM


126 Capítulo 4 demanda individual y del merCado

TABLA 4.6
Elasticidad precio cruzada de algunos pares de productos*

Bien o servicio Bien o servicio que ha variado de precio Elasticidad precio cruzada
mantequilla margarina +0.81
margarina mantequilla +0.67
gas natural gasóleo +0.44
Carne de res Carne de cerdo +0.28
electricidad gas natural +0.20
entretenimiento alimentos –0.72
Cereales pescado fresco –0.87

* tomado de H. Wold y l. Jureen, Demand Analysis, nueva york: Wiley, 1953; l. taylor y r. Halvorsen, “energy sustitution in u. s.
manufacturing”, en Review of Economics and Statistics, noviembre de 1977; e. t. fujii et al., “an almost ideal demand system for visitor
expenditures”, en Journal of Transport Economics and Policy, 19 de mayo de 1985, pp. 161-171; a. deaton, “estimation of own- and Cross-
price elasticities from Household survey data”, en Journal of Econometrics, 36, 1987: pp. 7-30.

EJERCICIO 4.7
Para los siguientes pares de bienes, ¿la elasticidad precio cruzada de la demanda será positiva o
negativa?: a) manzanas y naranjas, b) boletos de avión y neumáticos para automóviles, c) herra-
mientas para computación y programas de cómputo, d) plumas y papel, e) plumas y lápices.

RESUMEN
• El tema principal en este capítulo ha sido analizar cómo respon- cantidad demandada como resultado del cambio en el poder
den la demanda individual y de mercado a las variaciones de real de compra ocasionado por la alteración de precio. El efecto
precio y de ingreso. Para generar una curva de demanda para sustitución siempre va en dirección opuesta a la variación del
un consumidor individual de un determinado bien X, primero precio: si el precio aumenta [disminuye] la cantidad demandada
se traza la curva consumo-precio en el diagrama de curva de in- siempre disminuye [aumenta]. En el caso de bienes normales,
diferencia estándar. La CPC es la línea de paquetes óptimos que el efecto ingreso va en la dirección contraria a la variación de
se observan cuando varía el precio de X, si el ingreso y las pre- precio y de esta manera tiende a reforzar el efecto sustitución.
ferencias permanecen constantes. Después, de la CPC se toman En el caso de bienes inferiores el efecto ingreso va en la misma
los pares precio-cantidad relevantes y se grafican en un diagra- dirección que la variación de precio y por lo tanto tiende a dis-
ma separado para obtener la curva de demanda individual. minuir el efecto sustitución.
• El análogo de la CPC en el ingreso es la curva consumo-ingreso • El hecho de que los efectos ingreso y sustitución vayan en direc-
o CCI. Se construye también mediante el diagrama estándar de ciones opuestas cuando se trata de bienes inferiores sugiere la
la curva de indiferencia. La CCI es la línea de paquetes óptimos posibilidad teórica de un bien Giffen, para el cual el efecto total
que se traza al variar el ingreso del consumidor, si las preferen- de un aumento de precio es incrementar la cantidad demanda-
cias y los precios relativos permanecen constantes. La curva de da. No se han documentado ejemplos de bienes Giffen, y en este
Engel es el análogo en el ingreso de la curva de demanda indi- libro se supone que todos los bienes, a menos que se diga otra
vidual. Se genera al tomar los pares relevantes ingreso-cantidad cosa, son demandados en cantidades más pequeñas cuanto ma-
de la CCI y graficándolos en un diagrama aparte. yores son los precios.
• Bienes normales son los que el consumidor compra más cuando • Los bienes para los cuales las decisiones de compra responden
aumenta su ingreso y bienes inferiores son los que el consumi- con más fuerza al precio tienden a ser aquellos que tienen gran-
dor compra menos cuando aumenta su ingreso. des efectos ingreso y sustitución que funcionan en una misma
• El efecto total de un variación de precio puede descomponerse dirección. Por ejemplo, un bien normal que ocupa una pro-
en dos efectos distintos: 1) el efecto sustitución, que muestra porción grande de los gastos totales y para el cual hay muchos
el cambio en la cantidad demandada debido a que la variación sustitutos directos o indirectos tenderá a responder de manera
de precio hace que los bienes sustitutos resulten más o menos muy marcada a las variaciones de precio. Para muchos consu-
atractivos, y 2) el efecto ingreso, que denota la variación en la midores, la vivienda es uno de los principales ejemplos de tales

04_CHAPTER 4.indd 126 3/6/09 8:23:05 PM


preguntas de repaso 127

bienes. Los bienes menos sensibles a las variaciones de precio al precio que los bienes normales. 4) Tiempo. Los hábitos y los
serán aquellos a los que les corresponda una proporción muy compromisos existentes limitan el grado al que los consumido-
pequeña del presupuesto y para los que las posibilidades de sus- res responden a los cambios de precio en el corto plazo. La elas-
titución sean muy limitadas. Para la mayoría de la gente, la sal ticidad precio de la demanda tenderá a ser mayor cuanto mayor
tiene estas dos propiedades. sea el tiempo que tienen los consumidores para adaptarse.
• Existen dos técnicas equivalentes para generar curvas de deman- • Los cambios en el nivel de ingreso promedio en un mercado
da del mercado a partir de curvas de demanda individuales. La suelen desplazar la curva de demanda. La elasticidad ingreso de
primera consiste en trazar gráficamente las curvas individuales la demanda se define de manera análoga a la elasticidad precio.
y después sumarlas horizontalmente. La segunda es algebraica y Es el cambio porcentual en la cantidad que resulta de una varia-
en ella primero se despejan los valores de Q de las curvas de ción de uno por ciento en el ingreso. A los bienes cuya elastici-
demanda individuales, se suman y por último de la suma resul- dad ingreso de la demanda es mayor que cero se les llama bienes
tante se despeja P. normales, para los que es menor que cero se les llama bienes in-
• Un concepto analítico central en la teoría de la demanda es la feriores, para los que es mayor que 1 se les llama bienes de lujo
elasticidad precio de la demanda, una medida de la sensibilidad y para los que es menor que 1 se les llama bienes necesarios. En
de las decisiones de compra a variaciones pequeñas en el precio. el caso de los bienes normales, el aumento del ingreso desplaza
Se define como el cambio porcentual en la cantidad demandada la demanda de mercado hacia la derecha, y en el caso de bienes
debido a una variación de uno por ciento en el precio. Bienes inferiores hacia la izquierda. Para algunos bienes la distribución
para los cuales el valor absoluto de la elasticidad es mayor que del ingreso, no sólo su valor promedio, es un determinante im-
1 se dice que son elásticos, aquellos para los cuales es menor portante de la demanda de mercado.
que 1 son inelásticos y aquellos para los que es igual a 1 son • La elasticidad precio cruzada de la demanda es una medida de la
unitariamente elásticos. sensibilidad de la cantidad demandada de un bien ante variacio-
• Otra relación importante es la que existe entre la elasticidad nes pequeñas en el precio de otro bien. Se define como el cam-
precio y el efecto de una variación del precio sobre el gasto bio porcentual de la cantidad demandada de un bien como re-
total. Cuando la demanda es elástica, una reducción de precio sultado de una variación de uno por ciento en el precio de otro.
aumenta el gasto total; cuando es inelástica, el gasto total dismi- Si la elasticidad precio cruzada de la demanda de X con respecto
nuye cuando el precio se reduce. Cuando la demanda es elástica al precio de Z es positiva, X y Z son sustitutos; si es negativa,
unitaria, el gasto total está en su máximo. los bienes son complementos. Para recordar las fórmulas de las
• La elasticidad precio de la demanda depende en gran parte de diferentes elasticidades —del propio precio, precio cruzada y del
cuatro factores: 1) posibilidades de sustitución. Cuanto más fácil ingreso— muchas personas encuentran útil observar que cada
sea que los consumidores cambien a otros bienes más elástica una de ellas es el cambio porcentual en un efecto dividido entre
será la demanda. 2) Participación en el presupuesto. Cuanto que el cambio porcentual en el factor causal correspondiente.
tienen una participación importante en los gastos totales tien- • En el apéndice de este capítulo se examinan otros temas de
den a tener una elasticidad precio mayor. 3) Dirección del efecto la teoría de demanda, que incluyen la curva de demanda con
de ingreso. Si todos los demás factores permanecen constantes, elasticidad constante y la curva de demanda compensada por
los bienes inferiores tenderán a ser menos elásticos con respecto el ingreso.

pREgUNtaS dE REpaSo
1. ¿Por qué las cantidades de sal demandadas tienden a ser insen- 7. Resuma la relación entre elasticidad precio, variaciones de
sibles a las variaciones en su precio? precio y variación en el gasto total.
2. ¿Por qué la cantidad de educación demandada en las univer- 8. ¿Por qué no se mide la sensibilidad de la demanda a los cam-
sidades privadas es mucho más sensible a las variaciones de bios en el precio mediante la pendiente de la curva de deman-
precio que la sal? da en lugar de usar la expresión para la elasticidad que es más
3. Trace las curvas de Engel de un bien normal y de un bien in- complicada?
ferior. 9. Dada una curva de demanda lineal, ¿cuál es su elasticidad pre-
4. Dé dos ejemplos de bienes que sean bienes inferiores para la cio en el punto máximo de ingreso?
mayoría de los estudiantes. 10. ¿Cree usted que la educación universitaria en una determinada
5. ¿Es posible que la curva consumo-precio de un bien normal universidad tiene una elasticidad precio de la demanda alta o baja?
tenga pendiente negativa? 11. ¿Cómo pueden afectar la demanda de un producto las va-ria-
6. ¿Por qué para obtener la curva de demanda del mercado de ciones de la distribución del ingreso entre los consumidores?
un producto las curvas individuales de demanda se suman 12. Si usted espera que haya un largo periodo durante el cual el
horizontalmente y no verticalmente? PIB disminuya, ¿en qué tipo de empresas invertiría?

04_CHAPTER 4.indd 127 3/6/09 8:23:06 PM


128 Capítulo 4 demanda individual y del merCado

13. Verdadero o falso: si un presupuesto se gasta por completo en 15. Verdadero o falso: si cada individuo en un mercado tiene una
dos bienes, un aumento en el precio de uno necesariamente curva de demanda de un bien lineal, entonces la curva de de-
hará que disminuya el consumo de ambos, a menos que por manda del mercado de ese bien debe ser también una línea
lo menos uno de ellos sea un bien inferior. Explique. recta. Explique.
14. Mike gasta todo su ingreso en pelotas de tenis y en boletos para 16. Suponga que gasta todo su presupuesto en dos bienes: pan y
el basquetbol. Su curva de demanda para pelotas de tenis es mantequilla. Si el pan es un bien inferior, ¿puede la mantequi-
elástica. Verdadero o falso: si el precio de las pelotas de tenis au- lla ser también un bien inferior?
menta comprará más boletos para el basquetbol. Explique.

pRoblEMaS
1. Sam gasta 6 dólares/semana en jugo de naranja y jugo de manzana. El jugo de naranja se vende en
2 dólares/vaso y el jugo de manzana en 1 dólar/vaso. Para Sam un vaso de jugo de naranja es un
sustituto perfecto de 3 vasos de jugo de manzana. Encuentre su paquete óptimo de consumo de jugo
de naranja y jugo de manzana por semana. Suponga que el precio del jugo de manzana aumenta
a 2 dólares/vaso y que el precio del jugo de naranja permanece constante. ¿Qué ingreso adicional
necesita Sam para tener su paquete de consumo original?
2. Bruce tiene el mismo ingreso y también enfrenta los mismos precios que Sam en el problema 1,
pero para él un vaso de jugo de naranja es un sustituto perfecto de un vaso de jugo de manzana.
Encuentre el paquete óptimo de consumo para Bruce. Si el precio del jugo de manzana aumenta al
doble, ¿qué ingreso adicional necesitará Bruce para poder tener su paquete de consumo original?
3. Maureen tiene el mismo ingreso y enfrenta los mismos precios que Sam y Bruce, pero para ella un
vaso de jugo de naranja es un complemento perfecto de un vaso de jugo de manzana. Encuentre el
paquete óptimo de consumo para Maureen. Si el precio del jugo de manzana aumenta al doble, ¿de
cuánto será el ingreso adicional que necesita para tener su paquete de consumo original?
4. En el mercado de limonada hay 10 consumidores potenciales, la curva de demanda de cada uno de
ellos es P = 101 – 10Qi, donde P es precio en dólares por vaso y Qi es la cantidad de vasos que deman-
da semanalmente el consumidor i-ésimo. Emplee el álgebra para encontrar la curva de demanda del
mercado. Trace una curva de demanda individual y la curva de demanda del mercado. ¿Cuál es la
cantidad demandada por un consumidor y por el mercado como un todo al precio P = 1 dólar/vaso?
5. a) Dada la curva de demanda P = 60 – 0.5Q, encuentre la elasticidad en P = 10.
b) Si la curva de demanda se desplaza en forma paralela hacia la derecha, ¿qué pasa con la elasticidad
en P = 10?
6. Considere la curva de demanda Q = 100 – 50P.
a) Trace la curva de demanda e indique qué porción de ella es elástica, que porción es inelástica y
que porción es unitariamente elástica.
b) Sin hacer ningún cálculo, diga en qué punto de la curva se maximizan los gastos en bienes y
después explique las razones de su respuesta.
7. Suponga que la demanda por cruzar el puente Golden Gate está dada por Q = 10 000 – 1 000P.
a) Si el peaje es P = 3, ¿cuál es la recaudación?
b) ¿Cuál es la elasticidad precio de la demanda en este punto?
c) ¿Podrían las autoridades del puente aumentar sus ingresos modificando el precio?
d) Un servicio de transbordadores que compite con el puente Golden Gate empezó a utilizar aero-
deslizadores que hacen el transporte mucho más cómodo. ¿Cómo afectará esto a la elasticidad de
la demanda por el uso del puente?
8. Se piensa que el gasto de los consumidores en seguridad tiene una elasticidad ingreso positiva. Por
ejemplo, a medida que aumenta el ingreso, la gente tiende a comprar automóviles más seguros (más
grandes y con bolsas laterales de aire), hace más viajes en avión que en coche, se hace más revisiones
de salud y recibe más atención médica por cualquier problema que revelen los exámenes. ¿La seguri-
dad es un lujo o un bien necesario?
9. Los profesores Adams y Brown constituyen el total de la demanda del mercado de investigadores
asistentes en el departamento de economía durante el verano. Si la curva de demanda del profesor

04_CHAPTER 4.indd 128 3/6/09 8:23:06 PM


problemas 129

Adams es P = 50 – 2QA y la curva de demanda del profesor Brown es P = 50 – QB donde QA y QB son


las horas demandadas por Adams y Brown, respectivamente, ¿cuál es la demanda del mercado de
horas de investigación en el departamento de economía?
10. Suponga que a 400 dólares la demanda de boletos para volar de Ithaca, Nueva York, a Los Ángeles,
California, es de 300 boletos. Cuando el precio se incrementa 600 dólares la demanda es de 280 bole-
tos. Si se supone que la demanda de boletos es lineal, encuentre la elasticidad precio correspondiente
a los pares cantidad-precio (300, 400) y (280, 600).
11. La curva de demanda mensual de mercado para las calculadoras entre los estudiantes de ingeniería
está dada por P = 100 – Q, donde P es el precio en dólares por calculadora y Q es la cantidad de
calculadoras compradas por mes. Si el precio es de 30 dólares, ¿cuál será el ingreso mensual de los
fabricantes de calculadoras? Encuentre la elasticidad precio de la demanda de las calculadoras. ¿Qué
pueden hacer los fabricantes de calculadoras para aumentar su ingreso?
12. ¿Qué precio maximiza el gasto total a lo largo de la curva de demanda P = 27 – Q2?
13. La curva de demanda diaria de un vendedor de hot dogs es Q = 1 800 – 15P, donde P es el precio en
centavos y Q la cantidad vendida por día.
a) Si sus ventas han sido de 300 hot dogs por día, ¿de cuánto han sido sus ingresos?
b) ¿Cuál es la elasticidad precio de la demanda de hot dogs?
c) El vendedor desea incrementar su ingreso, ¿deberá subir o bajar el precio de sus hot dogs?
d) ¿A qué preció logrará el vendedor obtener un ingreso máximo?
14. Ordene de menor a mayor los valores absolutos de la elasticidad precio de la demanda en los puntos
A, B, C, D y E de las tres curvas de demanda que se observan a continuación.
P
P2

A
P1
E B
D

Q
Q1 Q2

15. Trace las curvas de Engel correspondientes a los siguientes bienes: alimento, vacaciones en Hawai,
nueces de anacardo, zapatillas deportivas marca Kmart (4.99 dólares/par).
16. En los siguientes pares de artículos, ¿es positiva o negativa la elasticidad precio cruzada de la demanda?
a) Raquetas de tenis y pelotas de tenis.
b) Mantequilla de cacahuate y gelatina.
c) Hot dogs y hamburguesas.
*17. En 2001 el precio de X era 3 dólares y se vendían 400 unidades. En ese mismo año el precio de un bien
relacionado con éste, Y, era 10 dólares y se vendían 200 unidades. En 2002 X continuaba ofreciéndose
en 3 dólares pero sólo se vendieron 300 unidades, el precio de Y subió a 12 dólares y se vendieron
sólo 150 unidades. Si todo lo demás permanece constante y se supone que la demanda de X es una
función lineal del precio de Y, ¿cuál es la elasticidad de precio cruzada la demanda de X con respecto
a Y en 2001?
*18. Para Smith no hay diferencia entre arroz y trigo y gasta todo su presupuesto para alimento, que es
de 24 dólares/semana, en estos dos granos. Si el arroz tiene un precio de 3 dólares/lb, trace la curva
consumo-precio de Smith para trigo y la correspondiente curva de demanda.

* Los problemas marcados con un asterisco (*) son más difíciles.

04_CHAPTER 4.indd 129 3/6/09 8:23:07 PM


130 Capítulo 4 demanda individual y del merCado

*19. Repita el problema anterior pero ahora suponga que arroz y trigo son complementos perfectos uno
a uno.
*20. Suponga que su cafetería hace la oferta siguiente: las personas que traigan su propio envase con un
cuarto de litro de leche podrán obtener una taza de capuchino a sólo 1.50 y no al precio normal de
2.50 dólares. Los envases con un cuarto de litro de leche pueden adquirirse en la tienda cercana a
sólo 0.50 centavos. Con esta oferta, la cantidad de capuchinos vendida aumentó 60 por ciento y en la
tienda cercana el ingreso por la venta de leche se duplicó.
a) Verdadero o falso: a pesar de la complicación para el cliente de llevar su propia leche, el valor
absoluto de la elasticidad precio de la demanda de capuchino es 3. Explique.
b) Verdadero o falso: se concluye que la demanda de leche en la tienda cercana es elástica con respecto
al precio. Explique.

RESpUEStaS a loS EjERcicioS dEl capítUlo


4.1. En su presupuesto original, B0, Paula consume el paquete A. En el nuevo presupuesto, B1, consume
el paquete D. (Decir que consume 1.5 pares de fijadores por año, significa que consume 3 pares de
fijadores cada dos años.) El efecto sustitución del aumento de precio (el paso de A a C) es cero.

Esquíes (pares/año)
9

8
B’
7

6
C
5
A
4 B1 I0

3 I1
D
2
B0
1

Fijadores (pares/año)
0 1 2 3 4 5 6 7 8

4.2. Todos los efectos, ingreso, sustitución y total son cero porque la variación de precio no altera el
paquete óptimo de consumo de Pam.

Té (tazas/semana)
12

10
B0
I0
8 B1

A
Café (tazas/semana)
0 12

* Los problemas marcados con un asterisco (*) son más difíciles.

04_CHAPTER 4.indd 130 3/6/09 8:23:09 PM


respuestas a los eJerCiCios del Capítulo 131

4.3. Las fórmulas para D1 y D2 son P = 16 – 2Qi y P = 8 – 2Qi, respectivamente. Para la región en la que
0 ≤ P ≤ 8, se tiene Q1 = 8 – (P/2) y Q2 = 4 – (P/2). Al sumar se obtiene Q1 + Q2 = Q = 12 – P, para 0 ≤
P ≤ 8. Para 8 < P ≤ 16, la curva de demanda del mercado es la misma que D1, es decir, P = 16 – 2Qi.
4.4. Primero se necesita reordenar la curva de demanda representativa del consumidor P = 120 – 60Qi,
de manera que se tenga la cantidad sola en un lado:
Q1 = 2 − 601 P .

Después se multiplica por el número de consumidores, n = 30,


Q = nQi = 30Qi = 30( 2 − 601 P ) = 60 − 12 P .

Precio ($/unidad)
120

D1

Cantidad (unidades/semana)
0 2 4 6 8 10 20

1
Por último se reordena la curva de demanda del mercado Q = 60 − 2 P de manera que se tenga el
precio sólo en un lado, P = 120 – 2Q, para volver a la forma pendiente-ordenada al origen.
4.5. Como la pendiente de la curva de demanda es –1, se tiene, e = –P/Q. En P = 24, Q = 8, y de esta
manera ε = −P / Q = − 24 8 = −3 .

Precio ($/unidad)
32

A
24

Cantidad (cantidad/semana)
0 8 32

4.6. Cuando P = 4 dólares/yd2 la elasticidad es 1–3, de manera que una disminución del precio hará que
disminuya el gasto total. En P = 4, el gasto total es 48 dólares/semana, más que los 39 dólares/sema-
na de gasto total en P = 3.
4.7. Los sustitutos, como a, b y e, tienen elasticidad precio cruzada positiva (un aumento del precio de
un bien eleva la cantidad demandada del otro). Complementos como c y d, tienen elasticidad precio
cruzada negativa (un aumento del precio de un bien hace que se reduzca la demanda del otro).

04_CHAPTER 4.indd 131 3/6/09 8:23:13 PM


04_CHAPTER 4.indd 132 3/6/09 8:23:13 PM
a p é N d i c E

4
TEMAS ADICIONALES SOBRE
LA TEORÍA DE LA DEMANDA

LAS CURVAS DE DEMANDA


DE ELASTIICIDAD CONSTANTE
Hasta ahora las curvas de demanda han sido lineales, por lo cual, como se observó, la elasticidad
precio disminuye a medida que se desciende por la curva. Sin embargo, no todas tienen esta propie-
dad; al contrario, hay otras en las que la elasticidad precio puede permanecer constante o incluso
aumentar a medida que se desciende por ellas. Curva de demanda con elasticidad constante es el nombre
que se da a aquellas en las que la elasticidad no varía con el precio y la cantidad. Mientras las curvas
de demanda lineales son de la forma P = a – bQ, las curvas de demanda de elasticidad constante son de
la forma
k
P= (A.4.1)
Q1 ε
donde k y ε son números positivos, valores específicos que determinan la forma y la posición exacta
de la curva.1 En la figura A.4.1 se presenta un ejemplo en el que k = 2 y ε = 1.
Se examinarán algunos puntos de la curva de la figura A.4.1 para verificar que, en efecto, tienen
la misma elasticidad precio. Considere primero el punto P = 2, Q = 1 y calcule la elasticidad precio
1
Mediante la definición formal de elasticidad es fácil demostrar que la elasticidad para cualquier par precio-cantidad sobre esta curva
de demanda es –ε:

P dQ( P ) k Q1 ε 1
= = −ε.
Q dP 2 (−1 ε)kQ−1 ε−1

133

04_CHAPTER 4.indd 133 3/6/09 8:23:15 PM


134 Capítulo 4 apÉndiCe 4: temas adiCionales sobre la teoría de la demanda

FIGURA A.4.1
P
Curvas de demanda con
elasticidad constante
mientras que a lo largo
de una curva de demanda 5
lineal la elasticidad precio
disminuye a medida que 4
la cantidad aumenta,
permanece igual a lo largo 3
de una curva de demanda
con elasticidad constante.
2

1 2
P=
Q
Q
0 1 2 3 4 5 6

como el producto del cociente P/Q multiplicado por la inversa de la pendiente de la curva de deman-
da. Para calcular la pendiente de la curva de demanda se necesita calcular el ∆Q que, como respuesta
a un ∆P muy pequeño se obtiene en la cercanía del punto (1, 2). Suponga, por ejemplo que se emplea
una variación de precio de +0.001. Si P = 2.001, se despeja Q de la curva de demanda (es decir de la
ecuación P = 2/Q) y se obtiene Q = 2/2.001 = 0.9995. Por lo tanto, ∆Q = 0.9995 – 1 = –0.0005, con
lo que la pendiente de la curva de demanda en (1, 2) será ∆P/ ∆Q, es decir 0.001/) (–0.0005) = –2. La
inversa de la pendiente es – –12 y, por lo tanto, la elasticidad precio es 2(− 12 ) = −1 .
Considere ahora el punto (2, 1). Una vez más se emplea un ∆P = 0.001, se obtiene una nueva Q,
que es 2/1.001 = 1.998, es decir un ∆Q = –0.002. Por lo tanto la pendiente de la curva de demanda
en (2, 1) es 0.001/(–0.002) = − 12 y su inversa es –2. La elasticidad precio en (2, 1) es por lo tanto
( 12 )(−2), es decir –1.

EJERCICIO A.4.1
En la curva de demanda de la figura A.4.1 verifique que en todos los puntos la elasticidad precio
es igual a –1. [En la respuesta que se da al final del capítulo se emplean los puntos (0.5, 4) y (4,
0.5).]

La curva de demanda dada por P = k/Q es un caso especial de curva de demanda con elasticidad
constante llamada curva de demanda de gasto constante. En todos los puntos a lo largo de esta curva,
el gasto total está dado por el producto PQ = k, donde k es de nuevo una constante positiva. Por lo
tanto, a diferencia del caso de la curva de demanda lineal, aquí la gente gasta exactamente la misma
cantidad sin importar que el precio sea alto o bajo. Por ejemplo, una persona que todos los meses
gasta todo su presupuesto en discos compactos, tendrá una curva de demanda de gasto constante en
discos compactos. La constante k será igual al monto de su presupuesto.
A medida que se desciende a lo largo de una curva de demanda con elasticidad constante (P =
k/Q1/ε) la disminución del cociente P/Q es exactamente compensada por el aumento de la inver-
sa de la pendiente. Una curva de demanda con elasticidad constante en la que e > 1 tiene la propiedad
de que una disminución de precio incrementará siempre el gasto total. Por el contrario, en el caso de
que e < 1, una disminución del precio reducirá el gasto total.

EJERCICIO A.4.2
¿Qué pasa con el gasto total cuando el precio baja de 4 a 3 en la curva dada por P = 4/Q1/2?

04_CHAPTER 4.indd 134 3/6/09 8:23:18 PM


Curva de demanda de ingreso Compensado 135

MÉTODO COCIENTE-SEGMENTO
A la elasticidad precio en un punto dado de una curva de demanda lineal puede dársele otra útil
interpretación geométrica. Suponga que la curva de demanda se divide en dos segmentos, AC y CE,
como se muestra en la figura A.4.2. La elasticidad precio de la demanda (en valor absoluto) en el
punto C, representada por εc , será igual al cociente de estos dos segmentos.2
CE
. εC = (A.4.2)
AC
La ecuación A.4.2 se llama cociente-segmento para el cálculo de la elasticidad precio de la de-
manda.

FIGURA A.4.2
P Método cociente-
A CE segmento
ε
AC el valor absoluto de la
elasticidad precio en
cualquier punto es el
F C cociente de los dos
segmentos de la curva
de demanda a partir de
dicho punto. en el punto
C el valor absoluto de la
elasticidad precio de la
0 G E
Q demanda es igual a CE/AC.

Saber que la elasticidad precio de la demanda en cualquier punto de la curva de demanda lineal
es el cociente entre dos segmentos de línea simplifica enormemente la tarea de dar datos cuantitativos
acerca de la elasticidad precio. Considere la curva de demanda que se presenta en la gráfica superior de
la figura A.4.3. En el punto medio de esa curva de demanda (punto M), por ejemplo, puede observarse
a simple vista que la elasticidad precio es –1. A un cuarto hacia abajo de la curva de demanda (punto K
en la figura A.4.3), la elasticidad es –3; a tres cuartos hacia abajo de la curva de demanda (punto L en la
figura A.4.3), la elasticidad es – 1–3 ; etc. La gráfica inferior de la figura A.4.3 resume la relación entre la
posición sobre una curva de demanda lineal y la elasticidad precio de la demanda.

CURVA DE DEMANDA DE INGRESO COMPENSADO


Las curvas de demanda individual que se han visto en este capítulo toman en cuenta los efectos susti-
tución e ingreso de las variaciones de precio. En muchas aplicaciones estas curvas de demanda serán
la herramienta relevante para predecir la respuesta de la gente a una variación de precio. Suponga,
por ejemplo, que el precio de la gasolina sube debido a un nuevo acuerdo de la OPEP. Tal incremento
de precio tendrá efectos tanto sustitución como ingreso y la curva de demanda individual descrita
antes será el instrumento apropiado para predecir la respuesta de la gente.
Sin embargo, en otras situaciones esta curva de demanda no será la herramienta adecuada. Du-
rante la administración Carter, por ejemplo, hubo una propuesta que consistía en gravar el petróleo
del exterior y amortiguar la carga de este impuesto reduciendo simultáneamente el impuesto sobre

2
Para ver por qué es así, se puede aplicar la geometría elemental. Primero, observe que la inversa de la pendiente de la curva de la
demanda en la figura A.4.2 es la relación GE/GC y que la relación entre precio y cantidad en el punto C es GC/FC. Al multiplicar
ambas se obtiene εc = (GE / GC )(GC / FC ) = GE / FC. Ahora observe que los triángulos AFC y CGE son similares, lo cual significa
que las relaciones entre sus lados correspondientes deben ser iguales. En particular, significa que la relación GE/FC, la cual como se
acaba de ver es igual a la elasticidad precio de la demanda en el punto C, debe ser igual a la relación CE/AC. Y éste, por supuesto, es
el resultado que se deseaba establecer.

04_CHAPTER 4.indd 135 3/6/09 8:23:21 PM


136 Capítulo 4 apÉndiCe 4: temas adiCionales sobre la teoría de la demanda

FIGURA A.4.3
P
Elasticidad en distintos
puntos a lo largo de 10 La demanda es elástica ( ε =3)
una curva de demanda 8
K La demanda es unitariamente elástica ( ε = 1)
lineal 6
si se emplea el método de M La demanda es inelástica ( ε = 13 )
4
cociente-segmento, puede L
calcularse la elasticidad 2
precio en los puntos K, M Q
y L (gráfica superior) en un 0 2 4 6 8
instante.
η

2 4 6 8
Q
– 13

–1

–2

–3

–4

el salario. Un impuesto sobre el petróleo, por sí mismo, hubiera aumentado el precio del petróleo y
producido los correspondientes efectos ingreso y sustitución. Pero el efecto de la reducción simultá-
nea del impuesto sobre el salario, dicho grosso modo, hubiera eliminado el efecto ingreso del aumento
de precio. El impuesto sale de un bolsillo y es puesto inmediatamente en otro.
curva de demanda de Para analizar el efecto de una política de este tipo, se debe usar la curva de demanda de ingreso
ingreso compensado la curva compensado, que proporciona las cantidades que comprarían los consumidores si fueran del todo
de demanda que indica cuánto compensados por los efectos ingreso de las variaciones del precio. Para generar esta curva para un
comprarían los consumidores individuo, simplemente se elimina el efecto ingreso del efecto total de las variaciones de precio. La
a cada precio si fueran gráfica superior de la figura A.4.4 muestra los efectos ingreso y sustitución de un aumento del precio
totalmente compensados de la vivienda de 6 dólares/yd2 a 12 dólares/yd2 para un consumidor cuyo ingreso semanal es 120 dó-
por los efectos ingreso de la lares. La curva ordinaria de demanda de vivienda para un individuo representada aquí asocia 6 dóla-
variación de precios.
res con 10 yd2/semana y 12 dólares con 6yd2/semana. La curva de demanda de ingreso compensado
siempre se construye con relación a un punto fijo de referencia, el precio actual. Igual que la curva de
demanda ordinaria, esta curva también asocia 10 yd2/semana con 6 dólares. Pero con el precio de 12
dólares no asocia 6 yd2/semana sino 7 yd2/semana, que es la cantidad de vivienda que el consumidor
hubiera comprado a 12 dólares/yd2 si se le hubiera dado suficiente ingreso para permanecer en la
curva de indiferencia original, I0.
Los individuos cuyas respuestas se describen en la figura A.4.4 que ven a la vivienda como un
bien normal, son aquellos cuya cantidad demandada aumenta a medida que se incrementa el ingreso.
Cuando se trata de un bien normal, la curva de demanda de ingreso compensado será necesariamen-
te más inclinada que la curva de demanda ordinaria. Sin embargo, en el caso de un bien inferior, la
curva de demanda ordinaria siempre será la más inclinada de las dos. En la figura A.4.5 se representa
la relación entre las dos curvas de demanda en el caso de un bien inferior.
En la práctica, la distinción entre las curvas de demanda ordinaria y las curvas de demanda de
ingreso compensado resulta ser de particular importancia para cuestiones relacionadas con la política
fiscal. En el caso de la propuesta del impuesto a la gasolina de Jimmy Carter, había una previsión explí-
cita para que lo recaudado por ese impuesto fuera devuelto a la gente que lo pagaba. Pero aun sin una
previsión de este tipo el impacto práctico de un nuevo impuesto será aproximadamente el mismo.

04_CHAPTER 4.indd 136 3/6/09 8:23:22 PM


Curva de demanda de ingreso Compensado 137

FIGURA A.4.4
Y($/semana) Curva de demanda
ordinaria versus curva
de demanda de ingreso
compensado para un
B’ bien normal
en la curva de demanda
120 ordinaria se representan
los efectos sustitución e
B1 ingreso de una variación
C
90 de precio. la curva de
demanda de ingreso
60 A compensado siempre será
48 más inclinada que la curva
D de demanda ordinaria.
I0
20 I1 B0

Vivienda (yd 2/semana)


0 56 7 10 15 20

Precio de la vivienda
(yd2/semana)

12

Curva de demanda no compensada (ordinaria)


Curva de demanda de ingreso compensado
Vivienda (yd 2/semana)
0 56 7 10

FIGURA A.4.5
Curva de demanda
Precio de la hamburguesa ($/lb) ordinaria frente a curva
de demanda de ingreso
compensado para un
bien inferior
Cuando se trata de un bien
inferior el efecto ingreso
contrarresta el efecto
Curva de demanda de ingreso compensado sustitución. en el caso de
un bien inferior, la curva
de demanda de ingreso
Curva de demanda ordinaria
compensado, en la que se
Hamburguesa (lb/semana) omite el efecto ingreso,
es por lo tanto menos
inclinada.

Después de todo, cuando el gobierno recauda más ingresos de una fuente necesitará recaudar
menos de otras. El resultado final es que la curva de demanda relevante para estudiar los efectos de
un impuesto sobre un bien es la curva de demanda de ingreso compensado.
En la práctica, decidir entre los dos tipos de curvas es relevante sólo cuando se trata de bienes
para los cuales los efectos ingreso son mayores que los correspondientes efectos sustitución. Para
que el efecto ingreso de una variación del precio de un determinado bien sea grande, es necesario
(pero no suficiente) que ese bien tenga una participación significativa en el gasto total. Sin embargo,
muchos de los bienes y servicios individuales que compra la gente representan sólo una fracción muy

04_CHAPTER 4.indd 137 3/6/09 8:23:24 PM


138 Capítulo 4 apÉndiCe 4: temas adiCionales sobre la teoría de la demanda

pequeña de su gasto total. En consecuencia, para tales bienes la distinción entre estos dos tipos de
curvas de demanda no tendrá importancia. Incluso en el caso de bienes con una participación grande
en el presupuesto, algunas veces el efecto ingreso de una variación de precio será pequeño. (El bien
puede encontrarse en la frontera entre un bien normal y un bien inferior.) En el caso de tales bienes,
también la distinción entre curva de demanda ordinaria y curva de demanda de ingreso compensado
tendrá poca utilidad práctica.

RESpUEStaS a loS EjERcicioS dEl apéNdicE


A.4.1. Considere primero el punto (0.5, 4). Si de nuevo se hace ∆P = 0.001, de manera que la nueva P
sea 4.001, la Q que se obtiene es 2/4.001 = 0.499875, lo que significa que ∆Q es –0.000125. Por
lo tanto la elasticidad precio es igual a (4/0.5)(–0.000125/0.001) = –1. Ahora considere el punto
(4, 0.5). Si otra vez se hace ∆P = 0.001, de manera que la nueva P sea 0.501, la Q que se obtiene
es 2/0.501 = 3.992, lo que significa que ∆Q es 0.008. Por lo tanto la elasticidad precio es igual a
(0.5/4)(–0.008/0.001) = –1.
A.4.2. Para P = 4, se tiene 4 = 4 / Q , lo que da Q = 1, de manera que el gasto total es 4(1) = 4. Para
P = 3, se tiene 3 = 4 / Q , lo que da Q = 16 — 16 16
9 , de tal forma que el gasto total es ( 3)( 9 ) = ( 3 ) .
Cuando e = 2 el gasto total aumenta cuando el precio disminuye.

04_CHAPTER 4.indd 138 3/6/09 8:23:26 PM


Capítulo

5
AplicAciones de lAs teoríAs
de lA elección rAcionAl
y de lA demAndA

E
n el año académico 2008-2009 el pago por inscripción y colegiatura en la Universidad de
Cornell era superior a 35 000 dólares. La universidad tenía una política especial según la
cual los hijos de los profesores que asistieran a la universidad sólo tenían que pagar la ins-
cripción, que era de aproximadamente 2 500 dólares/año. Por supuesto, esta política era un fuerte
incentivo para que los hijos de los profesores de la universidad ingresaran en Cornell.
El comité de profesores argumentó durante muchos años que la universidad debería extender
este beneficio a los hijos de profesores que decidieran asistir a otra universidad. La respuesta tradi-
cional de la institución era que no estaba en condiciones de hacer tal oferta. Sin embargo, debido a la
insistencia de los economistas del comité, la administración dio un paso tentativo en esta dirección
ofreciendo pagar un tercio de la colegiatura en otras universidades. Para su sorpresa, esta nueva
política no sólo no le costó ni un centavo a la universidad, sino que le ahorró una buena cantidad
debido a que el número de hijos de profesores que asistían a Cornell disminuyó de manera significa-
tiva en cuanto la política entró en vigor. Esto abrió una cantidad equivalente de lugares para nuevos
estudiantes que fueron ocupados por alumnos que sí pagaban colegiatura, Cornell salió ganando.
También ganaron las familias de los profesores que recibieron la nueva ayuda financiera y los nuevos
estudiantes que de otra manera no hubieran podido tener un sitio en Cornell. La universidad no se
había dado cuenta del costo de oportunidad de asignar plazas a los hijos de sus profesores y de que
muchos de éstos quedarían vacantes con la nueva oferta.
139

05_CHAPTER 5.indd 139 3/6/09 7:53:19 PM


140 Capítulo 5 apliCaCiones de las teorías de la eleCCión raCional y de la demanda

VISIÓN PRELIMINAR DEL CAPÍTULO


La política de Cornell con las colegiaturas proporciona otra lección de que los precios afectan el
comportamiento. En este capítulo se consideran aplicaciones y ejemplos de las teorías de la elección
racional y de la demanda que se vieron en los capítulos 3 y 4. Se empieza con dos ejemplos —un
impuesto a la gasolina y los vales escolares— que ilustran cómo el modelo de la elección racional
puede iluminar importantes cuestiones de política económica. Después se considera el concepto del
excedente del consumidor, una medida de cuánto se beneficia al consumidor al poder comprar cierto
producto a un determinado precio. Y se verá cómo el modelo de la elección racional puede emplear-
se para examinar cómo afectan al bienestar económico las variaciones de los precios y del ingreso.
A continuación en la agenda están dos casos de estudio que ilustran el papel de la elasticidad-pre-
cio en el análisis político. Se examina el efecto de un aumento de tarifa realizado por la Metropolitan
Atlanta Rapid Transit Authority y el efecto de un impuesto sobre el consumo de alcohol por parte de
los bebedores habituales.
Por último se considera cómo puede adaptarse el modelo de elección racional a casos que tie-
nen consecuencias futuras.

USO DEL MODELO DE LA ELECCIÓN RACIONAL


PARA RESPONDER A CUESTIONES POLÍTICAS
Muchas políticas del gobierno no sólo afectan el ingreso de las personas sino también los precios
que pagan. Algunas veces estos efectos son el objetivo último de las políticas públicas, pero en otras
ocasiones son consecuencias no intencionadas de políticas que tienen otros fines. En cualquier caso,
tanto el sentido común como el modelo de la elección racional indican que es de esperarse que las
variaciones en ingreso y precios modifiquen la manera en que los consumidores gastan su dinero.
Y, como se verá, el modelo de la elección racional puede proporcionar una comprensión crucial no
siempre disponible para los analistas políticos cuando sólo cuentan con el sentido común.

APLICACIÓN: UN IMPUESTO A LA GASOLINA


Y UNA POLÍTICA DE REEMBOLSO
Como un oportuno e interesante caso histórico, considere la propuesta política hecha durante la
administración del ex presidente Jimmy Carter de aplicar un impuesto a la gasolina como ayuda para
limitar su demanda y hacer a Estados Unidos menos dependiente de las fuentes extranjeras de petró-
leo. Una objeción inmediata a esta propuesta fue que el consecuente aumento del precio de la gasoli-
na sería una desventaja económica para los pobres. Anticipándose a esta objeción, la administración
de Carter propuso disminuirles la carga a los pobres empleando lo recaudado con este impuesto en
una reducción de las cotizaciones a la seguridad social de los trabajadores. Los críticos respondieron
de inmediato que devolver de esta manera lo recaudado anularía su propósito. Los críticos pensaron
que si el impuesto a la gasolina se devolvía en forma de sueldo a los consumidores, éstos comprarían
exactamente la misma cantidad de gasolina que antes. Pero como se verá, estos críticos necesitaban
con urgencia conocer los principios básicos de la elección racional.
Considere un ejemplo ilustrativo. Suponga que el precio de la gasolina es 1.00 dólar/galón, más
o menos el precio que tenía cuando Carter hizo su propuesta, y que se le aplica un impuesto de
0.50, lo cual da como resultado un aumento de 0.50 de dólar en el precio del combustible.1 Suponga
también que a un consumidor típico se le da como reembolso una cantidad fija que resulta ser exac-
tamente igual a la que paga por el impuesto a la gasolina. (Aquí el término “cantidad fija” significa
que el reembolso no varía con la cantidad de gasolina que consume.) Verdadero o falso: esta política no
tendrá efecto sobre la cantidad de gasolina que compre el consumidor. Los críticos de la propuesta

1
Recuerde que en el capítulo 2 se vio que el aumento en el precio de equilibrio será idéntico al impuesto cuando la curva de oferta
para la gasolina sea perfectamente horizontal, una suposición que se hace aquí para simplificar.

05_CHAPTER 5.indd 140 3/6/09 7:53:19 PM


uso del modelo de la eleCCión raCional para responder a Cuestiones polítiCas 141

de Carter contestarán, por supuesto, “verdadero”, pero una vez traducido al conocido contexto de la
elección racional se verá que la respuesta correcta es “falso”.
Para analizar esta combinación impuesto-reembolso, se considerará un consumidor cuyo ingre-
so es de 150 dólares/semana. La restricción presupuestaria de este consumidor antes del impuesto
se muestra cómo B1 en la figura 5.1.2 En B1, el consumidor elige C, que contiene 58 gal/semana de
gasolina. Su restricción presupuestaria, con un precio de la gasolina de 1.50 dls/gal, será B2 si no
obtiene ningún reembolso. En B2 consumirá A, que contiene únicamente 30 gal/semana de gasolina.
Pero ¿cómo se encuentra la restricción presupuestaria que corresponde a un reembolso igual a la
cantidad que se ha recabado de él en impuestos a la gasolina?

FIGURA 5.1
Otros bienes (dólares/semana)
Impuesto a la gasolina
y reembolso
el impuesto hace girar la
restricción presupuestaria
D original de B1 a B2. el
168
R reembolso desplaza
150 ICC B2 hacia afuera a B3. el
reembolso no altera
el hecho de que este
impuesto hace que la
A gasolina sea 50% más cara
C
en relación con los demás
bienes. el consumidor que
I1 se muestra en el diagrama
I2 I3 responde consumiendo 22
galones/semana menos de
gasolina.
B2 B1
B3
Gasolina
(galones/semana)
30 36 58 100 112 150

El primer paso es observar que, para cualquier cantidad de gasolina consumida, la distancia ver-
tical entre las restricciones presupuestaria B1 y B2 corresponde a la cantidad total de impuesto pagado
para esa cantidad de gasolina. Así, en 1 gal/semana de gasolina, la distancia vertical entre B1 y B2 será
0.50 de dólares; en 2 gal/semana, será 1 dólar, y así sucesivamente.
El siguiente paso es seguir la variación en el consumo en función del tamaño del reembolso.
Para esto, observe que el reembolso es como otra fuente de ingreso, de manera que, en realidad,
lo que se quiere hacer es seguir la respuesta del consumidor a la variación en el ingreso. Como se
vio en el capítulo 4, la herramienta apropiada para esto es la curva consumo-ingreso o CCI. En
consecuencia, se construye la CCI que pasa a través del paquete A, como se muestra en la figura 5.1.
Recuerde que, como se vio en el capítulo 4, la CCI en cuestión es el lugar geométrico de los puntos
de tangencia generados por una serie de restricciones presupuestaria paralelas a B2. La CCI muestra
cómo poco a poco aumenta el consumo de gasolina y de otros bienes a medida que el reembolso
crece en forma gradual. El objetivo es que continúe incrementándose hasta que sea suficiente para
pagar el impuesto de la gasolina que el consumidor elige comprar.
¿Cuál es el reembolso requerido? Observe el paquete D, el punto donde la CCI, que pasa a través
de A, interseca la restricción presupuestaria original B1. D es el paquete de equilibrio en la restricción
presupuestaria B3, donde el precio de la gasolina es 1.50 dls/gal y donde el consumidor tiene como
ingreso (150 dólares + R)/semana = 168 dls/semana. Observe que el consumo de gasolina en D es
de 36 gal/semana. Esto significa que si al consumidor se le da un reembolso de R = 18 dólares/se-
mana, consumirá el paquete D y pagará 18 dólares/semana en impuestos sobre la gasolina. (Observe

2
La ecuación para B1 es Y = 150 – G, para B2 es Y = 150 – 1.5G y para B3 es Y = 168 – 1.5G, donde G es gasolina (gal/semana) y Y son
todos los demás bienes ($/semana).

05_CHAPTER 5.indd 141 3/6/09 7:53:20 PM


142 Capítulo 5 apliCaCiones de las teorías de la eleCCión raCional y de la demanda

también que 18 dólares es la distancia vertical entre B1 y B2 cuando el consumo de gasolina es 36


gal/semana.)
El punto crítico es que D se encuentra bastante a la izquierda del paquete original C, lo que signi-
fica que, a pesar del reembolso, el consumidor reduce de manera sustancial su consumo de gasolina.
Si la gasolina es un bien normal, el efecto del reembolso es contrarrestar en forma parcial el efecto
ingreso del aumento de precio. No altera en absoluto el efecto sustitución.
Al final la propuesta del impuesto y del reembolso de la administración de Carter nunca pudo
ponerse en marcha debido en gran parte a las objeciones de los críticos que carecían de todo conoci-
miento económico para entenderla. El resultado es que Estados Unidos sigue siendo peligrosamente
dependiente del petróleo del extranjero. En efecto, con la creciente demanda global de petróleo, que
crece a tasas récord, debido en gran medida al rápido crecimiento económico de China e India, y
dada la inestabilidad económica en el Medio Oriente, lugar de origen de las reservas de petróleo más
grandes del mundo, imponer un impuesto a la gasolina es hoy una idea más convincente de lo que
fue cuando Carter hizo la propuesta hace ya más de 25 años.

APLICACIÓN: VALES ESCOLARES


En los últimos años ha habido una gran discusión sobre la necesidad de mejorar la calidad de la edu-
cación primaria y secundaria en Estados Unidos. Muchos análisis políticos han recomendado que se
introduzca más competencia en el mercado de servicios educativos. Con este fin se ha propuesto que
a cada familia se le dé un vale para educación que pueda usar en la escuela que prefiera.
Estas propuestas contrastan con el actual sistema en la mayoría de los distritos escolares, de
acuerdo con el cual a todas las familias se les pide que paguen impuestos, con lo que tienen derecho
a enseñanza “gratuita” en la escuela pública más cercana. Con este sistema las familias que optan
por una escuela privada no obtienen un reembolso de los impuestos que sufraga el gasto público en
educación. Los críticos del sistema actual se quejan de que debido a las colegiaturas que se cobran en
las escuelas privadas, éstas no pueden competir de manera efectiva con las escuelas públicas, con lo
cual estas últimas están poco presionadas para mejorar su desempeño.
Si se prescinde de la cuestión de si el sistema de vales puede conducir a una educación de mejor
calidad, se puede examinar el efecto que tendrían los vales sobre la cantidad de recursos dedicados a
la educación. Se puede emplear el modelo de la elección racional para examinar la elección educativa
con la que se encuentra una familia representativa.
Para simplificar, suponga que la cantidad de educación, medida en términos de horas de cla-
se por año, es fija y que cuando se habla de gastar más en educación no se quiere dar a entender
comprar más horas de educación, sino comprar educación de mejor calidad. Suponga que el actual
sistema de impuestos grava a cada familia con un impuesto Pe por una unidad de educación pública,
sea ésta utilizada o no, donde “una unidad” se define como el equivalente a un año de educación de
la calidad que hoy se ofrece en las escuelas públicas. Si la familia no manda a sus hijos a la escuela pú-
blica, entonces tiene la opción de comprar una o más unidades de educación en una escuela privada,
también al precio Pe por unidad. Por ejemplo, comprar 1.2 unidades de educación en una escuela
privada significa comprar educación de una calidad 20% superior a la que se ofrece en las escuelas
públicas. Las familias tienen, por ley, la obligación de proporcionar a sus hijos por lo menos una
unidad de educación, sea pública o privada.
Dados estos valores, puede hallarse ahora la restricción presupuestaria para educación y para
otros bienes de una familia típica cuyo ingreso antes de impuestos es Y. Si no hubiera impuestos ni
escuelas públicas, la restricción presupuestaria de esta familia sería la línea indicada como ABD en la
figura 5.2. Pero como cada familia tiene que pagar Pe de impuestos escolares, la intersección vertical
de la restricción de presupuesto no es Y sino Y – Pe. Como hay una unidad “gratis” de educación
pública, la restricción presupuestaria de la familia es horizontal a lo largo de una unidad. Si después la
familia desea comprar más de una unidad de educación, debe sacar a su hijo(a) de la escuela pública y
matricularlo(a) en una escuela privada con un precio adicional de Pe por unidad. Esto explica por qué
la restricción presupuestaria actual baja de manera vertical Pe en una unidad de educación. A partir
de aquí la restricción presupuestaria continúa con pendiente negativa a una tasa de Pe por unidad.
Así, la restricción presupuestaria de una familia, tomando en consideración la educación que desea
comprar, es la indicada por A' BCE en la figura 5.2.

05_CHAPTER 5.indd 142 3/6/09 7:53:20 PM


uso del modelo de la eleCCión raCional para responder a Cuestiones polítiCas 143

FIGURA 5.2
Elección educacional
acorde en el sistema
Otros bienes actual
la familia tiene un ingreso
antes de impuestos de Y, de
donde tiene que pagar Pe en
Y A impuestos escolares. esto
le da derecho a una unidad
A’ B de educación pública sin
Y – Pe
tener que pagar colegiatura.
en lugar de la educación
C pública puede comprar
Y – 2Pe I0 por lo menos una unidad
de educación privada al
E D precio de Pe por unidad.
Cantidad de educación entonces su restricción
1 (Y – Pe)/Pe Y/Pe
presupuestaria será A' BCE y
su paquete óptimo será B,
que contiene una unidad de
educación pública.

Observe en la figura 5.2 que la no linealidad de la restricción presupuestaria hace que para una
familia, como la que se muestra en la figura, una solución de tangencia no sea posible. Para dicha fa-
milia el paquete óptimo es, en efecto, una solución de esquina en la que se elige exactamente una
unidad de educación pública.
Ahora se compararán estos resultados con lo que ocurriría con el sistema de vales. En éste, las
familias también pagan un impuesto escolar Pe y obtienen un vale por una cuantía de Pe que puede
usarse para comprar educación pública o privada. Con el sistema de vales la ley también exige que
las familias proporcionen a sus hijos por lo menos una unidad de educación. Entonces, la restricción
presupuestaria en el sistema de vales estará dada por A' BD en la figura 5.3.

FIGURA 5.3
Otros bienes Elección educacional
en el sistema de vales
a diferencia del sistema
actual, el sistema de vales
A permite a los padres
Y
proporcionar pequeños
incrementos de educación
A’ superiores a una unidad
Y – Pe
B G a un precio de Pe por
H
I1 unidad. la restricción
I0 presupuestaria es ahora
A' BD y la familia que se
D muestra elige ahora el
Cantidad de educación paquete G, que contiene
1 (Y – H)/Pe Y/Pe más de una unidad de
educación.

Compare las figuras 5.2 y 5.3. Observe que la diferencia principal producida por el sistema de
vales es eliminar la discontinuidad en el punto B de la restricción presupuestaria. Los padres ya no
tienen que renunciar a sus impuestos escolares cuando cambian de una escuela pública a una priva-
da; ahora pueden comprar pequeños incrementos de educación mayores a una unidad sin tener que
“pagar doble”. Y en efecto, la familia de la figura 5.3 responde eligiendo el paquete G, que contie-ne
más de una unidad de educación. Este análisis sugiere entonces que cambiar al sistema de vales in-
crementaría el gasto en educación.

05_CHAPTER 5.indd 143 3/6/09 7:53:22 PM


144 Capítulo 5 apliCaCiones de las teorías de la eleCCión raCional y de la demanda

En tiempos de escasez presupuestaria mucha gente se ve tentada a concluir que se debe evitar
cualquier política que requiera recursos adicionales. Para estas personas habrá que hacer algunas
adventencias. Primero, este análisis no toma en cuenta la posibilidad de que la competencia entre
las escuelas pueda hacer que baje el costo por unidad haciendo que sean más eficientes en la pro-
ducción de servicios de educación. Así, mientras los padres pueden elegir comprar más unidades
de educación en el esquema de vales, la competencia puede hacer que el costo por unidad ba-
je, haciendo que sea difícil determinar el efecto neto en el gasto. Segundo, muchos de los recursos
adicionales dedicados a la educación como resultado del sistema de vales vendría directo de los pa-
dres, no del gobierno. Y de ninguna manera está claro que un objetivo de las políticas públicas deba
ser evitar que los padres gasten más en educación y menos en otros bienes. Por último, un análisis
más completo deberá considerar el efecto de la educación adicional sobre la productividad econó-
mica. Después de todo, cualquier aumento en la productividad que sea resultado de la educación
adicional deberá usarse para compensar el costo de producir esa educación. En resumen, el análisis
en este ejemplo se enfoca nada más en una parte de un escenario mucho mayor. Pero es una parte
importante que los políticos no deben descuidar.

EXCEDENTE DEL CONSUMIDOR


Cuando un intercambio se lleva a cabo de manera voluntaria, los economistas suponen que, por lo
general, beneficia a todos los participantes; de otra manera no hubieran tomado parte en el inter-
cambio. Tener una medida del grado en que la gente se beneficia con una transacción suele ser de
excedente del consumidor utilidad. Una medida de este tipo, llamada excedente del consumidor, sirve para evaluar programas
medida en dólares del grado públicos potenciales. Medir los costos, por ejemplo, de construir una nueva carretera, es relativa-
en que un consumidor se mente sencillo. Pero no se puede tomar una decisión inteligente acerca de si construirla o no sin una
beneficia al participar en una estimación confiable del grado en que los consumidores se beneficiarán con ella.
transacción.

USO DE LAS CURVAS DE DEMANDA PARA MEDIR


EL EXCEDENTE DEL CONSUMIDOR
La manera más fácil de medir el excedente del consumidor es empleando la curva de demanda del
consumidor para un producto. En las dos gráficas de la figura 5.4, la línea indicada como D repre-
senta una curva de demanda individual para vivienda, que en el mercado se vende a un precio de 3

Figura 5.4
La curva de demanda P (dólares/yd2) P (dólares/yd2)
mide el excedente del
consumidor 15 15
a) la altura de la curva de 14
demanda correspondiente 13
a cada cantidad mide lo 12
que el consumidor está 11 Excedente del consumidor
dispuesto a pagar por una 10
unidad más de vivienda. 9
esa cantidad menos el
precio de mercado es el
excedente que obtiene D D
el consumidor por
adquirir la última unidad.
b) el excedente total del 3 3
consumidor es el área
sombreada entre la curva
de demanda y el precio de
0 3 6 9 12 15 0 3 6 9 12 15
mercado.
Vivienda (yd2/semana) Vivienda (yd2/semana)

a) b)

05_CHAPTER 5.indd 144 3/6/09 7:53:23 PM


exCedente del Consumidor 145

dólares/yd2. En la gráfica a) observe que lo máximo que está dispuesto a pagar el consumidor por
la primera yarda cuadrada de vivienda son 14 dólares. Como la vivienda tiene un precio de sólo 3
dólares/yd2 esto significa que en su compra de la primera yarda cuadrada por semana el consumidor
obtiene un excedente de 11 dólares. Por la segunda yarda cuadrada de vivienda, lo más que está
dispuesto a pagar son 13 dólares, de manera que su excedente por la compra de esa unidad será
menor, sólo 10 dólares. Su excedente en la compra de la tercera unidad es aún menor, 9 dólares.
Tratándose de vivienda o de cualquier otro bien perfectamente divisible, la altura de la curva de de-
manda individual correspondiente a cada cantidad representa lo máximo que el consumidor pagará
por una unidad adicional de ese bien.3 En este ejemplo, si de ese valor se resta el precio de compra,
que es de 3 dólares/yd2, y se suman las diferencias correspondientes a cada cantidad a partir de 12
yd2/semana, se obtiene aproximadamente el área sombreada que se muestra en la gráfica b). (Si se
emplean incrementos infinitesimales a lo largo del eje horizontal, se obtiene exactamente el área
sombreada.) Ésta representa el excedente que el consumidor individual obtiene en la compra de 12
yd2/semana de vivienda.

La curva de demanda de gasolina de un individuo está dada por P = 10 – Q, donde P es el EJEMPLO 5.1
precio de la gasolina (dólares/gal) y Q es la cantidad que consume (gal/semana). Si el ingreso
semanal de este individuo es de 1 000 dólares y el precio de la gasolina es de 2 dólares/gal,
¿cuánto disminuirá su excedente si una restricción de importación hace que el precio de la
gasolina suba a 3 dólares/gal?

Al precio de 2 dólares/gal, el consumidor sólo consume 8 galones de gasolina por semana, lo que
constituye menos de 2% de su ingreso. Por lo tanto, es posible que el efecto ingreso del aumento de
precio sea insignificante, de manera que se puede usar la aproximación de la curva de demanda para
medir su excedente del consumidor antes y después del aumento de precio. (Vea nota a pie de página
número 3.) La figura 5.5 representa su curva de demanda. El excedente del consumidor al precio de
2 dólares/galón está dado por el área del triángulo AEF en la figura 5.5, CS = ½ (10 – 2) 8 = 32 dóla-
res/semana. Después del aumento de precio, su consumo baja de 8 a 7 gal/semana y su excedente se

FIGURA 5.5
Pérdida en el excedente
del consumidor debida
P (dólares/gal) a un aumento del precio
de la gasolina
A al precio de 2 dólares/gal, el
10
excedente del consumidor
Pérdida en el excedente está dado por el área del
del consumidor triángulo AEF. al precio de
3 dólares/gal, el excedente
C
del consumidor se reduce
D
3 E al área del triángulo ACD. la
2 pérdida en el excedente del
F
consumidor es la diferencia
Gasolina (gal/semana) entre estas dos áreas, que
7 8 10 es el área de la región
sombreada.

3 Estas afirmaciones acerca de “estar dispuesto a pagar” son literalmente verdaderas sólo si la curva de demanda de la que se habla

es de ingreso compensado como la que se vio en el apéndice del capítulo 4 (vea página 137). Si la curva de demanda fuera ordinaria,
del tipo que se ha estado usando, esto indicaría que el consumidor está dispuesto a comprar una unidad al precio de 14 dólares, 2
unidades al precio de 13 dólares, etc. De acuerdo con esto no sería del todo correcto concluir que, habiendo pagado ya 14 dólares
por la primera unidad, el consumidor esté dispuesto a gastar 13 dólares adicionales por la segunda. Si el efecto ingreso de la demanda
de este bien es positivo, el hecho de que el consumidor ahora sea 14 dólares más pobre que antes significa que estará dispuesto a
pagar un poco menos de 13 dólares por la segunda unidad. Pero como para la mayoría de los bienes los efectos ingreso en general
son pequeños, es aceptable la aproximación que mide el excedente del consumidor usando la curva de demanda ordinaria. En un
artículo muy citado, Robert Willig sostiene que el método de la curva de demanda casi siempre da una aproximación aceptable
del verdadero valor de los beneficios del consumidor. Vea R. Willig, “Consumer Surplus without Apology”, en American Economic
Review, 66, 1976: pp. 589-597.

05_CHAPTER 5.indd 145 3/6/09 7:53:24 PM


146 Capítulo 5 apliCaCiones de las teorías de la eleCCión raCional y de la demanda

reduce al área del triángulo ACD, CS' = ½ (10 – 3) 7 = 24.50 dólares/semana. Su pérdida en excedente
del consumidor es la diferencia entre estas dos áreas, que corresponde al área del trapezoide DCEF, la
región sombreada en la figura 5.5. Esta área es igual a CS – CS' = 32 – 24.5 = 7.50 dólares/semana.

EJERCICIO 5.1
¿De cuánto será la reducción en el excedente del consumidor en el ejemplo 5.1 si el precio de la
gasolina sube de 3 dólares/galón a 4 dólares/galón?

APLICACIÓN: PRECIO EN DOS PARTES


El razonamiento económico indica que entre un comprador y un vendedor tendrá lugar un inter-
cambio voluntario si y sólo si ambos se ven beneficiados. Desde el punto de vista del comprador, se
puede decir que su disposición al intercambio depende de sus expectativas de obtener un excedente
del consumidor en la transacción.
La teoría económica no da mucha información acerca de cómo se dividen las ganancias del
intercambio entre comprador y vendedor. Algunas veces el comprador estará en posición de ventaja,
lo que le permitirá captar la mayor parte de los beneficios de la negociación. Otras veces las opciones
del comprador son más limitadas y en esos casos es posible que su excedente del consumidor sea me-
nor. En realidad, como ilustra El naturalista económico 5.1, algunas veces el vendedor puede diseñar
estrategias de precio que le permiten captar todo el excedente del consumidor.

¿Por qué algunos clubes de tenis cobran una membresía anual además de la renta por hora
de la cancha?
En un club de tenis el alquiler de las
canchas asciende a 25 dólares por hora
EL NATURALISTA por persona. En la figura 5.6 se presen-
ECONÓMICO ta la curva de demanda por tiempo de
5.1 cancha de John, P = 50 – ¼ Q, donde
Q se mide en horas por año. Si se su-
pone que en esa localidad no hubiera
ningún otro club de tenis, ¿cuál es la
membresía anual máxima que estaría
dispuesto a pagar John por tener de-
recho a comprar tiempo de cancha de
tenis a 25 dólares/h?
La respuesta a esta pregunta es el
excedente del consumidor que obtie-
ne John por poder comprar la canti- ¿por qué muchos clubes de tenis cobran una membresía anual
además de una cuota por hora de las canchas?
dad de tiempo de cancha que desee al
precio de 25dólares/h. Esto es igual
al área del triángulo ABC en la figura 5.6, que es CS = ½ (50 – 25) 100 = 1 250 dólares/año. Si el club
cobra una membresía mayor a esta cantidad, John se verá más beneficiado si no alquila la cancha de
tenis.

EJERCICIO 5.2
En El naturalista económico 5.1, ¿de cuánto sería el costo de la membresía anual si el precio
por hora de la cancha de tenis fuera de 20 dólares/hora?

El naturalista económico 5.1 es un ejemplo de múltiples prácticas de precios que se observan


en la economía. Muchos parques de atracciones, por ejemplo, cobran una tarifa de admisión además
de un precio por cada atracción. Muchas empresas telefónicas cobran una tarifa mensual fija además

05_CHAPTER 5.indd 146 3/6/09 7:53:29 PM


ComparaCiones generales del bienestar eConómiCo 147

FIGURA 5.6
Precio (dólares/h) Curva de demanda
individual por cada
A precio por hora de
50
alquiler de la cancha de
tenis
25 B a 25 dólares/h, John
C obtiene 1 250 dólares/año
(el área sombreada) como
Tiempo de cancha (h/año) excedente del consumidor
0 100 200 por alquiler de la cancha.
la cuota anual máxima que
puede cobrar el club son
1 250 dólares.

de cobrar por las llamadas realizadas. Y algunos clubes de compras cobran una cuota fija para tener precio en dos partes
derecho a comprar los artículos que ellos venden. A estas estrategias de precios se les suele llamar estrategia de precios que
precio en dos partes. La función que tienen estas estrategias de precios es transferir parte del exce- consiste en una cuota fija y
dente del consumidor del comprador al vendedor. un cobro marginal por cada
unidad comprada (también se
le llama tarifa en dos partes).

¿Por qué algunos parques de atracciones cobran sólo una tarifa de entrada y ninguna canti-
dad adicional incluso por atracciones para las que hay largas colas?
En Disney Magic Kingdom el pase por día para niños menores de 10 años tiene un precio de 55 dó- EL NATURALISTA
lares. Con dicho pase se tiene acceso ilimitado a todas las atracciones del parque, el único problema ECONÓMICO
es que en ciertos juegos mecánicos —como en la popular montaña rusa Space Mountain— hay que
hacer filas de más de una hora. Dado que al precio de cero la demanda de algunos juegos es excesiva, 5.2
¿por qué no cobra Disney una cuota adicional en sus juegos
mecánicos más populares?
La teoría económica dice que el precio de cualquier bien
o servicio aumenta ante un exceso de demanda. Las largas
filas de espera que se describieron antes son un desafío para
los economistas. En este caso, una posible explicación puede
ser que las personas que pagan por las atracciones (los padres)
no son las mismas que los demandan (sus hijos). Como los
que pagan son los padres, los niños quieren subir a las atrac-
ciones más emocionantes sea su precio 0 o 50 dólares. A un
precio lo suficientemente alto para eliminar las filas, sería
posible ir a los juegos mecánicos más populares docenas de
veces al día y eso es lo que muchos niños desearían hacer.
Los padres, por supuesto, podrían racionar el acceso a estos
juegos. Pero a muchos padres no les entusiasman unas vaca-
ciones en las que tengan que pasar todo el día diciendo no.
Para ellos la actual política de precios de Disney es quizá
la solución ideal; les permite decirle a sus hijos “vayan a todos los
juegos que quieran todas las veces que quieran” y dejar que las
largas filas realicen la función de racionar.
¿por qué disney no cobra cuotas de entrada extra en sus juegos
más populares?
COMPARACIONES GENERALES
DEL BIENESTAR ECONÓMICO
El concepto de excedente del consumidor ayuda a identificar los beneficios (o costos) de las varia-
ciones que ocurren en determinados mercados. Con frecuencia se desea evaluar si los consumidores
son beneficiados o perjudicados por las variaciones no sólo en uno sino en varios mercados.

05_CHAPTER 5.indd 147 3/6/09 7:53:31 PM


148 Capítulo 5 apliCaCiones de las teorías de la eleCCión raCional y de la demanda

En este caso el modelo de elección racional también permite hacer diversas inferencias útiles.
Considere el ejemplo siguiente.

EJEMPLO 5.2 Jones gasta todo su ingreso en dos bienes: X y Y. Los precios que pagó y las cantidades que
consumió el año pasado son los siguientes: PX = 10, X = 50, PY = 20 y Y = 25. Este año tanto PX
como PY son 10 y el ingreso de Jones es 750 dólares. Si se supone que sus gustos permanecen
sin cambio, ¿qué año es mejor para Jones, éste o el pasado?

Para responder a esta pregunta se empieza por comparar las restricciones presupuestarias de Jones
para los dos años. Observe primero que el año pasado su ingreso fue igual a lo que gastó, a saber,
PXX + PYY = 1 000. Con los precios dados, la restricción presupuestaria que se tiene es entonces la que
muestra la figura 5.7a.

FIGURA 5.7
Restricciones de
presupuestarias para
dos años
Y Y
a) si la restricción
presupuestaria del Presupuesto
consumidor para este año 75 de este año 75
contiene el mismo paquete
que compró el año pasado
(paquete A), este año el 50 50 D
Presupuesto
consumidor estará tan bien del año pasado
económicamente como el A C
25 25 I1
año pasado. b) si, además, A
los precios relativos I0
de estos dos años son X X
diferentes, este año podrá 50 75 100 50 75 100
comprar un mejor paquete
(paquete D). a) b)

En la figura 5.7a se ve que la restricción presupuestaria de Jones para este año contiene exac-
tamente el mismo paquete que compró el año pasado. Dado que sus gustos no han cambiado, esto
indica que este año su situación económica no es peor que la del año pasado. Después de todo,
aún puede comprar el mismo paquete. Pero la suposición estándar acerca de los órdenes de pre-
ferencia permite hacer una inferencia aún más fuerte. Si sus curvas de indiferencia tienen la forma
convexa usual, se sabe que existe una curva de indiferencia —llámele I0— tangente en el punto A a
la restricción presupuestaria del año pasado, véase la figura 5.7b. También se sabe que la restricción
presupuestaria de este año tiene una inclinación mayor que la del año pasado, lo cual indica que parte
de I0 debe encontrarse dentro del triángulo de presupuesto de este año. En I0 la preferencia por el
paquete A es igual a la preferencia por el paquete C. Se concluye que D se prefiere en vez de A, con lo
que se sabe que este año Jones ha podido comprar un paquete de bienes que le gusta más que el que
compró el año pasado. Se concluye entonces que Jones tuvo una mejor situación económica este año
que el pasado.

EJERCICIO 5.3
Jones gasta todo su ingreso en dos bienes: X y Y. Los precios que pagó y las cantidades que
consumió el año pasado son: PX = 15, X = 20, PY = 25 y Y = 30. Este año los precios han cambiado
(PX = 15 y PY = 20) y el ingreso de Jones es ahora de 900 dólares. Si se supone que sus gustos
permanecen sin cambio, ¿qué año es mejor para Jones, éste o el pasado?

05_CHAPTER 5.indd 148 3/6/09 7:53:32 PM


ComparaCiones generales del bienestar eConómiCo 149

APLICACIÓN: EFECTOS DE LAS VARIACIONES EN EL PRECIO


DE LA VIVIENDA SOBRE EL BIENESTAR ECONÓMICO
Considere las dos situaciones siguientes:

1. Usted acaba de comprar una casa en 200 000 dólares. Al día siguiente de su compra, el precio de
todas las casas, incluyendo la suya, se duplica.
2. Usted acaba de comprar una casa en 200 000 dólares. Al día siguiente de su compra, el precio de
todas las casas, incluyendo la suya, baja a la mitad.

En cada caso, ¿cómo afecta la variación de precios su bienestar económico? (¿Está usted en una mejor
situación económica antes o después del cambio de precios?)
Hace poco hice esta pregunta a un grupo de estudiantes de economía. La inmensa mayoría
respondió que en la situación 1, después del aumento de precios, está uno en una mejor situa-
ción económica, pero en la situación 2, después de la baja del precio, está uno en una peor si-
tuación económica. Aunque la mayoría de los estudiantes parecían estar seguros de estas dos respues-
tas, sólo una era correcta. Para saber por qué, considere primero el caso en que el precio de todas las
casas se duplica. Suponga que su patrimonio total antes de comprar la casa era de 400 000 dólares.
Sea el tamaño de su casa actual equivalente a una unidad de vivienda y sea 1 el precio de los otros
bienes (el bien compuesto). Su restricción presupuestaria original en la situación 1 corresponderá a la
línea indicada como B1 en la figura 5.8. Su ordenada en el origen, 400 000 dólares, es la cantidad máxi-
ma que puede gastar en otros bienes. Su abscisa en el origen, 2 unidades de vivienda, corresponde a
la máxima cantidad de vivienda que podría haber comprado (es decir una casa del doble de tamaño
de la que compró). En B1 el punto de equilibrio en A representa su compra original. En A usted tiene
una unidad de vivienda y le quedan 200 000 dólares para otros bienes.

FIGURA 5.8
Otros bienes Aumento del precio de
la vivienda y bienestar
económico de los
600 000 propietarios de casas
Cuando el precio de
las casas se duplica, su
restricción presupuestaria
400 000 será B2, que contiene
O2 C también su paquete original
A. Como C, el paquete
óptimo en B2, se encuentra
200 000 en una curva de indiferencia
A
I2 superior a A, el efecto del
I1 aumento del precio de la
B2 B1 vivienda mejora su situación
Cantidad de vivienda económica.
H2 1 1.5 2

Después que el precio de su vivienda se duplica, su restricción presupuestaria será la línea indi-
cada como B2 en la figura 5.8. Para calcular la ordenada en el origen de B2 observe que ahora puede
vender su casa en 400 000 dólares, los cuales, sumados a los 200 000 dólares restantes después de
comprar la casa, dan una cantidad máxima disponible de 600 000 dólares para otros bienes. La abs-
cisa en el origen de B2 indica que cuando el precio de la vivienda se duplica a 400 000 dls/unidad, sus
600 000 dólares comprarán un máximo de sólo 1.5 unidades de vivienda. Observe finalmente que
sobre B2 su paquete óptimo es C, que contiene H2 < una unidad de vivienda y O2 > 200 000 dólares
para otros bienes. Y como el paquete C se encuentra en una curva de indiferencia superior a la del
paquete A, usted está en una mejor situación que antes del aumento de precio.

05_CHAPTER 5.indd 149 3/6/09 7:53:33 PM


150 Capítulo 5 apliCaCiones de las teorías de la eleCCión raCional y de la demanda

Es lógico que si el precio de la vivienda sube, su mejor respuesta será comprar menos unidades
de vivienda y más unidades de otros bienes. Observe que usted está resguardado del daño del efecto
ingreso causado por el aumento de precio debido a que esto hace que su propiedad valga más.
Hasta aquí vamos bien. Ahora considere lo que para muchos estudiantes fue el caso más compli-
cado; es decir, la situación 2, en la que el precio de la vivienda disminuye a la mitad. Si se emplean otra
vez las unidades de medida de la situación 1, su restricción presupuestaria después de la disminución
de los precios de la vivienda es la línea indicada como B3 en la figura 5.9. Para obtener la ordenada en
el origen, observe que la venta de su casa le proporcionaría sólo 100 000 dólares, los cuales, sumados
a los 200 000 que ya tiene, hacen un máximo de 300 000 dólares disponibles para la compra de otros
bienes. Para calcular la abscisa en el origen de B3, observe que cuando el precio de la vivienda baja a
100 000 dólares, con sus 300 000 comprará ahora máximo 3 unidades de vivienda. Dada la restricción
presupuestaria B3, el mejor paquete asequible es el indicado como D, que contiene H3 > una unidad
de vivienda y O3 < 200 000 unidades de otros bienes. Como en la situación 1, el efecto de la variación
relativa del precio es de nuevo llevarlo a usted a una curva de indiferencia superior. Sin embargo, esta
vez su dirección de sustitución es la contraria a la de la situación 1: como la vivienda es ahora más
barata que antes, usted responde comprando más unidades de vivienda y menos unidades de otros
bienes.

FIGURA 5.9
Disminución en los Otros bienes
precios de la vivienda
I1
y bienestar económico
de los propietarios de
casas I3
Cuando el precio de la
vivienda disminuye 400 000
a la mitad, su restricción
presupuestaria será B3, la 300 000
cual contiene también su
paquete original A. Como 200 000
D, el paquete óptimo de B3, A D
O3
se encuentra en una curva
de indiferencia superior
a la de A, el efecto de la B1 B3
disminución del precio de Cantidad de vivienda
1 H3 2 3
la vivienda es mejorar su
situación económica.

Observe en cada situación que su nueva restricción presupuestaria contiene su paquete original,
lo cual significa que después de la variación del precio usted está en una situación económica por lo
menos tan buena como la de antes. Observe también que en cada uno de estos casos la variación de los
precios relativos significa que su nueva restricción presupuestaria contiene paquetes que están más allá
de su curva de indiferencia original, lo que en ambos casos hace posible alcanzar un mejor resultado.

APLICACIÓN: SESGO EN EL ÍNDICE


DE PRECIOS AL CONSUMIDOR
El índice de precios al consumidor (IPC) mide las variaciones en el “costo de la vida”, el monto que
tiene que gastar un consumidor para mantener un determinado estándar de vida. El IPC es publicado
cada mes por la Oficina de Estadística Laboral (de Estados Unidos) y se obtiene calculando el costo
de un paquete representativo de bienes y servicios durante un periodo de referencia y dividiendo des-
pués esta cantidad entre el costo actual del mismo paquete. Así, si en el periodo de referencia cuesta
100 dólares comprar el paquete representativo y actualmente el mismo paquete cuesta 150 dólares,
el IPC será 1.5. Con la publicación de esta cifra, el vocero del gobierno manifiesta que esto significa
que el costo de la vida ha aumentado 50% en comparación con el periodo de referencia.

05_CHAPTER 5.indd 150 3/6/09 7:53:34 PM


ComparaCiones generales del bienestar eConómiCo 151

Sin embargo, lo que el IPC no toma en cuenta es que cuando los precios de varios bienes aumen-
tan en proporciones diferentes, los consumidores no suelen comprar el mismo paquete de bienes
que antes. En general sustituyen aquellos bienes cuyo precio ha aumentado más. Reasignando sus
presupuestos, los consumidores pueden escapar, por lo menos en parte, de los efectos perjudiciales
del aumento de precio. El IPC exagera el aumento del costo de la vida debido a que no toma en
cuenta la sustitución.
Un sencillo ejemplo que emplea el modelo de la elección racional aclara este punto de manera
inconfundible. Suponga que los únicos bienes en la economía fueran arroz y trigo y que en el perio-
do de referencia un consumidor representativo consumiera 20 lb/mes de cada uno de ellos. Si en
el periodo de referencia tanto el arroz como el trigo se venden en 1 dólar/lb, ¿cuál será el IPC en el
periodo actual si ahora el arroz tiene un precio de 2 dólares/lb y el trigo de 3 dólares/lb? A los precios
del periodo de referencia, el costo del paquete era de 40 dólares, mientras que ahora, a los precios
actuales, el mismo paquete cuesta 100 dólares. Entonces, el valor del IPC es 100 dls/40 dls = 2.5. ¿Pero
es correcto decir que ahora el costo de la vida es 2.5 veces mayor que antes?
Para ver un caso extremo, suponga que el consumidor representativo considera arroz y trigo
como sustitutos perfectos uno a uno, lo cual significa que sus curvas de indiferencia son rectas a 45
grados con pendiente negativa. En la figura 5.10 su paquete original es A y su curva de indiferencia
original (que coincide de manera exacta con su restricción presupuestaria original) es I0. ¿Cuánto
debe aumentar su ingreso en el periodo actual para lograr el mismo nivel de satisfacción que tenía
en el periodo de referencia? A los nuevos precios la pendiente de su restricción presupuestaria ya no
es –1, sino –3/2. Al tener una restricción presupuestaria con esta nueva pendiente, la manera menos
costosa de alcanzar su curva de indiferencia original es comprar el paquete C de la figura 5.10. Y dado
que el costo de C a los precios actuales es de sólo 80 dólares, se puede decir que el costo de conservar
el nivel de satisfacción original ha aumentado en un factor de 2.0 y no de 2.5.
En general, se puede decir que el grado en que el IPC exagera el costo de la vida se incrementa a
medida que las posibilidades de sustitución aumentan. El sesgo será también mayor si las diferencias
en las tasas de aumento de los diversos precios son mayores.

FIGURA 5.10
Sesgo inherente al índice
de precios al consumidor
para este consumidor arroz y
trigo son sustitutos perfectos.
Cuando, en el periodo de
referencia, el precio de cada
Arroz (lb/mes)
Paquete óptimo uno de ellos era 1 dólar/lb,
en el periodo presente el consumidor compraba 20
C
40 lb/mes, lo que suponía un
gasto total de 40 dólares/
mes. si los precios actuales
Paquete óptimo de arroz y trigo son 2 dls/lb
en el periodo de referencia y 3 dls/lb, respectivamente, la
cantidad que necesita gastar
A para comprar el paquete
20
original es de 100 dólares/
mes. el ipC es el cociente de
I0 estos dos gastos, 100 dls/40
dls = 2.5. pero el consumidor
Pendiente = – 3 B1
2 puede alcanzar su curva de
indiferencia original, I0, si
Trigo (lb/mes)
0 20 40 compra el paquete C, el cual,
a los precios actuales, sólo
cuesta 80 dólares. de manera
que el costo de conservar el
nivel de satisfacción original
ha aumentado en un factor
de 2.0.

05_CHAPTER 5.indd 151 3/6/09 7:53:35 PM


152 Capítulo 5 apliCaCiones de las teorías de la eleCCión raCional y de la demanda

VARIACIÓN EN LA CALIDAD: ¿OTRO SESGO EN EL IPC?


Recolectar datos sobre los precios de los bienes y servicios puede parecer una tarea sencilla. Sin em-
bargo, en la práctica esto es complicado debido a la existencia de descuentos, reembolsos y otras
ofertas promocionales con las que el precio real de la transacción puede ser sustancialmente diferen-
te al de la lista oficial de precios.
Sin embargo, a pesar de ser tan importantes, los datos exactos de los precios no son suficientes
para estimar las variaciones en el costo de la vida. También se deben tomar en cuenta las variaciones
de la calidad. Y esto, por desgracia, resulta ser una tarea mucho más complicada que medir las varia-
ciones de los precios.
Una breve ojeada a la industria automotriz ilustra esta dificultad. El Departamento de Comercio
de Estados Unidos informó que en 1994 el precio promedio de un vehículo nuevo era de 19 675 dóla-
res, un aumento de 5.1% en comparación con 1993 y de 72.8% en comparación con 1984.4 Durante
este mismo lapso de 10 años el IPC aumentó sólo 42.6%. ¿Significa esto que durante este periodo
los precios de los automóviles aumentaron con mucha más rapidez que los de los demás bienes y
servicios? No necesariamente. Después de todo, los automóviles producidos en 1994 poseían mu-
chas características que no tenían los modelos anteriores. Por ejemplo mientras que casi 90% de los
automóviles que se vendieron en Estados Unidos en 1994 estaban equipados con bolsas de aire y 40%
con frenos antibloqueo, en 1984 casi ningún automóvil tenía estas características. Durante esa misma
década la cantidad de automóviles con equipos como descongelador para el parabrisas trasero y ven-
tanas automáticas, alguna vez exclusivos de los modelos de lujo, aumentó más de 50%.
Ahora el Departamento de Comercio calcula un IPC especial para los automóviles en el que se
deduce el costo de estas características adicionales para tratar de medir las variaciones de precio de
automóviles equipados en forma similar. Este índice aumentó nada más 32.2% entre 1984 y 1994; es
decir, 10% menos que el IPC general.
Sin embargo, a pesar de que está claro que estos ajustes ayudan, sólo captan una pequeña parte
de las variaciones ocurridas en los últimos años en la calidad de los automóviles. Por ejemplo, el IPC
para los automóviles no toma en cuenta el hecho de que en la actualidad se han logrado reducciones
de 90% en las emisiones de hidrocarburos y de 60% en las de óxidos de nitrógeno en relación con los
automóviles de 1984. El índice tampoco toma en cuenta el hecho de que en 1994 los automóviles
eran mucho más confiables, más resistentes a los accidentes y a la corrosión que los automóviles de
una década antes.
El ritmo en la mejora de la calidad de los automóviles se ilustra con claridad mediante una com-
paración entre el Honda Civic DX sedán 1995 —uno los automóviles más pequeños y más baratos de
la empresa en ese año— con el Honda Accord sedán de 1982. Además de tener varias características
de seguridad que no se encuentran en el viejo Accord, el Civic 95 tenía un interior más amplio, un
motor más potente (102 caballos de fuerza contra 75), mejores llantas y mejores frenos y una suspen-
sión mucho más sofisticada. El Civic 95 aceleraba de 0 a 60 mph en 9.1 segundos a diferencia de los
12.2 segundos que necesitaba el Accord 82; el Civic 95 tenía un rendimiento de 40 millas por galón,
mientras que el Accord 82 daba 32. Mientras el Civic 95 podía resistir seis inviernos en condiciones
parecidas a la de una sala de exhibición, el Accord 82 con una exposición semejante quedaba cubierto
de óxido. El precio de lista del Accord 82 era 8 995 dólares, pero como escaseaba, muchos concesio-
narios de automóviles lo daban en aproximadamente 10 000. El precio de lista del Civic 95 era 12 360
dólares y la mayoría de los concesionarios de automóviles ofrecían un descuento sustancial. De ma-
nera que, a pesar de una diferencia de 13 años, el precio de la transacción en dólar constante no era
mucho mayor en el caso del Civic que en el del viejo Accord. El precio de lista del Civic 95, ajustado a
las variaciones del IPC general, se traduce en 8 852 dólares de 1982; en realidad, un automóvil mucho
mejor por menos dinero.
Si la comparación Civic-Accord es representativa, parece que el intento del gobierno de hacer
un ajuste de acuerdo con la mejora de la calidad quedó corto. Con la creciente competencia global,
ha habido una rápida mejora de la calidad no sólo en los automóviles sino también en otros bienes

4 Los datos de esta sección se obtuvieron de Csaba Csere, “Do Cars Cost Too Much, or Do We Just Like Costly Cars?”, en Car and

Driver, junio de 1995, p. 9.

05_CHAPTER 5.indd 152 3/6/09 7:53:36 PM


uso de la elastiCidad-preCio de la demanda 153

y servicios. Y puede uno estar seguro de que, como ocurre en el caso de la industria automovilística,
muchos de los cambios habrán escapado a la atención del Departamento de Comercio.
No tener en cuenta por completo la mejora de la calidad tiene el mismo efecto que pasar por
alto la sustitución. Ambas ocasionan que en el índice oficial del costo de la vida se exagere el verda-
dero aumento de precios.
El IPC tiene consecuencias extremadamente importantes para el déficit presupuestario federal,
porque éste es el índice que se emplea para determinar los ajustes al costo de la vida que reciben los
beneficiarios de la Seguridad Social y los beneficiarios de gran cantidad de diversos programas guber-
namentales. Incluso un ligero sesgo hacia arriba en el IPC puede hacer que el déficit presupuestario
se incremente en varios millones de dólares.

USO DE LA ELASTICIDAD-PRECIO DE LA DEMANDA


En el campo del análisis económico aplicado pocas herramientas son tan importantes como el con-
cepto de la elasticidad-precio de la demanda. En esta sección se examinarán aplicaciones históricas de
este concepto en dos casos muy diferentes.

APLICACIÓN: AUMENTA LA TARIFA DE MARTA


En 1987, para saldar un enorme déficit presupuestal, la Metropolitan Atlanta Rapid Transit Autho-
rity (MARTA), incrementó la tarifa básica de 60 a 75 centavos/viaje. En los dos meses siguientes al
aumento de la tarifa los ingresos totales del sistema subieron 18.3% en comparación con el mismo
periodo del año anterior.5 Si se supone una curva de demanda lineal y que las variaciones observadas
en la cantidad de pasajeros que utilizaron el transporte público fueron el resultado del aumento de
la tarifa, ¿qué indican estas cifras acerca de la elasticidad-precio original de la demanda de viajes en
el sistema MARTA? Si Q1 denota la cantidad original de viajes y ∆Q la variación en los viajes debido
al incremento de la tarifa y si ∆P y P1 denota la variación en el precio y el precio original, respectiva-
mente, se quiere usar la información dada para calcular la expresión e = (∆Q/Q1)/(∆P/P1). Suponga
que la curva de demanda de viajes en MARTA es la que se muestra indicada como D en la figura 5.11.
El hecho de que el ingreso total haya subido 18.3% puede expresarse como sigue:

75(Q1 + ∆Q ) – 60Q1
= 0.183, (5.1)
60Q1

FIGURA 5.11
P (centavos/viaje) Aumento de la tarifa de
MARTA
si se conoce la variación
porcentual en el gasto total
y la variación porcentual en
el precio se puede calcular
la elasticidad-precio de la
demanda.

75
60
D
Q (viajes/día)
Q1
Q1 + DQ

5 Vea Bert Roughton Jr., “MARTA Sees Ridership Dip with Fare Hike”, en Atlanta Constitution, 8 de octubre de 1987, p. 7.

05_CHAPTER 5.indd 153 3/6/09 7:53:37 PM


154 Capítulo 5 apliCaCiones de las teorías de la eleCCión raCional y de la demanda

donde ∆Q < 0 es la disminución en la cantidad de viajes. La ecuación 5.1 se reduce a


(5.2)
15Q1 + 75∆Q
= 0.183,
60Q1
de la que a su vez se obtiene
∆Q
= −0.0536. (5.3)
Q1
Como se sabe que ∆P/P1 = 15 60 = 0.25, esto indica que e = –0.0536/0.25 = –0.2144. De esta manera re-
sulta que la demanda de pasajes de MARTA es muy inelástica con respecto al precio, lo cual concuerda
con el hecho de que el incremento de la tarifa condujo a un aumento sustancial del gasto total.

EJERCICIO 5.4
En el ejemplo que se acaba de analizar suponga que MARTA incrementa su tarifa de 1.00 dólar
a 1.10 y que, como respuesta a esto, los ingresos del sistema suben 2%. ¿Qué indica esto acerca
de la elasticidad de la demanda de boletos de MARTA al precio original de 1.00 dólar? (De
nuevo suponga que la curva de demanda de los boletos es lineal.)

APLICACIÓN: LA ELASTICIDAD-PRECIO
DE LA DEMANDA DEL ALCOHOL
¿Cómo responde el consumo de bebidas alcohólicas a las variaciones de su precio? Durante muchas
décadas la sabiduría convencional respondía a esta pregunta, “no mucho”. Por desgracia las estima-
ciones de la elasticidad-precio de la demanda de alcohol tienden a ser poco confiables. El problema es
que los precios de las bebidas alcohólicas no suelen variar lo suficiente como para hacer estimaciones
exactas.
En un cuidadoso estudio,6 Philip Cook empleó algunos datos, no explotados hasta entonces,
sobre variaciones significativas del precio del alcohol. Sugirió que la elasticidad-precio de la demanda
de alcohol puede ser mucho más alta de lo que se piensa.
El método de Cook consistió en examinar las variaciones en el consumo de alcohol que se dan
como respuesta a cambios en los impuestos estatales. De los 48 estados confederados, 30 permiten
y gravan la venta privada de bebidas alcohólicas. De manera periódica la mayoría de dichos estados
incrementa sus impuestos nominales sobre las bebidas alcohólicas para compensar los efectos de la
inflación. La pauta es que el valor real de un impuesto estatal al licor es más alto justo después de que
uno de estos impuestos se incrementa, luego se diluye de manera continua a medida que aumenta
el costo de la vida. El hecho de que los impuestos no se ajusten en forma periódica para mantener
constante su valor real proporciona la variabilidad de precio real que se necesita para estimar la sensi-
bilidad de la adquisición de alcohol ante los cambios de precio.
Durante el periodo 1960-1975, hubo 39 aumentos del impuesto a las bebidas alcohólicas en los
30 estados estudiados por Cook. En 30 de estos 39 casos, Cook encontró que al año siguiente del
incremento del impuesto, el consumo de alcohol disminuía con respecto a la tendencia nacional.
Su estimación de la elasticidad-precio de la demanda fue –1.8, un valor mucho mayor que el hallado
en estudios previos. La interpretación que dio Cook a sus hallazgos proporciona un caso de estudio
interesante sobre los factores que rigen la elasticidad-precio. Un factor sobresaliente acerca del mer-
cado de las bebidas alcohólicas, observó Cook, es que los bebedores empedernidos, a pesar de ser
una parte pequeña de la población total, son los responsables de una gran parte del consumo total de
alcohol. Este hecho llevó a mucha gente a esperar que el consumo de alcohol fuera insensible a la va-
riación del precio. El punto de vista general de los bebedores habituales es, después de todo, que ellos
beben primordialmente por dependencia, no debido a deliberaciones racionales acerca del precio.
Dicho de otra manera, los analistas esperan que el efecto sustitución sea pequeño en estas personas.
6 Philip J. Cook, “The Effect of Liquor Taxes on Drinking, Cirrhosis and Auto Accident”, en Alcohol and Public Policy, Mark Moore y

Dean Gerstein (editores), Washington DC: National Academy Press, 1982.

05_CHAPTER 5.indd 154 3/6/09 7:53:39 PM


el modelo de eleCCión intertemporal 155

Pero aun cuando el efecto sustitución fuera cero en el caso de los bebedores empedernidos, quedaría
el efecto ingreso. Entre los bebedores habituales la parte del presupuesto que destinan al alcohol
tiende a ser grande por dos razones. La más obvia es que compran mucho alcohol. Quizá menos ob-
vio es que es su ingreso tiende a ser considerablemente menor que el promedio. Muchos bebedores
habituales tienen dificultades para tener un trabajo estable y con frecuencia aunque lo conserven no
logran ser productivos. El resultado es que el efecto ingreso de un aumento sustancial en el precio de
las bebidas alcohólicas obliga a muchos bebedores habituales a consumir menos. Como apoyo a esta
interpretación, Cook observó que la mortalidad por cirrosis hepática disminuyó de manera notable
en los años posteriores a un incremento significativo de los impuestos a las bebidas alcohólicas. Ésta
es una enfermedad que en su mayor parte afecta sólo a las personas con extensos antecedentes de
abuso del alcohol y la experiencia clínica muestra que la reducción en el consumo de alcohol por
parte de los bebedores habituales puede retardar o prevenir su aparición.

EL MODELO DE ELECCIÓN INTERTEMPORAL


En las elecciones que se han considerado hasta ahora las alternativas se refieren al presente: la elección
entre alimento y vestido ahora, entre viajar y un equipo estereofónico ahora, etc. No ha habido ningún
indicio de que la alternativa elegida hoy pueda afectar al menú de alternativas del futuro.
Sin embargo, tales efectos son una característica sobresaliente de muchas decisiones importan-
tes. En esta sección el objetivo es ampliar el modelo de elección del consumidor que se vio en el
capítulo 3 para considerar estos efectos.

PAQUETES DE CONSUMO INTERTEMPORAL


La gente puede consumir todo su ingreso ahora o ahorrar una parte para el futuro. ¿Cómo dis-
tribuyen los consumidores racionales su consumo a lo largo del tiempo? Para que el análisis siga
siendo manejable, suponga que sólo hay dos periodos, el actual y el futuro. En el modelo de elección
estándar o atemporal del capítulo 3, las alternativas eran entre bienes diferentes que podían ser consu-
midos en el periodo actual: manzanas ahora versus naranjas ahora, etc. En nuestro modelo de elección
intertemporal, las alternativas serán consumo actual (C1) versus consumo futuro (C2). Cada uno de éstos
es una amalgama: el equivalente funcional al bien compuesto (vea el capítulo 3). Para simplificar se
dejará a un lado la cuestión de cómo repartir los consumos actual y futuro entre los distintos bienes
de consumo.
En el modelo de elección atemporal cualquier paquete de bienes puede representarse como un
punto en un diagrama sencillo de dos dimensiones. En el modelo de elección intertemporal se usa
un procedimiento análogo. Por ejemplo, en la figura 5.12 se representa el consumo actual de 6 000
dólares combinado con el consumo futuro de 6 000 dólares por medio del paquete E. El paquete D
representa un consumo actual de 3 000 dólares y un consumo futuro de 9 000 dólares.

FIGURA 5.12
C2 (consumo futuro) Paquetes de consumo
intertemporal
las combinaciones de
D alternativas de consumo
9 000
actual y futuro se
representan como puntos
E
6 000 en el plano C1, C2. por
convención el eje horizontal
mide el consumo actual,
el eje vertical el consumo
futuro.

C1 (consumo presente)
3 000 6 000

05_CHAPTER 5.indd 155 3/6/09 7:53:40 PM


156 Capítulo 5 apliCaCiones de las teorías de la eleCCión raCional y de la demanda

LA RESTRICCIÓN PRESUPUESTARIA INTERTEMPORAL


Suponga que ahora tiene un ingreso de 50 000 dólares y en el futuro uno de 60 000 dólares. Suponga
además que si usted deposita parte de su ingreso actual en un banco, en un periodo futuro puede ob-
tener su capital principal más 20%. De manera similar, si desea pedir prestado con cargo a su ingreso
futuro puede recibir un dólar ahora por cada 1.20 que deba pagar en el futuro. (Vea la figura 5.13.)
Para construir su restricción presupuestaria intertemporal observe primero que en cada periodo us-
ted puede simplemente consumir su ingreso, de manera que C1 = 50 000 dólares y C2 = 60 000 dólares
debe ser un punto en su restricción presupuestaria intertemporal. Otra opción es depositar el total de
los 50 000 dólares (préstamo máximo) y recibir así 1.2 (50 000) = 60 000 dólares, además de su ingreso
futuro de 60 000 dólares, para tener un consumo futuro C2 = 120 000 dólares sin consumo actual
(C1 = 0). Pero otra opción es tomar prestados 60 000 dólares/1.2 = 50 000 dólares (préstamo máximo)
además de su ingreso actual de 50 000 dólares para tener un consumo actual C1 = 100 000 dólares
y ningún consumo futuro (C2 = 0). La ecuación de su restricción presupuestaria intertemporal es
C2 = 120 000 – 1.2C1, o lo que es equivalente, 1.2C1 + C2 = 120 000 dólares.

FIGURA 5.13
Restricción
C2
presupuestaria
120 000
intertemporal
por cada dólar de
reducción en el consumo
actual es posible aumentar
el consumo futuro en 1.2. B

60 000

Pendiente
= –1.2
C1
0 50 000 100 000

En general, suponga que usted recibe M1 de su ingreso en el primer periodo y M2 en el segundo


periodo y que puede pedir prestado o prestar a una tasa de interés r. ¿Cuánto es lo máximo que podrá
consumir en el periodo futuro? El consumo futuro máximo se presenta cuando usted ahorra todo
su ingreso actual para gastarlo en el futuro. Ahorrar M1 en el periodo actual a una tasa de interés r
significa que su depósito crecerá hasta M1(1 + r) en el periodo futuro. De manera que lo máximo que
podrá consumir en el futuro será esa cantidad más su ingreso futuro, es decir, M1(1 + r) + M2.
¿Cuánto es lo máximo que puede usted consumir en el periodo actual? La respuesta es su ingre-
so actual más la cantidad máxima que pueda pedir prestada con cargo a su ingreso futuro. Lo más
valor actual el valor actual de que puede pedir prestado contra su ingreso futuro de M2 se conoce como valor actual de M2, que
un pago de X dólares durante se representa como PV(M2). Es la cantidad que, depositada hoy a una tasa de interés r, valdrá exac-
T años a partir de ahora es tamente M2 en el periodo futuro. Por lo tanto, el valor actual de M2 puede encontrarse despejando
X/(1 + r)T, donde r es la tasa de PV(M2) de PV(M2)(1 + r) = M2:
interés anual.
M2
PV( M 2 ) = . (5.4)
1+ r

Por ejemplo, si M2 fuera 110 000 dólares y la tasa de interés fuera 10% (es decir, r = 0.10), el
valor actual de M2 sería 110 000/1.1 = 100 000 dólares. El valor actual es una sencilla relación de
equivalencia entre sumas de dinero que son pagables en momentos diferentes. Si r = 0.10, entonces

05_CHAPTER 5.indd 156 3/6/09 7:53:41 PM


el modelo de eleCCión intertemporal 157

FIGURA 5.14
C2 Restricción
presupuestaria
M1(1 + r) + M2 intertemporal con
ingreso en ambos
periodos cuando es
posible pedir prestado
B o prestar a una tasa r
el costo de oportunidad
de un dólar de consumo
M2 actual es (1 + r) dólares
de consumo futuro. la
abscisa en el origen de la
restricción presupuestaria
Pendiente intertemporal es el valor
= –(1 + r) actual del ingreso a lo
C1 largo de toda la vida,
0 M1 M1 + M2/(1 + r) M1 + M2/(1 + r).

100 000 dólares valdrá 110 000 en el futuro. De la misma manera, 110 000 dólares en el futuro vale
hoy 100 000 dólares si la tasa de interés es de 10%.
Por supuesto, no es necesario pedir prestado o ahorrar las cantidades máximas posibles. El
consumidor que desee desplazar parte de su ingreso futuro al periodo actual puede pedir prestada
cualquier cantidad hasta un máximo a una tasa de 1/(1 + r) dólares de hoy por cada dólar al que se
renuncia en el futuro. O puede ahorrar cualquier cantidad de su ingreso actual y obtener (1 + r)
dólares en el futuro por cada dólar no consumido hoy. La restricción presupuestaria intertemporal,
indicada como B en la figura 5.14, es de nuevo la línea recta que une los puntos de consumo máximo
actual y futuro. Y otra vez su pendiente será – (1 + r). Como en el modelo atemporal, aquí también la
pendiente de la restricción presupuestaria puede interpretarse como un cociente de precios relativos.
Esta vez se trata del cociente de los precios de consumo presente y futuro. El consumo actual tiene
un precio más alto que el consumo futuro debido al costo de oportunidad del interés perdido cuando
el dinero se gasta en lugar de ahorrarse. Es una convención referirse a la abscisa en el origen de la
restricción presupuestaria intertemporal como valor actual del ingreso a lo largo de toda la vida.

EJERCICIO 5.5
Usted tiene como ingreso actual 50 000 dólares y como ingreso futuro 42 000. Si la tasa de
interés entre el periodo actual y el periodo futuro es 5%, ¿cuál es el valor actual de su ingreso a
lo largo de toda la vida? ¿Cuál es la cantidad máxima que podría consumir en el futuro? ¿Cuál es
la ecuación que describe su restricción presupuestaria intertemporal?

Como en el caso atemporal considerado en el capítulo 3, la restricción presupuestaria intertem-


poral es una manera adecuada de resumir los paquetes de consumo que puede comprar una persona.
Y de nuevo como antes, esta restricción presupuestaria no dice nada acerca de qué combinación
elegirá comprar una persona.

CURVAS DE INDIFERENCIA INTERTEMPORAL


Para descubrir qué paquete elegirá el consumidor de entre los que tiene a su disposición se necesita
alguna manera de representar sus preferencias entre el consumo actual y el futuro. Aquí de nue-
vo el recurso analítico es del todo análogo al que se usa en el caso atemporal. Así como las prefe-
rencias del consumidor entre el consumo actual de dos bienes puede captarse mediante un mapa
de indiferencia, así también sus preferencias entre bienes futuros y actuales pueden representarse de

05_CHAPTER 5.indd 157 3/6/09 7:53:42 PM


158 Capítulo 5 apliCaCiones de las teorías de la eleCCión raCional y de la demanda

FIGURA 5.15
Mapa de indiferencia
intertemporal C2
Como en el modelo
atemporal, ir hacia el Incremento de
noreste significa aumentar la satisfacción
la satisfacción. el valor
absoluto de la pendiente de
una curva de indiferencia
en un punto se llama tasa
marginal de preferencia
temporal (tmpt) en ese
punto. la tmpt en A es I4
|∆C2/∆C1|. I3
A
DC2
I2
DC1
I1
C1

esta manera. En la figura 5.15, para el consumidor son indiferentes los paquetes en I1, los cuales son
menos deseable que aquellos en I2, y así sucesivamente.
El valor absoluto de la pendiente de la curva de indiferencia intertemporal en cualquier punto
es la tasa marginal de sustitución entre consumo futuro y consumo actual. En el punto A de la figura
5.15 está dada por |∆C2/∆C1| y este cociente es también conocido como tasa marginal de preferen-
tasa marginal de preferencia cia temporal (TMPT) en A.7 Si |∆C2/∆C1|> 1 en A, se dice que en ese punto el consumidor muestra
temporal el número de una preferencia temporal positiva. Esto significa que requiere más de una unidad de consumo futuro
unidades de consumo en el para compensar la pérdida de una unidad de consumo actual. Si |∆C2/∆C1|< 1 en un punto, se dice
futuro que un consumidor que el consumidor muestra una preferencia temporal negativa. Esta persona está dispuesta a renunciar
puede cambiar por una unidad a una unidad de consumo actual a cambio de menos de una unidad de consumo futuro. Por último,
de consumo en el periodo
si |∆C2/∆C1|= 1 en un punto, se dice que el consumidor tiene una preferencia temporal neutra. En este
actual.
último caso la disyuntiva entre consumo actual y futuro es de 1 a 1.
Como el caso atemporal, parece justificado suponer que la tasa marginal de preferencia tempo-
ral disminuye al bajar a lo largo de una curva de indiferencia. Cuanto mayor es el consumo actual de
una persona, mayor será la cantidad a la que está dispuesta a renunciar con el objetivo de obtener una
unidad más de consumo en el futuro. Entonces, para la mayoría de las personas la cuestión de si la
preferencia temporal es positiva, negativa o neutra es cuestión de dónde se encuentren en sus mapas
de indiferencia. El heredero de una familia acaudalada que me pueda tomar un préstamo contra los
5 000 millones de dólares que heredará en dos años es muy probable que tenga una preferencia
temporal fuertemente positiva. En cambio, el sencillo agricultor cuya producción es perecedera es
posible que tenga una preferencia temporal negativa cuando haya tenido una cosecha excepcional.
La asignación óptima entre consumo actual y futuro está determinada de la misma manera que
en el modelo atemporal. El consumidor elige a lo largo de su restricción presupuestaria el punto que
corresponda a la curva de indiferencia más alta alcanzable. Si las curvas de indiferencia intertemporal
tienen la forma convexa convencional, por lo general se obtiene una solución de tangencia como la
que se muestra en la figura 5.16. Si la TMPT es mayor (o menor) en todas partes que la pendiente de
la restricción presupuestaria se obtienen soluciones de esquina, como en el caso atemporal.
Observe en la figura 5.16 que la tasa marginal de preferencia temporal en ese paquete óptimo
(C1, C2) es positiva, debido a que el valor absoluto de la restricción presupuestaria es 1 + r > 1. En el

7 En términos del cálculo, la tasa marginal de preferencia temporal está dada por |dC2/dC1|.

05_CHAPTER 5.indd 158 3/6/09 7:53:43 PM


el modelo de eleCCión intertemporal 159

FIGURA 5.16
C2 Asignación
intertemporal óptima
Como en el modelo
atemporal, el paquete
óptimo de consumo
A intertemporal (paquete
C*2
A) se encuentra en la
M2 curva de indiferencia más
alta alcanzable. aquí, eso
I ocurre en el punto de
tangencia.
Pendiente
= –(1 + r)
C1
0 C*1 M1

ejemplo que se representa en el diagrama, el consumidor tiene el mismo ingreso en cada periodo,
pero en el segundo consume un poco más.
Por supuesto, la asignación óptima será diferente para cada consumidor. El óptimo que se pre-
senta en la figura 5.17a, por ejemplo, corresponde a un consumidor cuyas preferencias se inclinan a
favor de un consumo futuro. En cambio, el óptimo de la figura 5.17b, corresponde a un consumidor
al que le interesa más el consumo actual. Pero observe que en el punto óptimo la pendiente de la
curva de indiferencia es la misma en ambos casos. En tanto los consumidores puedan pedir prestado
y prestar a una misma tasa de interés r, la tasa marginal de preferencia temporal en el paquete ópti-
mo será (1 + r) (excepto, por supuesto, en el caso de soluciones de esquina). En el caso de soluciones
interiores, la preferencia temporal positiva es la regla, sin importar las preferencias del consumidor.

FIGURA 5.17
C2 C2 Paciencia e impaciencia
a) el consumidor paciente
pospone la mayor parte del
Paciente Impaciente
consumo para un periodo
futuro. b) el consumidor
impaciente consume más
Ip en el periodo actual. pero
en el equilibrio la tasa
marginal de preferencia
temporal (1 + r) es la
Pendiente Pendiente misma para los dos tipos
= –(1 + r) = –(1 + r) Ii de consumidores.
C1 C1

a) b)

Se acostumbra suponer que tanto en el consumo actual como en el futuro se trata de bienes nor-
males. Por tanto, un aumento en el valor actual del ingreso a lo largo de la vida, si todos los demás
factores permanecen constantes, hará que aumente tanto el consumo actual como el futuro.

Usted tiene un ingreso actual de 100 000 dólares y un ingreso futuro de 154 000 dólares y EJEMPLO 5.3
puede pedir prestado o prestar a una tasa r = 0.1. En estas condiciones usted consume exac-
tamente su ingreso en cada periodo. Verdadero o falso: si r aumenta a r = 0.4, esto hará que
usted ahorre parte de su ingreso actual.

05_CHAPTER 5.indd 159 3/6/09 7:53:45 PM


160 Capítulo 5 apliCaCiones de las teorías de la eleCCión raCional y de la demanda

FIGURA 5.18 C2 (miles)


Efecto de un aumento
en la tasa de interés Área amplificada
Cuando aumenta la 294
tasa de interés, la
restricción presupuestaria 264
intertemporal gira en torno
al punto de dotación actual.
si el punto de dotación D
actual (A) fuera óptimo a D
C*2
una tasa de interés menor,
el nuevo paquete óptimo 154
(D) tendrá menos consumo A A
I2 I2
actual y más consumo I1
futuro.
B

B’

B’ B

C1 (miles)
C*1 100 210 240

En la figura 5.18, la línea B es la restricción presupuestaria inicial. Su abscisa en el origen es el valor


actual del ingreso a lo largo de toda la vida cuando r = 0.1: 100 000 + 154 000/1.1 = 240 000 dólares.
Su ordenada en el origen es el ingreso futuro más (1 + r) multiplicado por el ingreso actual: 154 000
+ (1.1)(100 000) = 264 000 dólares. El paquete óptimo se presenta en A, lo que implica que en A la
TMPT es 1.1. Cuando la tasa de interés aumenta a 0.4, la restricción presupuestaria intertemporal
será B´. Su abscisa en el origen es 100 000 +154 000/1.4 = 210 000 dólares. Su ordenada en el origen es
154 000 + (1.4)(100 000) = 294 000 dólares. Como en A la TMPT es menor que el valor absoluto de la
pendiente de la restricción presupuestaria B´, se concluye que el consumidor estará mejor económi-
camente si consume menos ahora y más en el futuro que lo que consumió en A. El nuevo paquete se
muestra en D en la figura 5.18.

APLICACIÓN: HIPÓTESIS DEL INGRESO PERMANENTE


Y EL CICLO DE VIDA
Los economistas suponían que el consumo actual de una persona dependía sobre todo de su ingreso.
De manera que si una persona recibe una ganancia inesperada, aproximadamente igual a su ingreso
actual, su consumo será más o menos el doble.
Sin embargo, en la década de 1950, Milton Friedman, Franco Modigliani, Richard Brumberg y
otros argumentaron que el modelo de elección intertemporal sugiere otra cosa.8 Para ilustrar esto,
considere un consumidor cuyo ingreso actual y futuro es 120, que puede pedir prestado y prestar a
una tasa r = 0.2. En la figura 5.19 la línea B es la restricción presupuestaria intertemporal del consu-
midor y el paquete óptimo en ella es A. Observe que la abscisa en el origen de B es el valor actual del
ingreso a lo largo de toda la vida, es decir, 120 + (120/1.2) = 220.
Observe lo que ocurre cuando este ingreso actual del consumidor aumenta de 120 a 240. Ahora
su restricción presupuestaria es B´ y el paquete óptimo es D. El efecto de aumentar el ingreso actual
no es sólo aumentar el consumo presente (de 80 a 150) sino también el consumo futuro (de 168 a
228). Dado que las curvas de indiferencia intertemporal muestran tasas marginales decrecientes de

8 Vea Franco Modigliani y R. Brumberg, “Utility Analysis and the Consumption Function: An Interpretation of Cross-Section

Data”, en K. Kurihara (ed.), Post Keynesian Economics, Londres: Allen & Unwin, 1955; y Milton Friedman, A Theory of Consumption
Function, Princeton, NJ: Princeton University Press, 1957.

05_CHAPTER 5.indd 160 3/6/09 7:53:46 PM


el modelo de eleCCión intertemporal 161

FIGURA 5.19
C2 El principal
determinante del
408 I2 consumo actual es el
ingreso permanente y
no ingreso actual
I1
el efecto de un aumento
en el consumo actual (de
120 a 240) se manifestará
264 no sólo como un aumento
D del consumo actual (de
228 80 a 150), sino también en
un aumento del consumo
A futuro (de 168 a 228).
168
F
120
E

B B’
C1
120 220 340
80 150 240

preferencia temporal,9 en general lo mejor que puede hacer el consumidor es no concentrar dema-
siado de su consumo en ningún periodo. Al dividir su ganancia inesperada entre ambos periodos,
puede lograr un mejor resultado.
La hipótesis del ingreso permanente de Friedman dice que el principal determinante del consumo
actual no es el ingreso actual sino lo que él llama el ingreso permanente. En términos del modelo de ingreso permanente valor
elección intertemporal, el ingreso permanente es el valor actual del ingreso a lo largo de toda la vida. actual del ingreso a lo largo de
(Si se sigue el aumento del ingreso actual en la figura 5.19, el ingreso permanente es 240 + 120/1.2 = toda la vida.
340.) Cuando se considera que en realidad el futuro consta no sólo de uno sino de muchos periodos,
resulta claro que el ingreso actual es nada más una fracción pequeña del ingreso permanente. (Por
ejemplo, si hubiera 10 periodos futuros por considerar, entonces un aumento de 10% en el ingreso
actual haría que el ingreso permanente aumentará sólo 2%.)10 Por tanto, dice Friedman, una de-
terminada variación en el ingreso actual dará lugar a un cambio proporcional mucho menor en el
consumo actual, como se vio en la figura 5.19. (La hipótesis del ciclo de vida de Modigliani y Brumberg
cuenta en esencia la misma historia.)

FACTORES RESPONSABLES DE LAS DIFERENCIAS


EN LAS PREFERENCIAS TEMPORALES
La incertidumbre con respecto al futuro es una de las razones para preferir el consumo actual al
consumo futuro. En los países en guerra, por ejemplo, las personas suelen vivir como si no hubiera
un mañana, y en realidad para muchas de ellas no lo hay. En cambio, un clima de paz internacional,
seguridad en el empleo, redes sociales estables, buena salud y otros factores similares tienden a re-
ducir la incertidumbre acerca del futuro, justificando que se le dé un mayor peso al consumo futuro.
Los mapas de indiferencia temporal, como su variedad atemporal, varían también de acuerdo con la
disposición del individuo. Mi primer hijo, por ejemplo, tiene fuertes preferencias temporales positi-
vas en la mayoría de las situaciones. (Sus curvas de indiferencia son muy inclinadas con respecto al

9 La tasa marginal de preferencia temporal decreciente es el análogo intertemporal de la tasa marginal de sustitución decreciente

en el modelo atemporal.
10 Una vez más se supone una tasa de interés r = 0.2.

05_CHAPTER 5.indd 161 3/6/09 7:53:47 PM


162 Capítulo 5 apliCaCiones de las teorías de la eleCCión raCional y de la demanda

eje de consumo actual.) Cuando era pequeño comía primero la parte de la comida que era su favorita
y después se iba comiendo las cosas que le gustaban menos. Había que presionarlo para que comiera
las verduras y aun así las dejaba para el final. Mi segundo hijo es el caso opuesto. Siempre empieza
por lo que menos le gusta y tiene cuidado de dejar para el final lo que más le gusta. Este contraste en
sus conductas en la mesa se extiende casi a todos los demás aspectos de sus vidas.
Las preferencias temporales dependen también de las circunstancias del momento. Estudios
experimentales han aislado ciertas situaciones en las cuales la mayoría de las personas tienen prefe-
rencias temporales muy positivas y otras en las que muestran preferencias temporales fuertemente
negativas. El economista George Loewenstein de la Universidad Carnegie-Mellon, por ejemplo,
pidió a sujetos experimentales que se imaginaran que habían ganado un beso de su estrella de cine
favorita y les preguntó cuándo les gustaría recibirlo. Aun cuando una de las posibilidades era recibirlo
de inmediato, la mayoría prefirió esperar en promedio varios días. Esta elección implica una prefe-
rencia temporal negativa, y Loewenstein explicó que la mayoría de los sujetos únicamente deseaba
un poco de tiempo para saborear por anticipado el beso.11
A otro grupo Loewenstein le pidió que imaginaran que se le iba a dar un fuerte choque eléctrico
y preguntó a sus miembros cuándo preferían recibirlo. Esta vez la mayoría prefirió tener la experien-
cia en ese momento. Al parecer querían pasar el menor tiempo posible temiendo el choque. Pero
dado que un choque eléctrico es un “mal” y no un “bien”, esta elección también es una preferencia
temporal negativa.
Aunque en casos individuales en ocasiones se observan preferencias temporales negativas, y
éstas pueden ser invocadas en muchas personas mediante experimentos adecuados, lo más común
es una preferencia por el consumo actual sobre el consumo futuro. Por ejemplo, si se pone una lata
de nueces de la India enfrente de los sujetos experimentales de Loewenstein, no muchos desearán
esperar unos días para saborear por anticipado el placer de comérselas. Al contrario, las nueces des-
aparecerán en poco tiempo, aun cuando eso signifique perder el apetito para la hora de la comida.
El economista del siglo xix Eugen von Böhm-Bawerk sugirió que una de las razones de tal con-
ducta es que las oportunidades de consumo actual son percibidas directamente por nuestros senti-
dos, mientras que las de consumo futuro sólo pueden imaginarse. El placer de comer las nueces de la
India es tan intenso como inmediato. Aun las personas que preferirían la comida en vez de las nueces
suelen perder el control en la espera. Böhm-Bawerk creía que nuestra “débil facultad telescópica” no
es una buena razón para asignar mayor peso al placer presente que al placer futuro. Dejando a un
lado la incertidumbre, él pensaba que la gente lograría mayor satisfacción en sus vidas si ponderara
igualmente el presente que el futuro.

RESUMEN
• En este capítulo el interés principal lo constituyen las aplicacio- bienestar económico. Este modelo sugiere por qué el índice
nes de la teoría de la elección racional y de la demanda de los de precios al consumidor, una medida de la variación en el cos-
capítulos 3 y 4. Se consideró también el concepto de excedente to de la vida, suele exagerar el verdadero costo de obtener un
del consumidor, que mide la cantidad en que se beneficia un determinado nivel de satisfacción.
consumidor al comprar cierto producto a un determinado • El modelo de elección intertemporal es análogo al de elección
precio. Se vio que el excedente del consumidor puede aproxi- atemporal del capítulo 3. En el caso bidimensional se empieza
marse mediante el área limitada desde arriba por la curva de con una gráfica de mercancía en la que se representa el consu-
demanda individual y desde abajo por el precio de mercado. mo actual y futuro de un bien compuesto. La dotación inicial
Las estructuras de precio en dos partes son un medio por el del consumidor es el punto (M1, M2), que corresponde al ingre-
cual una parte del excedente del consumidor se transfiere del so actual y al ingreso futuro respectivamente. Si el consumi-
comprador al vendedor. dor puede pedir prestado y prestar a una tasa r, su restricción
• El modelo de elección racional también es útil para evaluar de presupuesto intertemporal es entonces la línea que pasa a
los efectos de las variaciones del precio y del ingreso sobre el través del punto de dotación con una pendiente de –(1 + r).

11Vea George Loewenstein, “Anticipation and the Valuation of Delayed Consumption”, en Economic Journal, 97, septiembre de
1987: pp. 666-684.

05_CHAPTER 5.indd 162 3/6/09 7:53:47 PM


problemas 163

El costo de oportunidad de una unidad de consumo actual es pectivamente. En el caso de soluciones interiores, el equilibrio
1 + r unidades de consumo futuro. La abscisa en el origen de se presenta en un punto de tangencia entre la restricción pre-
la restricción presupuestaria intertemporal es el valor actual supuestaria intertemporal y una curva de indiferencia. Debido
de todo el ingreso actual y futuro, a lo que también se le lla- a que la pendiente de la restricción presupuestaria intertempo-
ma valor actual del bienestar económico a lo largo de toda la ral es mayor que 1 cuando r > 0, los consumidores presentan
vida. preferencia temporal positiva en el equilibrio, sin importar
• Las preferencias intertemporales del consumidor se represen- cuál sea la forma de su curva de indiferencia.
tan mediante un mapa de indiferencia que tiene las mismas • Una aplicación importante del modelo de elección intertem-
propiedades que en el caso atemporal. Se dice que un consu- poral es la que se refiere al estudio de las decisiones acerca de
midor presenta preferencia temporal positiva, neutra o nega- cuánto ahorrar. Las hipótesis del ingreso permanente y del
tiva en un punto si su tasa marginal de preferencia temporal ciclo de vida emplean este modelo para demostrar que lo que
(el valor absoluto de la pendiente de su curva de indiferencia) influye en el consumo actual (y por lo tanto el ahorro) es el
en ese punto es mayor que 1, igual que 1 o menor que 1, res- valor actual de la riqueza a lo largo de toda la vida.

pREgUNtaS dE REpaSo
1. Con sus propias palabras explique por qué un impuesto a la ga- más la tasa de interés es el análogo al cociente de precio relati-
solina, cuyos ingresos recaudados se devuelven al consumidor vo en el modelo de elección del consumidor del capítulo 3.
en forma de una cantidad fija de dinero, reducirá, de cualquier 6. Los autobuses son por lo general consumidores de energía
manera, el consumo de gasolina. más eficientes que los automóviles; sin embargo, a pesar del
2. En sus propias palabras explique qué es un esquema de precio aumento en los precios de la energía, la tendencia en los últi-
en dos partes y por qué lo usan los vendedores. mos 30 años ha sido la de una disminución en la proporción de
3. ¿Considera usted que la educación universitaria tiene una elas- personas que usan el autobús para ir y volver del trabajo. ¿Por
ticidad-precio (colegiatura) de la demanda alta o baja? qué?
4. En sus propias palabras explique por qué incluso los bebedores 7. Jennifer, cuyo salario anual es de 20 000 dólares, gana 25 000
habituales serán sensibles a un aumento del precio del alcohol. dólares en la lotería. Explique por qué lo más probable es que
5. Explique por qué en el modelo de elección intertemporal 1 durante el próximo año no gaste todo su premio.

pRoblEMaS
1. Use un diagrama como el de la figura 5.2 para explicar por qué con nuestro actual método de finan-
ciamiento* educativo es más probable que una familia rica mande a sus hijos a una escuela privada
que una familia pobre.
2. Cuando el precio de la gasolina es 1 dólar/gal su consumo es 1 000 gal/año. Y entonces pasan dos
cosas: 1) el precio de la gasolina sube a 2 dólares/gal y 2) un tío muere y le deja a usted una herencia
de 1 000 dólares/año. Si no hay ningún otro cambio ni en los precios ni en el ingreso, ¿con estos dos
cambios queda usted en una mejor situación económica que antes?
3. La demanda de fresas de Larry está dada por P = 4 – (Q/2), donde P es el precio de las fresas (dólares/
pinta) y Q es la cantidad (pintas/semana). Si se supone que el efecto ingreso es despreciable, ¿cuánto
perjudicará a Larry que el precio de las fresas suba de 1 dólar/pinta a 2 dólares/pinta?
4. El único club de renta de DVD accesible para usted cobra 4 dólares por película diarios, si su curva de
demanda de renta de películas está dada por P = 20 – 2Q, donde P es el precio de renta (dólares/día)
y Q es la cantidad demandada (películas/año), ¿cuál es la cuota máxima que estaría dispuesto a pagar
por ser socio del club?
5. Jane gasta todo su ingreso en hamburguesas y caviar. Su curva de demanda de caviar era inelástica
para todos los precios de caviar. Por desgracia, un accidente en la planta nuclear hizo que el suminis-
tro de caviar disminuyera y el precio se elevara. ¿Qué pasó con el consumo de hamburguesas de Jane?
Explique. (Nota: suponga que el accidente no tuvo ningún efecto sobre el precio de las hamburguesas
ni en el gusto de Jane por el caviar.)
6. Jones gasta todo su ingreso en dos bienes, X y Y. Los precios que pagó y las cantidades que consumió
el año pasado son las siguientes: PX = 15, X = 20, PY = 25 y Y = 30. Si los precios del año siguiente son
PX = 6 y PY = 30 y el ingreso de Jones es de 1 020, ¿estará económicamente mejor o peor que el año
anterior? (Suponga que sus gustos no cambian.)
* En algunos países de habla hispana se utiliza el término financiación.

05_CHAPTER 5.indd 163 3/6/09 7:53:48 PM


164 Capítulo 5 apliCaCiones de las teorías de la eleCCión raCional y de la demanda

7. Smith vive en un mundo en el que sólo hay dos periodos. Su ingreso en cada periodo es 210 dólares y
lo recibe al inicio del mismo. Si la tasa de interés, expresada como una fracción, es 0.05 dls/periodo,
¿cuál es el valor actual de su ingreso a lo largo de su vida? Trace su restricción presupuestaria inter-
temporal. En los mismos ejes de coordenadas trace la restricción presupuestaria intertemporal de
Smith si r = 0.20.
8. Suponga que en el problema 7 Smith considera el consumo actual y el futuro como sustitutos perfec-
tos uno a uno. Encuentre su paquete de consumo óptimo.
9. Suponga que en el problema 7 Smith considera el consumo actual y futuro como complementos
perfectos 1 a 1. Encuentre su paquete de consumo óptimo.
10. Karen gana 75 000 dólares en el periodo actual y en el periodo futuro ganará 75 000 dólares.
a) Suponga que los anteriores son los únicos dos periodos y que en los bancos de su país la tasa de
interés por préstamo o ahorro es r = 0, trace su restricción presupuestaria intertemporal.
b) Ahora suponga que los bancos ofrecen depósitos con 10% de interés si se contratan en el periodo
actual y préstamos con esta misma tasa de interés. Trace su nueva restricción presupuestaria inter-
temporal.
11. Encuentre el valor actual de 50 000 dólares que serán recibidos después de un año si la tasa de interés
anual es
a) 8%.
b) 10%.
c) 12%.
12. Crusoe vivirá éste y el siguiente periodo como único habitante de su isla. Su único ingreso son 100
cocos que cosecha al inicio de cada periodo. Los cocos que no se consumen en el periodo actual se
echan a perder a una tasa de 10% por periodo.
a) Trace la restricción presupuestaria intertemporal de Crusoe. ¿Cuál será su consumo en cada
periodo si él considera el consumo futuro como un sustituto perfecto uno a uno del consumo
actual?
b) ¿Cuál será su consumo en cada periodo si él considera 0.8 unidades de consumo futuro como
equivalentes a una unidad de consumo actual?
13. Kathy gana 55 000 dólares en el periodo actual y en el periodo futuro ganará 60 000. ¿Cuál es la tasa
de interés máxima que le permitiría gastar 105 000 dólares en el periodo actual? ¿Cuál es la tasa míni-
ma de interés que le permitiría gastar 120 500 dólares en el periodo futuro?
14. Smith recibe 100 dólares de ingreso en este periodo y 100 dólares en el siguiente. A una tasa de interés
de 10%, consume todo su ingreso en cada periodo. Smith tiene una tasa marginal decreciente de pre-
ferencia temporal entre consumo en el periodo siguiente y consumo en el periodo actual. Verdadero o
falso: si la tasa de interés sube 20%, Smith ahorrará parte de su ingreso de este periodo. Explique.
15. A los precios actuales, la vivienda tiene un precio de 50 dólares por unidad, y el bien compuesto de 1
dólar por unidad. Un rico benefactor ha otorgado a Joe, una persona sin un centavo, una unidad de vi-
vienda y 50 unidades de bien compuesto. Ahora el precio de la vivienda disminuye a la mitad. Verdadero
o falso: Joe está en una mejor posición económica después de esta variación del precio. Explique.
*16. Tom y Karen son economistas. Para regular el uso que su hijo Harry hace del automóvil de la familia
le cobran una tarifa de uso de 20 centavos de dólar/milla. A ese precio Harry usa el automóvil más de
lo que ellos desearían, pero no están muy dispuestos a tener problemas con él por elevar este precio.
De manera que le hacen la siguiente pregunta: ¿cuál es el aumento mínimo en tu pensión semanal
que aceptarías a cambio de un aumento de la tarifa de uso del automóvil a 40 centavos de dólar/milla?
Harry, que se sabe que nunca miente y que tiene gustos convencionales contesta 10 dólares/semana.
a) Si Tom y Karen aumentan la pensión de Harry a 10 dólares/semana y le cobran 40 centavos de
dólar por milla, ¿usará el automóvil menos que antes? Explique.
b) ¿El rendimiento del precio adicional por milla será mayor, menor o igual que 10 dólares/semana?
Explique.
*17. Todos los compradores de libros tienen las mismas preferencias, y en las condiciones actuales, las
personas que compran libros usados en 22 dólares obtienen la misma utilidad que aquellos que com-
pran libros nuevos en 50 dólares. La tasa de interés anual es 10% y en la compraventa de libros usados

* Los problemas marcados con un asterisco (*) son más difíciles.

05_CHAPTER 5.indd 164 3/6/09 7:53:48 PM


respuestas a los eJerCiCios del Capítulo 165

no hay costos de transacción. Producir cada libro nuevo cuesta m dólares y el libro dura exactamente
dos años.
a) ¿Cuánto es lo máximo que un comprador pagaría por usar un libro nuevo durante un año?
b) ¿Cuál es el valor mínimo de m para que un editor decidiera que valdría la pena imprimir un libro
con tinta que desapareciera después de un año contado a partir del momento en que se compra
un libro nuevo, eliminando así el mercado de libros usados? (Suponga que la eliminación del mer-
cado de libros usados duplicará exactamente las ventas del editor.)
*18. Herb quiere trabajar exactamente 12 h/semana. Puede trabajar como empleado en una librería a
6 dólares/h o como tutor de estudiantes de primer año de economía. Aparte de las diferencias en
el pago, cualquiera de los dos empleos le da lo mismo. Uno de cada tres estudiantes de primer año
tienen una demanda de clases particulares dada por P = 10 – Q, donde P es el precio en dólares por
hora y Q es la cantidad de horas por semana. Si Herb tiene la opción de establecer una tarifa en dos
partes por sus servicios de tutoría, ¿cuántas horas de clases por semana deberá dar y cuántas horas
deberá trabajar en la librería? Si decide dar clases, ¿cuál debe ser la estructura de su tarifa?
†19. La Universidad de Cornell sigue con su política actual de permitir que los hijos de sus maestros
asistan a esta universidad sin pagar colegiatura. Suponga que la curva de demanda de los hijos de
los maestros de Cornell por plazas en otras universidades está dada por P =30 – 5Q0 , donde P es
la colegiatura que cobran otras universidades (en miles de dólares) y Q0 es la cantidad de hijos de
maestros de Cornell que asisten a esas universidades. La Universidad de Cornell está pensando en
una propuesta para subsidiar una proporción k de la colegiatura que pagan los hijos de sus profeso-
res en otras universidades. Suponga que la Universidad de Cornell sabe que puede ocupar todas las
plazas disponibles con estudiantes que no sean hijos de sus profesores y que paguen una colegiatura
de 45 000 dólares/año. Suponga que todos los hijos de los profesores que no asisten a otras univer-
sidades asistirán a Cornell, ¿qué valor k maximizará los ingresos netos por colegiatura, sin tener en
cuenta otros subsidios, si el precio de ésta en otras universidades es de 24 000 dólares/año?
†20. ¿Cómo variaría su respuesta al problema anterior si la colegiatura en otras universidades fuera de
12 000 dólares/año? ¿Cuál es la interpretación económica de un valor k mayor que 1?
*21. Harry tiene un pequeño cine cuyos clientes tienen todos gustos idénticos. El precio de entrada al cine
es 5 dólares y la curva de demanda de palomitas de cada cliente está dada por Pc = 4 – Qc donde Pc es
el precio de las palomitas en dólares y Qc es la cantidad de palomitas en cuartos de galón. Si el costo
marginal de permitir que otro cliente vea la película es cero y el costo marginal de las palomitas es un
dólar, ¿a que precio deberá Harry vender las entradas y las palomitas si su objetivo es maximizar las
utilidades? (Suponga que Harry puede hacer publicidad de su estructura de precios, sin costo, entre
clientes potenciales.)

RESpUEStaS a loS EjERcicioS dEl capítUlo


5.1. El excedente inicial del consumidor en P = 3 dólares (y Q = 7 gal/semana) es ½(10 –3)7 = 24.50
dólares/semana. El excedente del consumidor al precio más alto P' = 4 dólares (y Q' = 6 gal/semana)
es CS' = ½(10 –4)6 = 18 dólares/semana. La pérdida en el excedente el consumidor está dada por el
área DCEF, que es igual a 24.5 – 18 = 6.50 dólares/semana.
Precio (dólares/gal)

10 A Pérdida en el excedente
del consumidor

D C
4 E
3
F

Gasolina (gal/semana)
6 7 10
* Los problemas marcados con un asterisco (*) son más difíciles.
† Los problemas marcados con una espada (†) se resuelven con más facilidad empleando el cálculo.

05_CHAPTER 5.indd 165 3/6/09 7:53:49 PM


166 Capítulo 5 apliCaCiones de las teorías de la eleCCión raCional y de la demanda

5.2 Ahora la cuota máxima anual está dada por el área del triángulo AB'C', que es CS = ½(50 – 20)120 =
1 800/año.

Precio (dólares/h)

A
50

B’
20
C’

Tiempo de
cancha de tenis (h/año)
0 120 200

5.3 En el diagrama siguiente se muestran las dos líneas presupuestarias y el paquete óptimo del último
año. Si se ve el punto de tangencia más de cerca (área amplificada) se observa que este año Jones está
en condiciones de comprar un paquete que prefiere en lugar del que compró el año pasado.

Paquete de
este año

45 Paquete del
año pasado
42

30 Área amplificada

Paquete del
año pasado

Paquete de
este año

X
20 60 70

5.4 Sean de nuevo P1 y Q1 el precio y la cantidad originales y ∆P y ∆Q las variaciones respectivas, siendo
∆Q < 0. El hecho de que el nuevo ingreso total sea 2% mayor que antes indica que

[1.10 (Q1 + ∆Q) – 1.00 (Q1)] = 0.02 [1.00 (Q1)].

Al reordenar los términos se obtiene

0.08 Q1 = –1.10 ∆Q,

05_CHAPTER 5.indd 166 3/6/09 7:53:51 PM


respuestas a los eJerCiCios del Capítulo 167

de donde se obtiene

∆Q/Q1 = –0.08/1.10.

Y cómo se sabe que ∆P/P1 = 0.10, se tiene

ε = (∆Q/Q1) / (∆P/P1) = (–0.08/1.10)/0.10 = –0.727.

5.5 PV = 50 000 + 42 000/1.05 = 90 000 dólares. Consumo máximo futuro = 50 000(1.05) + 42 000 =
94 500 dólares. La ecuación de su restricción presupuestaria intertemporal es C2 = 94 500 –1.05 C1.

05_CHAPTER 5.indd 167 3/6/09 7:53:51 PM


05_CHAPTER 5.indd 168 3/6/09 7:53:51 PM
Capítulo

6
Economía dE la información
y ElEcción bajo
incErtidumbrE

C
uando un sapo y su rival compiten por la misma hembra, cada uno se enfrenta a una deci-
sión estratégica importante. ¿Deberá luchar por ella o buscar otra? Luchar es arriesgarse a
un daño, pero seguir buscando también implica costos. Por lo menos, supone un gasto en
tiempo. Y no hay ninguna garantía de que la siguiente hembra potencial no sea también elegida por
otro sapo.
Para tomar esta decisión, la evaluación que hace cada sapo de la capacidad de lucha de los otros
es muy importante. Si el rival es considerablemente mayor la posibilidad de imponerse será pequeña
y la de salir dañado será grande, de manera que lo prudente será seguir buscando. Si no es así, puede
que valga la pena luchar.
Con frecuencia, estas decisiones suelen tomarse de noche, cuando es difícil ver, de manera que
los sapos encuentran útil apoyarse en diversos indicios no visuales. El más útil es el tono de voz del
sapo rival. En general, los sapos más grandes tienen cuerdas vocales mayores y más gruesas y por
consiguiente un croar grave. Si en la noche un sapo oye un croar grave, puede inferir que es produ-
cido por un sapo grande. En efecto, los experimentos han demostrado que es mucho más probable
que los sapos se vean intimidados por un croar grave que por uno agudo.1

1
Véase John Krebs y Richard Dawkins, “Animal Signals: Mind Reading and Manipulation”, en J. Krebs y N. Davies (editores), Beha-
vioral Ecology: An Evolutionary Approach, 2a. ed., Sunderland, MA: Sinauer Associates, 1984.

169

06_CHAPTER 6.indd 169 3/6/09 7:55:02 PM


170 Capítulo 6 eConomía de la informaCión y eleCCión bajo inCertidumbre

VISIÓN PRELIMINAR DEL CAPÍTULO


La información es muy importante en la toma decisiones, no sólo en el caso de los sapos, sino
también en el caso de los consumidores y las empresas. Los modelos que se han visto en los ca-
pítulos anteriores suponen información perfecta. Sin embargo, tristemente, en la práctica se está
mal informado. En la primera parte de este capítulo la atención se centra en la pregunta de cómo
obtener y evaluar información fiable. Debido a que muchas de las cuestiones que interesarán en
este capítulo surgen de manera sencilla en el contexto del problema de los sapos, éste proporciona
un punto de partida adecuado para el análisis. Se verá que los principios que rigen la comunicación
entre los sapos sirven para entender asuntos tan diversos como las garantías de los productos,
las prácticas de contratación e, incluso, la manera en que las personas eligen a sus socios para
sus relaciones personales. Se examinará también la discriminación estadística, proceso mediante
el cual se emplean las características grupales como ayuda para estimar las peculiaridades de un
determinado individuo.
Aunque la calidad de las decisiones suele mejorar mediante una recolección inteligente de infor-
mación, es casi imposible obtener toda la que es potencialmente importante. En la segunda parte de
este capítulo la tarea es ampliar el modelo de elección del consumidor del capítulo 3 de manera que
se puedan tomar decisiones en condiciones de incertidumbre.

ECONOMÍA DE LA INFORMACIÓN
COMUNICACIÓN ENTRE ADVERSARIOS POTENCIALES
Los problemas de comunicación entre grupos cuyos objetivos están en conflicto potencial son por
completo diferentes de aquellos en los que las partes tienen objetivos comunes. El problema de los
sapos en busca de hembras cae dentro de la primera categoría, como por lo general es el caso de
los dos grupos que participan en un intercambio económico. El vendedor, por ejemplo, suele estar
motivado a exagerar la calidad de sus productos. El comprador, de la igual manera, suele tener un
incentivo para subestimar la cantidad que está dispuesto a pagar por ellos. También el empleado
potencial puede verse inclinado a exagerar sus cualidades para ocupar un puesto.
En cambio, parece claro que los compañeros de bridge tienen objetivos comunes. Cuando un ju-
gador utiliza los símbolos habituales de una apuesta para decirle algo a su compañero, éste toma ese
mensaje tal cual. Ninguno de ellos tiene nada que ganar si engaña al otro. Aquí la comunicación es
tan sólo un problema de transferencia de información. Basta que el mensaje sea descifrable. Aparte
de posibles errores, no se duda de su credibilidad.
Sin embargo, cuando los intereses de los supuestos comunicadores están en conflicto, o en
posible conflicto, la lógica es muy distinta. Suponga, por ejemplo, que un jugador de bridge le
susurra a su oponente de la izquierda, “yo siempre actúo de manera conservadora”. ¿Qué debe
hacer el oponente ante tal observación? El comentario es perfectamente inteligible. Sin embargo, si
se considera que todos los contrincantes son racionales, la relación entre ellos es de tal naturaleza
que la observación puede no contener ninguna información real. Si ser conocido como un pujador
conservador fuera ventajoso, eso sería razón suficiente para que un jugador se llamara a sí mis-
mo conservador, sea cierto o no. Esta observación no es ni creíble ni no creíble; no proporciona
ninguna información.
El comprador avispado sabe que debe tener cuidado con las afirmaciones exageradas acerca
de la calidad de los productos. Pero, ¿cómo puede distinguir con exactitud un buen producto de
uno malo? ¿De manera similar, cómo convence un productor a un rival potencial de que reducirá
sus precios de manera drástica si el competidor ingresa a su mercado? Afirmaciones como “bajaré
mis precios” son problemáticas en el mismo sentido que en el caso de los contrincantes del bridge.
Dado que el productor tiene razones para emitir tales afirmaciones, sean o no verdaderas, éstas no
transmiten ninguna información.
señalización comunicación El ejemplo de los sapos ilustra dos características importantes de la señalización entre adversa-
que transmite información. rios potenciales: 1) las señales deben ser difíciles de imitar, y 2) si algunos individuos emplean señales

06_CHAPTER 6.indd 170 3/6/09 7:55:03 PM


eConomía de la informaCión 171

que transmiten información favorable acerca de ellos mismos, otros se verán forzados a revelar in-
formación aun cuando esto sea mucho menos favorable. Cada uno de estos principios es importante
para entender cómo obtienen e interpretan la información los agentes económicos. Para empezar se
plantea cada principio en términos de su aplicación en el ejemplo de los sapos y después se examina-
rá su aplicación a diversos contextos económicos.

EL PRINCIPIO “DIFÍCIL DE IMITAR”


Para que una señal entre adversarios sea creíble, debe ser costoso (o, en sentido más general, difícil)
imitarla. Si los sapos pequeños pudieran imitar con facilidad el croar grave, éste dejaría de ser ca-
racterístico de los sapos grandes. Pero los sapos pequeños no pueden hacer esto. Los sapos grandes
tienen una ventaja natural, y es ese sencillo factor el que les permite emitir un croar más grave como
una señal fiable.
Este principio “difícil de imitar” tiene una clara aplicación a las señales entre personas. Funcio- principio “difícil de imitar”
na, por ejemplo, en el siguiente episodio de Fatal Vision de Joe McGinnis. El capitán Jeffrey MacDo- para que una señal enviada a
nald, médico del ejército, ha sido declarado sospechoso de haber matado a su esposa y a sus hijas. El un adversario sea creíble, debe
ser costoso imitarla.
ejército le ha asignado un abogado del ejército como defensor. Sin embargo, mientras tanto, la madre
de MacDonald contrata a Bernard Segal, un renombrado abogado de Filadelfia, para que defienda a
su hijo. Cuando Segal llama a MacDonald, que se encuentra en Fort Bragg, Carolina del Norte, para
presentarse, la primera pregunta que hace a MacDonald es acerca del abogado del ejército:

“¿Están bien lustrados sus zapatos?”

“¿Qué?” MacDonald no podía creer lo que oía. Había sido acusado nada menos que de ase-
sinar a su esposa y a sus hijas, y en esta primera conversación con el abogado de Filadelfia,
que se suponía había sido contratado para poner las cosas en orden, la primera pregunta que
le hacía era acerca del lustre de los zapatos del otro abogado.

Segal repitió la pregunta. “Y esta vez”, dijo después, “casi pude oír a Jeff riendo en el telé-
fono. Con lo que no sólo supe que tenía un cliente inteligente, sino que captaba todo con
rapidez. Él contestó que no, que los zapatos del abogado estaban algo desaliñados. Enton-
ces dije, ‘muy bien, en ese caso, confíe en él. Coopere con él hasta que yo llegue’. El punto
es, como se podrá ver, que si un abogado del ejército tiene zapatos bien lustrados, significa
que trata de impresionar al sistema. Y en esa situación, tratar de quedar bien con el siste-
ma —el cual ya había declarado especial interés en que su cliente fuera declarado culpable
al anunciar públicamente que lo encontraba sospechoso—, no le iba a hacer ningún bien a
Jeff. Unos zapatos no lustrados pueden significar que le importa más ser abogado.”

Es obvio que el estado de los zapatos del abogado no era una indicación perfecta de sus priorida-
des en la vida. Sin embargo, proporcionaba por lo menos cierto indicio para pensar que no era sólo un
lacayo del ejército. Un abogado que usara zapatos desaliñados nada más para no dar la impresión de
estar buscando una mejor posición en el ejército, ciertamente no lograría progresar. De manera que
las únicas personas que con seguridad pueden enviar tal señal son aquellas a las que en realidad les
preocupa más su papel como abogados.
A continuación se presentan algunas aplicaciones económicas del principio “difícil de imitar”:

Garantía de la calidad de un producto


Muchos productos son tan complejos que los consumidores no pueden inspeccionar de manera
directa su calidad. En tales casos, las empresas que ofrecen alta calidad necesitan algún medio de
comunicar este hecho a sus clientes potenciales. De otra manera no les será posible cobrar precios
suficientemente altos para compensar los costos agregados.

06_CHAPTER 6.indd 171 3/6/09 7:55:03 PM


172 Capítulo 6 eConomía de la informaCión y eleCCión bajo inCertidumbre

Una manera de resolver este problema es que la empresa desarrolle la reputación de proporcio-
nar alta calidad.2 Pero las circunstancias no siempre permiten a las empresas hacer esto. Considere
el caso de alguien que vende relojes de pulsera en la calle de una gran ciudad. Si una “empresa” de
este tipo decide dejar el negocio, lo puede hacer sin tener casi ninguna pérdida. No tiene un local ni
un equipo costoso ni clientes leales por quienes preocuparse –—en realidad, ningún costo de inver-
sión—. Aun cuando un vendedor haya proporcionado productos de calidad en una misma esquina
durante años, eso no es ninguna garantía de que mañana esté ahí con su negocio. Y si él estuviera
planeando dejar el negocio, su incentivo sería vender la mercancía de la menor calidad posible. En
pocas palabras, una empresa sin ninguna apuesta por el futuro tiene una dificultad inherente para
convencer a sus clientes potenciales de que cumplirá sus promesas.
Las motivaciones de las empresas con extensos costos hundidos son muy distintas. Si una em-
presa de este tipo cierra su negocio, pierde el valor sustancial de los bienes y servicios que no pueda
liquidar. Por consiguiente, el interés material de estas empresas es hacer todo lo que puedan para
seguir activas. Y si los clientes saben eso, pueden confiar mucho más en la promesa de un producto
de alta calidad. Si una compañía de este tipo cobra un precio correspondiente a una alta calidad y
provee mercancía de mala calidad, tendrá muy pocas posibilidades de sobrevivir, y de esta manera
habrá incurrido en vano en costos hundidos.
Estas observaciones sugieren una razón para creer que aquellos productos a los que se les hace
una gran publicidad resultan ser productos de alta calidad, tal como lo proclaman sus lemas. Una
amplia campaña de publicidad a nivel nacional es un costo hundido cuyo valor se perderá para siem-
pre si la empresa cierra el negocio. Habiendo hecho una inversión así, tienen todos los motivos para
producir. Que las empresas supongan que muchos consumidores saben esto es evidente por el hecho
de que en su publicidad en revistas dicen con frecuencia “…como se ha visto en la televisión nacio-
nal…”.

Elección de un empleado fiable


Hay múltiples situaciones en las que los empleados tienen oportunidad de engañar a sus empleado-
res. Muchas actividades productivas tendrían que abandonarse si las empresas fueran incapaces de
contratar empleados que no aprovecharían tales situaciones para engañar. Las empresas necesitan
una señal que identifique a los futuros empleados como fiables. Una base de tal señal puede ser la
relación entre el carácter de una persona y los costos o beneficios de la pertenencia a grupos espe-
cíficos. Por ejemplo, a las personas fiables por lo general les agrada trabajar como voluntarios en
organizaciones de caridad; en cambio, las personas poco fiables tienden a considerarlo como algo
molesto. En tales casos, los grupos a los que las personas deciden unirse trasmitirán información
estadísticamente fiable acerca del carácter de los individuos.
Esta idea parece confirmarse en el procedimiento mediante el cual muchas parejas de profesio-
nales de la ciudad de Nueva York contratan cuidadores para sus hijos. El cuidado de los niños es una
de esas tareas en las que la fiabilidad es de gran importancia debido a que es difícil vigilar en forma
directa la actuación de la persona. La simple razón de necesitar a alguien que cuide a los hijos indica,
después de todo, que usted no puede estar ahí para hacerlo por sí mismo. Amargas experiencias
han convencido a muchos neoyorquinos de que el mercado laboral local no es un buen lugar para
encontrar personas que trabajen de manera confiable sin supervisión.
La solución que han adoptado muchas parejas es buscar cuidadores mediante anuncios en los
periódicos de Salt Lake City; han descubierto que las personas que han crecido en la tradición mor-
mona son confiables en un grado que el neoyorquino promedio no lo es. Esta señal funciona porque
una persona que sólo quiera aparentar ser confiable encontrará desagradable, si no imposible, haber
estado en la tradición mormona, en la cual existe un intenso y continuo adoctrinamiento moral,
experiencia que la mayoría de las personas oportunistas encontrarían muy difícil de soportar. Así

2
Este ejemplo está basado en Benjamin Klein y Keith Leffler, “The Role of Market Forces in Assuring Contractual Performance”,
en Journal of Political Economy, agosto de 1981.

06_CHAPTER 6.indd 172 3/6/09 7:55:03 PM


eConomía de la informaCión 173

como el sonido grave del croar del sapo es una indicación de su tamaño, la pertenencia a la tradición
mormona es una buena señal de confiabilidad porque sería muy difícil para una persona oportunista
simular esto.

Elección de un empleado inteligente y trabajador


Como última ilustración del principio “difícil de imitar” considere un título universitario obtenido
con honores en una universidad de élite. Los empleadores buscan gente que sea inteligente y que
esté dispuesta a trabajar duro. Por supuesto, hay muchas personas en el mundo que tienen estas
dos características pero que no poseen un título de una universidad de prestigio. A pesar de lo ante-
rior, los empleadores tienen razón en suponer que una persona que ostenta un grado universitario
de este tipo debe ser inteligente y trabajadora, porque en el caso de no tener esta combinación de
características difícilmente podría haberse graduado. Nadie cuestiona el hecho de que los egresados
de estas instituciones resultan ser empleados productivos. Pero existe un fuerte debate acerca del
grado en que la asistencia a estas instituciones en realidad causa alta productividad. Las personas que
piensan esto apuntan al hecho de que los graduados de instituciones de élite ganan salarios mucho
más altos. Sin embargo, los escépticos alertan que esta diferencia no puede atribuirse más que a la
calidad de su educación. El problema es que los estudiantes de las mejores instituciones han sido sin
duda más productivos desde siempre. Después de todo, estas instituciones seleccionan con cuidado a
sus estudiantes y aceptan sólo a aquellos que tienen las mejores calificaciones.

PRINCIPIO DE APERTURA TOTAL


El ejemplo de los sapos puede servir para ilustrar un segundo principio importante ilustrado que principio de apertura total
puede llamarse principio de apertura total, el cual dice que si algunos individuos sacan provecho los individuos se ven obligados
mostrando el aspecto favorable de una característica, otros se verán forzados a mostrar sus aspectos a mostrar incluso cualidades
desfavorables, de manera que
menos favorables de ese rasgo. Este principio ayuda a responder a la pregunta inicial de qué objeto
su silencio no se interprete
tiene que croen los sapos pequeños.3 Croando les hacen saber a los demás sapos lo pequeños que
como que tienen algo aún peor
son. ¿Por qué no quedarse callados y dejar a los demás con la duda? que esconder.
Suponga que todos los sapos cuyo croar sea más alto que un umbral fijo permanecieran en
silencio. Imagine un índice de 0 a 10 que midiera el tono del croar de un sapo, siendo 10 el tono más
alto y 0 el más bajo, y suponga que, de manera arbitraria, los sapos cuyo valor índice sea superior a 6
se queden callados (vea figura 6.1).

FIGURA 6.1
Umbral inicial para croar La información
implícita en el silencio
Croan No croan Si sólo los sapos con un
croar menor que 6.0 se
molestan en croar, aquellos
que permanecen en silencio
0 1 2 3 4 5 6 7 8 9 10 revelan que, en promedio,
Tono bajo Tono alto su tono es superior a 6.0.

Es fácil ver por qué un patrón de este tipo sería inherentemente inestable. Considere un sapo
cuyo índice sea 6.1, apenas arriba del límite. Si permanece en silencio, ¿qué pensarán los demás sa-
pos? Por experiencia, sabrán que debido a que está en silencio, el tono de su croar debe ser mayor que
6. Pero, ¿cuánto?
Al carecer de información sobre este determinado sapo no pueden decir nada con exactitud.
Pero pueden hacer una estimación estadística. Suponga que los sapos estuvieran distribuidos de ma-
nera uniforme a lo largo de la escala del tono de croar. Esto significaría que si de toda la población

3
Vea Krebs y Dawkins, “Animal Signals”.

06_CHAPTER 6.indd 173 3/6/09 7:55:04 PM


174 Capítulo 6 eConomía de la informaCión y eleCCión bajo inCertidumbre

de sapos se toma uno al azar habría la misma posibilidad de que el tono de su croar fuera igual a
cualquiera de los valores de la escala. Pero, fijando el umbral del croar en 6, un sapo que permanece
en silencio será sistemáticamente diferente de uno elegido al azar. En particular, la experiencia indi-
cará que el índice promedio de los sapos que permanecen en silencio es 8 (el punto medio entre 6 y
10). Cualquier sapo cuyo índice sea menor que 8, por el hecho de su silencio, daría la impresión de
ser menor de lo que en realidad es. Así, el sapo cuyo índice es 6.1 hará mucho mejor en croar que en
abstenerse.
Por lo tanto, si el índice para permanecer en silencio fuera 6, para todos los sapos con un índice
menor que 8 valdría la pena croar. Si lo hacen, el umbral se desplazaría de 6 a 8. Pero un umbral de
8 tampoco sería estable. Si el límite estuviera en ese nivel, para todos los sapos con un índice menor
que 9 sería conveniente croar. Por razones similares, cualquier umbral menor que 10 está destinado
a elevarse. Este proceso no se debe a que los sapos pequeños quieran llamar la atención acerca de su
pequeñez mediante su croar. Más bien, se ven forzados a croar para evitar parecer más pequeños de
lo que son.
El principio de apertura-total tiene su origen en el hecho de que no todos los adversarios po-
tenciales tienen acceso a la misma información. En el caso de los sapos, la asimetría está en que los
sapos que se quedan en silencio saben con exactitud de qué tamaño son mientras que sus adversarios
sólo pueden hacer una estimación aproximada. Como demuestra la siguiente ilustración, asimetrías
similares dan lugar a señales importantes entre los agentes económicos.

Garantías de los productos


La asimetría de la información ayuda a explicar por qué los productores de artículos de baja calidad
revelan ese hecho ofreciendo una garantía muy limitada. Aquí la asimetría es que los productores
saben mucho mejor que los consumidores qué tan buenos son sus productos. Las empresas que
saben que tienen los mejores productos están muy motivadas a revelar esa información. Una manera
creíble de hacerlo es una garantía generosa. (Este recurso es creíble debido al principio “difícil de
imitar”: un producto de baja calidad se echa a perder con frecuencia, por lo que resulta muy costoso
ofrecer una garantía generosa.)
Una vez que aparece un producto con una garantía a largo plazo, los consumidores saben de
inmediato, no sólo mucho más acerca de su calidad, sino también acerca de la del resto de los pro-
ductos. En particular, saben que los productos que no tienen garantía no pueden ser de alta calidad.
A falta de cualquier otra información acerca de los productos sin garantía, un consumidor prudente
estima su calidad como la del promedio de tales productos. Pero esto significa que los consumidores
subestimarán la calidad de aquellos productos que sean sólo un poco inferiores a los mejores.
Considere la situación que afronta el productor del segundo mejor producto. Si continúa sin
ofrecer garantía, el consumidor pensará que su producto es peor de lo que en realidad es. De acuerdo
con lo anterior, el fabricante hará mejor en ofrecer una garantía; pero como su producto es de una
calidad un poco menor, los términos de aquélla no pueden ser tan generosos como los del mejor
producto.
Si se garantizan los productos que siguen de los mejores, la clase del resto de los productos
sin garantía es de una calidad promedio aún menor que antes. El proceso de apertura se pone en
marcha, y al final, todos los productores tienen que ofrecer alguna garantía o vivir sabiendo que los
consumidores clasificarán sus productos como los de menor calidad. Los términos de la garantía,
en general, serán menos generosos cuanto menor sea la calidad de un producto. Es evidente que
los productores no quieren anunciar su baja calidad ofreciendo garantías poco generosas, pero su
problema es que si no lo hacen, harán que los consumidores consideren sus niveles de calidad aún
menores de lo que en realidad son.

Regulación del entrevistador de empleo


Otra aplicación ilustrativa del principio de apertura total es la dificultad que supone para el gobierno
desarrollar políticas de protección de información que las empresas pueden exigir a una persona
que solicita empleo. Considere, por ejemplo, la legislación que prohíbe a los empleadores preguntar
acerca del estado civil y de los planes de tener hijos.

06_CHAPTER 6.indd 174 3/6/09 7:55:05 PM


eConomía de la informaCión 175

Antes de la entrada en vigor de la legislación, los empleadores solicitaban cierta información


de manera rutinaria, en especial a las mujeres, que ayudaba predecir la posibilidad de retiro de la
fuerza laboral; la razón por la cual los empleadores preguntaban lo anterior era evitar invertir en
la contratación y capacitación de trabajadores que no se quedarían por mucho tiempo en la empresa.
Dado que el estado civil es difícil de imitar (pocas personas se abstendrán de casarse con el fin de que
parezca menos probable que se retiren de la fuerza laboral), puede ser una señal entre partes cuyos
intereses pueden estar en conflicto. El objetivo de esta legislación era evitar que los empleadores
favorecieran a candidatos en razón de su estado civil.
Sin embargo, para lograr esto no es suficiente prohibir sólo a los empleadores que pregunten
acerca del estado civil, ya que si una mujer se da cuenta de que sus categorías particulares la colocan
en una posición más favorable para ser contratada, se verá motivada a ofrecer información acerca
de ella sin que se lo pidan. Esto pone en marcha el familiar proceso de apertura en el que toda la
información, salvo la menos favorable, será ofrecida en forma espontánea por el solicitante del em-
pleo. Del candidato que no da información de manera voluntaria, aun cuando ésta no sea favorable,
simplemente se supone que pertenece a la categoría menos deseable. Si la legislación desea lograr su
propósito, tendría que, de alguna manera, prohibir a quienes solicitan un empleo dar información
por su propia voluntad.
Las personas y las cosas pertenecen a categorías, algunas de las cuales, por consenso, son mejo-
res que otras. Ser confiable es mejor que no serlo, ser trabajador es mejor que ser perezoso, etc. El
mensaje general del principio de apertura total es que la falta de evidencias de que algo pertenece
a una categoría preferente suele sugerir que forma parte de la menos favorecida. Planteado de esta
manera, el principio parece transparente y sencillo. Sin embargo, sus consecuencias no suelen ser
nada obvias.

Principio de los limones


Por ejemplo, como lo ilustra a continuación El naturalista económico, el principio de apertura total
ayuda a resolver la vieja paradoja de por qué un automóvil nuevo suele perder gran parte de su valor
de mercado en el preciso momento en que sale de la concesionaria.

¿Por qué los automóviles usados “seminuevos” se venden mucho más baratos que los nuevos?
Un automóvil nuevo comprado el lunes en 30 000 dólares, puede venderse el siguiente viernes como
usado en sólo 22 000 dólares. Es claro que esta gran disminución del precio no se explica porque el EL NATURALISTA
automóvil haya sufrido una depreciación física mayor de 25% en menos de una semana. ECONÓMICO
Los economistas se han esforzado durante años para entender este curioso evento. Apartándose
de su posición profesional característica, algunos han especulado 6.1
que el consumidor tiene prejuicios irracionales contra los auto-
móviles usados. Sin embargo, George Akerlof, un economista de
Berkeley, sugiere que no son necesarias las supersticiones miste-
riosas. En su artículo “The Market for ‘Lemons?’” (“El mercado
de ‘limones’”), uno de los documentos de economía más citados
en las últimas décadas, proporciona una ingeniosa explicación al-
ternativa (que fue el primer planteamiento claro del principio de
apertura total).4
Akerlof parte del supuesto de que, grosso modo, puede decir-
se que los automóviles nuevos son de dos tipos: los buenos y los
“limones”. Ambos son, en apariencia, parecidos. Pero el dueño de
un automóvil sabe, por experiencia, de qué tipo es el suyo. Dado
que los futuros clientes no pueden saber de qué clase es cada uno,
los buenos automóviles y los limones deberían venderse al mismo
precio. Se tiende a pensar que este precio común es un promedio
ponderado de los valores de los dos tipos respectivos, con un peso

4
George Akerlof, “The Market for ‘Lemons’”, en Quarterly Journal of Economics, 1970. ¿por qué los automóviles usados se venden a precios tan bajos?

06_CHAPTER 6.indd 175 3/6/09 7:55:06 PM


176 Capítulo 6 eConomía de la informaCión y eleCCión bajo inCertidumbre

de ponderación correspondiente a la proporción de cada tipo. De hecho, en el mercado de automóvi-


les nuevos esta intuición prueba ser más o menos correcta.
En el mercado de automóviles usados las cosas funcionan de manera diferente, como los autos
buenos son más estimados por sus propietarios que los limones por los suyos, hay una mayor pro-
porción de limones que sale con rapidez al mercado. Cuando los compradores observan este patrón,
el precio de los automóviles usados comienza a disminuir, lo cual, a su vez, refuerza la tendencia
original de los propietarios de automóviles buenos a no venderlos. En casos extremos, los únicos
automóviles a la venta serán los limones.
La idea de Akerlof surge de la observación de que el simple hecho de que un automóvil esté a
la venta constituye una información importante acerca de su calidad. Esto no quiere decir que tener
un limón sea la única razón que lleva a la gente a vender sus automóviles. Pero, aun si ésta fuera
una razón menor, evitaría que los dueños de automóviles buenos pudieran obtener el valor de su
automóvil en el mercado de segunda mano. Y esto puede ser todo lo que se necesita para iniciar el
ahora ya familiar proceso de apertura. En realidad, los automóviles que no tienen ningún problema
rara vez llegan al mercado de automóviles usados, excepto como resultado de una fuerte presión por
circunstancias externas. (“Por viaje al extranjero vendo Volvo station wagon” o “Por daño en una
mano, vendo mi BMW con caja de velocidades estándar”.)
La explicación de Akerlof reivindica la intuición de que la depreciación física es una razón insu-
ficiente para la notable diferencia de precios entre un automóvil nuevo y uno usado. La brecha puede
entenderse mejor pensando en el hecho de que los autos ofrecidos a la venta, tomados como grupo,
tienen una calidad promedio inferior que los que no están en venta.

El estigma de los recién llegados


El principio de apertura total también sirve para entender por qué antes era más difícil que ahora
escapar de las consecuencias de una mala reputación mudándose de lugar. En el ambiente actual,
donde la movilidad es alta, una persona deshonesta puede sentirse atraída por la estrategia de mu-
darse a un nuevo lugar cada vez que es sorprendida. Pero en tiempos en que la movilidad era menor,
esta estrategia debió ser mucho menos efectiva, ya que cuando las sociedades eran más estables, las
personas confiables tenían mucho más que ganar permaneciendo en un lugar y cosechando el fruto
de su buena reputación adquirida. De la misma manera que al propietario de un buen automóvil
no le interesa venderlo, a una persona honesta no le interesaba mudarse de lugar. En ambientes en
general estables, las personas que se mudaban de lugar, como los automóviles usados, resultaban
sospechosas. Sin embargo, hoy en día hay tantas presiones externas para mudarse que el simple he-
cho de ser un recién llegado casi nunca acarrea tales suposiciones.

ELEGIR UNA RELACIÓN


La mayoría de las personas busca una pareja que sea amable, comprensiva, sana, inteligente, atracti-
va, etc. La información acerca del atractivo físico se puede obtener de un vistazo. Pero gran parte de
las demás características que la gente busca en una pareja son difíciles de observar y por lo general la
gente se apoya en señales de comportamiento que las indiquen. Para que dichas señales sean efecti-
vas, deben ser difíciles de imitar. Alguien que busque, por ejemplo, una pareja muy disciplinada, hará
bien en poner especial interés en personas que corran maratones en menos de 2½ horas. Incluso el
grado de interés que una persona muestra en una posible pareja suele revelar mucho de ella misma.

¿Por qué la timidez es una cualidad atractiva?


El comediante Groucho Marx dijo alguna vez que no ingresaría en ningún club del que él fuera
miembro. Seguir una estrategia similar al buscar una relación obviamente daría como resultado la
frustración. Sin embargo, Groucho quería decir algo. Puede haber buenas razones para evadir a un
EL NATURALISTA
buscador bastante atractivo que sea demasiado ansioso. Si esta persona es tan atractiva como parece,
ECONÓMICO ¿por qué esa ansiedad? Tal actitud indica con frecuencia valores no favorables en características difí-
6.2 ciles de observar. Por lo tanto, las propiedades de señales eficientes muestran con claridad por qué la

06_CHAPTER 6.indd 176 3/6/09 7:55:06 PM


eConomía de la informaCión 177

timidez, dentro de ciertos límites, es tan atractiva. Al parecer, las personas ansiosas encuentran difícil
disimular su ansiedad.

© the new yorker Collection 1991 mick Stevens from caartoonbank.com. all rights reserved

Estas mismas propiedades también tienen consecuencias para las condiciones institucionales de
acuerdo con las cuales la gente busca una pareja. Un problema, con frecuencia ignorado, de la vida
urbana moderna es que los pesados horarios de trabajo dificultan que las personas se encuentren.
En respuesta a esto, los servicios comerciales de citas ofrecen ayuda para que parejas con intereses y
gustos claramente similares se encuentren. Las personas que emplean estos servicios se ahorran, de
esta manera, el tiempo y el gasto de tener que conocer personas con las que tienen pocos intereses
en común. También evitan la incertidumbre de saber si el candidato estará interesado en conocer a
alguien. Y aunque estos servicios comerciales de citas llegan a resultar en matrimonios, a muchas
personas les ha parecido una mala inversión. La razón aparente es que, sin proponérselo, estos
servicios actúan como un dispositivo que identifica a las personas que tienen problema para iniciar
relaciones. Es seguro que el problema de algunos participantes es precisamente estar muy ocupados;
pero, con frecuencia, el empleo de estos servicios es resultado de problemas de personalidad o de
otras dificultades más preocupantes. Quienes emplean estos servicios son, en realidad, lo que dice la
publicidad, fáciles de hallar; pero la teoría de la señalización indica que, en promedio, no constituyen
un hallazgo valioso.

CONSUMO OSTENTOSO COMO INDICACIÓN DE CAPACIDAD


Suponga que usted ha sido acusado de manera injusta de un crimen serio y que está buscando un
abogado que lo represente. Suponga también que tiene que elegir entre dos abogados que, hasta
donde usted sabe, son idénticos en todos los aspectos, excepto en su nivel de consumo. Uno usa un
raído traje de poliéster y llega al juzgado en un Chevrolet sedán de hace 15 años, todo carcomido
por el óxido. El otro lleva un traje de casimir impecable, hecho a la medida y conduce un BMW 745i
nuevo. ¿Cuál contrataría usted?
El principio de señalización indica que el último abogado tal vez es el más indicado. La razón es
que, en un mercado competitivo, es factible que la capacidad de un abogado se refleje en su ingreso,
el cual, a su vez, está correlacionado de manera positiva con su consumo. Por supuesto, no hay ga-
rantía de que el abogado que gasta más tenga mayor capacidad. Pero como en otras situaciones que
implican riesgo, aquí también la gente debe guiarse por las leyes de la probabilidad. Y éstas indican en
forma inequívoca que se elija el abogado mejor vestido.
Cuando se trata de decisiones importantes acerca de personas que no se conocen bien, incluso
débiles señales de capacidad suelen ser decisivas. Las decisiones de empleo son un ejemplo obvio.

06_CHAPTER 6.indd 177 3/6/09 7:55:07 PM


178 Capítulo 6 eConomía de la informaCión y eleCCión bajo inCertidumbre

La primera impresión cuenta mucho durante una entrevista de trabajo, y como no dejan de
repetir los fabricantes de ropa, nunca hay otra oportunidad de dar una buena primera impresión. Los
asesores de colocación siempre han subrayado la importancia de la calidad del atuendo en el proceso
de búsqueda de empleo. Aun cuando el empleador sepa lo bueno que es un candidato, dará gran
importancia a la impresión que la persona les da a los demás. En especial en trabajos que implican un
amplio contacto con personas extrañas que no saben lo bueno que es el empleado.
A juzgar por sus gastos, muchas personas solteras parecen creer que sus prospectos de matri-
monio dependen críticamente de la ropa que usen y del automóvil que tengan. A primera vista esto
resulta curioso porque se supone que cuando la mayoría de las personas se casan ya se conocen lo
suficiente para que tales cosas no sean determinantes. Pero también muchas parejas potenciales han
sido rechazadas desde un principio por parecer “inadecuadas”. Los símbolos del éxito no garantizan
que una persona se case bien, pero refuerzan sus oportunidades.
La importancia de los bienes de consumo como señal de habilidad difiere según las profesiones.
Entre los investigadores universitarios el ingreso y las habilidades que cuentan no están fuertemente
correlacionadas y a la mayoría de los profesores universitarios no les preocupa tener un automóvil de
hace 15 años si éste les da un buen servicio. Pero para un aspirante a trabajar en un banco de inversio-
nes sería un grave error llegar con sus clientes potenciales en un automóvil así.

¿Por qué los habitantes de pueblos pequeños gastan menos en su vestuario profesional que
los de ciudades grandes?
Como en el caso de un banquero inversionista que espera atraer nuevos clientes, es evidente que
el incentivo para que una persona gaste más dinero en el consumo de bienes estará inversamente
EL NATURALISTA relacionado con la cantidad y confiabilidad de la información que otras personas tengan acerca de
ECONÓMICO sus habilidades. Cuanto más se conozca a alguien, el consumo de bienes observables influirá menos
6.3 en la valoración que los demás hacen de él. Esto puede ayudar a explicar por qué los patrones de
consumo en los pueblos pequeños, que tienen redes sociales altamente estables, son tan diferentes
de los que predominan en las grandes ciudades. El vestuario que un profesional “necesita” en la
ciudad de Iowa, por ejemplo, cuesta menos de la mitad de lo que costaría en Manhattan o en Los
Ángeles. De manera similar, debido a que la confiabilidad de la información referente a una persona
aumenta con la edad, la cantidad de ingreso dedicada al consumo de estos bienes debería disminuir
con el tiempo. Los patrones de gasto más maduros de la
gente mayor pueden indicar tanto los beneficios decre-
cientes de mostrar sus habilidades como el mayor grado
de sabiduría que da la edad.
Observe que el consumo ostentoso como señal de
habilidad lo coloca frente a un dilema. El concepto de
vestuario de buen gusto, al igual que el de un auto veloz,
indiscutiblemente es relativo. Para dar una buena prime-
ra impresión, no es suficiente utilizar ropa limpia y en
orden. Es necesario usar algo que sea mejor que lo que
usan casi todos los demás. Esto crea un incentivo para
que todo el mundo ahorre menos y gaste más en ropa.
Pero cuando todos gastan más en ropa, la apariencia re-
lativa no cambia. El consumo ostentoso es entonces en
esencia un bien posicional, uno cuyo atractivo depende
de su comparación con los bienes de la misma categoría
consumidos por otros. En la conocida metáfora del esta-
dio, todos los espectadores se ponen de pie para poder ver
una jugada emocionante, sólo para darse cuenta de que la
vista no mejora. Aquí también el resultado agregado de
¿por qué los abogados de los pueblos chicos compran trajes más baratos la conducta racional individual es muy diferente de lo
que los abogados de las ciudades grandes? que la gente espera.

06_CHAPTER 6.indd 178 3/6/09 7:55:08 PM


eleCCión Con inCertidumbre 179

Como grupo, puede ser mejor gastar mucho menos en consumo ostentoso y ahorrar mucho
más para la jubilación. Pero si un consumo suntuoso tiene una gran influencia sobre la estimación de
las capacidades, a ningún individuo que actúe solo le conviene tomar esta medida.

ELECCIÓN CON INCERTIDUMBRE


No importa cuánto tiempo y energía se invierta en obtener información, la mayor parte de las veces
hay que elegir sin conocer bien las alternativas relevantes. La elección entre, por ejemplo, ir a esquiar
o comprar un nuevo reproductor de DVD puede hacerse de manera más inteligente si se consultan
fuentes como el Servicio Meteorológico Nacional o la Revista del Consumidor. Y, sin embargo, en últi-
mo término, no se puede excluir la posibilidad de que haga mal tiempo o de que un chip salga malo.
Tales riesgos son una característica primordial de muchas de nuestras decisiones más importantes. El
objetivo de lo que resta de esta sección es ampliar el modelo de elección del consumidor del capítulo
3 para tomar en cuenta estos riesgos.

PROBABILIDAD Y VALOR ESPERADO


Cuando se elige una universidad, una persona para casarse, una ocupación o incluso qué película ir
a ver, es posible que en el momento de la elección existan características importantes de las que no
se tenga certeza. Algunas veces la elección es entre alternativas que implican el mismo riesgo (por
ejemplo la elección entre dos citas a ciegas); otras veces hay que elegir entre una alternativa poco
conocida y otra más o menos familiar (por ejemplo, cuando hay que decidir si cambiarse a otra
universidad o permanecer en la que se está).
Las decisiones económicas que se hacen con incertidumbre son en esencia apuestas. Se tienen
varias ideas intuitivas acerca de lo que hace atractiva una apuesta, y muchas de ellas se llevan al
ámbito de las elecciones económicas. Para ilustrar esto considere la siguiente serie de apuestas rela-
cionadas con el lanzamiento de una moneda no cargada.

Apuesta 1. Si cae cara usted gana 100 dólares; si cae cruz usted pierde 0.50.

Ésta es una apuesta que rara vez se le haría en un casino. Si gana la apuesta el premio es 200
veces mayor que la cantidad que tendrá que pagar si la pierde, y los dos resultados son igualmente
probables. Sólo las personas a las que su religión les prohíba hacer apuestas rechazarían esta apuesta,
pero aun para ellas la decisión sería difícil. (Puede que, pensándolo bien, encuentren que sea mejor
aceptar la apuesta y donarla a la caridad.)

Apuesta 2. Si cae cara usted gana 200 dólares; si cae cruz pierde 100.
Si gana la apuesta, la cantidad que gana es el doble de lo que podría perder. Es claro que esta
apuesta es menos atractiva que la primera, pero también sería aceptada por muchos.
Por último, considere una tercera apuesta. Ésta es igual que la segunda salvo que las cantidades
están multiplicadas por 100.

Apuesta 3. Si cae cara usted gana 20 000 dólares; si cae cruz pierde 10 000, pero si pierde puede pagar
en pequeños pagos mensuales a lo largo de 30 años.

Puesta a prueba, la mayoría de las personas rechazará esta apuesta, aun cuando las cantidades
estén en la misma proporción que en la apuesta 2. La tarea de la teoría de la elección con incertidum-
bre es explicar este patrón de conducta. valor esperado suma de
Una propiedad importante de una apuesta es su valor esperado, un promedio ponderado de todos los resultados posibles,
todos los posibles resultados, en el que los pesos son las probabilidades respectivas. La probabilidad ponderados mediante sus
de que una moneda no cargada caiga con la cara hacia arriba después de lanzarla al aire es 1/2. Una respectivas probabilidades de
manera de interpretar esto es decir que si la moneda se lanzara un número muy grande de veces, la ocurrencia.

06_CHAPTER 6.indd 179 3/6/09 7:55:08 PM


180 Capítulo 6 eConomía de la informaCión y eleCCión bajo inCertidumbre

mitad de ellas caería cara y la otra mitad cruz. Por lo tanto, las tres apuestas antes descritas tienen los
valores esperados siguientes:

EV1 = (1/2)100 + (1/2)(–0.50) = 49.75 dólares; (6.1)

EV2 = (1/2)200 + (1/2)(–100) = 50 dólares, (6.2)

EV3 = (1/2)20 000 + (1/2)(10 000) = 5 000 dólares, (6.3)

donde EVi significa valor esperado de la apuesta i, donde i = 1, 2, 3.


Por supuesto, una apuesta es más atractiva si su valor esperado es positivo. Pero de acuerdo con
la manera en que la mayoría de las personas responden a estas tres apuestas, es claro que tener un
valor esperado positivo no hace que una apuesta sea atractiva. Al contrario, la apuesta 3 es la que
tiene el mayor valor esperado de las tres y, sin embargo, es la que tiene menor posibilidad de ser
aceptada. En cambio, la apuesta 1, cuyo valor esperado es el menor, es la que tiene más posibilidades
de ser aceptada.
Ahora, es obvio que la lección no establece que tener un valor esperado elevado sea algo malo
en sí mismo, sino que además de considerar el valor esperado de una apuesta, la mayoría de las
personas consideran también cómo se sentirá con respecto a los posibles resultados. Lo que hace a la
apuesta 3 tan poco atractiva para la mayoría de las personas es que las posibilidades de obtener el des-
agradable resultado de perder 10 000 dólares están 1-1. La apuesta 2 también encierra una posibilidad
desagradable: 1-1 de perder 100 dólares; pero es un resultado que muchas personas no encuentran
gravoso. La apuesta 1 es, con mucho, la que representa una más fácil elección para la mayoría de las
personas debido a que su resultado positivo es suficientemente grande, mientras que su resultado
negativo es demasiado pequeño para tener importancia.

EL MODELO DE UTILIDAD ESPERADA


DE VON NEUMANN-MORGENSTERN
La teoría económica formal de la elección entre alternativas inciertas fue desarrollada por John von
Neumann, un distinguido matemático del Insititute for Advanced Study y por Oskar Morgenstern,
un economista de Princeton. Su premisa central es que la gente elige la alternativa que tiene una
utilidad esperada la utilidad utilidad esperada más alta. Su teoría de la maximización de la utilidad esperada supone una función
esperada de una apuesta es el de utilidad U que asigna una medida numérica a la satisfacción asociada con diferentes resultados. La
valor de la utilidad esperada en utilidad esperada de una apuesta es el valor de utilidad esperado en todos los resultados posibles.
todos los resultados posibles. Para simplificar, se considerará que el resultado de una apuesta está definido sólo por la cantidad
total de riqueza a la que corresponde. Por ejemplo, si un consumidor que tiene una riqueza inicial
de 1 000 acepta la apuesta 1 y gana, el resultado será una riqueza total de 1 000 + 100 = 1 100 y la
utilidad del consumidor será U(1100). Si pierde, su riqueza será 1 000 – 0.50 = 999.50 y su utilidad
será U(999.50). En general, si M0 es la riqueza inicial del consumidor, la utilidad esperada de aceptar
la primera apuesta será

EU1 = (1/2) U(M0 + 100) + (1/2) U(M0 – 0.50). (6.4)

Si se rechaza la apuesta 1, la utilidad esperada será la utilidad de la riqueza M0, que es, U(M0).
Ante esta elección, el criterio de utilidad esperada de Von Neumann-Morgenstern dice que una
apuesta debe aceptarse si y sólo si EU1 es mayor que U(M0).

06_CHAPTER 6.indd 180 3/6/09 7:55:08 PM


eleCCión Con inCertidumbre 181

Suponga que la función de utilidad de Smith está dada por U(M) = √ M. Si la riqueza inicial de EJEMPLO 6.1
Smith es 10 000, ¿cuál de las apuestas anteriores tiene la utilidad esperada más alta?

Las tres utilidades esperadas son:

EU111 =
EU = ((11// 22)) 10 100 +
10 100 999.50 =
+ ((11// 22)) 99 999.50 = 100
100..248
248,,

EU 222 =
EU (111// 22)) 10
= (1 10 200 + (1/ 2) 9 900 = 100.247,
200 + (1/ 2) 9 900 = 100.247,
y

EU 333 =
EU 00 +
30 0000
= ((11// 22)) 30 + ((11// 22)) 00 =
= 86
86..603
603,,

de manera que para Smith la apuesta 1 es la más atractiva.

La idea clave de la teoría es que los valores esperados de los resultados de un conjunto de alternativas no
necesariamente tienen el mismo orden que las utilidades esperadas de esas alternativas. Las diferencias entre
estos órdenes surgen debido a que la utilidad suele ser una función no lineal de la riqueza final. En el
caso empírico más común, se supone que la utilidad es una función cóncava de la riqueza total, lo cual utilidad marginal
significa que la función de utilidad tiene el perfil característico que se muestra en la figura 6.2. De decreciente en el caso de una
manera formal se dice que una función U(M) es cóncava si para cualquier par de valores M1 y M2 la función de utilidad definida con
función se encuentra por encima de la línea que une los puntos [M1, U(M1)] y [M2, U(M2)]. La función respecto a la riqueza, aquella
de utilidad U = √M es una función cóncava de M. Una función de utilidad que es cóncava en M se dice función en la que la utilidad
que muestra una utilidad marginal decreciente de la riqueza. La utilidad marginal es simplemente marginal disminuye a medida
la pendiente de la función de utilidad5 y una función de utilidad con utilidad marginal decreciente es que la riqueza aumenta.
aquella cuya pendiente disminuye a medida que M aumenta. De manera intuitiva, el significado de
aversión al riesgo
utilidad marginal decreciente de la riqueza es que cuanta más riqueza tiene un consumidor, menor preferencias descritas
será el aumento de utilidad que ocasiona un aumento de una unidad de riqueza. mediante una función de
Las personas cuyas funciones de utilidad son cóncavas en riqueza total se dice que tienen aver- utilidad con utilidad marginal
sión al riesgo, lo cual significa que siempre rechazarán una apuesta cuyo valor esperado sea cero. A decreciente de la riqueza.
las apuestas cuyo valor esperado es cero se les llama apuestas justas.
Considere, por ejemplo, una apuesta G en la cual usted gana 30 dólares si cae cara y pierde 30 apuesta justa apuesta cuyo
dólares si cae cruz. El valor esperado de esta apuesta es (1/2)30 + (1/2)(–30) = 0, de manera que es valor esperado es cero.

FIGURA 6.2
Una función de utilidad
U(M) cóncava
todo arco de una función
de utilidad cóncava se
U = U(M) encuentra por encima de la
U(M2) línea correspondiente.

U(M1)

M
M1 M2

5
Vea en el apéndice del capítulo 3 una explicación más amplia de la utilidad marginal.

06_CHAPTER 6.indd 181 3/6/09 7:55:10 PM


182 Capítulo 6 eConomía de la informaCión y eleCCión bajo inCertidumbre

una apuesta justa. Para una persona cuya riqueza inicial es 40 y cuya función de utilidad está dada
por U = U(M), la utilidad esperada de esta apuesta es

EUG = 0.5U(40 – 30) + 0.5U(40 + 30) = 0.5 U(10) + 0.5U(70). (6.5)

En cualquier apuesta justa, el valor esperado de la riqueza si se acepta la apuesta es igual al


valor seguro de la riqueza si se rechaza la apuesta. En este caso, el valor de la riqueza si se acepta la
apuesta es igual a 40, si se rechaza se tendrá una riqueza de 40, lo que da una utilidad igual a U(40).
La teoría de la utilidad esperada dice que si EUG > U(40) debe aceptarse la apuesta, si no es así debe
rechazarse.
La utilidad esperada de una apuesta tiene una interpretación geométrica sencilla. Primero se
traza una línea que una los puntos de la función de utilidad que corresponden a perder la apues-
ta y a ganarla, respectivamente (es decir, los puntos A y C de la figura 6.3). En la función de utilidad
que muestra la figura 6.3, la utilidad esperada de la apuesta es igual a 0.5(18) + 0.5 (38) = 28. Obser-
ve que este valor corresponde al punto de la línea que se encuentra entre A y C directamente sobre
el valor esperado de la riqueza con la apuesta (40). Observe que la utilidad esperada de rechazar la
apuesta es U(40) = 32, que con toda claridad es mayor que la utilidad esperada de la apuesta misma.

FIGURA 6.3
Una persona que tiene
aversión al riesgo U
siempre rechazará una
apuesta justa C U = U(M)
la utilidad esperada de una 38
apuesta se encuentra en la
línea que une los puntos 32
A y C. Si la probabilidad
de ganar es 1/2, la utilidad 28
esperada se encuentra en
el punto medio entre A
y C. Como un punto en A
el arco de una función 18
cóncava siempre está por
encima del correspondiente
punto en la línea, la utilidad
esperada en una apuesta
justa siempre será menor
que la utilidad de rechazar
la apuesta. M
10 40 52 70

En efecto, de acuerdo con la figura 6.3 es evidente que las personas con aversión al riesgo no sólo
rechazarán apuestas justas sino incluso algunas que tengan valor esperado positivo. En el caso de la
función de utilidad que se muestra, todas las apuestas que den como resultado un valor esperado
de la riqueza final menor a 52 dan una utilidad esperada más baja que la de quedarse con la riqueza
inicial de 40.
Las apuestas hasta ahora consideradas se deciden mediante el lanzamiento de una moneda. La
probabilidad de obtener un buen resultado es 1/2; la probabilidad de tener un mal resultado es 1/2.
Sin embargo, en general la probabilidad de ganar una apuesta puede ser cualquier número entre 0 y
1. Pero como ilustran el ejemplo y el ejercicio siguientes, la utilidad esperada de una apuesta siempre
puede interpretarse como un punto en la línea que une los puntos correspondientes a ganar y a
perder, aun cuando la probabilidad de ganar sea distinta de 1/2.

06_CHAPTER 6.indd 182 3/6/09 7:55:11 PM


eleCCión Con inCertidumbre 183

Su función de utilidad es U(M) = √ M. y su riqueza inicial es 36. ¿Aceptaría una apuesta en la EJEMPLO 6.2
que gane 13 con probabilidad de 2/3 y pierda 11 con probabilidad de 1/3?

La utilidad esperada de esta apuesta está dada por

EU G = ( 2 / 3) 36 + 13 + (1/ 3) 36 −11 = 14 / 3 + 5 / 3 = 19 / 3.

Si usted rechaza la apuesta, su utilidad será √ 36 = 6, la cual es menor que 19/3, de manera que
debe aceptar la apuesta.

EJERCICIO 6.1
Grafique la función de utilidad del ejemplo 6.2 para 0 < M < 50. En la función de utilidad localice
los puntos que corresponden a ganar y a perder la apuesta del ejemplo 6.2.Trace la línea entre
estos dos puntos e indique como C el punto correspondiente a tener como resultado ganar y
como A el punto correspondiente a tener como resultado perder. ¿Qué fracción de AC hay que
recorrer desde C antes de alcanzar la utilidad esperada de la apuesta?

La regla general ilustrada mediante el ejercicio 6.1 es que si la probabilidad de ganar es p y la


probabilidad de perder es 1 – p, entonces la utilidad esperada se encuentra en una fracción 1 – p a la
izquierda del punto C, el punto final correspondiente a ganar en la línea que une los puntos de ganar
y perder sobre la función de utilidad.
Una razón intuitiva para suponer que la mayoría de las personas tienen aversión al riesgo es
que los aumentos a la riqueza total dan utilidades marginales decrecientes; lo cual, a su vez, signifi-
ca que cuanto mayor sea la riqueza de un consumidor, menor será el aumento en su utilidad ocasio-
nado por un incremento de una unidad en su riqueza. La mayoría de las personas están de acuerdo
con la idea de que 100 dólares extra significan más para una persona cuya riqueza total sea 4 000
dólares que para una cuya riqueza total sea un millón de dólares. Observe que esta idea intuitiva es
equivalente a decir que la función de utilidad es cóncava en la riqueza total; lo que a su vez implica
que una determinada ganancia en la riqueza produce una ganancia de utilidad menor que la mer-
ma que ocasionaría una pérdida comparable en la riqueza.
Si la gente tiene aversión al riesgo es, por supuesto, una cuestión empírica. Se sabe que por lo
menos algunas personas no tienen aversión al riesgo en ciertas ocasiones (como aquellas que escalan
escarpados acantilados o que vuelan con ala delta). En efecto, algunos de nosotros algunas veces no
tenemos aversión al riesgo (por ejemplo, cuando jugamos ruleta en un casino o cualquier otro juego
de azar con un valor esperado negativo).
Considere una persona con una riqueza inicial M0 que se encuentra ante una apuesta que paga
B con probabilidad 1/2 y –B con probabilidad 1/2. Si esta persona siente atracción al riesgo, su fun- atracción al riesgo
ción de utilidad será como la que se muestra en la figura 6.4. Esta función es convexa en riqueza total, preferencias descritas por
lo cual implica que la utilidad esperada de aceptar una oferta justa, EUG, será mayor que la utilidad de una función de utilidad con
rechazarla, U(M0). Geométricamente, en una función de utilidad convexa la pendiente se incrementa utilidad marginal creciente de
la riqueza.
a medida que aumenta la riqueza total.

EJERCICIO 6.2
Considere una persona con una riqueza total de 100 ante una oportunidad de ganar 20 con
probabilidad de 1/2 y de perder 20 con probabilidad de 1/2. Si la función de utilidad de la perso-
na está dada por U(M) =M2, ¿aceptará la apuesta?

Por último, se dice que una persona es indiferente al riesgo si, en general, le es igual aceptar indiferencia al riesgo
o rechazar una apuesta justa. La función de utilidad de una persona indiferente al riesgo será lineal, preferencias descritas por
como la que se muestra en la figura 6.5. una función de utilidad con
utilidad marginal de la riqueza
constante.

06_CHAPTER 6.indd 183 3/6/09 7:55:12 PM


184 Capítulo 6 eConomía de la informaCión y eleCCión bajo inCertidumbre

FIGURA 6.4
U
La función de utilidad
de una persona amante
del riesgo es convexa en
riqueza total U = U(M)
todo arco de una función
convexa se encuentra
por debajo de la línea
correspondiente. para
un amante del riesgo, la U(M0 + B)
utilidad esperada de una
apuesta justa, euG, siempre EUG
será mayor que la utilidad U(M0)
de rechazar la apuesta, U(M0 – B)
U(M0). M
M0 – B M0 M0 + B

FIGURA 6.5 U
Indiferencia al riesgo
a un consumidor
indiferente al riesgo le da lo
U = U(M)
mismo aceptar o rechazar U(M0 + B)
una apuesta justa, debido
a que la utilidad esperada
de aceptarla, euG, es igual EUG = U(M0)
que la utilidad segura de
rechazarla, U(M0).
U(M0 – B)

M
M0 – B M0 M0 + B

EJERCICIO 6.3
Considere una persona con una riqueza total de 100 ante una oportunidad de ganar 20 con
probabilidad de 1/2 y de perder 20 con probabilidad de 1/2. Si la función de utilidad de esta
persona está dada por U(M) = M, ¿aceptará la apuesta?

EJEMPLO 6.3 Suponga que se sabe que cierta fracción z de todas las computadoras personales tiene algún
defecto. Sin embargo, no pueden identificarse, y sólo sus dueños saben que tienen alguna falla.
Los consumidores son indiferentes al riesgo y valoran una computadora sin defectos en 2 000
dólares. Las computadoras no se deprecian físicamente con el uso; las nuevas se venden en
1 000 dólares, las usadas en 500. ¿Cuál es el valor de z?

De acuerdo con el principio de los limones, se sabe que todas las computadoras usadas que están a
la venta deben tener algún defecto. (Los propietarios de computadoras que no tienen defectos
no las venderán por lo que obtendrán de ellas en el mercado de computadoras usadas, de manera que
las conservarán.) Por consiguiente, el precio de una computadora usada es igual al valor de una
nueva con defectos. (Recuerde que ser de segunda mano no hace que la máquina se deprecie.) Como
el consumidor es indiferente al riesgo, el precio de una computadora nueva, 1 000 dólares, es sólo
un promedio ponderado de los valores de las computadoras sin defectos y con defectos, y los pesos
de ponderación son las probabilidades respectivas. De manera que se tiene
1 000 = 500z + 2 000 (1 – z), (6.6)
de donde se despeja z para obtener z = 2/3.

06_CHAPTER 6.indd 184 3/6/09 7:55:14 PM


eleCCión Con inCertidumbre 185

EJERCICIO 6.4
Suponga que una de cada cuatro computadoras personales nuevas tiene algún defecto. Sin
embargo, sólo sus dueños pueden identificar las que tienen algún defecto. Los consumidores
son indiferentes al riesgo y valoran una computadora sin defectos en 2 000 dólares. Las compu-
tadoras no se deprecian físicamente con el uso. Si las usadas se venden en 600 dólares, ¿en
cuánto se venden las nuevas?

En las apuestas consideradas hasta ahora ha habido sólo dos resultados. Sin embargo, en ge-
neral, una apuesta puede tener cualquier cantidad de resultados posibles. El valor esperado de una
apuesta con más de dos resultados es, como antes, la suma ponderada de los resultados posibles y
los pesos son, una vez más, las probabilidades respectivas. Por ejemplo, el valor esperado de una
apuesta en la que haya tres posibles resultados, B1, B2 y B3, cuyas probabilidades de ocurrir sean p1,
p2 y p3, respectivamente, es p1B1 + p2B2 + p3B3. Como las probabilidades deben sumar 1, se sabe que
p3 = (1 – p1 – p2). Por lo tanto la utilidad esperada de esta apuesta es p1 U(B1) + p2 U(B2) + (1 – p1 – p2)
U(B3).
Para entender mejor los conceptos del modelo de utilidad esperada, es útil realizar algunos
ejemplos numéricos sencillos. En el siguiente ejemplo se usa el diagrama de árbol para ilustrar los
resultados y las probabilidades en una decisión con incertidumbre.

Sarah tiene una función de utilidad de Von Neumann-Morgenstern dada por U = 1 – 1/M, don- EJEMPLO 6.4
de M es el valor presente de su ingreso de por vida. Si Sarah decide ser maestra, tendrá un
ingreso M = 5 con probabilidad 1. Si Sarah decide ser actriz y se convierte en una estrella de
cine tendrá un ingreso M = 400, pero si no lo logra su ingreso será de M = 2. La probabilidad
de que se convierte en estrella de cine es 0.01. Smith es un juez infalible para detectar el
talento para la actuación. Después de una breve entrevista, podrá decir con seguridad si Sa-
rah puede convertirse en una estrella de cine en caso de que siga la carrera de la actuación.
¿Cuánto es lo más que estaría dispuesta a pagar Sarah por esta información?

Para responder esta pregunta hay que calcular primero la utilidad esperada si Sarah no tiene acceso a
esta información. Si decide ser maestra, su ingreso a lo largo toda la vida será de 5 con probabilidad 1,
de manera que su utilidad esperada será

UT = 1 –1/5 = 0.8.

En cambio, si sigue la carrera de la actuación, su utilidad esperada será

EUA = 0.01(1 – 1/400) + 0.99(1 – 1/2) = 0.505.

Como su utilidad esperada es mayor si decide ser maestra que si opta por ser actriz, será maestra
y su utilidad esperada será de 0.8.
Ahora suponga que puede tener la entrevista con Smith, quien le dirá con seguridad si decidién-
dose por seguir la carrera de actriz se convertirá en una estrella de cine. Y suponga que el costo de
esta entrevista es P, que se mide en las mismas unidades que M (vea la figura 6.6).
La ventaja que tiene al contar con la información de Smith es que si él dice que Sarah tendrá
éxito como actriz, ella podrá aprovechar esa lucrativa pero riesgosa carrera. Sin embargo, si Smith
dice que Sarah no tendrá éxito como actriz, ella podrá decidirse por ser maestra sin tener que dudar.
Su utilidad esperada si paga P por la entrevista está dada por

EU1 = 0.01[1 – 1/(400 – P)] + 0.99[1 – 1/(5 – P)] . (6.7)

06_CHAPTER 6.indd 185 3/6/09 7:55:14 PM


186 Capítulo 6 eConomía de la informaCión y eleCCión bajo inCertidumbre

FIGURA 6.6
Valor de reducir la
incertidumbre
de acuerdo con las
recompensas supuestas, la
utilidad esperada de ser Paga P
actriz es menor que la por entrevista
utilidad esperada de ser
“Te convertirás en una estrella” “No te convertirás en una estrella”
maestra. pero como una prob = 0.01 prob = 0.99
actriz de éxito gana mucho
más que una maestra, la Eliges la actuación Eliges ser maestra
información acerca de si ganas 400 ganas 5
una carrera de actriz tendrá
éxito tiene obviamente un
valor económico.

Para hallar la máxima cantidad que Smith puede cobrar por la entrevista, se iguala EU1 con la
utilidad esperada de Sarah si no cuenta con la información y se despeja P (recuerde que si no cuenta
con la información, será maestra y obtendrá UT = 0.8):

EU1 = 0.01[1 – 1/(400 – P)] + 0.99[1 – 1/(5 – P)] = UT = 0.8. (6.8)

Como ejercicio puede verificar que un valor de P de aproximadamente 0.0494 resuelve esta ecua-
ción. Por cualquier precio menor a esta cantidad, Sarah le pagará a Smith por su evaluación. Pero
si Smith cobra más de 0.0494, Sarah se olvidará de la entrevista y será maestra. Como ilustra con
claridad el ejemplo anterior, la información que ayuda a reducir la incertidumbre tiene un valor
económico. Y esto se refleja en el hecho de que por lo regular acostumbramos emplear servicios de
evaluación profesional, orientadores vocacionales y muchos otros profesionales para generar este
tipo de información.

EJEMPLO 6.5 Smith, a quien le hizo daño un producto en mal estado, está ponderando si demandar al
fabricante. Su función de utilidad está dada por U(M) = 1 – (I / M), donde M es su riqueza total.
Si no presenta la demanda su riqueza total es M0 = 7. Si demanda, la probabilidad de que gane
es 0.5, en cuyo caso recibirá una indemnización por daños de 5. Si pierde no recibirá ninguna
indemnización por daños. Suponga que el costo de oportunidad del tiempo que se requiere
para que un abogado presente una demanda a nombre de Smith es 2. ¿Presentará Smith la
demanda si tiene que pagarle 2 al abogado en honorarios? ¿Puede un abogado indiferente al
riesgo ofrecer a Smith un pago de honorarios cuyo valor esperado sea suficiente para saldar
el costo de oportunidad del abogado (es decir, cuyo valor esperado sea por lo menos 2) y que
de manera simultánea induzca a Smith a poner la demanda? Si es así describa tal pago de
honorarios.

Desde el punto de vista de Smith y de su abogado, el costo económico de presentar la demanda es


2, el tiempo del abogado. Si gana, el beneficio esperado de presentar la demanda es la probabilidad
de ganar (0.5) multiplicada por la indemnización del daño (5), es decir 2.5. Por lo tanto el valor espe-
rado de presentar la demanda es 2.5 – 0.5 = 0.5 > 0. Pero para Smith la pregunta no es si presentar
la demanda tiene un valor esperado positivo, sino si esto incrementará su utilidad esperada. Si Smith
demanda y paga al abogado 2, su riqueza total (neta sin los honorarios del abogado) será 7 + 5 – 2 =
10, en caso de que gane, y 7 – 2 = 5, en caso de que pierda. Si presenta la demanda su utilidad espera-
da será entonces 0.5[1 – (1/10)] + 0.5[1 – (1.5)] = 0.85. Si no demanda, su riqueza será con seguridad
7, lo que significa que su utilidad será 1 – (1/7) = 0.86 > 0.85. Por lo tanto si tiene que pagarle 2 al
abogado, no demandará.

06_CHAPTER 6.indd 186 3/6/09 7:55:15 PM


Seguro Contra maloS reSultadoS 187

La aversión al riesgo de Smith evitará que presente una demanda cuyo valor esperado es po-
sitivo. Por fortuna para él, su abogado es indiferente al riesgo. Esto significa que el abogado puede
diseñar un esquema de honorarios con el cual asuma todos los riesgos de no tener éxito. ¿Cuál sería
dicho esquema de honorarios?
Sea F1 los honorarios del abogado si Smith demanda y gana, y F2 los honorarios del abogado si
Smith pierde. Como Smith tiene aversión al riesgo, desde su perspectiva el mejor esquema de hono-
rarios será aquel en la cual su ingreso sea el mismo sin importar el resultado de la demanda:

7 + 5 – F 1 = 7 – F 2. (6.9)

¿Está su abogado dispuesto y en la posibilidad de ofrecer tal esquema de honorarios? Dado que
el abogado es indiferente al riesgo y la demanda tiene un valor esperado positivo, la respuesta será
sí. Indiferencia al riesgo significa que el abogado está dispuesto a aceptar el caso siempre que el valor
esperado de sus honorarios sea suficiente para pagar el costo de oportunidad de su tiempo, que se
supone es 2. De manera que se tiene

0.5 F1 + 0.5 F2 = 2. (6.10)

De las ecuaciones 6.9 y 6.10 se obtiene F1 = 4.5 y F2 = 0.5.


El valor negativo de F2 significa que si Smith presenta la demanda y pierde, su abogado pagará
a Smith 0.5. De esta manera, al margen del resultado de la demanda, con el esquema de honorarios
propuesto, Smith termina con M = 7.5, es decir 0.5 más que si no hubiera demandado. (Observe que
0.5 es el valor esperado de la demanda.) De manera que si el abogado le ofrece a Smith el esquema de
honorarios propuesto, éste demandará debido a que si lo hace su utilidad esperada —U = (1 – 1/7.5)
= 0.87— será mayor que si no demanda (0.86).
Este ejemplo ilustra el principio de que en una transacción que implique riesgo, un vendedor
indiferente al riesgo, que actúe como una empresa de seguros, puede hacer que la transacción resulte
más atractiva para un comprador con aversión al riesgo.

EJERCICIO 6.5
En el ejemplo 6.5 encuentre los valores de F1 y F2 que harían que a Smith le fuera indiferente
presentar la demanda o no presentarla.

La respuesta a la pregunta planteada en el ejercicio 6.5 se denomina valor equivalente cierto valor equivalente cierto el
de la apuesta asociada con presentar la demanda. El valor equivalente cierto de una apuesta es la valor equivalente cierto de
cantidad de dinero ante la que un individuo permanente indiferente al recibir dicha suma o tomar la una apuesta es la cantidad
apuesta. de dinero ante la cual a un
individuo le es indiferente
La exposición al riesgo es algo indeseable para los consumidores con aversión al riesgo, lo sufi-
recibir esa suma o aceptar la
ciente para que con frecuencia estén dispuestos a sacrificar recursos sustanciales para reducirlo. Por apuesta.
lo tanto, para los consumidores con aversión al riesgo el valor equivalente cierto de una apuesta es
menor que el valor esperado de la misma. Como lo demostró en el ejercicio 6.5, el consumidor del
ejemplo 6.5 habría estado deseoso de aceptar un valor esperado menor que 0.5 en vez de correr el
riesgo de tener que absorber el costo total de no ganar la demanda.

SEGURO CONTRA MALOS RESULTADOS


Cuando los riesgos que enfrentan diversos consumidores son independientes unos de otros (es decir,
cuando la probabilidad de que un consumidor obtenga un mal resultado es independiente de la pro-
babilidad de que otro también lo obtenga), suele ser posible que los consumidores actúen de manera
conjunta para lograr el resultado que todos prefieran.

06_CHAPTER 6.indd 187 3/6/09 7:55:16 PM


188 Capítulo 6 eConomía de la informaCión y eleCCión bajo inCertidumbre

RIESGO COMPARTIDO
Recuerde la apuesta 3 que se consideró antes en este capítulo en la cual al lanzar una moneda podían
ganar 20 000 dólares si caía cara o perder 10 000 dólares si caía cruz. Aunque es obvio que esto es
mucho mejor que una apuesta justa (su valor esperado es 5 000 dólares), la mayoría de las personas lo
pensarían antes de aceptarla por las dificultades que implica perder 10 000 dólares. Pero suponga que
puede reunir a otras 9 999 personas y hacer un acuerdo donde cada uno acepta la apuesta, reúnen
después las ganancias y las pérdidas que se obtengan y las comparten de manera equitativa. ¿Acepta-
ría este acuerdo?
Tendría que estar loco para no aceptarlo, porque en 10 000 lanzamientos independientes de una
moneda no cargada, más de 95% de las veces la cantidad de caras estará entre 4 900 y 5 100. Lo que da
como resultado que si acepta el acuerdo, no sólo el valor esperado de su participación en las ganan-
cias será 5 000 (lo mismo que si apostara solo), sino que además es muy posible que su participación
en las ganancias sea cercana a 5 000. La única manera en la que podría perder dinero si aceptara este
acuerdo es si en los 10 000 lanzamientos de la moneda no cargada cayera cruz más de dos terceras
partes de las veces. La posibilidad de que ocurra algo así es tan remota que en realidad no vale la pena
preocuparse por eso.
ley de los grandes números Compartir los riesgos funciona debido a una propiedad estadística llamada ley de los gran-
ley de la estadística que dice des números. Ésta establece que si a cada uno de los miembros de una cantidad grande de indivi-
que si un evento ocurre duos les sucede un evento de manera independiente con probabilidad p, entonces la proporción de
independientemente con individuos a los que les ocurre el evento, en un determinado año, rara vez será significativamente
probabilidad p en cada una de distinta de p. Suponga que el evento en cuestión sea que el fuego destruya una casa y que para cada
los N ocasiones, la proporción casa la probabilidad de que esto ocurra en un determinado año sea 0.001. Dado un pequeño grupo
de casos en los que ocurre
de casas, la proporción de ellas destruidas por el fuego puede variar en forma notable de año en año.
el evento se aproxima a p a
Pero en una muestra de, por ejemplo, 1 000 000 de casas se puede estar bastante seguro de que la
medida que N crece.
cantidad de ellas destruidas por el fuego en un determinado año será muy cercana a 1 000 (de manera
que la proporción de casas destruidas —1 000/1 000 000— será de 0.001).
Para los individuos, o incluso para pequeños grupos de personas, las pérdidas accidentales repre-
sentan un problema de incertidumbre implícita. Pero en el caso de un grupo grande de individuos,
la proporción de ellos que tendrá un accidente es en extremo estable y predecible. Esta propiedad
de la ley de los grandes números hace posible que las personas reduzcan sus riesgos mediante conve-
nios conjuntos.
Otra manera de compartir riesgos es la práctica de la propiedad conjunta de empresas. Cuan-
do un negocio empieza es posible que sucedan dos cosas: puede tener éxito, en cuyo caso sus pro-
pietarios ganan una gran cantidad de dinero, pero la otra alternativa —mucho más probable— es
que fracase, en cuyo caso los propietarios pierden toda su inversión inicial o una parte de ella. Con-
sidere un negocio que requiera una inversión inicial de 10 000 dólares. Suponga que la probabilidad
de que usted pierda esta inversión inicial es de 1/2 y la probabilidad de que no sólo la recupere, sino
que gane un dividendo de 20 000 dólares es 1/2. Esta empresa es entonces en esencia igual que la
apuesta 3 que se consideró antes. Es claro que aunque el valor esperado de esta empresa es positivo
(a saber, 5 000 dólares), mucha gente la encontraría inaceptablemente arriesgada. Pero si 100 perso-
nas reúnen sus recursos y comparten la inversión, de repente la empresa tendrá la misma apariencia
que la apuesta 2, cada persona podrá perder 100 dólares con probabilidad de 1/2 o ganar 200 dólares
con probabilidad de 1/2. Sin necesidad de modificar la empresa en absoluto, de repente se vuelve
atractiva para una gran cantidad de personas.
Las sociedades de todo tipo, los sindicatos y una gran cantidad de otros convenios institucionales
similares permiten a la gente traducir riesgos inaceptablemente grandes en otros más aceptables. Al-
guien que tenga una gran riqueza para invertir puede mantener sus riesgos en un mínimo invirtiendo
en diversos proyectos independientes. Existe una buena posibilidad de que algunos de estos proyectos
fracasen, pero una posibilidad muy pequeña de que una parte importante de ellos se arruine.
Otro ejemplo de acción colectiva para reducir la incertidumbre es la operación de mercados de
seguros. Considere el caso de los seguros de automóvil contra daños a terceros. Un accidente que
ocasione la muerte o lesiones serias fácilmente puede dar lugar a un juicio contra el conductor
que cueste varios millones de dólares. La posibilidad de que esto ocurra es remota, pero las conse-
cuencias son tan funestas que pocas familias podrían soportarlas.

06_CHAPTER 6.indd 188 3/6/09 7:55:16 PM


Seguro Contra maloS reSultadoS 189

Las compañías de seguros proporcionan a los consumidores un medio para compartir este ries-
go. Al contribuir cada uno con una cantidad anual de varios cientos de dólares, los asegurados crean
un fondo lo suficientemente grande para solucionar el puñado de juicios que se presenta cada año.
Debido a la ley de los grandes números, las empresas de seguros pueden predecir de manera muy
exacta los ingresos que necesitan para pagar sus reclamaciones. En efecto, cada una de las partes
acepta una pequeña pérdida (su prima de seguro) a cambio de no tener que soportar una mucho
mayor.
Para el consumidor promedio los seguros que se venden en el mercado privado siempre serán
una apuesta injusta, en el sentido específico que se definió antes. Para ver por qué, observe primero
que si una empresa de seguros pagara como beneficios exactamente la misma cantidad que recolectó
en primas, comprar un seguro sería una apuesta justa —la cantidad que un asegurado recuperaría
como beneficios sería igual, en promedio, a la cantidad pagada en primas—. Pero una empresa pri-
vada de seguros debe recolectar más en primas de lo que paga como beneficios, ya que tiene que
saldar también sus gastos administrativos. Los agentes de venta, los contadores, los investigadores y
el alquiler de las oficinas de la empresa, todo esto debe pagarse con lo que se obtiene de las primas.
De manera que, en promedio, los consumidores recuperan como beneficio menos de lo que pagan
como primas. El que la mayoría de las personas prefiera una pequeña apuesta injusta (comprar un
seguro) a una apuesta justa mucho mayor (correr los riesgos sin seguro) suele citarse como evidencia
de que la mayoría de ellas tiene aversión al riesgo.

PRECIO DE RESERVA DE UN SEGURO


¿Cuánto es lo máximo que pagará un consumidor por un seguro que proteja ante una pérdida? Supon-
ga que la función de utilidad de un consumidor con aversión al riesgo cuya riqueza inicial es de 700 es
U(M), como se muestra en la figura 6.7. Si se enfrenta a la perspectiva de una pérdida de 600 con proba-
bilidad de 1/3, su utilidad esperada será (1/3)U(100) + (2/3)U(700) = (1/3)(18) = (2/3)(36) = 30. (Vea
la figura 6.7 y observe que esta utilidad esperada se encuentra en la línea que une A y C, exactamente
en el punto encima de M = 500, su riqueza esperada sin seguro.) Suponga ahora que este consumidor
compra un seguro que respalda del todo la pérdida. ¿Cuánto es lo máximo que estará dispuesto a pa-
gar por ese seguro? De acuerdo con la figura 6.7, se ve que si paga 330 por el seguro, su utilidad será
U(700 – 330) = U(370) = 30, ocurra o no una pérdida. Como esto es idéntico que su utilidad esperada
si no compra el seguro, le sería indiferente comprarlo o no. Su precio de reserva por la póliza (es decir, lo
máxima que estaría dispuesto a pagar por ella), es por lo tanto 330. Observe que 700 – 370 = 330 es el
valor equivalente cierto de la apuesta que consiste en obtener 700 – 600 con probabilidad de 1/3 y 700

U FIGURA 6.7
Precio de reserva
C
de un seguro
U = U(M)
36 la riqueza inicial del
consumidor es 700 y se
33
encuentra con una pérdida
30 de 600 con probabilidad de
1/3. Su utilidad esperada es
30. Como de una riqueza
segura de 370 obtiene la
misma utilidad esperada,
A lo máximo que estará
18
dispuesto a pagar por un
seguro contra una pérdida
Precio de reserva es 700 – 370 = 330.
para un seguro = 330 dólares

M
100 370 500 700

06_CHAPTER 6.indd 189 3/6/09 7:55:17 PM


190 Capítulo 6 eConomía de la informaCión y eleCCión bajo inCertidumbre

con probabilidad de 2/3. Si I es el precio real del seguro y éste es menor que 330, entonces el consu-
midor comprará el seguro y de él obtendrá un excedente del consumidor de 330 – I.

SELECCIÓN ADVERSA
Cuando a un grupo heterogéneo de comerciantes potenciales se les presenta una oportunidad de
comerciar, aquellos que la aceptan serán diferentes —y con frecuencia peores— en promedio, que
los que la rechazan. Por ejemplo, los automóviles usados que están a la venta son de menor calidad
que los que no están a la venta y las personas que utilizan un servicio de citas son, en general, perso-
nas que vale menos la pena conocer que las que no usan tales servicios. Ambas situaciones ilustran
selección adversa proceso el principio de los limones y suele considerárseles como ejemplos de selecciones adversas. Una
mediante el cual un socio selección adversa es un proceso mediante el cual miembros “indeseables” de una población de com-
comercial potencial menos pradores o de vendedores tienen más posibilidad de participar en un intercambio voluntario.
deseable se ofrece como La selección adversa es especialmente importante en el mercado de seguros, donde suele eli-
voluntario para un intercambio. minar posibilidades de intercambio que beneficiarían tanto a los consumidores como a las empresas
aseguradoras. Para permanecer en el negocio, una empresa aseguradora debe obtener ingresos su-
ficientes de las primas que le permitan saldar tanto las reclamaciones de seguros por pagar como
de sus gastos administrativos. Por lo tanto, sus primas deben corresponder de manera estrecha a la
posibilidad de que se presenten reclamaciones de seguros. Sin embargo, no todos los consumidores
potenciales tienen la misma posibilidad de presentar una reclamación de seguro. En el caso de los
seguros contra siniestros automovilísticos, por ejemplo, algunos conductores tienen mucha más posi-
bilidad de tener un accidente que otros.
Si las empresas de seguros pudieran identificar a los conductores con más riesgo, podrían ajus-
tar sus primas de acuerdo con eso. En cierta medida, por supuesto, tratan de hacer precisamente lo
anterior y cobran tasas más elevadas a conductores con un historial de accidentes y de infracciones
de tránsito, o incluso a aquellos que no tienen historial de seguros. (Más adelante se dirá más sobre
cómo se establecen distintas tasas para personas con antecedentes idénticos que pertenecen a grupos
diferentes.) Pero estos ajustes son imperfectos. De hecho, existen personas que aunque no han tenido
accidentes ni multas de tránsito están en mayor riesgo que muchos conductores con malos antece-
dentes. Dentro de cada categoría de seguro habrá de manera inevitable una amplia variedad en el
riesgo potencial de los asegurados.
La presión de la competencia en el mercado de seguros en general hará que las primas reflejen
el riesgo promedio de las pólizas de seguro dentro de una determinada categoría. Esto significa que
conductores que saben que tienen mayor riesgo que el promedio se encuentran con un precio de
seguro atractivo. Sin embargo, el otro lado de la moneda es que esa misma prima no resulte atractiva
para los conductores que saben que tienen mucho menos riesgo que el promedio. Esto da como
resultado que muchos de los conductores con menor riesgo se vean inducidos a asegurarse por sí
mismos —es decir evitan comprar un seguro y simplemente asumen las consecuencias—. Y cuando
esto sucede, el riesgo promedio de los conductores que compran el seguro sube, lo que por fuerza
hace que aumente la prima. Esto hace los seguros aún menos atractivos para los conductores con
menos riesgo, con lo que una mayor cantidad de ellos opta por autoasegurarse. A la larga todos, ex-
cepto los peores conductores, pueden quedar excluidos del mercado de seguros. Éste es un resultado
desafortunado para muchos conductores cuidadosos que con gusto pagarían una prima de seguro
que correspondiera al valor esperado de sus pérdidas.

RIESGO MORAL
riesgo moral motivaciones Otro fenómeno que tiene consecuencias importantes en el mercado de seguros es el riesgo moral,
que llevan a las personas que se presenta cuando el seguro crea incentivos para que la gente se comporte de maneras ineficien-
a presentar reclamaciones tes o incluso fraudulentas. Considere el caso de “Nub City”, un pequeño pueblo de Florida al que los
fraudulentas de seguros o a ser investigadores de seguros le dieron este nombre por dudar del curioso patrón de sus reclamaciones
negligentes en el cuidado de
por accidente.
los bienes asegurados contra
robos o daños.
Más de 50 personas de ese pueblo habían sufrido algún “accidente” que consistía en la
pérdida de órganos y apéndices y el seguro había pagado reclamaciones hasta de 300 000

06_CHAPTER 6.indd 190 3/6/09 7:55:17 PM


Seguro Contra maloS reSultadoS 191

dólares. Sus investigadores estaban seguros de que esas mutilaciones se las infligieron ellas
mismas; muchos de los testigos de estos “accidentes” son personas que hicieron reclama-
ciones anteriores o parientes de víctimas, y un investigador observó que “de alguna mane-
ra siempre perdían partes que les eran menos necesarias”.6

Como las cantidades que las empresas aseguradoras cobran como primas deben ser suficientes
para pagar las reclamaciones, el cobro fraudulento de éstas ocasiona que las primas se eleven más
allá de lo que ocurriría de otra manera. Pero el riesgo moral ocasiona también que las primas suban
por razones que no están relacionadas con el fraude. Por ejemplo, mucha gente cuyos automóviles
están asegurados tiende a tomar menos medidas para reducir la posibilidad de sufrir daños o robo.
De manera similar, alguien cuya casa está asegurada contra incendios se ocupa menos de instalar un
sistema de rociadores para evitar daños por incendio.

DISCRIMINACIÓN ESTADÍSTICA
Como ya se dijo, las empresas aseguradoras de automóviles tratan de ajustar sus tarifas a los ante-
cedentes de los asegurados. Además, la mayoría de estas empresas tienen tasas diferenciadas para
personas con historiales de conducción idénticos pero que pertenecen a grupos de riesgo promedio
muy diferentes. Quizás el ejemplo más sobresaliente sea la tasa tan alta para conductores de sexo
masculino solteros menores de 25 años. La tasa promedio de accidentes entre estos conductores es
mucho más alta que la de cualquier otra categoría demográfica. A pesar de esto, una gran cantidad
de hombres menores de 25 años son conductores excepcionalmente buenos. Con seguridad conoce
usted varias personas así. El problema es que las empresas de seguros no pueden identificarlas a un
costo razonable.
En California, muchas empresas aseguradoras de automóviles tienen tarifas que difieren de
acuerdo con la zona de la ciudad en que usted vive. La razón de esto es que los congestionamientos
de tráfico, el robo, el vandalismo, los conductores no asegurados y otros factores que influyen en las
reclamaciones de seguros difieren de manera notable de una zona a otra.7 Sin embargo, lo desagra-
dable es que personas que viven a sólo 50 yardas de distancia unas de otras, pero en códigos postales
diferentes acaban pagando tasas de seguro muy diferentes. Muchas personas se han quejado de que
tales diferencias en las tasas son injustas. Pero antes de juzgar a las empresas aseguradoras es impor-
tante entender lo que ocurriría si alguna de ellas no tomara en cuenta estas diferencias. Suponga,
por ejemplo, que una empresa decide vender sus seguros a un mismo precio a todos los conductores
con un historial limpio. Si dicha empresa conservara a todos sus asegurados, esto significaría reducir
las tasas de las personas que viven en zonas inseguras, de los jóvenes adolescentes y de otros grupos
de alto riesgo y elevar las de todos los demás. Pero entonces, ¿por qué habrían de permanecer con
esta empresa los conductores de mayor edad que viven en zonas seguras? Estas personas ahorrarían
cambiándose a otra empresa que conservara las tasas anteriores y muchos así lo harían. De la misma
manera, los miembros de grupos de alto riesgo de otras empresas estarían muy motivados para cam-
biarse a la empresa con las nuevas tarifas. Por último, la empresa se quedaría nada más con asegura-
dos de los grupos de alto riesgo. La empresa podría conservar su nuevo programa de una misma tasa
a todo mundo, pero ésta tendría que ser suficientemente alta para pagar las reclamaciones generadas
por el grupo de más alto riesgo de todos.
Reconociendo este problema, algunos estados están buscando leyes que prohíban las tasas de
seguros por grupos. El argumento es que si todas las empresas se ven obligadas a ofrecer una sola
tasa, los miembros de los grupos de bajo riesgo podrán evadir el incremento de la tasa cambiándose
a otras empresas. Sin embargo, el problema con esto es que el gobierno no puede obligar a las em-
presas aseguradoras privadas a proporcionar seguros en contra de su voluntad. El resultado será que
muchas empresas se salgan de las áreas que son más costosas de atender, dejando que sus antiguos
clientes velen por sí mismos.

6
Wall Street Journal, 23 de diciembre de 1974.
7
Vea Eric Smith y Randall Wright, “¿Why Is Automobile Insurance in Philadelphia So Damned Expensive?” en American Economic
Review, 82, 1992: pp. 756-772.

06_CHAPTER 6.indd 191 3/6/09 7:55:18 PM


192 Capítulo 6 eConomía de la informaCión y eleCCión bajo inCertidumbre

APLICACIÓN: AUTOASEGURARSE SIEMPRE


CONTRA PÉRDIDAS PEQUEÑAS
Como ya se observó, los seguros que se proporcionan en el mercado privado tienen un valor esperado
negativo debido a los recursos que usa la empresa para administrar sus pólizas. El fenómeno de la se-
lección adversa es otra razón para autoasegurarse cuando el tamaño de la pérdida potencial es acepta-
ble. Esto significa que las primas de seguros deben ser lo suficientemente grandes para saldar el costo
de atender al asegurado típico, cuyo riesgo de pérdida es mayor que el de una persona promedio. Una
razón más por la cual las primas de seguro superan el valor esperado de las pérdidas promedio de una
persona es el problema del riesgo moral.
A pesar del hecho de que las primas de seguro deban pagar gastos administrativos, selección
adversa y los costos del riesgo moral, la mayoría de las personas encuentran prudente asegurarse
contra pérdidas mayores, como daños a sus casas por el incendio. Pero muchas otras se aseguran
también contra una enorme cantidad de pérdidas mucho más pequeñas.
Comprar seguros contra pérdidas menores viola el principio de escoger siempre la alternativa
con el resultado esperado más alto cuando sólo están en juego resultados pequeños. Los seguros
contra siniestros automovilísticos en general ofrecen una variedad de opciones para la cantidad
que no quedará cubierta —el llamado “deducible”*—. Si se escoge un deducible de 200 dólares, por
ejemplo, la póliza de seguro pagará todo excepto los primeros 200 dólares de cualquier reclamación
de daños. Las pólizas con estos deducibles son más baratas que las que no los tienen, debido a que la
empresa no sólo no tiene que pagar los primeros 200 dólares de los daños, sino también se evita el
problema y los gastos de procesar una gran cantidad de reclamaciones por daños pequeños. Debido
a esta reducción adicional del costo, la cantidad que usted espera ahorrarse de la prima es mayor que
la cantidad adicional que prevé gastar en una reparación. Y cuanto mayor sea el deducible que se
elija, mayores serán los ahorros esperados. En lugar de asegurarse por completo contra los daños de
un siniestro, es más razonable elegir un deducible grande y depositar los ahorros en una cuenta que
gane intereses.
¿De cuánto debe ser el deducible? Cuanto mayor sea, mejor, siempre y cuando se cuente con
recursos suficientes para atender la porción no cubierta de cualquier reclamación por daño. En rea-
lidad, para muchos consumidores con ingreso medio y alto, así como para aquellos que poseen au-
tomóviles más viejos, que no tienen un gran valor, la mejor estrategia es no comprar ningún seguro
contra accidentes.
¿Pero qué pasa si usted sigue esta estrategia y alguien le destroza de manera irreparable su nue-
vo automóvil de 20 000 dólares? Por supuesto, se sentirá mal. Pero tenga cuidado de no ser vícti-
ma de la falacia de que un mal resultado implica una mala decisión. Después de todo, las posibilidades
de un accidente así eran muy bajas, y aun cuando haya tenido uno, el ahorro de las primas a lo lar-
go de una vida serán más que suficiente para saldar los daños. En relación con la alternativa de com-
prar un seguro contra accidentes, no adquirirlo es una apuesta más que justa, y si usted cuenta con
los suficientes recursos económicos para soportar el peor resultado, debe aceptarlo.

Advertencia: emplee siempre un seguro


contra pérdidas mayores
A fin de que no haya ningún mal entendido, el consejo de la última sección no se aplica a pérdidas
mayores. Si una pérdida mayor se define como aquella que lo privará de una parte importante de su
riqueza de toda la vida, debe asegurarse contra ella. Debe tener un seguro médico, de cobertura am-
plia, tanto contra enfermedades desastrosas como contra pérdida del ingreso por incapacidad; debe
tener un seguro de responsabilidad civil amplia contra la posibilidad de un mal juicio; si usted vive en
una zona de inundaciones deberá asegurarse contra inundaciones, etcétera.
Sin embargo, de manera irónica muchas personas no se preocupan por protegerse contra estos
riesgos, pero sí se aseguran por completo contra el robo de sus equipos de televisión. La persona
prudente que sabe maximizar su utilidad esperada sabrá evitar estos patrones erróneos de conducta.
En el apéndice de este capítulo se analiza la teoría de la búsqueda y la maldición del ganador.

* En algunos países de habla hispana se denomina como franquicia.

06_CHAPTER 6.indd 192 3/6/09 7:55:18 PM


preguntaS de repaSo 193

RESUMEN
• Los participantes potenciales en un intercambio económico tados en la que los pesos de ponderación son las probabilida-
tienen muchos objetivos en común pero hay un aspecto im- des respectivas de que ocurran.
portante en el que deben ser considerados como adversarios. • La idea central del modelo de utilidad esperada es que la clasi-
Tanto en los mercados de productos como de trabajo, com- ficación de los valores esperados de una colección de apuestas
pradores y vendedores se encuentran ante incentivos podero- suele ser diferente del ordenamiento de las utilidades espera-
sos para representar erróneamente sus ofertas. das de dichas apuestas. Las diferencias surgen debido a la falta
• Para que los mensajes entre adversarios potenciales sean creí- de linealidad en la función de utilidad, lo que a su vez resume
bles, deben ser difíciles de imitar. Por ejemplo, una empresa la actitud del consumidor frente al riesgo. La función de utili-
con amplios costos hundidos, debe comunicar de manera dad cóncava, en la que cualquier arco de la misma se encuen-
creíble que ofrece un producto confiable porque de no lograr tra sobre la línea correspondiente, conduce a una conducta
satisfacer a sus consumidores puede perder mucho dinero. En de aversión al riesgo. Una persona con una función de utili-
cambio, para un vendedor callejero, para el que cerrar su ne- dad de este tipo aceptará una apuesta justa, la cual se define
gocio tiene costos muy bajos, es más difícil que persuada a los como una apuesta cuyo valor esperado es cero. Una perso-
compradores de que lo que ofrece es de alta calidad. na con una función de utilidad convexa, en la que cualquier
• Los mensajes entre adversarios potenciales también deben arco de la misma se encuentra debajo de la línea correspon-
satisfacer el principio de la apertura total, lo cual significa que diente, se dice que es amante del riesgo. Tales personas
si una de las partes es capaz de mostrar información favorable siempre aceptan una apuesta justa. Se dice que una perso-
acerca de ella misma, las otras partes se sentirán presionadas na con una función de utilidad lineal es indiferente al riesgo, y
a proporcionar información paralela, aun cuando sea conside- siempre le dará igual a aceptar o rechazar una apuesta justa.
rada menos favorable. El fabricante de un producto de baja • Debido a la selección adversa, las empresas se encuentran bajo
calidad no desea indicar el estado inferior de su producto ofre- una fuerte presión competitiva para hallar todo lo que pue-
ciendo sólo una garantía limitada. Pero si no lo hace, muchos dan acerca de los compradores y empleados potenciales. Esta
compradores harán una valoración aún menos favorable de su presión suele dar como resultado la discriminación estadística.
producto. En los mercados de seguros, las personas que pertenecen a
• Cuando una oportunidad de comerciar confronta a un grupo grupos que tienen diferentes tasas de accidentes suelen pagar
mixto de comerciantes potenciales, aquellos que la aceptan se- primas diferentes, aun cuando sus antecedentes como conduc-
rán diferentes —y de alguna manera peores— en promedio de tores sean idénticos. Este patrón de fijación de precios provoca
aquellos que la rechazan. Los automóviles que se ofrecen a la una comprensible sensación de injusticia en parte de los indi-
venta en un mercado de segunda mano son de menor calidad viduos afectados adversamente por el mismo. Sin embargo, en
que los que no están a la venta, vale menos la pena conocer a los mercados competitivos toda empresa que abandone esta
las personas que participan en un servicio de citas que a quie- política no puede esperar sobrevivir por mucho tiempo.
nes no lo hacen; y así sucesivamente. Éstos son ejemplos del • Un seguro comprado en los mercados privados es por lo ge-
principio de los limones. neral una apuesta injusta, no sólo debido a los costos adminis-
• La herramienta analítica para la elección con incertidumbre trativos que implica la prima del seguro, sino también a causa
es el modelo de utilidad esperada de Von Neumann-Morgen- de la selección adversa y del riesgo moral. El hecho de que la
stern. Éste parte de una función de utilidad que asigna una mayoría de las personas, a pesar de esto, compren cantidades
medida numérica de satisfacción a cada resultado, el cual se grandes de seguros se toma como una evidencia de la aversión
define en términos de la riqueza final a la que corresponde. al riesgo. Esta observación se ve también apoyada por el auge
El modelo dice que un consumidor racional, elegirá entre de los planes para compartir riesgos tales como la posesión
alternativas inciertas de manera que maximice su utilidad colectiva de acciones.
esperada, es una suma ponderada de las utilidades de los resul-

pREgUNtaS dE REpaSo
1. ¿Por qué una señal entre adversarios potenciales debe ser difí- tan de regular las preguntas que pueden hacerse en una entre-
cil de imitar? vista de trabajo a las personas que solicitan empleo?
2. Explique por qué a pesar de la relación de adversarios que exis- 4. ¿Cómo afecta la discriminación estadística a la distribución de
te entre vendedores y compradores, la propaganda comercial las primas promedio de seguros dentro de un grupo?
transmite información acerca de la calidad de un producto. 5. ¿Cómo afecta la discriminación estadística a la prima prome-
3. ¿Con qué dificultad práctica se encuentran las leyes que tra- dio de un seguro cobrada a los diferentes grupos?

06_CHAPTER 6.indd 193 3/6/09 7:55:19 PM


194 Capítulo 6 eConomía de la informaCión y eleCCión bajo inCertidumbre

6. ¿Por qué es intuitivamente aceptable suponer que la mayoría 8. Con sus propias palabras explique por qué es conveniente ase-
de las personas tiene aversión al riesgo? gurarse contra pérdidas menores.
7. Dé algunos ejemplos de conductas que parezcan inconsisten- 9. Dé algunos ejemplos de casos en los que la gente no se asegu-
tes con el supuesto de tener aversión al riesgo. ra contra pérdidas menores.

pRoblEMaS
1. Suponga que el desorden de los departamentos se midiera en una escala de 0 a 100, correspondiendo
el 0 al más ordenado y 100 al más desordenado. Suponga también que la distribución de los departa-
mentos en relación con el desorden es como se muestra en el diagrama siguiente. Es decir, suponga
que 10% de los departamentos están entre 0 y 20, 20% entre 20 y 40, etcétera.
40%

20% 20%

10% 10%

Más ordenado Más desordenado


0 20 40 60 80 100

Suponga, por último, que todos los padres tratan de enseñarles a sus hijos que si están más allá de
80 en la escala de desorden no dejen entrar a nadie en su departamento. Si tal regla fuera observada,
¿cuál sería su estimación del índice de desorden de alguien que dice, “no puedes entrar, mi departa-
mento es un desorden”? En un mundo en el que todos emplean la información disponible, ¿esperaría
que esta regla fuera estable? ¿Qué concluye usted del hecho de que en realidad, algunas veces, la
gente no permita la entrada a su departamento debido a que está muy desordenado?
2. Explique con detalle lo que le ocurriría a una empresa de seguros de automóvil que cobrara lo mis-
mo a los jóvenes adolescentes que a los demás clientes.
3. Se sabe que una fracción d de todos los automóviles nuevos tienen algún defecto. Los automóviles
con defecto sólo pueden ser identificados como tales por sus dueños. Todos los consumidores son in-
diferentes a este riesgo y valoran un automóvil sin defecto en 6 000 dólares. El precio de los automó-
viles nuevos es de 4 000 dólares, el de los usados es 1 000 dólares. Si los automóviles no se deprecian
físicamente con el uso, ¿cuál es el valor de d?
4. Una motocicleta nueva cuesta 9 000 dólares, mientras que una usada tiene un precio de 1 000 dólares.
Si no hay depreciación y los consumidores indiferentes al riesgo saben que 20% de las motocicletas
nuevas tienen algún defecto, ¿en cuánto valoran los consumidores una motocicleta sin defecto?
5. El sistema de escape de su Escort 1986 necesita cambiarse y usted sospecha que el precio de un nuevo
sistema de escape será igual a lo que obtendría si vende su automóvil. Si usted sabe que por lo demás
su automóvil está bien, ¿qué importancia tiene el modelo de los limones de Akerlof para su decisión
acerca de si debe comprar un nuevo sistema de escape?
6. ¿Qué razones hay para suponer que sea menos probable que un trabajador social elegido al azar haga
trampa en un juego de cartas que una persona elegida al azar?
7. Al comienzo de este siglo la mayoría de los bancos requerían que los cajeros tuvieran bachillerato.
A pesar de que para la mayoría de las tareas que desempeñan estos empleados en los bancos es su-
ficiente haber terminado el bachillerato, en la actualidad muchos bancos requieren que tengan un
grado universitario. Suponiendo que el costo actual de la educación universitaria sea menor hoy que
al comienzo del siglo y que dicho costo sea menor para personas con mayores capacidades, dé una
explicación de la posible razón por la cual los bancos han subido sus estándares.
8. ¿Cuál es el valor esperado del lanzamiento de un dado? [No está cargado (trucado) y tiene seis lados.]
9. Una moneda no cargada se lanza dos veces y la recompensa en cada uno de los cuatro posibles resul-
tados es:
Cara-Cara: gana 20; Cara-Cruz: gana 9; Cruz-Cara: gana 7; Cruz-Cruz: gana 16.
¿Cuál es el valor esperado en esta apuesta?

06_CHAPTER 6.indd 194 3/6/09 7:55:19 PM


problemaS 195

10. Suponga que su función de utilidad está dada por U = √ M, donde M es su riqueza total. Si el valor
inicial de M es 16, ¿aceptaría la apuesta del problema anterior?
11. Suponga que tiene 10 000 dólares para invertir. Un corredor le llama para darle información solici-
tada por usted acerca de algunos bonos basura. Si la empresa que emite los bonos obtiene ganancias
este año usted obtendrá una tasa de interés de 40% por bono. Si la empresa se declara en bancarrota,
usted perderá toda su inversión. Si la empresa ni gana ni pierde, usted obtendrá una tasa de interés
de 10%. Su corredor le dice que hay 50% de probabilidades de que la empresa no gane ni pierda y
20% de probabilidades de que se declare en bancarrota. Su otra posibilidad es invertir en un bono del
gobierno, libre de riesgo, que garantiza 8% de interés durante un año.
a) ¿Cuál es la tasa de interés esperada de una inversión en estos bonos basura?
b) ¿Qué inversión elegirá usted si su función de utilidad está dada por U = M2 ?
c) ¿Qué inversión elegirá usted si su función de utilidad está dada por U = √M ?
12. Suponga que su riqueza presente, M, es 100 y que su función de utilidad es U = M2. Usted tiene un
billete de la lotería con el que ganará 10 dólares con probabilidad de 0.25 y 0 dólares con probabilidad
0.75. ¿Cuál es la mínima cantidad por la que estaría dispuesto a vender su billete?
13. Su función de utilidad es √ M. Su riqueza presente es 400 000 dólares. Existe una probabilidad de
0.00001 de que su responsabilidad civil en un accidente automovilístico reduzca su riqueza a 0.
¿Cuánto es lo máximo que pagaría usted por un seguro que cubra este riesgo?
14. Las gallinas de un granjero ponen 1 000 huevos por día, los cuales vende el granjero a 10 centavos de
dólar cada uno, su única fuente de ingreso. Su función de utilidad es U = √ M, donde M es su ingreso
diario. Cada vez que el granjero recoge los huevos de las gallinas, existe 50% de posibilidad de que se
caiga y rompa los huevos. Si se supone que él no le asigna ningún valor a su tiempo, ¿estará en mejor
posición económica recogiendo todos los huevos en un solo viaje o recogiéndolos en dos viajes,
cada uno de 500 huevos? (Pista: cuando hace dos viajes hay tres posibilidades: 1 000 huevos rotos, 500
huevos rotos o ningún huevo roto. ¿Cuál es la probabilidad de cada uno de estos resultados?)
15. Su riqueza presente es M = 49 y usted se ve obligado a hacer la apuesta siguiente: si una moneda no
cargada cae cara, usted obtiene 15; si cae cruz usted pierde 13. Su función de utilidad es U = √ M.
a) ¿Cuál es el valor esperado de esta apuesta?
b) ¿Cuál es su utilidad esperada?
c) ¿Qué diferencia habría en su respuesta si la pérdida porque caiga cruz es 15?
d) ¿Cuánto es lo máximo que pagaría por no tener que hacer la apuesta que se describe en el inciso c)?
16. Smith tiene una oportunidad de inversión en la que gana 33 con probabilidad de 1/2 y pierde 30 con
probabilidad de 1/2.
a) Si su riqueza presente es M = 111, y su función de utilidad es U = √M, ¿hará esta inversión?
b) ¿Hará esta inversión si tiene dos socios a partes iguales? (Calcule las utilidades esperadas con por
lo menos dos cifras decimales.)
17. John tiene una función de utilidad de Von Neumann-Morgenstern dada por U = √ M, donde M es
su ingreso. Si decide ser profesor de economía, tendrá M = 81/año con probabilidad 1. Si decide ser
abogado, tendrá M = 900/año si se convierte en socio de una empresa de Wall Street, pero si no es así
sólo tendrá M = 25/año. La probabilidad de que pueda ser socio es 0.2. Smith es infalible para detec-
tar el talento de un abogado. Después de una breve entrevista puede decir con seguridad si John será
socio. ¿Cuánto es lo máximo que John estaría dispuesto a pagar por esta información? (Establezca la
ecuación relevante. No necesita resolverla.)
*18. En el problema anterior, si se supone que a Smith no le cuesta realizar la entrevista, ¿obtiene el ingre-
so esperado más alto posible si le cobra a John los mismos honorarios al margen del resultado de la
entrevista?
*19. Hay dos grupos del mismo tamaño, cada uno con una función de utilidad dada por U(M) = √ M,
donde M = 100 es la riqueza inicial de cada individuo. Cada miembro del grupo 1 tendrá una pérdida
de 36 con probabilidad de 0.5. Cada miembro del grupo 2 tendrá la misma pérdida con probabili-
dad de 0.1.
a) ¿Cuánto es lo máximo que un miembro de cada grupo estaría dispuesto a pagar para asegurarse
contra esta pérdida?

*Los problemas marcados con un asterisco (*) son más difíciles.

06_CHAPTER 6.indd 195 3/6/09 7:55:20 PM


196 Capítulo 6 eConomía de la informaCión y eleCCión bajo inCertidumbre

b) En el inciso a), si es imposible para alguien externo descubrir qué individuos pertenecen a cada
grupo, ¿sería práctico para los miembros del grupo 2 asegurarse contra esta pérdida en un mer-
cado competitivo de seguros? (Para simplificar, puede suponer que las empresas de seguros sólo
cobran en primas lo suficiente para saldar sus pagos esperados.) Explique.
c) Ahora suponga que las empresas de seguros del inciso b) tienen una prueba imperfecta para de-
terminar qué personas pertenecen a qué grupo. Si la prueba dice que una persona pertenece a un
determinado grupo, la probabilidad de que esto sea cierto es x < 1.0. ¿Qué tan grande debe ser x
para que usted modifique su respuesta al inciso b)?
*20. Existen dos grupos, cada uno con una función de utilidad dada por U(M) = √ M, donde M = 44 es la
riqueza inicial de cada individuo. Cada miembro del grupo 1 experimentará una pérdida de 44 con
probabilidad de 0.5. Cada miembro del grupo 2 experimentará la misma pérdida con probabilidad
de 0.1.
a) ¿Cuánto es lo máximo que un miembro de cada grupo estaría dispuesto a pagar para asegurarse
contra esta pérdida?
b) Si a una persona ajena le es posible descubrir qué individuo pertenece a qué grupo, en un conjun-
to de clientes potenciales ¿cómo puede ser la porción de miembros del grupo 1 para que resulte
imposible que una empresa privada con restricción de ganancia cero proporcione seguro a los
miembros del grupo 2? (Para simplificar, puede suponer que las empresas de seguros sólo cobran
la prima suficiente para saldar sus pagos de beneficios esperados y que la gente siempre compra
un seguro si su precio es igual o menor que su precio de reserva.) Explique.
*21. Ante una elección entre A (una ganancia segura de 100) y B (una posibilidad de 80% de ganar 150 y
una posibilidad de 20% de ganar 0), Smith elige A. Pero ante una elección entre C (50% de posibilidad
de ganar 100 y 50% de posibilidad de ganar 0) y D (40% de posibilidad de ganar 150 y 60% de ganar 0),
Smith elige D. Muestre que Smith elige de manera inconsistente con la maximización de la utilidad
esperada.

RESpUEStaS a loS EjERcicioS dEl capítUlo


6.1 La utilidad esperada de la apuesta corresponde al punto D, que se encuentra a un tercio de la distan-
cia desde C hasta A.
U(M)

C U= M
7

19/3 D
6

A
5

M
0 25 36 41 49

6.2 Si acepta la apuesta, su utilidad esperada está dada por EUG = (1/2)(1202) + (1/2)(802) = 10 400. Si
rechaza la apuesta, su utilidad es únicamente 1002 = 10 000, y por lo tanto debe aceptar la apuesta.
6.3 Si acepta la apuesta su utilidad esperada está dada por EUG = (1/2)(120) + (1/2)(80) = 100. Si rechaza
la apuesta, su utilidad es también 100, de manera que le da lo mismo aceptar o rechazar la apuesta.

06_CHAPTER 6.indd 196 3/6/09 7:55:21 PM


reSpueStaS a loS ejerCiCioS del Capítulo 197

6.4 Como en el ejemplo 6.3, aquí el paso clave es darse cuenta de que sólo están a la venta computadoras
usadas que tienen algún defecto. Debido al principio de los limones, los poseedores de una compu-
tadora buena no pueden venderla en el mercado de segunda mano por lo que ellos consideran que
vale. De manera que si el valor de una computadora defectuosa es de 600 dólares, entonces para un
comprador indiferente al riesgo el valor que tiene una computadora nueva deberá ser (1/4)(600) +
(3/4)(2 000) = 1 650 dólares.
6.5 A Smith le será indiferente presentar la demanda o no hacerlo si el esquema de honorarios es tal que
su ingreso, incluyendo indemnización por daños y honorarios para el abogado, sea exactamente 7 sin
importar cuál sea el resultado de la demanda. Esto ocurre si F1 = 5 y F2 = 0. En este caso, el valor espe-
rado de los honorarios del abogado de Smith será 0.5(5) + 0.5(0) = 2.5, es decir 0.5 más que el costo
de oportunidad del tiempo del abogado. De acuerdo con esta estructura de honorarios, el abogado
absorberá todo el valor esperado de la demanda.

06_CHAPTER 6.indd 197 3/6/09 7:55:21 PM


06_CHAPTER 6.indd 198 3/6/09 7:55:21 PM
a p é N d i c E

6
tEoría dE la búsquEda
y la maldición dEl ganador

BÚSQUEDA DE SALARIOS ALTOS Y PRECIOS BAJOS


Si en todos los empleos se obtuviera la misma remuneración y fueran igualmente deseables en todos
los aspectos, una vez que se obtuviera la primera oferta no habría razón para seguir buscando. Pero,
por supuesto, no todos los empleos son iguales. En particular, en algunos usted puede hacer un uso
más completo de su particular mezcla de talento, preparación y habilidades que en otros. Cuan-
to más coincidan los requerimientos de un determinado empleo con su inventario de características
personales, más productivo será usted y su empleador podrá pagarle más. Si usted es un hábil para-
dor en corto que puede anotar 30 cuadrangulares y robar 30 bases por año, por ejemplo, es mucho
más valioso para los Medias Rojas de Boston que para el equipo local de Burger King.
Sea cual sea su mezcla de habilidades, su problema es hallar el empleo correcto para usted. Lo
primero que hay que observar es que su búsqueda no puede —en realidad, no debe— ser exhaustiva.
Después de todo, en Estados Unidos cada momento se crean más nuevos empleos de los que una
persona podría investigar. Aun si se pudieran investigar todos, esa estrategia resultaría tan costosa
—tanto en términos de tiempo como de dinero— que sería imprudente hacerla.
Para simplificar suponga que se abstrae de todas las dimensiones de un empleo excepto el sala-
rio. Es decir, suponga que existe una distribución de los empleos vacantes por examinar, cada uno
con un salario diferente. También para simplificar, suponga que usted es indiferente al riesgo, que
planea trabajar durante un periodo y que en el total de empleos vacantes los salarios por periodo
están distribuidos de manera uniforme entre 100 y 200 dólares, como se muestra en la figura A.6.1.
Lo que esto significa es que si usted examina un empleo vacante tomado al azar, el salario correspon-
diente a ese empleo tendrá la misma posibilidad de tomar cualquiera de los valores entre 100 y 200
dólares. Por último, suponga que examinar una vacante tiene un precio de 5 dólares.
199

06_CHAPTER 6.indd 199 3/6/09 7:55:22 PM


200 Capítulo 6 apéndiCe 6: teoría de la búSqueda y la maldiCión del ganador

FIGURA A.6.1
Una hipotética
distribución uniforme
de los salarios
el salario que se ofrece por
un nuevo empleo tomado al
azar es igualmente posible Salario (dólares/periodo)
100 200
que tome cualquiera de los
valores entre 100 y 200
dólares por un periodo. en
promedio el salario de una
nueva oferta de empleo
será de 150 dólares.

Usted ha empezado con su búsqueda y en el primer empleo que examina el salario es de 150 dó-
lares. ¿Debe aceptarlo o pagar 5 dólares y buscar otro? (Si examina otro empleo y resulta ser peor que
la oferta actual, usted todavía tiene la posibilidad de aceptar esta última.) En este caso, para decidir
con inteligencia usted debe comparar el costo de examinar otra oferta con los beneficios esperados.
Si es que hay algún beneficio, la nueva oferta debe ser mayor que 150 dólares. La probabilidad de que
esto ocurra es 0.5 (en la figura A.6.2, la relación entre el área sombreada del rectángulo y toda el área
del rectángulo).

FIGURA A.6.2
Valor esperado de una
oferta mayor que 150
dólares
la probabilidad de que la
siguiente oferta de empleo
sea mayor que 150 dólares
es 0.5. una oferta que se Salario (dólares/periodo)
sabe que es mayor que 100 150 175 200
150 dólares es igualmente
posible que se encuentre
en cualquier punto entre
150 y 200 dólares. Su valor
esperado será de 175
dólares.

Suponga que la nueva oferta es superior a 150 dólares. ¿Cuál es su valor esperado? En la figu-
ra A.6.2, observe que como es igualmente posible que una oferta mayor que 150 dólares caiga en
cualquier lugar en el intervalo de 150 a 200 dólares, su valor promedio será de 175 dólares, lo cual
significa una ganancia de 25 dólares respecto a su oferta actual. Por lo tanto, la ganancia esperada de
considerar otra oferta cuando usted tiene una de 150 dólares en la mano —llámele EG(150)— es el
producto de los siguientes dos factores: 1) que la probabilidad de que la nueva oferta sea mayor que
la anterior y 2) que la ganancia esperada es mayor. De esta manera se tiene

EG(150) = (1/2)(25) = 12.50 dólares (A.6.1)

Como la ganancia esperada de ponderar otra oferta es mayor que el costo, 5 dólares, y usted es
indiferente al riesgo, deberá continuar buscando.
¿De cuánto debe ser una oferta para que usted la acepte? La respuesta a esta pregunta se llama
salario de aceptación, que se denota w*. Si usted es indiferente al riesgo, este salario de aceptación es
aquel para el cual el beneficio monetario esperado de considerar otra oferta es exactamente igual a
los costos. En general, sería el salario para el cual la ganancia de utilidad esperada de examinar otra
oferta es compensada de manera exacta por la pérdida de utilidad derivada del costo de la búsqueda.

06_CHAPTER 6.indd 200 3/6/09 7:55:23 PM


búSqueda de SalarioS altoS y preCioS bajoS 201

FIGURA A.6.3
Salario de aceptación
el salario de aceptación,
w*, es aquel para el cual
el costo de una búsqueda
más es exactamente igual
al beneficio esperado. el
beneficio esperado es el
Salario (dólares/periodo) producto de la probabilidad
100 w* 200 de que una nueva oferta
(200 + w *)/2 pague más que w* [(200 –
w*)/100] multiplicada por la
ganancia promedio cuando
es así [(200 – w*)/2]. para
calcular w*, este producto
se iguala al costo de la
búsqueda (aquí 5 dólares) y
se despeja w*.

El caso indiferente al riesgo es mucho más sencillo de analizar pero explica todos los aspectos
importantes.
Si la oferta actual es w*, observe en la figura A.6.3 que la probabilidad de obtener una oferta
mejor es (200 – w*)/100 (que de nuevo es la relación entre el área del rectángulo sombreado y al área
total del rectángulo). Si se supone que la nueva oferta se encuentra entre w* y 200, su valor esperado
estará en el punto medio entre estos dos, es decir (200 + w*)/2. Este valor esperado es (200 – w*)/2
unidades mayor que w*.
La ganancia esperada de considerar otra oferta es otra vez la probabilidad de que la nueva oferta
sea mayor que w* multiplicada por el aumento esperado del salario, si lo hay. Por lo tanto se tiene

 200 − w *  200 − w *  ( 200 − w *) 2


EG( w *) =    = (A.6.2)
 100  2  200
 200 − w *  200 − w *  ( 200 − w *) 2
EG( w *) =  200 − w *  200 − w *  = ( 200 − w *) 2
De acuerdo con la definición del salario EG(de =  100paraun buscador
w *)aceptación 2  =indiferente
200 al riesgo, esta
( 200 − w *)2 100  2   200
ganancia esperada es igual EG
al costo = ponderar otra
( w *) de =oferta:
5
200
( 200 − w *) 2
EG( w *) = ( 200 − w *) 2 = 5 (A.6.3)
EG( w *) = 200 =5
w * = 200 − 1 000 = 168.38 .
200
lo cual se reduce a
w * = 200 − 1 000 = 168.38.
w * = 200 − 1 000 = 168.38. (A.6.4)

Entonces, en este ejemplo la regla de decisión óptima será seguir buscando hasta que encuen-
tre una oferta de por lo menos 168.38. Cuando los salarios están distribuidos de manera uniforme,
como en este caso, usted deberá obtener, en promedio, un salario que se encuentre a la mitad entre
200 y w*. Sin embargo, observe que seguir esta regla no significa por fuerza que siempre será mejor
que aceptar una oferta menor que w*. Si usted no tiene suerte, por ejemplo, puede empezar con
una oferta de 160 dólares y buscar 20 veces antes de hallar una oferta mayor que w*. Su ganancia
total, sin los costos de búsqueda, será a lo sumo de 100 dólares, lo que obviamente es peor que 160
dólares.

EJERCICIO A.6.1
Si el costo de la búsqueda es de un dólar y las ofertas de empleo están distribuidas de manera
uniforme entre 10 y 60, ¿cuál es el salario más bajo que usted aceptaría?

06_CHAPTER 6.indd 201 3/6/09 7:55:25 PM


202 Capítulo 6 apéndiCe 6: teoría de la búSqueda y la maldiCión del ganador

Un razonamiento análogo conduce a una regla de decisión similar para una persona que busca
un producto de bajo precio. Como se verá en el capítulo 12, la mayoría de las empresas emplean di-
versos métodos de descuentos, lo que da como resultado que, por lo general, en los mercados exista
una amplia distribución de precios para la mayoría de los productos. Una vez más, para simplificar,
suponga una distribución de precios que es uniforme en el intervalo (0, P), como se muestra en la
figura A.6.4

FIGURA A.6.4
Distribución hipotética
de los precios
el precio de un producto
seleccionado al azar de esta
distribución es igualmente
posible que sea cualquier
número entre 0 y P. la
probabilidad de hallar un Precio (dólares)
0 P*/2 P* P
precio inferior a P* es
P*/P, que es la fracción de
todos los precios que se
encuentran a la izquierda
de P*. el valor promedio de
un precio que se sabe está
por debajo de P* es P*/2.

El precio de aceptación, P*, está determinado de manera muy similar a la del salario de acep-
tación en el caso de la búsqueda de empleo. Si el precio del producto que acaba de tomar como
muestra es P*, la probabilidad de obtener un precio menor en su siguiente búsqueda es P*/P (co-
mo antes, la relación entre el rectángulo sombreado y el área de todo el rectángulo). Si usted encuen-
tra un precio más bajo, su ahorro, en promedio, será P*/2. Su ganancia esperada de otra búsqueda
en P* es, por lo tanto
 P *  P *  P * 2
EG( P *) =    .
 2  PP * 2 P *  P * 2 (A.6.5)
EG( P *) =    .
 2  P  2P
Si el costo de otra búsqueda es C, la expresión para el precio de aceptación será
P * = 2PC.
P * = 2PC. (A.6.6)

Tanto en el caso del salario como en el caso del precio, los niveles de aceptación dependen del
costo de la búsqueda: si aumenta el costo de examinar una opción más, su salario de aceptación
disminuye y su precio de aceptación aumenta. En el caso de la búsqueda de precio, la relación entre
P* y C (como se indica en la ecuación A.6.6) se muestra en la figura A.6.5.

EJEMPLO A.6.1 Suponga que busca un precio bajo en una distribución de precios que es uniforme en el inter-
valo (1, 2). ¿Cómo se modificaría su precio de aceptación si el costo de la búsqueda aumenta-
ra de 0.05 a 0.10?

La expresión para el precio de aceptación que se da en la ecuación A.6.6 es para una distribución de
precios uniforme en el intervalo (0, P). Observe que ahora el valor mínimo de la distribución de pre-
cios no es 0 sino 1. El precio de aceptación en esta distribución de precios será exactamente una uni-
dad mayor que el precio de aceptación en una distribución uniforme en el intervalo (0, 1). Si el costo
de la búsqueda es 0.05 se ve, de acuerdo con la ecuación A.6.6, que el último precio de aceptación será
√ 0.10 = 0.316, lo cual significa que el precio de aceptación para la distribución uniforme de precios
en (1, 2) será 1.316. Con un costo de búsqueda de 0.10, el precio de aceptación para la distribución

06_CHAPTER 6.indd 202 3/6/09 7:55:27 PM


la maldiCión del ganador 203

P*
FIGURA A.6.5
Precio de aceptación
como función del costo
P de la búsqueda
a medida que la búsqueda
se vuelve más costosa
(conforme C aumenta),
vale la pena conformarse
con productos de precio
más elevado.
P* = 2CP

C
0 P/2

uniforme en (0, 1) aumenta a √ 0.20 =0.447, de manera que el nuevo precio de aceptación para la
distribución uniforme (1, 2) será 1.447.
Observe la semejanza esencial entre los problemas de búsqueda de salario y precio, y la decisión
del sapo que se consideró al principio de este capítulo. Como el buscador racional de precio o de sala-
rio, el sapo racional ponderará los costos de una búsqueda adicional contra los beneficios esperados.
El sapo llegará a un “tono de aceptación”, de manera que, si su rival croa en un tono más alto, el sapo
se quedará y luchará.

LA MALDICIÓN DEL GANADOR


Cuando el psicólogo Max Bazerman de la Universidad Northwestern estuvo como docente en la
Universidad de Boston hace unos años, él y su colega William Samuelson llevaron a cabo el siguiente
experimento en sus clases de microeconomía.1 Primero, en una jarra de vidrio transparente coloca-
ron una cantidad en monedas equivalente a ocho dólares. Después de permitir que los estudiantes
examinaran con cuidado la jarra, la subastaron al mejor postor. Además, pidieron a los estudiantes
dar una estimación por escrito del valor de las monedas en la jarra.
En promedio, los estudiantes se comportaron de manera conservadora tanto en sus ofertas
como en sus estimaciones. La evaluación promedio fue de sólo 5.13 dólares, más o menos un tercio
por debajo del valor real de las monedas. De manera similar, la mayoría de los estudiantes abandona-
ron la subasta mucho antes de que las ofertas llegaran a ocho dólares.
En cualquier subasta la magnitud de la oferta ganadora no depende sólo del comportamiento
del postor promedio, sino también del comportamiento del postor más alto. En 48 repeticiones de este
experimento la oferta más alta fue, en promedio, de 10.01 dólares, más de 20% mayor que el valor
de las monedas.
De manera que Bazerman y Samuelson obtuvieron una ganancia de casi 100 dólares a costa de
los postores ganadores.
A ese precio, se puede considerar que los ganadores aprendieron una importante lección a un
precio reducido. Cuando al principio de los años setenta los ejecutivos de empresas petroleras reci-
bieron la misma lección, las apuestas fueron mayores. Por primera vez el gobierno federal de Estados
Unidos realizaba una subasta de localizaciones petroleras potencialmente valiosas en el Golfo de
México. Las empresas no tenían experiencia en la licitación de tales locaciones, pero tenían acceso a
los métodos geológicos estándar para estimar su valor. Ahora, en retrospectiva, puede verse que las
locaciones alcanzaron precios demasiado altos. Los postores ganadores —que pagaron precios mi-
llonarios por ellas— hubieran tenido un rendimiento mucho mayor por sus inversiones si hubieran
depositado el mismo dinero en una cuenta de ahorros.

1
Para más detalles sobre este experimento, véase David Warsh, “The Winner’s Curse”, en Boston Globe, 17 de abril de 1988.

06_CHAPTER 6.indd 203 3/6/09 7:55:28 PM


204 Capítulo 6 apéndiCe 6: teoría de la búSqueda y la maldiCión del ganador

El principio general de que la oferta ganadora es superior al precio real de un artículo se conoce
como maldición del ganador. Lo sorprendente de ella es que no se requiere que nadie use una estima-
ción sesgada del valor del premio. El problema es que en toda estimación interviene algún elemento
de aleatoriedad. Se dice que una estimación es no sesgada si, en promedio, es igual al valor verdadero.
Los pronósticos de temperatura, por ejemplo son no sesgados; algunos días son muy altos, otros
muy bajos, pero su valor promedio a largo plazo refleja casi perfectamente las temperaturas ver-
daderas. De la misma manera, aun cuando la estimación de cada postor sea no sesgada, en algunas
ocasiones es demasiado alta y en otras demasiado baja. Y el ganador de la subasta será, por supuesto,
el postor cuya estimación sea demasiado alta con el mayor margen.
Un postor totalmente racional tomará en cuenta el hecho de que la oferta ganadora tiende a
ser demasiado alta. Volviendo a la subasta de las monedas, suponga que la mejor estimación de su
valor, calculada por alguna persona, es de nueve dólares. Sabiendo que la oferta ganadora tiende a
ser demasiado alta, esta persona puede protegerse ajustando su oferta hacia abajo. Si otros postores
son del todo racionales ajustarán también sus ofertas y la identidad del postor ganador será la misma
que antes.
¿De cuánto debe ser el ajuste hacia abajo? Un momento de reflexión muestra que cuantos más
postores haya, mayor debe ser el ajuste. Suponga que el verdadero valor de un objeto que se subasta
sea de 1 000 dólares. Para ilustrar lo que se quiere decir con una estimación no sesgada, imagine que
de manera consecutiva cada postor potencial extrae de una urna que contiene 201 canicas numera-
das del 900 al 1 100 una de ellas. (Cada canica, una vez inspeccionada, se devuelve a la urna.) El valor
esperado para el número en cualquier canica es 1 000, sin importar cuántos postores haya. Pero el
valor esperado del número mayor extraído de la urna aumentará con la cantidad de postores. (Si hay
un millón de personas que sacan una canica, es casi seguro que alguna extraerá la 1 100, el máximo
valor posible; pero si sólo cinco personas participan, el número mayor sería mucho menor en pro-
medio.) En consecuencia, cuantos más postores haya, más hacia abajo tendrá que ajustar usted su
estimación.
Para ilustrar las mecánicas de los procesos de ajuste, considere una subasta en la que el verdade-
ro valor de un artículo a la venta es 0.5 y que cada postor emplea una estimación que está distribuida
de manera uniforme entre 0 y 1. Dicha estimación tendrá la misma posibilidad de tomar cualquier
valor entre 0 y 1, así que tiene un valor esperado de 0.5, esto significa que es no sesgada (ver figura
A.6.6).
Si en esta subasta hubiera nada más un postor, el valor esperado de la estimación más alta sería
0.5 y entonces no tendría necesidad de hacer ningún ajuste. Sin embargo, si hubiera N postores,
¿cuál sería el valor esperado de la estimación más alta? Suponga que las N estimaciones se colocan en
orden ascendente X1, X2, …, XN de manera que XN denota la más alta de las N estimaciones. Dado que
todas las estimaciones provienen de una distribución uniforme, los valores esperados de X1, X2, …,
XN estarían repartidos de manera uniforme en todo el intervalo. Como se observó, cuando N = 1 sólo
se tiene X1 y su valor esperado, 0.5, se encuentra justo en el centro del intervalo.

FIGURA A.6.6
Estimación no sesgada
con una distribución
uniforme
Cada postor potencial tiene
una estimación del valor del
recurso, que es igualmente
posible que tome cualquier
0 0.5 1
valor entre 0 y 1. el
verdadero valor del recurso
es el valor promedio de
estas estimaciones, el cual
es 0.5.

06_CHAPTER 6.indd 204 3/6/09 7:55:29 PM


la maldiCión del ganador 205

¿Y qué pasa cuando N = 2? Ahora las dos estimaciones son X1 y X2 y sus valores esperados son los
que se muestran en la segunda gráfica de la figura A.6.7. Esta vez, el valor esperado de la estimación
más alta es 2/3. En la tercera gráfica observe que el valor esperado de la mayor de tres estimaciones
es 3/4. En la gráfica inferior, por último, el valor esperado de la más alta de cuatro estimaciones es
4/5. En general, el valor esperado de la más alta de N estimaciones es N/(N + 1).2 (Vea nota a pie de
página número 2.)

FIGURA A.6.7
Valor esperado de la
N=1
estimación más alta
0 X1 1 N = 1, 2, 3 y 4
Cuando más personas
ponderan el valor de un
N=2 recurso, el valor esperado
de la estimación más alta
0 X1 X2 1 aumenta. Si las estimaciones
están distribuidas de
manera uniforme en el
N=3
intervalo (0, 1), el valor
0 X1 X2 X3 1 promedio de la mayor de
las N estimaciones (XN) es
N/(N + 1).
N=4

0 X1 X2 X3 X4 1

Si todas las estimaciones se sacaran de una distribución uniforme en el intervalo (0, C) y no de


(0, 1), el valor esperado de la estimación más alta sería CN/(N + 1).

Suponga que hay 50 postores que pujan por un reloj antiguo y que cada uno tiene una esti- EJEMPLO A.6.2
mación no sesgada del verdadero valor del reloj extraída de una distribución uniforme en el
intervalo (0, C), donde C es desconocida. La estimación que usted hace del valor del reloj es
de 400 dólares. ¿Cuánto deberá ofrecer por él?

Su problema es ajustar su estimación de manera que si resulta ser la más alta de las 50, no ofrecerá
más, en promedio, que el verdadero valor del reloj, que es C/2. El valor esperado de la más alta de las
50 estimaciones es (50/51)C, de manera que si su estimación de 400 resulta ser la más alta, se tiene,
en promedio,
(50/51)C = 400, (A.6.7)

de donde se obtiene C = 408. Ésta es su estimación ajustada del valor de C suponiendo que la suya
sea la más alta de las 50 estimaciones no ajustadas. Como el verdadero valor del reloj es C/2, usted
deberá ofrecer nada más 204 dólares.

EJERCICIO A.6.2
Si en el ejemplo A.6.2 sólo hubiera habido cuatro postores, ¿de cuánto habría sido su oferta?

2
Las variables aleatorias X1…, XN se conocen como “estadísticos de orden” de una muestra de tamaño N. Para hallar el valor espe-
rado de XN, observe primero que XN es menor que cualquier valor z si y sólo si cada uno de los N valores tomados como muestra es
menor que z. En la distribución uniforme en (0, 1), la probabilidad de ese evento es ZN. Ésta es la función de distribución acumulada
de XN. La función de densidad de probabilidad de XN es entonces d(zN)/dz, es decir NzN–Y. El valor esperado de XN está dado por
1 N
∫ 0
zNz N –1 dz =
N +1
.

06_CHAPTER 6.indd 205 3/6/09 7:55:30 PM


206 Capítulo 6 apéndiCe 6: teoría de la búSqueda y la maldiCión del ganador

En la práctica, ¿los postores en realidad ajustan su comportamiento para eliminar la maldición


del ganador? Puede darse un argumento evolucionista de que habrá una tendencia automática a que
esto suceda. La idea es que los postores que no ajustan sus estimaciones hacia abajo acaban perdien-
do dinero cada vez que ganan una subasta y al final llegarán a la bancarrota.
Cuando están en juego sumas excesivamente grandes de dinero y la bancarrota es una posibili-
dad real el argumento evolutivo tiene una fuerza obvia. Pero Michael Manove, un economista de la
Universidad de Boston sugiere que este argumento no será aplicable en todas las circunstancias. Él
observa que aun cuando alguien pague demasiado por algo en una subasta, con frecuencia es posible
recuperar la pérdida mediante un esfuerzo adicional. En tales casos la persona se arrepentirá de ha-
ber pagado demasiado pero su supervivencia económica no estará en juego, sólo su tranquilidad y su
paz interior. En el esquema de Manove, son los optimistas descuidados los que ignoran la maldición
del ganador y ganan todas las subastas. Entonces tendrán el reto de permanecer en pie y al final
muchos de ellos lo logran. Su utilidad es menor que si no hubieran ganado las subastas. Lo irónico es
que logran sacar del mercado a muchos postores más realistas.
Para ejemplificar lo que quiere decir, Manove cita su propia decisión acerca de si debía aceptar
la vicepresidencia de su corporación. Una persona que pondera esta decisión sabe que ser vicepresi-
dente lleva consigo una retribución conocida e implica una cantidad desconocida de trabajo. Manove
aceptó, diciendo que la retribución parecía valer la pena de acuerdo con su estimación de la cantidad
de trabajo que ese puesto implicaría. Sin embargo, el puesto resultó consumir mucho más tiempo
de lo que él esperaba y al final Manove acabó ligeramente más rico pero mucho menos satisfecho
que si hubiera renunciado a él. Su anécdota ejemplifica la advertencia de que el modelo de elección
racional se queda corto en términos puramente descriptivos. Pero en este caso el modelo ofrece de
nuevo una útil orientación que llama la atención hacia situaciones específicas en las que usted puede
equivocarse.
Desde hace años conozco la maldición del ganador y, sin embargo, siempre que voy a una subas-
ta de antigüedades pujo por todo aquello que pienso que vale la pena. ¿Soy irracional? La respuesta
depende de lo que yo quiera hacer con lo que compro. En muchas de las subastas a las que voy hay
muchos participantes que son comerciantes; yo sería irracional si fuera comerciante y no ajustara
mis estimaciones. Pero no lo soy. Lo que compro es para mi uso personal. Como cada artículo es
único, no se trata de ver si lo podría comprar más barato en otro lado. Para mí la única prueba que
debe pasar mi oferta es si el artículo vale esa cantidad para mí.

EJEMPLO A.6.3 Suponga que usted es el asesor económico de una empresa que está por decidir si adquiere
la compañía Bumbler Oil, cuyo único recurso es un campo petrolero que tiene un valor neto
X, con su administración actual. Los propietarios de Bumbler conocen exactamente el valor
de X, pero su empresa sólo sabe que X es un número aleatorio distribuido de manera unifor-
me entre 0 y 100. Debido a la mejor administración de la empresa para la que usted trabaja,
el campo petrolero de Bumbler valdría 1.5X en sus manos. ¿Cuál es el valor P más alto que
puede usted ofrecer, sin esperar sufrir una pérdida, en caso de que su empresa adquiera Bum-
bler Oil?

Si la empresa de usted ofrece P y X es mayor que P, Bumbler rechazará la oferta y no habrá trato. En
ese caso su empresa tendrá una ganancia cero. Si Bumbler acepta la oferta de su empresa, entonces se
sabe que X < P. Dado que X está distribuida de manera uniforme entre 0 y 100, el valor esperado de X
dado que X < P, es P/2. Esto significa que para su empresa el valor esperado del campo petrolero es
1.5 veces eso, es decir 0.75P. Si su empresa ofrece P y Bumbler acepta, entonces la ganancia esperada
para su empresa será 0.75P – P = –0.25P. De manera que para cualquier P positiva, su empresa espera
perder 0.25P si Bumbler acepta su oferta. La mejor estrategia de su empresa es, por lo tanto, no hacer
ninguna oferta.

06_CHAPTER 6.indd 206 3/6/09 7:55:31 PM


algunoS eSColloS para maximizar la utilidad eSperada 207

ALGUNOS ESCOLLOS PARA MAXIMIZAR


LA UTILIDAD ESPERADA
El modelo de la utilidad esperada ofrece una orientación inteligente acerca de cómo elegir en forma
racional ante la incertidumbre. La utilidad normativa de este modelo se subraya por el hecho de
que la gente parece necesitar con urgencia de su orientación. Un ejemplo muy conocido, basado en
el trabajo del economista francés M. Allais, indica que la mayoría de las personas se comportan de
manera inconsistente con respecto a ciertos tipos de elecciones. Para ilustrar esto, considere primero
el par de alternativas:

A: una ganancia segura de 30 dólares


versus
A': una posibilidad de 80% de ganar 45 dólares.

Ante estas alternativas, la mayoría de las personas eligen A, la ganancia segura.3 Si una persona
tiene aversión al riesgo, esta elección no debe sorprender, aun cuando el valor esperado de la alterna-
tiva A' sea de 36 dólares.
Ahora considere el siguiente par de alternativas:

B: una posibilidad de 25% de ganar 30 dólares


versus
B': una posibilidad de 20% de ganar 45 dólares.

Esta vez, la mayoría de las personas elige la alternativa menos segura, es decir, B'. Tomada de manera
aislada, esta elección tampoco sorprende, ya que el valor esperado de B(7.50 dólares) es significati-
vamente menor que el de B' ($9) y ambas alternativas conllevan cierto riesgo. El problema es que el
par de alternativas más elegidas (A y B' ), juntas, contradicen el supuesto de la maximización de la
utilidad esperada. Para ver por qué, suponga que la persona que elige gusta de maximizar la utilidad,
que su función de utilidad es U(M) y que su riqueza inicial es M0. Su decisión de elegir A en lugar de
A' implica que

U(M0 + 30) > 0.8U(M0 – 45) + 0.2U(M0). (A.6.8)

Y su decisión de elegir B en lugar de B' implica que

0.2U(M0 + 45) + 0.8U(M0) > 0.25U(M0 + 30) + 0.75U(M0). (A.6.9)

Reordenando los términos de la desigualdad A.6.9, se tiene

0.25U(M0 + 30) < 0.2U(M0 + 45) + 0.05U(M0). (A.6.10)

Dividiendo ambos miembros de la desigualdad A.6.10 entre 0.25, al final se tiene

U(M0 + 30) < 0.8U(M0 + 45) + 0.2U(M0), (A.6.11)

que es precisamente la desigualdad contraria a la desigualdad A.6.8, la que se obtiene al elegir A en


lugar de A'. Por lo tanto se tiene una contradicción.
Los psicólogos Daniel Kahneman y Amos Tversky han denominado a esta inconsistencia “efec-
to de seguridad”. Como asunto puramente descriptivo argumentan que “la disminución de la proba-

3
Vea Amos Tversky y Daniel Kahneman, “The Framing of Decisions and the Psychology of Choice”, en Science, 211, 1981: pp.
453-458.

06_CHAPTER 6.indd 207 3/6/09 7:55:31 PM


208 Capítulo 6 apéndiCe 6: teoría de la búSqueda y la maldiCión del ganador

bilidad de un resultado por un factor constante tiene mayor impacto cuando el resultado es seguro
que cuando es nada más probable”.4
Así, en el primer par de alternativas, el paso de A a A' representa una disminución de 20% (de 100
a 80%) en la posibilidad de ganar, igual que la reducción al pasar de B a B' (de 25 a 20%). Pero debido a
que la primera reducción proviene de un resultado inicialmente seguro, la aversión es mayor.
Observe que Kahneman y Tversky no afirman que haya algo irracional en preferir lo seguro. Lo
que afirman es que elegir en una situación en la que ambas alternativas son arriesgadas parece entra-
ñar un menor grado de aversión al riesgo que otra en la que una de las opciones está libre de riesgo.
Parte de la atracción de la alternativa segura es el arrepentimiento que la gente espera sentir
cuando hace una apuesta y pierde. Sin embargo, el maximizador de la utilidad esperada querrá tener
cuidado de evitar la falacia de “mal resultado implica mala decisión”. Para ilustrar esta falacia, supon-
ga que alguien le propone la apuesta siguiente: de una urna que contiene 999 canicas blancas y una
canica roja usted tiene que sacar una canica. Si saca una blanca, que es lo más probable, ganará 1 000
dólares. Pero, si saca la única roja, perderá uno. Suponga que usted acepta la puesta y saca la roja.
Pierde un dólar. ¿Diría que tomó una mala decisión? Si es así, está siendo víctima de esta falacia. La
decisión, cuando la tomó, era obviamente buena. Casi cualquier persona racional hubiera decidido
de la misma manera. Lástima que perdió, pero eso no dice nada acerca de la calidad de su decisión.
De la misma manera, si usted elige una posibilidad de 80% de ganar 45 dólares en lugar de una
ganancia segura de 30 dólares, no hay razón para lamentarse por la calidad de su decisión en caso de
perder.
Por lo general la naturaleza humana prefiere la certeza en lugar del riesgo. Sin embargo, al mis-
mo tiempo el riesgo es parte ineludible del entorno. La gente desea la mayor ganancia y el menor
riesgo posibles, pero casi siempre se ve forzada a elegir entre riesgo y ganancia. Cuando estamos
obligados a elegir entre dos alternativas arriesgadas, tenemos que reconocer de manera explícita
esta disyuntiva. En tales casos no se puede eludir el esfuerzo cognitivo necesario para llegar a una
decisión sensata. Pero cuando una de las alternativas es menos arriesgada, suele ser más fácil simple-
mente elegirla y no desperdiciar demasiado esfuerzo en la decisión. Sin embargo, lo que este patrón
de comportamiento no reconoce es que el elegir una ganancia segura de 30 dólares en lugar de una
posibilidad de 80% de ganar 45 dólares reduce muy poco cualquiera de las incertidumbres que tienen
importancia real en la vida.
En cambio, cuando están en juego sólo sumas pequeñas de dinero, un argumento convincente es que la
única estrategia razonable es elegir la alternativa que tenga el mayor valor esperado. El argumento para esta
estrategia, igual que para comprar un seguro, descansa en la ley de los grandes números. Aquí, la ley
indica que si se acepta una cantidad grande de apuestas independientes y se reúnen todas, con gran
seguridad se obtendrá casi exactamente la suma de sus valores esperados. Al tomar la decisión, el
truco es recordar que cada riesgo pequeño es sólo parte de una colección mucho mayor. Después de
todo, estar consciente de que haber seguido cualquier otra estrategia hubiera llevado a una pérdida
grande casi de seguro quitará el sinsabor de una pequeña pérdida ocasional.
Para ilustrar esto, considere de nuevo la elección entre una ganancia segura de 30 dólares y una
posibilidad de 80% de ganar 45, y suponga que cada semana usted se enfrenta a una elección equiva-
lente. Recuerde que el valor esperado de la apuesta es de 36 dólares, 6 más que la opción segura. Si
elige siempre la alternativa más “arriesgada”, su ganancia esperada —muy por encima de la ganancia
de la alternativa segura— será 312 dólares por año. Los estudiantes que hayan tenido un curso básico
de probabilidad pueden mostrar que la probabilidad de obtener un mejor resultado eligiendo cada
año la alternativa segura es menor que 1%. A largo plazo el costo de oportunidad de seguir una estrategia
adversa al riesgo en decisiones en las que intervienen resultados pequeños es una PÉRDIDA casi segura de mag-
nitud considerable. Si piensa en su problema como la elección de una póliza para manejar un número
grande de opciones del mismo tipo, una estrategia arriesgada en apariencia se transforma en una
obviamente muy segura.

4
Ibídem, p. 456.

06_CHAPTER 6.indd 208 3/6/09 7:55:32 PM


reSpueStaS a loS ejerCiCioS del apéndiCe 209

pRoblEMaS
1. Usted busca un salario alto en una distribución de salarios que es uniforme en el intervalo (5, 8). El
costo de cada búsqueda es 0.06. ¿Cuál es el salario más bajo que deberá aceptar?
2. Los 100 estudiantes de una clase participan en una subasta para ver quién se queda con una jarra
que contiene monedas de 25 centavos de dólar. Cada estudiante tiene una estimación no sesgada del
valor de las monedas. Si las estimaciones están tomadas del intervalo (0, C), donde no se conoce C, y
si su estimación es de 50 dólares, ¿cuánto deberá ofrecer usted?
3. Suponga que usted es el asesor económico de un empresa que tiene que decidir si debe adquirir la
compañía Bumbler Oil, cuya única ventaja es un campo petrolero cuyo valor neto con su adminis-
tración actual es X. Los propietarios de Bumbler Oil saben con exactitud cuál es el valor de X, pero
su empresa sólo sabe que X es un número aleatorio distribuido de manera uniforme entre 0 y 100.
Debido a la mejor administración de la empresa para la que usted trabaja, en sus manos el valor del
campo petrolero de Bumbler Oil será X + 40.
a) ¿Cuánto es lo máximo que puede ofrecer su empresa sin esperar tener una pérdida?
b) Si se supone que su empresa sea el único postor, ¿cuál es la oferta que maximiza la ganancia espe-
rada de su empresa?
4. Si los salarios que le han ofrecido están distribuidos de manera uniforme entre 75 y 150 dólares y si el
costo de buscar otro empleo es de dos dólares, ¿cuál es el salario mínimo que deberá aceptar?
5. Un servicio de citas cobra 100 dólares por cada cita conseguida por ellos. Todas las personas que
van a estas citas aceptarán una oferta de matrimonio. De acuerdo con sus propias estimaciones, la
calidad de un cónyuge potencial ofrecido por el servicio de citas puede medirse mediante un índice
que va de 0 a 100. Los cónyuges potenciales están distribuidos de manera uniforme en este intervalo.
Suponga que usted valora un cónyuge en 50 dólares por punto índice. Si sus citas fueran tomadas
de manera aleatoria del fondo de citas del servicio que las concierta, ¿en qué valor del índice dejaría
de buscar?
6. Usted es un comprador que trabaja para un concesionario de autos usados. Asiste a las subastas y
puja por automóviles que venderá el concesionario. Los automóviles se venden “como están” y casi
nunca hay oportunidad de inspeccionarlos con cuidado. En estas condiciones el límite más bajo para
el valor de un automóvil puede ser cero. En la subasta se ha ofrecido un Dodge Aries 1981. Usted
es uno de 20 postores. Su estimación del valor del automóvil es de 200 dólares. Si todos los posto-
res tienen estimaciones no sesgadas obtenidas de una distribución uniforme con un límite superior
desconocido, ¿cuál es la cantidad máxima que puede ofrecer por el auto sin que su cliente pierda
dinero?

RESpUEStaS a loS EjERcicioS dEl apéNdicE


A.6.1 Sea de nuevo w* el salario de aceptación. La probabilidad de hallar un salario más alto es (60
– w*)/50. La ganancia promedio, si encuentra un salario más alto, es (60 – w*)/2. De manera que
la ganancia esperada es el producto de estos dos (60 – w*)2/100. Igualando lo anterior con el costo
de la búsqueda, 1, y despejando w* se tiene w* = 50.

Salario
10 w* = 50 60

A.6.2 Si hay cuatro postores, el valor esperado de la estimación más alta es (4/5)C. Igualando esto con
400 y resolviendo, se obtiene C = 500, de manera que usted debe ofrecer 250.

06_CHAPTER 6.indd 209 3/6/09 7:55:33 PM


06_CHAPTER 6.indd 210 3/6/09 7:55:33 PM
Capítulo

7
EXPLICACIÓN DE LOS GUSTOS:
IMPORTANCIA DEL ALTRUISMO
Y DE OTROS COMPORTAMIENTOS
NO EGOÍSTAS

E
l supuesto central del análisis microeconómico es que la gente es racional. Sin embargo,
de ninguna manera existe un acuerdo universal acerca de lo que es ser racional. Dos defi-
niciones importantes de racionalidad son los llamados estándares del objetivo presente y del
interés personal.1 Una persona es racional, desde el punto de vista del estándar de objetivo presente,
si es eficiente en la persecución de cualquier fin que tenga en el momento de la acción. De acuerdo
con este estándar, no se hace ningún intento de valorar si los fines mismos tienen algún sentido. Si,
por ejemplo, alguna persona tiene preferencia por las conductas autodestructivas, el único requisito
para ser considerada racional según el estándar en cuestión es que persiga esa conducta de la forma
más eficiente de que disponga. Por otra parte, conforme al estándar del interés personal, desde el
principio se supone que las motivaciones de la gente coinciden con sus estrictos intereses materiales.
Motivaciones como el altruismo, la lealtad a los principios, el deseo de justicia y cosas por el estilo, no
son considerados según el estándar del interés personal.

1
Vea Derek Parfit, Reasons and Persons, Oxford: Clarendon, 1984.

211

07_CHAPTER 7.indd 211 3/6/09 7:56:04 PM


212 Capítulo 7 expliCaCión de los gustos

¿Hola? No, papá La gente siempre supone

Calvin and Hobbes Copyright © 1995 Watterson. Distributed


¿Que yo tome un recado?

by Universal Press Syndicate.Reprinted with permission. All


que uno es una especie
no está. No sé, ¿qué gano con eso? de altruista.

rights reserved.
En los libros de texto, al tratar sobre la elección racional, los economistas suelen adoptar el
estándar del objetivo presente. Se dice que los gustos se dan de manera exógena, y que no hay base
lógica para cuestionarlos. De acuerdo con las palabras de Jeremy Bentham, economista del siglo xix,
el gusto por las tachuelas no es menos válido que el gusto por la poesía.
El problema con el estándar del objetivo presente es lo que se llama el problema del “aceite
para coche”. Si se ve que una persona se bebe el aceite usado de su automóvil y luego agonizante se
retuerce de dolor y muere, podrá decirse que realmente le gustaba el aceite para coche. (¿Qué otra ra-
zón tendría para beberlo?) Casi cualquier comportamiento, no importa cuán extraño sea, puede ser
“explicado” a posteriori suponiendo simplemente el gusto por él. De esta manera el principal atracti-
vo del modelo del objetivo presente resulta ser también su mayor debilidad. Por permitir explicarlo
todo, acaba por no explicar nada.
Con este problema en mente, la mayoría de los economistas adopta en sus investigaciones algu-
na versión del estándar de racionalidad del interés propio. Éste es el punto de vista adoptado en los
capítulos anteriores y, como se ha visto, arroja muchas luces sobre el comportamiento humano. Por
ejemplo, ayuda a explicar por qué, cuando aumenta el precio de la gasolina, las personas se agrupan
para ir juntas en un solo automóvil al trabajo; por qué los miembros de organizaciones de “servicio”
como el Club Rotario o el Club Kiwanis tienden a ser vendedores de bienes raíces, dentistas, agentes
de seguros o comerciantes, y no empleados de correos, pilotos de línea aérea o cosas por el estilo. Sin
duda, el interés personal es una importante motivación humana.
Aun así, el estricto interés personal seguramente no es la única motivación humana. En las ca-
rreteras, los viajeros dejan propina a meseras que nunca volverán a ver. Las personas que intervienen
en terribles pleitos familiares buscan venganza sin importarles el costo que tiene para sí mismas.
La gente da la espalda a transacciones rentables cuyos términos considera “injustos”. Los británi-
cos gastaron vastas sumas de dinero defendiendo las desoladas Malvinas de Argentina, a pesar de la
dificultad para volver a hacerlo en el futuro. (El escritor argentino Jorge Luis Borges comparó esta
guerra con dos calvos peleándose por un peine.) De esta manera y de otras muchas, la gente no
parece seguir intereses del tipo egoísta usual.

VISIÓN PRELIMINAR DEL CAPÍTULO


El modelo del interés personal ignora el hecho de que la mayoría de nosotros tenemos diversos ob-
jetivos que parecen estar en conflicto con el estricto interés personal. Este capítulo empieza con un
ejemplo de cómo incorporar las motivaciones desinteresadas al modelo de la elección racional; en
este caso, una sencilla aplicación del estándar del objetivo presente. Pero el verdadero desafío es
explicar cómo tales motivaciones pueden llegar a ocupar un lugar primordial. Para todos es cono-
cida la idea de que alguien que trata de ser más espontáneo de forma deliberada está condenado al
fracaso. Así, también, se verá que las personas cuyo único objetivo es promover su interés personal
se encuentran con una dificultad de tipo similar. Existen problemas importantes que las personas
egoístas no son capaces de resolver bien.
Se verá que el comportamiento de las personas puede predecirse mucho mejor tomando en
cuenta ciertas fuentes de motivación desinteresadas.

07_CHAPTER 7.indd 212 3/6/09 7:56:05 PM


una apliCaCión del estándar del objetivo presente: las preferenCias altruistas 213

UNA APLICACIÓN DEL ESTÁNDAR DEL OBJETIVO


PRESENTE: LAS PREFERENCIAS ALTRUISTAS
Por experiencia, se sabe que no todo el mundo tiene las preferencias estrictamente egoístas supuestas
en el modelo del interés personal, resulta tentador ampliar el análisis agregando gustos adicionales, si
se supone, por ejemplo, que la gente obtiene satisfacción de una diversidad de conductas que están en
conflicto con el interés personal definido en estricto, tales como donar dinero para obras de caridad,
poner la basura en su lugar, etc. A continuación se analiza la posibilidad de incorporar de manera
formal las preferencias altruistas de algunas personas a un modelo de elección racional.
Considere, por ejemplo, el caso de Franny, quien se preocupa no sólo por sus propios ingresos
sino también por los de Zooey. Tales preferencias pueden representarse mediante un mapa de indife-
rencia definido con base en sus niveles respectivos de ingreso tal y como se presenta en la figura 7.1.
Observe que las curvas de indiferencia de Franny tienen pendiente negativa, lo cual significa que está
dispuesta a tolerar una reducción de su propio ingreso a cambio de un incremento suficientemente
grande en el de Zooey. Observe también que sus curvas de indiferencia muestran RMS decrecientes,
lo cual significa que cuanto mayor sea el ingreso que tenga Franny, estará dispuesta a renunciar a una
mayor parte de él con el fin de que Zooey tenga más.

FIGURA 7.1
Ingreso de Franny Aumento de la satisfacción Mapa de indiferencia
de Franny, una persona
altruista
franny está dispuesta a
tener menos ingresos para
que Zooey tenga más.

I3
I2
I1

Ingreso de Zooey

La pregunta a la que se enfrenta Franny es si estará en una mejor situación dando parte de ese
ingreso a Zooey. Para responder esta pregunta, primero se necesita representar la restricción de
presupuesto relevante para Franny. Suponga que su ingreso inicial es de 50 000 dólares/año y que el
de Zooey es de 10 000, como se indica mediante el punto A de la figura 7.2. ¿Cuáles son las opciones
que tiene Franny? Puede quedarse con todo su ingreso, en cuyo caso estará en A. También puede dar
parte de su ingreso a Zooey; en este caso tendrá un dólar menos de sus 50 000 por cada dólar que dé
a Zooey. Aquí, su restricción de presupuesto es el lugar geométrico que se indica como B en la figura
7.2, cuya pendiente es –1.
Si Franny se queda con todo su ingreso, terminará estando en la curva de indiferencia indicada
como I1 en la figura 7.2. Pero como su RMS es mayor que la pendiente de su restricción de presu-
puesto en A, es claro que puede hacer algo mejor. El hecho de que en A la RMS > 1 indica que está
dispuesta a dar más de un dólar de su ingreso con tal de que Zooey tenga un dólar extra. Pero la
pendiente de su restricción de presupuesto indica que sólo le cuesta un dólar darle a Zooey un dólar
más. Por lo tanto será mejor para ella dar parte de su ingreso a Zooey. La transferencia óptima está
representada por el punto de tangencia indicado como C en la figura 7.2. Lo mejor que puede hacer
es dar 19 000 dólares de su ingreso a Zooey.
Sin embargo, observe que si en la figura 7.2 Franny hubiera partido del punto D y no de A, la
conclusión hubiera sido muy diferente. Entonces su restricción de presupuesto habría sido nada más
la porción del lugar geométrico B que está por debajo de D. (¡No tiene la opción de dar un donativo
negativo a Zooey!) Y dado que su RMS en D es menor que 1, lo mejor que puede hacer es no dar nin-

07_CHAPTER 7.indd 213 3/6/09 7:56:06 PM


214 Capítulo 7 expliCaCión de los gustos

FIGURA 7.2 Ingreso de Franny


Transferencia óptima
A
de ingreso de una 50 000
persona altruista
en el punto C, la rMs de
franny entre su ingreso
y el de Zooey es igual al
valor de la pendiente de su 31 000 C
restricción de presupuesto.
dadas sus preferencias, lo
mejor que puede hacer es
dar 19 000 dólares de su I2
D
ingreso inicial de 50 000 y I1
quedarse con 31 000.
B

Ingreso de Zooey
0 10 000 29 000 60 000

gún dinero a Zooey. (Recuerde que en el capítulo 3 se vio que tales resultados se llaman soluciones
de esquina.)

EJEMPLO 7.1 La función de utilidad de Smith está dada por US = MS Mj, donde MS es la riqueza de Smith
y MJ es la de Jones. Al inicio, Smith tiene 90 unidades de riqueza y Jones tiene 10. Si Smith es
un maximizador de la utilidad, ¿deberá transferir parte de su riqueza a Jones?, y si es así,
¿cuánto? Trace la curva inicial de indiferencia de Smith y la curva de indiferencia cuando cada
uno tenga 50 unidades de riqueza. Trace también la restricción de presupuesto de Smith en
el plano MS MJ.
Smith y Jones juntos tienen 100 unidades de riqueza. Esto significa que si la riqueza de Smith es MS,
entonces la de Jones es 100 – MS. Por lo tanto, la función de utilidad de Smith puede escribirse como
US = MS(100 – MS), que corresponde a la gráfica superior de la figura 7.3. Observe que la utilidad de
Smith alcanza un máximo de 2 500 cuando MS = 50. En el punto de asignación inicial de la riqueza,
la utilidad de Smith es de sólo 900. De manera que si Smith es un maximizador de la utilidad, deberá
transferir 40 unidades de su riqueza a Jones. La línea continua de la gráfica inferior es la restricción de
presupuesto de Smith en el plano MSMJ. Observe que la curva de indiferencia US = 2 500 es tangente
a la restricción de presupuesto en MS = MJ = 50.

PAPEL ESTRATÉGICO DE LAS PREFERENCIAS


Lo atractivo del estándar de racionalidad del objetivo presente es que permite ampliar el análisis para
abarcar motivaciones no egoístas cuya existencia está bien documentada. Sin embargo, como se ha
observado, el problema metodológico persistente es que, a menos que se impongan algunas restric-
ciones, el estándar del objetivo presente permite explicar casi cualquier conducta extraña con sólo
manifestar el gusto por ella. El problema es cómo expandir la visión de las motivaciones humanas sin
volverse al mismo tiempo vulnerable a la objeción del aceite para automóviles.
Los biólogos han descubierto una manera de salir de este dilema que descansa en un análisis de
carácter puramente microeconómico. En la biología, los gustos de un organismo no se dan de ma-
nera arbitraria, como en los modelos económicos. Los biólogos suponen que los gustos son forjados
por las presiones de la selección natural para ayudar a los organismos a resolver problemas impor-
tantes de su entorno. Considere por ejemplo el gusto humano por lo dulce. ¿Cómo lo explican los
biólogos? Su argumento es sencillo. Parten de la observación de que ciertos tipos de azúcares —en
particular, los que se encuentran comúnmente en los frutos maduros— eran más fáciles de digerir
por nuestros antecesores primates. El paso siguiente es suponer cierto grado de variabilidad en los

07_CHAPTER 7.indd 214 3/6/09 7:56:07 PM


papel estratégiCo de las preferenCias 215

Utilidad de Smith FIGURA 7.3


US Un altruista
maximizador de la
2 500 utilidad
en la gráfica superior se
muestra que la utilidad
US = MS(100 – MS) de smith se maximiza si
conserva sólo 50 unidades
de riqueza. en la gráfica
inferior, la recta continua
es la restricción de
900 presupuesto de smith en el
plano MSMJ. observe que
la curva de indiferencia US
= 2 500 es tangente a la
restricción de presupuesto
MS en MS = 50.
50 90 100
MJ

100

50

US = 2 500
US = 900
MS
50 90 100

gustos de los individuos: es decir, suponer que algunos individuos tenían mayor predisposición que
otros a “preferir” el tipo de azúcares que se encuentra en los frutos maduros.
Motivados por este gusto, era más probable que estos primates comieran frutos maduros. Dado
que los nutrientes eran escasos y que los azúcares en los frutos maduros eran más fáciles de digerir,
los individuos a los que les gustaban tuvieron mayor posibilidad de sobrevivir y de dejar descenden-
cia. Debido a esta ventaja, los genes del gusto por los azúcares de los frutos maduros acabaron por
extenderse entre la población. Por lo tanto, de acuerdo con esta explicación biológica, el gusto por
lo dulce es una característica heredada de nuestros ancestros, entre los que se desarrolló por razones
funcionales.
Existe evidencia de que este gusto por lo dulce ha dejado de ser funcional en el ambiente pre-
sente. En el pasado, los azúcares que se encontraban en los frutos maduros eran bastante escasos,
de manera que no había peligro de consumirlos en exceso. En la actualidad, con tanta abundancia de
dulces, el gusto por ellos puede llevar a la exageración, con las consecuencias que esto implica. Si
éstas fueran suficientemente graves, la presión evolutiva acabaría por hacer disminuir el gusto por lo
dulce, pero como las transformaciones de este tipo requieren miles de generaciones, es casi seguro
que tendremos que cargar con él por ahora.
El gusto por lo dulce es sólo una preferencia simple, en el sentido de que ha sido útil para un
individuo sin importar si los demás miembros de la población comparten este gusto. Sin embargo,
otros gustos son más complejos, en el sentido de que su utilidad depende del porcentaje del resto de
la población que lo comparten. A este segundo tipo de gustos se le llama preferencia estratégica, la cual
ayuda a los individuos a resolver problemas importantes de interacción social. Un antiguo ejemplo
de preferencia estratégica en la literatura biológica es el gusto por la conducta agresiva. Un examen
cuidadoso del modelo biológico de la evolución de este gusto ayudará en el análisis posterior de otras
preferencias estratégicas importantes.

07_CHAPTER 7.indd 215 3/6/09 7:56:08 PM


216 Capítulo 7 expliCaCión de los gustos

PARÁBOLA DE LOS HALCONES Y LAS PALOMAS


Para empezar, considere una población que consta de individuos iguales excepto con respecto a su
gusto por un comportamiento agresivo. Un tipo, llamado “halcón”, tiene una fuerte preferencia por
tal comportamiento.2 El otro tipo, llamado “paloma”, prefiere evitar el comportamiento agresivo. A
cuál de estos dos tipos pertenece un individuo que sólo entra en conflicto con otro por un motivo
importante: alimento, una hembra, o lo que sea. La estrategia del halcón siempre es luchar por el
recurso. La estrategia de la paloma es no pelear nunca.
Si estos dos tipos compiten por un recurso escaso y necesario para la supervivencia, ¿quién gana-
rá? A primera vista puede parecer que los halcones ganarán, ya que en sus conflictos con las palomas
dominarán siempre. Pero con esta visión se pasa por alto lo que ocurre cuando se enfrentan dos
halcones. Como ambos individuos están predispuestos a ser agresivos, puede asegurarse una batalla
cruenta. Dependiendo de las consecuencias de una batalla, puede ser arriesgado ser un halcón.
La desventaja potencial de ser un halcón se vuelve más clara cuando se examina lo que ocurre
cuando dos palomas se enfrentan entre sí por un recurso importante. En estos encuentros se evitan
los costos de una batalla cruenta y las palomas comparten el recurso.
En esta población hipotética interactúan entre sí de manera aleatoria pares de individuos, y pue-
den darse tres tipos de pares: 1) dos palomas, 2) dos halcones y 3) un halcón y una paloma. Para ver
cómo evoluciona esta población es necesario saber cuál es la recompensa en cada uno de estos tres
tipos de interacción. Para facilitar el análisis suponga que un nutriólogo ha obtenido datos que per-
miten expresar esta recompensa en determinadas unidades, por ejemplo, calorías. Suponga que el
conflicto se debe a un alimento que contiene 12 calorías. Cuando la interacción es entre dos palomas,
éstas comparten el alimento, de manera que cada una obtiene una recompensa de 6 calorías. Cuando
la interacción es entre un halcón y una paloma, ésta retrocede ante el halcón, de manera que el hal-
cón obtiene 12 calorías y la paloma ninguna. Por último, cuando la interacción es entre dos halcones,
el ganador de la lucha obtiene 12 calorías y el perdedor ninguna. En la lucha misma, sin embargo,
cada halcón consume 10 calorías, lo que significa que la ganancia neta es de 12 – 10 = 2 calorías
para el halcón ganador y de –10 calorías para el halcón perdedor. A lo largo de muchos encuentros
entre halcones, cualquiera de ellos puede esperar ganar la mitad de las veces y perder la otra mitad.
Entonces, si se consideran los halcones como un todo, la ganancia promedio de cada participante en
un encuentro halcón-halcón es (2 – 10)/2 = –4 calorías.
Si X y Y representan dos individuos de la población, las ganancias promedio en las diferentes
combinaciones de interacción se resumen en la tabla 7.1.

dos individuos entran en


conflicto por el alimento, que
tiene un valor de 12 calorías.
Cuando se encuentran dos
halcones, hay una batalla en
TABLA 7.1 la que cada uno consume
Matriz de recompensa halcón-paloma 10 calorías, quedando una
recompensa neta promedio
Individuo Y de –4 calorías por halcón.
Cuando se encuentran
Halcón Paloma palomas y halcones, aquéllas
Halcón –4 calorías cada uno 12 calorías para X ceden ante éstos, de manera
que los halcones obtienen
0 calorías para Y 12 calorías y las palomas
Individuo X
Paloma 0 calorías para X 6 calorías cada una cero calorías. Cuando se
encuentran dos palomas,
12 calorías para Y
comparten el alimento, de
manera que cada una obtiene
6 calorías.

2
Aquí, “halcón” y “paloma” son palabras que se usan en sentido metafórico para describir a miembros de la misma especie que
tienen gustos distintos con respecto al comportamiento agresivo.

07_CHAPTER 7.indd 216 3/6/09 7:56:08 PM


papel estratégiCo de las preferenCias 217

Desde el punto de vista de los biólogos, la cuestión de si es mejor ser halcón o ser paloma se
responde calculando cuál de ellos obtiene, en promedio, más calorías. Para esto, es necesario co-
nocer primero la posibilidad de cada tipo de interacción. Para ilustrarlo, suponga que la mitad de
la población está compuesta por halcones y la otra mitad por palomas. Por lo tanto, la mitad de las
interacciones entre estos individuos será de halcones y la otra mitad de palomas. Entonces, la recom-
pensa promedio de un halcón, representada por PH, será un promedio ponderado de los valores de
las dos recompensas:
PH = ( 12 )(−4) + ( 12 )(12) = 4. (7.1)

La correspondiente recompensa promedio de una paloma, representada por PD, será

PD = ( 12 )(0) + ( 12 )6 = 3. (7.2)

El supuesto implícito, desde el punto de vista biológico, en la competencia entre halcones y pa-
lomas es que la clase que obtenga mayor cantidad de calorías tenderá a formar familias más grandes
y de esta manera tendrán una proporción mayor del total de la población. Se acaba de ver que en una
población en la que hay mitad de cada clase, los halcones obtendrán más calorías que las palomas, lo
que significa que el porcentaje de aquéllos en la población total crecerá.
Suponga que h representa la fracción de la población que son halcones (de manera que, en el
ejemplo que se acaba de considerar, sería h = ½). Como los porcentajes de ambas clases deben
sumar 1, 1 – h será la fracción de la población que son palomas. En una población así, la recompen-
sa promedio de los halcones es de nuevo un promedio ponderado de los dos tipos de recom-
pensa, donde ahora los pesos de ponderación son las respectivas proporciones en la población h y
(1 – h):
(7.3)
PH = (h )(−4) + (1− h )(12) = 12 −16h.

La expresión general correspondiente para la recompensa promedio de las palomas es


(7.4)
PD = (h )(0) + (1− h )6 = 6 − 6h.

Por ejemplo, si el porcentaje de halcones fuera de cuatro quintos, los halcones se encontrarían
con otros halcones en sus interacciones cuatro quintos de las veces y el quinto restante se encontrarían
con palomas, con lo que la recompensa promedio de los halcones sería PH = ( 45 )(−4) + ( 15 )(12) = −0.8.
El promedio correspondiente de las palomas sería PD = ( 45 )(0) + ( 15 )(6) = 1.2. De manera que cuan-
do los halcones constituyen cuatro quintos de la población, su recompensa promedio será menor
que la recompensa promedio de las palomas, lo que significa que el porcentaje promedio de los
halcones en la población empezará a disminuir.
Para ver si las proporciones de la población llegarán a algún equilibrio, se grafican las curvas de
recompensa promedio de las dos clases y se busca un punto en el que se crucen. Como se muestra en
la figura 7.4, esto ocurre cuando h = 0.6. Esto significa que cuando 60% de la población son halcones
y 40% son palomas, cada clase obtiene una recompensa promedio de 2.4 calorías por interacción. Al
tener recompensas promedio iguales, los dos tipos tenderán a tener la misma cantidad de crías, lo
que implica que sus correspondientes participaciones en la población permanecerán sin variar.
Observe que el punto de equilibrio de la figura 7.4 es estable: si el porcentaje de los hal-
cones en la población se desviara en algún momento de 0.6, de inmediato sería obligada a
volver a 0.6. Por ejemplo, si el porcentaje de halcones disminuyera por alguna razón a 0.5, de
acuerdo con las curvas de la figura 7.4 la recompensa promedio de los halcones sería superior
a la de las palomas y esto haría crecer el porcentaje de halcones en la población. Al contrario, si
la población de halcones aumentará a 0.7, entonces su recompensa promedio sería menor que la de
las palomas y esto haría disminuir el porcentaje de halcones en la población.

07_CHAPTER 7.indd 217 3/6/09 7:56:12 PM


218 Capítulo 7 expliCaCión de los gustos

FIGURA 7.4
Recompensas promedio
de halcones y palomas Calorías por
las recompensas promedio interacción
tanto de los halcones
como de las palomas son 12 PH = recompensa promedio de los halcones
funciones decrecientes de
la fracción de la población
constituida por halcones.
la mezcla de los dos tipos Punto de equilibrio
está en equilibrio cuando
6
las recompensas promedio
son iguales para los dos PD = recompensa promedio de las palomas
tipos.
2.4

0 h
0.1 0.2 0.3 0.4 0.5 0.6 0.7 0.8 0.9 1.0
Proporción de halcones en la población
–4

EJERCICIO 7.1
En el ejemplo anterior, suponga que ahora las recompensas se alteran como sigue: cuando in-
teraccionan dos palomas cada una gana tres unidades; si lo hacen dos halcones, cada uno gana
una unidad y cuando interaccionan una paloma y un halcón, la primera obtiene dos unidades y
el último seis. ¿Cuál será el porcentaje de cada clase de la población en el punto de equilibrio?

El ejemplo de los halcones y las palomas ilustra cómo la utilidad de una preferencia por un cierto tipo
de comportamiento depende de la frecuencia con la que otros miembros de la población también lo
prefieren. Ser un halcón (preferir la agresión) puede ser ventajoso, pero sólo hasta cierto punto. Una
vez que los halcones son numerosos lo que resulta ventajoso es ser paloma. La población está en
equilibrio sólo cuando las recompensas promedio de los dos gustos son iguales.
El ejemplo de los halcones y las palomas muestra también una propiedad importante de la evo-
lución debida a la selección natural, según la cual los rasgos son favorecidos con frecuencia por sus
efectos en las ganancias individuales, no por sus efectos en las ganancias de las poblaciones en su
conjunto. Observe en la figura 7.4 que para la población total sería mejor si no hubiera halcones (h =
0); es decir, en una población que se integrara sólo de palomas, todos los individuos obtendrían seis
calorías por interacción, lo cual es una mejora importante con respecto al valor de equilibrio de 2.4.
Pero una población que constara únicamente de palomas no sería estable. Un halcón podría invadir
esta población y progresar de manera rápida debido a su éxito al interactuar con las palomas.
El modelo de elección racional que se presenta en el capítulo 3 da por sentados los gustos del
consumidor, como un conjunto de objetivos que el consumidor busca satisfacer. Los modelos eco-
lógicos como el de los halcones y las palomas van un paso atrás y se preguntan de dónde provienen
esos gustos. Estos modelos ven las preferencias no como un fin en sí mismas, sino como medios me-
diante los cuales los individuos logran objetivos materiales importantes (en el caso de los halcones y
las palomas, la adquisición de las calorías necesarias para sobrevivir y reproducirse).
Partiendo del funcionamiento del modelo de los halcones y las palomas, puede analizarse ahora
cómo habrán surgido otros diversos gustos. En particular, se verá cómo ciertas motivaciones des-
interesadas suelen ayudar a la gente a resolver un tipo importante de problemas que surge en la
interacción económica y social.

07_CHAPTER 7.indd 218 3/6/09 7:56:13 PM


el probleMa del CoMproMiso 219

EL PROBLEMA DEL COMPROMISO


Uno de los ejemplos más discutidos en el que la persecución del interés personal es contraproducen-
te es el llamado dilema del prisionero. Al matemático A. W. Tucker se le atribuye haber descubierto
este sencillo juego, cuyo nombre proviene de la anécdota empleada al inicio para ejemplificarlo.
Dos prisioneros se encuentran en celdas separadas debido a un delito grave que cometieron. Sin
embargo, el fiscal sólo tiene evidencias para condenarlos por un delito menor, por lo cual la pena es,
por ejemplo, de un año de prisión. A cada prisionero se le dice que si uno de ellos confiesa y el otro
permanece en silencio, el que confiese quedará libre mientras el otro recibirá 20 años de prisión. Si
ambos confiesan, obtendrán una sentencia intermedia, por ejemplo, 5 años. En la tabla 7.2 se resu-
men estas recompensas. A los prisioneros no se les permite comunicarse entre sí.

TABLA 7.2 sin importar lo que haga el


El dilema del prisionero otro prisionero, cada uno de
ellos obtiene una sentencia
Prisionero Y más corta confesando.Y
si los dos confiesan, cada
Confiesa Permanece en silencio uno recibe 5 años. sin
Confiesa 5 años cada uno 0 años para X embargo, si los dos hubieran
permanecido en silencio, cada
20 años para Y uno hubiera recibido sólo 1
Prisionero X
Permanece en silencio 20 años para X 1 año cada uno año de prisión. en este caso
la persecución del interés
0 años para Y
personal produce para cada
uno, un resultado peor.

La estrategia dominante en el dilema del prisionero es confesar. Sin importar lo que haga Y, X
obtiene una sentencia menor si habla; si Y también confiesa, X recibe 5 años en lugar de 20; y si Y
permanece en silencio, X sale en libertad en lugar de pasar un año en prisión. Las recompensas son
perfectamente simétricas, de manera que para Y también es mejor confesar, sin importar lo que haga
X. El problema es que si los dos se comportan de acuerdo con su interés personal, es peor para am-
bos. Así, cuando los dos confiesan, obtienen 5 años, en lugar de sólo un año si hubieran permanecido
en silencio.
Aunque a los prisioneros no se les permite comunicarse uno con el otro, sería un error suponer
que esto es la verdadera fuente del problema. Su problema es más bien de falta de confianza. La sim-
ple promesa de no confesar no modifica las ganancias materiales del juego. (Si cada uno prometiera
no confesar, de cualquier manera lo mejor sería romper su promesa.)
El dilema del prisionero es un ejemplo de una clase más amplia de problemas llamados problemas
de compromiso. La característica común de éstos es que la gente puede obtener mejores resultados
si se compromete a comportarse de una manera que después será inconsistente con sus intereses
materiales personales. Por ejemplo, en el dilema del prisionero, si los prisioneros pudieran compro-
meterse a permanecer en silencio, sería mejor para ellos que tener la libertad de buscar sus estrictos
intereses materiales.
El economista Thomas Schelling3 de la Universidad de Maryland da otro interesante ejemplo de
un problema de compromiso. Schelling describe un secuestrador que de repente siente miedo, quie-
re dejar en libertad a su víctima pero teme que lo denuncie ante la policía. A cambio de su libertad
la víctima promete no hacerlo. El problema, sin embargo, es que ambos se dan cuenta de que una
vez libre, la víctima no cumplirá su promesa. De esta manera el secuestrador concluye, que a pesar

3
Thomas Schelling, The Strategy of Conflict, Cambridge, MA: Harvard University Press, 1960.

07_CHAPTER 7.indd 219 3/6/09 7:56:14 PM


220 Capítulo 7 expliCaCión de los gustos

suyo, debe matar a la víctima. La creencia del secuestrador de que la víctima actuará de una manera
racional, de acuerdo con su interés personal es aparentemente lo que condena a la víctima.
Schelling sugiere la siguiente solución para este dilema: “Si la víctima ha cometido un acto cuya
revelación pudiera llevar a un chantaje, puede confesarlo; si no, podría cometer un acto de este tipo
en presencia de su secuestrador para crear una garantía que asegure su silencio”.4 (Quizá la víctima
podría permitirle a su secuestrador fotografiarla realizando algún acto degradante. El acto que po-
dría provocar un chantaje funciona como una estrategia de compromiso, algo que proporciona
estrategia de compromiso
a la víctima un incentivo para cumplir su promesa. Una vez libre le será desagradable cumplir su
mecanismo que compromete compromiso, pero mucho menos que no poder hacer una promesa creíble.
a una persona a comportarse En la interacción económica y social cotidiana, con frecuencia se encuentran problemas de com-
de cierta manera en el futuro, promiso como el dilema del prisionero o como el problema de Schelling que confronta al secuestra-
aun cuando pueda desear dor y a la víctima. La solución que sugiere Schelling trata de eliminar el problema alterando los
comportarse de otro modo incentivos materiales relevantes. Sin embargo, por desgracia este método no siempre será práctico.
cuando llegue el momento. Una forma alternativa es modificar las recompensas psicológicas que gobiernan el comporta-
miento; en términos económicos, tener preferencias que lleven a la gente a comportarse de manera
contraria al interés personal estricto. Suponga, por ejemplo, que la víctima del secuestrador fuera
conocida como una persona que se sentiría mal si rompiera una promesa. Tal sentimiento, si es su-
ficientemente fuerte, le impedirá ir a la policía aun cuando esto vaya en contra de su propio interés.
Saber esto permitirá que el secuestrador lo deje libre.

EJEMPLO: EL PROBLEMA DEL ENGAÑO


El papel funcional de las motivaciones desinteresadas puede verse con más claridad con ayuda de
un ejemplo de sociedad sencilla en el cual egoístas se enfrentan a no egoístas en una lucha por so-
brevivir. Este problema de compromiso al que se enfrentan surge en empresas colectivas, cada una
de las cuales consta de un par de individuos. En dichas empresas cada persona puede comportar-
se de dos maneras: puede “cooperar”, lo que significa negociar honestamente con su socio, o puede
“desertar”, lo que significa engañar a su socio. Las recompensas representativas para cada uno de dos
socios, Smith y Jones, dependen, como se muestra en la tabla 7.3, de la combinación de comporta-
mientos elegida. Estas recompensas confrontan a los socios con una versión monetaria del dilema
del prisionero. Observe que Jones obtiene una recompensa mayor desertando, sin importar lo que
Smith haga, y lo mismo ocurre con Smith. Si Jones cree que Smith se comportará de acuerdo con
su interés personal, esperará a que Smith deserte. Y para protegerse a sí mismo, se sentirá obligado
a desertar también. Cuando los dos desertan, cada uno tiene sólo dos unidades de recompensa. La

4 Ibid., pp. 43-44.

TABLA 7.3
Recompensas monetarias en una empresa conjunta las recompensas en la tabla
tienen la misma estructura
que las del dilema del
Smith prisionero. Manteniendo el
Deserta Coopera comportamiento del otro
jugador constante, para cada
Deserta 2 para cada uno 0 para smith jugador es mejor desertar.
6 para jones sin embargo, cuando los dos
Jones desertan, cada uno obtiene
Coopera 6 para smith 4 para cada uno nada más 2, mientras que
0 para jones cuando los dos cooperan
cada uno obtiene 4.

07_CHAPTER 7.indd 220 3/6/09 7:56:14 PM


ejeMplo: el probleMa del engaño 221

frustración, como en todos los dilemas de este tipo, es que ambos podrían haber obtenido un mejor
resultado. Si hubieran cooperado, cada uno habría obtenido una recompensa de 4 unidades.
Ahora suponga que no sólo se trata de Smith y Jones sino de una población grande. De nuevo,
pares de personas forman empresas colectivas y la relación entre comportamiento y ganancias de los
miembros de cada par es como se muestra en la tabla 7.3. Suponga, además, que cada miembro de
la población pertenece a uno de los dos tipos: cooperador o desertor. Un cooperador es una persona
que quizás, a través de un intenso condicionamiento cultural, ha desarrollado la capacidad de experi-
mentar un sentimiento moral que lo predispone a la cooperación. Un desertor es una persona que o
bien carece de esta capacidad o no la ha podido desarrollar.
En este esquema, los cooperadores son altruistas en el sentido de que se abstienen de engañar
aun cuando no exista la posibilidad de ser descubiertos. Visto en el estricto contexto de la elección,
este comportamiento es contrario a sus intereses personales. Los desertores, en cambio, son simples
oportunistas, siempre eligen lo que maximice sus recompensas personales. Como en el ejemplo de
los halcones y las palomas que se consideró antes, el objetivo es determinar lo que ocurrirá cuando
miembros de estos dos grupos entren en una lucha por la supervivencia. Como se verá, la respuesta
depende de la facilidad con que puedan distinguirse entre sí estos dos grupos. Se considerarán diver-
sas posibilidades.

MOVIMIENTOS POBLACIONALES CUANDO


COOPERADORES Y DESERTORES SE PARECEN
Suponga que los cooperadores y los desertores se ven idénticos, siendo imposible distinguir entre
unos y otros. En esta sociedad hipotética, esto significa que los individuos formarán pares de manera
aleatoria, como en el ejemplo de los halcones y las palomas. Naturalmente, los cooperadores (y los
desertores) desearán formar parejas con cooperadores, pero no tienen la posibilidad de elegir. Dado
que todos se ven igual, tienen que arriesgarse. Las ganancias esperadas tanto para desertores como
para cooperadores dependen de la posibilidad de formar una pareja con un cooperador, lo que a su
vez depende del porcentaje de cooperadores en la población.
Sea c el porcentaje de población que son cooperadores. Si uno de ellos interactúa con una per-
sona elegida al azar de la población, la probabilidad de que esa persona sea también un cooperador
será c y la probabilidad de que sea un desertor será 1 – c. Como un cooperador obtiene 4 cuando
interactúa con otro cooperador y 0 cuando interactúa con un desertor, la recompensa esperada, o
promedio, para cada cooperador en este caso puede expresarse como

Pc = c ( 4) + (1− c )(0) = 4c.


(7.5)

Por lo tanto, cuando la mitad de la población consta de cooperadores (c = ½), un cooperador tiene
50% de posibilidad de interactuar con otro cooperador, en cuyo caso obtendrá 4 unidades, y 50% de
posibilidades de interactuar con un desertor, en cuyo caso obtendrá 0 unidades. Aquí, su recompensa
esperada es un promedio ponderado de estos dos resultados, es decir, 2 unidades.

EJERCICIO 7.2
¿Cuál es la recompensa promedio de un cooperador si c = 0.97?

La expresión correspondiente para la recompensa promedio de los desertores está dada por

PD = 6c + 2(1− c ) = 2 + 4c.
(7.6)

En la figura 7.5 se muestran las relaciones entre las recompensas promedio con los valores mo-
netarios supuestos en este ejemplo.

07_CHAPTER 7.indd 221 3/6/09 7:56:16 PM


222 Capítulo 7 expliCaCión de los gustos

FIGURA 7.5
Recompensas promedio
cuando los cooperadores
y los desertores tienen la
misma apariencia Recompensa promedio
las recompensas esperadas 6
de los desertores
tanto para los cooperadores
como para los desertores Recompensa promedio
aumentan con el porcentaje 4 de los cooperadores
de los primeros en la
población. pero, sin importar
cuál sea el porcentaje 2
inicial de cooperadores,
éstos obtienen una menor
recompensa que los Proporción de cooperadores en la población (c)
desertores. esto significa 0 0.5 1.0
que los cooperadores están
destinados a la extinción.

Cuando tanto los cooperadores como los desertores se ven igual, ¿cómo evolucionará la pobla-
ción a lo largo del tiempo? Como en el ejemplo de los halcones y las palomas, la regla es que cada
individuo se reproduce en proporción con su recompensa promedio: aquellos con recompensas ma-
teriales más elevadas cuentan con los recursos necesarios para criar cantidades mayores de vástagos.5
Recuerde que en el ejemplo de los halcones y las palomas, las curvas de recompensa promedio de los
dos tipos se intersecan, dando como resultado una proporción poblacional estable de cada tipo. Sin
embargo, en el ejemplo actual las curvas de recompensa promedio no se intersecan. Como los deser-
tores obtienen siempre una recompensa promedio mayor, su proporción en la población crecerá a lo
largo del tiempo. Los cooperadores, aun cuando al principio representen casi toda la población, están
destinados a la extinción. Cuando los cooperadores y los desertores son aparentemente iguales, la
cooperación genuina no puede surgir. De una manera cruda, este caso proporciona un fundamento
para el supuesto del modelo del interés personal del comportamiento egoísta.
Observe en la figura 7.5 que en una población que esté conformada sólo de cooperadores (c
= 1.0), la recompensa de todos será 4, es decir, el doble de lo que obtienen cuando el equilibrio
consta sólo de desertores. Como en el ejemplo de los halcones y las palomas, en este caso también
se observa que los gustos evolucionan de acuerdo con su efecto sobre las recompensas individuales,
no de grupo.

MOVIMIENTOS POBLACIONALES CUANDO


LOS COLABORADORES SON FÁCILMENTE IDENTIFICABLES
Ahora suponga que todo es como antes, excepto que los cooperadores y los desertores se distinguen
perfectamente. Para concretar, suponga que la simpatía es la emoción que motiva la cooperación
y que hay un síntoma observable en aquellas personas que la experimentan (quizás una “actitud
amable”). Los desertores carecen de este síntoma observable o, de manera más general, tratan de
imitarlo, pero sin conseguirlo con exactitud.
Si este síntoma puede observarse a simple vista, la situación ha cambiado por completo. Los
colaboradores ahora pueden interactuar de manera selectiva unos con otros y asegurarse una re-
compensa de cuatro.

5
Por supuesto que en los últimos tiempos ha habido una relación negativa entre ingreso y tamaño de la familia. Pero si las prefe-
rencias fueron forjadas por la selección natural, la relación que importa es la existente durante la mayor parte de la historia de la
evolución. Y esa relación fue positiva: durante los periodos de hambruna y los individuos con mayores recursos materiales pudieron
ver a un mayor número de sus hijos llegar a la edad adulta. Además, la mayoría de las sociedades primitivas fueron polígamas, sus
miembros más ricos tenían varias mujeres, dejando a muchos de los pobres sin ninguna.

07_CHAPTER 7.indd 222 3/6/09 7:56:17 PM


ejeMplo: el probleMa del engaño 223

FIGURA 7.6
Recompensas
promedio cuando los
cooperadores y los
6 desertores se distinguen
perfectamente
Cuando los cooperadores
Pago promedio pueden ser identificados
4 para los cooperadores a simple vista, pueden
interactuar siempre unos
Pago promedio con otros y obtener una
2 para los desertores recompensa de 4. los
desertores son dejados a un
lado para que interactúen
Proporción de cooperadores en la población (c) entre sí y obtienen una
0 0.5 1.0 recompensa de 2. en este
caso son los desertores los
que se extinguen.

Ningún cooperador necesita interactuar con un desertor. Los desertores son segregados para
que interactúen unos con otros, de manera que su recompensa será sólo 2. Como todos los elemen-
tos de la casualidad han sido eliminados del proceso de interacción, las recompensas ya no dependen
del porcentaje de cooperadores en la población (vea figura 7.6), los cooperadores obtienen siempre 4
y los desertores obtienen 2.
En este caso vez las recompensas que obtienen los cooperadores les permiten tener familias
más grandes, lo cual significa que constituirán una proporción cada vez mayor de la población. Si
los cooperadores pueden ser identificados con facilidad, son los desertores los que se enfrentan a la
extinción.

MIMETISMO SIN COSTO NI RETRASO


Sin embargo, los desertores no necesitan darse por vencidos sin luchar. Suponga que existe un tipo
de individuo que se comporta exactamente igual que los demás desertores, pero presenta los mismos
síntomas de fidelidad que los cooperadores. Como esta variedad de desertores se confunde con los
cooperadores, es imposible que éstos detecten a sus miembros. Por lo tanto, cada impostor tiene la
misma posibilidad que un cooperador genuino de interactuar con otro cooperador. Esto, a su vez, sig-
nifica que los desertores impostores tendrán una recompensa esperada mayor que los cooperadores.
Los desertores genuinos —aquellos que siguen teniendo una apariencia distinta a la de los
cooperadores— tendrán una recompensa menor que la de los otros dos grupos y, como antes, están
destinados a la extinción. Pero a menos que los cooperadores se adapten de alguna manera, ellos
también correrán la misma suerte. Cuando los desertores pueden imitar a la perfección la caracte-
rística distintiva de los cooperadores sin costo ni retraso, ésta pierde su poder distintivo. De nuevo
los cooperadores y los desertores supervivientes se ven idénticos, lo que una vez más supone la
extinción para los cooperadores.

MIMETISMO IMPERFECTO Y LOS COSTOS DE LA VIGILANCIA


Por supuesto que los desertores no tienen el monopolio de la capacidad de adaptación. Si una muta-
ción aleatoria modifica la característica distintiva de los cooperadores, los desertores se enfrentarán a
un objetivo cambiante. Imagine que la característica por la cual los cooperadores al principio logran
distinguirse entre ellos ha sido imitada de manera imperfecta por los desertores. Si los dos tipos
pudieran distinguirse de un vistazo, sería de nuevo el final para los desertores. Pero suponga que
se requiere un esfuerzo para distinguir entre cooperadores y desertores. Para ser más concretos,
suponga que la inspección cuesta una unidad. Pagar este costo es como comprar un par de lentes de
contacto que permiten distinguir de un vistazo a los cooperadores de los desertores. Para quienes no
pagan, los dos tipos siguen siendo indistinguibles.

07_CHAPTER 7.indd 223 3/6/09 7:56:17 PM


224 Capítulo 7 expliCaCión de los gustos

Para ver lo que ocurre ahora, suponga que las recompensas son, otra vez, como en la tabla 7.3 y
considere la decisión que tiene que tomar un cooperador entre pagar o no el costo de la supervisión.
Si paga puede estar seguro de interactuar con otro cooperador, con lo que obtendrá una recompensa
de 4 - 1 = 3. Si no paga, su recompensa es incierta. Para él, cooperadores y desertores se verán
idénticos y tendrá que arriesgarse. Si interactúa con otro cooperador, obtendrá 4. Pero si lo hace con
un desertor, obtendrá cero. Si vale o no la pena pagar este costo de una unidad por la supervisión
depende, entonces, de la posibilidad de que se presenten estos resultados.
Suponga que en la población el porcentaje de colaboradores es 90%. Al no pagar el costo de
la supervisión, un cooperador interactúa con otro cooperador 90% de las veces y con un desertor
sólo 10% de las veces. Su recompensa tendrá entonces un valor promedio de (0.9)(4) + (0.1)(0) =
3.6. Como esto es mayor que la recompensa neta de 3 unidades que obtendría si paga el costo de la
supervisión, es claro que es mejor no pagarlo.
Ahora suponga que el porcentaje de cooperadores en la población no es 90 sino 50%. Si un
cooperador no paga el costo de la supervisión, tendrá únicamente 50% de posibilidad de interactuar
con otro cooperador. Su recompensa promedio será entonces sólo 2, es decir 1 menos que si hubiera
pagado el costo. En este caso, es claramente mejor pagarlo.
En este ejemplo los números implican la existencia de un “umbral de rentabilidad” que se obtie-
ne despejando c de la ecuación siguiente:

4c = 3, (7.7)

lo que da c = 0.75. De manera que cuando el porcentaje de cooperadores en la población es 75%,


la recompensa esperada de un cooperador que no paga el costo de supervisión (4c) es igual que su
recompensa segura si lo hace (3). Un cooperador que no paga este costo tiene una probabilidad de
75% de obtener una recompensa de 4 y 25% de probabilidad de obtener cero, lo que significa una
recompensa promedio de 3, lo mismo que si hubiera pagado el costo. Si en la población el porcentaje
de cooperadores es menor que 75%, siempre será mejor pagar el costo de vigilancia. Si el porcen-
taje de cooperadores en la población es mayor que 75%, nunca será mejor pagar el costo.

EJERCICIO 7.3
En una población en la que 60% son cooperadores y en la que el costo de supervisión es igual a
1.5, ¿deberán los cooperadores pagar el costo de la supervisión?

Si se tiene en mente la regla del umbral de rentabilidad puede decirse cómo evolucionará la
población a lo largo del tiempo. Si el porcentaje de cooperadores en la población es menor que 75%,
todos los cooperadores pagarán el costo de supervisión y, al cooperar unos con otros, obtendrán
una recompensa de 3 unidades. A los desertores no les interesará cargar con este costo, dado que
de ninguna manera los cooperadores vigilantes interactuarán con ellos. A los desertores se les deja
interactuar unos con otros, con lo que obtienen nada más una recompensa de 2 unidades. De mane-
ra que si se parte de una población en la que el porcentaje de cooperadores es menor que 75%, los
cooperadores obtendrán una recompensa mayor, lo cual significa que aumentará su porcentaje en la
población.
En poblaciones en las que el porcentaje de cooperadores es mayor que 75% la cosa cambia.
Ahora ya no tiene sentido pagar el costo de la supervisión. Los cooperadores y los desertores inter-
actúan, entonces, en forma aleatoria, lo cual significa que los desertores tendrán una recompensa
promedio mayor. Esta diferencia en las recompensas, a su vez, hará que disminuya el porcentaje de
cooperadores sobre el total la población.
Con los valores supuestos en este ejemplo, las recompensas promedio de los dos grupos serán
los que se muestran en la figura 7.7. Como se observó, cuando el porcentaje de cooperadores es
menor que 75%, las recompensas de los cooperadores son superiores a las de los desertores, pero
cuando su proporción es mayor, aquéllos son menores. La marcada discontinuidad en las recom-
pensas de los desertores refleja el hecho de que, a la izquierda de 75%, todos los cooperadores pagan
por la supervisión, mientras que a la derecha de 75%, ninguno lo hace. Una vez que el porcentaje de

07_CHAPTER 7.indd 224 3/6/09 7:56:18 PM


ejeMplo: el probleMa del engaño 225

FIGURA 7.7
Recompensas promedio
con costos de supervisión
Cuando el porcentaje de
cooperadores es mayor que
75%, no es rentable para los
cooperadores cargar con
el costo de la supervisión.
esto da como resultado que
Recompensa promedio la recompensa esperada de
6
de los desertores los desertores sea superior
5 a la de los cooperadores
Recompensa promedio y que el porcentaje de
4 de los cooperadores cooperadores disminuya.
3 Cuando el porcentaje de
cooperadores es menor que
2 75%, los cooperadores pagan
el costo de la supervisión,
Porcentaje de cooperadores con lo cual evitan interactuar
sobre el total de la población (c) con los desertores. en esta
0 0.5 0.75 1.0 región los cooperadores
obtienen recompensas
mayores que los desertores
y, por lo tanto, aumenta el
porcentaje de cooperadores.
desde cualquier punto del
que se parta, a largo plazo
la población se estabiliza en
75% de cooperadores.

operadores sobre el total de la población supera 75%, los desertores obtienen de repente acceso a sus
víctimas. La regla evolutiva es, de nuevo, que recompensas relativas superiores dan como resultado
un crecimiento en el porcentaje de la población. Esta regla indica que, en este ejemplo, la población
se estabiliza en 75% de cooperadores.
Ahora, es obvio que no hay nada mágico en esta cifra de 75%. Si el costo de la supervisión hubiera
sido menor que 1, por ejemplo, el porcentaje de cooperadores en la población hubiera sido mayor.

EJERCICIO 7.4
¿Cuál será el porcentaje de cooperadores sobre el total la población en equilibrio si el costo de
la supervisión es 0.5?

El aumento de la recompensa cuando los cooperadores forman pares entre sí incrementa el por-
centaje de cooperadores en la población en equilibrio. Lo importante de este ejemplo es que cuando
hay costos de supervisión, existen presiones que empujan a la población hacia alguna combinación
estable de cooperadores y desertores. Como en el ejemplo de los halcones y las palomas, una vez
que la población llega a esta mezcla los miembros de ambos grupos tienen la misma recompensa
promedio y por lo tanto la misma posibilidad de sobrevivir. En otras palabras, hay un nicho social
para ambos grupos. Este resultado contrasta con la visión de un mundo materialista tremendamente
competitivo en el que sólo los oportunistas pueden sobrevivir.
El supuesto central detrás de la afirmación de que ciertas motivaciones o preferencias no egoís-
tas pueden ayudar a resolver problemas del compromiso es que la presencia de dichas motivaciones
puede, de alguna manera, ser descubierta por los demás. Desde la publicación del libro de Charles
Darwin The Expression of Emotions in Man and Animals en 1872 se ha estudiado mucho acerca de las
manifestaciones observables de los estados motivacionales. Por ejemplo, los psicólogos han confir-
mado la afirmación de Darwin de que ciertas expresiones faciales son características de determinadas
emociones. Estas expresiones, que son el resultado de complejas combinaciones de movimientos de

07_CHAPTER 7.indd 225 3/6/09 7:56:19 PM


226 Capítulo 7 expliCaCión de los gustos

Figura 7.8
La expresión de dolor,
tristeza o preocupación
emociones específicas
provocan expresiones
faciales características.
debido a que éstas son
en extremo difíciles de
expresar mediante la
manipulación consciente
de los músculos faciales
relevantes, las expresiones
sirven como indicadores
confiables de los estados de
motivación subyacentes.

los músculos faciales, son muy difíciles de producir a voluntad, pero se producen de manera espontá-
nea cuando se experimenta la emoción correspondiente.
Considere, por ejemplo, la expresión esquemática de la figura 7.8. La configuración distintiva
de las cejas —elevadas en el centro e inclinadas hacia los lados— se produce debido a una precisa
combinación de los músculos piramidales (localizados cerca del puente de la nariz) y de los músculos
corrugadores (localizados cerca del centro de las cejas). Sólo 15% de las personas sujetas a expe-
rimentación son capaces de producir esta expresión a voluntad. En cambio, casi todos los sujetos
muestran esta expresión de manera espontánea cuando sienten aflicción, tristeza o preocupación.
Los psicólogos han encontrado también que la postura, otros elementos del lenguaje corporal,
el tono y el timbre de la voz, la velocidad en la respiración e incluso la cadencia al hablar están relacio-
nados de manera sistemática con los estados de motivación subyacentes. Como en la mayoría de las
personas los vínculos relevantes quedan fuera del control consciente, es difícil ocultar ante los demás
la experiencia de ciertas emociones y es igualmente difícil fingir las expresiones características de es-
tas emociones cuando en realidad no se sienten. Debido a lo anterior es posible emplear estas señales
para estimar la apariencia emocional de los demás, lo que a su vez ayuda a formar juicios acerca de
sus preferencias.6 Además de las expresiones faciales y de otros síntomas físicos de las emociones, la
reputación y otras diversas señales sirven para predecir el comportamiento potencial de los socios.7

UN SENCILLO EXPERIMENTO DE REFLEXIÓN


Quizás el siguiente sencillo experimento de reflexión le ayude a decidir si usted cree poder hacer
juicios confiables acerca del carácter de otras personas.
Imagine que acaba de llegar a casa después de un concierto muy concurrido y descubre que ha
perdido 1 000 dólares en efectivo. El dinero estaba en el bolsillo de su abrigo dentro de un sobre con
su nombre. ¿Conoce a alguien, que no sea de su familia, de quien usted esté seguro que le devolverá
el sobre si lo encontrara?
Para facilitar el análisis, se supondrá que usted no se encuentra en la nada envidiable situación
de tener que responder no. Piense un momento en la persona de la que está seguro le devolvería

6
El término “preferencias” puede no expresar por completo la esencia de lo que se trata de evaluar en los socios potenciales. “Carác-
ter” o “sentimientos morales” pueden ser más adecuados.
7
Para un análisis del papel de la reputación y de otros factores, vea el capítulo 4 de Passions Within Reason del mismo autor de la
presente obra.

07_CHAPTER 7.indd 226 3/6/09 7:56:19 PM


un senCillo experiMento de reflexión 227

el dinero; llámela “Virtud”. Trate de explicar por qué confía en ella. Observe que se trata de una
situación en la que, si esta persona se quedara con el dinero, usted no lo sabría. Con base en sus otras
experiencias con esta persona, lo más que usted puede saber es que no lo ha engañado en ninguna
situación semejante en el pasado. Aun cuando, por ejemplo, en el pasado le haya devuelto algún di-
nero que usted haya perdido, eso no probaría que no lo haya engañado en otra ocasión. (Después de
todo, si en una situación similar en el pasado esta persona lo ha engañado, usted no puede saberlo.)
De cualquier manera, es casi seguro que usted no tendrá ninguna base lógica para inferir que Virtud
no lo engañará ahora. Si usted es como la mayoría de los participantes en este experimento de re-
flexión, pensará que conoce los motivos de Virtud: está seguro de que le devolvería el dinero porque
sabe que ella se sentiría muy mal si no lo hiciera.
Para que las preferencias sirvan como una estrategia de compromiso, no es necesario predecir
con certeza las preferencias de otra persona. Así como un pronóstico del tiempo que indique 20% de
probabilidad de lluvia puede ser inestimable para alguien que tenga que planear actividades al aire
libre, así los juicios probabilísticos acerca de rasgos de carácter pueden serles útiles a las personas
que tengan que elegir alguien en quien confiar. Obviamente, sería ideal estar seguro en todas las
circunstancias. Pero suele ser suficiente acertar sólo en algunos casos. Y la mayoría de la gente cree
con firmeza que puede juzgar, con exactitud razonable, el carácter de las personas que conoce bien.
Si usted cree lo mismo, está en situación de ver con claridad por qué la persecución desenfrenada del
interés personal es con frecuencia contraproducente.
A continuación se dan dos ejemplos más de problemas de compromiso y de cómo las preferen-
cias no egoístas pueden ayudar a resolverlos:
n El problema de la disuasión. Suponga que Jones tiene un portafolio de piel que Smith codicia.
Si Smith lo roba, Jones tendrá que decidir si denunciarlo. Si lo hace tendrá que ir al juzgado,
recuperará su portafolio y Smith pasará 60 días en prisión; pero un día en el juzgado le costará
600 dólares en pérdidas. Como esta cantidad es mayor que el precio del portafolio, es claro que
Jones no tendrá interés material para hacer la denuncia. (Para eliminar una complicación evi-
dente, suponga que Jones piensa mudarse a una ciudad distante, de manera que no hay necesi-
dad de adoptar medidas más estrictas con el fin de evitar robos futuros.) Así, si Smith sabe que
Jones es una persona puramente racional, preocupada nada más por el interés personal, podrá
robar con impunidad el portafolio. Jones puede amagar con denunciarlo, pero su amenaza no
tendrá consecuencias.
Ahora suponga que Jones no es un egoísta y que si Smith le robara el portafolio estará muy
molesto y no le importará perder un día de ganancias, incluso una semana, con tal de que se
haga justicia. Si Smith sabe esto no robará el portafolio. Si la gente espera que respondamos de
manera no egoísta ante el robo de alguna de nuestras pertenencias, rara vez nos veremos en
esa situación, no les convendrá robarnos. Estar dispuesto a responder de manera no egoísta da
mejores resultados aquí que dejarse llevar sólo por el interés material propio.
n El problema de la negociación. En este ejemplo, Smith y Jones se encuentran de nuevo ante la
oportunidad de una empresa colectiva rentable. Existe un trabajo que sólo ellos pueden hacer y
que les proporcionará 1 000 dólares. Suponga que Jones no tiene ninguna necesidad urgente de
dinero extra, pero que Smith tiene cuentas importantes por pagar. Un principio fundamental
de la teoría de la negociación es que la parte que menos necesita la transacción se encuentra en
la posición más fuerte. Sus diferentes circunstancias dan la ventaja a Jones. Como Jones necesita
menos las ganancias, puede amenazar, de manera creíble, con abandonar la negociación a me-
nos que obtenga la mayor parte de las ganancias, por ejemplo, 900 dólares. A Smith le convendrá
capitular antes que ver fracasar la transacción.
Pero suponga que Jones sabe que a Smith no sólo le preocupa cuánto dinero va a recibir en
términos absolutos, sino cómo se dividirá el total entre los dos. En concreto, suponga que Jones
sabe que Smith sigue una norma de justicia, de acuerdo con la cual el total deberá dividirse en
forma equitativa. Si el compromiso de Smith con esta norma es suficientemente fuerte, rehu-
sará la oferta unilateral de Jones, aun cuando lo que más le convenga, en términos materiales,
sea aceptarla. Lo irónico es que si Jones sabe esto, no se atreverá a hacerle a Smith una oferta
unilateral.

07_CHAPTER 7.indd 227 3/6/09 7:56:20 PM


228 Capítulo 7 expliCaCión de los gustos

LOS GUSTOS NO SÓLO PUEDEN,


SINO QUE DEBEN SER DIFERENTES
Los ejemplos que se han examinado en este capítulo indican que las fuerzas económicas pueden
crear nichos ambientales estables no sólo para los egoístas sino también para los altruistas. Se vio que
las poblaciones que constan nada más de una de estas clases siempre pueden ser invadidas por la otra.
Así, los gustos de la gente no sólo pueden, sino que deben ser diferentes.
Esta afirmación se contrapone con el mensaje de un célebre artículo de los economistas George
Stigler y Gary Becker de la Universidad de Chicago, quienes en ese momento criticaron fuertemen-
te las explicaciones del comportamiento basadas en el supuesto de gustos diferentes.8 Decir que la
gente se comporta de manera diferente nada más porque tiene distintos gustos, argumentaron, es
una renuncia intelectual, un abandono de los eruditos de llegar al fondo de las cosas. La mejor expli-
cación de las discrepancias en el comportamiento, desde su punto de vista, es la que supone que la
gente tiene los mismos gustos pero diferentes ingresos y enfrenta distintos precios.
Observe que Stigler y Becker planteaban la crítica clásica a la versión del objetivo presente en la
teoría de la elección racional; es decir, que da una explicación inmediata a cualquier comportamien-
to. Se puede estar por completo de acuerdo con esta crítica; sin embargo, esto no impide aceptar el
hecho de que existen gustos diferentes —en realidad deben existir— y que suelen implicar diferen-
cias importantes de comportamiento. La ventaja del modelo del compromiso es que su visión de
los gustos, como medios y no como fines, ayuda a constreñir la naturaleza abierta del estándar del
objetivo presente. La visión funcional de las preferencias en el modelo del compromiso indica que
el repertorio de gustos se extiende más allá de los simples gustos egoístas que supone el modelo del
interés personal, pero sólo una vez que se demuestre que tener un determinado gusto sea ventajoso
(o por lo menos no fatalmente desventajoso) en sentido material.

¿Por qué vota la gente en las elecciones presidenciales?


La versión del interés personal del modelo de elección racional predice que la gente no votará en
las elecciones presidenciales. La razón es, en pocas palabras, que mientras votar implica costos, no
hay casi ninguna posibilidad
EL NATURALISTA de que un solo voto incline
ECONÓMICO la balanza en una elección, ni
siquiera en una tan cerrada
7.1 como la del año 2000 en Esta-
dos Unidos.
Considere una persona a
la que se le ha dicho que es de-
ber de los ciudadanos votar en
las elecciones presidenciales.
Si este mensaje ha penetrado
a un nivel muy profundo, su
efecto altera las preferencias
de la persona. Votar se ha con-
vertido en un fin en sí mismo,
Ya que las elecciones presidenciales nunca se deciden por un solo voto, algo que le proporciona satis-
¿qué hace que las personas incurran en costos para ir a votar? facción.9

8
George Stigler y Gary Becker, “De Gustibus Non Est Disputandum”, en American Economic Review, 67, septiembre de 1977. Para
ser justo, debo agregar que Gary Becker ha adoptado una visión mucho más amplia acerca de la gama de preferencias aceptable.
9
Aquí también parece ser más descriptivo decir que las personas votan no porque les produzca placer, sino porque piensan que es
lo correcto. Sin embargo, desde el punto de vista analítico, ambas descripciones pueden representarse diciendo que el acto de votar
incrementa la utilidad.

07_CHAPTER 7.indd 228 3/6/09 7:56:20 PM


los gustos no sólo PUeDen, sino que Deben ser diferentes 229

APLICACIÓN: PREDICCIÓN DE VARIACIONES


EN LA CANTIDAD DE VOTANTES
Si se supone que una persona considera que votar es una obligación, su decisión de votar o no puede
ser analizada de una manera muy parecida a como se examinan otras elecciones económicas. Para
ejemplificar esto, considere un consumidor cuya utilidad está dada por la siguiente función:

U = 2M + 100 V, (7.8)

donde M es el valor en dólares de su consumo anual del bien compuesto y V vale 1 si vota y 0 si
no lo hace. Suponga, además, que este consumidor financia su consumo trabajando en un empleo
en el que gana 50 dólares/hora, por tantas horas como desee trabajar. Y suponga, por último, que
para ir a votar necesita emplear 30 minutos en el recorrido de ida y vuelta al lugar de la votación y,
además, esperar en una fila. Si esta persona considera el tiempo de recorrido y de espera en la fila tan
desagradable como trabajar una cantidad equivalente de tiempo, ¿cuánto tiempo esperará en la fila
para que decida no votar?
Suponga que t representa el tiempo de espera, medido en horas, en la fila para votar. Entonces,
la cantidad total de tiempo requerida para votar, incluyendo el que se emplea en el recorrido, es
igual a (t + 0.5) horas. Dado que, en su empleo, esta persona puede ganar 50 dólares/hora, el costo
de oportunidad de su voto es entonces $(50t + 25). Y como obtiene 2 unidades de utilidad por cada
dólar de consumo del bien compuesto (ver ecuación 7.8), el costo de oportunidad de votar, en términos
de utilidad, es igual a 100t + 50. Por el lado del beneficio, esta persona puede ganar 100 unidades de
utilidad por votar. El valor máximo aceptable para el tiempo de espera en la fila es el valor de t que
equipare los costos y los beneficios de votar en términos de utilidad. Este valor de t se encuentra
resolviendo la ecuación

100t + 50 = 100, (7.9)

de donde se obtiene t = ½, es decir 30 minutos. De manera que el modelo predice que si el tiempo de
espera en la fila para votar es menor que 30 minutos, la persona votará; si es mayor, no lo hará, y si es
igual a 30 minutos le será indiferente votar o no. El mismo modelo muestra, también, cómo una tor-
menta hará que aumente el tiempo de recorrido y ocasionará que disminuya la cantidad de votantes.

APLICACIÓN: PREOCUPACIONES POR LA JUSTICIA


Como un ejemplo más de cómo cambian las predicciones cuando se toman en cuenta los motivos
no egoístas, considere el siguiente ejemplo expuesto por el economista alemán Werner Guth. Él y
sus colegas investigaron cómo se comporta la gente en un sencillo juego diseñado para probar la
presencia de las preocupaciones por la justicia.
El juego se llama juego del ultimátum y participan dos personas, un asignador y un receptor. El
juego comienza dando al asignador una cantidad fija de dinero, por ejemplo, 20 dólares. Entonces el
asignador debe hacer una propuesta de cómo se repartirá el dinero entre él y el receptor; por ejem-
plo, puede proponer 10 dólares para él y 10 para el receptor. Lo que tiene que hacer este último es
aceptar o rechazar la propuesta. Si acepta, cada uno recibe las cantidades propuestas; si la rechaza, no
reciben nada ninguno de los dos. Los 20 dólares son simplemente devueltos a los experimentadores.
Los jugadores no se conocen y pueden jugar sólo una vez.
¿Qué es lo que el modelo del interés personal predice que ocurrirá aquí? Para responder a la
pregunta se empieza por suponer que lo único que le preocupa a cada jugador es su nivel de ri-
queza final y no cuánto obtenga el otro. Suponga que el asignador propone quedarse con PA = 15
dólares y dar al receptor los restantes $20 – PA = $5, y que el receptor acepta esta propuesta. Si antes
del experimento MA y MR eran sus niveles de riqueza, sus niveles de riqueza finales serán entonces
MA + 15 dólares y MR + 5 dólares.

07_CHAPTER 7.indd 229 3/6/09 7:56:21 PM


230 Capítulo 7 expliCaCión de los gustos

Si, por otro lado, el receptor rechaza la propuesta del asignador, sus niveles finales de riqueza
serán MA y MR. Sabiendo esto el asignador puede concluir que el receptor obtendrá un nivel de ri-
queza mayor aceptando la propuesta que rechazándola, siempre que PA sea menor que 20 dólares.
Si el dinero no puede dividirse en intervalos menores que 1 centavo, el modelo del interés personal
predice de manera inequívoca que el asignador propondrá quedarse con 19.99 y dar al receptor el
centavo restante. El receptor puede no estar muy contento con esta oferta unilateral, pero el modelo
del interés personal dice que de cualquier manera aceptará la oferta, porque MR + $0.01 > MR. De
acuerdo con la lógica del modelo del interés personal, el receptor piensa que, aunque una ganancia
de 1 centavo no es mucho, es mejor que nada, que es lo que obtendría si rechazara la oferta. Dado
que el juego se juega sólo una vez, no tiene sentido rechazar la oferta con la esperanza de que esto
aliente al asignador para hacer una oferta más favorable la próxima vez.
¿Cuál será el resultado final si se sabe que al receptor no sólo le interesa el nivel final de riqueza
sino también la justicia? Quizá resulte natural decir que la división más justa de la ganancia en un
juego del ultimátum es mitad y mitad. Sea S la cantidad total de dinero por repartirse y sea P/S = (20
– PA)/S el porcentaje de esta ganancia que recibirá el receptor si acepta la propuesta. Una manera ade-
cuada de expresar la preocupación del receptor por la justicia, es decir, que su satisfacción disminuye
a medida que el cociente P/S se aleja —en cualquier dirección— de 0.5. De esta manera, el mapa de
indiferencia del receptor definido sobre MR y P/S será más o menos como se muestra en la figura 7.9.
Las curvas de indiferencia mostradas expresan el supuesto adicional de que una división unilateral es
más objetable si favorece a la otra persona, lo cual es otra manera de decir que la RMS aumenta de
manera más acentuada al moverse hacia la izquierda de P/S = 0.5 que al moverse hacia la derecha.10

FIGURA 7.9
Alternativa entre
riqueza absoluta y
ganancia relativa
en muchas situaciones, MR (riqueza del receptor)
la división más equitativa Satisfacción creciente
de una ganancia es que
cada parte reciba la misma
proporción. Cuando una I3
persona valora la justicia,
las curvas de indiferencia I2
entre riqueza final y
proporción de la ganancia
tienen forma de u, esto
significa que las personas I1
necesitan ser compensadas
por aceptar divisiones que
se desvían de la igualdad. P/S (proporción de la ganancia
obtenida por el receptor)
0 0.5 1.0

Ahora se evaluará la propuesta unilateral pronosticada por el modelo del interés personal, donde
PA = 19.99 dólares y P = 0.01. Si el receptor acepta esta propuesta, terminará en el punto (0.01/20,
MR + 0.01), indicado como C en la figura 7.10. Por otro lado, si rechaza la propuesta, tendrá casi el
mismo nivel de riqueza, MR. Si rechazar la oferta puede considerarse que da como resultado un valor
P/S de 0.5 (ya que ninguna de las partes gana a costa del otro), el receptor terminará en el punto D
de la figura 7.10. Y como D se encuentra en una curva de indiferencia más alta que C, lo mejor que

10
Lo que se dice en este ejemplo sería en esencia lo mismo si en la figura 7.9 la pendiente de las curvas de indiferencia del receptor
fuera negativa en todos los puntos.

07_CHAPTER 7.indd 230 3/6/09 7:56:22 PM


los gustos no sólo PUeDen, sino que Deben ser diferentes 231

FIGURA 7.10
Ganancia de rechazar
una oferta unilateral
aceptar una oferta
M unilateral coloca al
receptor en el punto C.
rechazando la oferta, el
receptor termina en el
Satisfacción creciente punto D donde, aun cuando
I2 su riqueza final es un poco
MR + 0.01 inferior, está en una curva
C D de indiferencia superior.
I1

MR

P/S
0.01/20 0.5 1.0

puede hacer es rechazar la propuesta. (Si la acepta, el escaso aumento de su riqueza no será suficiente
para compensar la pérdida de utilidad de una negociación unilateral.) Más importante, si el asignador
sabe que el receptor tiene tales preferencias, nunca hará una oferta unilateral.
En el ejemplo anterior, al receptor le cuesta sólo un centavo de dólar castigar al asignador por
hacer una oferta unilateral. ¿Está dispuesta la gente a rechazar ofertas unilaterales cuando las ganan-
cias son sustancialmente mayores? Werner Guth y colaboradores realizaron estos experimentos con
cantidades hasta de 50 dólares. Encontraron que, incluso en este nivel, es común ver que el receptor
rechace si el asignador ofrece menos de 20% del total.
Por supuesto, existe un punto en el que las preocupaciones por la justicia ceden ante aquellas por
la ganancia absoluta misma. Sería en verdad sorprendente que el receptor rechazara una propuesta
en la que obtuviera 10% de, por ejemplo un millón de dólares. En este caso es seguro que la mayoría
de las personas hallarían el par (0.1, MR + $ 100 000) más atractivo que (0.5, MR).

La función de utilidad de Hatfield está dada por UH = MH/√MM, donde MH es el nivel de rique- EJEMPLO 7.2
za de Hatfield y MM es el de McCoy. La función de utilidad de este último tiene una forma
similar: UH = MM/√MH. Suponga que al inicio MH = MM = 4 y que existe una tarea, ni agradable
ni desagradable, que Hatfield y McCoy pueden realizar juntos y que generará 2 unidades de
riqueza para ser dividida entre los dos. Ninguno de ellos puede realizar esta tarea solo ni con
ninguna otra persona. ¿Cuál es el menor pago que aceptara Hatfield por esta tarea? (A Mc-
Coy se le pagará la diferencia entre 2 y la cantidad pagada a Hatfield.) ¿Es factible esta tarea?
En este ejemplo las funciones de utilidad son de tal naturaleza que cada persona se siente mejor
cuando su riqueza aumenta y se siente peor cuando la de otros aumenta. El efecto de esta tarea es
aumentar la riqueza de las dos personas. La pregunta, entonces, es si el efecto positivo de obtener
más ingreso por realizar la tarea contrarresta el efecto negativo de que la otra persona también tenga
más ingreso. El nivel inicial de utilidad de Hatfield es 4/√4 = 2. Suponga que Hatfield realiza la
tarea con McCoy y obtiene un pago de P, dejando, entonces, 2 – P para McCoy. El nivel de utilidad
de Hatfield sería entonces UH = (4 + P)/√(4 + 2 – P) = (4 + P)/√(6 – P). Para Hatfield el menor pago
aceptable de utilidad será aquel que mantenga su utilidad en el mismo nivel como si no hubiera
participado en la realización de la tarea: (4 + P)/√(6 – P) = 2. Reordenando los términos, se tiene

07_CHAPTER 7.indd 231 3/6/09 7:56:23 PM


232 Capítulo 7 expliCaCión de los gustos

P2 + 12P – 8 = 0, de donde P = 0.63.11 Como el problema es simétrico, éste es también el pago míni-
mo que aceptará McCoy. Y como la ganancia total por realizar el proyecto (2) es más que suficiente
para que cada persona obtenga 0.63, Hatfield y McCoy realizarán la tarea. Por ejemplo, si cada uno
obtiene un pago de 1, cada uno obtendrá un nivel de utilidad de (4 + 1)/√(4 + 1) = √5, que es mayor
que al nivel de utilidad inicial, que es 2.

LA IMPORTANCIA DE LOS GUSTOS


El modelo del interés personal supone ciertos gustos y restricciones y después calcula las acciones
que sirven mejor a dichos gustos. Este modelo es muy empleado por los economistas y otros cientí-
ficos de las ciencias sociales, por los especialistas en teoría de juegos, en estrategias militares, por los
filósofos y por muchos otros. Sus resultados influyen sobre decisiones que nos afectan a todos. En su
forma estándar supone gustos claramente egoístas, es decir, por los bienes de consumo presente y
futuro de varios tipos, ocio, etc. La envidia, la culpa, la cólera, el honor, la simpatía, el amor y otros
sentimientos semejantes no suelen desempeñar ningún papel.
En cambio, los ejemplos de este capítulo subrayan el papel de estas emociones sobre el compor-
tamiento. El racionalista habla de gustos, no de emociones, pero para propósitos analíticos, los dos
puedan ejercer papeles paralelos. Así, una persona motivada por evitar la emoción de la culpa puede
describirse de manera equivalente como alguien con “gusto” por el comportamiento honesto.
Los gustos tienen consecuencias importantes para la acción. La inclusión de gustos que ayudan
a resolver problemas de compromiso altera de manera sustancial las predicciones de los modelos del
interés personal. Se vio que vale la pena que las personas se preocupen por la justicia, porque esto las
hace mejores negociadoras. Sin tomar en cuenta la preocupación por la justicia no puede esperarse
poder predecir los precios que se cobran en las tiendas, los salarios que demandan los trabajadores, el
tiempo que los ejecutivos soportarán una huelga, los impuestos que recaudarán los gobernantes, lo
rápido que aumentará el presupuesto militar o si el dirigente de un sindicato será reelecto.
La presencia de la conciencia modifica también las predicciones de los modelos del interés per-
sonal. Éstos predicen con claridad que cuando las interacciones entre las personas no se repiten, la
gente engaña cuando sabe que puede salirse con la suya. Sin embargo, las evidencias muestran de
manera constante que hay mucha gente que no engaña en estas circunstancias. El modelo del interés
personal indica también que los propietarios de negocios pequeños no contribuyen con los esfuerzos
por lograr acuerdos que realicen las asociaciones comerciales. Como el voto de una sola persona, su
contribución parece demasiado pequeña en comparación con el total para que pueda hacer alguna
diferencia. Sin embargo, muchos negocios pequeños pagan cuotas a las asociaciones comerciales y
mucha gente vota. También existen instituciones de caridad en una escala mucho mayor que la que
predicen los modelos del interés personal.
Las emociones que conducen a estos comportamientos no encierran nada místico. Al contrario,
son parte obvia del maquillaje psicológico de la mayoría de las personas. Se ha visto por qué puede
ser ventajoso, incluso en términos puramente materiales, tener preocupaciones que motiven un
comportamiento desinteresado.

¿ES LA GANANCIA MATERIAL UN MOTIVO


“ADECUADO” PARA LA MORALIDAD?
Alguien puede objetar que la perspectiva de una ganancia material es de alguna manera un motivo
inadecuado para adoptar valores morales. Pero esta objeción malinterpreta el mensaje fundamental
de este capítulo: que los motivos no egoístas confieren ventajas materiales únicamente si la satisfac-
ción que la gente obtiene por hacer lo correcto es intrínseca al comportamiento mismo. De no ser así,

11
Recuerde que la solución de una ecuación de la forma ax2 + bx + c = 0 está dada por

−b ± b 2 − 4ac
x= .
2a

07_CHAPTER 7.indd 232 3/6/09 7:56:24 PM


probleMas 233

la persona carecerá de la motivación necesaria para hacer elecciones sacrificándose a sí misma cuan-
do nadie la ve y una vez que los demás detecten ese aspecto de su carácter, las ventajas materiales,
en realidad, no se darán. Debido al carácter mismo del problema del compromiso, los sentimientos
morales no pueden conducir a ventajas materiales a menos que sean sinceros.

RESUMEN
• Los dilemas del prisionero y otras formas de problemas de com- personas oportunistas supuestas por los modelos convenciona-
promiso abundan en las negociaciones económicas. Ser conoci- les del interés personal.
do por no tener preferencias de estricto interés personal puede • Los dos puntos principales de este capítulo son 1) el modelo del
ser muy útil para resolver estos problemas. interés personal, que supone que todo mundo se comporta de
• Para que tales preferencias sean ventajosas, otros tienen que ser manera oportunista, está destinado a cometer errores impor-
capaces de darse cuenta de que uno las tiene. Si las preferencias tantes al predecir la conducta real, y 2) la gente preocupada por
pudieran ser observadas sin ningún costo ni incertidumbre, en los intereses de los demás no necesita sufrir por eso, ni siquiera
este mundo sólo habría personas cooperativas. Pero debido a en términos puramente materiales. Debido a que otros pueden
que los costos y la incertidumbre son partes inherentes del pro- reconocer la clase de personas que son, se les presentarán opor-
ceso, siempre habrá un nicho social para al menos algunas de las tunidades que no tendrán los ventajistas.

pREgUNtaS dE REpaSo
1. Con sus propias palabras resuma las dificultades principales 4. En el modelo del compromiso, ¿qué papel desempeña la ob-
de los estándares de racionalidad del objetivo presente y del servancia de las preferencias?
interés personal. 5. Explique por qué la carrera armamentista entre Estados Uni-
2. Explique el papel del análisis racional en el modelo de los psi- dos y la ex Unión Soviética tenía la misma estructura formal
cólogos sobre la motivación humana. que un dilema del prisionero.
3. Trate de recordar por lo menos dos problemas de compromi-
so que haya encontrado personalmente en el último año.

pRoblEMaS
1. Una población consta de dos tipos, “amigables” y “agresivos”. Cada individuo interactúa con un
miembro de la población elegido al azar. Cuando interactúan dos amigables, cada uno gana 3 uni-
dades. Cuando interactúan dos agresivos, cada uno gana 0. Cuando interactúan un amigable y un
agresivo, el primero obtiene una recompensa de una unidad y el último obtiene una recompensa de
5 unidades. La tasa de crecimiento de cada tipo es proporcional a su recompensa promedio. ¿Cuál
será el porcentaje de cada tipo en la población en equilibrio?
2. Considere una población en la que hay dos tipos de personas, los C y los D. Las interacciones entre
las diversas combinaciones de estos dos tipos producen las recompensas siguientes:
C-C: 6 cada uno
C-D: 8 para D, 0 para C
D-D: 4 cada uno
A un precio de una unidad se pueden tener lentes de contacto invisibles que permiten a quien los use
identificar con certeza cada tipo de persona. Sin los lentes los dos tipos son indistinguibles.
a) ¿En el equilibrio cuáles serán las proporciones poblacionales de las dos clases?
b) ¿Cuál será su respuesta si la recompensa en la interacción D-D es 5.5?
3. La función de utilidad de Alfonso está dada por
UA = MAMG,
donde MA y MG son los niveles de riqueza de Alfonso y de Gastón, respectivamente. Si el nivel inicial
de riqueza de Alfonso es 100 y el de Gastón es sólo 20, ¿cuánto de su riqueza dará Alfonso a Gastón?

07_CHAPTER 7.indd 233 3/6/09 7:56:24 PM


234 Capítulo 7 expliCaCión de los gustos

4. La función de utilidad de Abdulá está dada por

M A2
UA = ,
MB
donde MA es el nivel de riqueza de Abdulá y MB es el nivel de riqueza de Benjamín. La función de
utilidad de Benjamín está dada por

M B2
UB = .
MA
Suponga que al principio MA = MB = 10 y existe un proyecto conjunto que pueden llevar a cabo
Abdulá y Benjamín y que les generará 10 unidades adicionales de riqueza para dividir entre los dos.
El proyecto no es ni agradable ni desagradable. ¿Cuál es la recompensa mínima que habrá que darle
a Abdulá para que realice el proyecto? ¿Cuál es la recompensa mínima que habrá que darle a Benja-
mín? ¿Realizarán el proyecto?
5. Ahora suponga que la función de utilidad de Benjamín está dada por UB = M B2 . Abdulá firma un
contrato diciendo que donará 20 a una causa a la que él se opone en caso de que reciba menos de
90% de cualquier cantidad que gane con Benjamín. ¿Aceptará Benjamín una oferta, del tipo “tómalo
o déjalo”, de una unidad de Abdulá?
6. Describa las ventajas y las desventajas de elegir un líder político conocido por estar a favor de vio-
lentas represalias militares contra las agresiones extranjeras, aun cuando dichas represalias dañen
fuertemente los propios intereses nacionales.
7. La utilidad de Harold está dada por U = 3M + 60V, donde M es el valor en dólares de su consumo
anual del bien compuesto y V tiene valor 1 si vota y 0 si no vota. Harold financia su consumo tra-
bajando en un empleo en el que gana 30 dólares/h, tantas horas como desee trabajar. Para votar
necesita 20 minutos de recorrido de ida y vuelta al lugar de la votación, donde antes de depositar su
voto tiene que hacer una fila. Si considera que el tiempo invertido en el transporte y en la espera en la
fila no es ni más ni menos desagradable que trabajar en su empleo la cantidad equivalente de tiempo,
¿qué tan larga debe ser la espera en la fila para depositar su voto para qué Harold decida no votar?

RESpUEStaS a loS EjERcicioS dEl capítUlo


7.1 Sea h el porcentaje de halcones en la población, de manera que 1 – h representa el porcentaje de pa-
lomas. Como estos dos tipos interactúan de manera aleatoria con otros miembros de la población, la
recompensa esperada de las palomas está dada por

PD = 3(1 – h) + 2(h) = 3 – h.

La correspondiente recompensa esperada para los halcones es

PH = 6(1 – h) + 1(h) = 6 – 5h.

La población está en equilibrio cuando las recompensas esperadas de los dos tipos son iguales. Si h*
representa la proporción de halcones en el equilibrio, se tiene

3 – h* = 6 – 5h*,

de donde se obtiene h* = ¾. El porcentaje de equilibrio para las palomas es 1 – h* = ¼.


7.2 PC = 0.9(4) + 0.1(0) = 3.6.

07_CHAPTER 7.indd 234 3/6/09 7:56:26 PM


respuestas a los ejerCiCios del Capítulo 235

7.3 Si todos los cooperadores pagan el costo de supervisión, cada uno obtendrá una recompensa de
4 – 1.5 = 2.5. Si ninguno paga el costo de supervisión, la recompensa esperada será
PC = 0.6(4) + 0.4(0) = 2.4,
que es menor que 2.5. De manera que deben pagar el costo de supervisión.
7.4 La recompensa neta del costo de supervisión es ahora 4 – 0.5 = 3.5. Si los cooperadores no pagan
el costo de supervisión, su recompensa esperada estará dada de nuevo por PC = 4c. Para hallar el
umbral de rentabilidad de c, se despeja c' de 4c' = 3.5, con lo que se obtiene c' = 7/8.
Cuando c < 7/8 , los cooperadores obtienen una recompensa mayor si pagan el costo. Cuando
c > 7/8 , obtienen una recompensa mayor si simplemente asumen su riesgo. Cuando c < 7/8 , los de-
sertores se verán obligados a interactuar unos con otros, lo que les da una recompensa de 2. Sin
embargo, una vez que c > 7/8 , los cooperadores dejan de pagar el costo y la recompensa esperada de
los desertores se convierte en
PD = c6 + (1 – c)2 = 2 + 4c.
Las funciones de recompensa esperada para cooperadores y desertores se muestran en el diagrama
siguiente:

6 Recompensa promedio
5.5 para desertores

Recompensa promedio
4 para cooperadores
3.5

c
7
0 8
1.0

Observe que cuando c < 7/8 , la recompensa promedio para los cooperadores es mayor que para los
desertores, mientras que cuando c > 7/8 , la recompensa esperada de los desertores es superior a la
de los cooperadores. El resultado es que si se parte de c > 7/8 , el porcentaje de cooperadores en la po-
blación se reducirá a 7/8 debido a que la tasa de crecimiento de los desertores es mayor que la de los
cooperadores. Si se parte de c < 7/8 , el porcentaje de cooperadores en la población aumentará hasta
7
/8 .

07_CHAPTER 7.indd 235 3/6/09 7:56:28 PM


07_CHAPTER 7.indd 236 3/6/09 7:56:28 PM
Capítulo

8
LIMITACIONES COGNITIVAS
Y COMPORTAMIENTO
DEL CONSUMIDOR

L
a Universidad de Cornell tiene dos tipos de canchas de tenis, unas en el exterior y otras
cubiertas. Las canchas del exterior están disponibles para los socios por una cuota fija por
temporada, y no es necesario pagar ninguna cantidad adicional por usarlas. En cambio,
para las segundas no sólo hay que pagar una cuota fija por estación, sino, además, 20 dóla-
res por hora de uso de la cancha. El costo extra de las canchas cubiertas refleja los costos de calefac-
ción, electricidad y mantenimiento de las instalaciones. Las canchas cubiertas se pueden usar desde
principios de octubre, cuando el clima en Ithaca puede ser desde soleado con temperaturas agrada-
bles hasta lluvioso con nieve. Las canchas externas permanecen abiertas, siempre que el tiempo lo
permita, hasta principios de noviembre.
Las canchas cubiertas tienen una gran demanda y la gente que desea jugar con regularidad tiene
que reservar una hora específica cada semana. Una vez hecha la reservación hay que pagar el tiempo
se juegue o no. Durante el tiempo que dura el buen clima, casi todos prefieren jugar en las canchas
externas, que se encuentran en una de las cañadas panorámicas de Ithaca.
El problema es el siguiente: usted ha reservado una cancha cubierta para jugar a las 3 de la tarde
del sábado 20 de octubre, la única hora que tiene disponible ese día. Es una cálida y soleada tarde de
octubre. ¿Dónde deberá jugar, en la cancha cubierta o en la cancha en el exterior?
Es sorprendente la reacción de muchos de mis colegas que no son economistas cuando digo que
lo razonable es jugar en una cancha en el exterior. “Pero ya pagamos por una cancha cubierta”, res-
ponden de manera invariable. Entonces pregunto: “Si ambas canchas tuvieran el mismo precio, ¿cuál
elegirían?” De inmediato responden: “Una del exterior.” Entonces les explico que las dos canchas
237

08_CHAPTER 8.indd 237 3/6/09 7:57:25 PM


238 Capítulo 8 limitaCiones Cognitivas y Comportamiento del Consumidor

tienen el mismo precio, porque la cantidad pagada por la hora va a ser 20 dólares sin importar en qué
cancha juguemos; en realidad, sin importar si jugamos o no. Los 20 dólares son un costo irrecupera-
ble* y no deben afectar nuestra decisión. Sin embargo, aun después de esta explicación, mucha gente
se siente incómoda por desperdiciar la cancha por la que ha pagado. La alternativa es desperdiciar la
oportunidad de jugar al aire libre que, todos estamos de acuerdo, ¡es todavía más valiosa! Por supues-
to, no es bueno malgastar, pero de cualquier manera algo se va a tener que desperdiciar sin importar
dónde juguemos.
Por último, la mayoría de las personas aceptan que en un día soleado de verano es más razona-
ble abandonar la cancha cubierta, aun cuando se haya pagado por ella, y jugar al aire libre. El modelo
de la elección racional dice de manera inequívoca que esto es lo que se debe hacer. Pero ésta no
parece ser la tendencia natural de la mayoría de las personas; de no tener cerca a un economista, ellas
elegirán la cancha cubierta y terminarán jugando ahí, aun en el día más espléndido.

VISIÓN PRELIMINAR DEL CAPÍTULO


En el capítulo 7 se vio por qué los motivos distintos al interés personal pueden ser importantes. Di-
chos motivos suelen llevar a la gente a comportarse de maneras consideradas irracionales de acuerdo
con el modelo del interés personal. Pero irracionales o no, comportamientos como dejar propinas
en las carreteras o devolver a su propietario una cartera que se ha encontrado se realizan sin arre-
pentimiento. Si un racionalista dijera que es imposible que un mesero en una ciudad lejana pueda
tomar represalias por no haberle dejado propina la mayoría respondería: “¿Y eso que?” No por eso
lamentaría haber dejado propinas toda la vida.
El punto principal en este capítulo es un comportamiento irracional de un tipo muy distinto: la
consecuencia de no poder ver con claridad la mejor manera de alcanzar un resultado deseado. Un
ejemplo es no poder ignorar los costos no recuperables de una cancha de tenis cubierta. A diferencia
de los comportamientos considerados en el capítulo anterior, la gente desea modificar con frecuencia
los comportamientos que se exponen aquí una vez que conocen sus consecuencias.
Además de no poder ignorar los costos no recuperables, se verá que la gente viola, de muchas
otras maneras sistemáticas, lo que predicen los modelos de elección racional. Se examinarán diver-
sos modelos basado en la conducta que suelen funcionar mucho mejor para predecir las decisiones
reales que el modelo de elección racional. Sin embargo, es importante recordar que dichos modelos
no pretenden tener ningún significado normativo; dicen, por ejemplo, que con frecuencia existe la
tendencia a ignorar los costos no recuperables, no que se deba ignorarlos.
Según el modelo de elección racional, se pueden tomar mejores decisiones si se ignoran los cos-
tos no recuperables, y la mayoría de las personas, después de reflexionarlo, coinciden fuertemente
con esto. El valor de los modelos de conducta es que llaman la atención hacia situaciones en las que
es posible que se cometan errores. Los modelos son herramientas importantes como ayuda para
evitar escollos comunes en la toma de decisiones.

RACIONALIDAD LIMITADA
El hoy desaparecido Premio Nobel Herbert Simon fue uno de los primeros que trataron de convencer
a los economistas de que los seres humanos son incapaces de comportarse como los modelos conven-
cionales de la elección racional. Simon fue uno de los pioneros en el campo de la inteligencia artificial
y llegó a esta conclusión al tratar de dar instrucciones a una computadora para “razonar” acerca de
un problema. Descubrió que cuando las personas se encuentran ante un conflicto rara vez llegan a la
solución de una manera clara y lineal. Más bien, buscan al azar aspectos e información que pueda ser
potencialmente relevante y suspenden la pesquisa una vez que su comprensión llega a cierto umbral.
Con frecuencia sus conclusiones son inconsistentes, incluso del todo incorrectas. Pero la mayoría de
las veces llegan a soluciones útiles. Como decía Simon, son “satisfactorias”, no maximizadoras.
Economistas posteriores seguidores de las ideas de Simon han desarrollado una literatura rebus-
cada sobre la toma de decisiones con información incompleta. Ahora se sabe que cuando la obten-

* En algunos países de habla hispana se utiliza el término hundido.

08_CHAPTER 8.indd 238 3/6/09 7:57:26 PM


la funCión asimétriCa de valor 239

ción de información es costosa y la capacidad del procesamiento cognitivo es limitada, no es racional


hacer elecciones por completo informadas del tipo que presentan los modelos sencillos. De manera
paradójica, ¡es irracional estar completamente informado! La literatura sobre la toma de decisiones
con información completa, lejos de poner en duda el modelo de la elección racional, ha reforzado, en
realidad, la confianza en él.
Pero hay otro aspecto del trabajo de Simon, uno que encaja menos con el modelo de la elección
racional. Esta investigación, influenciada fuertemente por los psicólogos cognitivos Daniel Kahne-
man y Amos Tversky, demuestra que aun cuando se trate de problemas transparentes y sencillos, la
gente suele violar los axiomas más fundamentales de la elección racional. Decidir si jugar tenis en
una cancha cubierta o en el exterior es un ejemplo de esto. Los aspectos relevantes de este problema
no podrían ser más simples; sin embargo, como se observó, la gente elige consistentemente de ma-
nera irracional. Kahneman y Tversky demostraron que tales ejemplos no son aislados. Debido a la
solidez de su trabajo, a Kahneman le fue otorgado el Premio Nobel de Economía en 2002, honor que
sin duda hubiera compartido Tversky de no ser por su prematuro fallecimiento en 1996.
Uno de los principios más apreciados del modelo de la elección racional es que la riqueza es
fungible (es decir, que se consume con el uso). El que sea fungible implica, entre otras cosas, que la
riqueza total, no la cantidad que se tenga en una determinada cuenta, determina lo que se compra. Sin
embargo, Kahneman y Tversky proporcionaron una clara demostración experimental de lo contra-
rio.1 Estos investigadores pidieron a un grupo de personas que imaginaran que, habiendo comprado
con antelación entradas para el teatro, a un precio de 10 dólares, al llegar a la función descubren que
han perdido las entradas. A los miembros de otro grupo se les pidió que imaginaran que al llegar al
teatro justo antes del comienzo de la función se dan cuenta de que en el camino han perdido 10 dóla-
res. Después, a las personas de ambos grupos se les preguntó si de cualquier manera hubieran asistido
a la función. De acuerdo con el modelo de la elección racional, las fuerzas que rigen esta decisión son
las mismas en ambos grupos. Perder una entrada de 10 dólares debe tener el mismo efecto que perder
10 dólares. (Recuerde que en el capítulo 3 se vio la regla de que si los gustos y las restricciones de
presupuesto son iguales, las decisiones también deben ser iguales.) Sin embargo, en diversos ensayos
la mayoría de las personas del grupo que había perdido las entradas decidía no asistir a la función,
mientras que una gran mayoría —88%— del grupo que había perdido el dinero decidía lo contrario.
Kahneman y Tversky explicaban que al parecer la gente organiza sus gastos en distintas “cuentas
mentales” para alimentos, vivienda, entretenimiento, gastos generales, etc. Las personas que habían
perdido las entradas reaccionaban cargando a sus cuentas mentales de entretenimiento los 10 dó-
lares, mientras que aquellas que habían perdido los 10 dólares lo cargaban a sus cuentas de gastos
generales. Para las personas del primer grupo, la pérdida hacía que el costo aparente de ver la función
aumentará de 10 a 20 dólares, mientras que para las del segundo grupo seguía siendo de 10 dólares.
El modelo de la elección racional establece que la resolución correcta es la del segundo grupo.
Y pensándolo bien, la mayoría de la gente coincide en que, en realidad, perder una entrada no es una
razón más válida para no ver la función que perder 10 dólares.

LA FUNCIÓN ASIMÉTRICA DEL VALOR


El modelo de la elección racional establece que la gente debe evaluar los acontecimientos, o el con-
junto de éstos, en términos de su efecto global sobre su riqueza total. Suponga que A es un acon-
tecimiento que consiste en que usted recibe de manera inesperada 100 dólares como regalo, y B
es un acontecimiento que consiste en que al regresar de sus vacaciones encuentra una factura del
municipio por 80 dólares por la reparación de una tubería de agua. De acuerdo con el modelo de la
elección racional, ambos acontecimientos deberán ser considerados por usted como algo bueno, ya
que su efecto global es un aumento de 20 dólares en su riqueza.
Sin embargo, Kahneman y Tversky encontraron que la gente parece apreciar cada aconteci-
miento por separado y darle mucha menos importancia a la ganancia que a la pérdida, tanto que
muchas personas rechazan pares de acontecimientos que podrían aumentar su riqueza global.

1
Vea Amos Tversky y Daniel Kahneman, “The Framing of Decisions and the Psychology of Choice”, en Science, 211, 1981, pp.
453-458.

08_CHAPTER 8.indd 239 3/6/09 7:57:26 PM


240 Capítulo 8 limitaCiones Cognitivas y Comportamiento del Consumidor

Por supuesto, en el modelo de la elección racional esto nunca puede ocurrir. Ante los dos acon-
tecimientos A y B antes descritos, una persona que tenga una riqueza inicial de M0 sabe con exactitud
cómo reaccionar. El efecto combinado de A (una ganancia de 100 dólares) y B (una pérdida de 80
dólares) es incrementar la riqueza a M0 + 20. Y dado que la utilidad es una función creciente de
la riqueza total, los dos acontecimientos juntos hacen que la utilidad aumente de U0 a U1 como se
muestra en la figura 8.1.

FIGURA 8.1
Utilidad de un par de Utilidad
aontecimientos que
incrementan la riqueza U(W)
total U1
de acuerdo con el modelo U0
de la elección racional,
cualquier combinación
de acontecimientos que
incremente la riqueza total
también aumenta la utilidad
total.
Riqueza
M0 M0 + 20

Kahneman y Tversky proponen que la gente no evalúa las alternativas con la función de utilidad
convencional, sino con una función de valor definida con base en las variaciones de la riqueza. Una pro-
piedad importante de esta función de valor es que es mucho más inclinada en las pérdidas que en las
ganancias. En la figura 8.2, por ejemplo, observe cómo la función le asigna un valor mucho mayor,
en términos absolutos, a la pérdida de 80 dólares que a la ganancia de 100. Observe también que la
función de valor es cóncava en las ganancias y convexa en las pérdidas. Esta propiedad es el análogo
de la utilidad marginal decreciente en el modelo tradicional e indica que el impacto de las ganancias
o las pérdidas crecientes disminuye a medida que éstas son mayores.

FIGURA 8.2
Función de valor de Valor
Kahneman-Tversky V(G)
a diferencia de la función V(100)
de utilidad tradicional, la
de valor está definida con
base en las variaciones
en la riqueza total. esta –80
Pérdidas Ganancias
función es más inclinada 100
en las pérdidas que en las
ganancias, cóncava en las
ganancias y convexa en las
pérdidas.

V(–80)
V(L)

Kahneman y Tversky hacen hincapié en que su función de valor es un dispositivo puramente


descriptivo. Tratan de resumir las regularidades en las maneras en que las personas en realidad pare-
cen hacer sus elecciones. Estos investigadores no dicen que la gente deba elegir como lo predice su
función de valor.

08_CHAPTER 8.indd 240 3/6/09 7:57:28 PM


la funCión asimétriCa de valor 241

De acuerdo con Kahneman y Tversky, por lo general la gente evalúa por separado cada elemento
de un conjunto de acontecimientos y después decide con base en la suma de cada uno de los valores.
En este ejemplo, V(100) es mucho menor, en términos absolutos que V(–80). Como la suma algebrai-
ca es menor que cero, cualquier persona que emplee este mecanismo de decisión rechazará el par de
oportunidades A y B, aun cuando su efecto neto sea incrementar la riqueza total en 20 dólares.
En realidad hay dos aspectos importantes en la función de valor de Kahneman y Tversky. Uno
es que la gente considera ganancias y pérdidas de manera asimétrica, dándole a las últimas mucho
más peso que a las primeras al momento de tomar sus decisiones. El segundo es que la gente evalúa
primero los acontecimientos y después suma los valores de cada uno. El primero de estos aspectos
no implica un comportamiento irracional, después de todo no tiene nada de inconsistente pensar
que una pérdida causa más dolor que la alegría que ocasiona una ganancia de la misma magnitud. Lo
que sí suele parecer irracional es el segundo paso, considerar cada acontecimiento por separado, en
lugar de ponderar el efecto combinado de los dos.
Ésta es en esencia una cuestión acerca de cómo entender los acontecimientos. Si alguien le hace
ver a una persona que el efecto neto de los acontecimientos A y B es un incremento de su riqueza
de 20 dólares, es probable que ella acepte su realización. Entendidos como una entidad, los aconte-
cimientos son con claridad una mejoría del statu quo. El problema es que, en las decisiones reales,
parece más natural entenderlos por separado.
Otro ejemplo ayuda a ilustrar esto. Hace poco una empresa puso a disposición de sus empleados
un nuevo plan de seguro médico que sustituía el antiguo plan que pagaba 100% de los gastos médi-
cos incluidos y la prima era de aproximadamente 500 dólares/año por familia. El nuevo plan tenía un
deducible de 200 dólares; cada año las personas tienen que pagar los primeros 200 dólares de gastos
médicos, pero una vez alcanzado este umbral, el seguro paga 100%. En el nuevo plan la prima es de
250 dólares/año, la mitad de lo que era en el anterior. Los empleados tienen la opción de permanecer
con el plan anterior o cambiarse al nuevo.
Visto desde la perspectiva del modelo de la elección racional, el nuevo plan es mejor que el ante-
rior. Los 250 dólares que se ahorran de la prima son más que suficientes para compensar el deducible
de 200 dólares. Para las familias que incurren en gastos médicos menores que 200 dls/año, también
es mejor el plan nuevo. A pesar de esto, muchos de los empleados estaban decididos a permanecer
con el plan anterior. Si una persona considera como acontecimientos separados el ahorro de 250
dólares en la prima y los 200 de gastos médicos, entonces la función asimétrica de valor predice este
comportamiento. Como se indica en la figura 8.3, los 200 dólares de pérdida pesan mucho más que
los 250 de ganancia.

FIGURA 8.3
Valor Rechazo de un mejor
V(G) plan de seguro
V(250) dado que el ahorro en la
prima (250 dls.) es mayor
que el mayor aumento
posible en los gastos
–200 no incluidos (200 dls.),
Pérdidas Ganancias el plan nuevo es mejor
250 que el anterior. pero si
las personas consideran
por separado ganancias
y pérdidas, puede que, a
pesar de esto, no estén
dispuestas a cambiarse al
plan nuevo.

V(L) V(–200)

08_CHAPTER 8.indd 241 3/6/09 7:57:28 PM


242 Capítulo 8 limitaCiones Cognitivas y Comportamiento del Consumidor

COSTOS IRRECUPERABLES
Otro principio básico del modelo de la elección racional es que al tomar una decisión no se deben tener
en cuenta los costos irrecuperables. En el ejemplo del tenis que se presentó al principio de este capítulo
se vio que es precisamente este principio, no considerar los costos irrecuperables, el que suele ignorar-
se. El economista Richard Thaler argumenta que tales ejemplos no son raros, que en realidad la gente
presenta una tendencia general a no ignorar los costos irrecuperables. Thaler es el autor del experi-
mento “toda la pizza que pueda comer” que se vio en el capítulo 1. Recuerde que en este experimento
los comensales a los que se les devolvió el precio de la entrada comieron sustancialmente menos por-
ciones de pizza que los demás. Thaler ofrece algunos otros ejemplos claros de este comportamiento.
Uno es un experimento en el que primero se le pide que imagine haber comprado un par de za-
patos de moda en 600 dólares y que después se da cuenta de que le quedan terriblemente apretados.
Con el uso mejoran un poco, pero siguen siendo muy incómodos. ¿Usaría los zapatos o los desecha-
ría? ¿Cambiaría su respuesta si en vez de haber comprado los zapatos se los hubieran regalado?
De acuerdo con el modelo de la elección racional no debería importar si los compró o si se los
regalaron. De cualquier manera ahora son suyos y lo único que importa es si la incomodidad que
producen es lo suficientemente seria para desecharlos. En los dos casos debe ser idéntica la posibili-
dad de que la persona deseche los zapatos. Sin embargo, contrario a esta predicción, es mucho más
probable que una persona diga que los desechará si los ha recibido como regalo. Al parecer, haber
desembolsado 600 dólares hace que muchas personas decidan soportar los zapatos.
Un último ejemplo de costos irrecuperables: suponga que acaba de pagar 40 dólares por un
boleto para un partido de basquetbol que tendrá lugar esta noche en un estadio a 60 millas al norte
de donde usted vive. De repente empieza a nevar con fuerza y aunque la carretera que tiene que
cruzar es transitable será difícil recorrerla. ¿A pesar de esto va al partido? ¿Cambiaría su respuesta
si en lugar de haber comprado el boleto se lo hubieran regalado? Thaler encuentra que la mayor
parte de las personas que compraron el boleto irán al partido, mientras que la mayor parte de las que
lo recibieron como regalo se quedarán en casa. Por supuesto que, de acuerdo con el modelo de la
elección racional, la decisión debe ser la misma en cualquier caso. Si el placer que espera tener por
ver el partido es superior a la incomodidad de tener que conducir hasta el estadio, irá al partido; si no
es así, se quedará en casa. En este cálculo costo-beneficio ningún elemento debe depender de cómo
se obtuvo el boleto.

COSTO DESEMBOLSADO CONTRA COSTOS


DE OPORTUNIDAD
Thaler sugiere que a la tendencia a no ignorar los costos no recuperables puede dársele una expli-
cación sencilla en términos de la función de valor de Kahneman y Tversky. En el caso del ejemplo
del tenis, no jugar en una cancha en el exterior en un bello día se codifica mentalmente como una
ganancia no aprovechada, mientras que no jugar en la cancha cubierta por la que ya se han pagado 20
dólares se codifica como una pérdida. Aun cuando en este caso la ganancia es mayor que la pérdida,
la mayor inclinación de la función de valor en el dominio de las pérdidas crea un sesgo a favor de la
cancha cubierta.
Una interpretación muy parecida se encuentra en otros ejemplos convincentes.2 Considere una
persona que compró una caja de vino a 5 dólares/botella en 1982. Hoy el mismo vino se vende a 100
dólares/botella. Un vinatero le ofrece 60 dólares/botella y la persona se rehúsa a comprarlo, pues lo
máximo que estaría dispuesta a pagar por el vino serían 35 dólares/botella. El modelo de la elección
racional descarta tal comportamiento. Pero si los gastos desembolsados (por ejemplo, para la com-
pra de más vino) se codifican como pérdidas, mientras que los costos de oportunidad (por ejemplo,
no comprar el vino al vinatero) se codifican como ganancias no aprovechadas, entonces la función
asimétrica de valor explica estas respuestas.
Un ejemplo aún más común es el caso de los boletos para un evento de entretenimiento fuera
de lo común. En 2007 los boletos para el Súper Bowl tenían un precio de 700 dólares, en los lugares
oficiales de venta, pero en el mercado abierto los precios llegaron hasta 9 000 dólares. Miles de afi-
2
Vea R. Thaler, “Toward a Positive Theory of Consumer Choice”, en Journal of Economic Behavior and Organization, 1980.

08_CHAPTER 8.indd 242 3/6/09 7:57:29 PM


errores afeCtivos de prediCCión 243

cionados usaron sus boletos de 700 dólares para asistir al juego, desaprovechando la oportunidad
de venderlos a 9 000 dólares. Sin embargo, muy pocos de estos aficionados hubieran gastado 9 000
dólares para comprar un boleto.
Thaler presenta el ejemplo paralelo del hombre que rehusaría cortar el césped de su vecino por 40
dólares y, sin embargo, corta su propio césped, que es idéntico, aun cuando el hijo de su vecino estaría
dispuesto a cortarlo por sólo 20 dólares. Este comportamiento, como el de los aficionados al Súper
Bowl, coincide también con la noción de que mientras los gastos que salen del propio bolsillo son codi-
ficados como pérdidas, los costos de oportunidad son considerados como ganancias no aprovechadas.

ERRORES AFECTIVOS DE PREDICCIÓN


Para elegir con inteligencia entre un par de alternativas, suele ser necesario predecir con exactitud el
efecto que éstas tendrán en usted en ese momento. Por ejemplo, al elegir entre papas y arroz para la
cena, sólo es necesario predecir qué será más satisfactorio esa noche. La mayoría de las personas no
tienen problema en esto.
En muchos otros casos, una elección inteligente requiere no sólo predecir qué alternativa será
más satisfactoria en ese momento, sino también cómo será la experiencia con cada una a lo largo del
tiempo. Ante decisiones de compra suele dársele poca atención a la adaptación.3 En la mayoría de los
casos, lo atractivo de un bien o de una actividad se estima probándolo y viendo qué efecto produce
en cada quien. Para decidir si pagar un poco más por una casa con una bonita vista, por ejemplo,
se sienta un momento en la sala y contempla la vista; para decidir si compra un automóvil nuevo,
primero hace una prueba de manejo, etcétera.
¿Ignorar la adaptación al hacer una elección ocasiona que unas posibilidades resulten engaño-
samente más atractivas que otras? La respuesta, por supuesto, depende de si a lo largo del tiempo la
adaptación varía de acuerdo con las diferentes categorías. Si no, pasar por alto la adaptación no in-
troduce ningún sesgo. Por ejemplo, si su impresión inicial de cualquier actividad exagera su atractivo
final en un factor, por ejemplo, de tres, todas las actividades pueden decepcionarnos, pero la elección
entre ellas no estará sesgada.
Sin embargo, es seguro que las distorsiones surgirán si en unas áreas nos adaptamos con más ra-
pidez o por completo que en otras. Por lo tanto, si elegimos entre experiencias potenciales de acuer-
do con una reacción inicial, la consecuencia lógica será que se invertirá mucho en experiencias cuyo
atractivo disminuye fuertemente con el tiempo y muy poco en actividades cuyo atractivo disminuye
menos con el tiempo. Un tema central, aunque con frecuencia implícito, en la literatura psicológica
acerca de los determinantes de la satisfacción en la vida es que, en efecto, la adaptación varía mucho
entre las distintas categorías.
Los psicólogos Leaf van Boven y Thomas Gilovich encontraron, por ejemplo, que las personas
tienden a adaptarse mucho más rápido al consumo de bienes que al de experiencias.4 Así, aunque
la mayoría de las personas experimentan una enorme satisfacción al comparar televisores con una
pantalla más grande o un refrigerador con mayor capacidad, estas sensaciones, casi de manera inva-
riable, tienden a desaparecer con rapidez. Una vez que uno se acostumbra a la pantalla más grande o
al refrigerador más espacioso, sus características atrayentes se desvanecen y pasan a segundo plano.
Dejamos de ser conscientes de ellas.
En cambio, los perfiles temporales de las reacciones ante aumentos en muchas otras formas
de consumo tienen una configuración en esencia opuesta. Por ejemplo, la experiencia subjetiva de
hacer ejercicio o de tratar de tocar un instrumento musical puede ser, en un principio, un poco des-
agradable, pero, a medida que pase el tiempo, para la mayoría de las personas estas experiencias se
vuelven cada vez más placenteras. En la medida en que se ignoren las distintas tendencias a adaptarse
a diferentes esferas se invertirá demasiado en algunos bienes y muy poco en otros.
Considere, por ejemplo, un hombre que está por decidir si cambiar su Toyota Corolla por un
Porsche Boxster nuevo. Puede pagar las mensualidades de su automóvil nuevo trabajando un sábado

3
Para conocer más sobre este tema consulte George Loewenstein y David Schkade, “Wouldn’t It Be Nice? Predicting Future Fe-
elings”, en D. Kahneman, E. Diener y N. Schwartz (editores), Well Being: The Foundations of Hedonic Psychology, Nueva York, Russell
Sage Foundation, 1999.
4
Vea Leaf van Boven y Thomas y Gilovich, “To Do or to Have? That Is the Question”, en Journal of Personality and Social Psychology,
85, diciembre de 2003, pp. 1193-1202.

08_CHAPTER 8.indd 243 3/6/09 7:57:29 PM


244 Capítulo 8 limitaCiones Cognitivas y Comportamiento del Consumidor

más por mes, lo cual significaría no pasar ese día con sus amigos. El modelo de la elección racional
sugiere que trabajará un sábado más si la satisfacción que le proporcione el Porsche es mayor que la
que obtiene de estar en compañía de sus amigos. Pero si no ha tenido nunca un Porsche, no puede
estar seguro del efecto que tendrá en él esa experiencia. Tampoco puede saber lo que ocurrirá si
continúa encontrándose con sus amigos esos sábados. En ambos casos tendrá que conjeturar cómo
será el futuro.
La introspección puede proporcionar estimaciones razonablemente buenas de cómo afectarán
estas experiencias a su satisfacción a corto plazo. Pero es posible que los efectos relevantes a corto y
largo plazos de este problema sean distintos. Dado que el Porsche es mucho más rápido y se conduce
mucho mejor que su Toyota, la prueba de manejo le hará ilusionarse aún más. Sin embargo, con el
tiempo se acostumbrará a las características de su nuevo automóvil y las cualidades estimulantes de
éste disminuirán. La contribución a su bienestar subjetivo del tiempo que pasa con sus amigos tiene
un perfil temporal marcadamente distinto. A medida que las relaciones se prolongan a lo largo del
tiempo, la satisfacción que proporcionan tiende a aumentar más que a disminuir.
Entonces, a largo plazo, pasar más tiempo con sus amigos bien puede resultar ser la mejor elec-
ción. Sin embargo, es posible que a corto plazo el incremento de la satisfacción sea mayor con el nue-
vo automóvil. Y como los efectos inmediatos son la fuente de información más patente disponible en
el momento de la decisión, es posible que inclinen la balanza a favor del automóvil.
No tomar en cuenta la adaptación tiene consecuencias similares en las decisiones acerca de qué
tanto ahorrar. En este caso el problema surge de cómo la cantidad que se consume en un punto dado
en el tiempo afecta la capacidad para obtener, de ese mismo consumo, un placer en el futuro. A ma-
nera de ejemplo, considere la siguiente elección hipotética entre dos perfiles de consumo distintos
que pueden ser financiados por un mismo flujo de ingresos a lo largo de toda la vida. Suponga que
usted y todos los demás en su grupo de edad tendrán un sueldo de 50 000 dólares por año desde los
21 años hasta la edad de jubilación, a los 65 años. En un caso, suponga una situación en la que todos
ustedes gastan su sueldo completo de 50 000 dólares al año como se muestra en el perfil de nivel
de consumo que se indica como A en la figura 8.4. En otra situación suponga que usted y todos los
demás comienzan ahorrando 10 000 dólares por año (o, dicho de otra manera, que empieza consu-
miendo nada más 40 000 por año) y que después, de manera gradual, disminuye su tasa de ahorro
hasta que, a mediana edad, empieza a disponer de sus ahorros en cantidades cada vez mayores con
el objetivo de financiar un consumo adicional. Si, para simplificar, se ignora el hecho de que los
ahorros producen intereses, se obtiene el perfil de consumo creciente indicado como B. Observe que
aun cuando este perfil comienza 10 000 dólares abajo del perfil A y termina 10 000 dólares arriba del
mismo, el total de consumo a lo largo de toda la vida es el mismo en ambos perfiles. ¿Cuál elegiría
usted (suponga que en ambos casos recibirá la misma pensión generosa para su jubilación)?

FIGURA 8.4
Perfiles de consumo
estático y de consumo Consumo
creciente ($/año)
¿Qué perfil de consumo B
elegiría: A, en el cual todo
el mundo consume 50 000 A 60 000
dólares por año o B, en
el que todo el mundo 50 000
comienza consumiendo
40 000 dólares por año y 40 000
termina consumiendo
60 000? Cuando se planteó
esta pregunta un grupo
21 43 65
grande de estudiantes,
expresaron una fuerte Edad
preferencia por B.

08_CHAPTER 8.indd 244 3/6/09 7:57:30 PM


eleCCión Con inCertidumbre 245

Cuando hace algunos años planteé una pregunta en esencia similar a una muestra de más de 100
estudiantes de Cornell, casi 80% eligió B, el perfil de consumo creciente. Las evidencias indican que,
en efecto, este perfil es el más satisfactorio de los dos.5 Aquí, de nuevo, la idea es que las evaluaciones
de casi cualquier cosa —incluidos niveles materiales de vida— dependen de los marcos de referencia.
Un marco de referencia para evaluar niveles materiales de vida es el de comparar lo que se tiene con
lo que otros poseen. Pero como en este ejemplo se supone que todos tienen un mismo perfil de con-
sumo, la comparación no agrega nada de interés. Otro marco de referencia que resulta decisivo en
este problema es comparar lo que se tiene en este momento con lo que se tuvo en el pasado reciente.
Por ejemplo, alguien que consume 50 000 dólares este año es más probable que esté más satisfecho
con su nivel de vida si el año anterior consumió 45 000 dólares y menos si consumió 55 000. Por lo
tanto, la gente que consume mucho y muy rápido establece un marco de referencia de mayor gasto
dentro del cual tendrá que ser evaluado su consumo posterior.
Cuanto más se ahorre durante la primera etapa del ciclo de vida, más inclinados y, por lo tanto,
más satisfactorios serán los perfiles de consumo. Sin embargo, tan obvio como puede parecer esto,
muchas personas no consideran nunca de manera consciente el grado en que sus actuales hábitos
de gasto afectan sus evaluaciones. En la medida en que se ignore esta relación, se tenderá a ahorrar
muy poco. Aquí, de nuevo, no tomar en cuenta la adaptación puede conducir a errores costosos, si
bien, evitables.
Quizás el ejemplo más patente del error de no tomar en cuenta la adaptación futura es el que
cometen las personas que abusan de las drogas al no prever la creciente tolerancia a la droga elegida.
De esta manera, quien abusa de la cocaína por lo general se lanza hacia una espiral descendente al
esperar que la ingestión repetida de la droga le produzca la misma sensación de euforia que experi-
mentó la primera vez. Pero pronto descubre que para obtener ese mismo efecto necesita tomar do-
sis cada vez mayores. Y en breve le resulta necesario ingerir dosis enormes de droga sólo para evitar
sentir una profunda postración. Es seguro que pocas personas darían el primer paso hacia esta mise-
rable trayectoria si desde el principio conocieran claramente sus patrones de adaptación futura.

ELECCIÓN CON INCERTIDUMBRE


El modelo convencional de la elección racional con incertidumbre es el de la utilidad esperada de
Neumann-Morgenstern que se vio en el capítulo 6. Este modelo proporciona una valiosa orientación
acerca de la mejor manera de elegir entre alternativas inciertas. Pero Kahneman y Tversky demos-
traron que no siempre proporciona una buena descripción de la manera en que las personas deciden
en realidad.6 Para ejemplificar esto los investigadores presentaron a un grupo de voluntarios un con-
junto de elecciones. Comenzaron con el problema siguiente que produjo respuestas consistentes a la
perfección con el modelo de utilidad esperada:

Problema 1
Elección entre
A: una ganancia segura de 240 dólares (84%)
y
B: una posibilidad de 25% de obtener 1 000 dólares y una posibilidad de 75% de obtener
0 dólares (16%)
Las cantidades entre paréntesis indican el porcentaje de personas que eligió cada alternativa.
En este caso la mayoría eligió la ganancia segura de 240 dólares aun cuando el valor esperado de la
lotería, 250, era 10 dólares mayor. Para comprobar que este patrón es congruente con el modelo de
utilidad esperada, sea U la función de utilidad de una persona, definida sobre la riqueza total, y sea M

5
Vea D. C. Shin, “Does Rapid Economic Growth Improve the Human Lot?”, en Social Indicators Research, 8, 1980, pp. 199-221; y
Robert Frank y Robert Hutchens, “Wages, Seniority, and the Demand for Rising Consumption Profiles”, en Journal of Economic
Behavior and Organization, 21, 1993, pp. 251-276.
6
Vea Amos Tversky y Daniel Kahneman, “Judgment under Uncertainly: Heuristics and Biases”, en Science, 185, 1974, pp. 1124-
1131.

08_CHAPTER 8.indd 245 3/6/09 7:57:30 PM


246 Capítulo 8 limitaCiones Cognitivas y Comportamiento del Consumidor

su riqueza inicial en dólares. Entonces, la utilidad esperada de la elección A es U(M + 240), mientras
la de la elección B es 0.25U(M + 1 000) + 0.75U(M). Si la utilidad es una función cóncava de la riqueza
total (es decir, si la persona tiene aversión al riesgo), en la figura 8.5 puede verse por qué A fue más
atractiva que B.

FIGURA 8.5
Una persona que tiene U
aversión al riesgo U(W)
preferirá, por lo general,
una ganancia segura en
vez de una lotería con
un valor esperado un
U(M + 240)
poco superior
si la función de utilidad es 0.25U(M + 1 000) + 0.75U(M)
suficientemente cóncava,
U(M + 240) será mayor que
la utilidad esperada de una
apuesta que tenga valor
esperado positivo, 0.25U(M
+ 1 000) + 0.75U(M).

Riqueza
M M + 250 M + 1 000
M + 240

Después, se pidió a los voluntarios que consideraran un problema al parecer muy similar:

Problema 2
Elegir entre
C: una pérdida segura de 750 dólares (13%)
y
D: una posibilidad de 75% de perder 1 000 dólares y una posibilidad de 25% de perder 0
dólares (87%)
Esta vez la lotería tiene el mismo valor esperado que la opción segura. De acuerdo con el mode-
lo de utilidad esperada, las personas que tienen aversión al riesgo deberán elegir, una vez más, la al-
ternativa segura. Pero esta vez se observa un resultado impresionante. Casi siete veces más personas
eligieron la lotería que las que eligieron la pérdida segura de 750 dólares.
Por último, se les pidió a los voluntarios considerar el problema siguiente:

Problema 3
Elegir entre
E: una posibilidad de 25% de obtener 240 dólares y una posibilidad de 75% de perder 760
dólares (0%)
y
F: una posibilidad de 25% de obtener 250 dólares y una posibilidad de 75% de perder 750
dólares (100%)
Vistas de manera aislada, las respuestas al problema 3 no son en absoluto sorprendentes. La
lotería E es en todos los aspectos peor que la F y sólo una persona distraída la hubiera elegido.
Pero observe que la lotería E se obtiene al combinar las posibilidades A y D de los problemas 1 y
2 y que, de manera similar, la lotería F es el resultado de combinar las posibilidades B y C de los
dos problemas anteriores. En los dos primeros problemas, la combinación B y C fue elegida por

08_CHAPTER 8.indd 246 3/6/09 7:57:31 PM


heurístiCas CrítiCas y sesgos 247

menos personas (3%) que cualquier otra, mientras que la combinación A y D fue, con mucho, la
más elegida (por 73% de las personas), aun cuando la combinación A y D es estrictamente inferior
a la combinación B y C. Estos hallazgos, por supuesto, plantearon un fuerte desafío al modelo de
utilidad esperada.
Kahneman y Tversky argumentaron que el patrón observado era de manera exacta lo que
hubieran pronosticado mediante su función asimétrica de valor. Por ejemplo, observe que en el
problema 1 la elección es entre una ganancia segura y una lotería cuyos resultados posibles son no
negativos. Como la función de valor es cóncava en las ganancias y el valor esperado de la lotería sólo
es un poco mayor que la alternativa segura, la función de valor predice la elección de la última.
En cambio, en el problema 2, la elección es entre cierta pérdida y una lotería en la cual cada uno
de sus resultados es una pérdida. Como en las pérdidas la función de valor es convexa, se predice un
comportamiento de atracción al riesgo con respecto a tal elección y, por supuesto, esto es lo que se
observó. Como el problema 3 obliga a la gente a unificar las ganancias y las pérdidas relevantes, las
personas pudieron ver con facilidad que un par de alternativas dominaban sobre el resto y eligieron
en consecuencia.
Es tentador suponer que las violaciones al modelo de utilidad esperada sólo se presentan cuan-
do el problema es lo bastante complicado para que las personas tengan dificultad al calcular lo que el
modelo predice. Pero Kahneman y Tversky demostraron que se puede influir incluso sobre la más
simple de las decisiones formulando las alternativas de manera diferente.
Así, por ejemplo, pidieron a un grupo de personas elegir entre varias políticas de respuesta a
una rara enfermedad que podría costar 600 vidas si no se hacía nada. A un grupo se le pidió elegir
entre el programa A, con el que seguramente se salvarían 200 vidas y el programa B, con el que con
una probabilidad de 1/3 se salvarían 600 vidas y con una probabilidad de 2/3 cero vidas. En este
caso, 72% de las personas eligieron el programa A. A un segundo grupo le pidieron elegir entre el
programa C con el que morirían 400 personas o el programa D, que tenía 1/3 de probabilidad de que
nadie muriera y 2/3 de probabilidad de que 600 personas murieran. Esta vez, 78% de las personas
eligió el programa D.
Reflexionando un momento se ve que los programas A y C son idénticos, como también lo son
los programas B y D. Sin embargo, las personas de los dos grupos eligieron de maneras totalmente
diferente. Kahneman y Tversky explicaron que el primer grupo codificó “vida salvada” como ganan-
cias y por lo tanto tuvo aversión al riesgo al elegir entre A y B. De manera similar, el segundo grupo
codificó muertes como pérdidas, lo que los llevó a ser atraídos hacia el riesgo al elegir entre C y D.
Es tentador suponer que el comportamiento inconsistente, según predice el modelo de utilidad es-
perada, se reduce a situaciones en las que intervienen novatos en la toma de decisiones o en las que
están en juego asuntos de poca importancia. Sin embargo, Kahneman y Tversky encontraron que
incluso médicos experimentados pueden hacer recomendaciones similarmente incoherentes acerca
de los tratamientos cuando los problemas se plantean de una manera distinta. La moraleja es que
se recomienda ser cauteloso al tomar decisiones con incertidumbre. Debe tratarse de formular las
alternativas relevantes de varias maneras y ver si eso produce alguna diferencia. Y si éste es el caso, se
debe tratar de reflexionar sobre cuál de las formulaciones corresponde más a sus intereses.

HEURÍSTICAS CRÍTICAS Y SESGOS


Como lo muestran los ejemplos considerados hasta ahora, la gente suele tomar decisiones irracio-
nales aunque conozca al dedillo los hechos relevantes. Sin embargo, otras dificultades con las que
se encuentra el modelo de la elección racional es que con frecuencia se hacen inferencias erróneas
acerca de cuáles son los hechos relevantes. Y lo que es más importante, muchos de los errores que se
cometen son sistemáticos, no aleatorios. Kahneman y Tversky identificaron tres heurísticas, o reglas,
sencillas que la gente emplea para hacer juicios e inferencia acerca del entorno.7 Estas heurísticas son
eficientes en el sentido de que ayudan a economizar el esfuerzo cognitivo y la mayoría de las veces
proporcionan respuestas más o menos correctas.

7
Ibídem.

08_CHAPTER 8.indd 247 3/6/09 7:57:32 PM


248 Capítulo 8 limitaCiones Cognitivas y Comportamiento del Consumidor

Pero también dan lugar a grandes errores predecibles en muchos casos. A continuación se consi-
derará por separado cada una de las tres heurísticas.

DISPONIBILIDAD
La periodicidad de un acontecimiento, o de una clase de acontecimientos, por lo regular se estima
por la facilidad con que nos acordamos a ellos. La mayoría de las veces hay una estrecha correlación
positiva entre la facilidad con la que se puede hacer esto y la verdadera frecuencia con que ocurre.
Después de todo, es más fácil recordar ejemplos de cosas que se repiten en forma reiterada.
Pero la frecuencia con que ocurre un acontecimiento o evento no es el único factor que deter-
mina la facilidad con que se recuerda. Si se le pregunta a la gente, por ejemplo, si es verdad que cada
año hay más asesinos que suicidas en la ciudad de Nueva York, casi todos responderán con seguridad
que sí. Sin embargo, ¡siempre hay más suicidas! Kahneman y Tversky explican que se piensa que hay
más asesinos porque eso está más “disponible” en la memoria. La investigación acerca de la memoria
demuestra que un evento es mucho más fácil de recordar cuanto más vívido o sensacional sea. Aun
cuando se haya escuchado de más o menos la misma cantidad de asesinatos que de suicidios, es
posible que se recuerde una proporción mucho mayor de los primeros.
También existen otros factores en el funcionamiento de la memoria que afectan la facilidad
con que recordamos los diversos eventos. Por ejemplo, pregúntese si en inglés hay más palabras que
empiecen con la letra r que palabras que tengan una r como tercera letra. La mayoría de las personas
responderán con toda confianza que hay muchas más palabras que empiezan con r ; sin embargo, en
realidad hay muchas más que tienen una r como tercera letra. En la memoria se almacenan tantas
palabras como las que hay en un diccionario, en orden alfabético, empezando con la primera letra.
Conocemos multitud de palabras en las que la r es la tercera letra, pero recordarlas no es más fácil de
lo que es dar con ellas en un diccionario.
Los eventos también suelen ser más accesibles en la memoria si ocurrieron más recientemente.
Numerosas investigaciones indican que al hacer evaluaciones de acciones desempeñadas, las perso-
nas tienden a dar mucho más peso a la información reciente. Por ejemplo, en el béisbol, el promedio
de bateo de un jugador contra determinado lanzador a lo largo de toda la vida es el mejor instru-
mento disponible para predecir cómo se desempeñará frente a ese mismo lanzador en el futuro.
Sin embargo, parece que es frecuente que un entrenador deje en la banca a un bateador que en las
últimas semanas no ha tenido un muy buen desempeño contra un determinado lanzador, aunque
durante muchos años haya hecho muy buen papel contra él. El problema es que el entrenador valora
la actuación del jugador por los ejemplos que le llegan con rapidez a la mente. Y los más recientes
son los más fáciles de recordar.
Desde el punto de vista económico, el sesgo de la disponibilidad es importante porque con
frecuencia se tiene que estimar el desempeño relativo de opciones económicas alternativas. Los ge-
rentes de empresas, por ejemplo, deben ponderar los méritos de los diversos empleados para poder
promoverlos. Los administradores más eficientes serán aquellos que eviten la tendencia natural de
dar mucho más peso al desempeño reciente.

REPRESENTATIVIDAD
Kahneman y Tversky descubrieron también un interesante sesgo en la manera en que se trata de
responder preguntas de la forma, “¿cuál es la posibilidad de que el objeto A pertenezca a la clase
B?”: Por ejemplo, suponga que Steve es una persona tímida y que se desea estimar la posibilidad de
que sea bibliotecario y no vendedor. La mayoría de las personas se apresurarán a decir que es más
probable que sea bibliotecario porque se piensa que la timidez es una característica representati-
va de los bibliotecarios pero poco común en los vendedores. Sin embargo, estas respuestas están
con frecuencia sesgadas porque la posibilidad de pertenecer a la categoría en cuestión depende de
muchos otros factores importantes además de la representatividad. En este caso, la respuesta está

08_CHAPTER 8.indd 248 3/6/09 7:57:32 PM


heurístiCas CrítiCas y sesgos 249

fuertemente influenciada por la frecuencia relativa con que se encuentren los vendedores y los
bibliotecarios en la población total.
Un ejemplo sencillo muestra la esencia del problema. Suponga que 80% de los bibliotecarios son
tímidos y que sólo 20% de los vendedores lo son; además, suponga que en la población hay nueve
vendedores por cada bibliotecario. De acuerdo con estos supuestos razonables, si se sabe que Steve
es tímido y que es o bibliotecario o vendedor, ¿cuál es la probabilidad de que sea bibliotecario? En la
figura 8.6 se muestran las cifras relevantes para responder a esta pregunta. Ahí se ve que aun cuando
una proporción mucho mayor de bibliotecarios son tímidos, hay más del doble de vendedores que
también lo son. La causa, por supuesto, es que hay muchos más vendedores que bibliotecarios. Aquí,
de cada 100 personas, 26 son tímidos, 18 vendedores y 8 bibliotecarios. Esto significa que la proba-
bilidad de que una persona tímida sea bibliotecaria es de sólo 8/26, es decir, menos de un tercio. Sin
embargo, la mayoría de las personas que se encuentran con este ejemplo se muestran renuentes a
afirmar que Steve sea vendedor, porque la timidez no es nada representativa de los vendedores.

FIGURA 8.6
Distribución de
Bibliotecarios Vendedores bibliotecarios y
tímidos tímidos vendedores según
8 18 su tipo
aunque la timidez es
más representativa de
los bibliotecarios que
Bibliotecarios
de los vendedores, es
no tímidos
2
mucho más probable que
Vendedores una persona tímida sea
no tímidos vendedor que bibliotecario.
72 la razón es que hay mucho
más vendedores que
bibliotecarios.

EJERCICIO 8.1
Suponga que 90% de los bibliotecarios y 20% de los vendedores son tímidos; además, hay cuatro
veces más vendedores que bibliotecarios. ¿Cuál es la probabilidad de que una persona tímida
elegida al azar sea bibliotecario?

Otro ejemplo del sesgo de la representatividad es el fenómeno estadístico conocido como efecto
de regresión o regresión a la media. Suponga que a 100 personas se les hace una prueba para medir su
coeficiente intelectual y que las 20 que obtienen las puntuaciones más altas tienen una puntuación
promedio de 122, es decir, 22 puntos más que el promedio de la población. Si a estas mismas 20 se les
vuelve a hacer la prueba, casi siempre su puntuación promedio será sustancialmente menor que 122.
La razón es que existe cierta cantidad de aleatoriedad al realizar la prueba para medir el CI y que en-
tre las personas que obtuvieron un mejor resultado en la primera prueba es posible que haya muchas
cuyo desempeño fue por casualidad mejor que lo acostumbrado en esa determinada prueba.
En la vida diaria se tienen experiencias de primera mano de los efectos de la regresión (por ejem-
plo, los hijos de padres inusualmente altos tienden a ser de menor estatura). Sin embargo, Kahneman
y Tversky observan que, con frecuencia, esta circunstancia no es tomada en consideración al hacer

08_CHAPTER 8.indd 249 3/6/09 7:57:33 PM


250 Capítulo 8 limitaCiones Cognitivas y Comportamiento del Consumidor

los juicios de valor debido a que, de manera intuitiva, se piensa que un resultado (por ejemplo, la
descendencia) debe ser representativo del antecedente (por ejemplo, el padre) que lo produjo.

EL NATURALISTA ¿Por qué el novato del año en el beisbol suele tener una segunda temporada mediocre?
ECONÓMICO Eric Hinske, tercera base de Toronto, bateó .279 con 24 jonrones y 84 carreras empujadas cuan-
8.1 do ganó el premio del Novato del año de la Liga Americana en 2002. Sin embargo, en su segundo
año con los Blue Jays, su promedio bajó a .243 con sólo 12 jonrones y 63 empujadas. La caída en el
desempeño de Hinske en 2003 concuerda con un patrón observado en el beisbol y en otros depor-
tes profesionales en los que el novato del año suele tener un
desempeño mucho más bajo en la temporada siguiente. ¿A
qué se debe este patrón?
Al fenómeno en cuestión se le conoce como “maldición
del segundo año”. Otro hecho curioso relacionado con aquél
es la “maldición de la revista Sports Illustrated”, que consiste
en que un atleta que aparece una semana en la portada de
esta revista está destinado a un mal desempeño la semana
siguiente. Se dice que Shirley Babashoff, medallista olímpica
en natación, rehusó aparecer en la portada de la revista SI
por miedo a esta maldición.8 Sin embargo, estas dos supues-
tas maldiciones pueden explicarse como una regresión a la
media. Alguien llega a ser el novato del año sólo después de
haber tenido una temporada excepcionalmente buena. De
manera similar, en la portada de la revista SI aparecen atle-
tas que tuvieron una actuación inusualmente buena. En sus
actuaciones siguientes, aunque superiores al promedio, casi
de manera inevitable estarán por debajo de la media que les
¿a qué se debe la “maldición del segundo año”?
permitió ganar su galardón.

Una consecuencia en especial perniciosa, de no tomar en cuenta la regresión a la media, es el


efecto que ejerce sobre las estimaciones la eficacia relativa de la alabanza y la censura. Desde hace
mucho los psicólogos han demostrado que en la enseñanza de habilidades la alabanza y otras formas
de reforzamiento positivo son mucho más efectivas que el castigo o la reprobación. Pero es poco
probable que la gente llegue a esta conclusión a partir de la experiencia si no pone atención a la
importancia de la regresión a la media.
La razón es que, independientemente de si una persona es alabada o reprobada, lo más proba-
ble es que un buen desempeño vaya seguido de uno menos bueno y que después haya uno mejor.
Alguien que alaba un buen resultado es posible que concluya, en forma errónea, que la alabanza fue
la causa del empeoramiento del resultado posterior. A la inversa, alguien que desaprueba un mal
desempeño es posible que de manera equivocada atribuya esto una posterior mejoría, cuando en rea-
lidad sería consecuencia de los efectos de la regresión. Puede que este razonamiento convenza a casi
todo el mundo, salvo a los analistas más sofisticados, de que la reprobación funciona y la alabanza no.
Los directivos que tratan de obtener un desempeño más eficiente de sus empleados no pueden darse
el lujo de cometer este error.

ANCLAJE Y AJUSTE
En una estrategia de estimación muy usada, conocida como “anclaje y ajuste”, se hace primero una
estimación preliminar —un ancla— y después se ajusta con la información adicional que se tenga y
que parezca relevante. Kahneman y Tversky descubrieron que este procedimiento solía conducir

8
Vea Thomas Gilovich, How We Know What Isn’t So, Nueva York, The Free Press, 1991.

08_CHAPTER 8.indd 250 3/6/09 7:57:36 PM


la psiCofísiCa de la perCepCión 251

a estimaciones sesgadas por dos razones. Primera, porque el ancla inicial podía no tener relación
alguna con el valor por estimar. Y segunda, porque aunque estuviera relacionada se tendería a hacer
un ajuste muy pequeño de la estimación inicial.
Para demostrar el sesgo del anclaje y el ajuste, Kahneman y Tversky pidieron a un grupo de
estudiantes que estimara el porcentaje de países africanos que eran miembros de las Naciones Uni-
das. A cada estudiante se le pidió primero que hiciera girar una rueda que generaba un número del
1 al 100. Después se les preguntó si su estimación era mayor o menor que ese número. Y por último
se les pidió expresar el porcentaje de su estimación numérica. Los resultados fueron sorprendentes.
En el caso de los estudiantes que al girar la rueda obtuvieron un 10 o menos, la estimación me-
dia fue de 25%, mientras que la cifra correspondiente a los estudiantes que habían obtenido 65 o más
fue 45%.
Posiblemente los estudiantes sabían que el número aleatorio inicial no podía tener ninguna re-
levancia en la valoración del porcentaje de países africanos que pertenecían a las Naciones Unidas. A
pesar de esto, los números tuvieron un efecto impresionante sobre las estimaciones que dieron. La
conclusión era que, en problemas semejantes, parecía que el número que aleatoriamente se escogie-
ra era un punto de partida adecuado. Kahneman y Tversky informaron que dar a los estudiantes una
compensación monetaria por la exactitud de la estimación no alteró la magnitud de los sesgos.
En otro ejemplo se pidió a estudiantes de bachillerato que en 5 segundos estimaran el producto
de 8 números. Al primer grupo se le dio la expresión siguiente:

8 × 7 × 6 × 5 × 4 × 3 × 2 × 1,

mientras que al segundo grupo se le dieron los mismos números pero en orden inverso:

1 × 2 × 3 × 4 × 5 × 6 × 7 × 8.

El limitado tiempo de que disponían impidió que la mayoría de los estudiantes realizaran el cálculo
completo (que los hubiera llevado a la respuesta correcta de 40 320). Al parecer lo que muchos de
ellos hicieron fue realizar las primeras multiplicaciones (su ancla) y después hacer una estimación
del resultado final. En ambos grupos de estudiantes las anclas no resultaron ser muy adecuadas y las
estimaciones resultaron ser terriblemente insuficientes. Los sesgos obtenidos muestran con exacti-
tud el patrón previsto: la estimación media del primer grupo fue 2 250; la del segundo fue sólo 512.
Una aplicación económica importante del sesgo del anclaje y el ajuste es la estimación de las
tasas de fracaso de proyectos complejos. Considere, por ejemplo, el inicio de un negocio. Para tener
éxito es necesario que se den numerosas condiciones. Debe obtenerse un financiamiento satisfacto-
rio, encontrar una ubicación adecuada, diseñar un proceso de producción de bajo costo, contratar
mano de obra calificada, poner en marcha una campaña de comercialización eficiente, etc. Si cual-
quiera de estos pasos fallara, no se tendrá éxito. Cuando intervienen muchos factores, la tasa de
fracaso es invariablemente alta, aun cuando cada paso tenga una probabilidad alta de éxito. Por ejem-
plo, un programa en el que intervengan 10 pasos, cada uno de los cuales tiene una tasa de éxito de
90%, fracasará 65% de las veces. Al estimar tasas de fracaso para procesos de este tipo, la gente tiende
a anclar en una tasa de fracaso baja para el paso típico, a partir de lo cual hace ajustes terriblemente
insuficientes. El sesgo del anclaje y el ajuste puede ayudar a explicar el fracaso de la enorme mayoría
de negocios nuevos.

LA PSICOFÍSICA DE LA PERCEPCIÓN
Ley de Weber-Fechner
Existe otro patrón en la manera en que se percibe y procesa la información que es importante para propiedad de la percepción
las aplicaciones económicas. Dicho patrón se deriva de la llamada ley de Weber-Fechner de la psi- según la cual la diferencia
cofísica. Weber y Fechner lograron descubrir cuán grande tenía que ser la variación de un estímulo apenas notable de un estímulo
para que pueda percibirse la diferencia de su intensidad. Por ejemplo, la mayoría de las personas no tiende a estar en proporción
pueden distinguir la luz de un foco de 100 watts de la de un foco de 100.5 watts. Pero, ¿qué diferencia con el valor del estímulo.

08_CHAPTER 8.indd 251 3/6/09 7:57:36 PM


252 Capítulo 8 limitaCiones Cognitivas y Comportamiento del Consumidor

de intensidad luminosa debe existir para que las personas puedan identificarla? Weber y Fechner
encontraron que la diferencia mínima perceptible es aproximadamente proporcional a la intensidad
original del estímulo. De manera que cuanto más intenso sea el estímulo, mayor tendrá que ser, en
términos absolutos, la diferencia para que pueda ser percibida.
Thaler indica que la ley de Weber y Fechner parece funcionar cuando la gente decide cuáles son
las diferencias de precios por las que vale la pena preocuparse. Suponga, por ejemplo, que usted está
a punto de comprar un radio-reloj en una tienda en la que se vende en 25 dólares cuando un amigo
le informa que en otra tienda, a 10 minutos de distancia, el mismo radio-reloj tiene un precio de 20
dólares solamente. ¿Va usted a la otra tienda? ¿Variaría su respuesta si estuviera a punto de comprar
un televisor de 1 050 dólares y su amigo le dijera que puede obtener el mismo aparato en la otra
tienda a un precio de únicamente 1 045 dólares? Thaler encontró que la mayoría de las personas
responde con un sí a la primera pregunta y con un no a la segunda.
De acuerdo con el modelo de la elección racional es incongruente dar respuestas diferentes
a estas dos preguntas. Una persona racional iría a la otra tienda si y sólo si el beneficio de hacerlo
fuera superior al costo. En ambos casos el beneficio es de 5 dólares. El costo de ir a la otra tienda es
también el mismo, ya sea que se compre un radio o una televisión. Si tiene sentido ir a la otra tienda
en un caso, también lo tendrá en el otro.

LA DIFICULTAD REAL DE DECIDIR


En el modelo de la elección racional no debe haber decisiones difíciles. Si la elección entre dos alter-
nativas es difícil —es decir, si se prevé que las dos opciones generan aproximadamente la misma uti-
lidad— entonces debe dar igual cuál de ellas se elija. Ahora, si una de las opciones tiene una utilidad
esperada claramente mayor, la elección debe ser fácil otra vez. De cualquier manera, la persona que
elige no tiene razones obvias para experimentar ansiedad e indecisión.
Por supuesto, en realidad se sabe que las decisiones difíciles son más bien la excepción que
confirma la regla. Existen muchos pares de alternativas en las cuales las funciones de utilidad sim-
plemente no parecen asignar clasificaciones de preferencia claras y no ambiguas. La dificultad se
acentúa cuando las alternativas difieren en características difíciles de comparar. Si las tres cosas que
más importan de un automóvil son, por ejemplo, la comodidad, la economía de combustible y la
seguridad, será fácil decidir entre dos automóviles si uno es más seguro, más cómodo y consume
menos combustible que el otro. Pero, ¿qué pasa si uno es más cómodo pero no consume menos
gasolina? En principio se supone que se tienen curvas de indiferencia que indican a qué tasa se esta-
rá dispuesto a intercambiar una característica por la otra. Sin embargo, en la práctica parece difícil
utilizar la información implícita en estas curvas. Y la sola idea de pensar en hacerlo provoca cierta
incomodidad. Por ejemplo, es frecuente que a la gente le preocupe arrepentirse de la elección que
haga. (“Si me decido por un automóvil más cómodo, ¿qué pasa si después me cambio a un empleo en
el que tenga que recorrer una distancia mayor?”)
Estas dificultades parecen poner en duda un axioma fundamental de la teoría de la elección
racional, a saber, las decisiones deben ser independientes de las alternativas irrelevantes. Este axioma
suele ilustrarse mediante anécdotas como la siguiente. Un hombre entra en una salchichonería y pre-
gunta de qué tienen sándwiches. El empleado le contesta que tienen de rosbif y de pollo. El cliente
piensa unos momentos y, al final, pide un sándwich de rosbif. El vendedor dice: “Ah, se me olvidaba
decirle que también tenemos de atún.” A lo que el cliente responde: “Bueno, en ese caso creo que me
llevaré uno de pollo.” De acuerdo con el modelo de la elección racional, la alternativa del atún sólo
debe tener importancia si es la que prefiere el cliente. No hay ninguna razón lógica que explique que
el que haya de atún haga que el cliente cambie el rosbif por pollo.
En colaboración con Itamar Simonson, Tversky realizó interesantes experimentos que sugieren
que en realidad la elección no siempre es independiente de las alternativas irrelevantes.9 Uno de sus

9
Vea Itamar Simonson y Amos Tversky, “Choice in Context: Tradeoff Contrast and Extremeness Aversion”, en Journal of Marketing
Research, 29 de agosto de 1992, pp. 281-295.

08_CHAPTER 8.indd 252 3/6/09 7:57:36 PM


la difiCultad real de deCidir 253

ejemplos es el de la elección entre departamentos que difieren en dos características: alquiler men-
sual y distancia al campus universitario. Desde el punto de vista de un estudiante, un departamento
es mejor entre más cerca esté del campus universitario y menor sea la renta mensual. A un grupo de
estudiantes se les pidió que eligieran entre dos departamentos como los que se muestran en la figura
8.7. En ésta observe que ninguno de los departamentos es mejor que el otro. A es más caro, pero B
está más lejos del campus. Se espera que los estudiantes a los que les preocupa más el precio elijan el
departamento B, mientras que aquellos a los que les importa el tiempo de recorrido hasta el campus
elijan el A. Manipulando la distancia y la renta se puede hacer que un grupo de estudiantes se divida
aproximadamente a la mitad entre los dos departamentos.

FIGURA 8.7
Elección entre dos
Renta mensual departamentos
manipulando de manera
adecuada la renta mensual
A y la distancia al campus
universitario es posible
hacer que un grupo de
estudiantes se divida en dos
mitades iguales respecto a
B su elección entre A y B.

Distancia al campus universitario

Hasta ahora no hay nada sorprendente. Pero ahora los investigadores agregan un tercer depar-
tamento, C, a la lista de opciones, con lo cual se obtiene el conjunto que se representa en la figura 8.8.
Observe que B es mejor que C, es decir, C está más lejos del campus y es más caro que B. En términos
del modelo de la elección racional, éste es un ejemplo clásico de una alternativa irrelevante. Ante la
elección entre A, B y C, ningún consumidor razonable elegiría C. Y en efecto, en los experimentos
muy pocos lo hicieron.

FIGURA 8.8
Se agrega una
Renta mensual alternativa irrelevante
Como B es mejor que
C nadie lo elegirá. pero
A aunque nadie elija C, su
existencia hace que las
C personas se inclinen más
por B.

Distancia al campus universitario

Lo sorprendente es que opciones como C afectan la elección de las personas entre las opciones
restantes. Tversky y Simonson descubrieron que cuando al par de departamentos A y B se añade un

08_CHAPTER 8.indd 253 3/6/09 7:57:38 PM


254 Capítulo 8 limitaCiones Cognitivas y Comportamiento del Consumidor

departamento como C, las elecciones de las personas se desplazan en forma sustancial un favor de B.
Antes de agregar C, los estudiantes se dividieron en dos mitades entre A y B. Sin embargo, una vez
que se agregó C más de 70% de los estudiantes eligió B, la opción que es mejor que C.
Parece ser que la mayoría de las personas encuentran difícil la elección entre A y B. La aparición
de C les proporciona una comparación más fácil, es decir, la comparación entre B y C. La hipótesis de
los investigadores es que esto crea un “efecto halo” para B, lo que hace que aumenten las posibilida-
des de que sea elegido en lugar de A. Quizás un efecto similar sea el que haga que la posibilidad de
elegir atún provoque que alguien cambie su decisión de rosbif por pollo. Sea cual sea la razón de este
comportamiento, viola el axioma de que la elección es independiente de las alternativas irrelevantes.

¿Por qué los agentes inmobiliarios muestran a los clientes dos casas casi idénticas, aun cuan-
EL NATURALISTA do una sea menos cara y esté en mejores condiciones que la otra?
ECONÓMICO
Como en el ejemplo que se acaba de ver, el hecho de que una casa sea mejor que la otra puede dotar
8.2 a la primera de un halo que la haga más atractiva en relación con casas que sean mejores que ella
por lo menos en alguna característica. Por ejemplo, puede que un agente inmobiliario tenga un
cliente al que le cueste trabajo decidirse entre una casa estilo neoclásico y una victoriana. Al mostrar

EN VENTA EN VENTA
$200 000 $250 000

¿por qué un agente inmobiliario se toma el trabajo de mostrar una alternativa peor?

al cliente una casa victoriana parecida cuyo precio sea mayor y esté en condiciones menos buenas
que la primera, el agente puede resolver la venta a favor de la primera. Una vez más, parece que a la
gente no le gusta elegir entre alternativas que son difíciles de comparar. Los agentes inmobiliarios
experimentados suelen evitar este problema dando a sus clientes la oportunidad de enfocarse en una
elección más sencilla.

08_CHAPTER 8.indd 254 3/6/09 7:57:44 PM


el esCollo del autoControl 255

©The New Yorker Collection 1998 Mick Stevens from cartoonbank.com.


PELIGRO:
PAPAS FRITAS
ADELANTE

All Right Reserved


EL ESCOLLO DEL AUTOCONTROL
Otra razón por la cual los comportamientos no siempre coinciden con las predicciones de los mode-
los simples de elección racional es que la gente tiene cierta dificultad en llevar a cabo planes que con-
sidera en beneficio propio. Thomas Schelling observa, por ejemplo, que muchos fumadores dicen
que quieren dejar de fumar.10 Muchos de ellos lo consiguen con enorme esfuerzo. (Tanto Schelling
como yo pertenecemos a este grupo y puedo atestiguar lo difícil que es.) Sin embargo, muchos más
han tratado de dejar de fumar sin lograrlo.
Una manera de resolver el problema del autocontrol es captada por el ejemplo de Ulises relatado
por Homero, quien tenía que pasar navegando por peligrosos arrecifes donde se encontraban las si-
renas. Ulises sabía que en cuanto oyera el canto de las sirenas éste lo atraería de manera irresistible y,
para su perdición, navegaría hacia los arrecifes. Previendo este cambio temporal de sus preferencias,
se le ocurrió un eficiente mecanismo de compromiso. Pidió a los miembros de la tripulación que lo
ataran fuertemente al mástil y que no lo desataran hasta que hubieran pasado los arrecifes.
En la vida moderna existen otros conocidos mecanismos de compromiso. Ante el temor de
gastar sus ahorros, las personas participan en “Clubes de Navidad”, cuentas especiales que prohíben
hacer retiros antes de finales del otoño, o compran pólizas de seguro para la jubilación, que imponen
fuertes sanciones por retiros antes de tiempo. Temiendo echar a perder la cena, ponen los cacahuates
salados fuera de su alcance. Temiendo apostar demasiado, limitan la cantidad de efectivo que llevan
a los casinos de Atlantic City. Temiendo quedarse despiertas hasta muy tarde viendo la televisión, la
colocan fuera del dormitorio.
La moraleja de la floreciente literatura sobre autocontrol es que idear un plan de consumo inter-
temporal es sólo parte del problema. También está la tarea de ponerlo en marcha. Pero en este caso,
la deliberación racional puede ayudar a evitar algunos de los principales escollos. El consumidor que
acaba de dejar de fumar, por ejemplo, puede predecir que si el viernes en la noche sale a beber con
sus amigos deseará con desesperación un cigarrillo. Puede evitar la tentación al dedicarse a otro tipo
de actividades de fin de semana durante 1 mes. De la misma forma, quien quiere protegerse de la
tentación de gastar demasiado, puede asignar de manera automática parte de su paga a una cuenta
de ahorros, que es precisamente lo que hacen millones de personas.
Una vez más, estos ejemplos resaltan la diferencia entre los papeles positivo y normativo del mo-
delo de la elección racional que se vio en el capítulo 1. Así, debido a que dicho modelo no toma en
cuenta los problemas del autocontrol, algunas veces no logra predecir cuál será el comportamiento
real de la gente. Pero observe que esto no significa que el modelo, incluso en su forma más estricta,

10
Vea Thomas Schelling, Choice and Consequence, Cambridge, MA: Harvard University Press, 1984.

08_CHAPTER 8.indd 255 3/6/09 7:57:45 PM


256 Capítulo 8 limitaCiones Cognitivas y Comportamiento del Consumidor

esté equivocado o sea inútil. En ésta, como en otras circunstancias, el modelo tiene el importante
papel de orientar a la gente hacia mejores decisiones que coincidan más con sus objetivos reales.

RESUMEN
• Numerosos ejemplos del comportamiento contradicen las en el dominio de las ganancias y atracción al riesgo en el do-
predicciones del modelo convencional de la elección racional. minio de las pérdidas. Esto da como resultado que diferencias
La gente, con frecuencia, no logra ignorar los costos irrecupe- sutiles en la formulación del problema puedan desplazar el
rables. Juegan tenis en canchas cubiertas cuando, en realidad, punto medio de referencia empleado para calcular ganancias
preferirían jugar en las canchas del exterior. Se comportan de y pérdidas, lo que a su vez puede originar patrones de elección
manera diferente cuando pierden un boleto que cuando pier- radicalmente diferentes.
den una cantidad de dinero equivalente. Los psicólogos indi- • Otra desviación importante de la elección racional ocurre en
can que tal comportamiento es resultado de las limitaciones las heurísticas, o reglas generales, que emplea la gente para
de la capacidad cognitiva humana. La gente emplea sistemas hacer estimaciones de factores de decisión importantes. La
de cuentas mentales para disminuir la complejidad de sus de- heurística de la disponibilidad indica que una manera de es-
cisiones, con frecuencia a expensas de la coherencia con los timar la frecuencia de una determinada clase de eventos es
axiomas de la elección racional. mediante la facilidad con la que pueden recordarse ejemplos
• Un tipo importante de desviaciones de la elección racional relevantes de ellos. Esto conduce a sesgos predecibles debido
parece deberse a la función asimétrica de valor descrita por a que la frecuencia no es el único factor que rige la facilidad de
Kahneman y Tversky. Al contrario del modelo de la elección recordar ejemplos. Las personas tienden a sobrestimar la fre-
racional que emplea una función de utilidad definida sobre cuencia de hechos intensos o sobresalientes, así como de otros
la riqueza total, la teoría descriptiva de Kahneman y Tversky sucesos especialmente fáciles de traer a la memoria.
emplea una función de valor definida con base en las variacio- • Otra heurística importante es la representatividad. La gente
nes de la riqueza. A diferencia del modelo tradicional, asigna estima la posibilidad de que un objeto pertenezca a una de-
un peso mucho mayor a las pérdidas que a las ganancias en la terminada clase por lo representativo que sea de ella. Este
toma de decisiones. Esta característica hace que las decisiones comportamiento suele conducir a sesgos importantes, pues
sean en extremo sensibles a la manera en que se formulen las la representatividad es sólo uno de muchos factores que rigen
alternativas. Por ejemplo, si una pérdida se combina con una esta circunstancia. La timidez puede ser en efecto una carac-
ganancia un poco mayor, el efecto neto por lo general es una terística representativa de los bibliotecarios, pero dado que
evaluación positiva, como lo sería en el modelo de la elección hay muchos más vendedores que bibliotecarios es mucho más
racional. Pero Kahneman y Tversky indican que cuando las probable que alguien tímido, tomado al azar, sea vendedor
ganancias y las pérdidas ocurren como acontecimientos dis- que bibliotecario.
tintos, la gente tienden a evaluar sus efectos por separado, en • Anclaje y ajuste es la tercera heurística que suele conducir a es-
cuyo caso el impacto de la pérdida tiende a sobrepasar el de timaciones sesgadas de factores importantes en una decisión.
una ganancia mayor. Una pérdida combinada con una ganan- Esta heurística indica que la gente suele realizar estimaciones
cia un poco mayor produce un efecto positivo, mientras que numéricas eligiendo primero un ancla adecuada (aunque a
vistas por separado su efecto neto es negativo. veces irrelevante) y ajustándola después (por lo general de
• Otra fuente de decisiones no óptimas es no poder prever cómo manera insuficiente) con base en otra información potencial-
será la adaptación a las diferentes experiencias de consumo con mente relevante. Este procedimiento suele hacer que la gente
el paso del tiempo. Al elegir entre dos bienes, la gente tiende a subestime la tasa de fracaso de proyectos en los que intervie-
favorecer la alternativa que le proporcione mayor satisfacción nen muchos pasos. Tales proyectos fracasan si cualquiera de
en el momento de la decisión. Sin embargo, la evidencia sugie- sus elementos esenciales fracaso, lo cual significa que aun
re que la satisfacción que proveen algunos bienes y actividades cuando la tasa de fracaso de cada elemento sea en extremo
tiende a disminuir con rapidez a lo largo del tiempo, en tanto baja, es muy probable que un proyecto en el que intervienen
que para otras decae con más lentitud o incluso se incrementa. muchos elementos fracase. Como la gente tiende a anclar en
El resultado es una tendencia a gastar demasiado en bienes y la tasa de fracaso, y a partir de eso hace ajustes insuficientes,
actividades que entran en la primera categoría y muy poco en sobrestima enormemente la posibilidad de éxito. Esto ayuda a
las que se incluyen en la segunda. explicar el ingenuo optimismo de la gente que inicia un nuevo
• Las decisiones con incertidumbre también violan con fre- negocio.
cuencia las prescripciones del modelo de utilidad esperada. • Otra desviación de la elección racional apunta a la psicofísica
En este caso la función de valor simétrico provee también una de la percepción. Los psicólogos han descubierto que la varia-
descripción consistente de varias pautas de comportamiento ción apenas perceptible en cualquier estímulo es proporcional
importantes. Las personas tienden a sentir aversión al riesgo a su nivel inicial. Esto parece satisfacerse también cuando el

08_CHAPTER 8.indd 256 3/6/09 7:57:46 PM


problemas 257

estímulo en cuestión es el precio de un bien o servicio. A la • Con frecuencia los modelos de elección basados en la conducta
gente no le importa cruzar una ciudad para ahorrarse 5 dó- funcionan mucho mejor en la predicción de decisiones reales
lares en un radio que vale $25, pero de ningún modo lo hará que en el modelo de la elección racional. Sin embargo, es im-
para ahorrar 5 dólares en una televisión de 1 000 dólares. portante recordar que los modelos de conducta no pretenden
• Las desviaciones de la elección racional también pueden atri- tener ningún significado normativo. Es decir, el hecho de que
buirse a que la gente tiene dificultades para elegir entre alter- predigan, por ejemplo, que con frecuencia la gente ignora los
nativas difíciles de comparar. El modelo de la elección racional costos irrecuperables no debe entenderse como que la gente
supone que la gente tiene órdenes totales de preferencia, pero deba ignorarlos. El modelo de la elección racional señala que
en la práctica parece que se requiere de un gran esfuerzo para se tomarán mejores decisiones si se pasan por alto los costos
decidirse incluso entre alternativas muy sencillas. irrecuperables, y la mayoría de la gente, después de reflexionar
• Por último, las desviaciones de la elección racional pueden un poco, estará de acuerdo. En cuanto a esto, los modelos de
ocurrir debido a que a las personas les falta fuerza de voluntad elección basados en la conducta son herramientas importan-
para llevar a cabo planes que consideran que influyen en su tes para ayudar a evitar los escollos frecuentes al tomar una
propio interés. En tales situaciones la gente trata de colocar decisión.
fuera de su alcance alternativas tentadoras pero de menor
importancia.

pREgUNtaS dE REpaSo
1. Suponga que usted es el propietario de un pequeño negocio y 3. Distinga entre 1) la mejor decisión y 2) la decisión que condu-
que se le pregunta cuánto es lo máximo que estaría dispuesto ce al mejor resultado posible.
a pagar por asistir a un curso sobre la teoría tradicional de elec- 4. ¿Tiene algo de irracional dar menor peso a las ganancias que a
ción racional. ¿En qué caso sería mayor lo que estaría dispues- las pérdidas?
to a pagar: 1) si se supiera que la gente siempre se comporta 5. La política en una escuela es castigar a los alumnos por llegar
estrictamente de acuerdo con las predicciones de la teoría de tarde, mientras en otra, idéntica en todo lo demás, la política
elección racional, o 2) si se supiera que el comportamiento de correspondiente es premiar a los alumnos por llegar a tiempo.
la gente, incluyendo el suyo, suele apartarse sistemáticamente Si la efectividad se midiera por la conducta al día siguiente del
de las predicciones de la teoría de elección racional? castigo o del premio, ¿cuál de estas dos políticas parecería ser
2. ¿Por qué es racional tomar decisiones sin tener información más efectiva? ¿Se trata de un buen estándar de efectividad?
completa?

pRoblEMaS
1. Suponga que su felicidad está dada por una función de valor de Kahneman y Tversky como la que se
muestra en el diagrama.

Valor

V(G)

Pérdidas Ganancias

V(L)

08_CHAPTER 8.indd 257 3/6/09 7:57:47 PM


258 Capítulo 8 limitaCiones Cognitivas y Comportamiento del Consumidor

Usted ha decidido dar el efecto más favorable a las distintas combinaciones de acontecimientos que
ocurren en su vida. Dados cada uno de los pares siguientes de eventos, ¿sería usted más feliz si conside-
ra los efectos de los mismos por separado o si primero los combinara y después considerara el efecto
neto?
a) Una ganancia de 500 dólares y una pérdida de 50 dólares.
b) Una ganancia de 50 dólares y una pérdida de 500 dólares.
c) Una ganancia de 500 dólares y una ganancia de 600 dólares.
d) Una ganancia de 500 dólares y una pérdida de 600 dólares.
2. Sears Roebuck lo contrata a usted como asesor de mercadotecnia para saber cómo vender su nuevo
vehículo todo terreno. De acuerdo con el material que se vio en este capítulo, sugiera dos estrategias
de mercadotecnia para Sears.
3. Dé dos ejemplos de cómo la formulación de las alternativas tiende a producir efectos sistemáticos en
las elecciones de las personas.
4. Estudios han demostrado que en la ciudad de Nueva York la tasa de delincuencia en el Metro baja al
año siguiente de un aumento de los patrullajes de la policía. ¿Indica este patrón que el aumento de los
patrullajes sea la causa de la reducción de la delincuencia?
5. Clairborne es gastrónomo. Él nunca visita un restaurante por segunda vez a menos que la prime-
ra vez se le haya servido una comida excelente. Clairborne está desconcertado por lo raro que es
que, en la segunda ocasión, la calidad de los alimentos sea tan buena como la primera. ¿Debe sor-
prenderle?
6. Dalgliesh, el detective, cree conocer bien la naturaleza humana. En cuidadosos exámenes se ha des-
cubierto que 80% de las veces tiene razón cuando dice que un sospechoso, elegido al azar, miente.
Dalgliesh dice que Jones miente. El experto detector de mentiras, que acierta 100% de las veces,
afirma que 40% de los sujetos entrevistados por Dalgliesh dicen la verdad. ¿Cuál es la probabilidad de
que Jones esté mintiendo?
7. Un testigo asegura que el taxi que atropelló e hirió a Smith en un oscuro callejón era verde. En la
investigación, el abogado de la empresa de taxis encuentra que, en un callejón oscuro, los testigos
reconocen el color correcto de los taxis 80% de las veces. En la ciudad hay dos empresas de taxis, los
Verdes y los Azules. Los Verdes operan 15% de los taxis locales. La ley dice que la empresa Verde es la
responsable del daño causado a Smith si y sólo si la probabilidad de que lo haya causado es mayor que
0.5. ¿Es la empresa Verde la responsable? Explique.
8. Su agente de viajes le llamó la semana pasada para decirle que había encontrado una excelente tarifa
de 667 dólares para su vuelo al Reino Unido a finales del mes. La tarifa era casi 400 dólares inferior a
la usual. Usted le pide que haga una reservación de inmediato y le cuenta a todos sus compañeros del
departamento de la gran oferta que ha encontrado. Una hora más tarde su agente de viajes le llama
y le dice que en British Airways cometieron un error y que la tarifa de oferta no existe, pero que hará
todo lo posible para ayudarle. Unos días más tarde su agente de viajes encuentra un mayorista de
boletos que puede hacer una reservación en el mismo vuelo de British Airways a un precio de 708,
una cantidad aún bastante inferior a lo que al inicio había esperado pagar. Esta vez no presume ante
sus compañeros del departamento de su oferta. ¿Cómo puede emplearse el material visto en este
capítulo para entender su comportamiento?
9. Al planear sus próximas vacaciones usted decide entre dos opciones, dos paquetes que le ha ofrecido
su agente de viajes: una semana en Hawai al precio de 1 200 dólares o una semana en Cancún por 900
dólares. Cualquiera de ellas le da a usted lo mismo. En el periódico, en la sección de viajes, encuentra
una semana en Hawai, con un alojamiento idéntico al que le ha ofrecido su agente de viajes, a un
precio de 1 300 dólares. De acuerdo con la teoría de elección racional, ¿debe influir la información del
periódico sobre sus planes de vacaciones? Explique.
10. Mary estaría dispuesta a cruzar la ciudad para aprovechar una rebaja de 40% en una blusa de 40
dólares, pero no para aprovechar una rebaja de 10% en un estéreo de 1 000 dólares. Suponga que su
otra posibilidad sea comprar estos productos al precio normal en la tienda departamental cerca de su
casa, ¿es racional su comportamiento?
11. Hal no puede decidir entre dos raquetas de tenis, A y B. Como se muestra en el diagrama, B pro-
porciona más fuerza pero menos control que A. De acuerdo con el modelo de la elección racional,
¿qué influencia tendría una tercera posibilidad —la raqueta C— sobre la decisión de Hal? Si Hal se

08_CHAPTER 8.indd 258 3/6/09 7:57:47 PM


respuestas a los ejerCiCios del Capítulo 259

Fuerza

C B

Control

comporta como la mayoría de las personas al tomar una decisión en esta situación, ¿qué importancia
tendrá adicionar C a esta elección?
12. En otoño, Robinson Crusoe coloca 50 cocos de su cosecha dentro de una cueva, justo antes de que
una familia de osos llegue a hibernar. Por lo tanto, no podrá sacar los cocos hasta que los osos salgan
de la cueva en primavera. Los cocos se echan a perder a la misma velocidad sin importar dónde se
almacenen; sin embargo, Robinson Crusoe hace esto todos los años. ¿Por qué?

RESpUEStaS a loS EjERcicioS dEl capítUlo


8.1. Si hay cuatro veces más vendedores que bibliotecarios, entonces habrá 80 vendedores por cada 20
bibliotecarios. De los 80 vendedores, 20%, es decir 18, serán tímidos. Por lo tanto, la posibilidad de
que una persona tímida sea bibliotecaria es 18/(18 + 16) = 0.53.

08_CHAPTER 8.indd 259 3/6/09 7:57:48 PM


08_CHAPTER 8.indd 260 3/6/09 7:57:48 PM
Pa rt e

LLAA TEORÍA
3
O F E RTADEYLALEMPRESA
A D E M A YN DA
LA
ESTRUCTURA DE MERCADO
La teoría económica de la empresa supone que el objetivo principal de una
compañía es maximizar ganancias. esto requiere la expansión de la producción
siempre que el beneficio de hacerlo exceda al costo. el objetivo de los dos
primeros capítulos de la parte tres es desarrollar el aspecto de los costos de
este cálculo. el capítulo 9 comienza con la teoría de la producción, que muestra
cómo el trabajo, el capital y otros factores se combinan para originarla. Median-
te esta teoría, el capítulo 10 describe cómo varían los costos de la empresa
con la cantidad de producción que genera.
Los siguientes tres capítulos analizan el beneficio de la empresa de acuer-
do con cuatro formas diferentes de estructura de mercado. el capítulo 11
contempla a la compañía en competencia perfecta, para la cual el beneficio
de vender una unidad adicional de producto es exactamente igual a su precio.
el capítulo 12 la examina como monopolio o proveedor exclusivo de un bien
para el cual no hay sustitutos cercanos. en este caso, el beneficio de vender
una unidad adicional de producto en general es menor que su precio porque
para expandir su producción, aquél debe reducirse para las ventas existentes.
el capítulo 13 examina dos formas intermedias de la estructura de mercado:
la competencia monopolística y el oligopolio. al tomar decisiones acerca de
los niveles de producción, las empresas en competencia monopolística se
comportan como monopolios. en contraste, el oligopolio debe considerar las
respuestas estratégicas de sus rivales cuando calcula los beneficios de expandir
su producción.

261

09_CHAPTER 9.indd 261 3/6/09 8:45:53 PM


09_CHAPTER 9.indd 262 3/6/09 8:45:53 PM
CaPítuLo

9
PRODUCCIÓN

M
ucha gente piensa en la producción como un proceso altamente estructurado, a menudo
mecánico por el cual las materias primas se transforman en bienes terminados; y sin duda,
gran parte de la producción —como los ladrillos que coloca un albañil para las paredes de
una casa— es más o menos de este tipo. Sin embargo, los economistas enfatizan que es también un
concepto más general que abarca múltiples actividades que de manera ordinaria no se consideran
como tales. Se le define como cualquier actividad que cree utilidad presente o futura.
Así, el simple acto de contar un chiste constituye producción. Woody Allen (figura 9.1) relata
la historia del hombre que se queja con su analista de que su hermano piensa que es un pollo. “¿Por
qué no le dice que no es un pollo?”, pregunta el analista, a lo que el hombre responde: “No puedo,
necesito los huevos.” Una vez que se cuenta un chiste, no deja más rastro tangible que un recuerdo
agradable. Pero de acuerdo con la definición económica de la producción, Woody Allen es tanto un
trabajador de la producción como el artesano cuyos cincel y torno moldean un tronco de madera de
fresno para obtener un bate de beisbol Louisville Slugger. La persona que entrega un telegrama tam-
bién está produciendo; de la misma manera lo está el doctor que inyecta a un niño la vacuna contra
el tétanos; el abogado que redacta un testamento; los encargados de recoger la basura los miércoles
por la mañana; el empleado de correos que entrega una declaración de impuestos a la oficina de
Hacienda, e incluso los economistas que escriben sobre la producción.

VISIÓN PRELIMINAR DEL CAPÍTULO


En el estudio de la elección del consumidor durante los capítulos precedentes, se consideraba como
un hecho un menú existente de bienes y servicios. Pero, ¿de dónde provienen éstos? En el presente
capítulo se verá que su generación implica un proceso de decisión muy similar al que se examinó en
capítulos anteriores. Mientras que nuestro enfoque en ellos se centraba en las decisiones económicas
263

09_CHAPTER 9.indd 263 3/6/09 8:45:54 PM


264 CaPítuLo 9 ProduCCión

FIGURA 9.1
Un trabajador de la
producción
© associated Press

que subyacen en la parte de la demanda de la relación del mercado, el enfoque en los siete capítulos
siguientes está en las decisiones económicas que subyacen en el tema del suministro.
En este capítulo se describen las posibilidades de la producción disponibles para un estado de-
terminado de dotaciones de tecnología y recursos. Se desea saber cómo varía la producción con la
aplicación de factores productivos tanto a corto como a largo plazos. Las respuestas a estas preguntas
sentará las bases para describir, en el capítulo siguiente, cómo las empresas eligen entre métodos
alternativos técnicamente factibles para obtener un nivel dado de producción.

LA RELACIÓN INSUMO*-PRODUCTO,
O LA FUNCIÓN DE PRODUCCIÓN
Hay varias maneras de definir la producción. Una que se mencionó antes, es que se trata de cualquier
actividad que crea utilidad actual o futura. También puede describirse de forma equivalente como un
proceso que transforma los insumos (factores de producción) en productos. (Las dos descripciones
son equivalentes porque el producto es algo que crea utilidad presente o futura.) Entre los insumos
para la producción, los economistas han incluido tradicionalmente la tierra, el trabajo, el capital y la
categoría más etérea llamada espíritu emprendedor.1 Ha llegado a ser cada vez más común agregar a
esta lista factores tales como el conocimiento o la tecnología, la organización y la energía.
función de producción la Una función de producción es la relación mediante la cual los insumos se combinan para ob-
relación que describe cómo tener productos. De manera esquemática se le puede representar como la caja de la figura 9.2. Los
los insumos como el capital y insumos se introducen en ésta y el producto se obtiene de la misma. La caja incorpora en forma
el trabajo se transforman en implícita el estado actual de la tecnología, que ha ido mejorando de manera constante con el paso del
productos. tiempo. Así, de una combinación determinada de insumos productivos se obtendrá un número más
grande de coches con la tecnología de hoy que con la de 1970.
Una función de producción también se puede considerar como una receta de cocina. Enumera
los ingredientes y dice, por ejemplo, cuántos pasteles obtendrá si manipula los ingredientes de cierta
manera.2

1
El “espíritu emprendedor” se define como “el proceso de organizar, manejar y asumir responsabilidad para una empresa de nego-
cios” (Diccionario Random House College). Así, un emprendedor es, por definición, un tomador de riesgos.
2
En algunas recetas los ingredientes deben mezclarse en proporciones fijas. Otras recetas permiten la sustitución de ingredientes,
como en el caso de una receta de panqués que permite que la leche y el aceite sean reemplazados con huevos. Las funciones de
producción pueden ser de alguno de estos dos tipos.
* En algunos países de habla hispana se denomina como factor al término insumo.

09_CHAPTER 9.indd 264 3/6/09 8:45:55 PM


La reLaCión insuMo-ProduCto, o La funCión de ProduCCión 265

FIGURA 9.2
Insumos La función de
(tierra, trabajo, capital, etcétera) producción
La función de producción
transforma insumos como
tierra, trabajo, capital y
habilidad emprendedora
en productos. La caja en el
diagrama encarna el estado
actual del conocimiento
tecnológico. Como éste
Función de producción se ha acumulado con
el tiempo, se obtiene
más producción a partir
de cierta combinación
Producción de insumos hoy en
(automóviles, vacuna comparación con la que
contra la polio,
se habría obtenido en el
comidas caseras,
transmisiones de radio
pasado.
FM, etcétera)

Otra manera de describir la función de producción es ponerla en la forma de una ecuación ma-
temática. Considere un proceso de producción que emplee dos insumos, el capital (K) y el trabajo
(L), para producir comidas (Q). La relación entre K, L y Q se puede expresar como

Q = F(K, L), (9.1)

donde F es una función matemática que resume el proceso que se representa en la figura 9.2. No es
más que una regla simple que dice cuánto se obtiene de Q cuando se emplean cantidades específicas
de K y L. A manera de ilustración, suponga que la función de producción para las comidas está dada
por F(K, L) = 2KL, donde K se mide en horas-equipo por semana,3 L se mide en horas-persona por
semana y la producción se mide en comidas por semana. Por ejemplo, con 2 horas-equipo/sema-
na, combinadas con 3 horas-persona/semana, se obtendrían 2(2)(3) = 12 comidas/semana con esta
función de producción particular. La relación entre K, L y la producción semanal de comidas para la
función Q = 2KL se resume en la tabla 9.1.

TABLA 9.1
La función de producción Q = 2KL

Trabajo
(horas-persona/semana)
1 2 3 4 5 Las entradas en la
Capital 1 2 4 6 8 10 tabla representan
(horas-equipo/semana) 2 4 8 12 16 20 producción, medida en
comidas por semana,
3 6 12 18 24 30
y se calcula con la
4 8 16 24 32 40 fórmula Q = 2KL.
5 10 20 30 40 50

3
Aquí, 1 hora-sartén de freír/semana es 1 sartén de freír usado por una hora durante el curso de una semana. Así, una sartén de freír
que se usa durante 8 horas/día a lo largo de una semana de trabajo de 5 días constituiría 450 horas-sartén de freír/semana de insumo
de capital.

09_CHAPTER 9.indd 265 3/6/09 8:45:56 PM


266 CaPítuLo 9 ProduCCión

PRODUCTOS INTERMEDIOS
El capital (representado, por ejemplo, en forma de estufas y sartenes para freír) y el trabajo (por
ejemplo, los servicios de un chef ) con claridad son insuficientes por sí mismos para producir comi-
das. Los comestibles crudos también son necesarios. El proceso de producción representado por la
ecuación 9.1 transforma los comestibles crudos en el producto terminado llamado comida. En este
proceso los comestibles son los productos intermedios, tratados por muchos economistas como un
insumo más. Por simplicidad, en los ejemplos que se discuten en este capítulo se pasa por alto a los
productos intermedios. Pero éstos podrían incorporarse en todos los casos sin cambiar ninguna de
las conclusiones esenciales.

INSUMOS FIJOS Y VARIABLES


La función de producción dice cómo variará el producto si alguno o todos los insumos varían. En la
práctica hay muchos procesos de producción en los cuales las cantidades de por lo menos algunos
insumos no se pueden alterar de manera rápida. La transmisión por radio de FM de música clásica
es un proceso de este tipo. Para desarrollarlo es necesario equipo electrónico complejo, una biblio-
teca musical y una torre grande de transmisión. Las grabaciones y los discos compactos se pueden
comprar en cuestión de horas. Pero puede tomar semanas adquirir el equipo necesario para lanzar
una nueva estación, y meses o incluso años para comprar un lugar conveniente y construir una torre
nueva de transmisión.
largo plazo el periodo más El término a largo plazo para un proceso de producción específico se define como el periodo
corto que se requiere para más corto requerido para alterar las cantidades de todos y cada uno de los insumos. El término a cor-
alterar las cantidades de todos to plazo, por el contrario, se define como el periodo durante el cual uno o más insumos no pueden
los insumos que se usan en un variar. Un insumo cuya cantidad se puede alterar en el corto plazo se llama insumo variable. Aquel
proceso de producción. cuya cantidad no puede alterarse —excepto quizás a un costo prohibitivo— dentro de un periodo
dado se llama insumo fijo. En el largo plazo todos los insumos son variables por definición. En el
corto plazo el periodo más
largo durante el cual al menos
ejemplo de la transmisión de música clásica los discos compactos son insumos variables en el corto
uno de los insumos que se usan plazo, pero la torre de transmisión es un insumo fijo. Si transcurre suficiente tiempo, sin embargo,
en un proceso de producción incluso ésta se convierte en un insumo variable. En algunas actividades de producción, como las de
no puede variar. un puesto de salchichas en la esquina de una calle, incluso el término a largo plazo no implica un
periodo prolongado. La siguiente sección comienza con una consideración de la producción a corto
insumo variable un insumo plazo, para luego pasar a la producción a largo plazo en la siguiente sección.
que puede variar a corto plazo.

insumo fijo un insumo que PRODUCCIÓN A CORTO PLAZO


no puede variar a corto plazo.
Considere de nuevo el proceso de producción descrito por Q = F(K, L) = 2KL, la función de produc-
ción simple de dos insumos descrita en la tabla 9.1. Y suponga que le interesa la producción a corto
plazo, que en este caso es un periodo en el cual el insumo de trabajo es variable pero el insumo de
capital es fijo, por ejemplo, en el valor K = K0 = 1. Si el capital se mantiene constante, la producción se
convierte, en efecto, en una función exclusiva del insumo variable, trabajo: F(K, L) = 2K0L = 2L. Esto
significa que es posible trazar la función de producción en un diagrama bidimensional, como en la
figura 9.3a. Para esta F(K, L) específica, la función de producción a corto plazo es una línea recta que
pasa por el origen cuya pendiente es dos veces el valor fijo de K: así, ∆Q/∆L = 2K0. En la figura 9.3b,
observe que la producción a corto plazo rota hacia arriba a F(K1, L) = 6L cuando K asciende a K1 = 3.

EJERCICIO 9.1
__ __
Grafique la función de producción a corto plazo para F(K, L) = √ K √ L cuando K está fijo en
K0 = 4.

Como lo vio en el ejercicio 9.1, los gráficos de las funciones de producción a corto plazo no serán
siempre líneas rectas. La función de producción a corto plazo de la figura 9.4 tiene varias propiedades
que por lo común se encuentran en las funciones de producción que se observan en la práctica. Pri-

09_CHAPTER 9.indd 266 3/6/09 8:45:56 PM


ProduCCión a Corto PLazo 267

FIGURA 9.3
Q (comidas/semana) Q (comidas/semana)
Una función de
Q = F(K1, L) = 6L producción específica
a corto plazo
Q = F(K0, L) = 2L Q = F(K0, L) = 2L La parte a) muestra la
6 6 función de producción,
Q = 2KL, con K fijo en
K0 = 1. La parte b) muestra
cómo se desplaza la
función de producción a
corto plazo cuando K se
2 incrementa a K1 = 3.

L (horas-persona/ L (horas-persona/
0 1 3 semana) 0 1 3 semana)

a) b)

FIGURA 9.4
Q (comidas/semana)
Otra función de
producción a corto
100 plazo
La forma curvilínea que
86 se muestra aquí es común
81
para muchas funciones
72 de producción a corto
plazo. La producción
58 Q = F(K0, L)
crece inicialmente a una
tasa creciente a medida
43 que aumenta el trabajo.
Más allá de L = 4, la
producción crece a una
27 tasa que disminuye con los
incrementos en el trabajo.
14
4
L (horas-persona
0 1 2 3 4 5 6 7 8 9 10 /semana)

mero, pasa por el origen, lo que quiere decir que no se obtiene ningún producto si no se utiliza algún
insumo variable. Segundo, al principio la adición de insumos variables aumenta la producción a una
tasa creciente: el paso de 1 a 2 unidades de trabajo suministra 10 unidades adicionales de producto,
mientras que con el paso de 2 a 3 unidades de trabajo se obtienen 13 unidades adicionales. Por últi-
mo, la función que se muestra en la figura 9.4 tiene la propiedad de que más allá de cierto punto (L =
4 en el diagrama), las unidades adicionales del insumo variable dan lugar a incrementos cada vez más
pequeños de producción. Así, el paso de 5 a 6 unidades de trabajo suministra 14 unidades adicionales
de producto, mientras que del paso de 6 a 7 unidades de trabajo se obtienen nada más 9. Para algunas
funciones el nivel de producción puede reducirse a partir de cierto punto con unidades adicionales
del insumo variable, como sucede aquí para L > 8. Con una cantidad limitada de capital con el cual
trabajar, a la larga los empleados adicionales pueden comenzar a interferir entre sí.
La propiedad de que la producción inicialmente crece a una tasa creciente puede provenir de
los beneficios de la división de tareas y de la especialización del trabajo. Con un empleado, la misma
persona debe hacer todas las tareas, mientras que con dos o más empleados, las labores se dividen y
los trabajadores pueden realizar mejor las tareas asignadas. (Se aplica una lógica similar a la especiali-
zación en una tarea dentro de cualquier periodo.)
La propiedad final que se observa en la función de producción a corto plazo de la figura 9.4,
según la cual más allá de cierto punto la producción disminuye a una tasa decreciente respecto a

09_CHAPTER 9.indd 267 3/6/09 8:45:58 PM


268 CaPítuLo 9 ProduCCión

ley de los rendimientos sucesivos incrementos del insumo variable, se conoce como ley de los rendimientos decrecientes.
decrecientes si otros insumos Y aunque tampoco es una propiedad universal de las funciones de producción a corto plazo, es muy
están fijos, el incremento en común. Esta ley es un fenómeno a corto plazo. De manera formal, puede enunciarse como sigue:
la producción a partir de un
incremento en el insumo A medida que cantidades iguales de un insumo variable se agregan sucesivamente mien-
variable finalmente debe tras el resto de los insumos se mantienen fijos, los incrementos resultantes de la produc-
disminuir. ción acabarán disminuyendo.

¿Por qué la gente de todo el


mundo no puede ser alimentada
a partir de la cantidad de grano
cultivada en una sola maceta?
EL NATURALISTA La ley de rendimientos decre-
ECONÓMICO cientes sugiere que sin importar
9.1 cuánto trabajo, fertilizante, agua,
semilla, equipo de capital y otros
insumos se hayan utilizado, sólo
una cantidad limitada de grano po-
dría cultivarse en una sola maceta.
Con el insumo de tierra fijo en un
nivel tan bajo, los incrementos en
otros insumos dejarían rápidamen-
te de tener cualquier efecto sobre
la producción total.

El economista británico Thomas Malthus argumentó en 1798, con base en la lógica del Natura-
lista económico 9.1, que la ley de rendimientos decrecientes implicaba a largo plazo la miseria para la
raza humana. La dificultad es que la tierra agrícola está fija y, más allá de cierto punto, la aplicación
de trabajo adicional supondrá aumentos cada vez más pequeños en la producción de alimentos. El
resultado inevitable, como explicó Malthus, es que el crecimiento de la población reducirá el consu-
mo promedio de alimentos a nivel de hambruna.
Todavía falta comprobar si la predicción de Malthus se cumplirá en el futuro. Pero él nunca ha-
bría imaginado que la producción de alimentos per cápita crecería más de 20 veces durante los dos si-
guientes siglos. Sin embargo, observe con cuidado que la experiencia de los últimos 200 años no con-
tradice la ley de rendimientos crecientes. Lo que Malthus no previó fue el crecimiento explosivo de la
tecnología agrícola que ha rebasado con mucho el efecto de una provisión fija de tierra. No obstante,
permanece la lógica despiadada de la observación de Malthus. No importa qué tan avanzada sea la
tecnología, ya que si la población continúa creciendo, es sólo cuestión de tiempo que los límites de
la tierra cultivable causen escasez persistente de alimentos.
La población del mundo ha crecido con tal rapidez durante los años que han pasado desde que
Malthus escribió, que sólo en los últimos 50 años se duplicó. ¿Estamos condenados al hambre? Quizá
no. Como el finado economista Herbert Stein una vez comentó en forma célebre: “Si algo no puede
continuar para siempre, no lo hará.” Y, de hecho, los especialistas en población predicen ahora que
la población de la Tierra alcanzará un pico hacia el año 2070 y después comenzará a declinar.4 Si no
nos destruimos en ese lapso, entonces hay una buena oportunidad de escapar del destino calamitoso
que Malthus predijo.

4
Véase Wolfgang Lutz, Warren Sanderson y Sergei Sherbov, “The End of World Population Growth”, en Nature, 412, 2 de agosto de
2001: pp. 543-545.

09_CHAPTER 9.indd 268 3/6/09 8:45:59 PM


ProduCCión a Corto PLazo 269

Las mejoras tecnológicas en la producción se representan de manera gráfica por un despla-


zamiento ascendente en la función de producción. En la figura 9.5, por ejemplo, las curvas deno-
minadas F1 y F2 se utilizan para representar las funciones de producción agrícola en 1808 y 2008,
respectivamente. La ley de rendimientos decrecientes se aplica a cada una de estas curvas y aun así
el crecimiento en la producción de alimentos se ha mantenido a la par con el aumento del insumo
trabajo durante el periodo mostrado.

FIGURA 9.5
El efecto del progreso
tecnológico en la
producción de alimentos
F1 representa a la función
Q de producción para
alimentos en el año
Q2008 F2(K, L) 1808. F2 representa a la
correspondiente función
para 2008. el efecto del
F1(K, L)
progreso tecnológico
Q1808 en la producción de
alimentos causa que F2
esté situado arriba de
F1. aun cuando la ley de
rendimientos decrecientes
L es aplicable tanto a F1 como
L1808 L2008 a F2, el crecimiento de la
producción de alimentos
entre 1808 y 2008 se ha
mantenido más que a la par
con el crecimiento de los
insumos de trabajo para el
mismo periodo.

PRODUCTO TOTAL, MARGINAL Y PROMEDIO


Las funciones de producción a corto plazo, como las que aparecen en las figuras 9.4 y 9.5, se denomi-
nan a menudo curvas de producto total. Relacionan la cantidad total de producción con la cantidad curva de producto total una
del insumo variable. También de interés en muchas aplicaciones es el producto marginal de un insumo curva que muestra la cantidad
variable. Se define como el cambio en el producto total que ocurre en respuesta a un cambio unitario en el de producto como una función
insumo variable (el resto de los insumos se mantienen fijos). Un administrador de empresas que intenta de la cantidad de insumo
decidir si emplea o despide a otro trabajador tiene un interés obvio en saber cuál es el producto variable.
marginal del trabajo. producto marginal cambio
De manera más formal, si ∆L representa un cambio pequeño en el insumo variable y ∆Q repre- en el producto total debido a
senta el cambio resultante en la producción, entonces el producto marginal de L, representado PML, un cambio de una unidad en el
se define como insumo variable.
∆Q
PM L = . (9.2)
∆L
Desde el punto de vista de la geometría, el producto marginal en cualquier punto es tan sólo la
pendiente de la curva de producto total en dicho punto, como se muestra en la parte superior de la
figura 9.6.5 Por ejemplo, el producto marginal del trabajo cuando L = 2 es PML=2 = 12. Asimismo,
PML=4 = 16 y PML=7 = 6 para la curva de producto total de la figura 9.6. Observe, por último, que PML
es negativo para valores de L mayores que 8.
La curva de producto marginal se traza en la parte baja de la figura 9.6. Observe que primero
asciende, alcanza un máximo en L = 4 y entonces desciende, finalmente se hace negativa para valo-

5
La definición formal del producto marginal de un insumo variable está dada por PM(L) = ∂F(K, L)/∂L.

09_CHAPTER 9.indd 269 3/6/09 8:46:01 PM


270 CaPítuLo 9 ProduCCión

FIGURA 9.6
El producto marginal de Q (comidas/semana)
un insumo variable
Para cualquier punto, el PML = 6
90
producto marginal del
trabajo, PML, es la pendiente 80 ΔQ = 6
de la curva de producto ΔL = 1
70
total en ese punto (parte
superior). Para la función de 60 Q = F(K0, L)
producción que se muestra PML = 16
50
en la parte superior, la
curva de producto marginal 40 ΔQ = 16
(parte inferior) inicialmente
aumenta a medida que se 30 ΔL = 1
PML = 12
incrementa el trabajo. 20
sin embargo, más allá de ΔQ = 12
L = 4, el producto marginal 10
del trabajo disminuye a ΔL = 1
L
medida que aumenta el 0 1 2 3 4 5 6 7 8 9 10
trabajo. Para L > 8 la curva PM (comidas/persona-hora) Región de rendimientos decrecientes
de producto total declina
con L, lo que significa que 16
el producto marginal del
12
trabajo es negativo en esa
región. 6
L (horas-persona/
1 2 3 4 5 6 7 8 9 10 semana)

PML

res de L mayores que 8. Observe también que el punto máximo de la curva de producto marginal
corresponde al punto de inflexión en la curva de producto total, el punto donde su curvatura cambia
de convexa (aumentando con una tasa creciente) a cóncava (aumentando con una tasa decreciente).
Note también que la curva de producto marginal llega a cero en el valor de L en el cual la curva de
producto total alcanza un máximo.
Tal como se verá con mayor detalle en capítulos posteriores, la importancia del concepto de
producto marginal radica en el hecho de que las decisiones sobre la dirección de una empresa se
presentan de la manera más natural en forma de resoluciones referentes a cambios. ¿Se debe contratar
a otro ingeniero o a otro contador? ¿Se debe reducir el tamaño del personal de mantenimiento? ¿Se
debe adquirir otra copiadora? ¿Se debe alquilar otro camión de reparto?
Para contestar a estas preguntas de manera inteligente se debe comparar el beneficio del cambio
en cuestión con su costo; y como se verá, el concepto de producto marginal desempeña un papel
esencial en el cálculo de los beneficios cuando se altera el nivel de un insumo productivo. Al examinar
la figura 9.6 es posible identificar un intervalo de valores del insumo variable que un administrador
racional nunca emplearía. En particular, siempre que el costo del trabajo sea positivo, dicho adminis-
trador no deseará emplear el insumo variable en la región donde su producto marginal es negativo
(L > 8 en la figura 9.6). De forma equivalente, nunca emplearía un insumo variable más allá del
punto donde la curva de producto total alcanza su valor máximo (donde PML = 0).

EJERCICIO 9.2
¿Cuál es el producto marginal del trabajo cuando L = 3 en la función de producción a corto
plazo que se muestra en la figura 9.3a? ¿Y si L = 1? ¿Exhibe esta función de producción a corto
plazo rendimientos decrecientes para el trabajo?

09_CHAPTER 9.indd 270 3/6/09 8:46:02 PM


ProduCCión a Corto PLazo 271

El producto promedio de un insumo variable se define como el producto total dividido entre la producto promedio
cantidad de dicho insumo. Se le denota como PPL y está dado por producción total dividida entre
la cantidad del insumo variable.
Q
PPL = . (9.3)
L
Cuando el insumo variable es el trabajo, el producto promedio también se llama productividad del
trabajo.
Geométricamente, el producto promedio es la pendiente de la línea que une el origen con el
punto correspondiente en la curva de producto total. Tres líneas de este tipo, R1, R2 y R3, se trazan
para la curva de producto total que se muestra en la parte superior de la figura 9.7. El producto pro-
14 = 7. Observe que R interseca la curva de producto
medio para L = 2 es la pendiente de R1, que es ––2 2
total en dos lugares: primero, directo sobre L = 4, y luego justo sobre L = 8. Por consiguiente, el
producto promedio para estos dos valores de L será el mismo, es decir, la pendiente de R2, que es ––43
86 4
= ––8 = 10.75. R3 interseca a la curva de producto total en solamente un punto, directamente sobre L
= 6. El producto promedio para L = 6 es entonces la pendiente de R3, –– 72 = 12.
6

FIGURA 9.7
Q (comidas/semana) R2 Curvas de producto
R3 total, marginal y
86 promedio
el producto promedio
72 para cualquier punto de la
curva de producto total
Q = F(K0, L) es la pendiente del rayo
para ese punto. Para la
curva de producto total
43 que se muestra en la parte
superior, PPL asciende con
L = 6, luego declina.
R1 Para L = 6, PML = PPL. Para
14 cualquier L < 6, PML > PPL
y para cualquier L > 6, PML
L < PPL.
1 2 3 4 5 6 7 8 9 10

PM, PP (comidas/persona-hora)

16
12
10.75
PPL

L (horas-persona/
1 2 3 4 5 6 7 8 9 10 semana)

PML

EJERCICIO 9.3
Para la función de producción a corto plazo de la figura 9.3a, ¿cuál es el producto promedio del
trabajo para L = 3? ¿Y para L = 1? ¿Cómo es el producto promedio comparado con el producto
marginal en estos puntos?

09_CHAPTER 9.indd 271 3/6/09 8:46:04 PM


272 CaPítuLo 9 ProduCCión

LAS RELACIONES ENTRE CURVAS DE PRODUCTO


TOTAL, MARGINAL Y PROMEDIO
Debido a la manera en que se definen los productos total, marginal y promedio, existen relaciones
sistemáticas entre ellos. La parte superior en la figura 9.7 muestra una curva de producto total y tres
de los rayos cuyas pendientes definen el producto promedio del insumo variable. El más inclina-
do de los tres rayos, R3, es tangente a la curva de producto total en L = 6. Su pendiente, –– 72 = 12, es
6
el producto promedio del trabajo en L = 6. El producto marginal del trabajo en L = 6 se define como
la pendiente de la curva de producto total en L = 6, que resulta ser exactamente la pendiente de R3,
porque R3 es tangente a la curva de producto total. Así PPL = 6 = PML = 6, como se muestra en la
parte inferior por el hecho de que la curva PPL interseca a la curva PML en L = 6.
Para valores de L menores que 6, observe en la parte superior de la figura 9.7 que la pendiente
de la curva de producto total es mayor que la pendiente del rayo al punto correspondiente. Así, para
L < 6, PML > PPL, como se refleja en la parte inferior. Observe también en la parte superior que pa-
ra valores de L mayores que 6, la pendiente de la curva de producto total es menor que la pendiente
del rayo al punto correspondiente. Esto significa que para L > 6, se tiene PPL > PML, como se mues-
tra en la parte inferior de la figura 9.7.
Por último, observe en la figura 9.7 que para valores muy pequeños de L la pendiente del rayo a
la curva de producto total se hace indistinguible de la pendiente de la curva del producto total. Esto
indica que para L = 0, los productos promedio y marginal son iguales, lo que se refleja, en la parte
inferior de la figura 9.7, por el hecho de que ambas curvas parten del mismo punto.6
La relación entre las curvas de producto marginal y promedio puede resumirse como sigue:
cuando la curva de producto marginal está situada sobre la curva de producto promedio, esta última debe ser
ascendente; y cuando la curva de producto marginal está situada debajo de la curva de producto promedio,
ésta debe descender. Las dos curvas se intersecan en el valor máximo de la curva de producto promedio. Un
momento de reflexión sobre las definiciones de las dos curvas aclara la base intuitiva de esta relación.
Si la contribución a la producción de una unidad adicional del insumo variable excede a la contribu-
ción promedio de los insumos variables usados hasta el momento, la contribución promedio debe
ascender. Este efecto es análogo a lo que ocurre cuando un estudiante con un promedio general de
3.8 ingresa a una fraternidad cuyos miembros tienen un promedio general de 2.2: la presencia del
nuevo miembro hace que se eleve el promedio general del grupo. A la inversa, la adición de un insu-
mo variable cuyo producto marginal sea menor que el producto promedio de las unidades existentes
es como la adición de un nuevo miembro de la fraternidad con un promedio general de 1.7. Aquí, el
efecto es que desciende el promedio existente.7

EJERCICIO 9.4
Considere un proceso de producción a corto plazo para el cual PPL=10 = 7 y PML=10 = 12. ¿Será
PPL=10.1 mayor o menor que PPL=10 para este proceso?

EL SIGNIFICADO PRÁCTICO DE LA DISTINCIÓN


MARGINAL-PROMEDIO
La distinción entre los productos promedio y marginal es de importancia central para cualquier per-
sona que deba asignar un recurso escaso entre dos o más actividades productivas. La pregunta especí-

6
Para la función de producción que se muestra, ese punto resulta ser el origen, pero en general no tiene que ser así.
7
Desde el punto de vista matemático, el resultado de que MP interseque a AP en el valor máximo de AP se puede demostrar si se
observa que la condición necesaria para un máximo de AP es que su primera derivada parcial con respecto a L sea cero:

∂(Q/L)/∂L = [L(∂Q/∂L) – Q]/L2 = 0,

de lo que se concluye que ∂Q/∂L = Q/L.

09_CHAPTER 9.indd 272 3/6/09 8:46:05 PM


ProduCCión a Corto PLazo 273

fica es, ¿cómo debe asignarse el recurso para maximizar la producción total? Los siguientes ejemplos
aclaran los aspectos planteados por este problema y la regla general requerida para solucionarlo.
Repase el siguiente escenario con cuidado e intente contestar a la pregunta que viene al final:

Suponga que usted posee una flota pesquera que consiste en cierto número de barcos
y que puede enviarlos en cualquier cantidad que desee a cualquiera de dos extremos de
un lago muy ancho, este u oeste. De acuerdo con su asignación actual de barcos, los que
pescan en el extremo este regresan diario con 100 libras de pescado cada uno, mientras
que los del oeste regresan diariamente con 120 libras de pescado cada uno. Los bancos de
pescado en cada extremo del lago son del todo independientes y su producción actual se
puede sostener de manera indefinida. ¿Debe alterar su asignación actual de barcos?

La mayoría de las personas, en especial las que no han tenido un buen curso de microeconomía,
responden con confianza que la asignación actual debe alterarse. En específico, dicen que el dueño
de la flota pesquera debe enviar más barcos al lado oeste del lago. Sin embargo, como el siguiente
ejemplo lo ilustra, incluso una comprensión rudimentaria de la distinción entre los productos pro-
medio y marginal de un recurso productivo deja claro que esta respuesta no se justifica.

En el escenario de la flota pesquera apenas descrito, suponga que la relación entre el número EJEMPLO 9.1
de barcos enviados a cada extremo y la cantidad de libras pescadas por barco son los que se
resumen en la tabla 9.2. Suponga además que usted tiene cuatro barcos en su flota y que en
la actualidad dos pescan en el extremo este mientras los otros dos lo hacen en el extremo
oeste. (Observe que todas estas suposiciones son consistentes con los hechos que se esboza-
ron en el escenario.) ¿Debe mover uno de sus barcos del extremo este al extremo oeste?

TABLA 9.2
Producto promedio, producto total y producto marginal (lb/día) para dos áreas de pesca La captura promedio
por barco es constante
en 100 libras para cada
Extremo este Extremo oeste
uno de los barcos que
Número se mandan al extremo
de barcos PP PT PM PP PT PM este del lago. La captura
0 0 0 0 0 promedio por barco es
100 130
1 100 100 130 130 una función decreciente
100 110 del número de barcos
2 100 200 120 240 enviados al extremo
100 90
3 100 300 110 330 oeste.
100 70
4 100 400 100 400

A partir de las entradas de la tabla 9.2 se deduce que su producción total con la asignación actual es
de 440 libras de pescado por día (100 libras de cada uno de los dos barcos en el extremo este más,
120 de cada uno de los dos en el extremo oeste). Ahora suponga que transfiere un barco del extremo
este al extremo oeste, esto significa que ahora tiene tres barcos en el oeste y sólo uno en el este. De
las cifras de la tabla 9.2 se ve que su producción total será ahora de solamente 430 libras por día, o 10
libras diarias menos que con la asignación actual. Por lo tanto, no debe mover un barco adicional al
extremo oeste. Tampoco, de momento, deberá usted enviar ninguno de los barcos del extremo oeste
al extremo este. La pérdida de un barco del extremo oeste reduciría la pesca diaria total en ese ex-
tremo en 110 libras (la diferencia entre las 240 libras pescadas por dos barcos y las 130 obtenidas por

09_CHAPTER 9.indd 273 3/6/09 8:46:05 PM


274 CaPítuLo 9 ProduCCión

uno), que es más de las 100 libras adicionales que obtendría al tener un barco adicional en el extremo
este. La asignación actual de dos barcos a cada extremo es óptima.

El ejemplo 9.1 es una muestra de una clase importante de problemas en los cuales los geren-
tes deben decidir cómo asignar un insumo a varios procesos alternativos usados para producir un
producto específico. La regla general para asignar un insumo en forma eficiente en estos casos es destinar
la unidad siguiente de insumo a la actividad de producción en la que su producto marginal sea más alto. Esta
versión de la regla se aplica a los recursos, como los barcos, que no son perfectamente divisibles,
y también a los casos en los que el producto marginal de un recurso es siempre más alto en una
actividad que en otra.8 Para un recurso que sea perfectamente divisible y para las actividades para las
cuales el producto marginal del recurso no es siempre más alto en uno que en los otros, la regla es
asignar el recurso de modo que su producto marginal sea el mismo en cada actividad.
Sin embargo, mucha gente “soluciona” este tipo de problemas al asignar recursos a la actividad
con el producto promedio más alto o intentando igualar los productos promedio de las distintas acti-
vidades. La razón por la cual esta respuesta incorrecta a menudo tiene atractivo es que la gente suele
centrarse sólo en una parte del proceso de producción relevante. Si se envían nada más dos barcos al
extremo oeste, la captura promedio en ese extremo es 20 libras por día mayor que la pesca promedio
por barco en el extremo este. Pero observe que si envía un tercer barco al extremo oeste, su contribu-
ción a la cantidad total de peces capturados será de sólo 90 libras por día (la diferencia entre 330 libras
capturadas por tres barcos y 240 libras capturadas por dos). Lo que las personas en general pasan por
alto es que el tercer barco en el extremo oeste captura algunos de los peces que habrían sido pescados
de otra manera por los dos primeros.
Como ilustran las cifras en la tabla 9.2, el costo de oportunidad de enviar un tercer barco al
extremo oeste es 100 libras de peces que ya no serán capturados en el extremo este. Pero como ese
tercer barco agregará solamente 90 libras a la captura diaria en el extremo oeste, lo mejor que se
puede hacer es seguir enviando dos barcos a cada extremo del lago. El hecho de que cualquiera de
los dos barcos que en la actualidad pescan en el extremo este podría capturar 10 libras por día más
moviéndose al extremo oeste no es ninguna causa de inquietud para un dueño de flota pesquera que
entienda la diferencia entre los productos promedio y marginal.

EJERCICIO 9.5
Explique por qué no es posible concluir por fuerza que usted debe arrojar más bolas rápidas en
el siguiente escenario: usted es un lanzador de beisbol que hace dos tipos de tiros: bola rápida y
curva. El especialista en estadística del equipo le dice que a la tasa actual a la cual usted emplea
estos lanzamientos, los jugadores tienen un promedio de bateo de .275 contra su bola curva,
pero sólo de .200 contra su bola rápida. ¿Debe alterar su mezcla actual de tiros?

El ejemplo 9.1 produjo lo que los economistas llaman una solución interior, en la cual se utilizan
cada una de las actividades de producción. Pero no todos los problemas de esta clase tienen solucio-
nes interiores. Como el ejemplo siguiente lo aclarará, una actividad domina a veces a la otra.

EJEMPLO 9.2 Lo mismo que la flota pesquera del ejemplo 9.1, excepto que ahora el producto marginal de
cada barco enviado al extremo oeste del lago es igual a 120 libras/día.

La diferencia entre este ejemplo y el 9.1 es que esta vez no hay descenso de la tasa de captura de los
peces a medida que se envían más barcos al extremo oeste del lago. De modo que esta vez el produc-
to promedio de cualquier barco enviado a dicho extremo es idéntico a su producto marginal. Y como
este último siempre es más alto para los barcos enviados al extremo oeste, la asignación óptima es
enviar los cuatro barcos allá.

8
Véase el ejemplo 9.2.

09_CHAPTER 9.indd 274 3/6/09 8:46:06 PM


ProduCCión a Largo PLazo 275

Casos como el que se ilustra en el ejemplo 9.2 no son de ninguna manera inusuales. No obs-
tante, las decisiones de producción más frecuentes e interesantes son las que implican soluciones
interiores como la que vio en el ejemplo 9.1, en el que cierta cantidad positiva del insumo productivo
debe asignarse a cada actividad.

Suponga que de los últimos segundos que dedicó al problema I en su primer examen de EJEMPLO 9.3
economía usted ganó 4 puntos adicionales, mientras que de los últimos segundos que empleó
en el problema 2 ganó 6 puntos adicionales. El número total de puntos que obtuvo en estas
dos preguntas fue de 20 y 12, respectivamente, y el tiempo total que invirtió en cada una fue
el mismo. El número total de puntos posibles en cada problema era 40. ¿Cómo debió haber
reasignado su tiempo entre los problemas?

La regla para la asignación eficiente del tiempo ocupado en los exámenes es la misma que para la
asignación eficiente de cualquier recurso: su producto marginal debe ser el mismo en cada actividad.
A partir de la información que se da, el producto marginal del tiempo que ocupó en el problema
2 era más alto que el producto marginal del que empleó en el problema 1. Aunque el producto
promedio del tiempo que dedicó al problema 1 era más alto que el del problema 2, habría obtenido
más puntos si hubiera ocupado algunos segundos menos en el problema 1 y algunos segundos más
en el problema 2.

PRODUCCIÓN A LARGO PLAZO


Los ejemplos estudiados hasta el momento han implicado la producción a corto plazo, en la que
por lo menos un insumo de producción no varía. A largo plazo, por el contrario, todos los facto-
res de producción son variables por definición. A corto plazo, si K se mantiene fijo en la función
de producción Q = F(K, L), se podría describir la función de producción en un diagrama sim-
ple de dos dimensiones. Sin embargo, con K y L variables, ahora se requieren tres dimensiones en vez
de dos. Y cuando hay más de dos insumos variables, se requieren todavía más.
Esto crea un problema similar al que encontró en el capítulo 3 cuando el consumidor tenía que
enfrentar una selección entre productos múltiples: no existe mucha inclinación por las representacio-
nes gráficas que implican tres o más dimensiones. Para la producción con dos insumos variables, la
solución para este problema es similar a la que se adoptó en el capítulo 3.
Para ilustrarlo, considere otra vez la función de producción que se presentó antes en este capí-
tulo:

Q = F(K, L) = 2KL (9.4)

y suponga que desea describir todas las combinaciones posibles de K y L que dan lugar a un nivel
específico de producción; por ejemplo, Q = 16. Para hacer esto, se resuelve Q = 2KL = 16 para K en
términos de L, lo cual da
8
K= . (9.5)
L
Los pares (L, K) que satisfacen la ecuación 9.5 se muestran en la curva denominada Q = 16 en la
figura 9.8. Los pares (L, K) que dan 32 y 64 unidades de producto se muestran en la figura 9.8 como
las curvas denominadas Q = 32 y Q = 64, respectivamente. Tales curvas se llaman isocuantas y se isocuanta el conjunto de
definen de manera formal como todas las combinaciones de insumos variables de las que se obtiene cierto todas las combinaciones de
nivel de producción.9 insumos que dan cierto nivel
Observe la clara analogía entre la isocuanta y la curva de indiferencia de la teoría del consu- de producción.
midor. Así como un mapa de la indiferencia proporciona una representación simple de las preferen-

9
“Iso” viene de la palabra griega que significa “igual”, que también aparece, por ejemplo, en el término meteorológico “isobaras”,
que significa líneas de igual presión barométrica.

09_CHAPTER 9.indd 275 3/6/09 8:46:07 PM


276 CaPítuLo 9 ProduCCión

FIGURA 9.8
Parte de un mapa Capital (K)
de isocuantas para la
función de producción
Q = 2KL Producto creciente
una isocuanta es el 12
conjunto de todos los
pares (L, K) que dan un
nivel dado de producción.
Por ejemplo, cada par (L, C
K) en la curva rotulada
Q = 32 da 32 unidades
de producto. el mapa de D
isocuantas describe las
propiedades de un proceso
de producción casi de A
la misma manera que 4
un mapa de indiferencia
Q = 64
describe las preferencias
del consumidor. B Q = 32
1 Q = 16
Trabajo (L)
2 8 12

cias del consumidor, un mapa de isocuantas suministra una representación simple de un proceso de
producción.
En un mapa de indiferencia, los movimientos al noreste corresponden a niveles crecientes de
satisfacción. Los movimientos similares en un mapa de isocuantas corresponden a niveles crecientes
de producción. Un punto en una curva de indiferencia se prefiere a cualquier punto que esté situado
debajo de ella y es menos preferido que cualquier punto que esté situado encima de ella. Asimismo,
cualquier conjunto de insumos en una isocuanta da más producción que cualquier grupo de insu-
mos que esté situado debajo de dicha isocuanta y menos que cualquier conjunto de insumos que esté
situado sobre ella. Así, el grupo C en la figura 9.8 da más producción que el A, pero menos que el D.
La única consideración importante para la cual la analogía entre los mapas de isocuantas y los de
indiferencia está incompleta es la importancia de las notaciones asociadas a los dos tipos de curvas.
Recuerde del capítulo 3 que los números reales asignados a cada curva de indiferencia se utilizaron
para indicar sólo las clasificaciones relativas de los conjuntos sobre diversas curvas de indiferencia. El
número que se asigna a una isocuanta, por el contrario, corresponde al nivel real de producción que
se obtiene de un grupo de insumos a lo largo de esa isocuanta. Con los mapas de indiferencia se tiene
libertad para volver a rotular las curvas de indiferencia de cualquier manera que preserve la clasifi-
cación original de los conjuntos. Pero con los mapas de isocuantas las etiquetas están determinadas
exclusivamente por la función de producción.

LA TASA MARGINAL DE SUSTITUCIÓN TÉCNICA


Recuerde de la discusión de la teoría del consumidor en el capítulo 3 que en la tasa marginal de
sustitución el consumidor está dispuesto a intercambiar un bien por otro a lo largo de una curva
tasa marginal de sustitución de indiferencia. El concepto análogo en la teoría de producción se llama la tasa marginal de sustitu-
técnica (TMST) la tasa ción técnica (o TMST). Es la tasa para la cual un insumo puede intercambiarse por otro sin alterar
a la cual un insumo puede la producción. En la figura 9.9, por ejemplo, la TMST en A se define como el valor absoluto de la
intercambiarse por otro pendiente de la isocuanta en A, |∆K/∆L|.
sin alterar el nivel total de En la teoría del consumidor se supone que la tasa marginal de sustitución disminuye con los
producción.
movimientos hacia abajo a lo largo de una curva de indiferencia. Para la mayoría de las funciones de
producción, la TMST exhibe una propiedad similar. Si se mantiene constante la producción, cuanto

09_CHAPTER 9.indd 276 3/6/09 8:46:08 PM


ProduCCión a Largo PLazo 277

FIGURA 9.9
La tasa marginal de
K sustitución técnica
La tMst es la tasa a la
cual un insumo puede
TMSTA = ΔK/ΔL intercambiarse por otro
sin alterar la producción
A total. La tMst para
ΔK cualquier punto es el valor
ΔL absoluto de la pendiente
de la isocuanta que pasa
Q0 por ese punto. si ∆K
unidades de capital se
L
retiran en el punto A y se
añaden ∆L unidades de L, la
producción permanecerá
igual en Q0 unidades.

menos se tenga de un insumo, más se debe agregar de otro para compensar la reducción de una
unidad en el primero.
Existe una relación simple pero importante entre la TMST en cualquier punto y los productos
marginales de los insumos respectivos en ese punto. En una pequeña vecindad del punto A en la
figura 9.9, suponga que se reduce K en ∆K y que se aumenta L en una cantidad ∆L justo lo suficiente
para mantener el nivel original de producción. Si PMKA representa el producto marginal del capital en
A, entonces la reducción en la producción causada por la pérdida de ∆K es igual a PMKA∆K. Si se usa
PMLA para representar el producto marginal de L en A, se concluye igualmente que la ganancia en la
producción que resulta del ∆L adicional es igual a PMLA∆L. Por último, puesto que la reducción en la
producción que resulta de tener menos K es compensada exactamente por el aumento en producción
que resulta de tener más L, se concluye que

PMKA∆K = PMLA∆L. (9.6)

Al multiplicar en cruz se obtiene

PM LA ∆K
= , (9.7)
PM KA ∆L

que dice que la TMST en A es simplemente el cociente del producto marginal de L entre el producto
marginal de K. Esta relación tendrá una aplicación importante en el siguiente capítulo, donde se
examinará la cuestión de cómo obtener un nivel dado de producción al costo más bajo posible.

EJERCICIO 9.6
Dado el nivel actual de insumos de capital y de trabajo de una empresa, el producto marginal
de trabajo para su proceso de producción es igual a 3 unidades de producto. Si la tasa marginal
de sustitución técnica entre K y L es 9, ¿cuál es el producto marginal de capital?

En la teoría del consumidor, la forma de la curva de indiferencia indica cómo el consumidor está
dispuesto a sustituir un bien por otro. En la teoría de la producción, la forma de la isocuanta relata
una historia en esencia similar. La figura 9.10 ilustra los casos extremos de insumos que son sustitu-
tos perfectos a) y complementarios perfectos b). La figura 9.10a describe un proceso de producción
en el cual automóviles y gasolina se combinan para producir viajes. El insumo gasolina viene en dos
marcas de fábrica, Texaco y Amoco, que son sustitutos perfectos entre sí. Es posible suplir 1 galón

09_CHAPTER 9.indd 277 3/6/09 8:46:10 PM


278 CaPítuLo 9 ProduCCión

FIGURA 9.10
Mapas de isocuantas
para sustitutos
perfectos y Gasolina Texaco Máquinas de escribir
complementos
perfectos
en la parte a) se obtiene Sustitutos perfectos Complementos perfectos
el mismo número de viajes

Q
a partir de cierta cantidad Q = Q3

=
total de gasolina, sin

Q3
Q = Q2

Q
importar cómo se mezclen

=
Q2
Q = Q1
Q
las dos marcas. amoco
y texaco son sustitutos =
perfectos en la producción Q1 Gasolina Amoco Mecanógrafos
de viajes en automóvil. en a) b)
la parte b) las maquinas de
escribir y los mecanógrafos
son complementos
perfectos en el proceso de
escribir cartas.

de Amoco por uno de Texaco y aun así producir el mismo número de viajes que antes. La TMST
entre Texaco y Amoco permanece constante en 1 mientras nos movemos hacia abajo a lo largo de
cualquier isocuanta.
La figura 9.10b describe un proceso de producción para teclear letras que usa dos insumos: má-
quinas de escribir y mecanógrafos. En este proceso los dos insumos son complementos perfectos.
Aquí, los insumos se combinan de la manera más eficaz en proporciones fijas. El tener más de una
máquina de escribir por mecanógrafo no aumenta la producción, ni tampoco tener más de un meca-
nógrafo por máquina de escribir.

RENDIMIENTOS DE ESCALA
Una cuestión de importancia central para la organización de la industria es si la producción se obtiene
de la manera más eficiente posible tanto a gran escala como a pequeña escala (“grande” y “pequeño”
se definen en relación con la escala del mercado concreto). Esta cuestión es importante porque la
respuesta dicta si una industria terminará siendo servida por muchas firmas pequeñas o sólo por
algunas grandes.
La propiedad técnica de la función de producción que se usa para describir la relación entre
escala y eficacia se llama rendimientos de escala. El término dice qué le sucede a la producción cuando
todos los insumos se incrementan en la misma proporción. Como los rendimientos de escala se re-
fieren a una situación en la cual todos los insumos son variables, el concepto de rendimientos de escala es
intrínsecamente de largo plazo.
Se dice que una función de producción para la cual cualquier cambio proporcional en todos los
rendimientos crecientes insumos conduce a un cambio más que proporcional en la producción exhibe rendimientos cre-
de escala la propiedad de cientes de escala. Por ejemplo, si se duplican todos los insumos en una función de producción con
un proceso de producción rendimientos crecientes de escala, se obtiene más del doble de producción que antes. Como se verá
mediante el cual un incremento en los capítulos 12 y 13, tales funciones de producción por lo general dan lugar a condiciones en las
proporcional en cada insumo cuales un número pequeño de empresas provee a la mayoría del mercado.
da origen a más que un
Los rendimientos crecientes de escala resultan a menudo de las mayores posibilidades de es-
aumento proporcional en la
producción.
pecialización en organizaciones grandes. Adam Smith ilustró este punto al describir la división del
trabajo en una fábrica de tornillos.10

Un hombre estira el alambre, otro lo endereza, un tercero lo corta, un cuarto le saca pun-
ta, un quinto lo afila en la punta para recibir la cabeza; hacer la cabeza requiere dos o tres

10
Adam Smith, The Wealth of Nations, Nueva York: Everyman’s Library, 1910 (1776), Libro 1, p. 5.

09_CHAPTER 9.indd 278 3/6/09 8:46:11 PM


rendiMientos de esCaLa 279

operaciones diferentes... He visto una pequeña fábrica... de esta clase en donde


se empleaba a solamente diez hombres... [que] podrían, si se esforzaran, hacer
entre ellos cerca de 12 libras de tornillos en un día. Hay en una libra más de 4 000
tornillos de tamaño mediano. Esas 10 personas podrían hacer más de 48 000 tor-
nillos en un día. Por lo tanto, se puede considerar que cada persona que haga una
décima parte de 48 000 tornillos fabrica 4 800 tornillos en un día. Pero si todos
ellos hubieran trabajado por separado y en forma independiente... no habrían po-
dido hacer cada uno de ellos 20, quizá ni un tornillo en un día...

La industria de líneas aéreas se cita a menudo como una de las que tiene rendimientos a escala
creciente. Los profesionales de la industria han enfatizado desde hace mucho que tener una gran
cantidad de vuelos ayuda a la aerolínea a llenar cada avión al transbordar a los pasajeros de sus vue-
los que llegan hacia sus vuelos que salen. Las actividades locales del aeropuerto también exhiben
rendimientos a escala creciente. Por otra parte, debido a la ley de los grandes números11 se concluye
que las operaciones de mantenimiento, la programación de las tripulaciones de vuelo y otras ac-
tividades relacionadas con los inventarios se realizan de manera más eficiente a una escala grande
que a una pequeña. De manera semejante, el espacio del mostrador donde se expenden boletos, los
vendedores de boletos, el equipo que hace reservaciones, el que recoge las maletas, las unidades de
tierra y las instalaciones para el abordaje de los pasajeros son todos recursos que se utilizan con más
eficiencia en niveles altos de actividad. Los rendimientos a escala creciente constituyen la explicación
subyacente de por qué la industria se ha estado moviendo hacia líneas aéreas cada vez más grandes
en la última década.
Se dice que una función de producción para la cual un cambio proporcional en todos los in-
sumos causa que la producción cambie en la misma proporción exhibe rendimientos constantes rendimientos constantes
de escala. En estos casos, duplicar todas las entradas conduce a multiplicar por dos la producción. de escala la propiedad de
En industrias en las que la producción se obtiene con rendimientos constantes de escala, el tamaño un proceso de producción
grande no es una ventaja ni una desventaja. mediante el cual un aumento
Por último, se dice que una función de producción para la cual un cambio proporcional en proporcional en cada insumo
da un aumento proporcional
todos los insumos causa un cambio menor que el proporcional en la producción obtenida exhibe
igual en la producción.
rendimientos decrecientes de escala. Aquí el tamaño grande es una desventaja y no se espera ver
grandes empresas en una industria en la cual la producción se obtiene con rendimientos decrecientes rendimientos decrecientes
de escala. Como se verá en el capítulo 11, los casos de rendimientos constantes y decrecientes con de escala la propiedad de
frecuencia permiten a muchos vendedores coexistir dentro de los mismos mercados estrechamente un proceso de producción
definidos. mediante el cual un aumento
Una función de producción no necesita exhibir el mismo grado de rendimientos de escala para proporcional en cada insumo
la variedad completa de producción. Por el contrario, puede haber rendimientos crecientes de escala da menos que un aumento
a niveles bajos de producción, seguidos por rendimientos constantes de escala a niveles intermedios proporcional en la producción.
de producción y por último por rendimientos decrecientes de escala a niveles altos de producción.

¿Por qué los constructores utilizan marcos prefabricados para techos pero no para muros?
Cuando los equipos de construcción construyen una casa con marcos de madera, por lo general EL NATURALISTA
construyen las estructuras para las paredes en el sitio de la obra. Por el contrario, compran a menudo ECONÓMICO
bastidores prefabricados para el techo. ¿Por qué esta diferencia?
Hay dos diferencias clave entre las estructuras para pared y los bastidores para techos: 1) el corte 9.2
de la madera para el bastidor de techo implica muchos ángulos complicados, mientras los cortes en
ángulo recto que requiere el ensamblaje de pared son mucho más simples, y 2) las secciones del basti-
dor de techo de cierto tamaño son todas semejantes, mientras que las secciones de pared difieren se-
gún la colocación de las aberturas para ventanas y puertas. Ambas propiedades del bastidor de techo
conducen a economías de escala sustanciales en la producción. Primero, los cortes en ángulo que
requieren se pueden hacer mucho más rápidamente si puede construirse un marco o una “plantilla”
que guíe a la madera de construcción para que pase por la hoja de la sierra justamente en el ángulo

11
Véase el capítulo 6.

09_CHAPTER 9.indd 279 3/6/09 8:46:11 PM


280 CaPítuLo 9 ProduCCión

apropiado. Es económico elaborar estas plantillas en una fábrica


BASTIDOR DEL TECHO
donde se hacen miles de cortes cada día, pero por lo general
no es redituable utilizar este método para el número limitado
de cortes que se requieren en cualquier sitio de construcción.
Asimismo, los métodos automatizados son fáciles de emplear
en los bastidores de techo en virtud de su uniformidad. La na-
ENSAMBLAJE
DE PARED
turaleza variable de los ensamblajes de pared, por el contrario,
influye en contra del uso de los métodos automatizados.
Entonces, el hecho de que haya economías de escala mu-
cho mayores en la construcción de bastidores para techo que en
¿Por que los constructores elaboran marcos a la medida para paredes, pero usan marcos las estructuras de pared ayuda a explicar por qué estas últimas
prefabricados para techos? se suelen construir en el sitio de la obra mientras que el bastidor
de techo se prefabrica más a menudo.

CÓMO MOSTRAR LOS RENDIMIENTOS


DE ESCALA EN EL MAPA DE ISOCUANTAS
Existe una relación simple entre los rendimientos de escala de una función de producción y el es-
paciamiento de sus isocuantas.12 Considere el mapa de isocuantas en la figura 9.11. A medida que
se hace un movimiento hacia fuera en el mapa de isocuantas a lo largo del rayo denominado R,
cada insumo crece justo en la misma proporción. La función de producción específica cuyo mapa de
isocuantas se muestra en el diagrama exhibe rendimientos crecientes de escala en la región de A a C.
Observe, por ejemplo, que cuando se mueve de A a B, ambos insumos se duplican mientras que la
producción aumenta por un factor de 3; asimismo, cuando se mueve de B a C, ambos insumos crecen
cerca de 50% mientras que la producción se incrementa cerca de 100%. En la región de C a F, esta
misma función de producción exhibe rendimientos constantes de escala. Observe, por ejemplo, que
cuando se mueve de D a E, ambos insumos crecen cerca de 25% y la producción también aumenta
cerca de 25%. Por último, la función de producción cuyo mapa de isocuantas se muestra en la figura
9.11 exhibe rendimientos decrecientes de escala en la región al noreste de F. Así, cuando se mueve de
F a G, ambos insumos aumentan cerca de 16.7% mientras que la producción crece nada más 11.1%.

LA DIFERENCIA ENTRE LOS RENDIMIENTOS DECRECIENTES


Y LOS RENDIMIENTOS DECRECIENTES DE ESCALA
Es importante tener en mente que los rendimientos decrecientes de escala no tienen nada que ver con la ley de
rendimientos decrecientes. Los rendimientos decrecientes de escala se refieren a lo que sucede cuando
todos los insumos varían según una proporción dada. La ley de rendimientos decrecientes, por el
contrario, se refiere al caso en el cual un insumo varía mientras todos los demás se mantienen fijos.
Como una generalización empírica, se aplica por igual a las funciones de producción que tienen
rendimientos crecientes, constantes o decrecientes de escala.

EL ROMPECABEZAS LÓGICO DE LOS RENDIMIENTOS


DECRECIENTES DE ESCALA
Si la función de producción Q = F(K, L) es una descripción completa de un proceso de producción, es
difícil ver cómo alguna función de producción podría exhibir en la práctica rendimientos decrecien-
tes de escala. La dificultad es que se debe ser capaz de duplicar el proceso que se usó para producir
cualquier nivel dado de producción y con ello lograr rendimientos constantes de escala. Para ilustrar
lo anterior, suponga primero que Q0 = F(K0, L0). Si ahora se desea producir 2Q0 unidades de producto,
siempre es posible conseguirlo haciendo de nuevo lo que se llevó a cabo la primera vez; es decir,

12
La discusión en esta sección se aplica a las funciones de producción homotéticas, una clase importante definida por la propiedad de
que las pendientes de todas las isocuantas son constantes en puntos a lo largo de cualquier rayo.

09_CHAPTER 9.indd 280 3/6/09 8:46:13 PM


resuMen 281

FIGURA 9.11
Rendimientos de
K escala en el mapa de
30 isocuantas
en la región de A a
R C esta función de
27
H producción tiene
24 rendimientos crecientes
G de escala. Los incrementos
21 proporcionales en
Q = 420
F el insumo dan más
18
que incrementos
E Q = 400
15 proporcionales en
D la producción. en la
12 Q = 360 región de C a F hay
C rendimientos constantes
9 Q = 300 de escala. Los insumos
B y la producción crecen
6 Q = 240
en la misma proporción
A Q = 180 en esta región. en la
3
Q = 90 región noreste de F hay
Q = 30
L rendimientos decrecientes
0 1 2 3 4 5 6 7 8 9 10 de escala. Los incrementos
proporcionales en
Rendimientos Rendimientos Rendimientos
ambos insumos dan
crecientes constantes decrecientes
menos que incrementos
proporcionales en la
producción.

combinar otra vez K0 y L0 para obtener Q0 y agregarlo al Q0 que ya se tiene. De manera semejante,
se puede obtener 3Q0 desarrollando F(K0, L0) tres veces en sucesión. Simplemente con desarrollar
el proceso repetidas veces es posible lograr que la producción crezca en la misma proporción que
los insumos, lo que significa rendimientos constantes de escala. Y por razones similares a las que
se discutieron antes para la industria de las líneas aéreas, a menudo será posible hacer incluso algo
mejor que eso.
En los casos en que no es posible duplicar al menos la producción duplicando K y L, se verá
forzado a concluir que debe haber un cierto insumo importante además de K y L que no se está
incrementando al mismo tiempo. A este insumo se le denomina de manera variada como “organi-
zación” o “comunicación”, la idea es que cuando una empresa rebasa cierto tamaño se comienza
de alguna manera a perder el control. Otros argumentan que es la escasez de recursos directivos o
empresariales la que crea cuellos de botella en la producción. Si en verdad hay algún insumo que no
ha sido medido y que se mantiene fijo a medida que se expande K y L, entonces todavía se está en el
corto plazo por definición. Y no hay razón para esperar poder duplicar la producción al duplicar sólo
algunos de los insumos.
El apéndice de este capítulo considera varias extensiones matemáticas de la teoría de la produc-
ción. Los temas incluidos abarcan las aplicaciones de la distinción marginal-promedio, las formas
matemáticas específicas de la función de producción y un tratamiento matemático de los rendimien-
tos de escala en la producción.

RESUMEN
• La producción es cualquier actividad que crea una utilidad • El producto marginal de un insumo variable se define como el
actual o futura. Una función de producción resume la relación cambio en la producción causado por una unidad adicional del
entre insumos y productos. El corto plazo se define como insumo variable, manteniendo fijos todos los demás insumos.
aquel periodo durante el cual por lo menos algunos insumos La ley de rendimientos decrecientes dice que más allá de cierto
son fijos. En el caso de dos insumos, es el periodo durante el punto el producto marginal disminuye con las unidades adi-
cual un insumo está fijo y el otro es variable. cionales del insumo variable.

09_CHAPTER 9.indd 281 3/6/09 8:46:14 PM


282 CaPítuLo 9 ProduCCión

• El producto promedio de un insumo variable es el cociente • El largo plazo se define como el periodo requerido para que
de la producción total entre la cantidad del insumo variable. todos los insumos sean variables. La duración real del lapso
Siempre que el producto marginal esté situado por encima del temporal correspondiente a los plazos corto y largo diferirá
producto promedio, éste aumentará con los incrementos en en forma marcada para diversos casos. En el caso de dos insu-
el insumo variable. A la inversa, cuando el producto marginal mos, toda la información relevante sobre la producción a largo
esté situado por debajo del producto promedio, este último plazo se puede resumir de manera gráfica mediante un mapa
disminuirá con los incrementos en el insumo variable. de isocuantas. La tasa marginal de sustitución técnica se define
• Un problema práctico importante tiene que ver con la manera como aquella para la cual un insumo puede ser sustituido por
de asignar un insumo a dos actividades productivas para ob- otro sin alterar el nivel de producción. La TMST en cualquier
tener la máxima producción posible. En general, son posibles punto es el valor absoluto de la pendiente de la isocuanta en
dos tipos de soluciones. Se genera una solución de esquina ese punto. Para la mayoría de las funciones de producción, la
cuando el producto marginal del insumo es siempre más alto TMST disminuirá a medida que se mueva hacia abajo a la de-
en una actividad que en la otra. En ese caso, lo mejor que se recha a lo largo de una isocuanta.
puede hacer es concentrar todo el insumo en la actividad don- • Se dice que una función de producción exhibe rendimientos
de sea más productivo. constantes de escala si un incremento proporcional dado en
• Ocurre una solución interior siempre que el producto mar- todos los insumos produce el mismo incremento proporcional
ginal del insumo variable, cuando se le coloca por completo en la producción, rendimientos decrecientes de escala si un in-
en una actividad, es más bajo que el producto marginal de la cremento proporcional dado en todos los insumos conduce a
primera unidad del insumo en la otra actividad. En este caso, un incremento proporcional más pequeño en la producción y
la regla de maximización de la producción es distribuir el in- rendimientos crecientes de escala si un incremento proporcio-
sumo en las dos actividades de manera tal que su producto nal dado en todos los insumos causa un incremento proporcio-
marginal sea igual en ambas. Incluso los tomadores de deci- nal mayor en la producción. También se dice que las funciones
siones experimentados violan a menudo esta regla simple. El de producción con rendimientos crecientes de escala exhiben
error del cual hay que cuidarse es la tendencia a igualar no los economías de escala. Los rendimientos de escala constituyen
productos marginales sino los productos promedio en las dos un factor crítico importante en la determinación de la estruc-
actividades. tura de la organización industrial.

PREgUNTAS DE REPASO
1. Dé tres ejemplos de producción que una persona que no sabe 6. Distinga entre los rendimientos que disminuyen para un insu-
economía en general no considera como producción. mo variable y los rendimientos decrecientes de escala.
2. Dé un ejemplo de producción en el cual el corto plazo dure 7. Verdadero o falso: si el producto marginal es decreciente, enton-
por lo menos un año. ces el producto promedio también debe serlo. Explique.
3. ¿Por qué a una persona encargada de contratar insumos pro- 8. Una fábrica agrega un trabajador y descubre después que el
ductivos le importan más los productos marginales que los producto promedio de sus trabajadores ha subido. Verdadero
productos promedio? o falso: el producto marginal del nuevo trabajador es menor
4. Un bromista comentó una vez que cuando cierto funcionario que el producto promedio de los trabajadores de la planta an-
de gobierno se mudó de Nueva York a California, el nivel pro- tes de su llegada.
medio del coeficiente intelectual (CI) en ambos estados subió. 9. En la actualidad, 2 unidades de trabajo y una unidad de capi-
Interprete esta observación en el contexto de las relaciones tal producen una unidad de producto. Si usted duplica ambos
marginales promedio que se discutieron en el capítulo. insumos (4 unidades de trabajo y 2 unidades de capital), ¿qué
5. ¿En qué se parece un mapa de isocuantas a un mapa de indife- puede concluir sobre la producción obtenida con rendimien-
rencia? ¿En qué aspecto importante difieren las dos construc- tos constantes de escala? ¿Y con rendimientos decrecientes de
ciones? escala? ¿Y con rendimientos crecientes de escala?

PRObLEMAS
1. Grafique las curvas de producto total a corto plazo para cada una de las siguientes funciones de
producción si K está fija para K0 = 4.
a) Q = F(K, L) = 2K + 3L.
b) Q = F(K, L) = K2L2.
2. ¿Obedecen las dos funciones de producción en el problema 1 a la ley de rendimientos decrecientes?

09_CHAPTER 9.indd 282 3/6/09 8:46:14 PM


ProbLeMas 283

3. Suponga que el producto marginal del trabajo es actualmente igual a su producto promedio. Si us-
ted fuera uno de los 10 nuevos trabajadores que la empresa está a punto de contratar, ¿preferiría que
le pagaran el valor de su producto promedio o el valor de su producto marginal? ¿Sería interesante
para un patrón pagarle el valor de su producto promedio?
4. La siguiente tabla proporciona información parcial sobre el producto total, el producto promedio y
el producto marginal de una función de producción. Use las relaciones entre estas propiedades para
completar las celdas que faltan.

Producto Producto Producto


Trabajo total promedio marginal
0 0
1 180
140
2
3 420
4 120

5. El departamento de policía de Filadelfia debe decidir cómo asignar a los oficiales de policía entre el
oeste de Filadelfia y el centro de la ciudad. Medidos en detenciones por hora, el producto promedio,
el producto total y el producto marginal en cada una de estas dos áreas se dan en la tabla siguiente. El
departamento de policía asigna actualmente a 200 oficiales de policía al centro de la ciudad y 300 al
oeste de Filadelfia. Si la policía puede desplegarse solamente en grupos de 100, ¿cómo, si hay alguna
manera, debe el departamento de policía reasignar a sus oficiales para alcanzar el número máximo
de detenciones por hora?

Este de Filadelfia Centro de la ciudad


Número de
policías PP PT PM PP PT MP
0 0 0 0 0
40 45
100 40 40 45 45
40 35
200 40 80 40 80
40 25
300 40 120 35 105
40 15
400 40 160 30 120
40 5
500 40 200 25 125

6. Suponga que una onda criminal golpea al oeste de Filadelfia, de modo que el producto marginal y el
producto promedio de los oficiales de policía ahora sean de 60 detenciones por hora para cualquier
número de oficiales de policía. ¿Cuál es ahora la asignación óptima de 500 oficiales de policía entre
las dos áreas?
7. La función de producción a corto plazo de una empresa está dada por

Q = —12 L2 para 0 ≤ L ≤ 2

y
1 2
Q = 3L – 4L
— para 2 < L ≤ 7.

a) Grafique la función de producción.


b) Encuentre la máxima producción alcanzable. ¿Cuánto trabajo se utiliza en ese nivel?
c) Identifique los intervalos de utilización de L para los cuales el producto marginal del trabajo está
aumentando y disminuyendo.
d) Identifique el intervalo para el cual el producto marginal del trabajo es negativo.

09_CHAPTER 9.indd 283 3/6/09 8:46:15 PM


284 CaPítuLo 9 ProduCCión

8. Cada problema en un examen vale 20 puntos. Suponga que a partir de los últimos segundos que
usted dedicó al problema 10 en el examen ganó 2 puntos adicionales, mientras que de los últimos
segundos que dedicó al problema 8 ganó 4 puntos adicionales. El número total de puntos que ganó
en estos dos problemas fue 8 y 6, respectivamente, y el tiempo total que invirtió en cada uno fue el
mismo. ¿Cómo —si hay algún modo— debió usted reasignar su tiempo entre ellos?
9. Suponga que el capital está fijo en 4 unidades en la función de producción Q = KL. Dibuje las curvas
de producto total, marginal y promedio para el insumo trabajo.
10. Identifique las regiones de rendimientos crecientes, constantes y decrecientes de escala en el mapa de
isocuantas que se muestra.
K

G
14
F
Q = 950
12
E
10 Q = 900
D
8 Q = 800
C
6 Q = 500
B
4 Q = 300
A
2 Q = 200
Q = 100
L
0 1 2 3 4 5 6 7

11. Cuando Paul Samuelson se cambió de la física a la economía, se dice que Robert Solow comentó que
subió el coeficiente intelectual (IQ) promedio en ambas disciplinas. Una persona que estaba presente
respondió que el argumento de Solow debía ser incorrecto porque implicaba que el IQ promedio
para la academia como un todo (que es un promedio ponderado de los niveles promedio de IQ para
cada disciplina) también debía haber subido como resultado del cambio, lo cual es imposible. ¿Tenía
razón esta persona? Explique.

RESPUESTAS A LOS EjERCiCiOS DEL CAPÍTULO


9.1. Para K = 4, Q = √—
4 √—
L = 2√—
L.
Q
Q=2 L
12

L
0 4 16 36

9.2. La pendiente de la curva de producto total en la figura 9.3a es 2 para todos los valores de L. Entonces
PML=3 = 2.
9.3. La pendiente del rayo a cualquier punto en la curva de producto total es 2, y entonces PPL=3 = 2. Si la
curva de producto total es un rayo, como en este caso, PPL = PML es constante para todos los valores
de L.

09_CHAPTER 9.indd 284 3/6/09 8:46:17 PM


resPuestas a Los ejerCiCios deL CaPítuLo 285

9.4. Debido a que PPL=10 < PML=10, PP aumentará cuando L aumente, y entonces PPL=10.1 > PPL=10.

PM, PP

12
PPL = 10.1
7 PP(L)

L
10 10.1
PM(L)

9.5. No es posible decir que el lanzador deba tirar más bolas rápidas sin primero saber hasta qué punto
un cambio en la proporción de lanzamientos efectuados alteraría la efectividad de ambos tipos. En
particular, es probable que el lanzamiento de más bolas rápidas disminuya la efectividad no sólo de
las bolas rápidas adicionales, sino también de todas las demás bolas rápidas. Y si esta pérdida excede
la ganancia de cambiar de curvas a bolas rápidas, no deben lanzarse más bolas rápidas.
9.6. De la relación PML/PMK = TMST, se tiene 3/PMK = 9, lo cual da PMK = — 1 .
3

09_CHAPTER 9.indd 285 3/6/09 8:46:18 PM


09_CHAPTER 9.indd 286 3/6/09 8:46:18 PM
A P é N D i C E

9
exteNsIONes matemátICas De
la teORía De la PRODUCCIÓN

APLICACIÓN: LA DISTINCIÓN ENTRE


MARGINAL Y PROMEDIO
Suponga que cuando su oponente en el tenis llega a la red, su mejor respuesta es lanzar la pelota en
trayectoria alta y con poca velocidad o pasarla por alguno de los lados. Cada tipo de disparo es más
efectivo si toma a su adversario por sorpresa. Suponga que alguien que lanza la pelota en trayectoria
elevada permanentemente gana un punto sólo 10% de las veces, pero alguien que nunca lo hace gana
el punto 90% de las ocasiones. De modo semejante, suponga que alguien que pasa la pelota todo el
tiempo gana cualquier punto sólo 30% de las veces, pero alguien que nunca lo hace gana 40% de
las veces. Suponga por último que la tasa a la cual cada tipo de lanzamiento se hace menos efectivo
disminuye de manera proporcional a las veces que un jugador lo usa. ¿Cuál es la mejor proporción de
lanzamientos elevados y de tiros de pase que debe usar cuando su oponente llega a la red?1
Las ventajas de los dos tipos de lanzamientos se resumen de manera gráfica en la figura A.9.1.
En este caso, el problema de “producción” es obtener el porcentaje máximo posible de tiros gana-
dores cuando su oponente llega a la red. F(L) representa el porcentaje de puntos que ganará con un

1
Este ejemplo fue sugerido por los psicólogos de Harvard Richard Herrnstein y James Mazur, en “Making Up Our Minds: A New
Model of Economic Behavior”, en The Sciences, noviembre/diciembre de 1987, pp. 40-47.
287

09_CHAPTER 9.indd 287 3/6/09 8:46:19 PM


288 CaPítuLo 9 aPéndiCe 9: extensiones MateMátiCas de La teoría de La ProduCCión

FIGURA A.9.1 Tiros ganados (%)


Efectividad contra uso:
lanzamientos elevados
y tiros de pase 100
Porcentaje de puntos ganados
90 con un lanzamiento elevado = F(L)

Porcentaje de puntos ganados


con un lanzamiento de pase = G(L)

40

30

10
L (proporción de lanzamientos elevados)
0.1 0.2 0.3 0.4 0.5 0.6 0.7 0.8 0.9 1.0

lanzamiento elevado de la pelota como una función del porcentaje de veces que hace un lanzamiento
de este tipo (L). F(L) es entonces, en efecto, el producto promedio de L. G(L) representa el porcentaje
de puntos que ganará con un tiro de pase, de nuevo como una función del porcentaje de veces que
hace lanzamientos elevados. La pendiente negativa de F(L) refleja el hecho de que los lanzamientos
elevados se hacen menos efectivos mientras más los usa. De modo semejante, la pendiente positiva
de G(L) dice que los tiros de pase se hacen más efectivos mientras más lanzamientos elevados haga.
Su problema es seleccionar L*, la mejor proporción de veces que debe hacer lanzamientos elevados.
Para encontrar el valor óptimo de L primero debe descubrir cómo el porcentaje de puntos to-
tales ganados, denominado P, varía con L. Para cualquier valor de L, P es simplemente un promedio
ponderado de los porcentajes que gana con cada tipo de lanzamiento. El peso que se asigna a cada
tipo de lanzamiento es la proporción de veces que se utiliza. Si observa que (1 – L) es la proporción de
tiros de pase cuando L es la proporción de lanzamientos elevados, se tiene

P = LF(L) + (1 – L)G(L). (A.9.1)

La expresión LF(L) es el porcentaje de puntos totales que gana con los lanzamientos elevados de
pelota. (1 – L)G(L), de modo semejante, es el porcentaje de puntos totales ganados con los tiros de
pase. De la figura A.9.1 se ve que las fórmulas algebraicas para F(L) y G(L) están dadas por F(L) = 90
– 80L y G(L) = 30 + 10L. La sustitución de estas expresiones en la ecuación A.9.1 da

P = 30 + 70L – 90L2, (A.9.2)

que está representada gráficamente en la figura A.9.2. El valor de L que maximiza P resulta ser L* =
0.389 y el valor correspondiente de P es 43.61%.2
Observe en la figura A.9.3 que para el valor óptimo de L, la probabilidad de ganar con un lanza-
miento elevado está casi al doble de la altura (58.9%) que la de ganar con un tiro de pase (33.9%). A
muchas personas les parece que este estado de cosas es muy incómodo, tanto que rehúsan tener algo

2
El estudiante con entrenamiento en cálculo puede encontrar L* sin tener que graficar P como una función de L resolviendo sim-
plemente
dP/dL = 70 – 180L = 0,
lo que da L* = 7/18 = 0.389, y al sustituir este valor en la ecuación A.9.2, da P = 43.61.

09_CHAPTER 9.indd 288 3/6/09 8:46:20 PM


aPLiCaCión: La distinCión entre MarginaL y ProMedio 289

FIGURA A.9.2
P (%) Proporción óptima de
lanzamientos elevados
50 P = 30 + 70L – 90L2

43.61
40

30

20

10

L (proporción de lanzamientos elevados)


0 0.2 0.4 0.6 0.8 1.0

0.389

FIGURA A.9.3
Tiros ganados (%) En el punto óptimo, la
posibilidad de ganar con
100 un lanzamiento elevado
es mucho mayor que la
90 de ganar con un tiro de
pase

58.9

40
33.9
30

10
L (proporción de lanzamientos elevados)
0.1 0.2 0.3 0.4 0.5 0.6 0.7 0.8 0.9 1.0

0.389 0.667

que ver con él. En estudios experimentales extensos, los psicólogos de Harvard, Richard Herrnstein
y James Mazur encontraron que las personas tienden a dividir sus lanzamientos no para maximizar
sus oportunidades generales de ganar, sino para equilibrar el producto promedio de cada tipo. Observe
en la figura A.9.3 que esto ocurre cuando L = 2/3, punto en el cual el porcentaje de puntos ganados
con cada uno de los dos lanzamientos es 36.7. Sin embargo, para este valor de L, el producto marginal
de un tiro de pase será mucho más alto que para un lanzamiento elevado, porque aumentará fuerte-
mente la efectividad de todos los otros lanzamientos elevados. (Por supuesto, un tiro de pase también
reducirá la efectividad de sus otros tiros de pase, pero por un margen mucho más pequeño.)
La situación en este caso es análoga al ejemplo de asignación que los barcos pesqueros, men-
cionado en el capítulo 9. No hay más razones para desear que el rendimiento promedio de cada

09_CHAPTER 9.indd 289 3/6/09 8:46:21 PM


290 CaPítuLo 9 aPéndiCe 9: extensiones MateMátiCas de La teoría de La ProduCCión

lanzamiento de tenis sea el mismo que para desear que el producto promedio en cada extremo del
lago también lo sea. Sin embargo, la tendencia a equilibrar el promedio en vez de los productos
marginales es un error muy común, uno del cual tienen que cuidarse incluso los maximizadores
experimentados. Considere un ejemplo final.

EJEMPLO A.9.1 ¿Verdadero o falso? El mejor entrenador de futbol americano el aquel que siempre escoge el
tipo de juego que gane la mayor cantidad de yardas.

Si contestó “verdadero”, no ha estado prestando atención. El mejor entrenador es el que selecciona la


mejor combinación de jugadas, de la misma manera que el mejor jugador de tenis es el que seleccio-
na la mejor combinación de lanzamientos. En la Liga Nacional de Futbol Americano las jugadas de
pase obtienen casi el doble de yardas, en promedio, que las jugadas de correr. ¿Por qué los entrenado-
res no ordenan más pases? Porque el juego a base de pases pierde efectividad si se usa con demasiada
frecuencia. A pesar de la gran diferencia en la ganancia promedio entre los dos tipos de jugadas, es
evidente que la mayoría de los entrenadores son conscientes de que correr es necesario para realizar
un buen pase. Pero muchos comentaristas ostensiblemente expertos parecen olvidadizos con respec-
to a este punto. Si pierde por 4 puntos, con 20 segundos por jugar, en su cuarta y última oportunidad,
en la línea de la yarda 4, un equipo tiene más probabilidad de anotar si lanza la pelota. Sin embargo,
un equipo ganará más juegos a largo plazo si, en esta situación, usa una jugada de correr de vez en
vez. Pero si un entrenador ordena una jugada de correr y fracasa, tanto los aficionados en las gradas
como los locutores de la cabina insistirán en que es un idiota.

LOS MAPAS DE ISOCUANTAS


Y LA MONTAÑA DE PRODUCCIÓN
Antes se obtuvieron las isocuantas de manera algebraica manteniendo la producción constante en
la función de producción y luego despejando K en términos de L. Pero hay también una técnica
geométrica para obtener el mapa de isocuantas similar a la obtención del mapa de indiferencia que
se estudió en el apéndice del capítulo 3. Este método comienza con una gráfica tridimensional de la
función de producción, quizá como la que se muestra en la figura A.9.4. Se parece a la ladera de una
montaña.

FIGURA A.9.4
La montaña de Q
producción

D
C

B
Q1
A
L
Q0

09_CHAPTER 9.indd 290 3/6/09 8:46:22 PM


aLgunos ejeMPLos de funCiones de ProduCCión 291

El valor en el eje Q mide la altura de la montaña, o la producción total, que crece a medida que se
emplea más de K o de L.
Suponga en la figura A.9.4 que se tuviera que fijar la producción en alguna cantidad constante,
por ejemplo, Q0. Es decir, se corta la montaña de producción con un plano paralelo al plano KL, Q0
unidades por encima de él. La línea denominada AB en la figura A.9.4 representa la intersección de
ese plano con la superficie de la montaña de producción. Todos los grupos de insumos situados en
AB arrojan un nivel de producción de Q0. Si luego se proyecta la línea AB sobre el plano KL, se obtiene
la isocuanta Q0 que se observa en la figura A.9.5. Como se definió en el capítulo 9, una isocuanta es el
lugar geométrico de pares K, L de los que se obtiene el mismo nivel de producción.
Suponga que luego se interseca la montaña de producción con otro plano, esta vez a Q1 unida-
des por encima del plano KL. En la figura A.9.4, el segundo plano interseca la montaña de producción
a lo largo de la línea denominada CD. Representa todos los grupos de insumos que producen Q1.
Al proyectar CD sobre el plano KL, se obtiene así la isocuanta determinada Q1 en la figura A.9.5. De
manera similar es posible generar un mapa de isocuantas completo correspondiente a la función de
producción Q = F(K, L).

FIGURA A.9.5
K Mapa de isocuantas
Producción creciente obtenido de la montaña
de producción

Q2
Q1

Q0
L
0

ALGUNOS EJEMPLOS DE FUNCIONES


DE PRODUCCIÓN
En esta sección se examinarán dos de las muchas funciones de producción que en general se usan en
el análisis económico.

LA FUNCIÓN DE PRODUCCIÓN COBB-DOUGLAS


Quizá la función de producción más usada de todas es la Cobb-Douglas, que en el caso de dos insu-
mos adopta la forma
Q = mK α Lβ , (A.9.3)
donde α y β son números entre cero y 1, y m puede ser cualquier número positivo.
Para obtener una ecuación para la isocuanta Q0, se fija Q en Q0 y luego se despeja K en términos
de L. En el caso Cobb-Douglas, esto arroja
−1/α
 m
K =  ( L )−β α . (A.9.4)
 Q0 
Para la función Cobb-Douglas Q = K1/2L1/2, la isocuanta Q0 será
Q2
K= 0. (A.9.5)
L
Una porción del mapa de isocuantas para esta función de producción de Cobb-Douglas se muestra
en la figura A.9.6.

09_CHAPTER 9.indd 291 3/6/09 8:46:26 PM


292 CaPítuLo 9 aPéndiCe 9: extensiones MateMátiCas de La teoría de La ProduCCión

FIGURA A.9.6
Mapa de isocuantas
para la función de
K
producción de Cobb-
Douglas Q = K1/2L1/2
5

3
Q=4
2
Q=3
1
Q=2
Q=1
L
0 1 2 3 4 5

El número asignado a cada isocuanta específica de la figura A.9.6 es el nivel de producción al


cual corresponde. Por ejemplo, si se tienen 2 unidades de K y 2 unidades de L, se obtiene Q = √— 2 √—2=
2 unidades de producto. Recuerde del capítulo 3 que los números que se usan para rotular las curvas
de indiferencia en un mapa de indiferencia suministran información sólo acerca de niveles relativos de
satisfacción. Todo lo que se requirió del esquema de asignación de índices en ese contexto fue que el
orden de los números que se destinaran a las curvas de indiferencia reflejara la jerarquización correcta
de los correspondientes niveles de satisfacción. Con las isocuantas, la situación es por completo dife-
rente. En efecto, no se tiene ninguna opción acerca de qué etiquetas asignarles.
Los estudiantes entrenados en cálculo pueden verificar con facildad las siguientes expresiones
para los productos marginales de trabajo y capital en el caso Cobb-Douglas:
∂Q
PM K = = α mK α −1 Lβ (A.9.6)
∂K
y
∂Q
PM L = = β mK α Lβ −1 . (A.9.7)
∂L

LA FUNCIÓN DE PRODUCCIÓN DE LEONTIEF


O DE PROPORCIONES FIJAS
La más simple entre todas las funciones de producción que se usan es la de Leontief, denominada así
en honor del premio Nobel Wassily Leontief, que la inventó. Para el caso de dos insumos, está dada
por

Q = mín (aK, bL) (A.9.8)

Si está poco familiarizado con esta forma funcional curiosa, su interpretación es simplemente
que Q es igual ya sea a aK o a bL, el que sea menor. Suponga, por ejemplo, que a = 2, b = 3, K = 4 y L
= 3. Entonces, Q = mín (2 × 4, 3 × 3) = mín (8, 9) = 8. El mapa de isocuantas para Q = mín (2K, 3L)
se muestra en la figura A.9.7.

09_CHAPTER 9.indd 292 3/6/09 8:46:29 PM


una definiCión MateMátiCa de Los rendiMientos a esCaLa 293

FIGURA A.9.7
Mapa de isocuantas
K para la función de
5 producción de Leontief
K = ( 2)L
3
Q = mín (2K, 3L)

4 Q=8

3 Q=6

2 Q=4

1 Q=2

L
0 1 2 3 4 5

Para ver por qué la función de Leontief también se llama función de producción de proporcio-
nes fijas, observe primero en la figura A.9.7 que si se comienza con 3 unidades de K y 2 unidades de L
se obtienen 6 unidades de producto. Si luego se añade más L —de modo que se tengan, por ejemplo,
3 unidades de L en lugar de 2— todavía se obtienen sólo Q = mín (2 × 3, 3 × 3) = mín (6, 9) = 6 unida-
des de producto. Por el mismo motivo, el sumar más K cuando se está en K = 3 y L = 2 no conducirá
a ningún producto adicional. En el caso Leontief, K y L se usan con más eficacia cuando aK = bL —en
el ejemplo de referencia, cuando 2K = 3L—. En la figura A.9.7, el lugar geométrico de los puntos para
los cuales 2K = 3L se muestra como el rayo K = ( _32 )L. Es a lo largo de éste donde se sitúan los vértices
de todas las isocuantas en ángulo recto de esta función de producción de Leontief.
Recuerde del capítulo 3 que en el caso de los complementos perfectos, las curvas de indiferencia
tienen la misma forma en ángulo recto que las isocuantas para la función de producción de Leontief.
Esto implica que la TMS es infinita en la rama vertical de la curva de indiferencia, cero en la rama
horizontal e indefinida en la cúspide. Por razones similares, la TMST en el caso Leontief será infinita
en la rama vertical de la isocuanta, cero en la horizontal e indefinida en el vértice.

UNA DEFINICIÓN MATEMÁTICA


DE LOS RENDIMIENTOS A ESCALA
Desde el punto de vista matemático, aumentar todos los insumos en la misma proporción significa
multiplicar todos los insumos por el mismo número c > 1. A manera de ilustración, considere la fun-
ción de producción que se discutió en el capítulo 9, Q = F(K, L) = 2KL. Para esta función específica,
cuando se multiplica cada insumo por c se obtiene
F(cK, cL) = 2(cK)(cL) = c22KL = c2F(K, L). (A.9.9)
El resultado de multiplicar cada insumo por c en esta función de producción es entonces multiplicar
el nivel original de producción por c2. En este caso la producción crece entonces más que proporcio-
nalmente respecto al crecimiento del insumo [con un crecimiento proporcional, se habría tenido una
producción igual a cF(K, L)], de modo que esta función tiene rendimientos crecientes de escala. Así,
por ejemplo, si c = 2 (una duplicación de cada insumo) se obtiene F(2K, 2L) = 2(2K)(2L) = 4(2KL), una
producción cuadruplicada.

09_CHAPTER 9.indd 293 3/6/09 8:46:30 PM


294 CaPítuLo 9 aPéndiCe 9: extensiones MateMátiCas de La teoría de La ProduCCión

Teniendo en cuenta estas observaciones, las definiciones de los tres casos pueden resumirse
como sigue:
Rendimientos crecientes: F(cK, cL) > cF(K, L); (A.9.10)

Rendimientos constantes: F(cK, cL) = cF(K, L); (A.9.11)


y
Rendimientos decrecientes: F(cK, cL) < cF(K, L). (A.9.12)

Los siguientes dos ejercicios ayudarán a consolidar su habilidad para aplicar estas definiciones a
ejemplos específicos.

EJERCICIO A.9.1
— —
¿Tiene la función de producción Q = √ K √ L rendimientos crecientes, constantes o decrecientes
de escala?

EJERCICIO A.9.2
¿Tiene la función de producción Q = K1/3L1/3 rendimientos crecientes, constantes o decrecientes
de escala?

En el caso de la función de producción de Cobb-Douglas, Q = mKαLβ, las ecuaciones A.9.10 a A.9.12


implican una relación simple entre los parámetros α y β y el grado de los rendimientos de escala. De
manera específica, si α + β > 1, existen rendimientos crecientes de escala; α + β = 1 significa rendi-
mientos constantes de escala y α + β < 1 significa rendimientos decrecientes de escala. Para ilustrar
el caso de rendimientos constantes, suponga Q = F(K, L) = mKαLβ, con α + β = 1. Entonces se tiene

F(cK, cL) = m(cK)α(cL)β, (A.9.13)

que se reduce a

c(α + β)mKαLβ = cmKαLβ = cF(K, L), (A.9.14)

la cual, por la ecuación A.9.11, es la característica que define a los rendimientos constantes de escala.

PRObLEMAS
* 1. ¿Tienen las siguientes funciones de producción rendimientos crecientes, decrecientes o constantes de
escala? ¿Cuáles no satisfacen la ley de los rendimientos decrecientes?
a) Q = 4K1/2L1/2
b) Q = aK2 + bL2
c) Q = mín (aK, bL)
d) Q = 4K + 2L
e) Q = K0.5L0.6
f ) Q = K10.3K20.3L0.3
* 2. ¿Cuánto es el producto marginal del trabajo en la función de producción Q = 2K1/3L1/3 si K se fija en
27?

* Este problema se resuelve de manera más sencilla si se usa la definición de producto marginal del cálculo diferencial.

09_CHAPTER 9.indd 294 3/6/09 8:46:31 PM


resPuestas a Los ejerCiCios deL aPéndiCe 295

3. ¿Puede usarse la función de producción de Cobb-Douglas para ilustrar un proceso de producción


en el cual los rendimientos de escala aumenten a niveles bajos de producción y sean constantes o
decrecientes a niveles altos de producción?
4. Suponga que una compañía con la función de producción

Q = mín(2K, 3L)

está usando 6 unidades de capital y 5 unidades de trabajo. ¿Cuáles son los productos marginales de K
y L en este caso?
5. El número promedio de yardas ganadas por un equipo de futbol americano universitario en una
jugada de pase es 8 + 12r, donde r es la fracción de sus jugadas totales que son carreras. Su ganancia
promedio por carrera es 10 – 8r. ¿Cuál es su fracción óptima de jugadas de carrera? Para este valor de
r, ¿cuál es la ganancia promedio por pase? ¿Y la ganancia promedio por carrera? (Este problema y el
siguiente son similares al ejemplo del tenis que se consideró antes.)
6. Suponga que usted es un lanzador de beisbol que se especializa en dos modalidades: bola rápida y
curva. Los promedios de bateo de sus oponentes en comparación con los dos lanzamientos son los
que se muestran en el siguiente diagrama. Si su objetivo es minimizar el promedio de bateo general
de sus oponentes, ¿cuál es la proporción óptima de bolas rápidas? Para esta proporción, ¿cuáles son
los promedios de bateo de sus oponentes en comparación con los dos lanzamientos?

Promedio de bateo
del oponente Promedio del bateo del oponente
en comparación con curvas
.300
.250

.100 Promedio del bateo del oponente


en comparación con curvas

f (proporción de bolas rápidas)


0 0.1 0.2 0.3 0.4 0.5 0.6 0.7 0.8 0.9 1.0

RESPUESTAS A LOS EjERCiCiOS DEL APéNDiCE


— —
A.9.1. F(K, L) = √ K √—L, de modo que F(cK, cL) = √—cK √—cL = √c 2 √— K √—L = cF(K, L), por lo tanto se tienen
rendimientos constantes de escala.
A.9.2. F(K, L) = K1/3L1/3, de modo que F(cK, cL) = (cK)1/3(cL)1/3 = c2/3K1/3L1/3 = c2/3F(K, L) < cF(K, L), por lo
tanto, se tienen rendimientos decrecientes de escala.

09_CHAPTER 9.indd 295 3/6/09 8:46:33 PM


09_CHAPTER 9.indd 296 3/6/09 8:46:33 PM
Capítulo

10
Costos*

P
oco después de terminar la universidad fui maestro de ciencia y matemáticas en una prepa-
ratoria en Sanischare, un pueblo pequeño en el este de Nepal. Durante mis 2 años allí, una
de las pocas carreteras del país estaba en construcción a través de Sanischare. Una vez que
se dio el derecho de paso y se construyeron las
alcantarillas y los puentes, el siguiente paso era
extender grava sobre la superficie de la carretera.
Como en casi todas las demás etapas del proceso,
los métodos que se emplearon en ésta parecían
copiados de una página arrancada de otro siglo.
Los trabajadores nepaleses se pusieron en cucli-
llas a la orilla del camino bajo el sol resplande-
ciente, resquebrajando las grandes rocas con sus
martillos. En un día de 12 horas, cada trabajador
produciría un montículo pequeño de grava, insu-
ficiente para cubrir un pie lineal de superficie de
la carretera. Pero había mucha gente laborando y
al final la obra se terminó.
En Estados Unidos, por supuesto, no se con-
trata gente para resquebrajar rocas a mano hasta
© Martial Colomb/Getty Images/Dal

convertirlas en grava. En lugar de eso existen má-


quinas enormes que pulverizan varias toneladas
de roca por minuto. La razón de esta diferencia
me parecía obvia en ese momento: Nepal es un
país muy pobre que simplemente no podría em-
plear el equipo caro que se usa en las naciones
industrializadas. Pero ahora me percato de que

* En algunos países de habla hispana se emplea el término costes.

297

10_CHAPTER 10.indd 297 3/7/09 9:22:38 AM


298 Capítulo 10 Costos

esta explicación está equivocada. Como se verá, todavía tendría sentido que Nepal fabricara grava
con trabajo manual incluso si tuviera enormes ingresos excedentes en su tesorería nacional, porque
allí la mano de obra es muy barata comparada con el equipo de capital.

VISIÓN PRELIMINAR DEL CAPÍTULO


En este capítulo el objetivo es traducir la teoría de la producción que se desarrolló en el capítulo 9
en una teoría de costos coherente. En el capítulo 9 se estableció la relación entre las cantidades de
insumos empleados y el nivel correspondiente de producción. Aquí se relacionará el enlace entre la
cantidad de producto y el costo de generarlo.
El primer paso será abordar la pregunta de cómo varían los costos con la producción a corto pla-
zo. Esta pregunta resulta ser más compleja de lo que parece, pues hay siete tipos diferentes de costos
que deben examinarse: costo total, costo variable, costo fijo, costo marginal, costo total promedio,
costo variable promedio y costo fijo promedio. Este conjunto suena desconcertante al principio,
pero los enlaces entre los diferentes conceptos de costo son en realidad claros y simples. Y cada uno
resulta ser importante para el estudio del comportamiento de una empresa, la cuestión principal en
los siguientes capítulos.
Aun de mayor importancia para la estructura y la conducta de la industria es la pregunta de
cómo varían los costos con la producción a largo plazo. Aquí se comienza con la pregunta de cómo
generar cierto nivel de producción; por ejemplo, una milla de carretera, al costo más bajo posible.
Una cantidad determinada puede producirse de múltiples formas: se necesita encontrar la manera
más barata y el método más apropiado para los precios existentes de los factores. La respuesta a
esta pregunta permite explorar cómo se relacionan los costos con los rendimientos a escala en la
producción.

COSTOS A CORTO PLAZO


Para ver cómo varían los costos con la producción a corto plazo es conveniente comenzar con un
ejemplo de producción sencillo del tipo que se estudió en el capítulo 9. Suponga que la tintorería de
Kelly lava las bolsas de ropa sucia usando trabajo (L) y capital (K). El trabajo se compra en el mercado
abierto a un salario de w = 10 dólares/persona-hora.1 El capital se fija a corto plazo. La relación entre
el insumo variable y el número total de bolsas lavadas por hora se resume en la tabla 10.1. Observe
que la producción al inicio aumenta a una tasa creciente con unidades adicionales del insumo varia-
ble (a medida que L aumenta de 0 a 4 unidades), luego se desarrolla a una tasa decreciente (a medida
que L aumenta de 4 a 8 unidades).
El costo total de fabricar los diversos niveles de producción es tan sólo el costo de todos los facto-
res de producción empleados. Si Kelly posee su propio capital, su valor de renta implícito es un costo
de oportunidad, es decir, el dinero que podría haber ganado si hubiera vendido su capital e invertido
las ganancias en un bono gubernamental, por ejemplo (vea el capítulo 1). Suponga que el capital
de Kelly está fijo en 120 máquinas-hora/hora, el valor de renta de cada una de las cuales es r = 0.25
costo fijo (CF) costo que dls./máquina-hora,2 para una renta de capital total de 30 dólares/hora. Éste es un costo fijo (CF), lo
no varía con el nivel de cual significa que no varía a corto plazo conforme cambia el nivel de producción. En un sentido más
producción a corto plazo (el general, si K0 representa la cantidad de capital y r es su precio de renta por unidad, se tiene
costo de todos los factores
fijos de la producción). CF = rK0. (10.1)

1
Una persona-hora es una persona que trabaja durante 1 hora. En el capítulo 14 se analizará cómo se determinan los precios de los
insumos. Por el momento, simplemente se toman como dados.
2
Una máquina-hora es una máquina que trabaja durante 1 hora. Decir que el capital de Kelly está fijo en 120 máquinas-hora/hora
significa que él tiene 120 máquinas que pueden operar simultáneamente.

10_CHAPTER 10.indd 298 3/7/09 9:22:39 AM


Costos a Corto plazo 299

TABLA 10.1
La función de producción a corto plazo para la tintorería de Kelly

Cantidad de trabajo Cantidad de producción las entradas en cada fila


(persona-hora/hora) (bolsas/hora) de la columna derecha
contienen la cantidad
0 0
de producto que se
1 4 obtiene con la cantidad
de insumos variables en
2 14
la fila correspondiente de
3 27 la columna izquierda. Esta
4 43 función de producción
exhibe inicialmente
5 58 rendimientos crecientes
6 72 y luego decrecientes para
el insumo variable.
7 81
8 86

Otros ejemplos de costo fijo podrían incluir impuestos sobre bienes, pagos de seguro, interés
en los préstamos y otros pagos para los cuales la empresa está comprometida a corto plazo y que no
varían cuando lo hace el nivel de producción. Los gerentes comerciales a menudo se refieren a los
costos fijos como costos fijos generales.
El costo variable (CV) se define como el costo total del factor de producción variable para cada costo variable (CV) costo
nivel de producción.3 Para calcular el CV de cualquier nivel de producción en este ejemplo simple- que varía con el nivel de
mente se multiplica la cantidad de trabajo necesario para generarlo por el salario por hora. Así, el producción a corto plazo (el
costo variable de 27 bolsas/hora es (10 dólares/persona-hora)(3 personas-hora/hora) = 30 dólares/ costo de todos los factores
variables de la producción).
hora. En un sentido más general, si L1 es la cantidad del trabajo requerido para producir un nivel de
producción de Q1 y w es el salario por hora, se tiene

CVQ1 = wL1. (10.2)

Observe la dependencia explícita del CV respecto de la producción en el lado izquierdo de la ecuación


10.2, la cual está ausente en la ecuación 10.1. Esto es para enfatizar que el costo variable depende del
nivel de producción que se genera, el costo fijo no.
El costo total (CT) es la suma de CF y CV. Si Kelly desea lavar 43 bolsas/hora, el costo total de costo total (CT) todos los
hacerlo será 30 dólares/hora + (10 dólares/persona-hora)(4 personas-hora/hora) = 70 dólares/hora. costos de la producción: la
De manera más general, la expresión para el costo total de fabricar un nivel de producción de Q1 se suma del costo variable y del
escribe costo fijo.

CTQ1 = CF + CVQ1 = rK0 + wL1. (10.3)

La tabla 10.2 muestra los costos fijo, variable y total para los niveles correspondientes de produc-
ción para la función que se da en la tabla 10.1. Las relaciones entre las diversas categorías de costo se
ven con claridad si se exhibe la información de manera gráfica, no en forma de tabla. La función de
producción a corto plazo de la tabla 10.1 se grafica en la figura 10.1. Recuerde del capítulo 9 que la
región inicial de curvatura ascendente (0 ≤ L ≤ 4) de la función de producción corresponde a los ren-
dimientos crecientes del insumo variable. Más allá del punto L = 4, la función de producción exhibe
rendimientos decrecientes para el insumo variable.

3
En los procesos de producción con más de un insumo variable, el costo variable se refiere al costo de todos los insumos de este
tipo.

10_CHAPTER 10.indd 299 3/7/09 9:22:40 AM


300 Capítulo 10 Costos

TABLA 10.2
Producción y costos

Q CF CV CT
0 30 0 30
4 30 10 40 El costo de capital fijo
es de 30 dólares/hora y
14 30 20 50 el costo por unidad del
27 30 30 60 factor variable (L) es de
10 dólares/hora. El costo
43 30 40 70 total se calcula como la
58 30 50 80 suma del costo fijo y del
costo variable.
72 30 60 90
81 30 70 100
86 30 80 110

FIGURA 10.1
La producción como Q
una función de un
insumo variable 86 Q = F(K0, L)
Este proceso de 81
producción muestra
rendimientos crecientes 72
para el insumo variable
hasta L = 4 y rendimientos 58
decrecientes después.
43

27

14

4
L
1 2 3 4 5 6 7 8 9 10

REPRESENTACIÓN GRÁFICA DE LAS CURVAS


DE COSTO TOTAL, VARIABLE Y FIJO
Como es lógico, la forma de la curva de costo variable está relacionada de manera sistemática con la
forma de la función de producción a corto plazo. La conexión surge porque la función de producción
dice cuánto trabajo se necesita para producir un nivel dado de producción, y esta cantidad de trabajo
da el costo variable cuando se multiplica por el salario. Suponga, por ejemplo, que quiere graficar el
costo variable de producir 58 unidades de producto. (Ver las figuras 10.1 y 10.2.) Primero se observa,
en la función de producción que se muestra en la figura 10.1, que 58 unidades de producto requieren
5 unidades de trabajo, el cual, para un salario de 10 dólares/persona-hora, da lugar a un costo varia-
ble de (5)(10) = 50 dólares/hora. Así, en la figura 10.2, el nivel de producción de 58 corresponde a un
costo variable de 50 dólares/hora. De modo semejante, observe en la función de producción que 43
unidades de producto requieren 4 unidades de trabajo, lo cual, un salario de 10 dólares, da lugar en
la figura 10.2 a un costo variable de 40 dólares/hora. De manera similar, es posible generar tantos
puntos adicionales en la curva de costo variable como se quiera.

10_CHAPTER 10.indd 300 3/7/09 9:22:41 AM


Costos a Corto plazo 301

FIGURA 10.2
Curvas de costo total,
CT
variable y fijo
Estas curvas son para la
CT, CF, CV (dólares/h) función de producción de
la tintorería de Kelly que
se observa en la figura
CV 10.1. la curva de costo
80 CF = 30 variable pasa por el origen,
lo cual significa que el
70
costo variable de producir
60 cero unidades de producto
CF = 30 es igual a cero. la curva
50 Ct, que es la suma de las
40 curvas CF y CV, es paralela
a la curva CV y está situada
30 CF en CF = 30 unidades arriba
de ella.
20
10
Q (bolsas/h)
0 4 14 27 43 58 72 81 86

De particular interés es la relación entre la curvatura de la función de producción y la de costo


variable. Observe en la figura 10.1 que L = 4 es el punto en el cual comienzan los rendimientos decre-
cientes para el factor de producción variable. Para los valores de L menores que 4 hay rendimientos
crecientes para L, lo cual quiere decir que los incrementos en L producen aumentos sucesivamente
mayores de Q en esa zona. Dicho de otro modo, en esta zona un aumento dado en la producción,
Q, requiere incrementos sucesivamente más pequeños del insumo variable, L. En consecuencia, el
costo variable evoluciona a una tasa decreciente para niveles de producción menores que 43. Esto se
refleja en la figura 10.2 por la forma cóncava de la curva de costo variable para niveles de producción
entre 0 y 43.
Una vez que L sobrepasa a 4 en la figura 10.1 se entra en la zona de rendimientos decrecientes.
Aquí se requieren incrementos sucesivamente mayores de L para producir un incremento de Q. En
consecuencia, el costo variable aumenta a una tasa creciente en esta zona. Esto se refleja en la for-
ma convexa de la curva de costo variable en la figura 10.2 para niveles de producción que exceden
de 43.
Como los costos fijos no varían con el nivel de producción, su gráfica es una línea horizontal.
La figura 10.2 muestra las curvas de costos fijo, variable y total (CF, CV y CT) para la función de pro-
ducción de la figura 10.1. Observe en dicha figura que la curva de costo variable pasa por el origen,
lo cual significa que el costo variable es cero cuando no se genera producción. El costo total de la
producción cero es igual a los costos fijos, CF. Observe también en la figura que la distancia vertical
entre las curvas de CV y CT es igual a CF en todos los puntos. Esto significa que la curva de costo
total es paralela a la curva de costo variable y se sitúa a CF unidades por encima de ella.

Suponga que la función de producción está dada por Q = 3KL, donde K representa capital y L EJEMPLO 10.1
representa trabajo. El precio del capital es 2 dólares/máquina-hora, el precio del trabajo es 24
dólares/persona-hora y el capital está fijo en 4 máquina-hora/hora a corto plazo. Grafique las
curvas de CT, CV y CF para este proceso de producción.

A diferencia del proceso de producción de la figura 10.1, el de este ejemplo es uno en el cual hay
rendimientos constantes para el factor de producción variable en todos los puntos. Como se muestra
en la figura 10.3, la producción en este caso es estrictamente proporcional al insumo variable.

10_CHAPTER 10.indd 301 3/7/09 9:22:42 AM


302 Capítulo 10 Costos

FIGURA 10.3
La función de Q
producción Q = 3KL,
con K = 4
Esta función de producción
a corto plazo exhibe Q = 12L
rendimientos constantes
para L en el intervalo
completo de L. No existe ni
12
una zona de rendimientos
crecientes ni una zona de
rendimientos decrecientes
para L. L
0 1

Para poder derivar la función de costo total a partir de esta producción primero es preciso des-
cubrir cuánto capital y trabajo se requieren para crear un nivel dado de producción a corto plazo.
Como K está fijo en 4 máquinas-hora/hora, la cantidad requerida de insumo trabajo se encuentra al
resolver Q = 3KL = 3(4)L para L = Q/12. El costo total de producir Q unidades de producto por hora
está dado por

CT(Q) = (2 dólares/máquina-h)(4 máquinas-h/h) (10.4)


Q persona-h/h) = 8 dólares/h + 2Q dólares/h.
+ (24 dólares/persona-h) ( —
12

El gasto de 8 dólares/hora en capital constituye un costo fijo. El costo variable es el costo total
menos el costo fijo, o

CVQ = 2Q. (10.5)

Las curvas de costo total, variable y fijo se grafican en la figura 10.4.

FIGURA 10.4
Las curvas de costo
total, variable y fijo
para la función de
producción Q = 3KL
Con K fijo en 4 máquinas- Dólares/hr
hora/hora a corto plazo y CT
un precio de K de
r = 2 dólares/máquina-
hora, los costos fijos son CV
de 8 dólares/hora. para
producir Q unidades de 8 CF
producto por hora se
requieren Q/12 persona-
hora/hora de trabajo. Con Q
un precio del trabajo de 24 0 4
dólares/persona-hora, el
costo variable es 2 dólares
Q/hora. El costo total es 8
dólares/hora + 2 dólares
Q/hora.

EJERCICIO 10.1
Igual que el ejemplo 10.1 excepto que el precio de capital es r = 4 dólares/máquina-hora.

10_CHAPTER 10.indd 302 3/7/09 9:22:43 AM


Costos a Corto plazo 303

OTROS COSTOS A CORTO PLAZO


El costo fijo promedio (CFP) es el costo fijo dividido entre la cantidad de producción. Para la función costo fijo promedio
de producción de la tabla 10.1, por ejemplo, el costo fijo promedio de lavar 58 bolsas/hora es (30 dóla- (CFP) costo fijo dividido entre
res/hora) ÷ (58 bolsas/hora) = 0.517 dls./bolsa. Por lo general el costo fijo promedio de producir un la cantidad de producción.
nivel de producción Q1 se escribe

CF rK 0
CFPQ = = . (10.6)
1
Q1 Q1

Observe en la ecuación 10.6 que, a diferencia de CF, el CFP depende del nivel de producción gene-
rado.
El costo variable promedio (CVP) es el costo variable dividido entre la cantidad de producción. costo variable promedio
Si Kelly lava 72 bolsas/hora, su CVP será (10 dólares/persona-hora) (6 personas-hora/hora) ÷ 72 (CVP) costo variable
bolsas/hora = 0.833 dls./bolsa. El costo variable promedio de producir un nivel de producción Q1 dividido entre la cantidad de
puede escribirse como producción.

CVQ wL1
CVPQ = 1
= . (10.7)
1
Q1 Q1

El costo total promedio (CTP) es el costo total dividido entre la cantidad de producción. Y costo total promedio
como el costo total es la suma del costo fijo total y el costo variable total, se deduce que CTP es la (CTP) costo total dividido por
suma de CFP y CVP. Por ejemplo, el CTP de lavar 58 bolsas/hora es (30 dólares/hora) ÷ (58 bol- la cantidad de producto.
sas/hora) + (10 dólares/persona-hora)(5 personas-hora/hora) ÷ (58 bolsas/hora) = 0.517 dls./bolsa
+ $0.862/bolsa = $1.379/bolsa. El costo total promedio de elaborar Q1 unidades de producto está
dado por

rK 0 + wL1
CTPQ = CFPQ + CVPQ = . (10.8)
1 1 1
Q1

Por último, el costo marginal (CM) es el cambio en el costo total que resulta de elaborar una costo marginal (CM) cambio
unidad adicional de producto.4 Al pasar de 58 a 72 bolsas/hora, por ejemplo, los costos totales au- en el costo total que resulta de
mentan 10 dólares/hora, que es el costo de contratar al trabajador adicional necesario para lograr un cambio de una unidad en la
ese incremento en la producción. Como el trabajador adicional lava 14 bolsas/hora extras, el costo producción.
marginal de la producción adicional en términos de bolsas es (10 dólares/hora) ÷ (14 bolsas/hora) =
0.714 dls./bolsa. En sentido más general, si ∆Q denota el cambio en la producción de un nivel inicial
de Q1 y ∆CTQ1 denota el cambio correspondiente en el costo total, el costo marginal en Q1 está dado
por

∆CTQ
CM Q = 1
. (10.9)
1
∆Q

Como el costo fijo no varía con el nivel de producción, el cambio en costo total cuando se producen
∆Q unidades de producto adicionales es el mismo que el cambio en el costo variable. Por lo tanto,
una expresión equivalente para el costo marginal es

∆CVQ
CM Q = 1
, (10.10)
1
∆Q
donde ∆CVQ1 representa el cambio en el costo variable cuando se producen ∆Q unidades de produc-
ción adicional.

4
En términos del cálculo, la definición de costo marginal es simplemente CMQ = dCTQ/dQ.

10_CHAPTER 10.indd 303 3/7/09 9:22:47 AM


304 Capítulo 10 Costos

CÓMO GRAFICAR LAS CURVAS PROMEDIO A CORTO


PLAZO Y LAS CURVAS DE COSTO MARGINAL
Como CF no varía con la producción, el costo fijo promedio declina de manera continua a medida
que aumenta aquélla. Suponga que los costos fijos de CMGraw-Hill para producir este libro de texto
fueran de más o menos 200 000 dólares. Si sólo se produjeron 1 000 ejemplares, su costo fijo prome-
dio sería de 200 dólares/libro. Pero si el editor produce 20 000 ejemplares, el CFP se desplomará a 10
dólares/libro. El texto de principios de economía de mayor venta de CMGraw-Hill, escrito por Camp-
bell CCMonnell y Stanley Bruce, es considerablemente más grande que este libro; sin embargo, su
costo fijo promedio asciende a un poco más de l dólar/libro. Al proceso por medio del cual el CFP
desciende junto con la producción a menudo se le llama “repartir los costos fijos generales”.
Para la curva de costo fijo CF que se muestra en la parte superior de la figura 10.5, la curva de
costo fijo promedio correspondiente se muestra en la parte inferior rotulada como CFP. Al igual
que todas las demás curvas de CFP, adopta la forma de una hipérbola rectangular. A medida que la
producción disminuye hacia cero, el CFP aumenta sin límites y desciende cada vez más hasta cero
a medida que se incrementa la producción. Observe que las unidades en el eje vertical de la curva
CFP son dólares por unidad (dólares/unidad) de producto y que el eje vertical de la curva CF, por
contraste, se mide en dólares por hora (dólares/hora).

FIGURA 10.5 CT
Dólares/hora R1
Las curvas de costo
marginal, total
promedio, variable CV
promedio y fijo
promedio
la curva CM interseca a R2
las curvas Ctp y CVp en
sus respectivos puntos
mínimos.

CF

Q
Q1 Q2 Q3
Dólares/unidad de producto

CM

CTP
CVP

CFP
Q
Q1 Q2 Q3

10_CHAPTER 10.indd 304 3/7/09 9:22:48 AM


Costos a Corto plazo 305

Geométricamente, el costo variable promedio para cualquier nivel de producción Q, que es


igual a CV/Q, puede interpretarse como la pendiente de un rayo a la curva de costo variable en Q.
Observe en la parte superior de la figura 10.5 que la pendiente de un rayo a la curva CV disminuye
cuando la producción se eleva hasta el nivel Q2; a partir de ahí comienza a aumentar. Por consiguien-
te, la curva de costo variable promedio correspondiente, que se muestra en la parte inferior de la
figura 10.5, alcanza su valor mínimo en Q2, el nivel de producción para el cual el rayo R2 es tangente a
la curva de costo variable. Más allá de ese punto, la curva CVP aumenta con la producción.
La gráfica de la curva CTP se genera de manera análoga. Para cualquier nivel de producción,
CTP es la pendiente del rayo a la curva de costo total para dicho nivel de producción. Para la curva de
costo total en la parte superior de la figura 10.5, la curva CTP correspondiente se grafica en la parte
inferior del diagrama. Observe que el punto mínimo en el CTP en la parte inferior ocurre en Q3, el
nivel de producción para el cual el rayo R1 es tangente a la curva CT en la parte superior.
Recuerde que como CT = CF + CV, se concluye que CTP = CFP + CVP (simplemente dividien-
do ambos lados de la ecuación anterior entre la producción). Esto significa que la distancia vertical
entre las curvas CTP y CVP para cualquier nivel de producción siempre será el nivel correspondiente
de CFP. Así, la distancia vertical entre CTP y CVP tiende a infinito a medida que la producción cae
hacia cero y se contrae hacia cero a medida que la producción crece hacia el infinito. Observe también
en la figura 10.5 que el punto mínimo en la curva CVP ocurre para una unidad más pequeña de pro-
ducción que el punto mínimo en la curva CTP. Como el CFP disminuye de manera continua, el CTP
continúa descendiendo aun después de que CVP ha comenzado a cambiar de dirección hacia arriba.

Construya una tabla que muestre el costo fijo promedio, el costo variable promedio, el costo EJEMPLO 10.2
total promedio y el costo marginal. Use la información en la tabla 10.1 para la tintorería de
Kelly. Luego grafique estos costos promedio.

Producción y costos

Q CFP CVP CTP CM*


0 ∞ — ∞
2.50
4 7.50 2.50 10.00
1.0
14 2.14 1.43 3.57
0.77
27 1.11 1.11 2.22
0.63
43 0.70 0.93 1.63
0.67
58 0.52 0.86 1.38
1.71
72 0.42 0.83 1.25
1.11
81 0.37 0.86 1.23
2.0
86 0.35 0.93 1.28

*Los ingresos de costo marginal se colocan entre líneas en la tabla para indicar que cada ingreso representa el costo por
bolsa que implica moverse del nivel anterior de producción al siguiente.

10_CHAPTER 10.indd 305 3/7/09 9:22:48 AM


306 Capítulo 10 Costos

Se calcula el costo fijo promedio como el costo fijo dividido entre la cantidad CFP = CF/Q, el
costo variable promedio como el costo variable dividido entre la cantidad CVP = CV/Q y el costo
total promedio como el costo total dividido entre la cantidad CTP = CT/Q. Para llenar la parte baja
de la tabla se calcula el costo marginal dividiendo la diferencia en el costo total entre la diferencia en
cantidad: CM = ∆CT/∆Q. Las curvas de costo promedio se ilustran en la figura 10.6.

FIGURA 10.6
Cantidad contra costos Dólares/unidad de producto
promedio 4
Ctp es la suma de CVp y
CFp. CFp es decreciente 3
para todos los valores de Q.
2
CTP
1 CVP
CFP
Q
0 4 14 27 43 58 72 81 86

EJERCICIO 10.2
Si el CF adopta el valor de 20, ¿cuál es la distancia vertical entre las curvas del CTP y del CVP
en la figura 10.5 cuando Q = 10?

En términos de su papel en la decisión de la empresa de cuánto producir, con mucho la más im-
portante de las siete curvas de costos es la de costo marginal. La razón, como se verá en los siguientes
capítulos, es que la decisión operativa típica de la empresa incluye la pregunta de si se expande o se
contrae su nivel actual de producción. Para tomar esta decisión de manera inteligente, la empresa
debe comparar los costos y los beneficios representativos. El costo de expandir la producción (o los
ahorros que provienen de la contracción) es por definición igual al costo marginal.
Desde el punto de vista geométrico, el costo marginal en cualquier nivel de producción puede
interpretarse como la pendiente de la curva de costo total en dicho nivel. Y como las curvas de costo
total y de costo variable son paralelas, el costo marginal también es igual a la pendiente de la curva
de costo variable. (Recuerde que el componente del costo variable es todo lo que se modifica cuando
el costo total se altera, lo que significa que el cambio del costo total por unidad de producto debe ser
igual al cambio del costo variable por unidad de producto.)
Observe en la parte superior de la figura 10.5 que la pendiente de la curva de costo total dismi-
nuye cuando la producción cambia a Q1, y sube con la producción a partir de ahí.5 Esto indica que la
curva de costo marginal, denominada CM en la parte inferior, tendrá pendiente negativa hasta Q1 y
positiva a partir de ahí. Q1 es el punto en el cual comienzan los rendimientos decrecientes para esta
función de producción, y éstos explican la pendiente positiva de la curva de costo marginal a corto
plazo.
En el nivel de producción Q3, la pendiente de la curva de costo total es idéntica a la pendiente
del rayo a la curva de costo total (el rayo rotulado R1 en la parte superior de la figura 10.5). Esto
indica que el costo marginal y el costo total promedio adoptarán precisamente el mismo valor en
Q3. A la izquierda de Q3 la pendiente de la curva de costo total es menor que la pendiente del rayo
correspondiente, lo cual significa que el costo marginal será menor que el costo total promedio en
esa región. Para niveles de producción que exceden a Q3, la pendiente de la curva de costo total es
mayor que la pendiente del rayo correspondiente, de modo que el costo marginal será mayor que el

5
El punto donde cambia la curvatura se llama punto de inflexión.

10_CHAPTER 10.indd 306 3/7/09 9:22:49 AM


Costos a Corto plazo 307

costo total promedio para niveles de producción mayores que Q3. Estas relaciones se reflejan en las
curvas de costo total promedio y de costo marginal que se muestran en la parte inferior de la figura
10.5. Observe que la relación entre las curvas CM y CVP es cualitativamente similar a la que existe
entre las curvas CM y CTP. Una característica común es que CM interseca a cada curva en su punto
mínimo. Ambas curvas de costo promedio tienen la propiedad adicional de que cuando CM es menor
que el costo promedio (sea CTP o CVP), la curva de costo promedio debe decrecer con la producción; y cuando
CM es mayor que el costo promedio, este último debe aumentar con la producción.
Observe también que estas dos relaciones son muy parecidas a las existentes entre las curvas
de producto marginal y de producto promedio que se discutieron en el capítulo 9. Son una conse-
cuencia directa de la definición de costo marginal. Producir una unidad adicional cuyo costo excede
el costo promedio (sea total o variable) en que se incurre hasta entonces tiene el efecto de elevar el
costo promedio. A la inversa, una unidad adicional cuyo costo marginal sea menor que el promedio
necesariamente hará descender el promedio.
Por último, observe en la parte inferior de la figura 10.5 que las unidades en el eje vertical del
diagrama de la curva de costo marginal son otra vez dólares por unidad (dólares/unidad) de produc-
ción, lo mismo que para las tres curvas de costo promedio a corto plazo. Por lo tanto, estas cuatro
curvas pueden exhibirse en un solo diagrama. Pero usted nunca, jamás, debe tratar de colocar cual-
quiera de estas cuatro curvas en los mismos ejes junto a las curvas de costo total, costo variable o cos-
to fijo. Las unidades que se miden a lo largo de los ejes verticales simplemente no son compatibles.

Suponga que la producción está dada por la función Q = 3KL, donde K representa al capital EJEMPLO 10.3
y L al trabajo. El precio del capital es de 2 dólares/máquina-hora, el precio del trabajo es 24
dólares/persona-hora, y el capital se fija en 4 unidades a corto plazo (ésta es la misma función
de producción y los mismos precios de insumos que se dieron en el ejemplo 10.1). Grafique
las curvas de CTP, CVP, CFP y CM.

Recuerde del ejemplo 10.1 que la curva de costo total para este proceso está dada por

CTQ = 8 + 2Q. (10.11)

El costo marginal es la pendiente de la curva de costo total, que en este caso es igual a 2 dólares/
unidad de producto:

∆CTQ
CM Q = = 2. (10.12)
∆Q

El costo variable promedio está dado por CVQ/Q, que es también de 2 dólares/unidad de pro-
ducto:
2Q (10.13)
CVPQ = = 2.
Q

Cuando el costo marginal es constante, como en este proceso de producción, siempre será igual al
CVP.
El costo fijo promedio está dado por

8
CFPQ = (10.14)
Q

y el costo total promedio está dado por


8 (10.15)
CTPQ = 2 +
Q

10_CHAPTER 10.indd 307 3/7/09 9:22:52 AM


308 Capítulo 10 Costos

FIGURA 10.7
Curvas de costo para un
proceso de producción Dólares/hora
específico TC
20
para los procesos de
producción con un costo
marginal constante el VC
costo variable promedio
y el costo marginal son 8 FC
idénticos. El costo marginal
siempre está situado Q
debajo de Ctp para estos 0 1 2 3 4 5 6 7
procesos.
Dólares/unidad de producto

10

CTP
2 CM = CVP
CFP
Q
0 1 2 3 4 5 6 7

en este ejemplo. Las curvas de los costos marginal y promedio son como se muestran en la parte
inferior de la figura 10.7, donde la parte superior reproduce las correspondientes curvas de costo
total, variable y fijo.

ASIGNACIÓN DE LA PRODUCCIÓN
ENTRE DOS PROCESOS
En el capítulo 9 se vio que el problema de asignar un recurso fijo entre dos actividades de produc-
ción se soluciona cuando se iguala el producto marginal del recurso en cada uno. Un problema muy
relacionado puede solucionarse con los conceptos de costo que se han desarrollado en este capítulo.
Aquí el problema es dividir cierta cuota de producción entre dos procesos de tal manera que se gene-
re al costo más bajo posible.
Sea QT la cantidad total que debe producirse y sean Q1 y Q2 las cantidades producidas en el pri-
mero y el segundo procesos, respectivamente. Y suponga que el costo marginal en alguno de los dos
procesos a niveles muy bajos de producción es menor que el costo marginal para QT unidades de pro-
ducto en el otro (lo cual asegura que se usen ambos).6 Los valores de Q1 y Q2 que resuelven este problema
serán entonces los que den como resultado costos marginales iguales para los dos procesos.
Para ver por qué, suponga lo contrario, es decir, que la asignación que minimiza el costo dio
como resultado un costo marginal más alto en un proceso que en el otro. Luego se podría desplazar
una unidad de producto del proceso con el costo marginal más alto a aquel con el más bajo. Como el
resultado sería la misma producción total que antes para un costo total más bajo, la división inicial no
puede ser la que minimice el costo.

6
Suponga que el costo marginal para Q = QT usando la función de producción A fuera menor que el costo marginal para Q = 0 para
el proceso de producción B: CM QA < CM 0B . Entonces la forma más barata de producir QT sería usar nada más el proceso A.
T

10_CHAPTER 10.indd 308 3/7/09 9:22:53 AM


asIGNaCIóN DE la proDuCCIóN ENtrE Dos proCEsos 309

En el capítulo 9 se vio que dos procesos de producción podrían tener productos marginales igua-
les aun si sus productos promedio difirieran de manera sustancial. En este caso también es posible
que dos procesos de producción tengan costos marginales iguales aun cuando sus costos promedio
difieran de forma notable. La condición que minimiza el costo no requiere que los niveles de costo
promedio en los dos procesos sean iguales y, ciertamente, en la práctica, a menudo adoptarán valores
muy diferentes.

Suponga que los procesos de producción A y B dan lugar a las siguientes curvas de costo EJEMPLO 10.4
marginal y costo promedio total:
CMA = 12QA, CTPA = 16/QA + 6QA,
CMB = 4QB, CTPB = 240/QB + 2QB,
donde los superíndices representan a los procesos A y B, respectivamente. ¿Cuál es la forma
menos costosa de producir un total de 32 unidades de producto?

La condición de costo mínimo es que CM QA A = CM QB B , con QA + QB = 32. Si se igualan los costos


marginales se tiene

12QA = 4QB. (10.16)

Al sustituir QB = 32 – QA en la ecuación 10.16 se tiene

12QA = 128 – 4QA, (10.17)

cuya solución es QA = 8. QB = 32 – 8 = 24 considera a la producción restante y para estos niveles de


producción el costo marginal en ambas plantas será de 96 dólares/unidad de producto (vea la figura
10.8). La línea CMT = 3QT es la suma horizontal de CMA y CMB.7

FIGURA 10.8
La asignación de
CM A CM B CM T
producción a costo
mínimo
CMA = 12QA para generar cierta
producción total a
un costo mínimo,
CMB = 4QB ésta deberá asignarse
CMT = 3QT mediante las actividades
96 96 96
alternativas de
producción de modo que
QA QB QT el costo marginal de cada
8 24 0 32 actividad sea el mismo.

Los valores de costo total promedio que corresponden a esta asignación son CTPA = 50 dólares/
unidad de producto y CTPB = 58 dólares/unidad de producto. De las curvas de costo total promedio
es posible deducir curvas de costo total en este ejemplo (multiplique CTP por Q).8 Están dadas por
CTA = 16 + 6(QA)2 y CTB = 240 + 2(QB)2. La asignación que minimiza el costo resulta en CTA = 400
dólares y CTB = 1 392 dólares, que ilustra que la distribución que minimiza costos tampoco requiere
igualar los costos totales.

7
CMT se encuentra resolviendo QT = QA + QB = CM/12 + CM/4 = CM/3 para CMT = 3QT.
8
Observe que CMA = dCTA/dQA = d[16 + 6(QA)2]/dQA = 12QA y CMB = dCTB/dQB = d[240 + 2(QB)2]/dQB = 4QB.

10_CHAPTER 10.indd 309 3/7/09 9:22:55 AM


310 Capítulo 10 Costos

EJERCICIO 10.3
Igual que el ejemplo 10.4 excepto que la producción total es 12.

LA RELACIÓN ENTRE PM, PP, CM Y CVP


En el capítulo 9 se vio que la curva del producto marginal corta a la curva de producto promedio
en el valor máximo de la curva PP. En este capítulo se vio que la curva de costo marginal corta a la
curva de costo variable promedio en el valor mínimo de la curva CVP. Existe un enlace directo entre
estas relaciones. Para ver la conexión, observe primero que a partir de la definición de costo marginal
se tiene MC = ∆CV/∆Q. Cuando el trabajo es el único factor variable, ∆CV = ∆wL de modo que
∆CV/∆Q es igual a ∆wL/∆Q. Si los salarios son fijos, esto es lo mismo que w∆L/∆Q. Y como ∆L/∆Q
es igual a 1/PM, se concluye que

w (10.18)
CM = .
PM

De manera similar, observe de la definición de costo variable promedio que CVP = CV/Q = wL/Q, y
como L/Q es igual a 1/PP, se concluye que

w
CVP = . (10.19)
PP
A partir de la ecuación 10.18 se ve que el valor mínimo del costo marginal corresponde al valor
máximo de PM. Asimismo, se deduce de la ecuación 10.19 que el valor mínimo de CVP corresponde
al valor máximo de PP. La parte superior de la figura 10.9 grafica las curvas PP y PM como funciones
de L. La parte inferior usa las ecuaciones 10.18 y 10.19 para graficar las curvas correspondientes de

FIGURA 10.9 PM, PP


La relación entre PM,
PP, CM y CVP
Normalmente las curvas
CM y CVp se grafican con
Q en el eje horizontal.
En la parte inferior se PPL
muestran como funciones
de L. El valor de Q que
corresponde a un valor
dado de L se encuentra al
multiplicar L por el valor
correspondiente de ppl. El L
valor máximo de la curva L1 L2
PML
pM, para L = L1, en la parte
CM, CVP CM
superior, corresponde al
valor mínimo de la curva
CM, para Q = Q1, en la
parte inferior. De manera CVP
similar, el valor máximo de
la curva pp, para L = L2, en la
parte superior, corresponde
al valor mínimo de la curva
CVp, para Q = Q2, en la
parte inferior.
L
L1 L2
Q = L × PPL
Q1 Q2

10_CHAPTER 10.indd 310 3/7/09 9:22:57 AM


Costos a larGo plazo 311

CM y CVP como funciones de L. (En general las curvas MC y CVP se grafican como funciones de
Q. El valor de Q que corresponde a un valor dado de L en la parte inferior puede calcularse multipli-
cando L por el valor correspondiente de PPL.) Observe que la curva PM en la parte superior toma su
máximo valor en L = L1 y que el valor mínimo de la curva CM en la parte inferior ocurre en el nivel
de producción (Q1) que corresponde a L = L1. Observe también que la curva PP en la parte superior
toma su máximo valor en L = L2, y que el valor mínimo de la curva CVP en la parte inferior ocurre
en el nivel de producción (Q2) que corresponde a L = L2.

EJERCICIO 10.4
Para una función de producción en cierto nivel de producción a corto plazo el producto mar-
ginal del trabajo es mayor que el producto promedio del trabajo. ¿Cómo es el costo marginal
para ese nivel de producción comparado con el costo variable promedio?

COSTOS A LARGO PLAZO


A largo plazo todos los insumos son variables por definición. Si el gerente de la empresa desea ge-
nerar un nivel dado de producción al costo más bajo posible y tiene la libertad de escoger cualquier
combinación de insumos que le agrade, ¿cuál debe escoger? Como se verá en la siguiente sección la
respuesta a esta pregunta depende de los precios relativos del capital y del trabajo.

CÓMO ESCOGER LA COMBINACIÓN ÓPTIMA DE INSUMOS


No importa cuál sea la estructura de la industria: monopólica o competitiva en pequeña escala, capi-
talista o socialista, industrializada o menos desarrollada, el objetivo de la mayoría de los productores
es fabricar cualquier nivel y calidad de producción al costo mínimo posible. En forma equivalente, el
productor quiere generar tanta producción como sea posible a partir de cualquier gasto dado en los
insumos.
Para comenzar considere el caso de una empresa que quiere maximizar la producción a partir
de un nivel dado de gasto. Suponga que usa sólo dos insumos, capital (K) y trabajo (L), cuyos pre-
cios, medidos en dólares por unidad de insumo por día, son r = 2 y w = 4, respectivamente. ¿Qué
combinaciones diferentes de insumos puede comprar la empresa para un gasto total de C = 200
dólares/día? Observe que esta pregunta tiene la misma estructura que la que se encontró en la teoría
del comportamiento del consumidor en el capítulo 3 (“con un ingreso de M y al enfrentar precios de línea de isocosto conjunto de
PX y PY, ¿qué combinaciones de X y Y puede comprar el consumidor?”). Recuerde que en el caso del paquetes de insumos cada uno
consumidor la respuesta estaba fácilmente resumida por la limitación impuesta por el presupuesto. de los cuales cuesta la misma
La información paralela en el caso de la empresa está resumida por la línea de isocosto, que se cantidad.
muestra en la figura 10.10 para el ejemplo actual. Cualquiera de las combinaciones de insumos en el

FIGURA 10.10
K La línea de isocosto
C/r = 100 para precios de insumo
dados (r = 2 y w = 4
B en el diagrama), la línea
C = rK + wL de isocosto es el lugar
geométrico de todos
los posibles paquetes
de insumos que pueden
Pendiente = –w/r = –2 comprarse con un nivel
dado de gasto total C (200
dólares en el diagrama). la
pendiente de la línea de
L isocosto es el negativo del
C/w = 50 cociente de los precios de
los insumos, – w/r.

10_CHAPTER 10.indd 311 3/7/09 9:22:58 AM


312 Capítulo 10 Costos

lugar geométrico denominado B puede comprarse con un gasto total de 200 dólares/día. De manera
análoga para el caso de la limitación impuesta por el presupuesto, la pendiente de la línea de isocosto
es el valor negativo del cociente de los precios de los insumos: – w/r.

EJERCICIO 10.5
Si w = 3 y r = 6, dibuje las líneas de isocosto que corresponden al gasto total de 90 y 180 dólares
por unidad de tiempo.

El método analítico para encontrar la producción máxima que puede generarse para un costo
dado resulta ser similar al que se empleó para encontrar el paquete óptimo de consumo. Así como
puede alcanzarse un nivel dado de satisfacción con alguno de múltiples paquetes de consumo po-
sibles (los cuales están situados sobre la misma curva de indiferencia), también puede producirse
una cantidad dada de producto mediante una de diferentes combinaciones de insumos (las cuales
están situadas sobre la misma isocuanta). En el caso del consumidor se encontró el paquete óptimo
superponiendo la limitación impuesta por el presupuesto en el mapa de indiferencia y localizando el
punto correspondiente de tangencia.9 En este caso se superpone la línea de isocosto en el mapa de
isocuantas. En la figura 10.11 el punto de tangencia (L*, K*) es la combinación de insumos que aporta
la producción más alta posible (Q0) para un gasto de C.

FIGURA 10.11
La producción máxima
para un gasto dado K
una empresa que está
tratando de obtener C/r
la mayor producción
posible para un gasto
de C seleccionará la
combinación de insumos en
la cual la línea de isocosto K*
para C sea tangente a una
isocuanta.

Q2
Q0
Q1
L
L* C/w

Como se observa, el problema de generar la satisfacción más grande para cierto gasto se resuelve
en esencia de la misma forma que el problema de generar un nivel dado de producción al costo más
bajo posible. La única diferencia es que en este último caso se comienza con una isocuanta específica
(la que corresponde al nivel de producción que se está tratando de obtener), luego se superpone un
mapa de líneas de isocosto, cada uno correspondiente a un nivel de costos diferente. En el primer
ejercicio el costo fue fijo y la producción variaba; esta vez, la producción es fija y los costos varían.
Como se muestra en la figura 10.12, el paquete de insumos de menor costo (L*, K*) corresponde al
punto de tangencia entre una línea de isocosto y la isocuanta especificada (Q0).
Recuerde del capítulo 9 que la pendiente de la isocuanta en cualquier punto es igual a –PML/
PMK, el negativo de la relación del producto marginal de L entre el producto marginal de K en ese
punto. (Recuerde también del capítulo 9 que el valor absoluto de esta proporción se llama tasa mar-

9
Excepto, por supuesto, en el caso de soluciones de esquina.

10_CHAPTER 10.indd 312 3/7/09 9:22:59 AM


Costos a larGo plazo 313

FIGURA 10.12
K El costo mínimo
para un nivel dado de
C3/r producción
una empresa que está
tratando de producir
C2/r cierto nivel de producción,
Q0, al costo más bajo
posible seleccionará la
C1/r combinación de insumos
para la cual una línea de
K* isocosto sea tangente a la
isocuanta Q0.
Q0

L
L* C1/w C2/w C3/w

ginal de sustitución técnica.) Al combinar esto con el resultado de que el costo mínimo ocurre en un
punto de tangencia con la línea de isocosto (cuya pendiente es – w/r), se concluye que

PM L* w
= , (10.20)
PM K * r

donde K* y L* de nuevo representan los valores de costo mínimo de K y L. Al multiplicar en cruz se


obtiene
PM L* PM K * (10.21)
= .
w r
La ecuación 10.21 tiene una interpretación económica inmediata. Observe primero que PML* es
simplemente la producción adicional obtenida a partir de una unidad adicional de L en el punto de
minimización de costos. w es el costo en dólares de una unidad adicional de L. La relación PML*/w es
así la producción adicional que se obtiene del último dólar gastado en L. De modo semejante, PMK*/r
es la producción adicional que se obtiene del último dólar gastado en K. En palabras, la ecuación
10.21 dice que cuando los costos son mínimos, la producción adicional que se obtiene del último
dólar gastado en un insumo debe ser la misma para todos los insumos.
Es fácil demostrar por qué, si ése no fuera el caso, los costos no serían mínimos. Suponga, por
ejemplo, que las últimas unidades tanto de trabajo como de capital aumentaron la producción en 4
unidades. Es decir, suponga que PML = PMK = 4. Y de nuevo, suponga que r = 2 dólares y w = 4 dóla-
res. Entonces se habría obtenido sólo una unidad de producto por el último dólar gastado en L, pero
2 unidades por el último dólar gastado en K. Se podría reducir el gasto en L por un dólar, incrementar
el gasto en K por sólo 50 centavos y obtener el mismo nivel de producción que antes, ahorrando 50
centavos durante el proceso. Cada vez que los cocientes de los productos marginales entre los pre-
cios de los insumos difieran con cada uno de los insumos, siempre será posible hacer una sustitución
similar que ahorre costos a favor del insumo con el cociente PM/P más alto.10
Por lo general es posible considerar un proceso de producción que utilice no sólo dos sino N
insumos, X1, X2, ..., XN. En este caso la condición para la producción a un costo mínimo es una exten-
sión directa de la ecuación 10.21:
PM X PM X PM X
1
= 2
= … N
(10.22)
PX PX PX
1 2 N

10
De nuevo, esta afirmación es cierta excepto en el caso de las soluciones de esquina.

10_CHAPTER 10.indd 313 3/7/09 9:23:02 AM


314 Capítulo 10 Costos

¿Por qué la grava se fabrica a mano en Nepal pero con máquina en Estados Unidos?
Por simplicidad, suponga que el capital (K) y el trabajo (L) se emplean para transformar rocas en gra-
va, y que cualquiera de las combinaciones de insumos en la isocuanta denominada Q = 1 tonelada en
la figura 10.13 producirá una tonelada de grava. Así, la combinación denominada (L*EUA, K*EUA) po-
EL NATURALISTA dría corresponder a la técnica altamente intensiva en capital que se usa en Estados Unidos y (L*Nepal,
ECONÓMICO K*Nepal) a la técnica altamente intensiva en mano de obra que se usa en Nepal.
10.1
FIGURA 10.13
Diferentes maneras de
producir una tonelada K
de grava
los países donde la
mano de obra es barata
comparada con el capital
seleccionarán técnicas de
K*EUA
producción intensivas en
mano de obra. aquéllos Pendiente = –wEUA/r
donde el trabajo es más
caro emplearán técnicas
Pendiente = –wNepal/r
relativamente más
intensivas en capital. K*Nepal

Q = 1 ton
L
L*EUA L*Nepal

La razón por la cual las técnicas elegidas difieren


entre países no es que Estados Unidos sea más rico,
como yo originalmente había pensado cuando trabajé
en Nepal; más bien, es que los precios relativos del tra-
bajo y del capital difieren de manera muy marcada en
ambas naciones. En Nepal el trabajo es más barato que
en casi cualquier otra nación. Mientras viví allí pagué
10 centavos de dólar por un corte de pelo y un ajuste
quiropráctico del cuello (ambos realizados por la misma
persona). Los sueldos en Estados Unidos, por contraste,
están entre los más altos del mundo. El equipo de cons-
trucción se comercializa en los mercados mundiales y,
aparte de los costos de embarque, su precio no difiere
mucho de un país a otro. Si el precio del capital, r, es
aproximadamente el mismo en los dos países y el precio
del trabajo, w, es muy superior en Estados Unidos, se
deduce que la línea de isocosto es mucho más plana en
Nepal. Y como se muestra en la figura 10.13, este hecho
por sí solo es suficiente para explicar la diferencia dramá-
¿por qué la grava se fabrica manualmente en Nepal? tica en las técnicas de producción.

EJERCICIO 10.6
Suponga que el capital y el trabajo son complementos perfectos en una proporción uno a uno.
Es decir, suponga que Q = mín (L, K). Actualmente, el salario es w = 5 y la tasa de alquiler es r =
10. ¿Cual es el costo mínimo y el método que genera Q = 20 unidades de producto? Suponga
que el salario sube a w' = 20. Si se mantiene igual el costo total, ¿qué nivel de producción puede
conseguirse ahora y qué método de producción se usa (mezcla de insumos)?

10_CHAPTER 10.indd 314 3/7/09 9:23:04 AM


Costos a larGo plazo 315

EJERCICIO 10.7
Repita el ejercicio previo pero ahora suponga que el capital y el trabajo son sustitutos perfectos
en una proporción uno a uno: Q = K + L.

¿Por qué los sindicatos apoyan las leyes del salario mínimo con tal fuerza?
Los sindicatos obreros estadounidenses históricamente han estado entre los proponentes más activos
de la legislación del salario mínimo. Favorecen no sólo niveles más altos del salario mínimo, sino EL NATURALISTA
también una cobertura más amplia. Aun así, casi todos los miembros de los sindicatos Teamsters, ECONÓMICO
AFL-CIO, o United Auto Workers ya ganan mucho más que el salario mínimo y por esa razón no
se ven afectados de manera directa por cambios en la legislación. ¿Por qué, entonces, dedican un 10.2
esfuerzo y un gasto tan grandes a negociar a favor de los salarios mínimos?
Una razón podría ser que sus miembros están en verdad preocu-
pados por el bienestar económico de trabajadores menos afortunados
que ellos. Sin duda muchos sí se preocupan. Pero hay otros grupos
en desventaja (muchos de ellos merecen aun más ayuda que los tra-
bajadores de bajo salario) por quienes los sindicatos podrían haber
negociado. ¿Por qué la AFL-CIO no trabaja tan duro, por ejemplo,
tratando de obtener beneficios adicionales para niños sin hogar o para
los minusválidos?
Entender la condición para la producción a un costo mínimo ayu-
da a contestar estas preguntas. Observe primero que, en promedio, los
trabajadores afiliados tienden a poseer mejores habilidades que los no
afiliados. Los trabajadores no calificados y la mano de obra calificada
se sustituyen mutuamente en muchos procesos de producción, dando
lugar a isocuantas que tienen una forma parecida a la que se muestra
en la figura 10.14. La mezcla de las dos categorías de habilidad que
la empresa elige usar dependerá mucho de los precios relativos. La
figura 10.14 muestra la mezcla menos costosa para producir Q = Q0
tanto antes como después del establecimiento del salario mínimo. El ¿por qué los miembros de los sindicatos, que ganan mucho
salario para mano de obra calificada se representa por w. El salario más que el salario mínimo, favorecen el incremento del salario
de los trabajadores no calificados es w1, y se eleva a w2 después de la mínimo?
promulgación de la ley del salario mínimo. El efecto inmediato es au-
mentar el valor absoluto de la pendiente de la línea de isocosto de w1/w a w2/w , causando que la
empresa aumente el empleo de mano de obra calificada de S1 a S2 y reduce de manera simultánea el
empleo de trabajadores no calificados (no afiliados) de U1 a U2.

FIGURA 10.14
Trabajo calificado (S) El efecto de una ley de
salario mínimo sobre
el empleo de trabajo
calificado
El trabajo no calificado
y el trabajo calificado
S2 se sustituyen entre sí
Pendiente = –w2/w en muchos procesos
de producción. Cuando
Pendiente = –w1/w sube el precio del trabajo
no calificado, aumenta
S1 la pendiente de la línea
de isocosto, haciendo
Q = Q0 que muchas empresas
Trabajo no calificado (U) incrementen su empleo del
U2 U1 trabajo calificado (afiliado).

10_CHAPTER 10.indd 315 3/7/09 9:23:07 AM


316 Capítulo 10 Costos

Aunque la mayoría de los trabajadores afiliados no son afectados en forma directa por las leyes
de los salarios mínimos, éstas tienen la consecuencia indirecta de aumentar la demanda de trabajado-
res sindicalizados.11 Aun si los sindicatos carecieran de su preocupación declarada por el bienestar de
los trabajadores no calificados, en su mayoría trabajadores no sindicalizados, habría entonces poco
misterio acerca de por qué los sindicatos dedican tanto de sus recursos para apoyar la ampliación de
la legislación del salario mínimo.

¿Por qué un fabricante de equipos de baño pondría la imagen de una mosca en el centro de
sus mingitorios de cerámica?
La sustitución de capital por trabajo está algunas veces motivada
no por un cambio en los precios de los factores, sino por la intro-
EL NATURALISTA ducción de nuevas ideas. Considere, por ejemplo, el “proyecto
ECONÓMICO del inodoro oficial” iniciado por Jos van Bedaf, en ese tiempo
gerente de limpieza del aeropuerto Schiphol en Amsterdam.12
10.3 Su problema era que los sanitarios de caballeros del aeropuerto,
que eran usados por más de 100 000 personas al año, mostraban
una tendencia a ensuciarse y volverse malolientes a pesar de las
limpiezas frecuentes. La solución del señor Van Bedaf no fue
intensificar los esfuerzos de las brigadas de mantenimiento sino
¿Cómo reduce la imagen hacer un cambio menor en el equipo del sanitario. Pidió que
de una mosca los costos de el fabricante del equipo sanitario suministrara mingitorios al
mantenimiento de un aeropuerto? aeropuerto con la imagen de una mosca grabada al horno en el
centro de la superficie cerámica vitrificada de cada accesorio. Su
teoría fue que la presencia de este objetivo causaría que los parroquianos fueran mucho más exactos
en su uso de las instalaciones. ¿El resultado? Sanitarios mucho más limpios y 20% de reducción en los
costos de limpieza. Un periódico nacional en los Países Bajos clasificó a las instalaciones de Schiphol
en primer lugar en una lista de sanitarios limpios.

LA RELACIÓN ENTRE LA ELECCIÓN ÓPTIMA DE INSUMOS


Y LOS COSTOS A LARGO PLAZO
Con tiempo suficiente para ajustar, la empresa siempre puede comprar el paquete de insumos que
minimice los costos correspondientes a cualquier nivel determinado de producción y a los precios
relativos de los insumos. Para ver cómo los costos de la empresa varían con la producción a largo
plazo, es necesario comparar nada más los costos de los paquetes óptimos de insumos respectivos.
ruta de expansión de La curva denominada EE en la figura 10.15 muestra la ruta de expansión de la producción
la producción lugar de la empresa. Es el conjunto de paquetes de insumos que minimiza costos cuando el cociente del
geométrico de las tangencias precio de los insumos está fijo en w/r. Así, cuando el precio de K es r y el precio de L es w, la forma
(combinaciones de insumos más barata de producir Q1 unidades de producto es usar el paquete de insumos S, que contiene K1*
de costo mínimo) trazadas unidades de K, L1* unidades de L y cuesta CT1. El paquete S es por consiguiente un punto en la ruta
por una línea de isocostos de
de expansión de la producción. De manera similar, el nivel de producción Q2 está asociado con el
pendiente dada a medida que
paquete T, que tiene un costo total de CT2; Q3 está asociado con U, que cuesta CT3, etc. En la teoría
se desplaza hacia afuera dentro
del mapa de isocuantas para un
de comportamiento de la empresa, la ruta de expansión a largo plazo es análoga a la curva ingreso-
proceso de producción. consumo en la teoría del consumidor.
Para ir de la ruta de expansión a largo plazo a la curva de costo total a largo plazo, simplemente
se grafican los pares relevantes cantidad-costo de la figura 10.15. Así, el nivel de producción Q1 corres-
ponde a un costo total a largo plazo de CT1, Q2 a CT2, etc. El resultado es la curva rotulada CTL en la
parte superior en la figura 10.16. A largo plazo no hay necesidad para distinguir entre los costos total,
fijo y variable, ya que todos son variables.

11
Observe que en este ejemplo se supone que la empresa obtendrá el mismo nivel de producción después de que el salario mínimo
aumente. Sin embargo, como se verá en el próximo capítulo, la empresa por lo general obtendrá menos producción que antes. Si la
reducción de la producción es suficientemente grande, esto podría desalentar que la empresa cambie al trabajo calificado.
12
Este ejemplo se basa en Stefan Verhagen, “Fly in the Pot”, en Cornell Business, 21 de abril de 1992.

10_CHAPTER 10.indd 316 3/7/09 9:23:08 AM


Costos a larGo plazo 317

K FIGURA 10.15
La ruta de expansión a
CT3/r largo plazo
Con los precios de los
insumos r y w fijos, los
paquetes S,T, U y otros a lo
CT2/r largo del lugar geométrico
U E EE representan las maneras
menos costosas de generar
CT1/r T los niveles correspondientes
de producción.
S
K*1 Q3

E Q2
Q1
L
L* 1 CT1/w CT2/w CT3/w

FIGURA 10.16
Dólares/unidad de tiempo CTL Las curvas de costo
total, promedio y
marginal a largo plazo
a largo plazo, la empresa
siempre tiene la opción de
suspender las operaciones
y librarse de todos sus
CT3 insumos. Esto significa que
CT2 la curva de costo total a
CT1 largo plazo (parte superior)
siempre pasará por el
origen. las curvas de costo
Q marginal a largo plazo y
0 Q1 Q2 Q3 Q4 de costo promedio a largo
Dólares/unidad de producto plazo (parte inferior) se
CML obtienen de las curvas de
costo total a largo plazo de
una manera por completo
análoga al caso de corto
CPL plazo.

Q
0 Q1 Q2 Q3 Q4

La curva CTL siempre atravesará el origen porque a largo plazo la empresa puede liquidar todos
sus insumos. Si la firma elige no producir, no necesita retener o pagar por los servicios de ninguno
de sus insumos. La forma de la curva CTL que se observa en la parte superior se parece mucho a la
de la curva de costo total a corto plazo mostrada en la figura 10.2. Pero éste no siempre va a ser el
caso, como se verá ahora. Sin embargo, por el momento dé por sentada la forma de la curva CTL en
la parte superior de la figura 10.16 y pregúntese qué implica para las curvas de costo marginal y de
costo promedio a largo plazo.
De manera análoga para el caso a corto plazo, el costo marginal a largo plazo (CML) es la pen-
diente de la curva de costo total a largo plazo:
∆CTL Q
CML Q = . (10.23)
∆Q

10_CHAPTER 10.indd 317 3/7/09 9:23:10 AM


318 Capítulo 10 Costos

En palabras, CML es el costo para la empresa, a largo plazo, de expandir su producción por una
unidad.
El costo promedio a largo plazo (CPL) es el cociente del costo total a largo plazo entre la pro-
ducción:
CTL Q
CPL Q = . (10.24)
Q
De nuevo, no hay necesidad de discutir las diferencias entre los costos promedio total, fijo y variable
porque todos los costos a largo plazo son variables.
La parte inferior en la figura 10.16 muestra las curvas CPL y CML que corresponden a la curva
CTL de la parte superior. La pendiente de la curva CTL disminuye hasta el nivel de producción Q1 y
aumenta después, lo cual quiere decir que la curva CML toma su valor mínimo en Q1. La pendiente
de CTL y la pendiente del rayo a CTL son iguales en Q3, lo cual quiere decir que CPL y CML se inter-
secan en ese nivel de producción. Y, otra vez como antes, la relación tradicional marginal-promedio
es válida: CPL disminuye siempre que CML está situada debajo de ella y sube siempre que CML está
situada por encima de ella.
Para una función de producción con rendimientos constantes de escala la duplicación de la pro-
ducción duplica de manera exacta los costos.13 Triplicar todos los insumos triplica la producción y
también los costos, etc. Por lo tanto, para el caso de los rendimientos constantes de escala, los costos
totales a largo plazo son exactamente proporcionales a la producción. Como lo muestra la figura
10.17a, la curva CTL para una función de producción con rendimientos constantes de escala es una
línea recta que pasa por el origen. Como la pendiente de CTL es constante, la curva asociada CML es
una línea horizontal, y es exactamente igual que la curva CPL (figura 10.17b).

FIGURA 10.17
Las curvas CTL, CML y Dólares/unidad de tiempo Dólares/unidad de producto
CPL con rendimientos
constantes
de escala CTL
a) Con rendimientos
constantes, la curva
total a largo plazo es
estrictamente proporcional
CPL = CML
a la producción. b) El costo
marginal a largo plazo es Pendiente = CML = CPL
constante e igual al costo Q Q
promedio a largo plazo. 0 0
a) b)

Cuando la función de producción tiene rendimientos decrecientes de escala, un incremento pro-


porcional dado en la producción requiere un mayor incremento proporcional en todos los insumos
y por lo tanto un mayor aumento proporcional en los costos. En la figura 10.18 se muestran curvas
CTL, CML y CPL para una función de producción con rendimientos decrecientes de escala. Para la
curva específica CTL en la figura 10.18a, las curvas asociadas CPL y CML resultan ser lineales (figura
10.18b), pero esto no ocurre siempre. La propiedad general del caso de los rendimientos decrecientes
es que da lugar a una curva CTL con curvatura ascendente y a curvas CPL y CML con pendiente
positiva. Observe además otra aplicación de la relación marginal-promedio: el hecho de que CML
exceda a CPL asegura que esta última deba subir con la producción.
Considere por último el caso de los rendimientos crecientes de escala. Aquí la producción au-
menta en mayor proporción respecto al incremento en los insumos. En consecuencia, el costo total a

13
Se supone, por supuesto, que los precios de los insumos permanecen iguales a medida que varía la producción.

10_CHAPTER 10.indd 318 3/7/09 9:23:12 AM


los Costos a larGo plazo y la EstruCtura DE la INDustrIa 319

FIGURA 10.18
Las curvas CTL,
Dólares/unidad de tiempo Dólares/unidad de producto
CPL y CML para un
CML proceso de producción
CTL con rendimientos
CPL decrecientes de escala
Con rendimientos
decrecientes, la producción
crece menos que en
proporción al crecimiento
de los insumos, lo cual
significa que el costo
Q Q total crece más que
0 0
a) b) en proporción con
el crecimiento de la
producción.

FIGURA 10.19
Las curvas CTL, CPL y
Dólares/unidad de tiempo Dólares/unidad de producto
CML para un proceso
de producción con
rendimientos crecientes
CTL
de escala
Con rendimientos
crecientes, la empresa a
CPL gran escala tiene costos
marginal y promedio
CML más bajos que la firma a
pequeña escala.
Q Q
0 0
a) b)

largo plazo sube en menor proporción respecto a los incrementos en la producción, como se muestra
en la figura 10.19a. Las curvas asociadas CPL y CML se observan en la figura 10.19b. La característica
distintiva de las curvas CPL y CML con rendimientos crecientes de escala no es la forma lineal mos-
trada en este ejemplo particular, sino el hecho de que tienen pendiente negativa.
Los procesos de producción cuyas curvas de costos a largo plazo se ilustran en las figuras 10.17,
10.18 y 10.19 son “casos puros”, que exhiben rendimientos constantes, decrecientes y crecientes de
escala, respectivamente, para sus rangos completos de producción. Sin embargo, como se discutió
en el capítulo 9, el grado de rendimientos de escala de una función de producción no necesita ser el
mismo para la gama completa de producción.

LOS COSTOS A LARGO PLAZO


Y LA ESTRUCTURA DE LA INDUSTRIA
Como se observó en la visión preliminar de este capítulo, los costos a largo plazo son importantes
por su efecto en la estructura de la industria. Un análisis detallado de este papel será el tema de
los siguientes capítulos. Aquí, una visión general breve de los aspectos clave ayudará a establecer el
escenario para esa discusión.
Si, como en la figura 10.20a, hay en todos los casos costos promedio a largo plazo decrecientes,
la tendencia será que una sola empresa sirva al mercado. Si dos compañías intentaran atender a este
mercado y cada una de ellas produjera sólo una parte de la producción total vendida, tendrían costos

10_CHAPTER 10.indd 319 3/7/09 9:23:14 AM


320 Capítulo 10 Costos

monopolio natural industria promedio más altos que si sólo existiera una de ellas. La tendencia en tal mercado será que la empre-
cuya producción para el sa que resulte hacerse más grande tendrá una ventaja en el costo que le permitirá eliminar a su rival.
mercado se genera al costo Por esta razón los mercados caracterizados por curvas de costo promedio a largo plazo decrecientes
más bajo cuando la producción se denominan a menudo monopolios naturales.
se concentra en las manos de
una sola firma.

FIGURA 10.20
Las curvas CPL son
características de
estructuras industriales
altamente concentradas
a) las curvas Cpl que Dólares/Q Dólares/Q
tienen pendiente negativa
tienden a ser características
de los monopolios
naturales. los costos CPL
unitarios son los más bajos
cuando sólo una firma sirve
al mercado completo. b) las CPL
curvas Cpl con forma de
u cuyos puntos mínimos Q Q
ocurren para una parte 0 0 Q0
sustancial de la producción a) b)
total del mercado son
características de los
mercados atendidos sólo
por un puñado de firmas.

Considere ahora la curva CPL que se muestra en la figura 10.20b cuyo punto mínimo ocurre en
el nivel de producción Q0. En dicho nivel la empresa logra su costo de producción por unidad más
bajo posible. Q0 puede llamarse la escala mínima eficiente: el nivel de producción requerido por CPL
para alcanzar su punto mínimo. Si Q0 constituye una parte sustancial de la producción de la industria
(por ejemplo, más que 20%) la industria tenderá a ser dominada por un grupo pequeño de compa-
ñías. Como en el caso del monopolio natural, un gran número de empresas pequeñas tendría poca
probabilidad de sobrevivir en tal mercado, porque cada una tendría costos promedio mucho más
altos que las empresas más grandes. Sin embargo, en contraste con el caso del monopolio natural,
el incremento en el CPL más allá de Q0 dificultará que una empresa sirva al mercado completo.
Es posible que los mercados atendidos por compañías con curvas CPL como la de la figura 10.20b
sean “altamente concentrados”, lo cual quiere decir que un pequeño número de empresas tenderá a
representar la mayor parte de toda la producción vendida.
Es probable que la curva de costo promedio a largo plazo asociada con un mercado servido por
muchas empresas adopte una de las tres formas que se observan en la figura 10.21. Si Q0, el punto
mínimo en la curva de costo promedio con forma de U en el inciso a), constituye sólo una fracción
pequeña de la producción total, se espera ver una industria poblada por numerosas empresas, cada
una de las cuales genera sólo un pequeño porcentaje de producción de la industria total. El tamaño
pequeño tampoco es una desventaja cuando el proceso de producción da lugar a una curva horizon-
tal CPL como la que se muestra en la gráfica b). En tales procesos todas las compañías (grandes o
pequeñas) tienen los mismos costos unitarios de producción. Para la curva CPL con pendiente posi-
tiva que se muestra en la gráfica c) de la figura 10.21, el tamaño pequeño no es sólo compatible con
la supervivencia en el mercado sino que se requiere imperativamente, ya que las empresas grandes
siempre tendrán costos promedio más altos que las más pequeñas. Sin embargo, como una cuestión
práctica, es muy improbable que alguna vez pueda haber una curva CPL que tenga pendiente positi-
va aun para niveles sumamente bajos de producción. (Imagine, por ejemplo, los costos unitarios de
una empresa que tratara de producir 100—1 de una libra de azúcar.)

10_CHAPTER 10.indd 320 3/7/09 9:23:15 AM


la rElaCIóN ENtrE las CurVas DE Costo a larGo plazo y a Corto plazo 321

FIGURA 10.21
Curvas CPL
características de
estructuras industriales
no concentradas
El requisito para la
Dólares/Q Dólares/Q Dólares/Q supervivencia en cualquier
mercado es que una firma
tenga los costos unitarios
CPL más bajos posibles. si el
punto mínimo de una Cpl
con forma de u (Q0 en
CPL CPL la gráfica a) ocurre para
un fracción pequeña de la
producción del mercado o
si la Cpl es lineal en todos
Q Q Q lados o creciente (gráficas
Q0 b y c, respectivamente),
a) b) c) entonces son compatibles
el tamaño pequeño y la
sobrevivencia. Cada firma
tenderá a producir nada
más una pequeña parte
de la producción total del
mercado.

La relación entre la estructura del mercado y la forma de la curva de costo promedio a largo pla-
zo se deriva del hecho de que, frente a la competencia, la supervivencia del mercado requiere que las
empresas tengan los costos unitarios más bajos posibles con la tecnología de producción existente.
El que esto ocurra para niveles altos o bajos de producción depende por completo de la forma de la
curva CPL.

LA RELACIÓN ENTRE LAS CURVAS DE COSTO


A LARGO PLAZO Y A CORTO PLAZO
Una forma de considerar a la curva CPL es como una “envolvente” de todas las curvas de costo
total promedio a corto plazo (CTP). Suponga que las curvas CTP que corresponden a 10 000 niveles
diferentes de K se trazaran en un diagrama como la figura 10.22. Si luego se tomara una cuerda y se
amoldara al contorno exterior de estas curvas CTP, trazaría la forma de la curva CPL. Observe en
la figura 10.22 que para el nivel de producción para el cual un CTP dado es tangente a CPL, el costo
marginal a largo plazo (CML) para producir dicho nivel de producción es igual que el costo marginal
a corto plazo (CMC). Así, CML(Q1) = CMC(Q1), CML(Q2) = CMC(Q2) y CMC(Q3) = SCM(Q3).14 Ob-
serve también que cada punto a lo largo de una curva dada CTP, excepto el de tangencia, está situado
encima del punto correspondiente en la curva CPL. Por último, observe que para el punto mínimo
en la curva CPL en la figura 10.22 (Q = Q2), todos los costos promedio y marginal a largo y a corto
plazos toman exactamente el mismo valor.
Se puede aplicar algo de intuición acerca de la relación CTP-CPL para cierta curva CTP si se
observa que a la izquierda de la tangencia CTP-CPL, la empresa tiene “demasiado” capital, con el
resultado de que sus costos fijos son superiores a lo necesario; y que a la derecha de la tangencia tiene
“muy poco” capital, a fin de que los rendimientos decrecientes del trabajo hagan subir sus costos.
Sólo en el punto de tangencia la empresa tiene las cantidades óptimas de trabajo y capital para gene-
rar el nivel correspondiente de producción.

14
Estas relaciones se desarrollan con mayor detalle en el apéndice de este capítulo.

10_CHAPTER 10.indd 321 3/7/09 9:23:16 AM


322 Capítulo 10 Costos

FIGURA 10.22
La familia de curvas de Dólares/Q CMC3
costo asociada con una
CPL con forma de U
la curva Cpl es la
CMC1
“envolvente exterior” de CTP1
las curvas Ctp. CMl =
CMC2 CTP3
CMC para el valor Q en el CTP2
cual el Ctp es tangente a
la Cpl. En el punto mínimo
en la Cpl, CMl = CMC =
Ctp = Cpl.
CPL

CML

Q
Q1 Q2 Q3

El apéndice de este capítulo considera la relación entre los costos a largo y a corto plazos con
mayor detalle. También desarrolla el enfoque del cálculo diferencial para la minimización de costos.

RESUMEN
• De todos los temas incluidos en un texto intermedio de mi- la forma de una hipérbola rectangular que tiende a infinito a
croeconomía, los estudiantes suelen encontrar el material de medida que la producción disminuye hacia cero, y desciende
curvas de costos como el más difícil de digerir. Y por una buena hacia cero a medida que la producción crece.
razón, ya que sólo el volumen de conceptos específicos puede • El problema de asignar una cuota dada de producción a dos
parecer abrumador en el primer encuentro. Por consiguiente instalaciones diferentes de producción es similar al proble-
es importante tener en mente que todas las diversas curvas de ma de asignar un insumo disponible a dos instalaciones dife-
costos pueden obtenerse de las relaciones de producción sub- rentes. En este último caso el objetivo es maximizar la can-
yacentes de una manera simple e inmediata. tidad de producción que puede generarse con una cantidad
• Por ejemplo, todas las curvas de costo a corto plazo se obtie- dada de insumo. En el primer caso se debe obtener un nivel da-
nen directamente de la función de producción a corto plazo. do de producción al costo total más bajo. La solución es asig-
Todas las funciones de producción a corto plazo que se han nar la cuota de producción a fin de que el costo marginal sea
discutido incluyeron un factor fijo y uno variable, pero la teo- el mismo en cada proceso de producción. Esta solución no re-
ría sería exactamente igual en el caso de más de un insumo quiere que los costos promedio sean los mismos en cada pro-
fijo. Los costos totales a corto plazo se descomponen en costos ceso y, en la práctica, a menudo difieren de manera sustancial.
fijos y variables que corresponden, respectivamente, a los pa- • El paquete óptimo de insumos para producir un nivel dado de
gos para los factores de producción fijos y variables. Por la ley producción a largo plazo dependerá de los precios relativos
de los rendimientos decrecientes, más allá de algún punto se de los factores de producción. Estos precios relativos determi-
requieren incrementos aún mayores del insumo variable para nan la pendiente de la línea de isocosto, que es el lugar geomé-
producir una unidad adicional de producto. El resultado es trico de los paquetes de insumos que pueden comprarse a un
que el costo marginal a corto plazo, que es la pendiente de la costo total dado. El paquete óptimo de insumos será el que
curva de costo total a corto plazo, aumenta con la producción esté situado en el punto de tangencia entre una línea de isocos-
en la región de los rendimientos decrecientes. Estos últimos to y la isocuanta deseada. En el punto que minimiza costo, el
son también responsables del hecho de que las curvas de costo cociente del producto marginal de un insumo entre su precio
total promedio a corto plazo y de costo variable (que son, res- será el mismo para cada insumo. Dicho de otra manera, la
pectivamente, las pendientes de los rayos a las curvas de costo producción adicional obtenida del último dólar gastado en un
total y de costo variable a corto plazo) a la larga aumentan insumo debe ser igual que la producción adicional obtenida
con la producción. Los costos fijos promedio siempre adoptan del último dólar gastado en cualquier otro insumo. Otra ma-

10_CHAPTER 10.indd 322 3/7/09 9:23:17 AM


problEMas 323

nera de indicar la condición de costo mínimo es que la tasa horizontal. Un CPL con forma de U corresponde a un proceso
marginal de sustitución técnica en el paquete de optimización de producción que exhibe primero rendimientos crecientes de
debe ser igual que la pendiente de la línea de isocosto. escala, luego constantes y, por último, decrecientes. Al margen
• Estas propiedades de producción a un costo mínimo ayudan a de su forma, la curva CPL siempre será una envolvente de la
entender por qué los métodos de producción a menudo difie- familia correspondiente de curvas CTP, cada una de las cuales
ren mucho cuando los precios relativos de los factores difieren será tangente al CPL en un punto y sólo uno. Para los niveles
apreciablemente. Se vio, por ejemplo, que ayuda a explicar por de producción que corresponden a estos puntos de tangencia,
qué los países en vías de desarrollo a menudo usan técnicas el CML y el CMC correspondiente serán iguales.
intensivas en mano de obra mientras que los países industriali- • La relación entre la estructura de mercado y los costos a largo
zados escogen aquellas que son mucho más intensivas en capi- plazo se deriva del hecho de que la supervivencia en el merca-
tal, y por qué los sindicatos con frecuencia negocian a favor de do requiere que las empresas tengan los costos más bajos posi-
salarios mínimos elevados, si bien, para comenzar, casi todos bles con las tecnologías disponibles de producción. Si la curva
sus miembros ganan más que el salario mínimo. CPL tiene pendiente negativa, los costos más bajos ocurren
• Para un nivel dado de producción los costos totales a largo cuando una sola empresa sirve al mercado. Si la curva CPL
plazo nunca pueden ser mayores que los costos totales a corto tiene forma de U y su punto mínimo ocurre para una canti-
plazo por la simple razón que se tiene la oportunidad de ajustar dad que corresponde a una parte sustancial de la producción
todos los insumos a largo plazo y sólo algunos de ellos a cor- total del mercado, los costos más bajos ocurrirán cuando sólo
to plazo. La pendiente de la curva de costo promedio a largo algunas empresas sirvan al mercado. En contraste, si el punto
plazo es un reflejo directo del grado de rendimientos a escala mínimo en una curva CPL con forma de U corresponde sólo
en la producción. Cuando hay rendimientos crecientes, CPL a una fracción pequeña de la producción total de la industria,
disminuye con la producción. Con rendimientos decrecientes, es posible que el mercado sea atendido por muchas empresas
por contraste, CPL aumenta con la producción. Por último, los competidoras. Lo mismo será cierto cuando la curva CPL sea
rendimientos constantes en la producción dan lugar a un CPL horizontal o tenga pendiente positiva.

pREgUNtaS dE REpaSo
1. ¿Cuál es la relación entre la ley de rendimientos decrecientes y 4. ¿Por qué la curva CM a corto plazo corta a las curvas CTP y
la curvatura de la curva de costo variable? CVP en sus puntos mínimos?
2. ¿Cuál es la relación entre la ley de rendimientos decrecientes y 5. Si la curva CPL sube más allá de algún punto, ¿qué se puede decir
la pendiente de la curva de costo marginal a corto plazo? acerca del grado de los rendimientos a escala en la producción?
3. ¿Para qué proceso de producción es probable que el costo fijo 6. ¿Por qué la generación de una cantidad fija de producto debe
sea un porcentaje mayor de los costos totales a corto plazo, la repartirse entre dos actividades de producción a fin de que el
publicación de libros o la jardinería? costo marginal sea el mismo en cada una?

pRoblEMaS
1. En la siguiente tabla se han incluido de manera parcial los datos de los costos de la Compañía de
Embalsamado para la preservación. Después de la muerte repentina e inesperada del contador de la
compañía, se le solicita a usted que complete los ingresos que faltan.

Cuerpos Costo Costo Costo


CTP CVP CFP CM
embalsamados total fijo variable
0 24 — — —
16
1
2 50
3 108
52
4
5 39.2
6 47

10_CHAPTER 10.indd 323 3/7/09 9:23:18 AM


324 Capítulo 10 Costos

2. Trace las curvas CT, CV, CF, CTP, CVP, CFP y CM para la función de producción

Q = 3KL,

donde K está fijo en 2 unidades a corto plazo, con r = 3 y w = 2.


3. Si el producto promedio del trabajo es igual que el producto marginal del trabajo, ¿cómo es el costo
marginal comparado con el costo variable promedio?
4. Una empresa tiene acceso a dos procesos de producción con las siguientes curvas de costo marginal:
CM1 = 0.4Q y CM2 = 2 + 0.2Q.
a) Si quiere fabricar 8 unidades de producto, ¿cuánto deberá producir con cada proceso?
b) ¿Y si quiere fabricar 4 unidades de producto?
5. Una empresa usa dos insumos, K y L, en su proceso de producción y encuentra que independiente-
mente de cuánta producción obtenga o de cómo varíen los precios de los insumos, siempre minimi-
za sus costos si compra sólo uno de los dos insumos. Dibuje el mapa de isocuantas de esta empresa.
6. Una empresa encuentra que al margen de cuánta producción obtenga y de cómo varíen los precios
de los insumos, siempre minimiza sus costos si compra las unidades de capital en una cantidad que
sea la mitad de las unidades de trabajo. Dibuje el mapa de isocuantas de esta empresa.
7. Una compañía compra capital y trabajo en los mercados competitivos a precios de r = 6 y w = 4,
respectivamente. Con la mezcla actual de insumos de la empresa el producto marginal del capital
es 12 y el producto marginal del trabajo es 18. ¿Está minimizando sus costos esta empresa? Si es así,
explique cómo lo sabe. En caso de que no sea así, explique qué debe hacer la empresa.
8. Una compañía tiene una función de producción Q = F(K, L) con rendimientos constantes de escala.
Los precios de los insumos son r = 2 y w = 1. La ruta de expansión de la producción para esta función
con estos precios de los insumos es una línea recta que pasa por el origen. Cuando produce 5 unida-
des de producto, usa 2 unidades de K y 3 unidades de L. ¿Cuánto usará de K y L cuando su costo total
a largo plazo sea igual a 70?
9. Una empresa con la función de producción Q = F(K, L) produce un nivel de producción Q* a un costo
mínimo a largo plazo. ¿Cómo es su costo marginal a corto plazo cuando K está fijo comparado con
su costo marginal a corto plazo cuando L está fijo?
10. Una empresa usa una función de producción Q = F(K, L) para la cual sólo son posibles dos valores de
K: K1 y K2. Su curva CTP cuando K = K1 está dada por CTP1 = Q2 – 4Q + 6. La curva correspondiente
para K = K2 es CTP2 = Q2 – 8Q + 18. ¿Cuál es la curva CPL de esta firma?
11. Si la curva CML de una empresa está situada por encima de su curva CMC para un nivel dado de
producción, ¿cuál será la relación entre sus curvas CTP y CPL para ese nivel de producción?
*12. Una empresa tiene una función de costo total a largo plazo:

CTL(Q) = Q3 – 20Q2 + 220Q.

Obtenga expresiones para el costo promedio y el costo marginal a largo plazo, y trace estas curvas.
*13. Para la función de costo total a largo plazo

CTL(Q) = Q2 + 10,
trace CTP, CVP, CFP y CM.

* Estos problemas se resuelven con mayor facilidad si se usa la definición del costo marginal que se obtiene con el cálculo diferen-
cial.

10_CHAPTER 10.indd 324 3/7/09 9:23:18 AM


rEspuEstas a los EjErCICIos DEl Capítulo 325

RESpUEStaS a loS EjERcicioS dEl capítUlo


10.1. La curva de costos variables es la misma que antes; las curvas CF y CT están desplazadas hacia
arriba 8 unidades. (Vea la siguiente gráfica.)

CT

Dólares/hora

16 CF
CV

0 Q
4

10.2. La distancia vertical entre las curvas CTP y CVP es CFP. Así que se tiene que CTP10 – CVP10 =
CF/10 = 20/10 = 2.
10.3. Si se igualan los costos marginales, se tiene 12QA = 4QB. Al sustituir QB = 12 – QA se obtiene 12QA =
48 – 4QA, cuya solución es QA = 3. QB = 12 – 3 = 9 considera la producción restante, y para estos
niveles de producción el costo marginal en ambas plantas será de 36 dólares/unidad de producto.
10.4. Cuando el producto marginal está por encima del producto promedio, el costo marginal está deba-
jo del costo variable promedio. (Vea figura 10.9.)
10.5.

K
Gasto total = 90

Gasto total = 180


180/6 = 30

Pendiente = – w/r = –1/2

90/6 = 15

L
90/3 = 30 180/3 = 60

10.6. Para producir 20 unidades de producto, se necesitan L = K = 20. Como r = 10 y w = 5, los costos
son

C = 10K + 5L = 200 + 100 = 300,

que puede rescribirse como K = 30 – –2¹ L. Cuando el salario se eleva a w = 20, mantener los costos en
C = 300 requiere encontrar el punto en el cual K = L en la curva nueva de isocosto

C = 10K + 20L = 300,

que puede rescribirse como K = 30 – 2L. Si se hace K = L, se tiene

10K + 20L = 300 = 10L + 20L = 300 = 30L = 300, de modo que L = 10.

10_CHAPTER 10.indd 325 3/7/09 9:23:20 AM


326 Capítulo 10 Costos

Así, L = K = 10 y se produce Q = 10.

K
30 K=L

A
20
C = 10K + 5L
B
10
C' = 10K + 20L

L
0 10 20 60

10.7. Para producir 20 unidades de producto, se necesitará L = 20 o K = 20. Como r = 10 y w = 5, los


costos son

C = mín{10K, 5L} = mín{200, 100} = 100.

Cuando el salario sube a w = 20, mantener los costos en C = 100 implica que

100 100 
Q = máx  ,  = máx {10, 5} = 10.
 r w 

Así, no se usa trabajo (L = 0), sólo capital (K = 10) y se produce Q = 10.

K
20

10 C = 10K + 5L K = 20 – L

C' = 10K + 20L K = 10 – L


L
0 5 10 20

10_CHAPTER 10.indd 326 3/7/09 9:23:23 AM


a p é N d i c E

10
ExtEnsionEs matEmátiCas
dE la tEoría dE Costos

LA RELACIÓN ENTRE LAS CURVAS A LARGO


PLAZO Y LAS CURVAS A CORTO PLAZO
Considere primero con mayor detalle la relación entre los costos totales a largo y a corto plazos.
Recuerde que la curva CTL se genera al graficar el valor de Q para una isocuanta frente al nivel de
costo total correspondiente para la línea de isocosto tangente a dicha isocuanta. Así, por ejemplo, en
la figura A.10.1, Q = 1 se asocia con un costo total a largo plazo de CTL1, Q = 2 con CTL2, etcétera.
Cuando K es variable, tal como éste y todos los demás factores lo son a largo plazo, la ruta de
expansión está dada por la línea 0E. Sin embargo, ahora suponga que K está fija en K*2, el nivel óptimo
para la producción de Q = 2. La ruta de expansión a corto plazo será entonces la línea horizontal a
través del punto (0, K*2), que incluye los paquetes de insumos X, T y Z. El costo total a corto plazo de
generar cierto nivel de producción, por ejemplo, Q = 1, es simplemente el costo total asociado con la
línea de isocosto que atraviesa la intersección de la ruta de expansión a corto plazo y la isocuanta Q =
1 (el punto X en la figura A.10.1), es decir, CTC1.
Observe en la figura A.10.1 que los costos totales a corto y a largo plazos toman el mismo valor
para Q = 2, el nivel de producción para el cual se cruzan las rutas de expansión a corto y a largo
plazos. Para todos los demás niveles de producción, la línea de isocosto que atraviesa la intersección
de la isocuanta correspondiente y la ruta de expansión a corto plazo estará situada por encima de la
línea de isocosto que es tangente a la isocuanta. Así, para todos los niveles de producción excepto Q
= 2, el costo total a corto plazo será superior al costo total a largo plazo.
327

10_CHAPTER 10.indd 327 3/7/09 9:23:24 AM


328 Capítulo 10 apéNDICE 10: ExtENsIoNEs MatEMátICas DE la tEoría DE Costos

FIGURA A.10.1
Rutas de expansión a K
corto y a largo plazos
la ruta de expansión a CTC3
largo plazo es la línea 0E. r
Con K fijo en K*2, la ruta de CTL3
r
expansión a corto plazo
es una línea horizontal
que pasa por el punto
(0, K*2). Como K*2 es la E
cantidad óptima de K para
producir 2 unidades de CTL2 CTC2 Ruta de expansión a largo plazo
producto, las rutas de r = r
expansión a largo y a corto
plazos se intersecan en
T. El costo total a corto Ruta de expansión a corto plazo
U
plazo de producir cierto CTC1
nivel de producción está r T
X Z
asociado con la línea de K = K*2
CTL1
isocostos que atraviesa la r
intersección de la isocuanta Q=3
relevante y la ruta de
expansión a corto plazo.
así, por ejemplo, CtC3 es S Q=2
el costo total a corto plazo Q=1
de producir 3 unidades de L
producto. 0 CTL1 CTC1 CTL3 CTC3
w w w w
CTL2 CTC2
w = w

Las curvas de costo total a corto y a largo plazos que corresponden al mapa de isocuantas de la
figura A.10.1 se muestran en la figura A.10.2. Observe en la figura A.10.1 que mientras cuanto más
se acerque la producción a Q = 2, más pequeña será la diferencia entre el costo total a largo plazo y
a corto plazo. Esta propiedad se refleja en la figura A.10.2 por el hecho de que la curva CTC es tan-
gente a la curva CTL en Q = 2: entre más cercana sea Q a 2, CTCQ será más cercano a CTC2. Observe
también en la figura A.10.2 que la curva CTC interseca el eje vertical en rK*2, el costo fijo asociado con
K*2 unidades de K.

FIGURA A.10.2
Las curvas CTL y CTC Dólares/unidad de tiempo
asociadas con el mapa
de isocuantas en la
CTC
figura A.10.1
CTC3
a medida que Q se CTL
aproxima a 2, el nivel de
producción para el cual el CTL3
factor fijo está en su nivel
óptimo, CtCQ se aproxima
a CtlQ. las dos curvas son CTC2 = CTL2
tangentes en Q = 2.
CTC1
rK*2
CTL1

Q
1 2 3

10_CHAPTER 10.indd 328 3/7/09 9:23:26 AM


la rElaCIóN ENtrE las CurVas a larGo plazo y las CurVas a Corto plazo 329

FIGURA A.10.3
Dólares/Q
Las curvas CPL y CML
y dos curvas CTP
asociadas con las curvas
de costo de la figura
A.10.2
El costo promedio a corto
plazo es tangente al costo
CTPK = K*2 CTPK = K*3 promedio a largo plazo
en el mismo nivel de
producción para el cual las
curvas correspondientes
CPL = CML Ctl y CtC son tangentes.

Q
2 3

El proceso de producción cuyo mapa de isocuantas se muestra en la figura A.10.1 resulta tener
rendimientos constantes de escala. En consecuencia, sus curvas de costos promedio y marginal a lar-
go plazo serán la misma línea horizontal. La posición de esta línea está determinada por la pendiente
de la curva CTL en la figura A.10.2. La curva CTP asociada tendrá forma de U y será tangente a la
curva LAC en Q = 2, como se muestra en la figura A.10.3.
Hay curvas de costos a corto plazo no sólo para K = K*2, sino también para cualquier otro nivel
del insumo fijo. Por ejemplo, la curva de costo promedio a corto plazo cuando K está fijo en K*3 (la
cantidad óptima de K para producir Q = 3) se muestra en la figura A.10.3 como la curva rotulada
CTPK = K*3. Igual que la curva CTP asociada con K*2 también tiene forma de U y es tangente a la curva
CPL para Q = 3. Las curvas CTP en general tienen forma de U y son tangentes a la curva CPL en el
nivel de producción para el cual la magnitud del insumo fijo resulta ser óptimo.
Existe una relación similar en el caso de los procesos de producción que dan lugar a curvas CPL
en forma de U. Para tal proceso la curva CPL y tres de sus curvas CTP asociadas se muestran en la
figura A.10.4. Observe que cuando la curva CPL tiene forma de U, las tangencias entre ella y las cur-
vas CTP asociadas por lo general no ocurren en los puntos mínimos sobre las curvas CTP. La única

Dólares/Q CMC3 FIGURA A.10.4


La familia de curvas de
costo asociadas con una
CPL con forma de U
CMC1 la curva Cpl es la
CTP1 “envolvente exterior” de
CMC2 CTP3 las curvas Ctp, CMl =
CTP2 CMC en el valor de Q para
el cual la Ctp es tangente
a la Cpl. para el punto
mínimo en la Cpl, CMl =
CMC = Ctp = Cpl.
CPL

CML

Q
Q1 Q2 Q3

10_CHAPTER 10.indd 329 3/7/09 9:23:28 AM


330 Capítulo 10 apéNDICE 10: ExtENsIoNEs MatEMátICas DE la tEoría DE Costos

excepción es la curva CTP que es tangente al punto mínimo de la CPL con forma de U (CTP2 en la
figura A.10.4). En la porción con pendiente negativa de la curva CPL, las tangencias estarán situadas
a la izquierda de los puntos mínimos de las correspondientes curvas CTP; y en la porción con pen-
diente positiva de la curva CPL, las tangencias estarán situadas a la derecha de los puntos mínimos.
En el texto se hizo notar que una manera de considerar la curva CPL es como una “envolvente”
de todas las curvas CTP, como la que se muestra en la figura A.10.4. En el nivel de producción para
el cual un CTP dado es tangente a la CPL, el costo marginal a largo plazo (CML) necesario para
generar esa producción es igual que el costo marginal a corto plazo (CMC). Para ver por qué esto es
así, recuerde que el punto de tangencia representa la cantidad que es óptima para el nivel de factor
fijo que corresponde a la curva CTP específica. Si se cambia la producción por una cantidad muy
pequeña a corto plazo —ya sea aumentando o reduciendo la cantidad del insumo variable— se ter-
minará con una mezcla de insumos que es sólo marginalmente diferente de la óptima, y cuyo costo
es por consiguiente más o menos el mismo que el de la mezcla óptima. En consecuencia, para niveles
de producción muy cercanos al punto de tangencia relevante, CMC y CML son aproximadamente
iguales.
Observe también en la figura A.10.4 que las curvas CMC siempre son más inclinadas que la
curva CML. La razón está implícita en esta discusión de por qué CML y CMC son casi iguales en la
vecindad de los puntos de tangencia. Si comienza en un punto de tangencia —por ejemplo, en Q1 en
la figura A.10.4— suponga que desea producir una unidad adicional de producto a corto plazo. Para
hacerlo, tendrá que moverse de una mezcla de insumos óptima a otra que contenga un poco más L
y escasamente menos K para producir Q1 + 1 a largo plazo. Así que el costo de esa unidad adicional
será mayor a corto plazo que a largo plazo, lo cual es otra forma de decir que CMCQ1 + 1 > CMLQ1 + 1.
Ahora suponga que comienza en Q1 y quiere producir una unidad de producto menos que antes.
Para hacerlo tendrá que moverse a un paquete de insumos que contenga menos L y más K que el
óptimo para producir Q1 – 1. En consecuencia, los ahorros de costos serán más pequeños a corto
plazo que a largo plazo cuando se tiene libertad de ajustar L y K. Esto quiere decir que CMLQ1 – 1 >
CMCQ1 + 1. Expresar que CML excede a CMC cada vez que la producción es menor que Q1, pero es
menor que CMC cuando la producción es mayor que Q1, es lo mismo que decir que la curva CML
es menos inclinada que la curva CMC en Q1.

EJERCICIO A.10.1
Considere una función de producción Q = F (K, L) para la cual sólo son posibles dos valores de
K. Éstos dan lugar a las curvas CTP que se muestran en el diagrama. ¿Cuál es la curva CPL para
esta empresa?

Dólares/unidad de producto
CTP1

CTP2

EL ENFOQUE DEL CÁLCULO APLICADO


A LA MINIMIZACIÓN DE COSTOS
Si se usa la técnica de Lagrange que se discutió en el apéndice del capítulo 3, es posible demostrar
que la igualdad de las proporciones PM/P (ecuación 10.22) emerge como una condición necesaria
para el siguiente problema de minimización de costos:
mín PKK + PLL sujeto a F (K, L) = Q0. (A.10.1)
K, L

10_CHAPTER 10.indd 330 3/7/09 9:23:29 AM


problEMas 331

Para encontrar los valores de K y L que minimizan costos, primero se forma la expresión la
grangiana:

£ = PKK + PLL + λ[F (K, L) – Q0]. (A.10.2)

La condición de primer orden para un mínimo está dada por

∂£
∂ λ∂F
= PK + λ∂ = 0,

∂K
K
∂K
∂ (A.10.3)

∂£
∂ λ∂F
= PL + λ∂ = 0, (A.10.4)
∂L ∂L
∂ L ∂

y
∂£

= F ( K , L ) – Q0 = 0. (A.10.5)
∂λ 0
∂λ

Al dividir la ecuación A.10.3 entre la ecuación A.10.4 y reordenar términos se tiene

∂F / ∂K ∂F / ∂L
= , (A.10.6)
PK PL

que es el resultado de la ecuación 10.21 en el capítulo 10. (Como un ejercicio, encuentre las condi-
ciones de primer orden para un nivel máximo de producción sujeto a un límite de costo de C para
obtener el mismo resultado.)
Una alternativa a la técnica de Lagrange es solucionar la restricción de la función de producción
en la ecuación A.10.1 para K en términos de L, luego sustituir el resultado de nueva cuenta en la
expresión para el costo total. Para ilustrar este método alternativo, considere el siguiente ejemplo.

Para la función de producción Q = F(K, L) = √ K √ L con PK = 4 y PL = 2, encuentre los valores de EJEMPLO A.10.1
K y L que minimizan el costo de producir 2 unidades de producto.

El problema en este caso es minimizar 4K + 2L sujeto a F(K, L) =√ K √ L = 2. Aquí la restricción de la


función de producción es Q = 2 =√K √ L que da K = 4/L. Así es que el problema es minimizar 4(4/L)
+ 2L con respecto a L. La condición de primer orden para un mínimo está dada por
d (16 / L) + 2L 
  = 2 − 16 = 0, (A.10.7)
dL L2

que arroja L = 2 √ 2. Si se sustituye otra vez en la restricción de la función de producción se tiene


K = 4/(2 √2 ) = √ 2.

pRoblEMaS
1. Una empresa genera producción con la función

Q =√K √L,

donde K y L denotan sus insumos de capital y de trabajo, respectivamente. Si el precio del trabajo es
1 y el precio del capital es 4, ¿qué cantidades de capital y de trabajo debe utilizar si su meta es fabricar
2 unidades de producto?

10_CHAPTER 10.indd 331 3/7/09 9:23:31 AM


332 Capítulo 10 apéNDICE 10: ExtENsIoNEs MatEMátICas DE la tEoría DE Costos

2. Dibuje las curvas CTL, CPL y CML para la función de producción que se dio en el problema 2.
¿Tiene rendimientos constantes, crecientes o decrecientes de escala?
3. Suponga que una empresa tiene la siguiente función de producción:

Q(K, L) = 2L √ K.

a) Si el precio del trabajo es 2 y el precio del capital es 4, ¿cuál es el cociente óptimo entre capital y
trabajo?
b) Para un nivel de producción de Q = 1 000, ¿cuánto se usará de cada insumo?
4. Una firma con la función de producción

Q(K, L) = 2L √ KL
utiliza actualmente 8 unidades de trabajo y 2 de capital. Si ésta es la mezcla de insumos óptima y si
los costos totales son iguales a 16, ¿cuáles son los precios del capital y del trabajo?
5. Para una firma con la función de producción

Q(K, L) = 3 ln K + 2 ln L,

encuentre el cociente óptimo entre capital y trabajo si el precio del capital es 4 y el del trabajo es 6.

RESpUEStaS al EjERcicio dEl apéNdicE


A.10.1. La curva CPL (parte inferior) es la envolvente exterior de las dos curvas CTP (figura superior).

Dólares/unidad de producto
CTP1

CTP2

Dólares/unidad de producto

CPL

10_CHAPTER 10.indd 332 3/7/09 9:23:32 AM


Capítulo

11
COMPETENCIA PERFECTA

I
magínese como miembro de la legislatura estatal de Colorado. Le han pedido que vote
por un proyecto de ley que tiene el propósito de aliviar la pobreza de los agricultores en
las zonas rurales. Ellos son arrendatarios de terratenientes y pueden quedarse con las ga-
nancias por la venta del producto de sus cosechas. Como escasean las lluvias, las cosechas suelen ser
magras y el trabajador promedio recibe un ingreso muy bajo. En el proyecto de ley se autorizaría
financiamiento público para construir un sistema de riego que duplicaría la cosecha de los campos
de la región.
Usted respalda con firmeza el objetivo del proyecto de ley y está a punto de votar a favor cuando
se encuentra con su asesora legislativa, una interna con una licenciatura en economía que le pide
insistentemente que no vote por la ley. Ella acepta que el proyecto va a duplicar la cosecha de granos
y también simpatiza con la meta de mejorar las condiciones de los agricultores; pero insiste en que
la ley no tendrá grandes efectos a largo plazo en las ganancias de los agricultores. Su asesora ya le ha
dado consejos sensatos sobre materias parecidas, así que decide oírla.

VISIÓN PRELIMINAR DEL CAPÍTULO


En este capítulo vamos a desarrollar las herramientas analíticas necesarias para que el hipotético
legislador estatal pondere el consejo de su asistente; se incluye un modelo de determinación de pre-
cios y productos en mercados perfectamente competitivos. El primer paso será determinar que el
objetivo competitivo de la empresa es obtener las mayores utilidades* posibles. Es evidente que no es
la única meta que persigue una empresa, pero se verán los motivos por los cuales algunas empresas
operan como si las utilidades fueran todo lo que les importara.
A continuación se verán las cuatro condiciones que definen un mercado perfectamente compe-
titivo: 1) la existencia de un producto estandarizado, 2) el comportamiento como tomador de precios

* En algunos países de habla hispana se utiliza el término beneficios.

333

11_CHAPTER 11.indd 333 3/7/09 9:38:08 AM


334 Capítulo 11 CompetenCia perfeCta

por parte de las empresas, 3) la movilidad perfecta a largo plazo de los factores de la producción y
4) la información perfecta por parte de consumidores y empresas. Resulta que, en la práctica, no es
probable que ninguna de estas condiciones quede satisfecha en ninguna empresa. Sin embargo, se
verá que el modelo económico de competencia perfecta genera ideas útiles aunque sus condiciones
estructurales sólo se cumplan de manera aproximada.
Luego, mediante las curvas de costo que se estudiaron en el capítulo 10, se va a derivar la con-
dición necesaria para maximizar las utilidades a corto plazo. La regla dice que la empresa genere
un nivel de producción en el que su costo marginal de corto plazo sea igual al precio del producto.
Se verá que, por fortuna, implantar esta regla no exige que las empresas comprendan con detalle el
concepto de costo marginal de los economistas.
A partir de la decisión sobre la oferta de una empresa en particular, se obtiene la oferta en toda la
industria. La técnica para generar el programa de la oferta de la industria es bastante parecida a la de
la suma de curvas de demanda individuales para generar una curva de demanda de todo un mercado:
simplemente se suman en sentido horizontal las curvas de oferta de las empresas individuales.
Las curvas de oferta y demanda a corto plazo de la industria interactúan para determinar el
precio de mercado a corto plazo, que es la base sobre la cual las empresas toman sus decisiones
de producción. Se verá que la rentabilidad de una empresa a corto plazo es una señal que rige el
movimiento de recursos hacia adentro y hacia afuera de la industria; específicamente, verá que los
beneficios incitan la entrada de recursos, mientras que las pérdidas estimulan su salida.
Verá que a la larga, si no cambian los gustos ni la tecnología, una industria competitiva con
empresas que tengan curvas de costo promedio a largo plazo (CPL) en forma de U alcanzará un
precio de equilibrio igual al valor mínimo de esas curvas. Y verá también que en ciertas condiciones
no será posible que en dicho mercado alguien realice transacciones adicionales que beneficien a unos
sin perjudicar al mismo tiempo a otros.

LA META DE LA MAXIMIZACIÓN
DE LAS UTILIDADES
Al estudiar no sólo la competencia perfecta, sino también otras estructuras de mercado, es tradi-
cional que los economistas supongan que el objetivo central de una empresa es maximizar las uti-
lidades. Hay que decir dos cosas sobre este supuesto. En primer lugar, aclarar qué se entiende por
“utilidades”; la segunda es explicar por qué a veces es lógico suponer que las empresas tratan de
maximizarlas.
Las utilidades o, con más precisión, las utilidades económicas, se definen como la diferencia entre
los ingresos totales y los costos totales, los cuales comprenden cualquier costo (explícito o implícito)
en que se incurra con los recursos que consume la empresa. Esta definición es muy diferente de la
que usan los contadores y muchos que no son economistas, que no restan los costos de oportunidad
ni los costos implícitos de los ingresos totales. Las utilidades contables son los ingresos totales menos
todos los costos explícitos en que se haya incurrido.
Para ilustrar la distinción, suponga que una empresa genera 100 unidades de producto por se-
mana usando 10 unidades de capital y 10 de mano de obra. Suponga también que el precio semanal
de cada factor es de 10 dólares/unidad y que la empresa es dueña de sus 10 unidades de capital. Si el
producto se vende en 2.50 dólares por unidad, los ingresos totales de la compañía serán de 250 dólares
por semana. Para calcular las utilidades económicas de la semana, se resta a 250 los 100 dólares gas-
tados en mano de obra (un costo explícito) y los 100 dólares de costos de oportunidad del capital (un
costo implícito), lo que deja 50 dólares. (Si se supone que la empresa pudo haber rentado su capital a
otra compañía a una tasa semanal de 10 dólares/unidad, el costo de oportunidad de 100 dólares son
simplemente las ganancias perdidas por usar el capital en sus propias operaciones.) En cambio, las
utilidades contables de la compañía en esa semana son de 150 dólares, que es la diferencia entre el
ingreso total de 250 dólares menos el desembolso de 100 dólares en mano de obra.
Puede pensarse que las utilidades contables son la suma de dos componentes: 1) las utilidades
normales, que son el costo de oportunidad de los recursos que posee la empresa (en este ejemplo,

11_CHAPTER 11.indd 334 3/7/09 9:38:08 AM


la meta de la maximizaCión de las utilidades 335

100 dólares) y 2) las utilidades económicas, según se definieron antes (en el ejemplo, 50 dólares). Las
utilidades económicas son las que superan el nivel de las utilidades normales.
La importancia de la distinción entre utilidades contables y económicas se deja en claro sin du-
das (aunque con algo de fantasía) en el ejemplo siguiente. EJEMPLO 11.1

Cullen Gates dirige un minicampo de golf en Valdosta, Georgia. Alquila el campo y el equipo
a una compañía grande de recreación y aporta su propio trabajo. Sus ganancias mensuales
netas después de pagar el alquiler son de 800 dólares y piensa que trabajar en el minicampo
de golf es tan atractivo como su única alternativa, que es trabajar como empleado de una
tienda de abarrotes por un sueldo de 800 dólares/mes.
Cullen se acaba de enterar de que su tío Bill le cedió unas tierras en Nueva York (parcela
delimitada por las calles que se muestran en la figura 11.1). El terreno está sin edificar y
Cullen descubre que una constructora está dispuesta a instalar y mantener un minicampo de
golf por un pago mensual de 4 000 dólares. Cullen también encarga una encuesta de mercado
que revela que recaudaría 16 000 dólares mensuales si abriera ahí un minicampo de golf (por-
que hay más golfistas potenciales en Manhattan que en Valdosta). Después de deducir el pago
de 4 000 dólares mensuales a la constructora, le quedarían 12 000 dólares libres y limpios.
Dadas estas cifras, y suponiendo que el costo de la vida es el mismo en Nueva York y en Val-
dosta, ¿debe Cullen (que quiere maximizar sus utilidades) llevarse su negocio a Manhattan?

FIGURA 11.1
Posible ubicación de un
minicampo de golf en
Calle 59
Manhattan

Quinta Avenida
Avenida Madison

Calle 58

Como quiere maximizar sus utilidades, debe cambiarse a Manhattan sólo si sus utilidades económi-
cas allá son mayores que en Valdosta. Pero suponga que Cullen no maneja el concepto de utilidades
económicas y en cambio compara sus utilidades contables en los dos lugares. En Valdosta sus utilida-
des contables son de 800 dólares/mes, la suma que le queda después de pagar las cuentas. En Man-
hattan la cifra correspondiente será de 12 000 dólares/mes. Según esta comparación, debe apurarse a
abandonar Valdosta para irse a Nueva York.
Pero si compara las utilidades económicas, llega justo a la conclusión opuesta. En Valdosta sus
utilidades económicas son igual a cero cuando se considera el costo de oportunidad de su trabajo
(podría haber ganado 800 dólares al mes como empleado, lo cual es el monto exacto de sus utilidades
contables). Para calcular cuáles serían sus utilidades económicas en Nueva York, tiene que deducir de
sus utilidades contables de 12 000 dólares/mes no sólo los 800 dólares mensuales del costo de opor-
tunidad de su trabajo, sino también el costo de oportunidad del terreno. Pocos lugares del mundo
tienen suelos con precios tan altos como el centro de Manhattan. Suponga que se hace la estimación
conservadora de que el terreno de Cullen tiene un precio de 100 millones de dólares en el mercado
inmobiliario actual y que la tasa de interés es de un punto porcentual al mes. Entonces, el costo de
oportunidad de dedicar el terreno a un minicampo de golf será (0.01) × (100 000 000 dólares) =
1 000 000 dólares/mes, con lo cual las utilidades económicas en Manhattan son iguales a 12 000 – 800
– 1 000 000 = –988 800 dólares. De esta manera, si se asigna cualquier valor razonable al costo de
oportunidad del terreno de Cullen, es obvio que le conviene más venderlo o alquilarlo y quedarse

11_CHAPTER 11.indd 335 3/7/09 9:38:09 AM


336 Capítulo 11 CompetenCia perfeCta

en Valdosta. Los bienes raíces son tan caros en Manhattan porque la gente construye rascacielos y
cobra rentas elevadas a una multitud de inquilinos. Construir un minicampo de golf en el centro de
Manhattan sería como llevar diamantes en la suela de los zapatos.

EJERCICIO 11.1
En el ejemplo 11.1, ¿qué tan baja tendría que ser la tasa de interés para que Cullen se mude a
Manhattan?

De vuelta a la premisa de la maximización de utilidades. Para pronosticar qué hará cualquier entidad
(empresa, persona, comité o gobierno) en condiciones específicas, debe formularse algún supuesto
en cuanto a sus metas. En efecto, si se sabe a dónde quiere ir la gente, es mucho más fácil pronosticar
qué hará cuando llegue. Los economistas suponen que la meta de las empresas es maximizar las utili-
dades económicas; entonces, tratarán de descubrir qué conductas particulares fomentan esa meta.
Se han sugerido numerosas objeciones al supuesto de la maximización de utilidades. Algunos
críticos dicen que la meta de una empresa es maximizar sus oportunidades de sobrevivir; otros creen
que lo que quiere es maximizar las ventas totales o los ingresos totales, y otros más dicen que las
empresas no tratan de maximizar nada.
Una razón para el escepticismo es que abundan ejemplos en los que los administradores de las
empresas no parecen calificados para emprender las acciones requeridas para maximizar las utilida-
des. Pero es importante entender que la premisa de la maximización de las utilidades no se refuta por
la existencia de administradores incompetentes. Por el contrario, puede argumentarse que incluso
en un mundo en el que las acciones de las empresas empiezan por ser aleatorias, al final dominará
una tendencia a maximizar las utilidades.1
El argumento es análogo a la teoría de la evolución por selección natural de Charles Darwin y
es más o menos como sigue. En primer lugar, en un mundo de actos azarosos, algunas empresas, por
pura casualidad, se acercarán mucho más que otras a mostrar un funcionamiento de maximización
de las utilidades. Dichas empresas tendrán más ingresos excedentes a su disposición, lo que les per-
mitirá crecer más rápidamente que sus rivales. En el otro lado de la moneda están las empresas con
un comportamiento que se desvía bastante de la maximización de las utilidades, las cuales tienen
más probabilidades de quebrar. En el reino animal la comida es un recurso esencial para la super-
vivencia, y de igual forma las utilidades lo son para el mercado competitivo. Las empresas con las
mayores utilidades tienen más probabilidades de sobrevivir. El argumento evolutivo concluye que,
durante periodos largos, el comportamiento tiende a la maximización de las utilidades nada más
como resultado de las presiones selectivas en un entorno competitivo.
Pero las fuerzas que apoyan la maximización de las utilidades no se limitan a las presiones no
deliberadas de la selección natural. También incluyen los actos de personas que son muy conscientes
de perseguir sus propios intereses. Por ejemplo, banqueros y otros prestamistas quieren correr los
menores riesgos posibles; por eso, prefieren hacer negocios con empresas muy rentables. Además de
tener más recursos internos, a estas empresas se les facilita el acceso a recursos externos de capital
para financiar su crecimiento. Otra fuerza importante que sostiene la maximización de utilidades es
la amenaza de una compra hostil. El precio de las acciones de una empresa se basa en su rentabilidad
(verá más sobre este punto en el capítulo 15), con el resultado de que las acciones de una empresa
que no maximiza sus utilidades se venderán muy por debajo de su valor potencial. Esto abre una
oportunidad para que un tercero compre las acciones a precio de subasta y luego suba su precio
modificando el funcionamiento de la empresa.
Otra presión a favor de la maximización de las utilidades es que los propietarios de muchas
empresas pagan en forma parcial a sus administradores dándoles una parte de las utilidades. Éste es
un claro incentivo financiero para que los administradores eleven la rentabilidad siempre que surjan
las oportunidades.
Por último, hagamos notar que el supuesto de la maximización de utilidades no significa que las
empresas no realicen en todo momento sus operaciones de la manera más eficiente que quepa con-

1
Véase, por ejemplo, Armen Alchian, “Uncertainly, Evolution, and Economic Theory”, en Journal of Political Economy, 1950.

11_CHAPTER 11.indd 336 3/7/09 9:38:10 AM


las Cuatro CondiCiones de la CompetenCia perfeCta 337

cebir. En el mundo en que vivimos no hay muchos administradores inteligentes y competentes, sino
una multitud que no posee ninguno de estos atributos. No hace falta decir que no es posible asignar
todas las tareas a la persona más competente del universo. En un mundo sensato las tareas más im-
portantes serían realizadas por los mejores administradores y las menos importantes por los menos
competentes. Entonces, el mero hecho de que existan empresas que hacen cosas absurdas no signi-
fica que no se dediquen a maximizar sus utilidades; hacerlo significa hacer lo mejor que uno pueda,
dadas las circunstancias, y a veces esto significa trabajar con administradores poco inspirados.
En conjunto, estos comentarios justifican el supuesto de la maximización de utilidades. Hasta
se diría que ponen la carga de la prueba en quienes insisten en que las empresas no maximizan sus
utilidades. Pero es obvio que no establecen de manera concluyente que las empresas siempre per-
siguen las utilidades a expensas de todas las demás metas. Ésta sigue siendo una cuestión empírica,
y en los capítulos siguientes veremos pruebas de que a veces las empresas no alcanzan sus metas.
Con todo, el supuesto de la maximización de las utilidades es un buen sitio para empezar el análisis
del funcionamiento de las empresas, y no hay duda de que ofrece conocimientos útiles sobre cómo
reaccionan las empresas a los cambios de los insumos o los precios de los productos, los impuestos y
otras características importantes de su entorno operativo.

LAS CUATRO CONDICIONES


DE LA COMPETENCIA PERFECTA
Para anticipar la producción de una empresa competitiva los economistas desarrollaron una teoría de
la competencia perfecta. Cuatro condiciones definen la existencia de un mercado perfectamente com-
petitivo. Considérelas una por una.
1. Las empresas venden un producto homogéneo En un mercado perfectamente competitivo
se supone que el producto que vende una empresa es un sustituto perfecto del que vende otra. In-
terpretado en sentido literal, es una condición que casi nunca se cumple. Por ejemplo, los catadores
de vinos finos insisten en que pueden diferenciar entre vinos hechos con la misma especie de uva
cultivada en tierras separadas por pocos metros. También es difícil hablar de un mercado de artículos
tan simples como las camisas, porque hay de muchos estilos y de diferente calidad. Pero si se define
un mercado muy acotado, a veces es posible alcanzar un grado razonable de similitud entre los pro-
ductos elaborados por empresas rivales. Por ejemplo, “el trigo primavera de medio oeste” quizá no
sea idéntico en las diversas granjas, pero se asemeja tanto que a la mayoría de los compradores no les
importa de cuál de ellas venga.
2. Las empresas son tomadoras de precios Esto significa que las empresas dan por sentado el
precio del producto en el mercado. De manera más específica, deben creer que el precio de mer-
cado no se verá afectado por la producción que generan. Es probable que esta condición quede
satisfecha cuando muchas empresas atienden el mercado y cada una de ellas genera una fracción
insignificante de la producción total de la industria. Pero no siempre se necesitan muchas empresas
para que haya que tomar o aceptar el precio. Por ejemplo, aun si hay nada más dos empresas en un
mercado, pueden aceptar los precios si creen que otras empresas están en guardia para entrar en el
mercado sin previo aviso.
3. Entrada y salida libres, con factores de producción perfectamente móviles a largo plazo Una
consecuencia de esta condición es que si una empresa percibe una oportunidad comercial rentable en
un determinado tiempo y lugar, podrá contratar los factores de producción requeridos para aprove-
charla. Del mismo modo, si su ámbito comercial actual ya no parece atractivo en relación con otros,
está libre de retirar sus factores de producción y llevarlos a industrias donde haya oportunidades más
sólidas. Desde luego, nadie cree que los recursos sean perfectamente móviles. En particular, no es
probable que la mano de obra satisfaga esta condición. La gente compra casas, entabla amistades,
inscribe a sus hijos en la escuela y establece un cúmulo de compromisos que le dificultan mudarse a
otro lugar. Sin embargo, en la práctica el supuesto de la perfecta movilidad queda razonablemente
bien satisfecho, sobre todo si se tiene en cuenta que no siempre es necesario que la mano de obra se
desplace para que sea móvil en un sentido económico. De hecho, una empresa puede mudarse hacia

11_CHAPTER 11.indd 337 3/7/09 9:38:10 AM


338 Capítulo 11 CompetenCia perfeCta

el trabajador, como ocurrió cuando fábricas de calzado y textiles de Nueva Inglaterra se reubicaron
en el sur para emplear mano de obra más barata.
4. Empresas y consumidores tienen información perfecta Una empresa no tiene ningún mo-
tivo para dejar su industria actual si no se entera de la existencia de oportunidades más rentables
en otra parte. Del mismo modo, un consumidor no tiene incentivos para cambiar de un producto
caro a uno barato de la misma calidad si desconoce la existencia del último. Aquí también la condi-
ción requerida nunca se cumple en sentido literal. El mundo es tan complejo que de manera inevi-
table habrá características relevantes que se oculten a la vista. Como asunto práctico, el supuesto
de la información perfecta se suele interpretar en el sentido de que las personas pueden conseguir
sin grandes dificultades la mayor parte de la información que es más pertinente para sus opciones.
Como se vio en el capítulo 8, la gente tiene la información pertinente en la punta de los dedos
y, sin embargo, no hace buen uso de ella. Pese a estas observaciones, se verá que el estado del
conocimiento es suficiente para dar una aproximación razonable a una condición de información
perfecta.
Para evaluar si las premisas en que se basa el modelo de la competencia perfecta son irremedia-
blemente restrictivas, es útil compararlas con las premisas que subyacen en el modelo de los objetos
en movimiento de los físicos. Si en preparatoria o en la licenciatura tomó un curso de física, sabe (o
supo alguna vez) que una fuerza aplicada a un objeto en una superficie sin fricción lo acelera a una
tasa inversamente proporcional a su masa. Así, una fuerza dada aplicada a un objeto de 10 kilogra-
mos lo acelerará al doble que cuando se aplica la misma fuerza a un objeto de 20 kilogramos.
Para ilustrar esta teoría los maestros de física pasan películas de lo que sucede cuando se aplican
diversas fuerzas a un disco de hockey sobre una superficie de hielo seco. Los físicos entienden a la
perfección que hay cierta fricción que es fácil medir entre el disco y el hielo. Pero también son cons-
cientes de que la fricción es ínfima y el modelo de todos modos hace predicciones precisas.
En las situaciones que son comunes en la práctica, la fricción casi nunca es tan poca como entre
un disco y una superficie de hielo. Habrá aprendido esto con dolor si se cayó de su motocicleta en un
camino asfaltado. Pero incluso en este caso las leyes físicas del movimiento son válidas y es posible
hacer ajustes correspondientes a la fricción para estimar cuánto se deslizará un motociclista caído. Y
aunque el modelo no se calibre con precisión, dirá que el motociclista se proyectará más lejos cuanto
más rápido haya ido cuando cayó, y que también se proyectará más si el pavimento estaba húmedo o
cubierto de arena o grava, que estando limpio y seco.
En el modelo económico de la competencia perfecta los problemas son semejantes. En algunos
mercados, como el de camiones de basura o de equipos para remover tierra, por lo menos algunas
condiciones no se satisfacen ni por aproximación. Pero incluso en estos casos, el modelo competitivo
indica algo útil si se le interpreta con cuidado.

LA CONDICIÓN DE CORTO PLAZO


DE LA MAXIMIZACIÓN DE UTILIDADES
La primera pregunta que se desea que responda el modelo del funcionamiento de una empresa com-
petitiva es ¿cómo decide su nivel de producción a corto plazo? De acuerdo con el supuesto de que la
meta de la empresa es maximizar las utilidades económicas, va a escoger el nivel de producción en el
que sea mayor la diferencia entre los ingresos totales y los costos totales.
Considere una empresa con la curva de costo total de corto plazo marcada CT en la sección
superior de la figura 11.2. Como muchas empresas que estudió en el capítulo 10, ésta tiene ren-
dimientos sobre sus insumos variables que primero aumentan y luego se reducen, lo que genera
la forma conocida de curvatura de la curva de costo total. Suponga que esta empresa puede ven-
der su producción a un precio de P0 = 18 dólares/unidad. Sus ingresos totales por semana serán
de 18 dólares/unidad de producción multiplicados por las unidades de producción vendidas en
la semana. Por ejemplo, si la empresa no vende su producción, gana cero ingresos totales; si ven-
de 10 unidades por semana, gana 180 dólares semanales; si vende 20 unidades, gana 360 dólares,
etc. Entonces, para la empresa perfectamente competitiva, que puede vender cuanta producción
quiera a un precio constante de mercado, los ingresos totales son exactamente proporcionales a la

11_CHAPTER 11.indd 338 3/7/09 9:38:10 AM


la CondiCión de Corto plazo de la maximizaCión de utilidades 339

producción. Para la empresa de este ejemplo, la curva de ingresos totales es la recta marcada IT
en la sección superior de la figura 11.2. Es una recta con pendiente igual al precio del producto,
P0 = 18.

FIGURA 11.2
Ingresos totales y costos totales (dólares/semana) CT Ingresos, costos y
utilidad económica
la curva de ingresos
IT totales es la recta marcada
it en la parte superior.
su diferencia con el costo
total (Ct en la sección
superior) representa las
133.2 utilidades económicas (ΠQ
120.6 en la sección inferior). Con
Q = 0, ΠQ = –Cf = –30.
las utilidades económicas
llegan a un máximo (12.60
dólares/semana) cuando
Q = 7.4.
CF = 30
Pendiente = P0 = 18
Q
4.7 7.4 8.7
Utilidades (dls./sem)
12.6

Q
4.7 7.4 8.7

–CF = –30
ΠQ = IT – CT

En la sección inferior de la figura 11.2 se da la gráfica de la diferencia entre IT y CT, que es la


curva designada ΠQ, la notación tradicional de la economía para representar las utilidades econó-
micas. En este caso ΠQ es positiva con niveles de producción entre Q = 4.7 y Q = 8.7 y llega a un
máximo con Q = 7.4. Para niveles de producción de menos de 4.7 o mayores que 8.7 la empresa sufre
pérdidas económicas, que es otra manera de decir que sus utilidades económicas son negativas para
los valores indicados de Q.
Observe también, en la parte inferior de la figura 11.2, que la ordenada en el origen de la curva de
utilidades es igual a –30 dólares/semana, el valor negativo de los costos fijos de la empresa. Cuando
la empresa no genera producción, no gana ingresos ni incurre en costos variables, pero todavía debe
pagar los costos fijos, así que cuando Q = 0, las utilidades son –CF. Si no hay un volumen positivo de
producción con el que la empresa pueda ganar utilidades aparte de –CF, la mejor opción es no tener
ninguna producción a corto plazo. ingreso marginal cambio del
El punto de las mayores utilidades también puede caracterizarse en términos de la relación en- ingreso total que es resultado
tre el precio de la producción y el costo marginal de corto plazo. El precio de la producción, que de un cambio de una unidad en
es igual a la pendiente de la curva de ingreso total, se llama ingreso marginal (IM)2 y se define las ventas.

2
Como se verá en el capítulo siguiente, el precio de la producción y el ingreso marginal no son lo mismo para un monopolista.

11_CHAPTER 11.indd 339 3/7/09 9:38:11 AM


340 Capítulo 11 CompetenCia perfeCta

formalmente como el cambio del ingreso que ocurre cuando la venta de la producción cambia una unidad.
En la terminología del costo-beneficio del capítulo 1, IM es el beneficio para la empresa de vender
una unidad adicional de producto. Si la empresa quiere maximizar sus utilidades, tiene que ponderar
este beneficio y el costo de vender una unidad adicional de producto, que es su costo marginal.
Las curvas de costo marginal a corto plazo y el costo variable promedio que corresponde a la
curva CT de la figura 11.2 también se muestran en la figura 11.3, en la que se vuelve a suponer que la
empresa puede vender su producción a un precio de P0 = $18/unidad. Para maximizar sus utilidades
económicas, la empresa debe seguir esta regla: siempre que P0 sea mayor que el valor mínimo de
CVP (más adelante se dirá más sobre las causas de esta condición), la empresa debe generar un nivel de
producción para el cual el ingreso marginal, P0 = 18, sea igual al costo marginal de la parte ascendente de la
curva CM. En particular para las curvas que se muestran en la figura 11.3, P0 = 18 es mayor que el
valor mínimo de CVP y es igual al costo marginal en el nivel de cantidad Q* = 7.4. El requisito para
que el ingreso marginal interseque el costo marginal en su parte ascendente implica que el ingreso
marginal interseque al costo marginal desde arriba. Así, el ingreso marginal estará debajo del costo
marginal a partir de este punto de intersección y la empresa no tendrá incentivos para ampliar la
producción más allá de ese punto (porque unidades adicionales reducirían las utilidades).

FIGURA 11.3
Nivel de producción
de maximización de
utilidades a corto plazo
una condición necesaria Dólares/unidad de producción CM
para la maximización de las
utilidades es que el precio
CMQ2
sea igual al costo marginal
en la parte ascendente de
la curva de costo marginal. 18 P0 = 18 = IM
aquí, esto sucede con un CVP
volumen de producción CMQ1
Q* = 7.4.

Q
Q1 Q* = 7.4 Q2

Como se muestra en el ejercicio siguiente, las definiciones de IM y CM indican algo acerca de los
valores relativos de las pendientes de las curvas IT y CT en el punto de máxima utilidad de la figura
11.2.

EJERCICIO 11.2
Compare las pendientes de las curvas CT e IT de la figura 11.2, con Q = 7.4

¿Por qué una condición necesaria para maximizar las utilidades es que el precio sea igual al cos-
to marginal? Suponga que se escoge otro nivel de producción, por ejemplo, Q1, que es menor que
Q* = 7.4. El beneficio para la empresa de vender una unidad adicional de producto será P0 = 18

11_CHAPTER 11.indd 340 3/7/09 9:38:12 AM


la CondiCión de Corto plazo de la maximizaCión de utilidades 341

dólares (su ingreso marginal). La adición al costo total de producir una unidad extra de producto
en Q1 será su costo marginal en el nivel de producción CMQ1, que en la figura 11.3 es evidentemente
menor que 18 dólares. Se deduce que para cualquier nivel de producción en la parte ascendente de
la curva CM a la izquierda de Q* = 7.4, el beneficio de expandirse (medido por el ingreso marginal)
será mayor que el costo de esa expansión (medido por el costo marginal). Esto equivale a decir que
las utilidades aumentan si se expande la producción a partir de Q1.
Ahora considere cualquier nivel de producción a la derecha de Q* = 7.4, como Q2. En Q2, el
beneficio de reducir la producción una unidad será el ahorro de costos, que es el costo marginal en
ese nivel de producción, a saber, CMQ2. (Observe que se usa el término beneficio para referirse a que
se evita un costo.) El costo para la empresa de contraer la producción una unidad será su ingreso
marginal, P0 = 18, la pérdida del ingreso total cuando vende una unidad menos. (En este caso, no
obtener un beneficio es un costo.) Puesto que CMQ2 > 18 dólares, la empresa ahorrará más de lo que
pierde cuando contrae la producción una unidad. Se deduce que para cualquier nivel de producción
mayor que Q* = 7.4, las utilidades de la empresa crecerán si contrae la producción. El único nivel
de producción en el que la empresa no puede obtener utilidades mayores ni con expansión ni con
contracción es Q* = 7.4, el nivel para el cual el costo de cualquier movimiento es exactamente igual
a su beneficio.3

LA CONDICIÓN DE CIERRE
Recuerde que la regla para la maximización de las utilidades a corto plazo es fijar un precio que sea
igual al costo marginal, siempre que el precio exceda el valor mínimo del costo variable promedio.
¿Por qué el precio tiene que ser mayor que el punto mínimo de la curva CVP? La respuesta es que
si no se cumple esta condición, a la empresa le conviene cerrar a corto plazo, es decir, no producir
nada. Para entender por qué, observe que el ingreso promedio (IP) por unidad de producto vendido
es el precio al que se vende el producto. (Cuando el precio es constante en todos los niveles de pro-
ducción, el ingreso promedio y el ingreso marginal son iguales.)4 Si el ingreso promedio es menor
que el costo variable promedio, la empresa asimila una pérdida por cada unidad de producto que
vende. El ingreso total de la empresa (el ingreso promedio multiplicado por la cantidad) será menor condición de cierre si el
que el costo variable total (CVP por la cantidad) y esto significa que lo mejor sería que no produjera precio cae por debajo del
nada. En este contexto, cerrar quiere decir generar una producción nula a corto plazo. La empresa mínimo del costo variable
reanudará la producción si el precio vuelve a subir por arriba del valor mínimo del CVP, el punto que promedio, la empresa debe
inicia la condición de cierre. cerrar a corto plazo.

3
El problema de la empresa es maximizar Π = PQ – CTQ, donde CTQ es el costo total a corto plazo de producir Q unidades. La
condición de primer orden para un máximo está dada por

d∏ dCTQ
= P− = P − CM Q = 0,
dQ dQ

que da la condición P = MCQ. La condición de segundo orden de un máximo está dada por

d 2 ∏ −dCM Q
= <0
dQ 2 dQ

o bien

dCM Q
> 0,
dQ

lo cual indica por qué se debe estar en un punto en la parte ascendente de la curva de costo marginal.
4
Observe que IP = IT/Q = PQ/Q = P.

11_CHAPTER 11.indd 341 3/7/09 9:38:14 AM


342 Capítulo 11 CompetenCia perfeCta

Como se vio en la figura 11.2, una empresa con producción nula obtendrá beneficios económi-
cos iguales al negativo de sus costos fijos. Si el precio de su producto es menor que el valor mínimo
de sus costos variables promedio, tendrá pérdidas todavía mayores si genera un volumen positivo de
producción.
Las dos reglas —1) que el precio debe ser igual al costo marginal en una parte ascendente de la
curva de costo marginal y 2) que el precio debe exceder el valor mínimo de la curva de costo variable
promedio— definen la curva de oferta de corto plazo de la empresa perfectamente competitiva. La
curva de oferta de la empresa señala cuánta producción quiere generar la empresa a diversos precios.
Como se muestra en la figura 11.4, es la parte ascendente de la curva de costo marginal de corto
plazo a partir del valor mínimo de la curva de costo variable promedio (que en este ejemplo es de 12
dólares por unidad de producto). Debajo de P = 12, la curva de oferta coincide con el eje vertical, lo
que indica que la empresa genera una producción nula cuando el precio es menor que el CVP míni-
mo. Para precios mayores que 12, la empresa ofrecerá un volumen de producción en el que P = CM.
Así, los precios de 14 y 20 harán que la empresa suministre 6.4 y 7.8 unidades de producto, respecti-
vamente. La empresa competitiva actúa como tomadora de precios y maximizadora de utilidades:
al dar el precio del mercado por sentado, escoge el nivel de producción que maximiza los beneficios
económicos a ese precio.

FIGURA 11.4
Curva de oferta a
corto plazo de una
empresa perfectamente
competitiva Dólares/unidad de producción CM
Cuando el precio está por
debajo del valor mínimo del
costo variable promedio 20 CTP
(en este caso, 12 dólares
por unidad de producto),
14 CVP
la empresa tendrá pérdidas
CVP mín. = 12
en todos los niveles de
producción y las llevará
al mínimo si no produce
nada. para los precios que
están sobre el CVp mínimo,
la empresa genera ese Q
0 5.6 6.4 7.8
volumen de producción
para el cual P = Cm en
la parte ascendente de la
curva Cm.

Observe en la figura 11.4 que la empresa obtiene una producción positiva cuando el precio supe-
ra el CVP mínimo y recuerde que el costo variable promedio es menor que el costo total promedio,
siendo la diferencia el costo fijo promedio. Se deduce que sin importar qué tan pequeño sea el CFP,
hay un intervalo de precios que está entre las curvas CVP y CTP. Para cualquier precio en dicho
intervalo, la empresa genera el nivel de producción en el cual P = CM, lo que significa que perderá
dinero porque P es menor que CTP. Por ejemplo, la empresa cuyas curvas de costo se muestran en
la figura 11.4 no puede cubrir todos sus costos a un precio de 14 dólares. Aun así, su mejor opción es
abastecer 6.4 unidades de producción por semana, porque perdería más si tuviera que cerrar. Poder
pagar costos variables no le asegura a la empresa que vaya a tener utilidades económicas positivas.
Pero basta inducir a la empresa a generar producción a corto plazo.
Observe también en la figura 11.4 que la curva de oferta a corto plazo de la empresa tiene pen-
diente positiva. Esto se debe a que la parte relevante de la curva de costo marginal a corto plazo de
la empresa tiene una pendiente positiva, lo cual a su vez es una consecuencia directa de la ley de
rendimientos decrecientes.

11_CHAPTER 11.indd 342 3/7/09 9:38:15 AM


oferta de la industria CompetitiVa a Corto plazo 343

OFERTA DE LA INDUSTRIA COMPETITIVA


A CORTO PLAZO
La curva de oferta a corto plazo para una industria competitiva se obtiene de manera análoga a
como se hizo para obtener la curva de demanda del mercado generada en el capítulo 5. En este caso,
se anuncia un precio y luego se suman las cantidades que cada empresa desea suministrar a ese pre-
cio. La suma es la oferta de la industria a ese precio. Los puntos adicionales de la curva de oferta de la
industria se generan relacionando otros precios con las sumas de las ofertas de empresas individuales
a esos precios.
La figura 11.5 ilustra el procedimiento para uno de los casos más simples, una industria com-
puesta por dos empresas. A un precio de 2 dólares/unidad de producto, sólo la empresa 1 (sección
de la izquierda) quiere ofrecer alguna producción, así que su oferta, Q1 = 2 unidades de produc-
ción por semana, constituye toda la oferta de la industria en P = 2 (sección de la derecha). En
P = 3, la empresa 2 entra en el mercado (sección central) con una oferta de Q2 = 4. Sumada a la
oferta de la empresa 1 en P = 3 (a saber, Q1 = 3), la oferta de la industria que resulta en P = 3 es
Q = 7 (sección de la derecha). De la misma manera, se ve que la oferta de la industria en P = 7
es Q = 7 + 8 = 15. En el capítulo 5 se vio que la curva de demanda del mercado es la suma horizontal
de las curvas individuales de la demanda de consumo. Aquí se ve que la curva de oferta del mercado
es la suma horizontal de las curvas de oferta de las empresas por separado.

FIGURA 11.5
Empresa 1 + Empresa 2 = Industria total Curva de oferta a corto
plazo de una industria
Dólares/unidad Dólares/unidad
competitiva
de producto CM1 CM2 de producto S = ΣCM para obtener la curva
de oferta de la industria
7 7 (sección de la derecha), se
suman horizontalmente
las curvas de oferta de
las empresas individuales
(secciones de la izquierda y
el centro).
3 3
2 2

Q1 Q2 Q
0 23 7 0 4 8 0 2 3 7 15
Q = Q1 + Q2

La suma horizontal de las ofertas individuales de las empresas de una industria tiene una forma
simple si las empresas de dicha industria son idénticas. Suponga que n empresas tienen una curva
de oferta P = c + dQi. Para sumar todas las cantidades de las n empresas que conforman la oferta de
la industria, se reordena la curva de oferta de la empresa P = c + dQi para despejar la cantidad en el
lado izquierdo: Qi = –(c/d) + (1/d)P. Entonces, la oferta de la industria es la suma de las cantidades
abastecidas, Qi, por cada una de las n empresas,

 c 1  nc n
Q = nQi = n− + P  = − + P .
 d d  d d

Es posible volver a ordenar la oferta de la industria Q = –(nc/d) + (n/d)P para obtenerla otra vez con
el precio despejado en el lado izquierdo: P = c + (d/n)Q. Se intuye que cada unidad abastecida por
la industria es 1/n unidad que suministra cada empresa. Estos cálculos apuntan a una regla general

11_CHAPTER 11.indd 343 3/7/09 9:38:17 AM


344 Capítulo 11 CompetenCia perfeCta

para trazar la curva de oferta de la industria cuando las empresas son idénticas. Si se tienen n curvas
de oferta individuales P = c + dQi, la curva de oferta de la industria es P = c + (d/n)Q.

EJEMPLO 11.2 Suponga que una industria está compuesta por 200 empresas, cada una con una curva de
oferta de P = 100 + 1 000Q. ¿Cuál es la curva de oferta de la industria?
En primer lugar, es necesario reordenar la curva de oferta de la empresa representativa P = 100 +
1 000Q, para despejar la cantidad:
1 1
Qi = − + P.
10 1000

Luego se multiplica por el número de empresas, n = 200:


 1 1  1
Q = nQi = 200Qi = 200− + P  = −20 + P.
 10 1000  5
Por último, se reordena la curva de oferta de la industria Q = –20 + (1–5 )P para despejar el precio en un
solo lado P = 100 + 5Q para volver a la forma pendiente-ordenada al origen.

EJERCICIO 11.3
Suponga que una industria tiene 30 empresas, cada una con una curva de oferta de P = 20 +
90Qi. ¿Cuál es la curva de oferta de la industria?

EQUILIBRIO COMPETITIVO A CORTO PLAZO


La empresa competitiva tiene que escoger el volumen de producción más redituable que va a gene-
rar como reacción a un precio dado. Pero, ¿cómo se determina el precio? Como se vio en el capítulo
2, se obtiene de la intersección de las curvas de oferta y demanda del producto. Recuerde que en el
precio de equilibrio los vendedores ofrecen la cantidad que quieran y los compradores adquieren la
que desean.
En la sección de la izquierda de la figura 11.6, la curva marcada con D es la de demanda del mer-
cado de un producto que se vende en una industria perfectamente competitiva. La curva marcada

FIGURA 11.6
Precio de corto plazo
y determinación de Mercado Empresa individual
la producción en
competencia pura Precio (dólares/unidad Precio (dólares/unidad
las curvas de oferta y de producto) de producto)
demanda a corto plazo se
intersecan para determinar el S = ΣCM
CTPi Πi = 640 dls/sem
precio de equilibrio a corto CMi
plazo, P* = 20 (sección de P* = 20 20 P = IMi = 20
la izquierda). la curva de
demanda de la empresa es
una línea horizontal en P* = 12
20 (sección de la derecha).
si se da por sentado P* = 20 CVPi
(sección de la derecha), la
empresa maximiza su
beneficio económico si D
produce Q*1 = 80 unidades/ Q Qi
0 Q* 0 80
semana, con lo que genera Unidades de producto/sem
utilidades económicas de Π i
= 640 dólares/semana (el
rectángulo sombreado en
la sección de la derecha).

11_CHAPTER 11.indd 344 3/7/09 9:38:19 AM


equilibrio CompetitiVo a Corto plazo 345

TABLA 11.1
Utilidades económicas contra pérdidas económicas

Q CTP CM Π(P = 20) Π(P=10) a un precio de


20, la empresa
40 14 6 240 –160 gana utilidades
60 12 10 480 –120 económicas, pero
a un precio de 10,
80 12 20 640 –160 sufre pérdidas.
100 15 31 500 –500

con S es la curva correspondiente de la oferta de la industria a corto plazo, la suma horizontal de la


parte relevante de las curvas individuales de costo marginal a corto plazo.5 Estas dos curvas se inter-
secan para establecer el precio de equilibrio competitivo de corto plazo, que aquí se representa como
P* = 20 dólares/unidad de producto. P* = 20 es el precio en el que las empresas basan sus decisiones
de producción.
Las condiciones que rigen a la empresa típica se muestran en la sección derecha de la figura 11.6.
La curva de demanda a la que se enfrenta la empresa es una línea horizontal en P* = 20. Esto significa
que puede vender tanto o tan poco como quiera al precio de mercado de 20 dólares por unidad. Di-
cho de otra manera, cualquier empresa puede vender cuanto quiera sin afectar al precio del mercado.
Si una empresa cobrara más de 20 dólares, no vendería ningún producto, porque los compradores
se cambiarían a la competencia, que vende a 20 dólares. Desde luego, una empresa puede cobrar
menos de 20 dólares, pero no tendría ningún motivo para hacerlo si su objetivo fuera maximizar los
beneficios económicos, puesto que ya puede vender todo lo que quiera a 20 dólares. El resultado es
que aunque la curva de demanda del mercado tiene una pendiente negativa, la curva de demanda
de la empresa individual es perfectamente elástica (recuerde la definición de la elasticidad-precio del
capítulo 5, según la cual una curva de demanda horizontal tiene una elasticidad precio infinita, lo que
significa que es “perfectamente elástica”).
En la sección de la derecha de la figura 11.6, la empresa representativa maximiza sus utilidades
igualando P* = 20 dólares/unidad con el costo marginal en un volumen de producción Qi* = 80
unidades/semana. En ese nivel de producción su ingreso total es P*Q*i = 1 600 dólares/semana y sus
costos totales son CTPQ*i = (12 dólares/unidad)(80 unidades/semana) = 960 dólares/semana. Su
utilidad económica es la diferencia entre el ingreso total y el costo total, 1 600 semana/semana – 960
dólares/semana = 640 dólates/semana y está representada por el rectángulo sombreado denotado
Πi. Dicho en forma equivalente, las utilidades pueden calcularse como la diferencia entre el precio
(20 dólares/unidad) y el costo total promedio (12 dólares/unidad) multiplicado por la cantidad ven-
dida (80 unidades/semana).
Recuerde que el costo de oportunidad de los recursos que posee la empresa constituye parte
del costo incluido en su curva de costo total promedio. Por eso se dice que los ingresos totales que
superan los costos totales representan utilidades económicas. Si los ingresos de la empresa fueran
idénticos a sus costos totales, obtendría sólo utilidades normales, que equivale a decir utilidades eco-
nómicas nulas.
Tener un precio igual al costo total promedio significa que el costo total es igual al ingreso total
y que la empresa obtiene utilidades económicas nulas. Así, el precio igual al mínimo del costo total
promedio puede llamarse punto de equilibrio: el precio más bajo al cual la empresa no sufre utilida-
des negativas a corto plazo.
En la situación plasmada en la figura 11.6 y la tabla 11.1 el precio de equilibrio de corto plazo
permite a la empresa tener una utilidad económica positiva. Otra posibilidad es que las curvas de
oferta y demanda de corto plazo se intersequen en un precio de equilibrio que sea lo bastante alto
para inducir a las empresas a ofrecer producción, pero no tanto que les permita pagar todos sus cos-
tos. Esta situación se muestra en la figura 11.7 y la tabla 11.1. En la sección de la izquierda, la oferta y
5
Aquí, las “partes relevantes” son las que están arriba de los valores respectivos de CVP mín.

11_CHAPTER 11.indd 345 3/7/09 9:38:20 AM


346 Capítulo 11 CompetenCia perfeCta

FIGURA 11.7
Precio de equilibrio a Mercado Empresa individual
corto plazo que resulta en
pérdidas económicas
Precio (dólares/unidad Precio (dólares/unidad
a veces, las curvas de oferta de producto) de producto)
y demanda de corto plazo se
intersecan para alcanzar un S = ΣCM Πi = –120 dls./sem
precio de equilibrio P* = 10
dólares/unidad de producto
(sección de la izquierda) que CMi
está debajo del valor mínimo CTPi
de la curva Ctp de la empresa 12
característica (sección de la P = IMi = 10
P* = 10 10
derecha), pero sobre el punto CVPi
mínimo de su curva CVp. en
el nivel de producción que
maximiza la producción, Q*i D
= 60 unidades/semana, la Q Qi
0 Q* 0 60
empresa sufre una pérdida
Unidades de producción/semana
económica de Πi = –120
dólares/semana.

la demanda se intersecan en un precio P* = 10 dólares/unidad de producto, que está por encima del
valor mínimo de la curva CVP de la empresa que se muestra en la sección de la derecha, pero está
debajo de la curva CTP de la empresa en el nivel producción que maximiza las utilidades, Q*i = 60
unidades de producto por semana. El resultado es que la empresa tiene una pérdida económica de
P*Q*i – CTPQ*i Q*i = –120 dólares/semana. Esta pérdida se muestra en la sección derecha de la figura
11.7 en el rectángulo sombreado marcado Πi. Observe que esta pérdida es menor que –CFT, el valor
de las utilidades económicas cuando la producción es cero. Así, es lógico producir incluso si las utili-
dades económicas bajan por debajo de cero a corto plazo.

EJERCICIO 11.4
Si las curvas de costo variable promedio y de costo marginal a corto plazo de una empresa
competitiva están dadas por CVP = Q y CM = 2Q, ¿cuántas unidades de producción generará
la empresa a un precio de mercado de P = 12? ¿En qué nivel de costos fijos percibirá utilidades
económicas de cero?

EFICIENCIA DEL EQUILIBRIO


COMPETITIVO A CORTO PLAZO
Una de las características más atractivas de los mercados competitivos es el hecho de que dan lugar
eficiencia distributiva a una eficiencia distributiva, lo que significa que explotan por completo las posibilidades de ganan-
condición en la que se cias mutuas por medio de intercambios. Para ejemplificarlo, piense en el equilibrio a corto plazo
materializan todas las ganancias representado en la sección de la izquierda de la figura 11.8 y suponga que las curvas de costos que
posibles de un intercambio. aparecen en la sección de la derecha son las mismas para las 1 000 empresas de la industria.
En un mercado competitivo a corto plazo, los consumidores dan dinero a las empresas y éstas
lo usan para comprar diversos insumos para elaborar los productos que ofrecen a los consumido-
res. Decir que el equilibrio competitivo no deja espacio para mayores intercambios mutuamente
benéficos es igual que decir que no hay posibilidad de que un productor y un consumidor se pongan
de acuerdo en una transacción privada a ningún precio diferente de 10 dólares. Desde luego, los
consumidores pagarían de buena gana menos de 10 dólares por una unidad adicional de producto,
pero como 10 dólares es igual al valor de los recursos necesarios para producir otra unidad (CM en
la sección derecha de la figura 11.8), ninguna empresa estaría dispuesta a aceptarlo. Por su parte, las
empresas querrían producir una unidad extra si el precio fuera mayor de 10 dólares. Pero si ya están

11_CHAPTER 11.indd 346 3/7/09 9:38:21 AM


exCedente del produCtor 347

FIGURA 11.8
El equilibrio
competitivo es eficiente
Mercado Empresa individual a corto plazo
P S = ΣMC P Con el precio y la cantidad
de equilibrio, el valor de
los recursos adicionales
necesarios para producir la
MCi última unidad de producto
$10 $10 P = MRi = 10 de cada empresa (Cm en la
sección de la derecha) es
CVPi
equivalente al valor que los
consumidores otorgan a la
última unidad de producto
D consumida (el precio de
demanda en la sección
Q Qi de la izquierda). esto
100 000 100
significa que no hay otro
intercambio que beneficie
más a las dos partes.

en el mercado 100 000 unidades del producto, no quedan consumidores dispuestos a pagar más de 10
dólares (parte izquierda de la figura 11.8). Con el precio y la cantidad en equilibrio competitivo a cor-
to plazo, el valor de los recursos dedicados a producir la última unidad (medido por el costo marginal
de corto plazo) es exactamente igual al valor que otorgan los consumidores a esa unidad de producto
(medido por el precio que están dispuestos a pagar). Las empresas quisieran que los precios fueran
más altos y los consumidores se quejarían de que ya son muy elevados. Pero ambas partes carecen de
incentivos para comerciar a ningún precio distinto del de equilibrio.

EXCEDENTE DEL PRODUCTOR


Afirmar que un mercado competitivo es eficiente es decir que maximiza los beneficios netos para sus
participantes. En el análisis de políticas muchas veces conviene estimar el monto real que las perso-
nas y las empresas ganan por su participación en mercados específicos. Por ejemplo, suponga que un
gobierno del Tercer mundo sabe que puede abrir nuevos mercados para los mariscos si tiende una
carretera de la costa a una región interior. Si su meta es aprovechar los recursos del país con la mayor
eficiencia, su decisión sobre abrir el camino dependerá de si los beneficios que cosechen personas y
empresas en estos mercados nuevos van a superar el costo de la carretera.
En el capítulo 4 se expuso el concepto de excedente del consumidor como medida de los be-
neficios que obtiene al participar en los intercambios del mercado. Hay una medida análoga para
los productores. Los economistas la llaman excedente del productor y mide cuánto mejoran las excedente del productor
empresas por ofrecer un nivel de producción que maximice las utilidades. Alguien se sentiría tentado suma de dinero que gana una
a decir que el excedente del productor no es más que la utilidad económica, pero de ordinario el empresa por generar un nivel
excedente y las utilidades no son lo mismo. Para entenderlo, recuerde primero que, a corto plazo, de producción que maximice
las utilidades.
si la empresa no produce nada, sufre una pérdida igual a su costo fijo. En cambio, si el precio supera
el valor mínimo del CVP, le conviene más ofrecer un nivel positivo de producción. ¿De cuánto? La
ganancia de la empresa comparada con la alternativa de no producir nada es la diferencia entre los
ingresos totales y el costo variable total en el nivel de producción donde P = CM. Ahora recuerde
que la utilidad económica es la diferencia entre el ingreso total y el costo total y que este último
difiere del costo variable por el costo fijo; se concluye que el excedente del productor es la suma
de la utilidad económica y el costo fijo.6 Gráficamente, es el área del rectángulo sombreado que se

6
Si Π = IT – CT y CT = CV + CF, entonces el excedente del productor = IT – CV = IT – CT + CF = Π + CF.

11_CHAPTER 11.indd 347 3/7/09 9:38:23 AM


348 Capítulo 11 CompetenCia perfeCta

FIGURA 11.9
Dos medidas
equivalentes del
excedente del
productor
P P
la diferencia entre el
ingreso total y el costo CMi
variable total es una CMi
medida del excedente P* P*
del productor, la ganancia Excedente Excedente
para éste de producir Q*i del productor CVPi del productor
unidades en lugar de cero.
puede medirse como la
diferencia entre P*Q*i y
CVpQ*i Q*i (el rectángulo
sombreado en la sección Q Q
de la izquierda) o como la Q*i Q*i
diferencia entre P*Q*i y el a) b)
área bajo la curva de costo
marginal (área superior
sombreada, sección de la
derecha).

muestra en la sección izquierda de la figura 11.9. A corto plazo, el excedente del productor es mayor
que las utilidades económicas porque la empresa perdería más que sus utilidades económicas si se le
impidiera participar en el mercado. A largo plazo, todos los costos son variables. Por ende, a la larga,
el excedente del productor es igual que la utilidad económica.
En la sección de la derecha de la figura 11.9 se muestra una representación equivalente del ex-
cedente del productor. Esta otra medida se funda en el hecho de que el costo variable en cualquier
nivel de producción es igual al área bajo la curva de costo marginal (debajo del área sombreada en
la sección de la derecha). Para entenderlo, observe que el costo variable de generar una unidad de
producto es igual al costo marginal en una unidad, CM1; el CV para dos unidades es la suma de CM1
y CM2, etc., de modo que CVQ = CM1 + CM2 + . . . + CMQ, que es exactamente el área bajo la curva
de CM. Por lo tanto, la diferencia entre el ingreso total y el costo variable total también puede expre-
sarse como el área sombreada superior de la sección derecha de la figura 11.9.
Cuál de las dos mediciones del excedente del productor es más útil depende del contexto espe-
cífico. Si lo que interesa es el cambio en un excedente del productor, es más fácil trabajar con el mé-
todo que se muestra en la sección de la derecha de la figura 11.9. Pero si se quiere medir el excedente
total del productor, normalmente es más sencillo calcular el excedente con el método que se indica
en la parte izquierda.
Para medir el excedente acumulado en un mercado, basta sumar el excedente del productor de
todas las empresas participantes. En los casos en que la curva de costo marginal de cada empresa tie-
ne una pendiente positiva para la mayor parte de su intervalo, el excedente del productor acumulado
se aproxima al área entre la curva de oferta y la recta del precio de equilibrio, P*, como se muestra en
la figura 11.10.
Recuerde del capítulo 4 que una aproximación general del excedente del consumidor para el
mercado en su conjunto está dada por el área entre la curva de demanda y la recta del precio de equi-
librio, como se indica en el triángulo sombreado superior de la figura 11.11.7 Los beneficios totales
derivados del intercambio en el mercado pueden medirse como la suma del excedente del consumi-
dor y del productor.

7
Recuerde que esta medida del excedente del consumidor es más exacta cuando los efectos del ingreso son menores.

11_CHAPTER 11.indd 348 3/7/09 9:38:24 AM


exCedente del produCtor 349

FIGURA 11.10
Excedente acumulado
del productor cuando
las curvas de costo
marginal individuales
tienen pendiente
P S = ΣCM positiva
para cualquier cantidad,
la curva de oferta mide
P* el precio mínimo al que
las empresas estarían
Excedente del
productor
dispuestas a suministrarla.
acumulado la diferencia entre el
precio de mercado y el
D precio de oferta es la
contribución marginal al
excedente del productor
Q acumulado en ese nivel de
Q* producción. si se suman
estas contribuciones
marginales hasta la cantidad
de equilibrio Q*, se obtiene
el área sombreada, que es
el excedente del productor
acumulado.

FIGURA 11.11
P Beneficio total del
intercambio en un
mercado
la suma del excedente
S = ΣCM del productor acumulado
Excedente (triángulo inferior
del consumidor sombreado) y el excedente
acumulado del consumidor (triángulo
P* superior sombreado)
Excedente mide el beneficio total del
del producto intercambio.
acumulado

Q
Q*

Suponga que hay dos tipos de usuarios de fuegos artificiales: descuidados y cuidadosos. Los EJEMPLO 11.3
segundos nunca salen lastimados, pero los primeros a veces se lesionan a sí mismos y también
a testigos inocentes. Las curvas de costo marginal a corto plazo de las 1 000 empresas de la
industria de los fuegos artificiales están dadas por CM = 10 + Q, donde Q se mide en kilogra-
mos de petardos por año y el CM se mide en dólares por kilogramo de petardos. La curva de
demanda de fuegos artificiales de los usuarios cuidadosos está dado por P = 50 – 0.001Q (las
mismas unidades que para el CM). Los legisladores quisieran seguir autorizando a los usua-
rios cuidadosos el disfrutar de la pirotecnia, pero como no es práctico distinguir entre los dos
tipos de consumidores, decidieron prohibir totalmente los fuegos. ¿Les iría mejor a los consu-
midores y productores si los legisladores tuvieran un medio de permitirlos parcialmente?
Si todo el mercado de fuegos artificiales se prohíbe, el excedente total de consumidores y productores
es nulo. Entonces, para medir los beneficios de una veda parcial, es necesario encontrar la suma de

11_CHAPTER 11.indd 349 3/7/09 9:38:25 AM


350 Capítulo 11 CompetenCia perfeCta

los excedentes de consumidores y productores en un mercado de fuegos artificiales restringido a los


usuarios cuidadosos. Para generar la curva de oferta de este mercado se suman en forma horizontal
las curvas de costo marginal de las empresas, lo que da por resultado la curva marcada S en la figura
11.12. La curva de demanda de los usuarios cuidadosos intersecaría S en un precio de equilibrio de 30
dólares y una cantidad de equilibrio de 20 000 libras/año.

FIGURA 11.12
Excedente del productor P ($/Q)
y el consumidor en un 50
mercado compuesto por
usuarios cuidadosos de
fuegos artificiales S = ΣCM = 10 + 0.001Q
el triángulo superior
sombreado es el excedente
del consumidor (200 000
dólares anuales). el triángulo 30
sombreado inferior es el
excedente del productor
(200 000 dólares anuales). el
beneficio total de mantener D (Usuarios
abierto este mercado es la cuidadosos)
suma de los dos, o sea, 10
400 000 dólares al año.
Q (lb/año)
20 000

Al proscribir por completo la venta de fuegos artificiales, los legisladores eliminan los valores de
los excedentes de productores y consumidores dados por las áreas de los dos triángulos sombreados
en la figura 11.12, que suman 400 000 dólares anuales. En el lenguaje del análisis de costo-benefi-
cio, es el costo impuesto a los productores y usuarios cuidadosos. El beneficio de la prohibición es
cualquier valor que asigne la gente a las lesiones prevenidas (el costo neto de negar a los usuarios
descuidados el derecho a continuar). Como es obvio, no es una mera cuestión de asignar un valor
monetario al dolor y sufrimiento asociado con los dedos reventados por los petardos. En el capítulo
14 se va a explicar cómo se ha intentado hacer estimaciones aproximadas en situaciones semejantes.
Pero aun en ausencia de una medida cuantitativa formal del valor de las lesiones evitadas, el público
puede preguntarse si el excedente perdido de 400 000 dólares anuales es un precio razonable. Como
casi todos los países han vedado la venta y el uso de fuegos artificiales por parte de particulares, la
respuesta debería ser un sí enfático.

EJERCICIO 11.5
¿Cuál sería la suma de los excedentes de consumidores y productores en el ejemplo 11.3 si la
curva de demanda de los usuarios cuidadosos estuviera dada por P = 30 – 0.001Q?

AJUSTES A LARGO PLAZO


El objetivo de la empresa, a corto y largo plazos, es obtener las mayores utilidades que pueda. En
la sección anterior se vio que a veces una empresa considera ventajoso seguir produciendo a corto
plazo aunque tenga pérdidas económicas. Pero, a la larga, una empresa preferiría salir del negocio si
no puede ganar por lo menos una utilidad normal en su industria actual.
Suponga que la oferta y la demanda de la industria se intersecan en un nivel de precios P = 10,
como se muestra en la sección izquierda de la figura 11.13. Las curvas de costos para una empresa re-
presentativa se muestran en la sección derecha de la figura 11.13. En Q = 200, el precio de 10 dólares
por unidad de producto excede el CTP2, como resultado la empresa gana una utilidad económica de
600 dólares en cada periodo. El rectángulo sombreado indica estas utilidades.

11_CHAPTER 11.indd 350 3/7/09 9:38:27 AM


ajustes a largo plazo 351

La situación representada en la figura 11.13 es inherentemente inestable, porque la utilidad


económica positiva es un incentivo para que otros entren en la industria. Recuerde que las curvas
de costo total promedio ya incluyen el costo de oportunidad del capital que una empresa necesita
para hacer negocios. Esto significa que un tercero puede comprar todo lo que necesita para duplicar
las operaciones de cualquier empresa dentro de la industria y obtener utilidades económicas de 600
dólares en cada periodo.

FIGURA 11.13
Mercado Empresa individual Nivel de precios
P $/Q que genera utilidad
económica
en un nivel de precios
S = ΣCM P = 10 dólares/unidad, la
empresa ajustó el tamaño
de su planta de manera
CTP2 CMC2 que CmC2 = Cml = 10.
en el nivel de producción
10 10 que maximiza las utilidades,
Πi = 600 Q = 200, la empresa gana
7 una utilidad económica
CML igual a 600 dólares en
CPL cada periodo, indicada
D por el área del triángulo
sombreado.
Q Qi
200

A medida que más empresas entran en la industria, sus curvas de costo marginal a corto plazo se
suman a las de las empresas anteriores, lo que desplaza a la derecha la curva de oferta de la industria.
Si sólo una empresa entra en la industria, no habría ningún efecto notable en el precio. Si el precio
es prácticamente el mismo que antes, cada empresa seguiría obteniendo sus utilidades económicas
de 600 dólares por periodo, las cuales seguirán siendo la zanahoria que atraiga a otras empresas y, al
acumularse el desplazamiento de la oferta a la derecha, los precios bajarán en forma gradual.
En la sección de la izquierda de la figura 11.14 se representa el desplazamiento a la derecha de
la oferta de la industria que se genera con numerosas entradas. La curva de la oferta, S', interseca
la curva de la demanda en P = 8, y este precio inferior da a las empresas un motivo para ajustar
sus existencias de capital. Observe en la sección derecha de la figura 11.14 que el monto del capital
que da lugar a las curvas de costo a corto plazo CTP3 y CMC3 es óptimo para el nivel de precios P
= 8. Observe también que el nivel de producción que maximiza las utilidades en P = 8 es Q = 180 y
que esto da por resultado una utilidad económica de 540 por periodo, como lo indica el rectángulo
sombreado.
Observe que el ajuste de las empresas existentes al precio menor desplaza a la izquierda sus
curvas de costo marginal de corto plazo. Desde el punto de vista de su efecto sobre la curva de oferta
de la industria, este ajuste opera en dirección opuesta al ajuste causado por la entrada de nuevas
empresas. Pero el efecto neto de los dos ajustes debe ser un desplazamiento a la derecha de la oferta
de la industria. En caso contrario, el precio no habría bajado nunca y no habría motivo para que las
empresas redujeran su existencia de capital.
Aun después de realizar los ajustes indicados anteriormente, las empresas anteriores y nuevas
de la industria siguen recibiendo utilidades económicas positivas. El nuevo nivel de utilidad es menor
que antes, pero todavía es un incentivo para entradas adicionales en la industria. Las nuevas entradas
desencadenan una nueva ronda de ajustes, porque al continuar la disminución del precio, las exis-
tencias de capital se vuelven excesivas. En las industrias con empresas que tienen curvas de costo
promedio a largo plazo en forma de U, la entrada de nuevas empresas, la disminución de precios y el
ajuste de las existencias de capital persisten hasta que no se cumplen las siguientes dos condiciones:
1) el precio llega al punto mínimo de la curva CPL (P* en la sección de la derecha de la figura 11.15)

11_CHAPTER 11.indd 351 3/7/09 9:38:28 AM


352 Capítulo 11 CompetenCia perfeCta

FIGURA 11.14
Un paso en la ruta
hacia el equilibrio de
largo plazo Mercado Empresa individual
la entrada de empresas P $/Q
nuevas desplaza la oferta
a la derecha y reduce
el precio de 10 a 8. el S
S’ CMC2
precio menor hace que las
empresas ajusten a la baja CMC3
sus existencias de capital, CTP2
lo que da lugar a las nuevas CTP3
curvas de costo a corto 10 10
plazo Ctp3 y CmC3. en 8 8
tanto el precio permanezca Π’i = 540
por arriba del costo 5
promedio de corto plazo CML
(en este caso, Ctp3 = 5), CPL
D
las utilidades económicas
serán positivas (Π = 540 Q Qi
dólares por periodo) y no 180 200
desaparecen los incentivos
para que entren empresas
nuevas.

y 2) todas las empresas se han ajustado a las existencias de capital que dan lugar a una curva de
costo total promedio a corto plazo que sea tangente a la curva CPL en su punto mínimo (CTP* en
la sección derecha de la figura 11.15). Observe en la misma figura que cuando todas las empresas
llegan a este punto, su utilidad económica es de cero. La curva de costo marginal a corto plazo en
la sección derecha es similar a la curva de costo marginal a corto plazo de todas las empresas de la
industria; y cuando estas curvas se suman horizontalmente, se tiene la curva de oferta de la industria
que se muestra en la sección izquierda, la cual interseca la curva de demanda del mercado en el
precio de equilibrio a largo plazo P*. Ésta es la posición de equilibrio competitivo a largo plazo para

FIGURA 11.15
Equilibrio a largo
plazo en competencia Mercado Empresa individual
perfecta
P $/Q
si el precio empieza arriba
de P*, sigue habiendo
entradas de empresas y las
existencias de capital de S = ΣCM
las empresas establecidas
CMC*
se siguen ajustando hasta CTP*
que el movimiento a la
CPL
derecha de la curva de
oferta de la industria rebaja
el precio a P*. en P*, el P* P*
CML
nivel de producción que
maximiza las utilidades
de cada empresa es Q*i , el D
nivel de producción en el
que P* = CmC = Cml =
Q Qi
Ctp* = Cpl. las utilidades Q* Qi*
económicas de todas las
empresas son iguales a
cero.

11_CHAPTER 11.indd 352 3/7/09 9:38:31 AM


la mano inVisible 353

la industria. Una vez que se alcanza, no hay nuevos incentivos para que nuevas empresas entren en la
industria, porque todas las empresas existentes obtienen una utilidad económica nula.
Al estudiar el movimiento hacia el equilibrio competitivo de largo plazo, se parte de una situa-
ción inicial en la que el precio estaba por arriba del valor mínimo del costo promedio a largo plazo
y todas las empresas obtenían utilidades económicas. Supongamos que, por el contrario, se comien-
za con una situación en la que el precio es menor que el valor mínimo de CPL. En este caso, las
empresas existentes tendrían utilidades económicas negativas (es decir, pérdidas económicas), que
motivaría que algunas salieran de la industria. El éxodo desplazaría a la izquierda la curva de oferta
y aumentaría los precios y las iniciativas de las empresas para acrecentar sus existencias de capital. El
proceso continuaría hasta que todas las empresas tuvieran otra vez la posición de equilibrio a largo
plazo que se representa en la sección derecha de la figura 11.15.

LA MANO INVISIBLE
Como lo observó con claridad Adam Smith hace más de dos siglos, la mano in-
visible del afán de lucro, en especial la zanahoria de las ganancias o el palo de
las pérdidas económicas, conduce a las industrias competitivas a sus respectivas
posiciones de equilibrio a largo plazo. Pero aun cuando ninguna empresa pre-
tende fomentar de manera consciente el bienestar social general, hay algunas
características increíblemente atractivas del equilibrio competitivo a largo plazo.
Como describiera Smith, las acciones de un industrial,

sólo buscan su seguridad; y al dirigir dicha industria de tal modo que su


producto sea el de mayor valor, sólo busca su propia ganancia; y, como en
muchos otros casos, se deja llevar por una mano invisible que estimula un
final que no era su intención. Tampoco es siempre lo peor para la sociedad
que no formaba parte de ella. Al buscar su propio interés, con frecuencia
promueve el de la sociedad con más eficacia de lo que en realidad preten-
día.8

¿En qué sentido es atractivo el equilibrio de largo plazo en los mercados


competitivos desde la perspectiva de la sociedad como un todo? Para empezar,
el precio es igual al costo marginal, tanto a corto como a largo plazo, lo cual
significa que el equilibrio es eficiente en el sentido antes analizado: agota todas
las posibilidades de intercambios mutuamente beneficiosos. La última unidad
de producto consumida vale exactamente lo mismo para el comprador que los
recursos que se necesitan para producirla. Más aún, el precio es igual al punto
mínimo en la curva de costo promedio a largo plazo, esto significa que no hay
una forma más económica de fabricar el producto. Por último, los productores
sólo ganan una tasa de utilidad normal, que es el costo de oportunidad de los
recursos que invirtieron en sus empresas. El público no paga un centavo más de
lo que le costó a la empresa darle el servicio.
Es más sorprendente aún que estas propiedades de eficiencia sean el volu-
men total de actividad que coordina el mecanismo del mercado. En Ithaca, du-
rante el invierno, un camión con alimentos se estaciona toda la noche afuera de
los dormitorios de Cornell para que cualquier estudiante pueda salir a las 3 a.m. y
a unos cuantos pasos compre una taza de café caliente por un dólar. Ningún estu-
diante le dijo al chofer del camión que se estacionara ahí ni que comprara vasos
desechables ni gas propano para su estufa portátil. Una tienda cerca de mi casa
“¿Quién necesita un clavo de ese tamaño?”
me vende un cartucho nuevo para mi impresora en el momento que sea o una “¿A quién le importa? Lo esencial es que se cumplió el proyecto de
hoja nueva con punta de carburo para la sierra de brazos radiales que tengo en la clavos de un solo golpe.”

8
Adam Smith, The Wealth of Nations, capítulo 2. La edición original en inglés se encuentra en http://www.online-literature.com/
view.php/wealth_nations/24?term=invisbible%20hand.

11_CHAPTER 11.indd 353 3/7/09 9:38:32 AM


354 Capítulo 11 CompetenCia perfeCta

cochera. En el departamento de carnes del supermercado venden conejo fresco viernes y sábados,
y todas las mañanas llega al amanecer un camión con pez espada capturado en las costas de Maine.
En cuestión de horas, varias líneas aéreas están listas para trasladarme a Nueva York, Los Ángeles o
Cedar Rapids, Iowa. Toda esta actividad y mucha más ocurre sin coordinación central absoluta, es el
resultado de muchos agentes económicos que se esfuerzan por obtener utilidades económicas.
En economías planificadas, los recursos no los asignan los mercados sino los comités de pla-
neación central. Debido a los límites naturales de la información que pueden procesar los comités,
no tienen la capacidad de especificar con detalle las características de los productos que exigen sus
planes. Por lo tanto, los trabajadores y administradores de economías planificadas con frecuencia
tienen la capacidad de interpretar sus pedidos de producción a su modo.
El ejemplo de la famosa caricatura rusa reimpresa en la página anterior muestra la respuesta del
administrador de una fábrica de clavos para techos a quien, según el plan, se le pidió que abasteciera
10 000 libras de clavos para techos durante el mes de agosto. Tuvo el ingenio de descubrir que la
forma más sencilla de cumplir con su cuota era producir un solo clavo de 10 000 libras.
Cualesquiera que sean los fallos que tenga, no se puede acusar al sistema del mercado de pro-
ducir artículos que la gente no quiera comprar. En el sistema del mercado, el consumidor manda y
las empresas que no logran proveer a los consumidores lo que quieren, se enfrentan a la extinción
económica.9 La cuestión de si los planes centrales son más eficientes que los incentivos del mercado
fue tema de un acalorado debate durante casi todo el siglo xx. Pero ya no lo es. Antes de su disolu-
ción a finales de la década de 1980, las economías controladas del mundo introdujeron incentivos de
mercado en un intento desesperado por recuperar sus niveles de producción.
Lo anterior no significa que en todos los casos los mercados competitivos lleven al mejor resul-
tado posible. Por el contrario, en los últimos capítulos verá que los sistemas de mercado son insufi-
cientes en muchas ocasiones.
Además, los reclamos de eficiencia en nombre de asignaciones competitivas dependen de la
distribución inicial de los recursos entre los miembros de la sociedad. Los mercados son eficientes
para producir lo que la gente exige y lo que se produce depende del ingreso que reciben personas
específicas. Si usted no cree que sea justa la distribución subyacente de recursos, no hay un motivo
convincente para que apruebe el modelo de bienes y servicios proporcionados por mercados com-
petitivos. Pero no es necesario que tenga un punto de vista ingenuo del mercado competitivo para
apreciar su poder realmente sorprendente para generar orden a partir de la complejidad.

APLICACIÓN: EL COSTO
DE INSUMOS EXTRAORDINARIOS
EL PROYECTO DE RIEGO
Ahora nos encontramos en situación de volver a la pregunta con la que empezó el capítulo, a saber,
si un sistema de riego respaldado por el Estado que duplique la cosecha aumentará el ingreso de los
agricultores pobres. Hay que recordar que los agricultores en cuestión viven en un condado aislado y
alquilan a los terratenientes sus parcelas.
Considere primero la situación actual sin sistema de riego. Los agricultores podrían considerar-
se como los operadores de pequeñas empresas competitivas. Alquilan la tierra, proveen su mano de
obra y se quedan con las ganancias de vender su grano en un mercado tan grande que sus ofertas no
influyen de manera notable en el precio del grano, que es de 10 dólares/bushel. En aras de la senci-
llez, no se tiene en cuenta el costo de la semilla, la herramienta y los insumos menores.
Suponga que un agricultor puede cultivar 40 acres y que, sin riego, la tierra produce 30 bushels
de grano por acre al año. Su ingreso total de la venta del grano sería entonces de 12 000 dólares/año,
de los cuales debe descontar el alquiler de la tierra. ¿Cómo se determinará el alquiler?
Suponga que la alternativa de trabajar como agricultor es emplearse en una fábrica por 6 000
dólares/año y que laborar en una fábrica por lo general no se considera ni más ni menos agradable

9
El afamado economista de Harvard John Kenneth Galbraith objetaba este punto de vista. En el capítulo 13 se ponderan sus argu-
mentos.

11_CHAPTER 11.indd 354 3/7/09 9:38:32 AM


apliCaCión: el Costo de insumos extraordinarios 355

que la agricultura. Si, por ejemplo, el alquiler sólo fuera de 5 000 dólares/año por una parcela de
40 acres, entonces todos los trabajadores del condado preferirían cultivar la tierra que trabajar en
fábricas porque sus ingresos netos serían de 7 000 dólares en vez de 6 000. Si hubiera mucho más
obreros que pudieran labrar la limitada oferta de tierra para cultivo del condado, habría un exceso de
demanda de parcelas a un precio de alquiler de 5 000 dólares/año. Los obreros pujarían unos contra
otros y la licitación continuaría hasta que el precio de alquiler por una parcela de 40 acres llegara a
6 000 dólares/año. A ese precio, un agricultor tendría 6 000 dólares de la venta de su cosecha, pero
las opciones de trabajar en el campo y en la fábrica le serían indiferentes. En estas condiciones, el
alquiler de la tierra nunca podría ser mayor que 6 000 dólares durante mucho tiempo porque, de
aumentar, el ingreso neto de la granja sería menor que los 6 000 dólares al año y cualquiera preferiría
trabajar en la fábrica en vez de en el campo.
Ahora se verá qué pasa con la introducción del proyecto de riego. Con una producción de grano
de 60 bushels/acre en lugar de 30, el ingreso total anual por la producción de una granja de 40 acres
sería de 24 000 dólares en vez de 12 000. Si el alquiler de la tierra se mantuviera en su nivel original
de 6 000 dólares/año, un agricultor ganaría 18 000 dólares/año en vez de 6 000/año. De hecho, el
proyecto de un incremento tan drástico del ingreso en el campo fue lo que originalmente atrajo
tanto apoyo para el proyecto de ley para el riego.
Sin embargo, los simpatizantes del proyecto de ley no tomaron en cuenta que, después de la in-
troducción del sistema de riego, los alquileres de las tierras no se mantendrán en 6 000 dólares/año.
Está de más decir que los obreros de las fábricas estarían encantados de pujar por una oportunidad
para alquilar una parcela con la que sus ingresos aumentarían de 6 000 a 18 000 dólares/año. Ante
esta presión de la puja, el precio de alquiler de la tierra seguiría subiendo hasta llegar a 18 000 dóla-
res/año. (Si, por ejemplo, sólo fuera de 17 000 dólares, el obrero de una fábrica podría dedicarse al
campo y su ingreso anual aumentaría de 6 000 a 7 000 dólares.) Una vez que el alquiler anual de una
parcela de 40 acres llega a 18 000 dólares, se restablece el balance entre las oportunidades de trabajo
en el campo y en la fábrica.
Cabe recordar que nuestra hipotética asesora en legislación estatal está en contra del proyecto
de riego basándose en que, a largo plazo, no aumentaría el ingreso de los agricultores. Tiene ra-
zón al percibir que el proyecto de riego respaldado por el Estado no beneficiaría a los empobrecidos
agricultores, sino a los propietarios de la tierra. En vista de que los ingresos de estos propietarios ya
son altos, gastar dinero de los contribuyentes para aumentar aún más sus ingresos no tiene ningún
sentido social.10
Este ejemplo ilustra la idea importante de que fuerzas poderosas tienden a igualar los costos to-
tales promedio de distintas empresas en una industria competitiva. En este caso se ajustó el precio de
la tierra para equilibrar los costos promedio de las granjas con riego respecto a los costos promedio
de cultivar en otras partes.

UNA ADMINISTRADORA EFICIENTE


Suponga que una empresa es como cualquier otra, salvo que contrata a una administradora ex-
traordinariamente eficiente. Es tal su eficiencia que la empresa obtiene 500 000 dólares en ganancias
económicas al año en una industria en la que las utilidades económicas de las demás empresas son
prácticamente nulas. Como recibe el mismo salario que los demás administradores, los costos de la
empresa que la contrató son mucho más bajos que los del resto de las empresas en la industria. No
obstante, genera un fuerte incentivo para que otras empresas busquen llevársela ofreciéndole un
salario más alto.
Suponga que una nueva empresa le ofrece 300 000 dólares más que su sueldo anual vigente y
ella acepta. Esa nueva empresa tendría entonces utilidades económicas de 200 000 dólares/año. No
es tan bueno como 500 000 dólares/año, pero 200 000 dólares/año sigue siendo mejor que la ganan-
cia normal que obtendría su patrón original sin ella.
Pero algunas otras empresas tienen el incentivo de ofrecerle aún más a la administradora. La
teoría dice que la puja debería continuar hasta que el ahorro en costos, de los que ella es responsable,

10
Desde luego, el proyecto de riego seguiría siendo atractivo si su costo fuera inferior al valor del grano extra resultante.

11_CHAPTER 11.indd 355 3/7/09 9:38:33 AM


356 Capítulo 11 CompetenCia perfeCta

se incorpore por completo en su salario, es decir, hasta que su salario sea de 500 000 dólares/año
superior al de un administrador ordinario. Y en cuanto la oferta de su salario llega a ese nivel, la em-
presa que la contrata ya no disfruta de una ventaja de costos sobre las demás empresas en la industria.
La existencia de dichas ofertas competitivas por los insumos permite suponer que las empresas de
una industria competitiva tienen escasamente los mismos costos totales promedio en equilibrio.

EJERCICIO 11.6
Suponga que las empresas de una industria tienen administradores “competentes” y sus uti-
lidades económicas son nulas. El administrador de una de las empresas renuncia de repente y
cuando la empresa anuncia el puesto con un salario original de 50 000 dólares al año (que es el
salario actual para administradores competentes en la industria) sólo responden solicitantes
incompetentes. Si la empresa pagara este salario a un administrador incompetente, su pérdida
económica sería de 20 000 dólares anuales. ¿Cuál sería el salario adecuado para que la empresa
contrate a un administrador incompetente?

CURVA DE OFERTA A LARGO PLAZO


EN INDUSTRIAS COMPETITIVAS
Ya se vio que la curva de oferta a corto plazo en una industria perfectamente competitiva es la suma
horizontal de las curvas de costo marginal a corto plazo de las empresas individuales. Pero la curva
de oferta a largo plazo correspondiente para una industria competitiva no es la suma horizontal de
las curvas de costos marginales a largo plazo de las empresas individuales. En las siguientes seccio-
nes, la tarea será deducir la curva de oferta a largo plazo para industrias competitivas que operan en
una variedad de condiciones de costos.

CURVA DE OFERTA A LARGO PLAZO


CON CURVAS CPL EN FORMA DE U
¿Cómo se ve la curva de oferta a largo plazo en una industria en la que todas las empresas tienen cur-
vas de costo promedio a largo plazo (CPL) con forma de U? Suponga, en especial, que estas curvas
son como la CPL1 marcada en la sección de la derecha de la figura 11.16 y que la curva de demanda a
la que se enfrenta la industria inicialmente es la marcada como D1 en la sección de la izquierda. Con
esta curva de demanda, la industria se encontrará en equilibrio a largo plazo cuando cada empresa
disponga de unas existencias de capital que hagan subir la curva de costo marginal a corto plazo
marcada como CMCi en la sección de la derecha. La cantidad de empresas en la industria se ajusta de
modo que la curva de oferta a corto plazo, señalada como SSR en la sección de la izquierda, interseque
a D1 en un precio igual al valor mínimo de CPLi. (Si hubiera más o menos empresas que las indicadas,
cada una representaría una pérdida o utilidad económica.)
Ahora suponga que la demanda cambia hacia la derecha de D1 a D2, intersecando la curva de
oferta a corto plazo de la industria en el precio P2. El efecto a corto plazo sería que cada empresa
aumentaría su producción de Q*i1 a Q*i2, lo cual provocaría una utilidad económica medida por el rec-
tángulo sombreado de la sección derecha de la figura 11.16. Con el paso del tiempo estas ganancias
atraen a más empresas hacia la industria hasta que el desplazamiento de la oferta a la derecha (a S* en
la sección de la izquierda) otra vez da como resultado un precio de CPL mín. Entonces, la respuesta
a largo plazo ante un incremento en la demanda es aumentar la producción de la industria mediante
una mayor cantidad de empresas dentro de ella. Mientras la expansión de la producción de la indus-
tria no provoque el incremento en los precios de capital, mano de obra y demás insumos, no habrá
un incremento a largo plazo en el precio del producto.11

11
Se dirá más sobre lo que sucede cuando los cambios en la producción de la industria provocan variaciones en los precios de los
insumos.

11_CHAPTER 11.indd 356 3/7/09 9:38:33 AM


CurVa de oferta a largo plazo en industrias CompetitiVas 357

Mercado Empresa individual FIGURA 11.16


Curva de oferta a largo
P $/Q plazo en industrias
SSR = ΣCMC competitivas
S* Cuando las empresas
CMCi tienen la libertad de
CPLi CTPi entrar o salir del mercado,
P2 el precio no se puede
separar del valor mínimo
CPL mín. SLR de la curva Cpl a largo
plazo. si los cambios en la
producción de la industria
D2 no afectan los precios de
D1 los insumos, la curva de
oferta a largo plazo es Slr,
Q Qi una línea horizontal en el
Q*1 Q*2SR Q*2LR Q*i1 Q*i2
valor mínimo de Cpl.

Si la demanda se hubiese desplazado a la izquierda de D1, se tendría otra historia paralela: el


precio habría caído a corto plazo, las empresas habrían ajustado sus ofertas y las pérdidas económicas
resultantes habrían provocado que algunas empresas abandonaran la industria. El éxodo movería la
oferta de la industria a la izquierda hasta que el precio subiera de nuevo a CPL mín. Aquí, una vez
más, la respuesta a largo plazo ante un cambio en la demanda se ajusta mediante el cambio en la
cantidad de empresas. Con las curvas CPL en forma de U no existe tendencia hacia un descenso en
la demanda para producir una disminución del precio a largo plazo.
En resumen, la curva de oferta a largo plazo de una industria competitiva con curvas CPL en
forma de U y precios de insumos constantes es una línea horizontal en el valor mínimo de la curva
CPL. A largo plazo, todos los ajustes a las variaciones en la demanda no ocurren por cambios en los
precios sino por variaciones en la cantidad de empresas que sirven al mercado. Después de posibles
desviaciones sustanciales a corto plazo, el precio muestra una tendencia constante a inclinarse hacia
el valor mínimo del costo promedio a largo plazo.

OFERTA DE LA INDUSTRIA CUANDO


CADA CURVA CPL ES HORIZONTAL
Igual que en el caso de las curvas CPL en forma de U, la curva de oferta de la industria a largo plazo
cuando la curva CPL de cada empresa es horizontal será de nuevo una línea horizontal (si se supone
que los precios de los insumos no varían con los cambios en la producción de la industria). Pero hay
una diferencia sobresaliente entre ambos casos: cuando las empresas tienen curvas CPL en forma de
U idénticas, es posible predecir que cada empresa producirá la cantidad que corresponda al punto
mínimo en su curva CPL. Por lo tanto, se tiene una industria que consta de empresas que generan el
mismo nivel de producción.
Por el contrario, con las curvas CPL horizontales no hay un punto de costo mínimo único. El
CPL es igual en cualquier nivel de producción lo cual conduce a una indeterminación que no se
presenta en el caso anterior. No es posible pronosticar cómo será la distribución de tamaño de las
empresas en el caso de las curvas CPL horizontales. Quizás haya un puñado de empresas grandes,
muchas pequeñas o una mezcla de diferentes tamaños. Lo único que es posible decir de manera
confiable es que el precio a largo plazo se inclinará hacia el valor de CPL.

CÓMO AFECTAN LOS CAMBIOS EN LOS PRECIOS


DE LOS INSUMOS LA OFERTA A LARGO PLAZO
En el análisis de las curvas de costo del capítulo 10, que conforma la base del presente análisis de la
oferta en competencia perfecta, un supuesto importante fue que los precios de los insumos no varían

11_CHAPTER 11.indd 357 3/7/09 9:38:35 AM


358 Capítulo 11 CompetenCia perfeCta

con la cantidad de producto generado. Para una sola empresa cuyas compras de insumos constituyen
sólo una pequeña fracción del mercado total de insumos, esta hipótesis es factible. Además, en mu-
chos casos, incluso las demandas de insumos de toda la industria sólo constituyen una pequeña parti-
cipación del mercado general de insumos. Por ejemplo, aunque la industria de seguros emitiera 20%
más pólizas este año que el anterior, emplea un porcentaje tan pequeño de los suministros totales
disponibles de secretarias, computadoras, ejecutivos y demás insumos que no debe influir de forma
significativa en los precios de los mismos. De manera que en este caso también se puede suponer que
los precios de los insumos no dependen de la producción.
Pero hay al menos algunas industrias en las que el volumen de insumos adquiridos constituye
una participación valiosa de todo el mercado. Por ejemplo, el mercado de las líneas aéreas comercia-
les consume una parte importante de la cantidad total de titanio que se vende cada año. En dichos
casos, un incremento importante en la producción de la industria se acompaña con frecuencia de
alzas significativas en los precios de los insumos.
deseconomía pecuniaria Cuando ocurre lo anterior, se tiene lo que se conoce como deseconomía pecuniaria, un au-
aumento de los costos de mento de los precios de los insumos cuando se incrementa la producción de la industria.12 Aun cuan-
producción que ocurre cuando do la industria puede expandir la producción de manera indefinida sin usar más insumos por unidad
una expansión de producción de producto, el punto mínimo en la curva CPL de cada empresa de todas maneras es una función
de la industria ocasiona el creciente de la producción de la industria. Por ejemplo, observe en la sección de la izquierda de la
aumento de precios de los
figura 11.17 que la curva CPL de la empresa para una producción de la industria de Q2 se encuentra
insumos.
por encima de su curva CPL para una producción de la industria de Q1 < Q2 y que la curva CPL para
una producción de la industria de Q3 > Q2 es todavía más alta. A cada nivel de producción de la indus-
tria le corresponde una curva CPL diferente porque los precios de los insumos son distintos en cada

FIGURA 11.17
Curva de oferta de
largo plazo en una
industria de costos
crecientes
Cuando los precios de
los insumos aumentan
con la producción de la
Empresa individual Mercado
industria, la curva Cpl
de cada empresa también
se incrementa con la Dólares/unidad de producto Dólares/unidad de producto
producción de la industria
(sección de la izquierda).
por lo tanto, la curva Cpl CPLQ = Q3 SLR
de la empresa cuando la
producción de la industria
CPLQ = Q2
es Q2 se encuentra arriba
de su curva Cpl cuando la
producción de la industria CPLQ = Q1
es Q1 (sección de la
izquierda). las empresas se
seguirán inclinando hacia
los puntos mínimos de las Qi Q
curvas Cpl (Q*i , sección Q*i Q1 Q2 Q3
de la izquierda), pero
como este punto mínimo
depende de la producción
de la industria, la curva
de oferta de la industria a
largo plazo (Slr, sección de
la derecha) tendrá ahora
una pendiente positiva.

12
Por consiguiente, una deseconomía pecuniaria implica que los precios de los insumos bajarán cuando la producción de la industria
se contraiga.

11_CHAPTER 11.indd 358 3/7/09 9:38:36 AM


CurVa de oferta a largo plazo en industrias CompetitiVas 359

nivel de producción. La curva de oferta a largo plazo para una industria así trazará los puntos mínimos
de estas curvas CPL. Por lo tanto, en la curva de oferta de la industria a largo plazo (SLR, sección de
la derecha), Q1 corresponde al punto mínimo de la curva CPL de la empresa cuando la producción
de la industria es Q1 (sección de la izquierda); Q2 corresponde al punto mínimo de la curva CPL para
Q2; y así sucesivamente. Con las deseconomías pecuniarias, la curva de oferta a largo plazo tiene
una pendiente positiva aun cuando la curva CPL de la empresa tenga forma de U. Las deseconomías
pecuniarias también producen una curva de oferta de la industria con pendiente positiva cuando la
curva CPL de cada empresa es horizontal. Las industrias competitivas en las que el incremento de
los precios de los insumos provoca curvas de oferta con pendiente positiva se denominan industrias
de costos crecientes.
También hay casos en los que los precios de los insumos pueden caer de manera drástica con la
expansión de la producción de la industria. Por ejemplo, lo anterior ocurre si los insumos se fabrican
mediante tecnologías en las que las economías de escala son sustanciales. Por ejemplo, un incremen-
to drástico en la construcción de carreteras podría provocar una mayor explotación de las economías
de escala en la producción de equipo de excavación, lo que deriva en un precio más bajo para ese in-
sumo. Dichos casos se llaman economías pecuniarias y dan origen a una curva de oferta de la industria
a largo plazo con pendiente negativa, incluso cuando la curva CPL de cada empresa sea horizontal
o en forma de U. Las industrias competitivas en las que la disminución en los precios de los insumos
genera curvas de oferta con pendiente negativa se llaman industrias de costos decrecientes.

¿Por qué las fotos a color tienen un precio menor que las fotos en blanco y negro?
Cuando era niño las fotografías a color eran un lujo, su precio era varias veces mayor que el de las
fotos en blanco y negro. Hoy, la tienda de revelado de fotos cerca de mi casa cobra 14.91 dólares por
el revelado e impresión de un rollo de 36 exposiciones en blanco y negro, y sólo 6.99 dólares por un
rollo del mismo tamaño de película a color. Esta caída en el precio relativo de las fotografías a color EL NATURALISTA
ocurre a pesar del hecho de que el proceso a color sigue siendo más complejo que el que se emplea ECONÓMICO
para blanco y negro.
Si el proceso para color es más complejo que para blanco
11.1
y negro, ¿por qué su precio es menor? La respuesta se debe en
parte a las economías de escala de la producción de maquinaria
empleada para ambos tipos de impresiones. En los inicios de la
fotografía a color la película era cara y los colores solían desvane-
cerse rápidamente, por lo que casi todo mundo usaba el blanco
y negro. El elevado volumen del procesamiento de la fotografía
en blanco y negro permitió a su vez la producción de máquinas
para ese fin más económicas debido a las economías de escala.
A medida que el precio de la película a color bajó al paso del
tiempo y mejoró su calidad, más personas empezaron a usarla
y la demanda del equipo de procesamiento de color aumentó 4x6 4x6
ImPresIón ImPresIón en
de manera gradual. De nuevo, debido a las economías de escala a color Blanco y negro

en la producción de equipo de procesamiento, bajó el costo de


un insumo importante para el revelado a color: una economía cada una cada una

pecuniaria. Al mismo tiempo, la disminución en la producción


de equipo de procesamiento en blanco y negro condujo a un
incremento en su precio: una deseconomía pecuniaria.
Los cambios resultantes en los precios en equilibrio y las
cantidades de los dos tipos de revelado se presentan de manera
breve en la figura 11.18. Observe que las posiciones relativas de ¿Por qué las fotos a color tienen un precio menor que las fotos en blanco y negro?
las dos curvas de demanda son iguales para ambos años. Esto
significa que la industria de revelados de película estaría dispues-
ta a ofrecer cualquier cantidad total de impresiones en blanco y negro por un precio más bajo que por
la misma cantidad de impresiones a color en ambos años. El cambio en los esquemas de la demanda,
aunado a las curvas de oferta con pendiente negativa de ambos mercados, explica la inversión obser-
vada en los precios relativos.

11_CHAPTER 11.indd 359 3/7/09 9:38:38 AM


360 Capítulo 11 CompetenCia perfeCta

FIGURA 11.18
Economías pecuniarias y
precio de las fotos a color
y en blanco y negro
debido a las economías
de escala en la producción P Dcolor P
del equipo que se usa para 1955 DByN 2005
procesar películas, las curvas
de oferta a largo plazo de los
revelados a color y en blanco Pcolor
y negro tienen pendientes
negativas. en 1955, cuando la DByN Dcolor
calidad de la película de color PByN
era mala, la mayoría de la gente
demandaba el blanco y negro, Pcolor
lo que daba por resultado
precios más bajos. en 2005, PByN Scolor
por el contrario, la demanda Scolor
de película a color fue mucho
SByN SByN
mayor que la de blanco y
negro. el resultado es que los Q Q
revelados a color ahora son Qcolor QByN QByN Qcolor
menos caros que los de blanco Impresiones/sem Impresiones/sem
y negro, aun cuando el equipo
para el procesamiento de
película a color sigue siendo
más complicado.

ELASTICIDAD DE LA OFERTA
En el capítulo 5 se definió la elasticidad precio de la demanda como una medida de la sensibilidad de
la cantidad demandada a las variaciones del precio. Existe un concepto análogo para medir la sensi-
elasticidad-precio de la bilidad de la cantidad ofrecida a las variaciones de precio. Como es natural, se denomina elasticidad-
oferta cambio porcentual precio de la oferta. Suponga que está en un punto (Q, P) sobre la curva de demanda de la industria
de la cantidad ofrecida que se que se muestra en la figura 11.19, donde un cambio en el precio de ∆P genera un cambio de ∆Q en la
presenta como reacción a un cantidad ofrecida. Por lo tanto, la elasticidad precio de la oferta, indicada como eS, está dada por
cambio de 1% en el precio del
producto.
∆Q P
εS = (vea nota a pie de página abajo). (11.1)
∆P Q

Igual que en el caso de la elasticidad de la demanda, la elasticidad de la oferta tiene una interpreta-
ción simple en términos de la geometría de la curva de oferta de la industria. Cuando ∆P es pequeña,
la relación ∆P/∆Q es la pendiente de la curva de oferta; esto significa que la relación ∆Q/∆P es
la inversa de la pendiente. Por consiguiente, la elasticidad precio de la oferta se interpreta como el
producto de la relación precio/cantidad y la inversa de la pendiente de la curva de oferta:

P 1
εS = . (11.2)
Q pendiente

Debido a la ley de rendimientos decrecientes, la curva de oferta competitiva a corto plazo de


la industria siempre tiene pendiente positiva, lo cual significa que la elasticidad de la oferta a corto

13
En términos del cálculo, la elasticidad de la oferta se define como
P dQ
εS = .
Q dP

11_CHAPTER 11.indd 360 3/7/09 9:38:41 AM


apliCaCión del modelo CompetitiVo 361

Precio (dólares/unidad de salida) FIGURA 11.19


S Elasticidad de la oferta
en el punto A, la elasticidad
de la oferta está dada por
(∆Q/∆P) (P/Q). Como la
A curva de oferta a corto
P ∆P plazo siempre tiene
pendiente positiva, la
∆Q elasticidad de la oferta a
corto plazo siempre será
positiva. a largo plazo, la
elasticidad de la oferta
puede ser positiva, cero o
negativa.
Cantidad de salida/unidad de tiempo
Q

plazo siempre es positiva. En el caso de las industrias con una curva de oferta a largo plazo horizon-
tal, la elasticidad de la oferta a largo plazo es infinita. La producción se puede expandir de manera
indefinida sin que cambie el precio. Debido a las economías y deseconomías pecuniarias, las curvas
de oferta de la industria competitiva a largo plazo pueden tener, en casos específicos, pendiente po-
sitiva o negativa. En estos casos, las elasticidades de la oferta correspondientes a largo plazo serán
negativas o positivas.
Como se observó antes, la mayoría de las industrias sólo emplean una parte pequeña del vo-
lumen total de los insumos que se comercian en el mercado, lo cual significa que las moderadas
variaciones en la producción de la industria no influyen de manera significativa en los precios de los
insumos de la mayoría de las industrias. Por lo tanto, en las aplicaciones prácticas del modelo com-
petitivo, la mayoría de los economistas adoptan la hipótesis funcional de que las curvas de oferta a
largo plazo son horizontales. Desde luego que esta suposición siempre se puede modificar si se tiene
evidencia de que las economías o deseconomías pecuniarias son importantes.

APLICACIÓN DEL MODELO COMPETITIVO


Anteriormente en este capítulo se mencionó que los economistas aceptan que ninguna industria
satisface en forma estricta los cuatro requisitos de la competencia perfecta: un producto homogéneo,
empresas como tomadoras de precios, movilidad perfecta de los factores e información perfecta.
Para efectos prácticos, la pregunta importante es cuánto le falta a una industria de estas condiciones
antes de que falle la aplicación de las tendencias generales del modelo competitivo. Por desgracia,
no hay reglas rígidas para juzgar. En industrias en las que es fácil entrar y salir del mercado, como
la de líneas aéreas, una empresa se puede comportar como una tomadora de precios incluso en un
mercado en el cual sea el único competidor.14 En industrias en las que la entrada y salida es más
difícil, la existencia de un número relativamente grande de empresas establecidas no garantiza el
comportamiento de toma de precios. Es probable que en especial a corto plazo las empresas lleguen
a un acuerdo tácito para restringir la competencia de precios incluso cuando se tengan utilidades
fuera de lo normal.
A pesar de esta dificultad, la experiencia indica que muchas de las propiedades a largo plazo más
importantes del modelo competitivo son aplicables en la mayoría de las industrias, con la notable
excepción de aquéllas donde el gobierno crea barreras legales para evitar la entrada (por ejemplo,
requerir una licencia gubernamental para participar en un mercado, como solía ser el caso de la
industria de las líneas aéreas).
Como ejemplo, tome en cuenta tres aplicaciones breves que resaltan algunas de las perspectivas
que brinda el modelo perfectamente competitivo.
En algunos aeropuertos grandes es difícil entrar incluso en la industria de las líneas aéreas. Estos aeropuertos necesitan construir
14

nuevas infraestructuras para dar cabida a las líneas de transporte, lo cual podría llevar años, incluso décadas.

11_CHAPTER 11.indd 361 3/7/09 9:38:42 AM


362 Capítulo 11 CompetenCia perfeCta

SUBSIDIOS DE LOS PRECIOS COMO MEDIO PARA


RESCATAR GRANJAS FAMILIARES
A principios del siglo xx, más de 20% de la mano de obra estadounidense se ganaba la vida en el
campo. Ahora, la cifra correspondiente es inferior a 3%. Es obvio que el cambio no se debe a una dis-
minución drástica del consumo de alimentos. Más bien, se trata de una de las muchas consecuencias
de que los métodos agrícolas se hayan vuelto mucho más productivos en el transcurso del siglo.
Mientras la maquinaria agrícola es más grande y complicada, el tamaño de las parcelas cuyas
curvas de costo promedio a largo plazo dejaron de disminuir ha aumentado más que nunca. Antes,
las granjas familiares de menos de 100 acres eran lo común en el corazón de Estados Unidos; ahora,
las empresas grandes dedicadas al campo con varios miles de acres se han convertido en la norma.
En términos del modelo competitivo que se desarrolló en este capítulo, la granja familiar puede
considerarse una empresa en la que las existencias de capital dan origen a curvas de costo a corto
plazo indicadas en la figura 11.20 como CTPF y CMCF. Las curvas de costo correspondientes para la
granja corporativa se presentan como CTPC y CMCC. La competencia tiene el efecto de empujar el
precio de equilibrio a largo plazo hacia P*, el punto mínimo en la curva CPL. En P*, las granjas cor-
porativas obtienen una ganancia normal mientras las granjas familiares, con costos más altos, tienen
pérdidas económicas de ΠF, medidas por el rectángulo sombreado de la figura 11.20.

FIGURA 11.20
Curvas de costos de
granjas familiares y Dólares/unidad de producto
corporativas
Con los métodos agrícolas
modernos al alcance, las CTPF
granjas grandes tienen
costos unitarios mucho CMCF CPL
CTPC
más bajos que las granjas CMCC
pequeñas. un precio
que pague el costo de
las granjas corporativas ΠF
grandes implicará pérdidas P*
económicas importantes
para las granjas familiares. Q

A pesar de la intensa determinación de muchos agricultores familiares de conservar sus tierras,


las grandes pérdidas simplemente no son sostenibles durante muchos años. La mayoría de los agri-
cultores continúan trabajando tiempo después de que ya no son capaces de obtener una ganancia
equivalente al costo de oportunidad de su tierra. Muchos persisten incluso mucho después de que
dejaron de ganar el costo de oportunidad de su propia mano de obra. Y una cantidad importante
sigue pidiendo prestado con la garantía del valor de su único activo significativo: sus tierras. Pero el
crédito no se puede extender sin límite y, a falta de intervención gubernamental, la tendencia a largo
plazo para los agricultores familiares ha sido abandonar la industria y vender lo que les queda de
tierras a las granjas corporativas más eficientes.
Contrario a los supuestos estilizados del modelo de competencia perfecta, este proceso de mo-
vilidad de recursos dista mucho de ser perfecto. La agricultura familiar es una forma de vida que la
gente no abandona porque sí cuando tienen en contra los condicionantes comerciales. Existe una
gran simpatía entre los votantes estadounidenses por la grave situación de los agricultores familiares.
A nadie le gusta ver en el noticiario de la noche escenas de familias abrazadas y llorando porque el
subastador vende lo que queda de sus posesiones. Se valora que estas familias se hayan esforzado
toda su vida trabajando cuando muchos otros se ganan la vida vendiendo drogas o asaltando tiendas.
Los agricultores familiares a quienes el Congreso ha incluido en programas legislativos diseñados
para que puedan permanecer en sus granjas cuentan con la simpatía de la gente.

11_CHAPTER 11.indd 362 3/7/09 9:38:43 AM


apliCaCión del modelo CompetitiVo 363

Los subsidios a los precios de productos agrícolas están entre los programas más importantes.
Los detalles de los programas de subsidio son sumamente complejos pero, para efectos del presente
análisis, baste decir que el gobierno anuncia un precio para un producto determinado y después se
dispone a adquirir lo que los compradores particulares no compran.15 Una de las metas más impor-
tantes de los programas de subsidio a los precios, aunque no siempre la mejor, es mantenerlos lo
suficientemente altos para evitar la quiebra de las granjas familiares.
Da tristeza decirlo, pero estos programas han fracasado de manera rotunda. Pero es mucho más
triste que incluso quienes tienen pocos conocimientos sobre la dinámica del mercado competitivo le
podrían haber explicado al Congreso por qué era inevitable este resultado. Para ejemplificarlo, su-
ponga que el subsidio a los precios se establece en PG en un mercado en el cual el precio sin subsidio
sería de P*. En la figura 11.21 se observa que el efecto a corto plazo es provocar el aumento de pro-
ducción de las granjas familiares a QP y que las granjas corporativas incrementen las suyas a QC. Con
estos niveles de producción, las granjas familiares tienen una pérdida económica que se indica en la
figura 11.21 con el rectángulo en pantalla más clara marcado como ΠF, mientras las granjas corpora-
tivas obtienen utilidades económicas que se indican con el rectángulo con pantalla más oscura ΠC.

Dólares/unidad de producto FIGURA 11.21


Efecto a corto plazo
de los subsidios a los
precios agrícolas
CTPF al inicio, los subsidios
CPL
CMCF de precios reducen las
CTPC CMCC pérdidas de las granjas
familiares, al tiempo
ΠF que generan ganancias
PG económicas para las granjas
ΠC
P* corporativas. sin embargo,
a largo plazo, sólo sirven
Q para aumentar los precios
QF QC de la tierra.

En el corto plazo, hasta el punto en el que la nueva pérdida sea menor a la que era soportada por
los agricultores familiares, la intención del efecto del subsidio a los precios es ayudarles. Pero el alivio
es temporal. Para ver por qué, observe primero que el mismo subsidio a los precios que reduce las
pérdidas a corto plazo de las granjas familiares genera ganancias positivas para las granjas corpora-
tivas. Sin embargo, se sabe que las ganancias positivas no se sostienen en una industria con entrada
libre. Atraerán a los externos para que pujen por la tierra, de modo que ellos también puedan ganar
un rendimiento mayor del normal. Esta puja provocará que los precios de la tierra aumenten hasta el
punto en que las granjas corporativas ya no obtengan ganancias económicas. Pero con los precios de
la tierra superiores a los de antes, las curvas de costo de las granjas corporativas y familiares suben.
En el caso de los propietarios, parte de esta incomodidad de las pérdidas económicas se alivia con el
mayor valor implícito de su tierra. Pero muchas de las familias agrícolas que alquilan la tierra, no
tienen dicha compensación.
Conforme avanza la tecnología, y con ella la escala de las granjas más eficientes, se establece un
escenario de mayor tensión para las granjas familiares y la presión correspondiente para aumentar
el nivel de subsidio gubernamental a los precios. Se ha puesto en marcha una rueda en la que el
subsidio a los precios sigue en escalada y sólo generará otra ronda de aumento en los precios de la
tierra. Como política para proteger la viabilidad económica a largo plazo de las granjas familiares,
el programa del subsidio a los precios agrícolas difícilmente podría haberse concebido en peores
circunstancias.

15
Los programas recientes de subsidios a los precios agrícolas, como el programa de préstamos, funcionan mediante el pago a los
agricultores de la diferencia entre un precio objetivo específico y el precio real en el mercado. De acuerdo con estos programas, el
precio que pagan los compradores puede ser inferior al precio de equilibrio no regulado.

11_CHAPTER 11.indd 363 3/7/09 9:38:45 AM


364 Capítulo 11 CompetenCia perfeCta

Los economistas no están en posición de decirle al Congreso si conservar la granja familiar es


un objetivo que valga la pena. Es una cuestión política, pero dado que el Congreso ha decidido per-
seguir este objetivo, los economistas pueden dar consejos sobre las políticas que tal vez sean las más
eficaces. Los subsidios de precios fracasaron debido a la puja competitiva que provocaron por la tie-
rra. Sus efectos a largo plazo fueron generar un alza en los precios de la tierra, mientras que ofrecían
poca garantía de supervivencia de las pequeñas granjas familiares. Una forma mucho más directa y
eficiente de ayudar a los agricultores familiares sería reducir los impuestos sobre los ingresos; o, en
caso de necesidad extrema, proporcionarles subvenciones en efectivo inmediatas.

EL ATRACTIVO ILUSORIO DE GRAVAR A LAS EMPRESAS


Como se vio en el capítulo 2, a los líderes políticos con frecuencia les parece más sencillo gravar
con nuevos impuestos a las empresas en lugar de cobrar más impuestos a las personas físicas. Las
propuestas de gravar a las empresas normalmente incluyen declaraciones con respecto a que “corpo-
raciones adineradas pueden permitirse pagar más impuestos que los trabajadores”. Pero como se vio
en el capítulo 2, un impuesto aplicado al producto que vende una industria en general se transmite,
al menos en parte, a los consumidores.
A continuación se estudia una industria perfectamente competitiva donde las empresas indivi-
duales tienen curvas CPL con forma de U como la marcada con CPL en el sección de la derecha de
la figura 11.22. En el caso más común, variaciones moderadas en la producción de la industria no
tienen un efecto apreciable en los precios de sus insumos, con el resultado de que la curva de oferta a
largo plazo para esta industria es una línea horizontal en el punto mínimo de CPLi (la curva marcada
como SLR en la sección de la izquierda). Si D es la curva de demanda del mercado, entonces el precio
de equilibrio será P*.

FIGURA 11.22
Efecto de un impuesto
a la producción en una Mercado Empresa individual
industria perfectamente
competitiva Precio Precio
un impuesto de T dólares (dólares/unidad de producto) (dólares/unidad de producto) CPLi + T
por unidad de producto
aumenta las curvas Cpl y CMCi + T
CmC en T dólares (sección CPLi
SLR + T T
de la derecha). la nueva
curva de oferta a largo plazo P* + T P* + T CMCi
de la industria una vez más
es una línea horizontal en SLR
el valor mínimo de Cpl P* P*
(sección de la izquierda). el D
precio de equilibrio aumenta Q Qi
0 Q*2 Q*1 0 Q*i
en T dólares (sección de la
izquierda), lo cual significa Unidades de producto/semana
que 100% del impuesto se
transfiere a los consumidores.

Ahora suponga que se cobra un impuesto de T dólares por cada unidad de producto vendido en
el mercado. El efecto de este impuesto es cambiar las curvas CPL y CMC de cada empresa en forma
ascendente en T dólares (sección de la derecha de la figura 11.22). La nueva curva de oferta de la
industria a largo plazo una vez más es una línea horizontal en el valor mínimo de la curva CPL; en
esta ocasión, la curva SLR + T en la sección de la izquierda de la figura 11.22. El efecto del impuesto
es que aumenta el precio del producto en exactamente T dólares. La producción de la industria se
contrae de Q*1 a Q*2 (sección de la izquierda) y dicha contracción se logra a costa de las empresas que
abandonan la industria.
Se ve así que, en el caso de industrias competitivas cuyas empresas tienen curvas CPL en forma
de U y cuyos precios de los insumos son fijos (el caso empíricamente más relevante), la carga de

11_CHAPTER 11.indd 364 3/7/09 9:38:46 AM


apliCaCión del modelo CompetitiVo 365

un impuesto sobre la producción recae por completo en los consumidores. Como se verá en capí-
tulos posteriores, existe una variedad de motivos reales para aplicar impuestos a la producción de
industrias específicas. Sin embargo, la afirmación de que las corporaciones tienen más dinero que
las personas no es uno de los motivos. Afirmar lo contrario es un fraude y es poco probable que una
población con conocimientos de economía reelija a políticos que defiendan la imposición fiscal a
empresas con base en este fundamento.
Si las industrias competitivas de costos constantes pueden trasladar 100% de los impuestos a los
compradores, ¿por qué los grupos de presión de la industria se oponen de manera tan terminante a
los impuestos? En la figura 11.22 se ve que un efecto de un impuesto es reducir la producción total
de la industria. Dicha reducción se logra con la salida del negocio de algunas empresas. La quiebra
nunca es una experiencia agradable para los propietarios de una empresa y, a este respecto, no es de
sorprender que las asociaciones de empresarios industriales se opongan de manera tan tajante a los
nuevos impuestos.

ADOPCIÓN DE INNOVACIONES QUE AHORRAN COSTOS


El énfasis del economista en una empresa competitiva como tomadora de precios a veces provoca
la impresión de que las empresas competitivas hacen poco más que responder en forma pasiva a
las señales impersonales de los precios que ofrece el mercado. Esta impresión confunde tremenda-
mente. Lo cierto es que, por ejemplo, un camionero no puede hacer mucho por influir en las tarifas
del transporte de carga establecidas en el mercado abierto, pero sí puede y debe hacer algo para
garantizar su supervivencia.
La respuesta a corto plazo al aumento drástico del precio del combustible en la década de 1970
produjo de manera automática el tipo de ajustes que pronostica el modelo competitivo: pérdidas a
corto plazo, salida de la industria, incremento gradual de los precios y una recuperación paulatina
de la rentabilidad de las empresas que sobrevivieron. Pero el cambio en el ambiente también generó
oportunidades que algunas empresas explotaron para beneficio propio. En la siguiente sección de El
naturalista económico se pone como ejemplo un caso al respecto.

¿Por qué los camiones de 18 ruedas empezaron a usar perfiles aerodinámicos a mediados de
la década de 1970?
Antes de 1970, el perfil de un camión de remolque típico de 18 ruedas era como el que se muestra en
el primer dibujo. El área ancha y plana de la parte superior del tráiler estaba expuesta directamente EL NATURALISTA
a la fuerza del aire entrante, que a velocidades de carretera es considerable. Pero en 1970 el costo del ECONÓMICO
diesel era apenas de 0.30 dls./galón, por lo que en esos días forzar un poco más el motor no era una
desventaja tan grave. 11.2
Sin embargo, cuando los precios del diesel aumenta-
ron a más de un dólar por galón a principios de la década
de 1980, esa desventaja adquirió más importancia, hasta
el punto de que los empresarios buscaron la manera de
reducirla. Una de las innovaciones más exitosas fue el
perfil aerodinámico simple que ahora adorna la cabina de
casi todos los camiones que se ven en la carretera. Como
se aprecia en el dibujo inferior, su propósito es desviar el
viento hacia la parte superior del tráiler. El perfil de los
camiones de la actualidad sigue sin ser una obra maestra
de la aerodinámica, pero los camioneros consideran que
una menor resistencia al viento permite que su kilome-
traje aumente 15% a velocidades de carretera.
Los primeros camioneros en instalar el perfil aero-
dinámico lo hicieron cuando el nivel de precios de la in-
dustria estaba determinado por los costos más elevados ¿Por qué a mediados de la década de 1970
de manejar camiones que no lo tenían. Como resultado, aparecieron de pronto los perfiles aerodiná-
los primeros que adoptaron el perfil obtuvieron ganan- micos?

11_CHAPTER 11.indd 365 3/7/09 9:38:47 AM


366 Capítulo 11 CompetenCia perfeCta

cias económicas por su esfuerzo. Sin embargo, conforme pasó el tiempo, cada vez más camiones
empezaron a usar los perfiles y el nivel de precios de la industria disminuyó de manera gradual como
respuesta a los costos más bajos que los perfiles hicieron posibles. Hoy es raro ver un camión de
18 ruedas sin perfil aerodinámico. Ahora es posible suponer con seguridad que el ahorro de costos
resultante se refleja ampliamente en las tarifas más bajas del transporte de carga. El resultado es que
el dueño de un camión ahora debe instalar un perfil aerodinámico para ganar una tasa de utilidad
normal. Quienes no lo hacen, pagan el precio de las pérdidas económicas.

La lección de este ejemplo es que el empresario que obtiene ganancias económicas es quien
adopta las innovaciones de ahorro de costos antes que la competencia. La búsqueda de dicha inno-
vación es lo que evita que incluso la empresa tomadora de precios reaccione de forma pasiva a las
fuerzas económicas que están más allá de su control.

RESUMEN
• El objetivo declarado de la empresa es maximizar las utilidades • Los ajustes a largo plazo consisten no sólo en alteraciones al ta-
económicas. Las presiones competitivas del mercado la convier- maño de las existencias de capital de las empresas, sino también
ten en una suposición verosímil, aunque parezca imponer un en la entrada y salida de empresas. Si las empresas tienen curvas
grado excesivo e irreal de deliberación en los actos de muchos de costo promedio a largo plazo con forma de U, el precio de
administradores. Las utilidades económicas son la diferencia equilibrio de largo plazo es el valor mínimo de esas curvas y
entre los ingresos totales y los costos (implícitos y explícitos) de todas las empresas producirán el volumen correspondiente.
todos los recursos consumidos en la producción. Las utilidades • Las posiciones de equilibrio a corto y largo plazos son eficientes
económicas no deben confundirse con las utilidades contables, en el sentido de que el valor de los recursos que se usan para
que son la diferencia entre los ingresos totales y el costo explíci- elaborar la última unidad de producto es exactamente igual al
to de los recursos empleados. valor de ese producto para el comprador. Esto significa que la
• En el modelo económico de la competencia perfecta se supone posición de equilibrio agota todas las posibilidades de un inter-
un producto estandarizado, empresas que funcionan como to- cambio mutuamente benéfico. El equilibrio a largo plazo tiene
madoras de precios, movilidad perfecta de los recursos e infor- otros dos atractivos: 1) la producción se genera con el menor
mación perfecta de compradores y empresas. En este sentido, costo unitario posible y 2) al vendedor se le paga nada más el
es semejante al modelo físico del movimiento en superficies sin costo de elaborar el producto. No se obtienen utilidades econó-
fricción. Los dos modelos describen condiciones ideales que micas del comprador.
rara vez o nunca se encuentran en la práctica y, con todo, ge- • En situación de competencia perfecta, con precios de insumos
neran predicciones y explicaciones útiles de los sucesos que se constantes, la curva de oferta de la industria a largo plazo es una
observan en el mundo. línea horizontal, no sólo cuando las curvas de costo promedio
• La regla de maximización de utilidades a corto plazo consiste de corto plazo son horizontales, sino también cuando adoptan
en generar el nivel de producción para el cual el precio es igual la forma de U. Cuando los precios de los insumos crecen en fun-
al costo marginal a corto plazo sobre la porción ascendente de ción de la producción industrial, las curvas de oferta de la indus-
dicha curva. Si el precio baja a menos del valor mínimo del cos- tria tienen una pendiente positiva en ambos casos. Cuando los
to promedio variable, lo mejor para la empresa es no producir precios de los insumos bajan con la producción de la industria,
a corto plazo. La curva de oferta a corto plazo de la empresa in- la curva de oferta de una industria competitiva tiene una pen-
dividual es por lo tanto el lado ascendente de su curva de costo diente negativa.
marginal a corto plazo que está a la derecha del punto mínimo • El efecto de la competencia por la compra de insumos de calidad
de su curva de costo variable promedio. inusitadamente alta es elevar el precio de dichos insumos hasta
• La curva de oferta de la industria a corto plazo es la suma ho- que la empresa que los utiliza ya no obtiene un beneficio eco-
rizontal de las curvas de oferta de las empresas individuales. nómico. Ésta es una parte en extremo importante de los ajustes
Interseca la curva de demanda de la industria y determina el a largo plazo y no tenerla en cuenta es la causa del fracaso de
precio de equilibrio a corto plazo. La curva de demanda de una muchas políticas económicas bien intencionadas.
empresa competitiva es una línea horizontal en el precio de • Aun las empresas que son tomadoras de precios tienen que
equilibrio. Si resulta que el precio está sobre el valor mínimo de buscar de manera activa los medios de abatir sus costos de hacer
la curva de costos promedio a largo plazo, todas las empresas negocios. Quienes adoptan primero las innovaciones que aho-
ganarán una utilidad económica positiva. Si el precio es menor rran costos reciben un caudal temporal de utilidades; quienes las
que ese valor, todas sufrirán pérdidas. adoptan tarde deben atravesar periodos de pérdidas económicas.

11_CHAPTER 11.indd 366 3/7/09 9:38:48 AM


problemas 367

pREgUNtaS dE REpaSo
1. ¿Cuál es la diferencia entre utilidades económicas y utilida- 8. Verdadero o falso: en una industria de costos constantes, el im-
des contables y cómo influye en las decisiones comerciales puesto de una cantidad fija, constante, aplicada a cada unidad
actuales? de producto vendido no influirá en la cantidad de producto
2. ¿En qué condiciones se espera que se comporten las empresas que vende a largo plazo una empresa perfectamente competi-
como tomadoras de precios aun cuando sólo haya una canti- tiva. Explique.
dad reducida de otras empresas en la industria? 9. Suponga que todas las empresas de una industria competitiva
3. ¿El mercado de la lavandería en seco sería perfectamente com- operan en niveles de producción cuyo precio es igual al costo
petitiva en ciudades como San Francisco o Nueva York? ¿Por marginal a largo plazo. Verdadero o falso: esta industria se en-
qué? ¿Que hay de una ciudad pequeña como Athens, Ohio o cuentra en un equilibrio a largo plazo.
Meredith, New Hampshire? 10. Verdadero o falso: el excedente del consumidor es el área entre
la curva de demanda y el precio. En el caso de una empresa
4. La curva de ingresos totales de una empresa se obtiene con
perfectamente competitiva, la curva de demanda es igual a
IT = aQ – 2Q2. ¿Es una empresa perfectamente competitiva?
la línea del precio. Por lo tanto, una industria perfectamente
Explique por qué.
competitiva no produce un excedente del consumidor.
5. ¿El hecho de que un gerente comercial no sepa la definición de
11. ¿Por qué se dice que las economías y deseconomías pecunia-
costo marginal contradice la teoría de la competencia perfecta?
rias son la excepción y no la regla?
6. Verdadero o falso: si el costo marginal está por debajo del costo 12. ¿Esperaría que una empresa que adopta innovaciones ahorra-
fijo promedio, la empresa debe cerrar a corto plazo. Explique. doras de costos más rápidamente que 80% de las empresas de
7. ¿A qué se refieren los economistas cuando dicen que el equili- su industria obtenga utilidades económicas? De ser así, ¿estas
brio competitivo a corto plazo es eficiente? ganancias tenderán a desaparecer?

pRoblEMaS
1. En la tabla siguiente se describe la estructura de costos de una empresa competitiva. Grafique las
curvas de costo marginal, costo variable promedio y costo total promedio. ¿Cuántas unidades de
producto generará a un precio de mercado de 32? Calcule las ganancias y preséntelas en su gráfica.

Q CTP CVP CM
1 44 4 8
2 28 8 16
4 26 16 32
6 31 24 48
8 37 32 64

2. Si las curvas de costo variable promedio y marginal a corto plazo de una empresa competitiva se
obtienen con CMC = 2 + 4Q y CVP = 2 + 2Q, ¿cuántas unidades de producto generará a un precio
de mercado de 0? ¿En qué nivel de costo fijo tendrá esta empresa utilidades económicas de cero?
3. Cada 1 000 empresas idénticas de la industria competitiva de la mantequilla de cacahuate tiene una
curva de costo marginal a corto plazo que se obtiene mediante

CMC = 4 + Q.

Si la curva de demanda de esta industria es

2Q
P = 10 − ,
1000
¿cuál será la pérdida a corto plazo de los excedentes del productor y del consumidor si de pronto un
brote de aflatoxina impide la producción de mantequilla de cacahuate?

11_CHAPTER 11.indd 367 3/7/09 9:38:49 AM


368 Capítulo 11 CompetenCia perfeCta

4. Si persiste el brote de aflatoxina del problema 3, ¿la pérdida a largo plazo en los excedentes del pro-
ductor y el consumidor será mayor, menor o igual a la pérdida a corto plazo?
5. Una empresa perfectamente competitiva se enfrenta a un precio de 10 y actualmente tiene un nivel
de producción en el que el costo marginal es igual a 10 en una porción ascendente de su curva de
costo marginal a corto plazo. Su costo marginal a largo plazo es igual a 12. Su costo variable prome-
dio a corto plazo es igual a 8. El punto mínimo en su curva de costo promedio a largo plazo es igual
a 10. ¿La empresa obtiene utilidades económicas a corto plazo? ¿Debe alterar su producción a corto
plazo? ¿Qué debe hacer la empresa a largo plazo?
6. Todas las empresas de una industria competitiva tienen curvas de costo total a largo plazo que se
obtiene mediante
CTLQ = Q3 – 10Q2 + 36Q,
donde Q es el nivel de producción de la empresa. ¿Cuál será el precio de equilibrio a largo plazo de
la industria? (Sugerencia: encuentre el valor mínimo de la curva de costo promedio a largo plazo
asociada mediante el cálculo o una gráfica.) ¿Cuál será el nivel de producción de equilibrio a largo
plazo de la empresa representativa?
7. Igual que el problema 6, excepto que ahora
CTLQ = Q2 + 4Q.
¿Una empresa realmente podría tener esta curva CTL en particular? ¿Por qué?
8. Las curvas de costo promedio y marginal de los taxis en Metrópolis son constantes en 0.20 dls./mi-
lla. La curva de demanda para viajes en taxi en Metrópolis se obtiene mediante P = 1 – 0.00001Q,
donde P es la tarifa en dólares por milla y Q se mide en millas al año. Si la industria es perfectamente
competitiva y cada taxi puede proporcionar exactamente 10 000 millas/año de servicio, ¿cuántos
taxis habrá en equilibrio y cuál será la tarifa de equilibrio?
9. Ahora suponga que el ayuntamiento de Metrópolis decide desviar el congestionamiento en la zona
centro limitando la cantidad de taxis a 6. Los solicitantes participan en un sorteo y los seis ganadores
obtienen un medallón, que es una licencia permanente para operar un taxi en Metrópolis. ¿Ahora
cuál será la tarifa de equilibrio? ¿A cuánto ascenderán las utilidades económicas de cada poseedor
de un medallón? Si se pueden negociar los medallones en el mercado y la tasa de interés es de 10%
anual, ¿en cuánto se venderán? (Sugerencia: ¿cuánto tendría que depositar en un banco para ganar
un interés anual equivalente a la ganancia de un medallón de taxi?) ¿La persona que compra un
medallón a este precio obtiene utilidades económicas?
10. Merlín es como cualquier otro administrador de una industria perfectamente competitiva, excepto
en un aspecto: debido a su gran sentido del humor, la gente está dispuesta a trabajar con él por la
mitad del salario actual. Las curvas de costo total a corto plazo de las demás empresas de la industria
se obtienen mediante
CTCQ = M + 10Q + wQ2 (vea la nota 16 a pie de página),
donde M es el salario que reciben los administradores normales y w es el salario actual de la indus-
tria. Si las empresas de la industria se enfrentan a un precio de producción de 28, y si w = 2, ¿cuánto
más ganará Merlín que los demás administradores de la industria?
11. Usted es dueño y administrador de una pequeña empresa competitiva que fabrica pinturas para
casas. Usted y el total de sus 1 000 competidores tienen curvas de costo total dadas por
CT = 8 + 2Q + 2Q2
y la industria se encuentra en equilibrio a largo plazo.
Un inventor se acerca a usted con una patente de un proceso que reducirá sus costos a la mitad
en cada nivel de producción.
a) ¿Cuál es la cantidad máxima que está dispuesto a pagar por el derecho exclusivo de usar este
invento?
b) ¿El inventor estaría dispuesto a vender a ese precio?

16
La curva de costo marginal asociada es dCTCQ/dQ = CMQ = 10 + 2wQ.

11_CHAPTER 11.indd 368 3/7/09 9:38:50 AM


respuestas a los ejerCiCios del Capítulo 369

12. A corto plazo, una empresa perfectamente competitiva genera productos usando servicios de capital
(un factor fijo) y servicios de mano de obra (un factor variable). En su nivel de producción de maxi-
mización de las ganancias, el producto marginal de la mano de obra es igual al producto promedio
de la mano de obra.
a) ¿Cuál es la relación entre el costo variable promedio de la empresa y su costo marginal? Explique.
b) Si la empresa tiene 10 unidades de capital y el precio de alquiler de cada unidad es de 4 dólares/
día, ¿cuál será la ganancia de la empresa? ¿Debe permanecer abierta a corto plazo?
13. Una empresa de una industria competitiva tiene una función de costo total de CT = 0.2Q2 – 5Q + 30,
cuya curva de costo marginal correspondiente es CM = 0.4Q – 5. Si la empresa enfrenta un precio de 6,
¿qué cantidad debe vender? ¿Qué ganancia obtiene la empresa a este precio? ¿La empresa debe cerrar?
14. La demanda de gasolina es P = 5 – 0.002Q y la oferta es P = 0.2 + 0.004Q, donde P está en dólares y Q
en galones. Si se aplica un impuesto de 1 dólar/galón de gasolina, ¿cuál es la incidencia fiscal? ¿Cuál
es el excedente del consumidor que se pierde? ¿Cuál es el excedente del productor que se pierde?
15. Suponga que una industria de costos constantes perfectamente competitiva produce bicicletas. ¿Cuál
de los siguientes eventos influirá más en el precio a largo plazo de las bicicletas: 1) un programa gu-
bernamental para anunciar los beneficios que aporta a la salud andar en bicicleta o 2) un programa
gubernamental que aumente la demanda de acero, un insumo en la manufactura de bicicletas que se
produce en una industria de costos crecientes?
16. Suponga que una empresa representativa perfectamente competitiva en una industria de costos
constantes tiene una función de costo CT = 4Q2 + 100Q + 100.
a) ¿Cuál es el precio de equilibrio a largo plazo de esta industria?
b) Si la demanda del mercado se obtiene con la función Q = 1 000 – P, donde P es el precio, ¿cuántas
empresas operarán en este equilibrio a largo plazo?
c) Suponga que el gobierno subsidia a cada empresa que fabrica el producto. Si el subsidio total
equivale a 36, ¿cuál sería el nuevo precio de equilibrio a largo plazo de la industria?
17. Las curvas de oferta y demanda nacionales del grano de café Jolt están dadas por P = 10 + Q y
P = 100 – 2Q, respectivamente, donde P es el precio en dólares por bushel y Q es la cantidad en
millones de bushels por año. Estados Unidos produce y consume sólo una fracción insignificante de
la producción de café Jolt y los acontecimientos en el mercado de ese país no afectan al precio actual
mundial de 30 dólares/bushel. Los costos de transporte tampoco influyen.
a) ¿Cuánto pagarán los consumidores estadounidenses por el café Jolt y cuántos bushels consumi-
rán al año?
b) ¿Cómo se modificarán sus respuestas al inciso a) si el Congreso aprueba un arancel de 20 dóla-
res/bushel?
c) ¿Cuál será el efecto total del arancel en el excedente del productor y consumidor internos? ¿A
cuánto ascienden los ingresos derivados del arancel?
18. Un investigador australiano descubrió un fármaco que debilita las fibras de lana de borrego justo
encima de la piel del animal. El fármaco disminuye de manera drástica el costo de trasquilar (cortar
la lana) los borregos porque se retira con facilidad la capa completa. El mercado mundial de la lana
está bastante cerca del modelo de la competencia perfecta en ambos lados, el producto y el agente de
venta. Trace los efectos de la introducción de este nuevo fármaco.

RESpUEStaS a loS EjERcicioS dEl capítUlo


11.1. Sea r* la tasa de interés mensual a la que las utilidades económicas de Cullen serían nulas. Luego, r*
debe satisfacer 16 000 dólares – 4 000 dólares – 800 dólares – r* (100 000 000 de dólares) = 0, lo cual
produce r* = 0.000112 o 0.0112%/mes. Cullen se debe reubicar sólo si la tasa de interés es inferior a r*.
11.2. El costo marginal es la pendiente de la curva de costo total y el ingreso marginal es la pendiente de
la curva de ingreso total. En el punto de ganancia máxima, Q = 7.4, las pendientes de estas curvas
son exactamente iguales.

11_CHAPTER 11.indd 369 3/7/09 9:38:50 AM


370 Capítulo 11 CompetenCia perfeCta

11.3. Primero, es necesario expresar la curva de demanda de la empresa representativa P = 20 + 90Qi


para que la cantidad quede en función del precio.
2 1
Qi = − + P .
9 90
Luego, se multiplica por el número de empresas, n = 30.

 2 1  20 1
Q = nQi = 30Qi = 30− + P  =− + P .
 9 90  3 3

Por último, se expresa la curva de oferta de la industria Q =− 203 + 13 P para que el precio quede en
función de la cantidad, P = 20 + 3Q, para regresar a la forma pendiente-ordenada al origen.
11.4. La maximización de ganancias a corto plazo de una empresa perfectamente competitiva ocurre en
la cantidad donde el precio es igual al costo marginal, P = CM, siempre que P > CVP mín. (de lo
contrario, la empresa cierra). Como el costo marginal es CM = 2Q, el precio del mercado P = 12
es igual al costo marginal 12 = 2Q en la cantidad Q = 6. Observe que en este caso el CVP mín. = 0.
Es posible expresar las ganancias (con costos fijos separados) como π = (P – CVP)Q – CF. Como el
costo variable promedio es CVP = Q = 6, la empresa obtendría ganancias de
π = (12 – 6)6 – CF = 36 – CF.
Por lo tanto, con un costo fijo de CF = 36, las ganancias de la empresa serían nulas.

P
CM

12 P

CVP
6

Q
0 6

11.5. El excedente total es igual a la suma de los dos triángulos sombreados abajo, es decir, 100 000 dóla-
res/año.

P
CS = 1 (30 − 20) 10 000 = 50 000
2
30 S
PS = 1 (20 − 10) 10 000 = 50 000
2
20

10 D
Q
10 000

11.6. Si la empresa sólo paga a un administrador incompetente 30 000 dólares, sus utilidades económicas
seguirán siendo nulas. No puede pagar más sin sufrir una pérdida económica.

11_CHAPTER 11.indd 370 3/7/09 9:38:54 AM


Capítulo

12
MONOPOLIO

asi en todos los cines cobran diferentes precios de entrada a los cinéfilos de diversos grupos.
C Los estudiantes pagan un precio, los adultos otro y los adultos mayores otro. En algunos
cines ofrecen “abonos” con varios boletos más baratos que los que se venden en taquilla.
Además, quienes van a la hora de la comida muchas veces pagan menos que quienes van
al atardecer. No se esperaría ninguna de estas prácticas en el modelo de competencia perfecta, el
cual establece que todos los compradores paguen un precio único por un producto completamente
estandarizado (la llamada ley del precio único).
Los mismos operadores de cines que cobran diversos precios a grupos diferentes siguen una
práctica muy distinta cuando se trata de la venta de artículos concesionados; en este caso casi siem-
pre prevalece la ley del precio único. Estudiantes, adultos, personas mayores, beisbolistas de las ligas
mayores, miembros del clero, empleados de gasolinera y otros clientes pagan exactamente el mismo
precio por sus palomitas. La misma observación se aplica a los precios de refrescos y dulces. Pero
dichos precios son mucho mayores que los que se piden por los mismos artículos en las tiendas y
otros locales comerciales, mucho más altos que cualquier medida razonable del costo marginal de
tenerlos a la venta.
Los dos comportamientos: cobrar precios de entrada diferentes, por un lado, y por el otro tener
precios similares y elevados para las concesiones son, como se verá, del todo congruentes con lo que
pronostica el modelo económico sobre el vendedor único de un bien o servicio.

VISIÓN PRELIMINAR DEL CAPÍTULO


En este capítulo el cometido será examinar la estructura de mercado que menos se parece a la com-
petencia perfecta, a saber, el monopolio, un mercado atendido por un único vendedor de un producto
que no tiene sustitutos cercanos. Se van a exponer cinco factores que llevan a esta estructura de

371

12_CHAPTER 12.indd 371 3/6/09 8:03:22 PM


372 Capítulo 12 monopolio

mercado: 1) control de los insumos clave, 2) economías de escala, 3) patentes, 4) economías de red y
5) licencias gubernamentales. A continuación se verá que la regla del monopolista para maximizar
las utilidades en el corto plazo es la misma que aplican las empresas perfectamente competitivas. El
monopolista amplía la producción si el aumento de los ingresos supera el incremento de los costos y
la contrae si la pérdida de ingresos es menor que la reducción de los costos. Después se examinará el
comportamiento del monopolista cuando se confronta con las opciones de vender en varios merca-
dos. Otra vez, la lógica del análisis de costos y beneficios dará un marco conveniente para examinar
la decisión de la empresa sobre si modifica su funcionamiento actual.
El siguiente paso será examinar las propiedades de eficiencia del equilibrio del monopolio es-
tándar. A diferencia del caso de la competencia perfecta, el equilibrio del monopolio no agota las
ganancias potenciales del intercambio. En general, el valor para la sociedad de una unidad adicional
de producto supera el costo que tienen para el monopolista los recursos necesarios para elaborarlo.
Este resultado se interpreta en el sentido de que el monopolio es menos eficiente que la competencia
perfecta; pero es una interpretación de poca importancia práctica, porque las condiciones que dan
lugar al monopolio (en particular, las economías de escala en la producción) casi nunca son compati-
bles con las que pide la competencia perfecta.
El enfoque político del capítulo se centra en las cuestiones de cómo debe tratar un gobierno a
los monopolios naturales: mercados caracterizados por curvas de costo promedio a largo plazo con
pendiente negativa. Se considerarán cinco políticas públicas alternativas: 1) empresas paraestatales,
2) empresas privadas con regulación oficial de los precios, 3) licitaciones competitivas de parte de
empresas privadas para tener el derecho exclusivo de prestar un servicio, 4) aplicación enérgica de le-
yes antimonopolio dirigidas a evitarlos y 5) una política liberal completa. Todas las alternativas tienen
problemas, y la mejor política será diferente en distintas circunstancias.

DEFINICIÓN DE MONOPOLIO
Un monopolio es una estructura de mercado en la que un único vendedor de un producto sin susti-
tutos cercanos atiende a todo ese mercado. Esta definición no podría ser más simple; sin embargo, es
extremadamente difícil de aplicar en la práctica. Considere el ejemplo de los cines con el que empezó
el capítulo. ¿Un cine local es un monopolio de acuerdo con nuestra definición? Por lo menos en
ciudades pequeñas, es probable que sea el único que proyecte determinada película en un momento
dado. Como es obvio, que sea un monopolio depende de lo que se entienda por sustituto cercano.
Por ejemplo, si el cine pasa actualmente Halloween parte 8, es probable que haya muchos sustitutos
cercanos de este producto. En efecto, todos los años se estrenan cientos de películas baratas, san-
grientas y violentas, y los aficionados a este género no tienen que buscar muy lejos si se sienten
insatisfechos con las películas que se ofrecen en un cine en particular.
¿Pero qué pasa con un cine que está en pleno estreno de la última película de El hombre araña
durante una temporada de seis meses de exhibición exclusiva? Para los fanáticos de la serie no hay un
sustituto parecido. Los que quieren ver el estreno sólo dispondrán de un proveedor.
La característica principal que distingue a un monopolio de una empresa competitiva es la elas-
ticidad-precio de la demanda que tiene la compañía. Recuerde que para una empresa perfectamente
competitiva, la elasticidad-precio es infinita. Si una empresa competitiva eleva un poco su precio,
perderá todas sus ventas. En cambio, un monopolio tiene un control importante sobre el precio que
cobra.
Empíricamente, una medida práctica para decidir si una empresa tiene un amplio poder mono-
pólico es examinar la elasticidad precio cruzada de la demanda para sus sustitutos más próximos. En
un famoso caso antimonopolio, DuPont Corporation fue acusada de tener el privilegio de la venta
de celofán. Aunque la compañía vendía más de 80% de todo el celofán comerciado, pudo defenderse
de la acusación argumentando que las elasticidades precio cruzadas entre el celofán y sus sustitutos
cercanos (en ese entonces, principalmente papel encerado y hoja de aluminio) eran bastante eleva-
das como para justificar que se agruparan en un mercado único todos estos productos para envolver.

12_CHAPTER 12.indd 372 3/6/09 8:03:22 PM


CinCo Causas de los monopolios 373

Con esta definición amplia del mercado, DuPont vendía menos de 20% de la producción total de la
industria. En un fallo polémico, el tribunal juzgó que era tan poco que permitía una competencia
eficiente.
Sin embargo, esto no es decir que la elasticidad precio cruzada provea una medida clara y directa
que distinga a un producto con sustitutos cercanos de otro que no los tiene. Aunque no haya nada
muy parecido a la última película de El hombre araña, siempre hay muchas alternativas para entrete-
nerse 2 horas. Para la persona que desea ver El hombre araña, el cine es un monopolista, pero para al-
guien que sólo quiere ver una buena película, el mismo cine enfrenta una dura competencia. Muchas
veces, la diferencia entre la competencia perfecta y el monopolio se reduce a la pregunta sobre cuál
de los dos grupos de consumidores es más numeroso. Como en tantos casos de la economía, la tarea
de distinguir entre competencia y monopolio es tanto un arte como una ciencia.
En ambos casos, observe que la distinción entre competencia y monopolio no estriba en nin-
guna diferencia entre la elasticidad-precio de sus respectivas curvas de demanda del mercado. Por el
contrario, la elasticidad-precio de la demanda del mercado con respecto a los productos abastecidos
por empresas competitivas es mucho menor que la elasticidad-precio de la demanda que enfrenta
un monopolista. La elasticidad-precio de la demanda es menor para el trigo que para las cámaras
Polaroid, aunque el grano se produce en condiciones de competencia casi perfecta, mientras que las
patentes de Polaroid sólo las tiene el único vendedor legal en la mayoría de sus mercados. La distin-
ción importante entre monopolio y competencia es que la curva de demanda a la que se enfrenta una empresa
competitiva es horizontal (cualquiera que sea la elasticidad-precio de la correspondiente curva de demanda del
mercado), mientras la curva de demanda del monopolista es la curva de demanda con pendiente negativa de
todo el mercado.

CINCO CAUSAS DE LOS MONOPOLIOS


¿Cómo logra una empresa ser la única en abastecer a su mercado? Los economistas estudian cinco
factores que solos o combinados permiten que una empresa se convierta en un monopolio. Vamos a
considerarlos en orden.
1. Control exclusivo de insumos importantes La corporación Perrier de Francia vende agua
mineral embotellada. Cada año gasta millones de dólares en divulgar las propiedades exclusivas de
su agua, que son el resultado —dice— de una confluencia irrepetible de factores geológicos que
crearon su manantial mineral. En el estado de Nueva York, la compañía Adirondack Soft Drink
ofrece un producto que es, en esencia, agua embotellada con anhídrido carbónico. Yo no distingo
entre la Adirondack Seltzer y la Perrier, pero a otros les sabe diferente y para muchos no hay un
sustituto satisfactorio de Perrier. La posición de monopolio de la compañía francesa con respecto a
estos compradores es el resultado de su control exclusivo sobre un insumo que no se puede repetir
con facilidad.
Hay una posición de monopolio parecida en el control exclusivo de deBeers Diamond Mines
sobre la mayor parte de la oferta mundial de diamantes. La calidad de los diamantes sintéticos ha
mejorado en la actualidad hasta el punto de que ocasionalmente han engañado incluso a joyeros ex-
perimentados. Pero para muchos compradores, la preferencia por una piedra extraída de la tierra no
es mera cuestión de mayor dureza y brillo. Quieren diamantes verdaderos y deBeers es la compañía
que los tiene.
El control exclusivo de insumos clave no es una garantía de un poder monopólico permanente.
Por ejemplo, la preferencia por tener diamantes verdaderos se basa sobre todo en el hecho de que
han sido en realidad superiores a los sintéticos. Pero si se supone que los diamantes sintéticos a la
larga se volvieran indistinguibles por completo de los naturales, ya no habría bases para esta prefe-
rencia. Como resultado, el control de deBeers sobre la oferta de diamantes genuinos dejará de con-
ferirle un poder monopólico. Continuamente se idean nuevos métodos para elaborar los productos
actuales y es probable que el insumo exclusivo que genera al monopolio de hoy se vuelva obsoleto
mañana.

12_CHAPTER 12.indd 373 3/6/09 8:03:23 PM


374 Capítulo 12 monopolio

2. Economías de escala Cuando la curva de costo promedio a largo plazo (con precios fijos de
los insumos) tiene pendiente negativa, la forma más barata de atender el mercado es concentrar la
producción en las manos de una sola empresa. Por ejemplo, observe en la figura 12.1 que una única
empresa puede generar una producción industrial de Q* a un costo promedio de CPLQ*; en cambio,
con dos empresas que compartan el mismo mercado el costo promedio se eleva a CPLQ*/2. Un mer-
cado que se atiende a menor costo con una sola empresa es un monopolio natural. Un ejemplo muy
citado es el suministro de líneas terrestres para telefonía local.

FIGURA 12.1
Monopolio natural
Cuando la curva Cpl $/Q
desciende en toda su
CPL
extensión, siempre es más
barato que toda la industria
sea abastecida por una sola
empresa.
CPLQ*/2

CPLQ*

Q
Q*/2 Q*

Recuerde que en el capítulo 11 se vio que es posible que la curva CPL tenga una pendiente nega-
tiva aun sin que haya economías de escala. Así ocurre, por ejemplo, si el precio de un insumo impor-
tante baja de manera significativa cuando la producción de la industria se expande (en los términos
del capítulo 11, una economía pecuniaria). Pero fíjese bien que este caso no da lugar a un monopolio
natural. Aquí, los precios de los insumos dependen del nivel de la producción industrial, no de lo pro-
ducido por alguna empresa individual. Las economías pecuniarias se cumplen con la misma fuerza al
margen de que atiendan el mercado una o muchas empresas.
Así pues, hablando en rigor, lo que determina que se tenga un monopolio natural es el grado
de los rendimientos de escala, no la pendiente de la curva CPL. Desde luego, con los precios de los
insumos fijos, siempre hay una relación uno a uno entre los rendimientos de escala y la pendiente de
la curva CPL (vea el capítulo 10).
3. Patentes La mayoría de los países protegen los inventos con algún sistema de patentes. Una
patente concede a su titular el derecho a beneficiarse de manera exclusiva de todos los intercambios
relacionados con el invento al que se aplica. Las patentes implican tanto costos como beneficios. Por
el lado de los costos, el monopolio que crean, como se verá, aumenta los precios que pagan los consu-
midores. En el lado de los beneficios, la patente posibilita muchos inventos que de otro modo se frus-
trarían. Aunque algunos inventos son fortuitos, la mayor parte es el resultado de grandes esfuerzos
y gastos en complejos laboratorios de investigación. Si una empresa no pudiera vender su producto
a un precio que recuperara estos desembolsos, hubiera faltado el motivo económico original para
emprender la investigación y el desarrollo. Sin una patente, la competencia bajaría el precio hasta
su costo marginal y el ritmo de las innovaciones se haría notablemente más lento. La protección
contra la competencia, que concede una patente, posibilita que una empresa recupere los costos de la
innovación. En Estados Unidos las patentes se extienden a 17 años, una cifra de compromiso que es
excesiva para muchos inventos y muy breve para otros tantos. En particular, se esgrime el convincen-
te argumento de que la duración de las patentes debiera extenderse en la industria farmacéutica, en la
que pruebas y trámites de aprobación consumen varios años del periodo actual de las patentes.1

1
Henry Grabowski, Drug Regulation and Innovation, Washington, DC: American Enterprise Institute, 1976.

12_CHAPTER 12.indd 374 3/6/09 8:03:24 PM


CinCo Causas de los monopolios 375

4. Economías de red En lo que respecta a la demanda de


muchos mercados, un producto es más valioso conforme lo
usan más consumidores.2 Un viejo ejemplo vívido es la de- HOLA, SOY BILL GATES. RECUERDE

©The New Yorker Collection 1998 Mick Stevens from cartoonback.com All Rights Reserved.
rrota del formato Beta para las videograbadoras domésticas QUE NADIE TIENE EL MONOPOLIO DE
por parte de la tecnología VHS. El atractivo de VHS sobre
las versiones iniciales de Beta radicaba en que permitía hacer
LA SEGURIDAD, ASÍ QUE PÓNGASE EL
grabaciones más largas. Después Beta corrigió esta deficien- CINTURÓN.
cia y los expertos consideraban que en los aspectos técnicos
más importantes era superior a VHS, pero la ventaja inicial de
ventas de este último formato resultó insuperable. Cuando la
fracción de consumidores que tenían VHS rebasó un límite
crítico, los motivos para preferirlo se hicieron irresistibles:
variedad y surtido de cintas para alquilar, centros de repara-
ción, posibilidad de intercambiar películas con los amigos,
etcétera.
En casos extremos, estas economías de red funcionan,
igual que las economías de escala, como fuente de monopo-
lios naturales. Por ejemplo, el sistema operativo Windows
de Microsoft alcanzó su posición dominante en el mercado
por la fuerza de poderosas economías de red. Como la ven-
taja inicial de ventas de Microsoft le dio a los programadores
(software) un firme incentivo para escribir en el formato de
Windows, el inventario de software para Windows es ahora
mucho más amplio que el de cualquier sistema operativo competidor. Y aunque todavía se venden
programas de uso general, como procesadores de texto y hojas de cálculo en muchos sistemas opera-
tivos, el software profesional especializado y los juegos normalmente aparecen primero en formato
Windows y sólo después en el otro sistema. Esta ventaja hace que la gente escoja Windows aun si,
como en el caso de muchos usuarios de Macintosh de Apple, creen que el sistema competidor es
mejor. El resultado último es que más de 90% de las computadoras personales del mundo llevan el
sistema operativo Windows de Microsoft. Si no es un monopolio puro, le falta poco.
5. Licencias o franquicias gubernamentales En muchos mercados la ley prohíbe que alguien
que no sea una empresa con autorización del gobierno haga un negocio. Por ejemplo, en las zonas de
descanso de la autopista Turnpike de Massachusetts no se puede instalar cualquier restaurante. Las
autoridades negocian con varias compañías, escogen una y le conceden una licencia exclusiva para
atender una zona particular. Como me gustan más las Whopper que las Big Mac, estoy contento de
que en la autopista se haya abierto un Burger King en lugar de un McDonald’s. Pero esta elección va
a decepcionar a muchos otros compradores. El propósito de las autoridades de la autopista Turnpike
al restringir el acceso es que en esos lugares no hay espacio para más de un establecimiento. En tales
casos, la licencia gubernamental como fuente de un monopolio es una economía de escala que actúa
de otra forma. Pero también se requieren licencias gubernamentales en otros mercados, como el
de los taxis, en el que las economías de escala no parecen ser un factor importante. Para elevar sus
ingresos, muchas universidades (como la Estatal de Ohio) venden derechos exclusivos para máquinas
expendedoras (por ejemplo, sólo Coca o sólo Pepsi).
A veces las licencias del gobierno están acompañadas por regulaciones estrictas que detallan lo
que puede hacer el concesionario y lo que no. Por ejemplo, si el gobierno da a una cadena de res-
taurantes una licencia de exclusividad, se le pedirá que cobre, por ejemplo, precios 10% más altos
que los que cobra en sus locales no regulados. En otros casos, el gobierno cobra una tarifa muy ele-
vada por la licencia con lo que obliga a la empresa a aplicar primas en los precios. Es lo que hacen

2
Vea, por ejemplo, Joseph Farell y Garth Saloner, “Standardization, Compatibility, and Innovation”, en Rand Journal of Economics,
16, 1985, pp. 70-83; y M. L. Katz y Carl Shapiro, “Systems Competition and Network Effects”, en Journal of Economic Perspectives,
primavera de 1994, pp. 93-115.

12_CHAPTER 12.indd 375 3/6/09 8:03:24 PM


376 Capítulo 12 monopolio

algunas autoridades aeroportuarias, que de hecho subastan al mejor postor los espacios de mostra-
dor de las terminales aéreas. Por lo tanto, ante su enojo por tener que pagar 5 dólares por un hot dog
en el aeropuerto de La Guardia debe dirigirse mejor a la autoridad aeroportuaria de Nueva York que
al proveedor.
El más importante de los cinco factores que explican los monopolios que persisten a largo plazo
son las economías de escala. Los procesos de producción cambian con el tiempo, de modo que el
control exclusivo sobre los insumos importantes es una fuente pasajera de monopolios. Las paten-
tes también son de naturaleza efímera. Las economías de red, cuando se han arraigado, pueden
ser la fuente de un monopolio natural tanto como las economías de escala. En sentido estricto, las
economías de red operan en la demanda del mercado e influyen en lo que los compradores están
dispuestos a pagar por un producto. Pero pueden concebirse igualmente en el lado de la oferta, co-
mo otra característica de la calidad del producto. Cuantas más personas tengan el producto, mayor
es el nivel de la calidad efectiva. Así, podrá decirse que un producto que se beneficia de las econo-
mías de red puede producirse con menor costo en la medida en que aumenten los volúmenes de
ventas. Visto de esta manera, las economías de red son otra forma de economías de escala en la
producción y así es como se les va a considerar en la siguiente exposición. Las licencias del gobierno
pueden alargarse durante periodos prolongados, pero muchas de ellas sólo suponen la aceptación
implícita de las economías de escala que de todos modos llevarían a un monopolio.

LA INFORMACIÓN COMO FUENTE CRECIENTE


DE ECONOMÍAS DE ESCALA
En 1984, a comienzos de la era de la computadora personal, aproximadamente 80% del costo de una
máquina correspondía al hardware y apenas 20% al software. Sólo seis años después los porcentajes
se habían invertido. Ahora, nada más que una fracción diminuta del costo total de sacar al mercado
una computadora personal se relaciona de una u otra manera con la producción de información.
Aunque esta transformación ha sido en especial notable en el caso de las computadoras personales,
también han ocurrido otras parecidas con la mayoría de los productos.
La característica distintiva de la información es que casi todos los costos que implica producirla
son fijos, a diferencia del hardware, en el que una parte importante de los costos de producción son
más o menos proporcionales al volumen. El resultado es que la elaboración de productos con mucha
información se caracteriza por enormes economías de escala.
Como el concepto de economías de escala se refiere por definición al largo plazo, la referencia
del párrafo anterior a economías de escala y costos fijos al mismo tiempo parecería incongruente.
Los costos fijos son gastos relacionados con insumos fijos y, como se vio en el capítulo 9, a la larga no
hay insumos fijos.
Sin embargo, como cuestión práctica, los costos cuantiosos suelen desembolsarse una sola vez,
como la investigación de productos y la generación de información, antes de que se lance el produc-
to. Por lo regular dichos costos no se repiten, ni siquiera en ciclos de productos que abarcan décadas.
En rigor, no son costos fijos, puesto que, en principio, los insumos que se emplean para generar la
información pueden variar; pero cuando el producto se lanza, no hay motivos para cambiarlos. Así
que, para fines prácticos, estos costos son fijos en esencia. En cualquier caso, el punto importante
es que la curva de costo promedio a largo plazo de la empresa tiene pendiente negativa cuando una
parte sustancial de sus costos totales se asocia con inversiones iniciales en información.
Ejemplos notables son el microprocesador que activa las computadoras personales y un cúmu-
lo creciente de otros productos. La inversión fija requerida para producir el último chip de Intel
fue de alrededor de 2 000 millones de dólares. Pero ya con el chip diseñado y construido el cen-
tro de manufactura, el costo marginal de producir cada chip es de pocos centavos. Por lo tanto, no
es de sorprender que Intel suministre más de 80% de los microprocesadores que se venden en la
actualidad.
Las economías de escala siempre han sido una característica importante del paisaje industrial
moderno. Pero a medida que una mayor parte del valor incorporado en los productos consista en
información, la importancia de las economías de escala no dejará de aumentar.

12_CHAPTER 12.indd 376 3/6/09 8:03:24 PM


el monopolista que maximiza utilidades 377

Con estas breves generalidades de las causas de los monopolios en mente, ahora se responderá
la pregunta de cuáles son las consecuencias de los monopolios. Para esto, se procederá de la misma
manera que en el estudio de la empresa competitiva; es decir, se examinará la decisión de producción
de la empresa y se preguntará si lleva a una situación en la que se agoten todas las posibles ganancias
derivadas del intercambio. La respuesta es que, en general, no. Pero al formular una política guber-
namental para mejorar los resultados de un monopolio sin regular, se verá que es crucial entender la
fuente original del monopolio.

EL MONOPOLISTA QUE MAXIMIZA UTILIDADES


Como en el caso competitivo, se supone que la meta del monopolista es maximizar las utilidades eco-
nómicas. E igual que antes, en el corto plazo esto significa escoger el nivel de producción en el que la
diferencia entre el ingreso total y el costo total de corto plazo sea mayor. La defensa de este motivo
es menos convincente que en el caso de la competencia perfecta: la supervivencia del monopolista
está menos asediada que la del competidor, así que el argumento evolutivo de la maximización de
utilidades se aplica con menos fuerza en el caso del monopolio. Sin embargo, se explorará qué com-
portamientos se derivan de la meta monopolista de maximizar las utilidades.

CURVA DE INGRESO TOTAL DEL MONOPOLISTA


La principal diferencia entre el monopolista y el competidor perfecto está en la manera en que varía
el ingreso total (y, con éste, el marginal) con la producción. Recuerde que en el capítulo 11 vio que la
curva de demanda para un competidor perfecto es una línea horizontal en el precio del mercado en
equilibrio a corto plazo, denominado P*. La empresa competitiva es una tomadora de precios, bási-
camente porque su producción es demasiado pequeña para tener alguna influencia discernible en el
precio del mercado. En estas circunstancias, la curva de ingreso total de la empresa perfectamente
competitiva es una diagonal con pendiente P*, como se muestra en la figura 12.2.

FIGURA 12.2
Curva de ingreso total
de un competidor
$/unidad de tiempo
IT = P*Q perfecto
el precio del competidor
perfecto se queda en el
nivel de equilibrio de corto
plazo P*, cualquiera que
sea la producción de la
empresa. así, su ingreso
total es el producto de P*
y la cantidad que vende:
Pendiente = P* it = P*Q.
Q

Ahora considere a un monopolista que tiene una curva de demanda con pendiente negativa P =
80 – ( –15 )Q representada en la sección superior de la figura 12.3. También para esta empresa el ingreso
total es el producto del precio por la cantidad. Por ejemplo, en el punto A de su curva de demanda
vende 100 unidades por semana a un precio unitario de 60 dólares, lo que da un ingreso total de 6 000
dólares/semana. En B vende 200 unidades a un precio de 40 dólares, así que su ingreso total en B será
de 8 000 dólares/semana, etc. La diferencia entre el monopolista y el competidor es que para que el
primero venda más volumen de producción, debe bajar su precio, no sólo de la unidad marginal, sino
también de todas las unidades anteriores. Como se vio en el capítulo 5, el efecto de una curva de de-

12_CHAPTER 12.indd 377 3/6/09 8:03:25 PM


378 Capítulo 12 monopolio

FIGURA 12.3
Demanda, ingreso total P ($/unidad de producción)
y elasticidad
80
para que el monopolista
aumente sus ventas, tiene
A
que reducir los precios 60
(sección superior). el
ingreso total aumenta a B
un valor máximo y luego 40
disminuye (sección central).
C
la cantidad para la cual 20
la elasticidad-precio de
la demanda es unitaria
corresponde al punto Q (unidades/sem)
0 100 200 300 400
central de la curva de
demanda y en ese valor el Ingresos totales ($/sem)
ingreso total es máximo. 8 000

6 000

4 000

2 000

Q (unidades/sem)
0 100 200 300 400
ε

0 100 200 300 400


Q (unidades/sem)
– 13
–1

–3

manda con pendiente negativa es que el ingreso total ya no es proporcional a la producción vendida.
Como en el caso competitivo, la curva de ingreso total del monopolista (sección central de la figura
12.3) pasa por el origen porque en cada caso no vender productos no genera ingresos. Pero a medida
que baja el precio, el ingreso total del monopolista no aumenta de manera lineal con la producción,
sino que llega a un valor máximo en la cantidad que corresponde al punto central de la curva de
demanda (B en la sección superior) y luego empieza a disminuir. Los valores correspondientes de la
elasticidad precio de la demanda se muestran en la sección inferior de la figura 12.3. Observe que el
ingreso total llega a su valor máximo cuando la elasticidad-precio de la demanda es unitaria.

EJERCICIO 12.1
Haga un diagrama de la curva de ingreso total de un monopolista cuya curva de demanda está
dada por P = 100 – 2Q.

En la sección superior de la figura 12.4 se representa la curva de costo total y la curva de ingreso
total a corto plazo de un monopolista que tiene la curva de demanda que se indica en la figura 12.3. La

12_CHAPTER 12.indd 378 3/6/09 8:03:26 PM


el monopolista que maximiza utilidades 379

utilidad económica, graficada en la sección inferior, es positiva en el intervalo de Q = 45 a Q = 305 y es


negativa a ambos lados del intervalo. El punto de utilidad máxima está en Q* = 175 unidades por se-
mana, que se halla a la izquierda del nivel de producción para el cual el ingreso total está en el máximo
(Q = 200).
Observe en la figura 12.4 que la distancia vertical entre la curva de costo total a corto plazo y
la curva de ingreso total es mayor cuando las dos son paralelas (cuando Q = 175). Suponga que no
fuera así. Por ejemplo, estime que en el punto de utilidad máxima la curva de costo total fuera más
inclinada que la curva de ingreso total. Sería posible ganar utilidades mayores generando menos
producción, porque los costos bajarían más que la reducción correspondiente en el ingreso total. Por
el contrario, si la curva de costo total fuera menos inclinada que la del ingreso total, el monopolista
obtendría mayores utilidades si ampliara la producción porque el ingreso total subiría mucho más
que el costo total.

FIGURA 12.4
IT, CT ($/sem) Curvas de costo, ingreso
y utilidad totales de un
8 000 monopolista
7 900 CT
la utilidad económica
[Π(Q)] en la sección
6 000 inferior es la distancia
vertical entre el ingreso
4 400 total y el costo total (it y
4 000 IT Ct en la sección superior).
observe que el punto de
2 000 máxima utilidad, Q* = 175,
está a la izquierda del nivel
de producción en el que it
Q está en un máximo
45 100 175 200 305 400 (Q = 200).
Utilidad económica ($/sem)
3 500

0 Q
45 100 175 200 305 400
–2 000

Π(Q)

INGRESO MARGINAL
Por definición, la pendiente de la curva de costo total en cualquier nivel de producción es igual al cos-
to marginal en ese nivel. En el mismo sentido, la pendiente de la curva de ingreso total es la definición
de ingreso marginal.3 Como en el caso de la empresa perfectamente competitiva se puede pensar en
el ingreso marginal como el cambio en el ingreso total cuando la venta del producto cambia por una
unidad. Con más precisión, suponga que ΔITQ es el cambio del ingreso total que ocurre en respuesta
a un pequeño cambio de la producción ΔQ. El ingreso marginal, denotado IMQ, está dado por:

∆ITQ
IM Q = . (12.1)
∆Q

3
En términos del cálculo, el ingreso marginal se define como la derivada dIT/dQ.

12_CHAPTER 12.indd 379 3/6/09 8:03:28 PM


380 Capítulo 12 monopolio

Con esta definición, un monopolista que maximiza utilidades en el corto plazo va a escoger el
nivel de producción Q* en el que

CM Q * = IM Q * (vea nota 4 a pie de página), (12.2)

siempre que el ingreso marginal interseque el costo marginal desde arriba. La ecuación 12.2 define
condición óptima para un la condición óptima para un monopolista. El monopolista quiere vender todas las unidades para
monopolista un monopolista las cuales el ingreso marginal supera el costo marginal, de modo que el primero debe estar arriba del
maximiza las utilidades segundo antes de la intersección (en algunas estructuras de costos el costo marginal primero dismi-
escogiendo el volumen de nuye y luego aumenta, lo que genera dos intersecciones del costo y el ingreso marginales).
producción en el que el
Recuerde que la condición análoga de la empresa perfectamente competitiva es escoger el nivel
ingreso marginal es igual al
costo marginal.
de producción en el que el precio y el costo marginal son iguales y que el ingreso marginal y el precio
(P) son exactamente iguales para la empresa competitiva (cuando ésta amplía su producción una
unidad, su ingreso total aumenta a P), se ve que la condición de maximización de las utilidades para
la empresa perfectamente competitiva no es más que un caso especial de la ecuación 12.2.
En el caso de la empresa monopolista, el ingreso marginal siempre será menor que el precio.5
Para ver por qué, considere la curva de demanda representada en la figura 12.5 y suponga que el
monopolista quiere aumentar la producción de Q0 = 100 a Q0 + ΔQ = 150 unidades/semana. Su
ingreso total al vender 100 unidades/semana es (60 dólares/unidad)(100 unidades/semana) =
6 000 dólares/semana. Para vender un adicional ΔQ = 50 unidades/semana, tiene que rebajar el precio a
60 – ΔP = 50 dólares por unidad, lo que significa que su nuevo ingreso total será (50 dólares/unidad)(150
unidades/semana), que es igual a 7 500 dólares/semana. Para calcular el ingreso marginal, se resta el
ingreso total original, 6 000 dólares/semana, del nuevo ingreso total y se divide entre la variación de la

FIGURA 12.5
Cambios en el ingreso
total como resultado
de una rebaja del precio
el área del rectángulo A P ($/unidad)
(1 000 dólares/semana)
es la pérdida de ingresos 80
ocasionada al vender el Pérdida de ingresos
volumen de producción por ventas al precio nuevo
anterior a un precio más 60
bajo. el área del rectángulo ΔP A
50 Ganancias de ingresos por
B (2 500 dólares/semana)
las ventas adicionales
es la ganancia de ingresos
al vender la producción
adicional al nuevo precio
más bajo. el ingreso Curva de demanda
B del mercado
marginal es la diferencia
entre estas dos áreas
(2 500 dólares – 1 000
Q (unidades/semana)
dólares = 1 500 dólares/ 100 150 400
semana) dividida entre el
cambio en la producción ΔQ
(50 unidades/semana). en
este caso, im es igual a
30 dólares/unidad, que es
menos que el nuevo precio
de 50 dólares por unidad.

4
Esta condición también puede justificarse aduciendo que la condición de primer orden para obtener utilidades máximas está dada
por:
dΠ d ( IT − CT )
= = IM − CM = 0.
dQ dQ
5
En realidad, hay una excepción a esta afirmación, a saber, el caso del monopolista perfectamente discriminador de precios, que se
analiza más adelante.

12_CHAPTER 12.indd 380 3/6/09 8:03:30 PM


el monopolista que maximiza utilidades 381

producción ΔQ = 50 unidades/semana. El resultado que se obtiene es IMQ0=100 = (7 500 dólares/sema-


na – 6 000 dólares/semana)/(50 unidades/semana) = 30 dólares/unidad, que es una cantidad infe-
rior al precio inicial de 60 dólares/unidad.
Otra manera útil de abordar el estudio del ingreso marginal es analizarlo como la ganancia
de ingresos por nuevas ventas menos las pérdidas causadas por vender el volumen de producción
anterior al nuevo precio más bajo. En la figura 12.5, el área del rectángulo B (2 500 dólares/semana)
representa la ganancia de ingresos generada por las ventas adicionales al precio menor. El área del
rectángulo A (1 000 dólares/semana) representa la pérdida de ingresos por vender las 100 unidades
semanales originales en 50 dólares en lugar de 60 dólares. El ingreso marginal es la diferencia entre
la ganancia en ingresos por las ventas adicionales y la pérdida de ingresos por vender a menor precio
dividido entre el cambio en la cantidad. Esto da como resultado (2 500 dólares/semana – 1 000 dóla-
res/semana)/(50 dólares/semana), que otra vez es igual a 30 dólares/unidad.
Para estudiar cómo varía el ingreso marginal conforme se avanza en la curva de la demanda,
considere el ejemplo que se representa en la figura 12.6 y suponga que el monopolista quiere aumen-
tar la producción de Q0 a Q0 + ΔQ unidades. Su ingreso total generado por la venta de Q0 unidades
es P0Q0. Para vender ΔQ unidades adicionales, debe reducir el precio a P0 – ΔP, lo que significa que su
nuevo ingreso total será (P0 – ΔP)(Q0 + ΔQ), que es igual a P0Q0 + P0ΔQ – ΔPQ0 – ΔPΔQ. Para calcular
el ingreso marginal se resta el ingreso total original, P0Q0, del nuevo ingreso total y se divide entre el
cambio en la producción, ΔQ. Esto deja IMQ0 = P0 – (ΔP/ΔQ)Q0 – ΔP, que es claramente menos que P0.
A medida que ΔP se acerca a cero, la expresión para el ingreso marginal se aproxima a6
ΔP
MR Q0 = P0 − Q0 . (12.3)
ΔQ

FIGURA 12.6
Ingreso marginal y
posición en la curva de
P ($/unidad) demanda
Cuando Q está a la izquierda
del punto medio (M) de una
MR > 0 curva de demanda lineal (por
ejemplo, cuando Q = Q0),
P0 las ganancias derivadas del
A aumento de las ventas (área
P0 – ΔP MR = 0
de B) compensan las pérdidas
derivadas de las ventas
M anteriores a un precio más
B MR < 0
bajo (área A). Cuando Q está
a la derecha del punto central
P1 (por ejemplo, cuando Q =
C
Q1), las ganancias producidas
D por el aumento las ventas
Q (unidades/sem) (área D) son menores que las
Q0 Q0 + ΔQ Q1
pérdidas provocadas por las
ventas anteriores a un precio
más bajo (área de C). en el
punto central de la curva de
demanda, pérdidas y ganancias
son iguales, lo cual significa
que el ingreso marginal es
cero.

La ecuación 12.3 tiene sentido intuitivo si se piensa en ΔQ como un cambio de una unidad en
la producción; P0 sería entonces la ganancia en el ingreso generada por la venta de la unidad extra, y

6
Observe que cuando ΔP disminuye hacia cero, la ΔQ correspondiente también lo hace. Como ΔP y ΔQ son positivas en este caso, el
cociente ΔP/ΔQ es el negativo de la pendiente de la curva de demanda.

12_CHAPTER 12.indd 381 3/6/09 8:03:32 PM


382 Capítulo 12 monopolio

(ΔP/ΔQ)Q0 = ΔPQ0 sería la pérdida de ingresos por la venta de las unidades que ya se tienen al precio
más bajo. Se ve de nuevo en la ecuación 12.3 que el ingreso marginal es menor que el precio para
todos los niveles positivos de producción.
El hecho de que el área de B sea mayor que A en la figura 12.6 significa que el ingreso marginal es
positivo en Q0. Sin embargo, una vez que la producción rebasa el punto central (M en la figura 12.6)
en una curva de demanda en forma de línea recta, el ingreso marginal de un nuevo aumento será
negativo. Así, el área del rectángulo C es mayor que el área del rectángulo D en la figura 12.6, lo que
significa que el ingreso marginal en el nivel de producción Q1 es menor que cero.

INGRESO MARGINAL Y ELASTICIDAD


Existe otra relación útil que vincula el ingreso marginal con la elasticidad-precio de la demanda en el
punto correspondiente de la curva de demanda. Recuerde que en el capítulo 5 vio que la elasticidad-
precio de la demanda en un punto (Q, P) está dada por
ΔQ P
ε= = . (12.4)
ΔP Q

En la ecuación 12.4, los términos ΔQ y ΔP tienen signos opuestos, porque la curva de demanda
tiene pendiente negativa. En contraste, recuerde que los términos ΔQ y ΔP de la ecuación 12.3, que
también representan cambios en P y Q conforme se avanza en la curva de demanda, son ambos
positivos. Suponga que se vuelven a definir ΔQ y ΔP a partir de la ecuación 12.4 de manera que los dos
términos sean positivos. La ecuación se convierte en
ΔQ P
ε= . (12.5)
ΔP Q

El propósito de hacer positivos los términos ΔQ y ΔP es poder relacionar la ecuación 12.5 con la
12.3. Si ahora se despeja de la ecuación 12.5 ΔP/ΔQ = P/(Q|ε|) y se sustituye en la ecuación 12.3, se
tiene

 1
IM Q = P  1 −  . (12.6)
 ε

La ecuación 12.6 indica que cuanto menos elástica sea la demanda con respecto al precio, tanto
más elevado será el precio en relación con el ingreso marginal.7 También señala que en el caso límite
de una elasticidad-precio infinita, el ingreso marginal y el precio son exactamente iguales (recuerde
que en el capítulo 11 se vio que el precio y el ingreso marginal eran iguales para la empresa competi-
tiva, que se enfrenta a una curva de demanda horizontal, o sea, infinitamente elástica).

GRÁFICA DEL INGRESO MARGINAL


La ecuación 12.6 permite representar fácilmente los valores del ingreso marginal que corresponden
a diferentes puntos sobre la curva de demanda. Para ejemplificarlo, considere la curva de demanda
lineal de la figura 12.7 que interseca el eje vertical para un precio P = 80. La elasticidad de la deman-
da es infinita en ese punto, lo cual significa que IM0 = 80(1 – 1/ε) = 80. Aunque el ingreso marginal
será en general menor que el precio para un monopolista, los dos son exactamente iguales cuando
la cantidad es cero. La explicación es que con una producción nula no hay ventas que afecten a la
reducción de precios.

7
La ecuación 12.6 puede derivarse mediante el cálculo como sigue:

IM =
dTR
=
( ) = P + Q dP = P  1 + QdP  = P  1 + 1  = P  1 − 1  .
d PQ
 
PdQ 
 
ε
 
dQ dQ dQ  ε 

12_CHAPTER 12.indd 382 3/6/09 8:03:36 PM


el monopolista que maximiza utilidades 383

FIGURA 12.7
La curva de demanda
P ($/unidad de producción)
y la curva de
80 ingreso marginal
correspondiente
∈ =3 para el caso de una curva
60 de demanda lineal, la
A
∈ =1 curva del ingreso marginal
40 correspondiente también
B es recta. tiene la misma
∈ = 13 intersección con el eje
20 Demanda
C vertical que la curva de
demanda y su intersección
0 Q (unidades/sem) horizontal es la mitad que
100 200 300 400 la de la curva de demanda.

–40 Ingreso marginal

Ahora suponga que se mueve, por ejemplo, un cuarto hacia abajo por la curva de demanda
hasta el punto A (100, 60). En ese punto, |ε| = 3. Así, se tiene IM100 = (60)(1 – –13 ) = 40.
Más hacia abajo, a la mitad de la curva de demanda, en el punto B (200, 40),  ε  = 1, lo que
da IM200 = (40)(1 – –11 ) = 0. Esto confirma el resultado del capítulo 5, según el cual el ingreso total
alcanza su máximo en el punto central de una curva de demanda en forma de línea recta en la cual la
elasticidad es unitaria.
Por último considere el punto C (300, 20), que está a tres cuartas partes hacia abajo sobre la
curva de demanda. En este caso |ε| = –13 , así que se tiene IM300 = (20)[1 – (1/–13 )] = (20) (–2) = –40. Así,
en Q = 300, el efecto de vender otra unidad de producto es reducir el ingreso total en 40 dólares por
semana.
Si se sigue este procedimiento, se observa con rapidez que la curva del ingreso marginal asocia-
da con una curva de demanda lineal es también una recta y que tiene una pendiente dos veces mayor
que la de la curva de demanda. La curva del ingreso marginal corta el eje de las abscisas justo debajo
del punto central de la curva de demanda, y es negativa para todas las cantidades mayores que el in-
greso marginal. Observe que todos los puntos a la derecha del centro de la curva de demanda tienen
valores absolutos de elasticidad menores que 1. El hecho de que el ingreso marginal sea negativo
en esta región concuerda con la observación del capítulo 5 de que una rebaja del precio reduce el
ingreso total cuando la demanda es inelástica con respecto al precio.

Obtenga la curva de ingreso marginal que corresponde a la curva de demanda P = 12 – 3Q. EJEMPLO 12.1
La curva del ingreso marginal tendrá la misma ordenada en el origen y una pendiente dos veces
mayor que la curva de demanda, lo cual da IM = 12 – 6Q, tal como se representa en la figura 12.8.
La fórmula general para la curva de demanda lineal es P = a – bQ, donde a y b son números po-
sitivos. La curva de ingreso marginal correspondiente será IM = a – 2bQ (vea la nota a pie de página
número 8).

8
Observe que el ingreso total de la curva de demanda P = a – bQ está dada por IT = aQ – bQ2. La curva correspondiente del ingreso
marginal es
dIT
IM = = a − 2bQ.
dQ

12_CHAPTER 12.indd 383 3/6/09 8:03:37 PM


384 Capítulo 12 monopolio

FIGURA 12.8
Curva de demanda $/Q
lineal y curva del 12
ingreso marginal
correspondiente
la curva de ingreso
marginal tiene la misma
ordenada en el origen y
dos veces la pendiente de
la curva de demanda lineal
correspondiente. P = 12 – 3Q

IM = 12 – 6Q
Q
0 2 4

EJERCICIO 12.2
Trace las curvas de demanda y de ingreso marginal de un monopolista con una curva de de-
manda del mercado dada por P = 100 – 2Q.

INTERPRETACIÓN GRÁFICA DE LA CONDICIÓN DE MAXIMIZACIÓN


DE UTILIDADES A CORTO PLAZO
Recuerde que en el capítulo 11 se vio la representación gráfica del punto de máximas utilidades a
corto plazo para una empresa competitiva. De manera análoga, hay una representación gráfica para
el monopolista. Considere un monopolista que tiene las curvas de demanda, ingreso marginal y cos-
to a corto plazo que se ilustran en la figura 12.9. El nivel de producción que maximiza las utilidades
para esta empresa es Q*, en el que se intersecan las curvas del ingreso y el costo marginales. Para esta
cantidad, el monopolista puede cobrar un precio P* y ganar con ello utilidades económicas iguales al
rectángulo sombreado marcado Π.

FIGURA 12.9
Precio y cantidad $/Q
de maximización de
utilidades para un
monopolista
las mayores utilidades
ocurren en un nivel de CMC CTP
P*
producción Q*, en el
cual el aumento de los
ingresos por incrementar Π
la producción (o la pérdida CTPQ*
de ingresos por contraerla),
im, es exactamente igual
al costo de aumentar la
producción (o a los ahorros D
de contraerla), CmC. en Q*, IM
Q
la empresa cobra P* y gana Q*
unas utilidades económicas
de Π.

12_CHAPTER 12.indd 384 3/6/09 8:03:39 PM


el monopolista que maximiza utilidades 385

Un monopolista tiene una curva de demanda dada por P = 100 – 2Q y una curva de costo EJEMPLO 12.2
total a corto plazo de CT = 640 + 20Q. La curva de costo marginal correspondiente es CM
= 20. ¿Cuál es el precio que maximiza las ganancias? ¿Cuánto venderá el monopolista y qué
utilidades económicas ganará a ese precio?
La curva del ingreso marginal para esta curva de demanda es IM = 100 – 4Q. El costo marginal
es la pendiente de la curva de costo total, que en este ejemplo es constante e igual a 20. Si se estable-
ce que IM = CM, se tiene 100 – 4Q = 20, que permite obtener la cantidad de mayores utilidades,
Q* = 20. Si se sustituye Q* = 20 en la curva de demanda, se tiene el precio que maximiza las uti-
lidades, P* = 60. Esta solución se muestra de manera gráfica en la figura 12.10, en la que también
se observa la curva de costo total promedio para el monopolista. Observe que en Q* el costo total
promedio es 52, lo cual significa que el monopolista obtiene utilidades económicas de 60 – 52 = 8
para cada unidad vendida. Con Q* = 20, se tienen utilidades económicas totales de 160.

FIGURA 12.10
$/Q Precio y cantidad
100 que maximizan
las utilidades para
funciones específicas de
costo y demanda

60
Π
52
CTP

20 CM
IM D
Q
20 25 50

Observe en la figura 12.10 que el costo fijo del monopolista es irrelevante para determinar el
nivel de producción y el precio que maximizan las utilidades. De manera intuitiva esto tiene senti-
do porque el costo fijo no influye en las pérdidas y las ganancias que ocurren cuando cambia la
producción.

EJERCICIO 12.3
¿Cómo cambiarían el precio y la cantidad que maximizan las utilidades del ejemplo 12.2 si la
curva de costo total del monopolista estuviera dada por CT = 640 + 40Q? La curva de costo
marginal es CM = 40.

UN MONOPOLISTA QUE MAXIMIZA UTILIDADES NUNCA PRODUCE


EN LA PARTE INELÁSTICA DE LA CURVA DE DEMANDA
Si el objetivo de un monopolista es maximizar las utilidades, se deduce que nunca va a generar un ni-
vel de producción en la parte inelástica de su curva de demanda. Si aumentara su precio en ese nivel
de producción, provocaría un aumento del ingreso total. El aumento del precio también reduciría la
cantidad demandada, lo cual reduciría el costo total para el monopolista. Como las utilidades eco-
nómicas son la diferencia entre el ingreso y el costo totales, las utilidades tendrían que aumentar como
respuesta a un incremento de los precios si el monopolista se encontraba situado en el tramo inelás-
tico de la curva de demanda. Por lo tanto, el nivel de producción que maximiza las utilidades está en

12_CHAPTER 12.indd 385 3/6/09 8:03:40 PM


386 Capítulo 12 monopolio

la parte elástica de la curva de demanda, donde nuevos aumentos de precios disminuyen los ingresos
y los costos.

EL MARGEN DE MAXIMIZACIÓN DE UTILIDADES


La condición de maximización de utilidades IM = CM puede combinarse con la ecuación 12.6 que
establece IM = P[1 – (1/|ε|)], para obtener el margen de máximas utilidades para el monopolista:

P − CM 1
= ,
P ε (12.7)

que es la diferencia entre el precio y el costo marginal, expresada como una fracción del precio que
maximiza las utilidades. Por ejemplo, si la elasticidad precio de la demanda a la que se enfrenta un
monopolista fuera igual a –2, el margen de mayores utilidades sería –12 , lo cual implica que el precio
de maximización de las utilidades es el doble del costo marginal. En la ecuación 12.7 se indica que
el margen de máximas utilidades disminuye conforme la demanda se vuelve más elástica. En el caso
extremo de una demanda infinitamente elástica, el margen de mayores utilidades es cero (lo cual
implica que P = CM), igual que en el caso de competencia perfecta.

LA CONDICIÓN DE CIERRE DEL MONOPOLISTA


En el caso de una empresa perfectamente competitiva, se vio que le conviene cerrar en el corto plazo
cuando el precio es menor que cierto valor mínimo del costo variable promedio (CVP). La condición
análoga para el monopolista es que no exista una cantidad en la cual la curva de demanda esté por
encima de la curva de costo variable promedio. Por ejemplo, el monopolista con las curvas de de-
manda, ingreso marginal, CMC y CVP que se muestran en la figura 12.11 no tiene un nivel positivo
de producción en el cual el precio supere el CVP, así que lo mejor que podría hacer es suspender la
producción a corto plazo. Así tendría una pérdida económica a corto plazo igual a sus costos fijos,
pero estaría peor con cualquier nivel positivo de producción.

FIGURA 12.11
Un monopolista que
debe cerrar a corto
plazo $/Q
Cuando el ingreso
promedio (el valor del
precio en la curva de CVP
demanda) es menor que el CMC
costo variable promedio
en todos los niveles de
producción, lo mejor que
puede hacer el monopolista
es suspender la producción D
en el corto plazo. IM
Q
Q0 Q1

Otra manera de establecer la condición de cierre de un monopolista es decir que debe suspender
la producción siempre que el ingreso promedio sea menor que el costo variable promedio en todos

12_CHAPTER 12.indd 386 3/6/09 8:03:41 PM


el monopolista que maximiza utilidades 387

los niveles de producción. El ingreso promedio es otra manera de referirse al precio: el valor de P
sobre la curva de demanda del monopolista.9
En la figura 12.11 se ilustra también otro punto importante que es la condición IM = CM necesa-
ria, aunque no suficiente, para maximizar las utilidades. Observe en la figura que el ingreso marginal
es igual al costo marginal en el nivel de producción Q0. ¿Por qué éste no es el punto de maximización
de utilidades? Recuerde que en el caso de la empresa perfectamente competitiva, la condición de
máximas utilidades obligaba a que el precio fuera igual al costo marginal en una parte ascendente
de la curva de costo marginal, por encima del punto mínimo de la curva CVP. En el caso del mono-
polista se aplica una condición un tanto diferente. Observe en la figura 12.11 que en Q0 la curva IM
interseca la curva CM desde abajo.10 Esto significa no sólo que Q0 no es el punto de maximización
de utilidades, sino que de hecho corresponde a un nivel de utilidades menor que el de otros niveles
de producción cercanos. Por ejemplo, considere un nivel de producción apenas menor que Q0. En
cualquier nivel como éste, las ganancias de contraer la producción (CM) excederán las pérdidas (IM),
así que la empresa hace bien en contraer Q0. Ahora considere un nivel de producción un poco mayor
que Q0. En ese nivel, las ganancias de expandirse (IM) superan los costos (CM), así que a la empresa
le conviene crecer. Así, cuando la empresa está en Q0, gana mayores utilidades si se expande o si se
contrae. Q0 se denomina el punto de utilidades mínimo local.11
Observe también en la figura 12.11 que la curva IM interseca la curva CM por segunda vez en
el nivel de producción Q1. En este caso la intersección procede de arriba y es fácil demostrar, como
ejercicio, que Q1 arroja mayores utilidades que el resto de niveles de producción cercanos (es un
argumento paralelo al del párrafo anterior). Se hace referencia a los puntos del tipo Q1 como máximos
locales de utilidades. Aunque Q1 arroja más utilidades que cualquier nivel de producción cercano, la
empresa no salda su costo variable promedio en el nivel de producción, así que lo mejor es que no
produzca nada. El punto Q* que se vio arriba en la figura 12.9 es tanto un punto máximo local como
un punto máximo global de utilidades; esta última designación indica que ningún otro volumen de
producción, incluyendo cero, arroja mayores utilidades. En el caso de un monopolista, un punto
máximo global de utilidades se encuentra en los lados ascendentes o descendentes de la curva CM.
Pero debe estar en un punto en el que la curva IM interseque a la curva CM desde arriba.

EJERCICIO 12.4
Encuentre el precio y la cantidad óptimos para el monopolista que describe la información de
la tabla siguiente.

Q P IM CMC CVP
0 100 100 150 150
15 86 71 71 107
25 75 50 41 84
34 66 33 33 72
50 50 0 63 63

9
De manera más formal, observe que el ingreso promedio = IT/Q = PQ/Q = P.
10
“Intersecar desde abajo a Q0” significa que Q se aproxima a Q0 desde la izquierda, IM está debajo de CM y luego la cruza cuando
Q = Q0.
11
La condición de segundo orden de las utilidades máximas está dada por

(
d IM − CM ) = dIM − dCM < 0,
dQ dQ dQ
la cual establece que la pendiente de la curva de ingreso marginal debe ser menor que la pendiente de la curva de costo marginal.

12_CHAPTER 12.indd 387 3/6/09 8:03:43 PM


388 Capítulo 12 monopolio

Para recapitular, se ha visto que el monopolista se comporta como una empresa perfectamente
competitiva en el sentido de que escoge un nivel de producción ponderando los beneficios de aumen-
tarlo (o contraerlo) en función de sus costos respectivos. Tanto en el caso del competidor perfecto
como del monopolista, el costo marginal es la medida relevante del costo de ampliar la producción.
En los dos casos, los costos fijos son irrelevantes para las decisiones de producción a corto plazo.
Para el monopolista y el competidor perfecto los beneficios de aumentar la producción se miden por
sus valores respectivos de ingreso marginal. Para el competidor el ingreso marginal y el precio son
equivalentes. En cambio, para el monopolista el ingreso marginal es menor que el precio. El compe-
tidor maximiza las utilidades aumentando la producción hasta que el costo marginal es igual al pre-
cio. El monopolista maximiza las utilidades al aumentar la producción hasta que el costo marginal
es igual al ingreso marginal, y así opta por un nivel de producción inferior al que hubiera elegido de
haber seguido el criterio del competidor. Tanto el monopolista como el competidor perfecto hacen
bien en cerrar en el corto plazo si el precio es menor que el costo variable promedio para todos los
niveles de producción posibles.

UN MONOPOLISTA NO TIENE CURVA DE OFERTA


Como se vio en el capítulo 11, la empresa competitiva tiene una curva de oferta bien definida. Da
por sentado el precio del mercado y responde eligiendo el nivel de producción pera el cual el costo
marginal y el precio son iguales. Por lo que se refiere a la industria, un desplazamiento de la curva
de demanda permite obtener una curva de la oferta de la industria bien definida, que es la suma
horizontal de las curvas de oferta de todas las empresas.
El monopolista no tiene tal curva, porque no es tomador de precios, lo cual significa que no
hay una correspondencia única entre el precio y el ingreso marginal cuando se desplaza la curva de
demanda del mercado. Así, un valor del ingreso marginal para una curva de demanda puede corres-
ponder a un precio, mientras que el mismo valor del ingreso marginal para otra curva de demanda
corresponde a otro precio. Como resultado, es posible observar que el monopolista produce Q*1 y
vende en P* en un periodo y luego, en otro, vende Q*2 en P*.
Para ilustrarlo, considere a un monopolista con una curva de demanda de P = 100 – Q y con
las mismas curvas de costo que en el ejemplo 12.2, en particular con CM = 20. La curva de ingreso
marginal de este monopolista está dada por IM = 100 – 2Q y al igualar IM con CM se obtiene el nivel
de producción de máximas utilidades correspondientes Q* = 40. El precio que maximiza las utilida-
des correspondientes es P* = 60. Observe que es el mismo precio de maximización de las utilidades
que se vio para el monopolista del ejemplo 12.2, aunque en este caso la curva de demanda está a la
derecha de la anterior.
Cuando se desplaza la curva de demanda del monopolista, la elasticidad-precio de la demanda
para un precio dado por lo regular también se desplaza. Pero dichos desplazamientos no tienen que
ocurrir en la misma dirección. Por ejemplo, cuando la demanda se desplaza a la derecha, la elasticidad
a un precio dado puede aumentar o reducirse, y lo mismo ocurre con un desplazamiento de la de-
manda a la izquierda. El resultado es que no puede haber una correspondencia única entre el precio
que cobra un monopolista y lo que elige producir. Por lo tanto, se dice que el monopolista no tiene
curva de oferta, sino una regla de demanda, que consiste en igualar el ingreso y el costo marginales.

AJUSTES A LARGO PLAZO


A largo plazo, el monopolista es libre de ajustar todos los insumos, al igual que la empresa compe-
titiva. A largo plazo, ¿cuál es la cantidad óptima para un monopolista con una tecnología dada? Lo
mejor que éste puede hacer es producir la cantidad en la que el costo marginal a largo plazo es igual
al ingreso marginal. En la figura 12.12 esto significa escoger unas existencias de capital que generen
curvas de costo total promedio y de costo marginal a corto plazo, que se marcan como CTP* y
CMC*. Para ese nivel de existencias de capital la curva de costo marginal a corto plazo pasa por la

12_CHAPTER 12.indd 388 3/6/09 8:03:43 PM


disCriminaCión de preCios 389

FIGURA 12.12
$/Q Equilibrio a largo plazo
para un monopolista
que maximiza
utilidades
la cantidad que maximiza
las utilidades a largo
plazo es Q*, el nivel de
producción en el que Cml
P* CMC* = im. el precio de máximas
Π CTP* utilidades a largo plazo es
P*. la existencia óptima de
capital a largo plazo genera
CPL la curva de costo marginal
a largo plazo, CmC*, que
CML pasa por la intersección de
Cml e im.
D
Q
Q*
IM

intersección de las curvas de costo marginal a largo plazo y de ingreso marginal. Q* será la cantidad
que maximiza las utilidades a largo plazo y se venderá en un precio de P*. Para las condiciones repre-
sentadas en la figura 12.12, el nivel de las ganancias económicas a largo plazo, Π, será positivo y está
indicado por el área del rectángulo sombreado.
Como se vio en el capítulo 11, las utilidades económicas tienden a desvanecerse a largo plazo
en las industrias perfectamente competitivas. Esta tendencia se observa a veces en el monopolio; en
la medida en que los factores que dieron lugar a la posición monopólica de la empresa sean atacados
en el largo plazo, habrá una presión que reduzca las utilidades. Por ejemplo, empresas competidoras
pueden desarrollar sustitutos de insumos importantes que antes estaban bajo el control del mono-
polista. O, en el caso de los productos patentados, los competidores pueden desarrollar sustitutos
parecidos que no infrinjan las patentes existentes, que de todos modos son temporales.
Pero en otros casos puede haber una tendencia para que las utilidades de un monopolio persis-
tan. Por ejemplo, la empresa mostrada en la figura 12.12 tiene una curva de costo promedio a largo
plazo descendente, lo cual significa que puede disfrutar de una ventaja de costo persistente sobre sus
rivales potenciales. En estos monopolios naturales, las utilidades económicas pueden ser muy esta-
bles al paso del tiempo. Desde luego, lo mismo puede ser cierto para una empresa cuyo monopolio
tiene su origen en una licencia del gobierno. Las utilidades económicas persistentes son una de las
mayores preocupaciones políticas respecto de los monopolios, como se verá más adelante.

DISCRIMINACIÓN DE PRECIOS
Hasta aquí se ha supuesto en esta exposición que el monopolista vende toda su producción a un
precio único. Pero en realidad los monopolistas cobran precios diferentes a distintos compradores,
una práctica que se conoce como discriminación de precios. El descuento de los cines que se refirió al
comienzo del capítulo constituye un ejemplo. En las siguientes secciones se analiza cómo se com-
porta el monopolista que maximiza utilidades cuando es posible cobrar precios diferentes a distintos
consumidores. Cuando se pueden discriminar los precios, un monopolista puede transferir parte de
las ganancias de los consumidores a sus propias utilidades. Sin embargo, se verá que no todos los
incrementos de las ganancias por discriminación de precios son a expensas de los consumidores. La
eficiencia mejora cuando el monopolista aumenta la producción hacia el nivel en el que la demanda
interseca el costo marginal.

12_CHAPTER 12.indd 389 3/6/09 8:03:44 PM


390 Capítulo 12 monopolio

VENTA EN DIFERENTES MERCADOS


Suponga que el monopolista tiene dos mercados por completo distintos en los que puede vender su
producción. Quizás es el único proveedor de su producto en el mercado nacional y en el mercado
extranjero. Si maximiza sus utilidades, ¿qué precios debe cobrar y qué cantidades debe vender en
cada mercado?
Suponga que las curvas de demanda y de ingreso marginal de los dos mercados son las que se
dan en las secciones de la izquierda y el centro de la figura 12.13. Observe primero que si el monopo-
lista maximiza sus utilidades, su ingreso marginal debe ser el mismo en cada mercado (de lo contra-
rio, podría vender una unidad menos en el mercado con menor IM y una unidad más en el mercado
con mayor IM, lo que aumentaría sus utilidades). Dado que IM debe ser igual en los dos mercados,
la cantidad total que maximiza las utilidades será aquella en la que este valor común sea igual que el
costo marginal. De manera gráfica, la solución consiste en sumar en sentido horizontal las curvas de
ingreso marginal de los dos mercados y generar el nivel de producción en el que la curva resultante
interseque la curva de costo marginal. En la sección de la derecha de la figura 12.13 el producto total
óptimo está indicado por Q* = 10 unidades. Q*1 = 4 se vende en el mercado 1 al precio P*1 y el resto Q*2
= 6 en el mercado 2 al precio P*2.

FIGURA 12.13
Monopolista que
maximiza utilidades $/Q1 Mercado 1 $/Q2 Mercado 2 $/Q Total
y que vende en dos
mercados
para un monopolista que
vende en dos mercados, el P*2
nivel de producción que CM
maximiza las utilidades
es donde la curva Σim P*1
interseca la curva Cm,
en este caso Q* = 10. el CM*
ingreso marginal de cada
mercado será el mismo D1 D2
cuando Q*1 = 4 y Q*2 = 6 se ΣIM
vendan en los mercados 1 y IM1 IM2
2, respectivamente. Q1 Q2 Q1 + Q2
10 3 9 3 19
Q*1 = 4 Q*2 = 6 Q* = 10

EJEMPLO 12.3 Un monopolista tiene costos marginales MC = Q y una demanda del mercado nacional de P =
30 – Q. El monopolista también puede vender en un mercado extranjero a un precio constan-
te PF = 12. Calcule y grafique la cantidad producida, la cantidad vendida en el mercado nacio-
nal, la cantidad vendida en el mercado extranjero y el precio cobrado en el mercado nacional.
Explique por qué las utilidades del monopolista bajarían si produjera la misma cantidad pero
vendiera más en el mercado nacional.
La curva lineal de la demanda P = 30 – Q tiene un ingreso marginal asociado de IM = 30 – 2Q. El
nivel de producción que maximiza las utilidades de un monopolista que vende en mercados segmen-
tados ocurre cuando ΣIM = CM. La suma horizontal de los ingresos marginales de los dos mercados
coincide con la función del ingreso marginal nacional IMH hasta en nivel de producción nacional en
el que IMF = IMH, y a partir de entonces es la función de ingreso marginal exterior IMF = 12 (vea la

12_CHAPTER 12.indd 390 3/6/09 8:03:45 PM


disCriminaCión de preCios 391

FIGURA 12.14
Monopolista con un
Precio (P) mercado extranjero de
30 elasticidad perfecta
la curva Σim sigue a imH
siempre que imH ≥ imF, y
CM luego sigue a imF. el nivel de
21 producción que maximiza
las utilidades es donde
la curva Σim interseca la
12 ΣIM curva Cm, aquí Q* = 12.
IMF

IMH
Cantidad (Q)
0 9 12 15 30

figura 12.14). El ingreso marginal total es igual al costo marginal cuando IMF = CM, que se resuelve
para Q = 12. El costo marginal para este nivel de producción es igual al ingreso marginal nacional en
30 – 2QH = 12, de modo que QH = 9 y las unidades restantes se venden en el extranjero:

QF = Q – QH = 12 – 9 = 3.

En el mercado nacional, el monopolista cobra:

PH = 30 – QH = 30 – 9 = 21.
Las unidades adicionales vendidas en el país arrojarían un ingreso marginal menor que 12. Como
las ventas en el mercado extranjero arrojan un ingreso marginal constante de 12, trasladar las ventas
al mercado interno reduciría las utilidades debido a la pérdida de ingreso marginal de cada unidad
desplazada.

EJERCICIO 12.5
Suponga que un monopolista vende en dos mercados por separado con curvas de demanda
dadas por P1 = 10 – Q1 y P2 = 20 – Q2, respectivamente. Si su curva de costo total está definida
por CT = 5 + 2Q (para la cual la curva de costo marginal asociada se calcula mediante CM = 2),
¿qué cantidades debe vender y qué precio debe cobrar en ambos mercados?

Observe que, en el ejercicio 12.5, el monopolista que vende en dos mercados cobra un precio
más alto en el mercado donde la demanda es menos elástica con respecto al precio.12 Muchas veces,
el cobro de distintos precios a los compradores de mercados totalmente separados se conoce como
discriminación de precios de tercer grado. El término “tercer grado” no tiene un significado especial
más allá del hecho de que este tipo de discriminación de precios fue el tercero que apareció en una
taxonomía anterior.
Abundan los ejemplos de discriminación de precios de tercer grado. Por ejemplo, este libro de
texto que tiene en sus manos también se presenta en una edición internacional que se vende a una
tercera parte del precio de la edición nacional. Como los ingresos de los estudiantes por lo general
son mucho más bajos en los mercados extranjeros que en Estados Unidos, la elasticidad-precio de la

12
Este resultado se deduce de la ecuación 12.6, que dice que IM = P(1 – 1/ε). Establecer que IM1 = IM2 produce P1/P2 =
(1 – 1/ε2)/(1 – 1/ε1). Por lo tanto, se cobrará el precio más alto a los clientes con la elasticidad precio de la demanda más baja.

12_CHAPTER 12.indd 391 3/6/09 8:03:46 PM


392 Capítulo 12 monopolio

demanda tiende a ser más alta en el extranjero que en los mercados estadounidenses. El precio que
maximiza las utilidades en el mercado de Estados Unidos desalentaría la compra del libro por parte
de los estudiantes de países en vías de desarrollo.

¿Por qué algunos médicos y abogados hacen descuentos a personas de bajos ingresos?
En la medicina, el derecho, la
odontología y otras profesiones,
muchos profesionales fijan sus
EL NATURALISTA honorarios en una “escala móvil”,
ECONÓMICO con el objetivo de prestar sus ser-
12.1 vicios a consumidores de bajos in-
gresos con descuentos importan-
tes. Se ha dicho que esta práctica
surge porque a los profesionales
les preocupan los problemas eco-
nómicos de los pobres. No cabe
duda que dichas inquietudes a
menudo son sinceras. Aunque
también se debe tomar en cuenta
que los servicios que ofrecen estos
profesionales son bienes norma-
les, esto significa que las curvas de
demanda de los clientes de bajos
ingresos se encuentran muy por
debajo del resto de clientes más
¿por qué los médicos cobran menos a los pacientes pobres? adinerados. Así pues, los honora-
rios en escala deslizable pueden
verse también como un intento
de los profesionales por aumentar sus ganancias al ajustar sus precios a las diferencias de elasticidad
entre los diferentes grupos de clientes. En el mercado de los boletos para el cine se ve un patrón seme-
jante, pues es común que los propietarios de las salas establezcan precios más bajos para estudiantes,
personas de la tercera edad y otros grupos que se considera que tienen mayores elasticidades-precio
de la demanda.

arbitraje compra de algo Asimismo, la discriminación de los precios sólo es factible cuando es imposible, o al menos
para revenderlo a un precio poco práctico, que los compradores hagan tratos comerciales entre sí. Por ejemplo, si los estudiantes
mayor sin riesgos de costos. de otros países pudieran negociar con los de Estados Unidos, no sería posible vender en esencia el
mismo libro a 30 dólares en Calcuta y a 100 dólares en Nueva York. Los estudiantes empresariales
comprarían los libros a 30 dólares en el extranjero y los venderían a estudiantes estadounidenses por,
digamos, 95 dólares; otros, esperando entrar en acción, rebajarían el precio todavía más y, con el
tiempo, desaparecería el diferencial de precio. Comprar a un precio bajo y revenderlo a uno más alto
se denomina con frecuencia arbitraje. Cuando el arbitraje es práctico no pueden persistir grandes
diferenciales de precios para un solo producto. El arbitraje, por ejemplo, garantiza que el precio del
oro en Londres nunca difiera en gran medida del que tiene en Nueva York.

¿Por qué los dueños de los cines ofrecen descuento a los estudiantes en el boleto de entrada
pero no en las palomitas?
EL NATURALISTA El arbitraje sólo es práctico en algunos casos. Los descuentos a los estudiantes en los boletos de
ECONÓMICO entrada permiten a los operadores de las salas de cine dividir sus mercados porque no es posible que
una persona vea una película a un precio bajo y después venda la experiencia a alguien más a un pre-
12.2 cio más alto. Del mismo modo, con base en las diferencias de elasticidad precio de la demanda, para
los abogados y médicos es práctico cobrar diferentes honorarios a las personas. Pero dicha división

12_CHAPTER 12.indd 392 3/6/09 8:03:49 PM


el monopolista Con disCriminaCión perfeCta 393

del mercado es más difícil para


productos como las palomitas.
Si los operadores de las salas de
cine intentaran vender a los estu-
diantes las palomitas a un dólar y
a tres dólares a los adultos, algún
estudiante con dotes empresaria-
les aprovecharía la oportunidad
del arbitraje para venderles a los
adultos inconformes las palo-
mitas a sólo dos dólares. Y con
la presión de la competencia de
otros que pusieran en práctica el
arbitraje, el diferencial de precio
caería hasta que apenas fuera lo
suficiente para que valiera la pena
que los estudiantes realizaran la
transacción.
¿por qué los propietarios de las salas de cine ofrecen descuentos
a los estudiantes en los precios de las entradas pero no en las
golosinas?

EL MONOPOLISTA CON DISCRIMINACIÓN PERFECTA


El término discriminación de precios de primer grado se utiliza para describir la división del mercado
más grande posible. Como ejemplo, suponga que un monopolista tiene N clientes potenciales, cada
uno con una curva de demanda con pendiente negativa Di como la que se muestra en la figura 12.15.
¿Cuál es el mayor ingreso que obtendría el monopolista de la venta de Q' unidades de producto a un
cliente así? Si el monopolista tuviera que vender todas las unidades al mismo precio, lo mejor que
podría hacer es cobrar P', lo que generaría un ingreso total de P'Q'. Pero si puede cobrar diferentes
precios para las distintas unidades de producción, le iría mucho mejor. Por ejemplo, puede vender
las primeras Q' unidades a un precio de P1, las siguientes Q2 – Q1 unidades a un precio de P2 y así en lo
sucesivo. Si los intervalos en los que el monopolista divide el producto son arbitrariamente peque-
ños, esta forma de fijación de precios aumenta los ingresos totales conforme al área del triángulo
sombreado en la figura 12.15.

FIGURA 12.15
P Discriminación perfecta
P1 de precios
P2 si el monopolista vende
P3 cada unidad de producción
a un precio diferente, cobra
al comprador el máximo
que está dispuesto a pagar
P’ por cada unidad. en dicha
situación, el monopolista
captura todo el excedente
del consumidor.

Di

Qi
Q1 Q3 Q’
Q2

12_CHAPTER 12.indd 393 3/6/09 8:03:51 PM


394 Capítulo 12 monopolio

Si el monopolista hubiera sido forzado a asignar un precio único para todas las unidades, ese
triángulo sombreado hubiera sido el excedente del consumidor. Sin embargo, si puede cobrar dis-
tintos precios por cada unidad, el monopolista captura el excedente del consumidor, que paga el
máximo que hubiera estado dispuesto a dar por cada unidad y, como resultado, se queda sin exce-
dente.
¿Cuánta producción genera un monopolista con discriminación perfecta que maximiza las ga-
nancias? Como siempre, la regla es igualar el ingreso marginal como el costo marginal. La figura
12.16 presenta las curvas de demanda, de costo marginal a corto plazo y de costo total promedio de
un monopolista con discriminación perfecta. Pero, ¿cuál es la curva de ingreso marginal para este
monopolista? Es idéntica a su curva de demanda. Como logra una discriminación perfecta, puede bajar
su precio para vender más producción sin tener que recortar el precio de la producción vendida origi-
nalmente. El precio y el ingreso marginal son uno solo, como en el caso de la competencia perfecta.
Lo mejor que puede hacer esta empresa es producir Q* unidades de producto, cada una de las cuales
se vende al precio más alto que está dispuesto a pagar cada comprador.

FIGURA 12.16
El monopolista con $/Q
discriminación perfecta
la curva de ingreso
marginal del monopolista
que practica una
discriminación perfecta
es idéntica a su curva de CMC
demanda. la producción Π
que maximiza las ganancias
CTP
es Q*, donde se intersecan
las curvas de CmC y
de demanda. el área
sombreada es la ganancia D
económica (Π).
Q
Q*

Hay dos puntos salientes de comparación entre el monopolista de discriminación perfecta y


el que no logra discriminar por completo. El primero es que el discriminador perfecto produce un
mayor nivel de producción porque no le preocupa el efecto de un precio recortado sobre el ingreso
derivado de la producción obtenida hasta ese momento. Puede recortar el precio para las personas
que no estuvieran dispuestas a comprar y mantener los precios más altos para quienes sí los pagan.
La segunda diferencia importante es que, por lo general, hay un excedente del consumidor positi-
vo en el caso del monopolista sin discriminación, mientras que con el discriminador perfecto no existe.
Como el monopolista que no discrimina debe cobrar el mismo precio a todos los compradores, tiene
la presión de no fijar un precio demasiado alto. Si lo fija en el nivel que están dispuestos a pagar los de-
mandantes menos elásticos, perderá a los demás clientes. El resultado es que no lo hará y por lo tanto
los demandantes menos elásticos terminarán por pagar un precio muy por debajo de sus respectivos
precios de reserva, de aquí el excedente del consumidor.
La discriminación perfecta de precios es un límite teórico inalcanzable. Si un cliente tuviera
tatuada en la frente una curva de demanda, sería posible que el vendedor ajustara cada precio para
obtener la máxima cantidad posible de cada comprador. Aunque, en general, los detalles de un
demandante sólo son conocidos de manera imperfecta por el vendedor. En muchas ocasiones los
comerciantes calculan la elasticidad individual con base en la información que conocen sobre los
grupos a los que pertenece el individuo. Por ejemplo, un comerciante por catálogo puede imprimir
ediciones especiales a precios más altos para enviar por correo a zonas de código postal de altos
ingresos, como 90213 (Beverly Hills, California).

12_CHAPTER 12.indd 394 3/6/09 8:03:52 PM


el monopolista Con disCriminaCión perfeCta 395

Quizá lo más cercano que podamos observar acerca de una evaluación profunda de las elasti-
cidades individuales es el comportamiento de los comerciantes de los bazares en el Medio Oriente.
El astuto comerciante de camellos tiene muchos años de experiencia intentando evaluar cuánto está
dispuesto a pagar un comprador con un perfil demográfico y psicológico determinado. Su objetivo
en el negocio es interpretar el gesto incongruente o la mirada furtiva. Incluso en este caso es proba-
ble que el comprador taimado sepa cómo ocultar su ansiedad por ser propietario del camello.

DISCRIMINACIÓN DE PRECIOS DE SEGUNDO GRADO


Otra forma de discriminación de precios es la práctica mediante la cual los vendedores no fijan un
precio único sino un esquema en el que disminuye el precio conforme aumenta la cantidad que se compra.
Por lo tanto, muchos servicios públicos de electricidad emplean lo que se llama las estructuras tarifa-
rias con base decreciente en la que, por ejemplo, los primeros 300 kilowatts-hora al mes se facturan
a 10 centavos, los 700 siguientes a 8 centavos y las cantidades superiores a 1 000 kilowatts-hora/
mes a 5 centavos cada uno. Dichas estructuras tarifarias son una forma de discriminación de precios de
segundo grado.
La figura 12.17 ilustra el efecto de una estructura tarifaria de este tipo para un consumidor con
una curva de demanda marcada como Di. En comparación con la alternativa de cobrar un precio de
P3 por cada unidad, el esquema de descuento por cantidad aumenta el pago total del consumidor en
un monto igual al del área sombreada.

FIGURA 12.17
Discriminación de
precios de segundo grado
P el vendedor ofrece el primer
bloque de consumo (de
0 a Q1) a un precio alto
(P1), el segundo bloque (de
P1 Q1 a Q2) a un precio más
bajo (P2), el tercer bloque
P2 (de Q2 a Q3) a un precio
P3 incluso más bajo (P3) y así
sucesivamente. aun cuando la
discriminación de precios de
segundo grado no pretende
adecuar las tarifas a las
Di características de individuos
Q
o grupos específicos,
0 Q1 Q2 Q3 muchas veces permite al
monopolista captar una parte
sustancial del excedente
del consumidor (el área
sombreada).

La discriminación de precios de segundo grado es como la de primer grado en cuanto a que


intenta obtener el excedente del consumidor de cada comprador. Las dos diferencias principales son:
1) con los esquemas de segundo grado cada consumidor tiene la misma estructura tarifaria, lo cual
significa que no se intenta adecuar los precios a las diferencias de elasticidad entre compradores; y 2)
el número limitado de categorías tarifarias tiende a limitar la cantidad de excedente del consumidor
que se puede obtener. Los esquemas de primer grado abarcan todo el triángulo, mientras que en la
figura 12.17 se muestra que los esquemas de segundo grado sólo captan una parte de él.

MODELO DE OBSTÁCULO PARA LA DISCRIMINACIÓN DE PRECIOS


A todo vendedor le gustaría practicar la discriminación perfecta de precios. Como se mencionó antes,
lo difícil es que los vendedores no cuentan con la información de las curvas de demanda individuales

12_CHAPTER 12.indd 395 3/6/09 8:03:53 PM


396 Capítulo 12 monopolio

para hacerlo. Otra forma de discriminación de precios aún más importante consta de una técnica
en la que una empresa incita a los compradores más elásticos para que se identifiquen. Se le llama
modelo de obstáculo para la discriminación de precios. La idea básica es que el vendedor establece cierto
tipo de obstáculo y fija un precio de descuento a disposición de los compradores que eligen pasar
dicho obstáculo. La lógica es que los compradores más susceptibles al precio estarán más dispuestos
a superar el obstáculo que los demás.
Un ejemplo de un obstáculo es una forma de bonificación incluida en el paquete del producto.
En este caso, superar el obstáculo significa llenar el formulario, buscar un timbre y un sobre e ir al
correo para enviarlo. La empresa espera que las personas a las que no les importa mucho el precio
no se molesten en realizar el proceso. De ser así, las personas cuyas demandas son menos elásticas
terminan pagando el precio “normal”, mientras aquellas con demandas más elásticas pagan el precio
de descuento más bajo.
Es raro el producto cuyo vendedor no usa el modelo de obstáculo de precios diferenciales. Los
vendedores de libros ofrecen ediciones de pasta dura a un precio alto durante el primer año de la
publicación. Los compradores que no se preocupan mucho por el precio compran estas ediciones
en cuanto salen a la venta. Otros esperan 1 o 2 años y compran una edición de pasta suave mucho
más barata. En este caso el obstáculo es la espera. Los vendedores de aparatos electrodomésticos
ofrecen ventas regulares de productos maltratados en las cuales las máquinas con defectos estéticos
insignificantes se venden a menos de la mitad de su precio normal. Aquí se presentan dos obstáculos
comunes: averiguar cuándo y dónde se realiza la venta y aceptar el rasguño o la abolladura (que
pocas veces está a la vista). Las líneas aéreas ofrecen descuentos “de gran ahorro” de hasta la mitad
de la tarifa normal en clase turista. Una vez más, se ven dos obstáculos comunes: tener que hacer las
reservaciones con una semana o más de anticipación y tener que quedarse un sábado por la noche.
Muchos minoristas incluyen cupones de descuento en los anuncios del periódico. Los obstáculos aquí
son tener que leer los anuncios, recortar los cupones e ir a la tienda antes de que venzan. Algunos
vendedores colocan letreros detrás del mostrador con la leyenda: “Pregunte por nuestras ofertas.”
Aquí, el obstáculo simplemente es preguntar. Pero incluso este obstáculo insignificante puede ser
muy eficaz porque muchos compradores de buena posición económica ni siquiera se tomarían la
molestia de preguntar por la oferta.
Ninguno de estos esquemas distingue a la perfección a los compradores de alta elasticidad y a
los de baja. Por ejemplo, algunas personas esperan las rebajas en blancos para comprar sus toallas,
aun cuando comprarían la misma cantidad si no estuvieran en oferta. Pero en general, el obstáculo
parece funcionar más de lo pensado. Un obstáculo perfecto sería el que impusiera sólo un costo
insignificante para que los compradores lo superaran, pero que dividiera perfectamente a los com-
pradores según su elasticidad de la demanda. De manera analítica, la figura 12.18 muestra el efecto
de un obstáculo así, donde PH representa el precio “normal” y PL es el precio de descuento. Con un

FIGURA 12.18
Obstáculo perfecto
Cuando un obstáculo P
es perfecto, los únicos
compradores con derecho
al precio de descuento
(PL) por haberlo superado PH
son los que no estarían
dispuestos a pagar el precio
PL
normal (PH). además, un
obstáculo perfecto no
impone costos significativos
a quienes lo superan. QL D
Q
QH QL + QH

12_CHAPTER 12.indd 396 3/6/09 8:03:54 PM


pérdida de efiCienCia del monopolio 397

obstáculo perfecto, nadie que pague el precio de descuento tiene un precio de reserva superior o
igual al precio normal, lo cual significa que, de estar disponible el precio normal, se excluiría del
mercado a todas las personas.
El modelo de obstáculo no tiene que limitarse a la versión de dos precios de la figura 12.18. Al
contrario, muchos vendedores lo han convertido en un arte muy complejo que incluye literalmente
docenas de combinaciones de obstáculos de precios. Por ejemplo, United Airlines ofrece docenas de
tarifas en su ruta Los Ángeles-Honolulú, cada una con sus propias restricciones. Pero a pesar de cuán
simple o complejo sea el esquema, la meta es la misma: ofrecer descuentos a los clientes que de otro
modo no comprarían el producto.
El modelo de obstáculo es como la discriminación de precios de primer grado en cuanto a que
busca adecuar los precios a las elasticidades de los compradores individuales. La principal diferencia
es que incluso en su forma más sofisticada, no se puede esperar que el modelo de obstáculo capte
todo el excedente del consumidor.

PÉRDIDA DE EFICIENCIA DEL MONOPOLIO


Recuerde que en el capítulo 11 se hizo la afirmación de que la competencia perfecta conduce a una
asignación de recursos eficiente. Esta afirmación se basó en la observación de que en el equilibrio
competitivo a largo plazo no hay posibilidad de obtener ganancias adicionales del intercambio. El
valor para los compradores de la última unidad de producción es exactamente igual al valor en el
mercado de los recursos requeridos para producirla.
¿Cómo se mide el equilibrio a largo plazo en el monopolio con los mismos criterios? Pues no
muy bien. Para ejemplificarlo, considere a un monopolista con costos marginales y promedio a largo
plazo constantes y la estructura de la demanda que se observa en la figura 12.19. La cantidad que
maximiza las utilidades de este monopolista es Q*, que vende a un precio de P*. Observe que en Q*
el valor de una unidad adicional de producto para los compradores es P*, que es mayor que el costo
de producir otra unidad, CML. Esto significa que el monopolista de un precio único no agota todas
las ganancias posibles derivadas del intercambio. Como se vio antes, si el monopolista pudiera cobrar
diferentes precios a cada comprador, la producción se expandiría a QC, que es la misma cantidad que

FIGURA 12.19
Pérdida del bienestar en
un monopolio de precio
único
$/Q un monopolista que cobra
un precio único a todos
los compradores producirá
Q* y venderá a P*. una
S2
industria competitiva
P* que opera en las mismas
condiciones de costo
Π
S1 produciría QC y vendería a
PC CPL = CML PC. en comparación con el
resultado de competencia
perfecta, un monopolio
de precio único da como
resultado una pérdida del
IM D excedente del consumidor
equivalente al área de
Q Π + S1, dado que el
Q* QC monopolista gana Π, el
costo para la sociedad es
S1, llamada pérdida de peso
muerto del monopolio.

12_CHAPTER 12.indd 397 3/6/09 8:03:55 PM


398 Capítulo 12 monopolio

se vería en una industria perfectamente competitiva con las mismas condiciones de demanda y costo.
Si la producción aumentó de Q* a QC debido a la discriminación perfecta de precios, la ganancia en
el excedente del productor sería igual a las áreas combinadas de los triángulos marcados como S1
y S2. Con la competencia perfecta el triángulo S1 sería parte del excedente del consumidor. Para la
sociedad, el costo de tener una industria que abastece un monopolista con un precio único, en vez de
vendedores con competencia perfecta, sería perder ese excedente del consumidor.
Por lo tanto, en términos de eficiencia pura, el monopolista con discriminación perfecta y la
industria de competencia perfecta conducen al mismo resultado. La diferencia es que, en el primer
caso, el beneficio está en el excedente del productor y, en el segundo, en el excedente del consumi-
dor. La pérdida de eficiencia del monopolio es el resultado de no lograr la discriminación perfecta
del precio. Esta pérdida (el área del triángulo S1 de la figura 12.19) se llama pérdida de peso muerto
del monopolio.
En el análisis anterior tenía sentido hablar de la pérdida del bienestar por tener un monopolio
en lugar de competencia porque la estructura de costos era compatible con la competencia perfecta.
Pero con esa clase de estructura de costos, sólo los impedimentos legales podrían evitar que surgiera
la competencia. La existencia de las utilidades económicas (Π en la figura 12.19) atraería a los compe-
tidores a la industria hasta que el precio y la cantidad se dirigieran a PC y QC, respectivamente.
Suponga que el motivo por el que el monopolio tiene una curva CPL plana es que la empresa
disfruta de una patente para su producto. ¿Se puede decir ahora que la pérdida de bienestar por tener
un monopolio de precio único es igual a la pérdida del excedente del consumidor que se mide en la
figura 12.19? Antes de contestar, primero se debe hacer la pregunta “¿cuál es la alternativa para la
situación actual?” Si se trata de una sociedad que carece de protección de una patente, es muy pro-
bable que, para empezar, nunca se hubiera adquirido el producto, así que no tiene sentido quejarse
de que, en comparación con la competencia pura, el monopolio produce una pérdida de bienestar.
En realidad, el monopolio de un precio único que está protegido con una patente no agota todas las
ganancias posibles de la transacción. Pero con el monopolio protegido con patente se obtiene un
excedente del consumidor además del excedente del productor de S2 + Π; quizá no se obtendría nada
sin la protección de la patente.

POLÍTICA PÚBLICA HACIA UN


MONOPOLIO NATURAL
Estas observaciones dejan claro que la pregunta relevante no es si el monopolio es eficiente en com-
paración con cierto ideal en teoría inalcanzable, sino cómo se compara con las alternativas que se
confrontan en la realidad. Esta pregunta adquiere su mayor importancia en el caso del monopolio
natural.
Para mantener el análisis de manera simple, considere una tecnología en la que el costo total se
obtiene con

CT = F + MQ, (12.8)

donde Q es el nivel de producción. Y suponga que las curvas de ingreso marginal y de demanda de un
monopolista de un precio único que se produce con esta tecnología son como las que se presentan
en la figura 12.20. La asignación ideal teórica de este mercado sería producir una cantidad de Q** y
venderla al costo marginal, que aquí es igual a M. Por el contrario, el monopolio de precio único sólo
produce Q* y lo vende a P*.
Básicamente, hay dos objeciones para el par precio-cantidad en el equilibrio del monopolio
natural de precio único: 1) la objeción de justicia se refiere a que el productor gana una utilidad econó-
mica (Π); y 2) la objeción de eficiencia tiene que ver con que el precio sea superior al costo marginal, lo
que resulta en la pérdida del excedente del consumidor (S).
Quienes hacen las políticas pueden responder a una variedad de objeciones de equidad y eficien-
cia. Las cinco opciones que se consideran a continuación representan las alternativas más importan-
tes.

12_CHAPTER 12.indd 398 3/6/09 8:03:55 PM


polítiCa púBliCa haCia un monopolio natural 399

FIGURA 12.20
$/Q Monopolio natural
las dos objeciones
principales para el
monopolio natural de
precio único son que gana
la utilidad económica (Π) y
que da como resultado la
pérdida del excedente del
P* consumidor (S).
Π

S CTP
M CM = M

IM D
Q
0 Q* Q **

I. PROPIEDAD Y GESTIÓN ESTATAL


La eficiencia requiere que el precio sea igual al costo marginal. El pro-
blema que genera es que, en el caso del monopolio natural, el costo
marginal está por debajo del costo total promedio. Como las empresas
privadas no pueden cobrar precios menores al costo promedio y man-
tenerse en el negocio a largo plazo, la empresa con precio único no
tiene otra alternativa que cobrar más que el costo marginal. Una op-
ción para evitar este problema en particular es que el Estado controle la
industria. La característica atractiva de esta opción es que el gobierno
no está obligado, como una empresa privada, a obtener al menos una
utilidad normal. Por eso, podría fijar un precio igual al costo marginal
y absorber las pérdidas económicas resultantes de los ingresos fiscales
en general.
Pero la propiedad estatal también tiene características poco atrac-
tivas. La más importante es el hecho de que a menudo parece debili-
tar los incentivos de una administración eficiente y consciente de los
costos. Como lo enfatizara el economista Harvey Leibenstein de la
Universidad de Harvard, los costos de una organización no sólo depen-
den de su tecnología, sino también de la energía con la que busque
la eficiencia. En palabras de Leibenstein, se dice que una organización
que no actúa de manera enfática para controlar los costos muestra una
ineficiencia X.13
La ineficiencia X no es en modo alguno patrimonio del gobierno.
También se encuentra en grados muy variados en las empresas priva- “luego, después de muchos años de fracasar en los
das. Leibenstein argumentaba que el grado en el cual la ineficiencia negocios, Bertram al fin logró hacer su fortuna vendiendo
X sea un problema, dependerá de los incentivos económicos, lo cual letreros para el sistema postal que decían: “disculpe, esta
sugiere una razón teórica para creer que es probable que esté más ex- ventanilla está cerrada.”
tendida en el gobierno. Cuando una empresa privada recorta un dólar
de sus costos, sus utilidades aumentan un dólar. Por el contrario, cuando la persona encargada de ineficiencia X condición en
la dependencia gubernamental reduce un dólar de su presupuesto, el efecto es simplemente una la que una empresa no obtiene
disminución de su feudo. la máxima producción a partir
de una combinación dada de
13
Harvey Leibenstein, “Allocative Efficiency vs. X-Efficiency”, en American Economic Review, junio de 1966, pp. 392-415. insumos.

12_CHAPTER 12.indd 399 3/6/09 8:04:00 PM


400 Capítulo 12 monopolio

Varios catedráticos reconocidos sostienen que el objetivo de la mayoría de los burócratas es


maximizar sus presupuestos operativos.14 Con ello no se niega que gran parte de los burócratas sean
servidores públicos sinceros y dedicados. Pero quizás el burócrata, por naturaleza humana, considere
que su dependencia en particular tiene la misión gubernamental más importante y por consiguiente
que presione en consecuencia.
Más adelante se tomarán en consideración ciertas pruebas cuantitativas que se sustentan en la
eficiencia relativa de las empresas privadas y estatales. Sin embargo, hasta la experiencia común pro-
porciona ciertos experimentos naturales relevantes. Por ejemplo, en las ciudades del norte de Esta-
dos Unidos cuando llega la primavera las cuadrillas de obreros del municipio reparan los baches que
se hicieron en invierno; mientras, al mismo tiempo, compañías privadas de pavimentación reparan
los estacionamientos de supermercados, centros comerciales y demás vías privadas. Muchas veces,
el contraste entre ambos grupos es impresionante. Es muy común ver a siete miembros de una cua-
drilla del municipio apoyados en las palas fumando un cigarro en tanto le dan órdenes ocasionales
al recién llegado, que con pereza apisona el asfalto en los agujeros. Las cuadrillas de las empresas
privadas que trabajan en los estacionamientos en general rellenan más baches al día, aunque muchas
veces están formadas por menos trabajadores.
Otro caso que ilustra los efectos de la administración estatal es el Departamento de registro de
automóviles. Intente recordar la última vez que fue a esta oficina. ¿Iría de nuevo si no tuviera que
hacerlo? Si es como la mayoría de las personas, salió de la oficina con la impresión de que hasta el
gerente menos talentoso habría encontrado la forma para que el servicio fuera más expedito. Es
posible especular con toda tranquilidad que pocas personas elegirían vivir en una sociedad en la que
el Departamento de registro de automóviles estuviera a cargo de una parte sustancial de la actividad
productiva.
A pesar del problema de ineficiencia X, en algunos casos la mejor solución es el monopolio natu-
ral operado por el Estado.15 Pero hay otras alternativas políticas que ofrecen muchos de los mismos
beneficios con menos costos aparentes.

2. REGULACIÓN ESTATAL DE MONOPOLIOS PRIVADOS


Una de tales alternativas es dejar la propiedad en manos privadas, al tiempo que se proporcionan
lineamientos o regulaciones que limiten la discreción de los precios. El ejemplo típico de este plan-
teamiento es la regulación pública de compañías privadas que proveen servicios de electricidad, agua
y teléfonos.
La principal forma de regulación de precios por parte del gobierno que se emplea en Estados
Unidos se denomina regulación de la tasa de rendimiento, en la cual se fijan los precios para permitir que
la empresa gane una tasa de rendimiento predeterminada sobre su capital invertido. Lo ideal es que
la tasa de rendimiento permita a la empresa recuperar con exactitud el costo de oportunidad de su
capital, es decir, idealmente sería igual a la tasa de rendimiento competitiva sobre la inversión.
Sin embargo, en la práctica, las comisiones reguladoras nunca pueden asegurar cuál será la tasa
de rendimiento competitiva en un periodo determinado. Si la tasa que establecen es inferior al ren-
dimiento competitivo, la empresa tendrá el incentivo de reducir la calidad de su servicio y, con el
tiempo, salir del negocio. Si, por el contrario, los reguladores establecen una tasa de rendimiento
demasiado alta, los precios serán más elevados de lo necesario y la empresa obtendrá un beneficio
fuera de lo normal. Ninguno de estos resultados es atractivo, aunque las comisiones reguladoras por
tradición han decidido que los problemas provocados por una tasa de rendimiento insuficiente son
mucho más graves que los causados por una tasa excesiva.
Harvey Averch y Leland Johnson fueron los primeros en explorar con detalle las consecuencias
de una tasa de rendimiento reguladora superior al costo de capital.16 Su conclusión es, en resumidas
14
Vea, por ejemplo, William Niskanen, Bureaucracy and Representative Government, Chicago: Aldine-Atherton, 1971; y Gordon Tu-
llock, The Politics of Bureaucracy, Washington, DC: Public Affairs Press, 1965. Pero para un punto de vista contrastante, vea Albert
Breton y Ronald Wintrobe, The Logic of Bureaucratic Conduct, Cambridge: Cambridge University Press, 1982.
15
Vea Elliott D. Sclar, You Don’t Always Get What you Pay For, Ithaca, NY: Cornell University Press, 2000.
16
Harvey Averch y Leland Johnson, “Behavior of the Firm under Regulatory Constraint”, en American Economic Review, diciembre
de 1962: pp. 1052-1069. Vea también R. M. Spann, “Rate of Return Regulation and Efficiency in Production: An Empirical Test of
the Averch-Johnson Thesis”, en Bell Journal of Economics, primavera de 1974, pp. 38-52.

12_CHAPTER 12.indd 400 3/6/09 8:04:01 PM


polítiCa púBliCa haCia un monopolio natural 401

cuentas, que esta práctica da a la empresa un incentivo para sustituir el capital por otros insumos en
una manera que aumente el costo de hacer el negocio. Si el objetivo de la empresa de servicio pú-
blico regulada es maximizar las utilidades, el comportamiento a seguir será crear su “tasa base” —el
capital invertido sobre el cual gana la tasa de rendimiento permitida— lo más grande posible. Si el
monopolista regulado puede pedir prestado capital a 8% anual y se le permite ganar 10% al año sobre
cada dólar invertido, tendrá 20 000 dólares de utilidad extra por cada millón de dólares adicional de
fondos prestados que invierta.
La discrepancia entre la tasa de rendimiento permitida y el costo de capital real genera por lo
menos dos distorsiones importantes. A la primera se le podría llamar el efecto de enfriador de agua
dorado. Se refiere al hecho de que el monopolista regulado tiene un incentivo para comprar más
equipo de capital que el necesario para generar un nivel de producción determinado. Ante la opción
de comprar, por ejemplo, un enfriador de agua normal y uno dorado más caro, el monopolista regu-
lado tiene el incentivo de optar por el último. Para ejemplificar, suponga que la tasa de rendimiento
permitida sobre el capital es de 10% anual y el costo de capital real de sólo 8%. Si el enfriador de agua
dorado tiene un precio 1 000 dólares más que el normal, entonces el monopolista obtendría una uti-
lidad mayor que 20 dólares al año por la instalación del enfriador de agua más caro. Las comisiones
reguladoras intentan evitar la compra de equipo innecesario, pero las complejidades de las operacio-
nes diarias son demasiado grandes para permitir la supervisión cuidadosa de cada decisión.
Una segunda distorsión inducida por la regulación de la tasa de rendimiento es típica del mono-
polista que da servicio a más de un mercado por separado y se le llama el efecto de subsidio cruzado.
Como la tasa de rendimiento permitida supera el costo de capital, el monopolista tiene el incentivo
de vender por debajo del costo en el mercado más elástico y subsidia en forma cruzada las pérdidas
resultantes al vender por arriba del costo en el mercado menos elástico. La idea es que el precio
por debajo del costo en el mercado elástico impulse las ventas para compensar la restricción de las
mismas provocada por el precio por arriba del costo en el mercado menos elástico. El aumento resul-
tante en la producción incrementa los requisitos de capital para generarla y, por ende, incrementa las
utilidades que permite la regulación.
Para ejemplificar, considere al monopolista regulado cuyas curvas de demanda y de costo para
dos mercados se presentan en la figura 12.21. Se trazaron las curvas CTP para incluir la tasa de uti-

FIGURA 12.21
Subsidios cruzados para
aumentar la producción
total
$/Q Mercado 1 $/Q Mercado 2 por lo general se permite
que un monopolista
CTP2 regulado obtenga una tasa
de rendimiento superior al
costo de capital real, lo cual
ofrece un incentivo para
P ’1 adquirir la mayor cantidad
D2 de capital posible. para
Π1
Π2 aumentar la producción
(y por lo tanto la reserva
de capital requerida), el
CTP1 P ’2
CM1 monopolista puede vender
CM2 por arriba del costo en su
IM1 D1
mercado menos elástico
Q1 Q2 (mercado 1 en la sección
0 Q’ 1 Q’2 a) y usar las utilidades
resultantes (Π1 > 0) para
a) b)
subsidiar las pérdidas (Π2
< 0) sostenidas con las
ventas por debajo del costo
en su mercado más elástico.

12_CHAPTER 12.indd 401 3/6/09 8:04:02 PM


402 Capítulo 12 monopolio

lidad permitida, que supera el costo de capital. Así, cuando el monopolista obtiene una utilidad de
cero en términos de las curvas de costo que se muestran en la figura 12.21, en realidad gana

Π = (r a – r c)K, (12.9)

donde r a es la tasa de rendimiento permitida, r c es el costo de capital real y K es el tamaño de la


reserva total de capital. Por lo tanto, para maximizar la utilidad, el monopolista quiere que K sea lo
más grande posible, lo cual a su vez significa hacer que la suma de la producción vendida en los dos
mercados sea lo más grande posible. Para hacerlo establecerá IM = CM en el mercado con la de-
manda menos elástica (mercado 1 de la sección a) y usará las utilidades ganadas en ese mercado (Π1)
para subsidiar un precio por debajo del costo promedio en el mercado con una demanda más elástica
(mercado 2 de la sección b). De nuevo, el objetivo es impulsar las ventas en el último mercado más de
lo que se restringieron en el primero. Con la venta de la máxima producción posible el monopolista
tiene la capacidad de emplear la mayor reserva de capital posible y así tener la utilidad más grande
posible.
Las trampas reguladoras no han impedido que los gobiernos de casi todas partes del mundo
sigan interviniendo en las decisiones del precio y la producción de monopolios naturales importan-
tes como el servicio eléctrico público y el de telefonía local. Si estas intervenciones hacen más bien
que mal, en términos puramente económicos sigue siendo una pregunta sin respuesta. Pero es claro
que parecen desempeñar una función psicológica importante a nombre de un público incómodo, y
con razón, por el hecho de no tener un amortiguador entre ellos y el único proveedor de un bien o
servicio crucial.

3. CONTRATO EXCLUSIVO PARA UN MONOPOLIO NATURAL


En el título de un artículo muy citado de Harold Demsetz, economista de la UCLA, se plantea la
apabullante pregunta: “¿Para qué regular un monopolio?”17 Su punto de vista era que aunque las
condiciones de los costos dictaran que un solo proveedor diera servicio a un mercado, de todas ma-
neras habría una competencia fuerte para ver quién lograría ser ese proveedor. En la propuesta de
Demsetz, el gobierno especificaría con detalle el servicio que desea proporcionar, como protección
contra incendios, colecta de basura, entrega postal, lo que sea, y después solicitaría a las compañías
privadas que enviaran sus cotizaciones para proporcionar ese servicio y el contrato sería para quien
cotizara más bajo.
Este esquema se ha probado con éxito en varios municipios de Estados Unidos. Por ejemplo,
la ciudad de Scottsdale, Arizona, tiene un contratista privado elegido de esta forma para ofrecer los
servicios de protección contra incendios. Oakland Scavenger Company, una empresa privada con
fines de lucro, recoge cada semana la basura de los residentes de Oakland, California, en lugar de los
recolectores de basura municipales. En ambos casos los costos en que se incurre por proporcionar el
servicio son de más o menos la mitad de lo que se paga por servicios similares que ofrecen de manera
directa los gobiernos municipales. Además, en el caso de la protección contra incendios, es contun-
dente la evidencia de que las reducciones en los costos no se logran a través de una menor calidad en
el servicio. Las compañías de seguros contra incendios con fines de lucro, cuya supervivencia depen-
de su capacidad para evaluar con precisión los riesgos, no cobran más por el seguro contra incendio
en Scottsdale que en las comunidades con departamento de bomberos municipal.18 La ventaja de
una contratación privada para la provisión de servicios monopólicos por naturaleza es que se toma la
producción de manos de los burócratas, que no son muy buenos para mantener bajos los costos.
Sin embargo, el científico político Elliott Sclar advierte que las ventajas de contratar servicios
públicos muchas veces son más aparentes que reales (vea nota a pie de página número 15). Por ejem-
plo, la mayoría de los contratos con proveedores privados contienen cláusulas de contingencia que
permiten tarifas más altas en caso de cambios imprevistos en las circunstancias. Es probable que los
empleados de gobierno que supervisan la celebración de contratos privados puedan ser incapaces

17
Harold Demsetz, “Why Regulate Monopoly?”, en Journal of Law and Economics, abril de 1968, pp. 55-65.
18
Para una extensa búsqueda de los estudios en los que se comparan costos públicos y privados, vea E. S. Savas, Privatizing the Public
Sector, Chatham, NJ: Chatham House Publishers, 1982.

12_CHAPTER 12.indd 402 3/6/09 8:04:02 PM


polítiCa púBliCa haCia un monopolio natural 403

de controlar esas tarifas adicionales. Aunado a ello está el problema de la corrupción. En la práctica,
a menudo los contratos públicos no son asignados a la empresa con la cotización más baja, sino a la
que paga el soborno más alto.
Aunque se haga caso omiso de estos problemas, es probable que la contratación privada en
muchos casos no sea una opción atractiva. Como el contrato debe especificar los detalles del servicio
que se va a proporcionar, debe ser en extremo minucioso en el caso de un servicio complejo como las
telecomunicaciones. Además, deben tomarse medidas acerca de cómo elegir a los contratistas nue-
vos. En el caso de los servicios públicos de electricidad, el cambio de contratistas implica de manera
necesaria la transferencia de una amplia y compleja variedad de equipos generadores y distribuido-
res. ¿A qué precio se debe vender este equipo? Para cuando se terminen de poner todos los puntos
sobre las íes, los contratos exclusivos para proporcionar el servicio de monopolio pueden ser tan
detallados como para ser indistinguibles de la regulación económica directa.

4. APLICACIÓN ENÉRGICA DE LAS LEYES ANTIMONOPOLIO


Un elemento importante en el arsenal de políticas relacionadas con un monopolio son las leyes an-
timonopolio de Estados Unidos. La más importante es la Ley Sherman (1890), que considera ilegal
“monopolizar o intentar monopolizar [...] cualquier parte del comercio entre los estados” y la Ley Cla-
yton (1914), una de cuyas disposiciones previene a las corporaciones contra la adquisición de acciones
de un competidor con el fin de “reducir sustancialmente la competencia o crear un monopolio”.
Al interpretar las leyes antimonopolio, el Departamento de Justicia de Estados Unidos ha desa-
rrollado lineamientos para prohibir las fusiones entre compañías en competencia cuya participación
combinada en el mercado supere cierta fracción predeterminada de la producción total de la indus-
tria. Estos lineamientos se aplican con grados de celo muy variables en diferentes administraciones
públicas. Como regla general, los demócratas han sido mucho menos tolerantes con las fusiones que
los republicanos.
En el caso de las industrias con curvas de costo promedio a largo plazo descendentes, el costo
de la producción es mucho más alto si se tienen los servicios de muchas empresas en vez de unas
cuantas. Quienes más defienden las leyes antimonopolio subrayan que las leyes no impiden la for-
mación de monopolios naturales. Pero como se verá en el capítulo 13, pueden posponer de manera
sustancial el momento en que se logren por completo las economías de escala.
Una respuesta a este problema sería aplicar las leyes antimonopolio para evitar sólo aquellas
fusiones en las que no se obtengan ahorros significativos en los costos. Pero el gobierno no se en-
cuentra en una buena posición para distinguir una de la otra. El Congreso estaba bastante consciente
de ello y descartó de manera explícita considerar el ahorro en los costos como una justificación para
permitir las fusiones. El resultado es que la política antimonopolio impide las fusiones aun cuando
conduzcan a reducciones sustanciales en los costos.

5. POLÍTICA LIBERAL HACIA LOS MONOPOLIOS NATURALES


Como quinta y última alternativa para manejar un monopolio natural, considere la posibilidad de
una política liberal laissez faire; es decir, dejar que el monopolista produzca la cantidad que elija y
la venda al precio que el mercado soporte. Las objeciones obvias a esta política son las dos con las
que se inició el presente análisis: los problemas de equidad y eficiencia. Sin embargo, en esta sección
se verá que puede haber ciertas circunstancias en las que estos problemas tienen una importancia
mínima.
Considere en particular a un monopolista natural que usa el modelo de obstáculo para discri-
minar los precios. Para que el análisis sea simple, suponga que cobra un precio normal y uno de
descuento, este último disponible para clientes que superen cierto obstáculo, como enviar por co-
rreo una forma de reembolso. ¿De qué manera la presencia de este dispositivo de precio diferencial
influye en las objeciones de equidad y eficiencia ante el monopolio natural?
Tome en cuenta primero la objeción de eficiencia. Debe recordar que el problema es que el mo-
nopolista de precio único cobra un precio por arriba del costo marginal, el cual excluye del mercado
a muchos compradores potenciales, quienes tasan el producto muy por encima del valor que tienen
los recursos necesarios para producirlo.

12_CHAPTER 12.indd 403 3/6/09 8:04:03 PM


404 Capítulo 12 monopolio

Para efectos de ejemplificarlo, considere un monopolista natural con una curva de costo total
dada por F + MQ y una curva de demanda lineal definida por P = A – BQ. En la figura 12.22a se mues-
tran las curvas de demanda y de costo marginal de dicho monopolista. Si él es un maximizador de
utilidades de precio único, producirá Q* y venderá a P*. Pero si es capaz de cobrar a los compradores
un precio a lo largo de la parte superior de la curva de demanda y un precio menor a los demás
compradores (figura 12.22b), su estrategia de maximización de utilidades será vender QH al precio de
PH y QL al precio de PL.19

FIGURA 12.22
Pérdidas de eficiencia $/Q $/Q
en monopolios de A A
precio único y de dos Monopolio de precio único Monopolio de dos precios
precios
al poder ofrecer un precio PH
de descuento a la mayor
porción de la curva de P*
demanda, el monopolista
de dos precios (sección PL
b) expande el mercado, W
provocando así una pérdida Z
M M
de eficiencia mucho menor
(área Z, sección b) que en
el caso del monopolista Q Q
de precio único (área W, Q* QH QH + QL
sección a). a) b)

Observe que la pérdida de eficiencia relacionada con el monopolista de dos precios (el excedente
del consumidor que se pierde, que es el área del triángulo Z de la sección b) es mucho menor que la
pérdida correspondiente para el monopolista de precio único (el área del triángulo W de la sección a).
En general, cuanto más fino divida su mercado el monopolista conforme al modelo de obstácu-
lo, más pequeña será la pérdida de eficiencia. Como se observó antes, en casi todas las empresas es

19
Para el monopolista de precio único, la función de utilidades está definida por
Π1 = (A – BQ)Q – F – MQ.
La condición de primer orden para un máximo se obtiene mediante
d ∏1
= A – 2 BQ – M = 0,
dQ
que produce una cantidad de maximización de utilidades de Q' = (A – M)/2B y un precio correspondiente de P' = (A + M)/2.
La función de utilidad para el monopolista de dos precios es, por el contrario,
Π2 = (A – BQH)QH + (A – BQH – BQL)QL – F – M(QH + QL).
Las condiciones de primer orden para un máximo están dadas por
∂ ∏2
= A – 2 BQ H – BQ L – M = 0
∂QH
y
∂ ∏2
= A – BQH – 2 BQL – M = 0,
∂QL
que se puede resolver con
A–M A + 2M 2A + M
QH = = QL y PL = y PH = .
3B 3 3

12_CHAPTER 12.indd 404 3/6/09 8:04:06 PM


polítiCa púBliCa haCia un monopolio natural 405

común ver no uno sino una variedad de precios de descuento, cada uno con una serie de restricciones
(cuanto mayor el descuento, más estricta es la restricción). Dada la amplia libertad que tienen muchas
empresas para expandir sus mercados mediante la fijación de precios de obstáculo, la importancia del
problema de eficiencia del monopolio natural suele ser secundaria.
¿Qué hay del problema de equidad? Para empezar, ¿cuál es el problema? La percepción popular
es que el monopolista transfiere los recursos de personas que los necesitan con desesperación (en
particular los consumidores de bajos ingresos) a otras que, de antemano, tienen más de lo que ne-
cesitan (sobre todo accionistas adinerados). A continuación se verá que, definido así, el problema a
veces es menos grave de lo que parece.
La pregunta más general de lo que constituye una distribución equitativa de los recursos de la
sociedad es filosófica y profunda, más allá del alcance de lo que se discute aquí. Sin embargo, por lo
menos se puede decir que ninguna empresa tiene derecho a adquirir, a la fuerza o por coerción, el
poder de obtener recursos excesivos de otras personas. Pero suponga que el monopolista se volvió
el único vendedor en su mercado a través de medios totalmente benévolos, algo que es probable.
Como monopolista natural, por definición, sus costos son más bajos de lo que serían si el mismo
mercado fuese atendido por otras empresas. Y quizá su servicio amable y alegre también ayude a
que afiance su posición. ¿Puede entonces generar una injusticia si cobra precios por arriba del costo
marginal?
Sin duda, a los consumidores les daría más gusto pagar sólo el costo marginal de la producción.
Pero éste es inferior al costo promedio en un monopolio natural, y no puede ser que todos paguen un
costo marginal y que el proveedor se mantenga en el negocio. Si acaso, algunos consumidores pueden
pagar precios cercanos al costo marginal, aunque otros tendrán que pagar mucho más. Aún así, si el
monopolista obtiene una utilidad económica, se sabe que, en promedio, los compradores pagan más
que el costo de los recursos requeridos para brindarles el servicio. ¿Cómo se puede defender esto en
nombre de la equidad?
Antes se vio que los precios de obstáculo hacen que el reparto del monopolio sea más eficiente.
Sería una exageración decir que el modelo de obstáculo hace que las utilidades de monopolio parez-
can ser por completo justas. Pero sí mitiga algunas de las objeciones más graves que se le achacan.
Primero, tome en cuenta la fuente de un dólar determinado de utilidad de monopolio. ¿De qué
comprador provino este dólar? Es fácil demostrar que no proviene del comprador con un precio de
descuento. Los precios de descuento típicos varían de 15 a 50% del llamado precio normal y rara
vez más de la mitad de los compradores pagan el precio de descuento. Si se toma como ejemplo un
caso en que el precio de descuento es de 30% y la mitad de los compradores lo reciben, se ve que el
ingreso del monopolista se reduciría 15% si todos pagaran el precio de descuento. Pocas empresas
continuarían siendo redituables ante una disminución de 15% del ingreso total.
Se concluye que si el monopolista obtiene una utilidad económica, la fuente de ésta es el com-
prador que paga el precio normal. El hecho de que este comprador podría haber pagado un precio
de descuento si hubiera estado dispuesto a superar el requisito del obstáculo indica que la carga que
se le impone no es superior al problema de superar el obstáculo. Obviamente esto no es lo mismo
que decir que quien compra al precio normal da una contribución voluntaria para beneficiar al mo-
nopolista, pero por lo menos aligera en parte la desagradable idea de que los clientes son víctimas del
cruel monopolista.
En lo que respecta a la fuente de la utilidad del monopolio. ¿Que hay de su disposición? ¿Quién
la recibe? Si se supone una tasa de interés sobre la renta corporativa de 40%, 40 centavos de cada dó-
lar de utilidades del monopolio van a la Tesorería de Estados Unidos. El resto se paga a los accionis-
tas, ya sea directamente a través de dividendos o indirectamente con la reinversión en la compañía.
Es un hecho, el ingreso promedio de los accionistas es superior al de los ciudadanos como un todo.
Pero hay muchos accionistas de bajos ingresos en Estados Unidos. Por ejemplo, la mayoría de los
fondos de pensiones para empleados se invierte en el mercado accionario, al igual que los valores
en cartera de aseguradoras privadas de muchos individuos de bajos ingresos. Así que una fracción
considerable de cualquier dólar de utilidad de monopolio terminará en manos de accionistas de
bajos ingresos.
Pero, para tomar el peor caso posible desde el punto de vista de la distribución, suponga que lo
que queda de un dólar de utilidad de monopolio va completo a la residente más adinerada de Man-

12_CHAPTER 12.indd 405 3/6/09 8:04:07 PM


406 Capítulo 12 monopolio

hattan. Si ella paga un impuesto federal sobre la renta a una tasa de 33% sobre los 60 centavos que el
gobierno federal no ha recaudado, le quedan 40 centavos. Los impuestos estatales y locales sobre la
renta y la venta reclamarán siete centavos más, dejando sólo 33 centavos en manos de la accionista
adinerada.
En resumen, la fuente de un dólar de utilidad para el monopolio es entonces quien compra a
precio normal, alguien que pudo haber pagado un precio de descuento si se hubiera tomado una
pequeña molestia extra. De ese dólar, 60 centavos van al tesoro federal y otros siete centavos a los
gobiernos estatales y locales. Por lo tanto, la disposición de más de dos terceras partes del dólar
está sujeta al control gubernamental. El resto se convierte en ingreso en manos de los accionistas
que, para empezar, al menos parte de ellos son de bajos ingresos. Así que en definitiva no queda
establecido que la utilidad económica relacionada con el monopolio natural genera inequidades de
distribución del tipo que usualmente se percibe.
Claro está que los obstáculos rara vez son perfectos. Es inevitable que eliminen a algunos con-
sumidores que no comprarán a precio normal. Y muchas veces, se deben gastar recursos reales para
superar dichos obstáculos. Puede ser que enviar por correo un cupón de reembolso no requiera mu-
cho tiempo, pero este bien podría aprovecharse mejor. Y por lo menos en algunos casos la evasión de
impuestos impide que el gobierno cobre tanto como lo especifican los tabuladores fiscales.
¿Entonces cuál puede ser la conclusión acerca de este breve análisis de las cinco opciones de
políticas para manejar un monopolio natural? La respuesta corta es que cada una tiene sus proble-
mas. Ninguna elimina por completo las dificultades que surgen cuando un solo vendedor atiende al
mercado. A veces la solución menos costosa es la contratación competitiva, otras lo es la propiedad
estatal directa. La regulación representa una función importante en industrias específicas, sobre todo
en las de servicios públicos tradicionales. Y a pesar de sus muchos defectos, las leyes antimonopolio
funcionan para el público al desalentar la fijación de precios y otras prácticas que afectan la compe-
tencia. Pero en algunos casos, en particular aquéllos en los cuales el monopolista ha ideado medios
para segmentar de manera importante el mercado, puede ser que la mejor opción sea no intervenir.

¿LOS MONOPOLIOS SUPRIMEN LA INNOVACIÓN?


Uno de los temas de conversación más continuos entre los entusiastas de la conspiración económica
es la noción de que los monopolistas privan a los consumidores de un espectro de innovaciones
tecnológicas en extremo variables. ¿Quién no ha escuchado, por ejemplo, de los fabricantes de focos
que conspiraron para evitar que nuevos diseños revolucionarios más duraderos llegaran al mercado?
¿La supresión de la innovación es otro costo de monopolio más que se debe tener en considera-
ción en este análisis de las opciones de política pública? Como se establece en el ejemplo a continua-
ción, la lógica de maximización de utilidades postula que los monopolistas no siempre pueden estar
tan ansiosos por suprimir la innovación.

EJEMPLO 12.4 Suponga que el diseño del foco actual dura 1 000 horas. Ahora, el monopolista de focos des-
cubre cómo hacer que un foco dure 10 000 horas por el mismo costo de producción por
unidad. ¿El monopolista introducirá en el mercado el foco nuevo?

Suponga que se mide la cantidad que produce el monopolista no como focos, sino como el número
de foco-horas de iluminación. De manera que si el costo de producir el diseño actual es de, por
ejemplo, 1 dólar/foco-hora, entonces el costo del nuevo diseño es de sólo 0.10 dls./foco-hora. En la
figura 12.23, D representa la curva de demanda del mercado para la iluminación y MR es la curva de
ingreso marginal asociada.
Observe que el precio de maximización de utilidades y la cantidad para el diseño actual, cuyo
costo marginal es de 1 dólar/foco-hora, son P1 y Q1, respectivamente. Para el diseño nuevo, cuyo cos-
to marginal es de 0.10 dl./foco-hora, el precio de maximización de utilidad y la cantidad son P2 y Q2.
La utilidad del monopolista con el diseño actual es el área del rectángulo FGHK. Y como la utilidad
del monopolista es superior con el diseño nuevo, tiene todos los incentivos para hacer que el diseño

12_CHAPTER 12.indd 406 3/6/09 8:04:07 PM


resumen 407

FIGURA 12.23
Precio ¿Los monopolios
($/foco-hr) suprimen la innovación?
el costo de producir el
nuevo foco eficiente a 0.10
dl./foco-hora es apenas una
P1 A B
décima parte del costo de
producir el diseño actual
P2 F G de 1 dólar/foco-hora.
1.00 C Como las utilidades del
E monopolista con el diseño
eficiente (área FGHK)
MR D superan sus utilidades
con el diseño actual (área
K ABCE), ofrecerá el diseño
0.10 H
Kilohoras de iluminación nuevo.
Q1 Q2

esté disponible. De hecho, como recordará, hace unos años se anunció la disponibilidad de un nuevo
foco así de eficiente.
Este ejemplo no implica que los incentivos del monopolista para introducir innovaciones siem-
pre serán iguales a los de una empresa competitiva en todas partes. Pero lo debe prevenir contra la
aceptación no crítica de afirmaciones de que los monopolistas siempre privan a los consumidores de
los beneficios de la tecnología más reciente.

RESUMEN
• Monopolio es el nombre que se da a la estructura de mercado • A diferencia del caso de la competencia perfecta, el equilibrio
en la que una única empresa atiende a todo ese mercado. Cinco del monopolio no agota las posibles ganancias derivadas del
factores, solos o combinados, dan lugar al monopolio: 1) con- intercambio. En general, el valor para la sociedad de una uni-
trol sobre insumos clave, 2) economías de escala, 3) patentes, dad adicional de producto supera el costo para el monopolista
4) economías de red, 5) licencias gubernamentales. A la larga, de los recursos necesarios para elaborarlo. Este resultado se
y por mucho, el factor más importante son las economías de interpreta en el sentido de que el monopolio es menos eficien-
escala, en parte porque también explican las economías de red te que la competencia perfecta; pero es una interpretación de
y las licencias gubernamentales. poca importancia práctica, porque las condiciones que dan
• Como el monopolista es el único vendedor del mercado, su lugar al monopolio (en particular, las economías de escala en
curva de demanda es la curva de demanda del mercado con la producción) casi nunca son compatibles con las que pide la
pendiente negativa. A diferencia del competidor perfecto, que competencia perfecta.
puede vender cuanto quiera al precio del mercado, el mono- • En el capítulo se centró el enfoque sobre la cuestión de cómo
polista debe reducir el precio para expandir su producción. debe tratar el gobierno a los monopolios naturales: mercados
La regla del monopolista para maximizar las utilidades es la caracterizados por curvas de costo promedio a largo plazo con
misma que aplican las empresas perfectamente competitivas: pendiente negativa. Se consideraron cinco políticas públicas al-
expandir la producción, si la ganancia en los ingresos (ingreso ternativas: 1) empresas paraestatales, 2) empresas privadas con
marginal) supera el aumento de los costos (costo marginal) y regulación oficial de precios, 3) licitaciones en competencia de
contraerla si la pérdida del ingreso es menor que la reducción empresas privadas para tener el derecho exclusivo a prestar
de los costos. La diferencia fundamental es que para el mono- un servicio, 4) aplicación enérgica de las leyes antimonopolio
polista el ingreso marginal es menor que el precio, pero igual dirigidas a evitarlos, y por último 5) una política por completo
al precio del competidor perfecto. liberal. Todas las alternativas tienen problemas, y la mejor
• Cuando el monopolista puede vender en varios mercados, política será diferente en distintas circunstancias. La postura
distribuye la producción entre éstos de manera que el ingreso liberal es más atractiva en mercados en los que el monopolista
marginal sea el mismo en todos. En este caso otra vez la lógi- puede emplear el modelo del obstáculo para diferenciar los
ca familiar del análisis costo-beneficio proporciona un marco precios. Dejar que los compradores decidan por sí mismos si
conveniente para analizar la decisión de la empresa en cuanto se hacen acreedores a un descuento aminora las objeciones a
a modificar su funcionamiento presente. la eficiencia y la equidad de un monopolio natural.

12_CHAPTER 12.indd 407 3/6/09 8:04:08 PM


408 Capítulo 12 monopolio

pREgUNtaS dE REpaSo
1. ¿Cuáles son los cinco factores que dan lugar a un monopolio? 7. Suponga que la elasticidad de la demanda es ∈ = –3. ¿En
A la larga, ¿por qué las economías de escala son el factor más cuánto excederá el precio de un monopolista que maximiza
importante? utilidades al costo marginal? Compare este margen de precio
2. Si Estados Unidos tiene cientos de cementeras pero en un pue- sobre el costo marginal con el que se presenta en la competen-
blito hay sólo una, ¿ésta es un monopolista? Explique. cia perfecta.
3. ¿Cuándo es el ingreso marginal de un monopolista menor que 8. Falso o verdadero. Un impuesto único a un monopolista siem-
el precio? Explique. pre aumenta el precio que cobra y disminuye la cantidad de
4. ¿Por qué un monopolista que maximiza utilidades nunca producto que vende.
produce en la parte inelástica de la curva de demanda? ¿Un 9. Falso o verdadero. Si un monopolista se enfrenta a una curva de
monopolista que maximiza sus utilidades produce alguna vez demanda perfectamente horizontal, la pérdida de peso muer-
en la parte inelástica de la curva de demanda? to de la economía es cero.
5. ¿Por qué un nivel de producción en el cual IM interseca CM 10. ¿Qué fuerzas van en contra de la ineficiencia X en los monopo-
desde abajo nunca es el nivel de máximas utilidades? lios de particulares?
6. ¿Qué efecto tendrá el establecimiento de un impuesto de 50% 11. ¿Por qué el método de obstáculo para la discriminación de
sobre las utilidades económicas en las decisiones de precio y precios aligera los problemas de eficiencia y equidad asociados
producción de un monopolista? (Sugerencia: recuerde que el con un monopolio?
objetivo supuesto es escoger el nivel de producción que maxi-
mice las utilidades económicas.)

pRoblEMaS
1. Usted es un asesor independiente en maximización de utilidades que se especializa en monopolios.
En estas épocas, cinco monopolios de precio único y maximización de utilidades buscan su consejo, y
aunque la información que le dieron es incompleta, sus conocimientos expertos le permiten ir hacia
atrás y hacer una recomendación clara para cada caso. Seleccione una de las siguientes recomenda-
ciones para cada empresa en el corto plazo.
a) Mantener el nivel actual de producción.
b) Aumentar la producción.
c) Reducir la producción.
d) Cerrar.
e) Vuelva a calcular las cifras, porque las que le dieron no pueden estar correctas.

Empresa P IM IT Q CT CM CTP CVP Su recomendación


A 3.90 3.00 2 000 7 400 2.90 3.24
B 5.90 10 000 5.90 4.74 4.24
C 9.00 44 000 4 000 9.00 11.90 10.74
D 35.90 37.90 5 000 37.90 35.90
E 35.00 3 990 1 000 3 300 a valor 23.94
mínimo

2. Un monopolista tiene una curva de demanda dada por P = 100 – Q y una curva de costo total dada
por CT = 16 + Q2. La curva asociada de costo marginal es CM = 2Q. Calcule la cantidad y el precio
que maximizan las utilidades para el monopolista. ¿A cuánto ascenderán las utilidades económicas
que ganará?
3. Ahora suponga que el monopolista del problema 2 tiene una curva de costo total dada por CT = 32
+ Q2. La curva correspondiente del costo marginal es todavía CM = 2Q, pero los costos fijos se du-
plicaron. Calcule la cantidad y el precio que maximizan las utilidades para el monopolista. ¿A cuánto
ascienden las utilidades económicas que ganará?

12_CHAPTER 12.indd 408 3/6/09 8:04:08 PM


proBlemas 409

4. Ahora suponga que el monopolista del problema 2 tiene una curva de costo total dada por CT = 16
+ 4Q2. La curva correspondiente del costo marginal es CM = 8Q y los costos fijos vuelven al nivel
original. Calcule la cantidad y el precio que maximizan las utilidades para el monopolista. ¿A cuánto
ascienden las utilidades económicas que ganará?
5. Suponga ahora que el monopolista del problema 2 también tiene acceso a un mercado extranjero en
el que puede vender cualquier cantidad que desee a un precio constante de 60. ¿Cuánto venderá en el
mercado extranjero? ¿Cuáles serán la cantidad y el precio nuevos en el mercado original?
6. Ahora suponga que el monopolista del problema 2 tiene una curva de costo marginal a largo plazo
de CM = 20. Calcule la cantidad y el precio que maximizan las utilidades para el monopolista. Calcu-
le la pérdida de eficiencia de este monopolio.
7. Suponga que un monopolista discriminador perfecto tiene una demanda del mercado de P = 100
– 10Q y un costo marginal constante CM = 20 (sin costos fijos). ¿Cuánto vende el monopolista? ¿Qué
utilidades gana? ¿Cuál es la tarifa máxima de licencia por periodo que el gobierno puede cobrar a la
empresa sin que ésta tenga que salir del negocio?
8. La demanda de adultos mayores de exhibiciones en el cine local tiene una elasticidad precio constan-
te igual a –4. La curva de demanda de los demás clientes tiene una elasticidad precio constante de
–2. Si el costo marginal por cliente es de un dólar por proyección, ¿cuánto debe cobrar el cine a los
miembros de cada grupo?
9. Durante la guerra Irán-Irak, los mismos traficantes de armas vendían a los dos bandos del conflicto.
En esta situación, hubiera podido ofrecerse un precio diferente a cada uno porque había poco peligro
de que el país al que se le vendiera más barato, comerciara las armas con su enemigo para sacar
una utilidad por el diferencial de precios. Suponga que un traficante de armamento francés tiene el
monopolio de los misiles aire-mar Exocet y está dispuesto a venderlos a los dos bandos. La demanda
de los Exocet en Irak es P = 400 – 0.5Q y la de Irán, P = 300 – Q, donde P está dado en millones de
dólares. El costo marginal de los Exocet es CM = Q. ¿Qué precio le cobrará a cada país?
10. Si ha tenido que ir a la tienda de autoservicio alguna tarde entre semana, habrá notado que gente
mayor recorre con lentitud los pasillos buscando entre sus cupones de descuento uno que corres-
ponda a sus compras. ¿Cómo se explica este comportamiento con el modelo del obstáculo para la
discriminación de precios?
11. El precio de un monopolista es de 10 dólares. A este precio el valor absoluto de la elasticidad de la
demanda es 2. ¿Cuál es el costo marginal del monopolista?
12. Suponga que el gobierno impone un tope de precios a un monopolista (un límite superior al precio

que puede cobrar). Sea P el tope de precios y suponga que el monopolista no incurre en costos de
producción. Falso o verdadero: si la curva de demanda que enfrenta el monopolista es inelástica en el

precio P , el monopolista no estaría mejor si el gobierno quitara el tope de precios.
13. El New York Times, un periódico que maximiza sus utilidades, tiene un esquema de demanda de anun-
cios con pendiente negativa. Cuando el periódico se anuncia en sus propias páginas (por ejemplo, un
anuncio que reza: “Lea a Maureen Dowd en el Times del domingo”), ¿el costo de oportunidad de un
anuncio de dimensiones dadas es el precio que cobra a los anunciantes externos? Explique.
*14. El monopolista Crazy Harry tiene una curva de costo total dada por CT = 5Q + 15. Fija dos precios
para su producto, un precio normal, PH, y uno de descuento, PL. Todos tienen el derecho de comprar
el producto a PH. Para tener derecho a comprarlo a PL, es necesario presentar al empleado un recorte
del último anuncio de Crazy Harry en el periódico. Suponga que los únicos compradores que pre-
sentan el anuncio son los que no están dispuestos a comprar el producto a PH.
a) Si la curva de demanda de Crazy Harry está dada por P = 20 – 5Q, ¿cuáles son los valores de PH y PL
que maximizan las utilidades?
b) ¿Qué utilidades económicas gana Crazy Harry?
c) ¿Qué utilidades habría obtenido si se hubiera visto obligado a cobrar el mismo precio a todos los
compradores?
d) ¿Los compradores están mejor o peor como resultado de que Harry pueda cobrar los dos precios?
15. Una escritora firmó un contrato en el cual el editor promete pagarle 10 000 dólares más 20% de las
entradas brutas por la venta de su libro. Falso o verdadero: si el editor y la escritora sólo se interesan

* Este problema se resuelve con más facilidad con el método de cálculo que se describe en la nota 19.

12_CHAPTER 12.indd 409 3/6/09 8:04:09 PM


410 Capítulo 12 monopolio

por su propio rendimiento económico del proyecto, la escritora preferiría que el libro costara
más que lo que propone el editor.
16. Una cineasta firmó un contrato por el que el estudio de producción promete pagarle 1 millón de
dólares más 5% de los ingresos del estudio sobre la renta de la película, la cual tiene costos de pro-
ducción y distribución fijos. Falso o verdadero: si la cineasta y el estudio sólo se interesan por su propio
rendimiento económico del proyecto, la directora preferiría un precio de alquiler de la película me-
nor que el que quiere el estudio.

RESpUEStaS a loS EjERcicioS dEl capítUlo


12.1.
IT ($/unidad de tiempo)
1 500

1 000

500

Q
10 20 30 40 50

12.2.
$/unidad del producto

100

Q
0 25 50
IM

12.3. CM = 40 = 100 – 4Q, lo cual se resuelve para Q* = 15, P* = 100 – 2Q* = 70.
12.4. El nivel de producción que maximiza las utilidades de un monopolista de precio único ocurre cuan-
do IM = CM. El ingreso marginal es igual al costo marginal en Q = 15 y Q = 34, pero en Q = 34
interseca con el ingreso marginal desde arriba y por eso es máxima. Sin embargo, incluso en Q =
34 el precio no paga el costo variable promedio (66 = P < CVP = 72). La curva de costo variable
promedio se encuentra arriba de la curva de demanda en todas partes (vea figura), de manera que
lo mejor que puede hacer la empresa es ganar utilidades iguales al negativo de los costos fijos. Así, la
cantidad óptima es Q = 0: ¡la empresa debe cerrar!

Precios (P)
150

CMC
100
86 CVP

66
D

IM
Cantidad(Q)
0 15 34 50

12_CHAPTER 12.indd 410 3/6/09 8:04:12 PM


respuestas a los ejerCiCios del Capítulo 411

12.5. IM1 = 10 – 2Q1 (sección de la izquierda) y IM2 = 20 – 2Q2 (sección central), de modo que la suma
horizontal de las curvas IM está dada por ΣIM (sección de la derecha). La cantidad que maximiza las
utilidades es 13 unidades, cuatro de las cuales deben venderse en el mercado 1 y las nueve restantes
en el mercado 2. Los precios que maximizan las utilidades son P*1 = 6 y P*2 = 11.

$/unidad de $/unidad de $/unidad de


producción producción producción
20

IM2

P*2 = 11 10
10
IM1
P*1 = 6 IM = ΣIMi

2 2
2 CM
Q1 Q2 Q
5 10 5 10 20 5 15
Q*1 = 4 Q*2 = 9 Q* = 13

12_CHAPTER 12.indd 411 3/6/09 8:04:14 PM


12_CHAPTER 12.indd 412 3/6/09 8:04:14 PM
Capítulo

13
COMPETENCIA IMPERFECTA:
ENFOQUE DE LA
TEORÍA DE JUEGOS

S
e cuenta un chiste sobre tres abogados y tres economistas que se encuentran una mañana
cuando llegan a la estación de la Calle 30 en Filadelfia para tomar el tren a Nueva York,
donde van a asistir a la reunión anual de la Asociación de Derecho y Economía. Los aboga-
dos compran boletos de ida y, sorprendidos, ven que los economistas, que están en la fila contigua,
compran un boleto para los tres. “¿Qué pretenden?”, pregunta un abogado a sus colegas mientras los
seis hombres toman sus asientos en el tren.
Poco después de la salida del tren, los abogados ven que el cobrador perfora los boletos en el
vagón de enfrente. En ese momento los economistas se meten en uno de los baños de la parte pos-
terior. El cobrador llega, toma los boletos de los abogados y, de camino al siguiente vagón, toca en la
puerta del baño ocupado: “Su boleto, por favor.” La puerta se abre apenas y se estira una mano con el
boleto, que el cobrador perfora con toda seriedad y lo devuelve.
Cuatro días después los hombres llegan a la estación Penn, para su viaje de regreso. Inspirados
por los economistas, los abogados compran un solo boleto y se felicitan por ser tan listos. Pero se
quedan perplejos al ver que los economistas no compran ningún boleto.

413

13_CHAPTER 13.indd 413 3/6/09 8:05:18 PM


414 Capítulo 13 CoMpEtENCIa IMpERFECta: ENFoQuE DE la tEoRía DE JuEGoS

Los seis toman el tren y en cuanto ven que el cobrador se acerca, los abogados se esconden en
uno de los baños. Al mismo tiempo los tres economistas se ponen de pie y se van a la parte posterior
del vagón. Dos se meten en el otro baño, mientras que el tercero toca la puerta del baño de los
abogados: “Su boleto, por favor.” Uno de los abogados saca el boleto, el economista lo toma y se va
con sus colegas.

VISIÓN PRELIMINAR DEL CAPÍTULO


Como enseña la parábola de los abogados y los economistas, los beneficios de una acción dependen
no sólo de la acción misma, sino también de cómo se relaciona con las acciones emprendidas por
otros en el mismo ambiente. En los capítulos anteriores vio que quienes toman las decisiones econó-
micas enfrentan un ambiente que es pasivo en esencia. Por ejemplo, en el ambiente perfectamente
competitivo del capítulo 11 se supone que las empresas ignoran las acciones de sus adversarias. En
el ambiente monopolista del capítulo 12 la empresa no tiene que preocuparse por ninguna compe-
tencia. Tanto la competencia perfecta como el monopolio representan formas idealizadas. Son útiles
para generar conocimientos sobre tendencias, pero en la práctica se encuentran pocas veces o nunca.
En este capítulo, la tarea es describir y explorar las formas híbridas de la organización industrial con
las que se trata todos los días, a saber, oligopolio y competencia monopólica.
Para comenzar se presentan los conceptos elementales de la teoría matemática de juegos. En-
seguida se aplicarán al análisis del comportamiento de los oligopolistas y se van a comparar varios
modelos simples en los que las empresas formulan supuestos alternativos sobre la conducta de sus
rivales. Después de considerar un modelo tradicional de competencia monopólica, se expondrá un
modelo espacial simple de competencia monopolista en el cual los clientes tienen preferencia por
determinadas características sobre las situaciones y los productos. Como se verá, en esta situación las
empresas compiten con más intensidad por los compradores de los productos que son más parecidos
a los suyos.

INTRODUCCIÓN A LA TEORÍA DE JUEGOS


Al precio de equilibrio del mercado la demanda mundial de café es inelástica con respecto al precio.
Así, cuando una helada destruye la mitad de la cosecha de un país que exporta una proporción gran-
de del café del mundo, su precio aumenta tanto que los agricultores obtienen ganancias mayores
que las que habrían tenido sin helada. Si los cafetaleros ganaran más vendiendo menos café, ¿por
qué tienen que esperar a que una helada destruya la mitad de sus siembras? ¿Por qué no plantar
inicialmente menos café?
Desde luego, la respuesta es que aunque sea para bien de los agricultores en conjunto reducir la
producción, ninguno individualmente tendría jamás incentivos para hacerlo. A cada granjero, en lo
individual, le va mejor si todos, excepto él, producen menos. Así, vende el mismo volumen que antes
pero más caro. En una industria competitiva como la del café, la existencia de muchos productores
independientes hace difícil montar conspiraciones para reducir la producción.
Esta colusión es menos difícil de conseguir en los oligopolios, es decir, en industrias atendidas
por pocas empresas. Pero aunque a veces coludirse beneficia a las empresas de tales industrias, re-
sulta sorprendentemente difícil de sostener. En efecto, un tema constante de la economía de los
oligopolios es que lo que a una empresa le conviene en lo individual resulta dañino para los intereses
de las empresas tomadas en conjunto.
El problema básico que enfrentan los oligopolios en colusión tiene la misma estructura que
el dilema del preso que vio en el capítulo 7. Recuerde que en la historia original que se usó para
ejemplificar el dilema, dos presos están en celdas separadas por un delito grave que sí cometieron.
Sin embargo, el fiscal sólo tiene pruebas contundentes para demostrar un delito menor, cuyo castigo
es, por ejemplo, un año de cárcel. A cada preso se le dice que si uno confiesa mientras el otro guarda
silencio, el que confiese quedará libre mientras el otro pasará 20 años en la cárcel. Si los dos confiesan
tendrán una sentencia intermedia de, por decir un número, cinco años. Estas posibilidades se resu-
men en la tabla 13.1. Los presos no pueden comunicarse entre ellos.

13_CHAPTER 13.indd 414 3/6/09 8:05:19 PM


INtRoDuCCIóN a la tEoRía DE JuEGoS 415

TABLA 13.1
El dilema del prisionero

la estrategia
prisionero Y dominante de cada
Confiesa Guarda silencio preso es confesar.
pero si los dos
Confiesa 5 años 0 años para X confiesan están
para los dos 20 años para Y peor que si hubieran
prisionero X guardado silencio.
Guarda silencio 20 años para X 1 año
0 años para Y para los dos

Situaciones como la del dilema del prisionero se pueden analizar con la teoría matemática de
juegos desarrollada por John von Neumann y Oskar Morgenstern en la década de 1940.1 La teoría
empieza con una descripción de tres elementos comunes a todos los juegos: 1) los participantes,
2) la lista de estrategias posibles y 3) las recompensas que correspondan a cada combinación de estra-
tegias. En la tabla 13.1 los dos participantes del juego son el preso X y el preso Y. Cada uno tiene dos
estrategias: confesar o guardar silencio. Las recompensas de cada combinación de estrategias son las
condenas que reciben, como se resume en la matriz de ganancias de la tabla 13.1.
Algunos juegos, como el dilema del prisionero, tienen una estrategia dominante, que produce estrategia dominante en
los mejores resultados sin importar qué estrategia siga el contrincante. En el dilema del prisionero, un juego, la estrategia que
la estrategia dominante es confesar. Haga lo que haga Y, X tiene una condena más leve si confiesa: si produce mejores resultados,
Y también confiesa, X es condenado a cinco años en lugar de 20; y si Y guarda silencio, X queda libre sin importar la estrategia
elegida por el oponente.
en lugar de pasar un año en la cárcel. Las recompensas son perfectamente simétricas, así que a Y tam-
bién le conviene confesar, sea lo que sea que haga X. La dificultad es que cuando los dos se muestran
egoístas empeoran su situación. Así, cuando los dos confiesan son condenados a cinco años en lugar
del año que estarían en la cárcel si ambos callaran.
Para ilustrar la analogía entre el dilema del prisionero y el problema que enfrentan los oligopo-
listas que tratan de coludirse, piense en dos empresas que son las únicas proveedoras de agua mineral
de cierto mercado. Suponga que la curva de demanda del mercado asume la forma específica P =
20 – Q y que cada empresa puede producir agua mineral de su propio manantial a un costo marginal
nulo. Suponga también que las dos empresas piensan en llegar a un acuerdo según el cual cada una
produzca la mitad del volumen del monopolio y la venda al precio de monopolio. Para la curva de
demanda específica supuesta, la cantidad del monopolio (es decir, aquella en la cual IM = CM = 0)
es 10 y el precio de monopolio también es 10. Si las empresas aceptan y cumplen tal acuerdo, cada
una venderá cinco unidades a un precio de 10, de lo que sacarán utilidades de 50. Según un criterio
estricto de utilidades, no hay ninguna posibilidad con la que cualquiera de las empresas esté mejor.
Pero esto no garantiza que las empresas respeten el acuerdo. Observe que la recompensa de
cada una depende de la combinación de comportamientos que escojan. Cada empresa tiene dos
opciones, a saber: respetar el acuerdo o cancelarlo. Para fines de la exposición, suponga que retirarse
significara rebajar el precio una unidad, de 10 a nueve. ¿Qué pasa si una empresa respeta el acuerdo y

1
Vea John von Neumann y Oskar Morgenstern, Theory of Games and Economic Behavior, 3a. ed., Princeton, NJ, Princeton University
Press, 1953.

13_CHAPTER 13.indd 415 3/6/09 8:05:19 PM


416 Capítulo 13 CoMpEtENCIa IMpERFECta: ENFoQuE DE la tEoRía DE JuEGoS

la otra no? Como las dos venden productos idénticos, la que renuncia se queda con todo el mercado
debido a su precio más bajo. Vender 11 unidades a precio de 9 le dará utilidades de 99. La empresa
cooperadora no vende su producción y por lo tanto sus utilidades son nulas.
Si las dos empresas incumplen el acuerdo, se dividen las 11 unidades de producción vendidas al
precio de 9 y tendrán ganancias de 49.50. Como cada una tiene dos opciones (cooperar y cobrar 10,
como acordaron, o renunciar y cobrar 9), hay cuatro combinaciones de conductas posibles. Éstas y
las utilidades que produce cada una se resumen en la tabla 13.2.

TABLA 13.2
Utilidades por cooperar y por incumplir

Empresa 1 la estrategia
dominante es que las
dos incumplan, porque
Coopera Incumple
con eso obtienen
(P = 10) (P = 9)
más utilidades, al
margen de la opción
Coopera Π1 = 50 Π1 = 99 que escoja la otra.
(P = 10) Π2 = 50 Π2 = 0 pero cuando las
Empresa 2 dos incumplen,
Incumplir Π1 = 0 Π1 = 49.50 ganan menos que si
(P = 9) Π2 = 99 Π2 = 49.50 cooperaran.

Observe en la tabla que desertar es la estrategia dominante de las dos empresas. Es decir, tie-
nen mayores recompensas si no respetan el acuerdo, sin importar la opción que escoja la otra. Para
ejemplificarlo, considere las opciones de la empresa 2. Los directores se dicen: “Supongamos que la
empresa 1 coopera. ¿Cuál es nuestra mejor opción?” Al cooperar cuando la empresa 1 coopera, las
empresas se colocan en la celda superior izquierda de la matriz de utilidades de la tabla 13.2, lo cual
significa que ganan 50. Pero si la empresa 2 incumple o deserta, el resultado será la celda inferior
izquierda en la que terminaría ganando 99. Ahora los directores de la empresa 2 dicen: “Supongamos
que la empresa 1 deserta. ¿Cuál es nuestra mejor opción?” Si la empresa 2 coopera, queda en la celda
superior derecha, donde gana cero. Pero si la empresa 2 deserta, gana 49.50. Por lo tanto, al margen
de lo que haga la empresa 1, la 2 gana más si deserta.
Con un razonamiento del todo paralelo, la renuncia es también la estrategia dominante para la
empresa 1. Pero observe que cuando las dos empresas desertan están peor que si cooperaran. En esta
situación, la conducta que le viene mejor a cada empresa no redunda en un resultado que vaya con el
interés de las dos empresas en general.
Como está planteado el ejemplo, las empresas no están mucho peor si desertan que si cooperan.
Pero las empresas que encuentran una vez que conviene a sus intereses renunciar a un acuerdo, tien-
den a pensar que les beneficia volver a hacerlo. Por ejemplo, si una cobra ahora 8, mientras la otra se
queda en 9, la primera ganará utilidades de 96, mientras la segunda gana 0. No es necesario que una
empresa sienta un deseo irresistible de superar a un rival para que una defección le parezca una op-
ción atractiva. Por el contrario, su motivación puede ser la pura protección, motivada por el miedo
racional a que su rival deserte. El proceso resultante de rebajas del precio de competencia no termina
hasta que se desploma al nivel del costo marginal. En este punto ninguna de las dos empresas obtiene
utilidades, así que el costo de no respetar un acuerdo de cooperación puede ser grave.
Los oligopolistas no sólo compiten en precio, sino también en el uso de publicidad. En este
caso también los intereses de una empresa suelen entrar en conflicto con los de las dos tomadas en
conjunto, como se ejemplifica en El naturalista económico.

13_CHAPTER 13.indd 416 3/6/09 8:05:19 PM


INtRoDuCCIóN a la tEoRía DE JuEGoS 417

¿Por qué las tabacaleras se anuncian “demasiado”?


Cuando una empresa anuncia su producto, la demanda aumenta por dos razones. En primer lugar,
quienes no habían consumido el producto se enteran de su existencia y algunos lo compran. En
segundo, otras personas que ya consumen otra marca del mismo producto podrían cambiarla debido EL NATURALISTA
a la publicidad. El primer efecto impulsa las ventas de toda la industria. El segundo sólo redistribuye ECONÓMICO
las ventas entre las empresas.
En la industria cigarrera estadounidense se cree que el efecto más importante de la publicidad
13.1
es el cambio de marcas. En tales industrias, la decisión de anunciarse plantea a las empresas el dilema
del prisionero. En la tabla 13.3 se muestran las ganancias de dos productores hipotéticos de cigarros
en cuatro posibles combinaciones de las decisiones de anunciarse o no. Si las dos empresas se anun-
cian (celda inferior derecha), cada una gana apenas 250, en comparación con las utilidades de 500
que ganarían si ninguna se anunciara (celda superior izquierda). Así, es evidente que es mejor que
ninguna empresa se anuncie, a que se anuncien las dos.

TABLA 13.3
Publicidad como un dilema del prisionero

Cuando el
Empresa 1
principal efecto
de la publicidad
No se anuncia Se anuncia es el cambio de
marca, la estrategia
dominante consiste
Π1 = 500 Π1 = 750 en anunciarse mucho
No se anuncia (celda inferior
Π2 = 500 Π2 = 0
derecha), aunque
Empresa 2
las empresas juntas
Π1 = 0 Π1 = 250 estarían mejor si no
Se anuncia
Π2 = 750 Π2 = 250 lo hicieran (celda
superior izquierda).

Pero observe los incentivos a que se enfrentan las em-


presas en lo individual. La empresa 1 ve que si la empresa
2 no se anuncia, ganará más con la publicidad (750) que sin
ella (500). La empresa 1 ve que si la empresa 2 se anuncia,
también gana más si se anuncia (250) que si no se anuncia
(0). Así, anunciarse es la estrategia dominante para la em-
presa 1. Y como las recompensas son simétricas, también
es la estrategia dominante para la empresa 2. Entonces,
aquí también cuando cada empresa hace lo que es racional
desde su propio punto de vista, las empresas como grupo
están peor que si actuaran de común acuerdo.
El Congreso estadounidense promulgó una ley que
prohibió los anuncios de cigarros por televisión desde el 1
de enero de 1971. Su propósito era proteger a las personas
de mensajes que pudieran convencerlas de consumir un
producto que ha demostrado ser peligroso para la salud.
Al parecer, la ley cumplió su propósito, por lo menos en
parte, como lo prueba la reducción siguiente en la pro-
porción de estadounidenses que fuman. Pero la ley tuvo
también un efecto impensado, que resolvió al menos de
manera temporal el dilema del prisionero que enfrentaban ¿Por qué se anuncian tanto las tabacaleras?

13_CHAPTER 13.indd 417 3/6/09 8:05:21 PM


418 Capítulo 13 CoMpEtENCIa IMpERFECta: ENFoQuE DE la tEoRía DE JuEGoS

las tabacaleras. En el año anterior a la promulgación de la ley, los fabricantes gastaron más de 300
millones de dólares en anunciar sus productos. La cifra correspondiente para el año siguiente fue
más de 60 millones de dólares inferior, y buena parte de la diferencia se tradujo en mayores utilidades
para la industria. Así, la prohibición de la publicidad logró para las tabacaleras lo que los imperativos
de la búsqueda individual de utilidades no consiguieron: una manera eficaz de limitar la carrera ar-
mamentista publicitaria. No obstante, en los años siguientes las presiones competitivas llevaron a las
tabacaleras a aumentar sus gastos en las formas publicitarias que todavía eran legales, lo que socavó
sus ganancias.

EL CONCEPTO DE EQUILIBRIO DE NASH


Cuando, como en el dilema del prisionero, las dos partes tienen una estrategia dominante en un
juego, el equilibrio de éste se establece cuando cada cual ejecuta la estrategia dominante. Pero hay
muchos juegos en los que no todos los participantes tienen una estrategia dominante. Por ejemplo,
considere la variación del juego de publicidad que se muestra en la tabla 13.4. No importa lo que
haga la empresa 2, a la empresa 1 le conviene anunciarse, así que anunciarse es su estrategia domi-
nante. Pero ahora no puede decirse lo mismo de la empresa 2. Si la empresa 1 se anuncia, la empresa
2 también hace mejor en anunciarse. Pero si la empresa 1 no se anuncia, la empresa 2 hace mejor en
no anunciarse. A diferencia del dilema del prisionero, en este caso la mejor estrategia de la empresa 2
depende de la que escoja la empresa 1.

TABLA 13.4
Juego en el que la empresa 2 no tiene estrategia dominante la estrategia
dominante de
la empresa 1 es
Empresa 1 anunciarse. pero la
empresa 2 no tiene
No se anuncia Se anuncia estrategia dominante.
Si la empresa 1 se
Π1 = 500 Π1 = 750 anuncia, a la empresa
No se anuncia 2 le conviene
Π2 = 400 Π2 = 100
Empresa 2 anunciarse; pero si
la empresa 1 no se
Se anuncia Π1 = 200 Π1 = 300 anuncia, a la empresa
Π2 = 0 Π2 = 200 2 le conviene no
anunciarse.

Aunque la empresa 2 no tiene una estrategia dominante en este juego, algo se puede decir acer-
ca de lo que es probable que pase. En particular, la empresa 2 es capaz de prever que la empresa 1 se
anunciará porque es su estrategia dominante. Y como la empresa 2 conoce esto, sabe que su mejor
equilibrio de Nash estrategia es anunciarse. En este juego, la celda inferior derecha se llama equilibrio de Nash,2 que se
combinación de estrategias define como una combinación de estrategias tal, que la de cada participante es la mejor que puede
en un juego de manera que escoger dadas las elecciones del otro jugador. Así, en el equilibrio de Nash, ningún participante tiene
ninguno de los participantes ningún incentivo para apartarse de su estrategia actual. Observe que cuando cada participante sigue
tiene incentivos para cambiar su estrategia dominante en un dilema del prisionero, el resultado es un equilibrio de Nash. Pero
de estrategia dada la estrategia
como se ha visto, un equilibro de Nash no requiere que los dos participantes tengan una estrategia
de su oponente.
dominante.
Una buena regla práctica para determinar si una combinación de estrategias constituye un equi-
libro de Nash consiste en preguntarse si alguno de los participantes tiene incentivos para elegir otra

2
Se llama así por John F. Nash, el matemático estadounidense que propuso el concepto en 1951.

13_CHAPTER 13.indd 418 3/6/09 8:05:22 PM


INtRoDuCCIóN a la tEoRía DE JuEGoS 419

estrategia. Por ejemplo, considere la combinación de estrategias de la celda superior izquierda de la


tabla 13.4. ¿Alguno de los participantes tiene un incentivo para escoger algo diferente?
Para responder la pregunta, observe primero que cada participante tiene la opción de moverse
sólo a otra celda de la tabla. Por ejemplo, la empresa 2 sólo puede escoger la fila superior o la fila in-
ferior, así que no puede moverse a una celda de otra columna. Si quiere moverse de la celda superior
izquierda, su única opción es la celda inferior izquierda. Pero como sus utilidades (0) serían menores
que en la celda superior izquierda (400), no quiere hacer el movimiento.
Del mismo modo, la empresa 1 puede escoger sólo la columna izquierda o derecha; no puede
moverse a una celda de otra fila. Así, a partir de la celda superior izquierda, la única opción de la
empresa 1 sería moverse a la celda superior derecha. Y como sus utilidades en esa celda (750) serían
mayores que en la celda superior izquierda (500), preferiría moverse. Así, si comienza en la celda
superior izquierda, al menos un participante (la empresa 1) querría moverse, lo cual significa que la
celda superior izquierda no puede ser un equilibrio de Nash.
En cambio, consideremos la celda inferior derecha de la tabla 13.4. La empresa 2, que escoge
únicamente la fila, sólo puede moverse a la celda superior derecha, lo cual no querría hacer porque
sus utilidades serían de (100), menos que en la celda inferior derecha (200). Del mismo modo, la
empresa 1, que escoge sólo la columna, nada más puede moverse a la celda inferior izquierda, pero
no querría hacerlo, porque sus utilidades ahí (200) serían menores que en la celda superior derecha
(300). Y como ninguna empresa quisiera moverse de la celda inferior derecha de la tabla 13.4, esa
combinación de estrategias satisface la definición de un equilibro de Nash.

EJERCICIO 13.1
¿Alguna de las dos empresas tiene una estrategia dominante en el juego siguiente? ¿El juego
tiene un equilibrio de Nash?

Empresa 1

Alto presupuesto Bajo presupuesto


para investigación para investigación

Alto presupuesto Π1 = 200 Π1 = 60


para investigación Π2 = 40 Π2 = 100
Empresa 2
Bajo presupuesto Π1 = 0 Π1 = 40
para investigación Π2 = 30 Π2 = 80

LA ESTRATEGIA MAXIMIN
En el juego que se muestra en la tabla 13.4 se vio que si la empresa 1 sigue su estrategia dominante
de anunciarse, a la empresa 2 también le conviene hacerlo. Si la empresa 2 cree que la empresa 1
actúa en forma racional, su mejor opción es anunciarse. Pero quizá la empresa 2 no está segura de
que la empresa 1 vaya a escoger de manera racional. En ese caso pensaría en considerar por lo menos
la posibilidad de que la empresa 1 decida no anunciarse. En tal situación, si la empresa 2 se anuncia
ganará utilidades nulas, mucho peor que las 400 que ganaría si no se anunciara.
¿Qué debe hacer la empresa 2 si no tiene una estrategia dominante y no está segura sobre la
decisión de la empresa 1? La respuesta depende de la probabilidad que le asigne a las opciones de la
empresa 1 y del efecto que éstas tengan en sus propias recompensas. Dadas las circunstancias, es en
extremo difícil pronosticar qué hará la empresa 2.
Sin embargo, suponga que la empresa 2 adopta una postura muy cauta al enfrentar dicha incer- estrategia maximin escoger
tidumbre. Entonces, podría seguir la llamada estrategia maximin, lo que significa escoger la opción la opción que aumenta lo más
que maximice el valor mínimo posible de sus recompensas. Vuelva otra vez a la tabla 13.4, observe posible el menor rendimiento
que si la empresa 2 decide no anunciarse, el menor rendimiento que puede recibir es 100 (que ocurriría que se puede recibir.

13_CHAPTER 13.indd 419 3/6/09 8:05:22 PM


420 Capítulo 13 CoMpEtENCIa IMpERFECta: ENFoQuE DE la tEoRía DE JuEGoS

si la empresa 1 se anunciara). Pero si la empresa 2 escoge anunciarse, el rendimiento más bajo posible
que puede recibir es cero (lo cual sucedería si la empresa 1 decide no anunciarse). Así, si se sabe que
la empresa 2 sigue la estrategia maximin en esta situación, es posible pronosticar que va a escoger no
anunciarse.

ESTRATEGIAS DE JUEGO REPETIDO


EN DILEMAS DEL PRISIONERO
Los juegos que se han examinado hasta este momento han tenido sólo dos participantes. Pero es fácil
extender los juegos a muchos jugadores. Por ejemplo, en el dilema del prisionero con muchos parti-
cipantes, las dos opciones estratégicas son también cooperar o retirarse, y la deserción sigue siendo
la estrategia dominante de los participantes. Y la condición definitoria del juego es también que cada
participante reciba una recompensa mayor cuando todos cooperan que cuando todos desertan.
Decir que los costos de no cooperar son elevados en la disyuntiva del preso no es más que otra
manera de decir que hay poderosos incentivos financieros para encontrar alguna manera de hacer
acuerdos de colusión. Lo que necesitan los participantes potenciales en un acuerdo de colusión es una
manera de castigar a los que desertan, de modo que no les convenga hacerlo. Esto es muy difícil de ha-
cer cuando en un dilema del prisionero se confrontan participantes que se relacionarán una sola vez.
Pero si los participantes esperan interactuar repetidamente en el futuro surgen nuevas posibilidades.
En investigaciones experimentales de la década de 1960 se identificó una estrategia muy simple
que resulta ser muy eficaz para controlar a los posibles evasores.3 Se llama ley del talión y opera como
sigue: la primera vez que usted interactúa con alguien, coopera. En cada interacción siguiente hace
lo mismo que la otra persona hizo la vez anterior. Así, si la asociada desertó en la primera ocasión,
entonces usted renunciará en su segunda interacción con ella. Si a continuación la segunda empresa
coopera, el siguiente movimiento de la primera empresa sería cooperar también.
La ley del talión es una estrategia “cordial” por la propensión de los participantes a cooperar en
la primera interacción. Si dos participantes en esta estrategia se relacionan durante mucho tiempo,
el resultado será la cooperación en cada interacción. Pero la ley del talión también es una estrategia
“dura”, porque quienes la siguen están dispuestos a castigar a los desertores en la siguiente interac-
ción. Por último, es una estrategia “de perdón”, en el sentido de que una empresa está dispuesta a
cooperar con una antigua desertora si dio pruebas de que está lista para cooperar.
Robert Axelrod, un politólogo de la Universidad de Michigan, realizó un análisis extenso de lo
bien que se desempeña la ley del talión en comparación con otras estrategias para el juego repetido
del dilema del prisionero.4 En una ronda inicial de las simulaciones por computadora de Axelrod, la
ley del talión fue la estrategia más exitosa, en el sentido de que la gente que la seguía ganaba más,
en promedio, que quienes usaban cualquier otra de las tácticas probadas. A continuación, Axelrod
publicó estos resultados e invitó a expertos de todo el mundo a tratar de diseñar una estrategia mejor.
Su desafío produjo un cúmulo de respuestas ingeniosas, pero Axelrod encontró que estas estrategias,
muchas de las cuales fueron ideadas con el propósito específico de superar la ley del talión, no la
superaron.
El éxito de dicha ley requiere un conjunto más o menos estable de participantes y que todos
puedan recordar lo que hicieron los demás en interacciones previas. También requiere que los par-
ticipantes tengan intereses en lo que ocurra en el futuro, porque sólo así el miedo a las represalias
los disuade de retirarse. Cuando se cumplen estas condiciones, los cooperadores se identifican entre
ellos y discriminan a los desertores.
Las condiciones que exige la estrategia de la ley del talión se encuentran también en poblaciones
humanas. Muchas personas se relacionan de manera constante y casi todas llevan la cuenta de cómo
las trataron. Axelrod reunió pruebas convincentes de que estas fuerzas explican cómo se comportan
las personas. Quizá las más impresionantes de estas pruebas vengan de la relación del sistema “vive y
deja vivir” que se desarrolló en las trincheras europeas durante la Primera Guerra Mundial. En mu-

3
Vea Anatol Rapoport y A. Chammah, Prisoner’s Dilema, Ann Arbor, University of Michigan Press, 1965.
4
Robert Axelrod, The Evolution of Cooperation, Nueva York, Basic Books, 1984.

13_CHAPTER 13.indd 420 3/6/09 8:05:22 PM


INtRoDuCCIóN a la tEoRía DE JuEGoS 421

chas zonas bélicas, las mismas unidades enemigas se


atrincheraron frente a frente durante varios años.
Las unidades estaban muy equilibradas, de modo
que ningún bando tenía mucha esperanza de derro-
tar con rapidez al otro. Sus opciones eran combatir
con intensidad y causarse muchas bajas unos a otros
o reprimirse.
Las condiciones de interacción descritas por
el historiador Tony Ashworth en su recuento de
las guerras de trincheras se asemejan mucho a las
que se necesitan para el éxito de la estrategia de la
ley del talión.5 Las identidades de los participantes

library of Congress/Dal
eran más o menos estables. Las interacciones entre
ellos se repitieron, en ocasiones varias veces al día,
durante periodos prolongados. Cada lado sabía con
facilidad cuando el otro faltaba al acuerdo y ambos
tenían un interés en mantener sus pérdidas futuras
en un mínimo.
No hay dudas de que la ley del talión surgió
como la estrategia preferida tanto para las unidades de los Aliados como de los alemanes en la Primera
Guerra Mundial. Aunque muy reprobada por razones de políticas oficiales, a veces la represión era del
todo patente. Ashworth escribe sobre las patrullas nocturnas que operaban fuera de las trincheras:

ingleses y alemanes de los sectores tranquilos suponían que en caso de que ocurriera un
encuentro fortuito, ninguna patrulla iniciaría una agresión, sino que se moverían para evi-
tarse. Cada patrulla concedía la paz a la otra cuando la agresión era no sólo posible, sino
obligada, siempre que el gesto fuera recíproco, porque si una patrulla disparaba, la otra
también lo hacía.6

En palabras de uno de los participantes en el conflicto:

De pronto, al rodear un montículo o una excavación, nos topábamos con una patrulla
alemana [...] Estaríamos a unas 20 yardas de distancia, completamente visibles. Agité una
mano cansada, como para preguntar qué sentido tenía que nos matáramos. El oficial
alemán pareció entender y los dos bandos dimos la vuelta y nos marchamos a nuestras
propias trincheras.7

Con frecuencia los bombardeos ocurrían sólo a ciertas horas del día y se dirigían lejos de las
posiciones más vulnerables. Por ejemplo, la hora de comer y las tiendas hospitales quedaban tácita-
mente fuera de los límites.
Las condiciones que examinó Axelrod explican no sólo cuándo van a cooperar las personas,
sino también cuándo es más probable que no lo hagan. Por ejemplo, señala que la contención mu-
tua en la guerra de trincheras comenzó a debilitarse cuando quedó claro que se acercaba el final de
la guerra.
Así como en la guerra, lo mismo sucede en el mundo de los negocios. Axelrod postula que las
compañías pagan sus facturas a tiempo no porque sea lo correcto, sino porque más adelante van a
requerir envíos de los mismos proveedores. Cuando no parece probable que se repitan las interaccio-
nes, muchas veces desaparece esta tendencia a cooperar: “Un ejemplo es el caso de la empresa que
está al borde de la quiebra y vende sus cuentas por cobrar a un tercero llamado ‘factor’.’” La venta se
hace con un descuento sustancial porque

5
Tony Ashworth, Trench Warfare: The Live and Let Live System, Nueva York, Holmes and Meier, 1980.
6
Ibíd., p. 103.
7
Herbert Read, citado en ibíd., p. 104.

13_CHAPTER 13.indd 421 3/6/09 8:05:23 PM


422 Capítulo 13 CoMpEtENCIa IMpERFECta: ENFoQuE DE la tEoRía DE JuEGoS

cuando un fabricante comienza a hundirse, incluso sus mejores clientes se niegan a pagar
la mercancía y alegan defectos de calidad, incumplimiento de las expectativas, entregas
retrasadas o lo que se les ocurra. El gran guardián de la moral en el comercio es la relación
continuada, la idea de que habrá de volver a comerciarse con este cliente o aquel provee-
dor, y cuando una empresa en problemas pierde este guardián automático, ni siquiera un
factor de mano dura llega a encontrar un sustituto.8

Otro requisito para el éxito de la ley del talión es que no haya un número fijo y conocido de inte-
racciones futuras. En efecto, si los participantes saben con exactitud cuántas veces van a relacionarse,
su cooperación mutua en cada movimiento no puede ser un equilibrio de Nash. Para entenderlo,
suponga que todas las empresas saben que van a interactuar con sus rivales, por ejemplo, otras 1 000
veces. Todas sabrían que la otra desertará en la última interacción, porque no hay posibilidad de que
la castiguen por eso. Pero como todas lo saben, tampoco hay razones para no fallar en la interacción
999, porque pase lo que pase en esa ronda, habrá una defección en la interacción 1 000. El mismo
argumento puede aplicarse hacia atrás, una por una hasta la primera interacción, con el resultado de
que la estrategia del talión se deshaga por completo.
Este problema no aparece si no hay un número fijo conocido de interacciones.9 Por ejemplo,
si se supone que siempre hay alguna probabilidad positiva de que haya nuevas interacciones, no es
posible identificar una interacción como la última, lo cual significa que la amenaza de un castigo
futuro siempre tendrá por lo menos alguna fuerza. En estas situaciones, en las que se encuentran la
mayoría de las empresas (con la excepción del caso ya citado de una quiebra), parece posible suponer
que siempre habrá alguna posibilidad de interacción en el futuro.
Entonces, ¿es inevitable que la estrategia del talión acabe por producir una colusión general en-
tre las empresas? De ninguna manera. Una dificultad es que la eficacia de la ley del talión depende de
que sólo haya dos participantes en el juego. En industrias competitivas y monopólicas competitivas
hay muchas empresas y hasta en los oligopolios hay varias. Cuando hay más de dos empresas y una
deserta en este periodo, ¿cómo castigan las que cooperan a la traidora en el siguiente periodo? ¿Re-
ducen los precios? Esto castigaría a todas, no sólo a la que defeccionó. Aun si sólo hay dos empresas
en la industria, queda el problema de que otra empresa pueda entrar en la industria. Así, las posibles
cooperadoras tienen que preocuparse no sólo una de la otra, sino también de la lista completa de
empresas que decidan hacerles competencia. Cada empresa ve esto como una tarea sin esperanza
y pensará en desertar ahora, con la expectativa de por lo menos cosechar utilidades mayores que lo
normal a corto plazo.
En las secciones que vienen se detalla la amenaza de la entrada potencial. Por ahora, observe
que, como cuestión puramente teórica, en el pasado se han producido acuerdos de cárteles y otras
formas de colusión que han sido muy inestables. Al parecer, los problemas prácticos involucrados en
el establecimiento de la ley del talión en los entornos donde operan las empresas dificultan mucho
que se sostengan los acuerdos de colusión durante largo tiempo.

JUEGOS EN SECUENCIA
En los juegos que se han considerado hasta ahora, los dos participantes deben escoger su estrategia
en forma simultánea. Cada participante tiene que escoger su estrategia conociendo nada más los
incentivos de su oponente, no su decisión estratégica concreta. Pero en muchos juegos un partici-
pante se mueve primero y entonces el otro puede escoger su estrategia con pleno conocimiento de
la primera decisión del otro. Esta descripción se aproxima a las circunstancias en que se encontraron
Estados Unidos y la Unión Soviética en la Guerra Fría.

8
Mayer, citado en Axelrod, op. cit., pp. 59, 60.
9
Otra manera de evitar el problema de que se rompa la ley del talión es que haya alguna probabilidad positiva de que otros sigan la
estrategia aunque, en rigor, no sea racional que lo hagan. Vea David Kreps, Paul Milgrom, John Roberts y Robert Wilson, “Rational
Cooperation in Finitely Repeated Prisoner’s Dilemmas”, en Journal of Economy Theory, 27, 1982, pp. 245-252.

13_CHAPTER 13.indd 422 3/6/09 8:05:24 PM


INtRoDuCCIóN a la tEoRía DE JuEGoS 423

En esa época las estrategias militares de las dos naciones se basaban en la doctrina de la destruc-
ción mutua asegurada (MAD, por sus siglas en inglés). La idea de la MAD era simple: los dos lados te-
nían arsenales nucleares lo bastante grandes y bien resguardados para garantizar que serían capaces
de tomar represalias si el otro lanzaba el primer ataque. La perspectiva de una respuesta devastadora,
según la teoría MAD, evitaba que cada lado pensara en dar el primer golpe.
Algunos interpretan el hecho de que ninguno haya lanzado el primer ataque como prueba de
que la estrategia MAD funcionó. Pero es evidente una falla lógica de la estrategia, que induce a pen-
sar que la causa de la contención debió ser otra. Para apreciar el problema, póngase en el lugar de
un presidente estadounidense que acaba de enterarse de que los rusos lanzaron el primer ataque. En
ese momento sabe que la estrategia MAD fracasó. Por la razón que sea, la amenaza de un contra-
ataque no disuadió a los soviéticos de emprender el primer ataque. ¿Ordena usted el contraataque?
Se da cuenta de que con eso aumentará las probabilidades de una destrucción total. Es verdad que
los intereses estadounidenses sufrirían un daño cruel, pero en ese momento un contraataque sólo
aumentaría sus daños.
Entonces, la dificultad lógica de la MAD es que cada lado sabe perfectamente bien que si se lanza
el primer ataque, no es lo mejor para el otro cobrar represalias. Y como ambos lo saben, la amenaza
de un contraataque pierde todo su poder disuasorio.
O, por lo menos, así parece en la teoría. Quizá la amenaza disuade porque cada lado teme que
el otro no responda de manera racional si es la víctima del primer ataque (una revisión somera de las
autoridades estadounidenses y soviéticas de la posguerra presta alguna credibilidad a esta interpre-
tación). Pero sea MAD o no una estrategia de defensa eficaz, es perturbadora su patente insuficien-
cia. Con la misma lógica con que se expone la falla se propone una manera sencilla de remediarla.
Consiste en instalar la llamada máquina del fin del mundo, un dispositivo a prueba de manipulación
que tomaría represalias automáticamente cuando se lanzara el primer ataque. Si cada lado supiera que
estaba en operación esa máquina, la estrategia MAD quedaría completa y entonces sería impensable
dar el primer golpe.
Para ver cómo los economistas tratan de manera analítica los juegos en secuencia, suponga
que la Unión Soviética pensaba en si debía lanzar un ataque nuclear contra Estados Unidos. La
decisión podría retratarse en un “árbol de juego”, como el diagrama de la figura 13.1. Si la URSS
mueve primero, el juego empieza en el punto A. Las primeras dos ramas del árbol representan
las alternativas de la URSS, atacar o no atacar. Si ataca, Estados Unidos se encuentra en el pun-
to B, en la rama superior del árbol de juego, donde debe decidir si responde o no. Si responde,
termina en el punto D, donde las recompensas son –100 para cada país. Si Estados Unidos no
toma represalias, pasa al punto E, donde las recompensas son 100 para la URSS y –50 para Estados
Unidos (las unidades de las recompensas son arbitrarias y se escogen con la intención de reflejar la
valoración hipotética de cada país sobre los resultados). La mitad inferior del árbol representa la
alternativa en la que la URSS no ataca. Si la URSS escoge esta opción, Estados Unidos se encuentra
en el punto C, donde vuelve a enfrentarse a una decisión sobre si lanza o no los misiles contra la
URSS. En aras del argumento, suponga que las recompensas de los dos países con cada alternativa
se indican al pie de las dos ramas del árbol en los puntos F y G. Dadas las recompensas supues-
tas para los cuatro posibles resultados del juego, la URSS puede analizar lo que hará Estados Uni-
dos en cada alternativa. Si la URSS ataca (punto B), la mejor opción que se le ofrece a Estados
Unidos es no contraatacar (punto E). Si la URSS no ataca (punto C), la mejor opción para Esta-
dos Unidos es no atacar (punto G). Entonces, la URSS sabe que si Estados Unidos quiere maximizar
sus recompensas, el juego termina en el punto E si la URSS ataca y en el punto G si no ataca. Como
la URSS tiene más recompensas en el punto E, lo que más le conviene es atacar. Estados Unidos
pueden amenazar con represalias, pero siempre que sus rivales crean que es un maximizador de
recompensas, a su amenaza le falta credibilidad.
Ahora suponga que Estados Unidos puede instalar una “máquina del fin del mundo”, un apa-
rato que tome represalias automáticamente en caso de un ataque de la URSS. El efecto sería elimi-
nar la mitad inferior de la rama alta del árbol de juego de la figura 13.1. La URSS sabría entonces
que si ataca, el juego terminaría en el punto D, donde obtiene una ganancia de –100. Y como esto
es peor que el resultado si no ataca (punto G), la mejor opción que tiene sería dejar los misiles en
sus silos.

13_CHAPTER 13.indd 423 3/6/09 8:05:24 PM


424 Capítulo 13 CoMpEtENCIa IMpERFECta: ENFoQuE DE la tEoRía DE JuEGoS

FIGURA 13.1 Lanza los D –100 para E.U.


Disuasión nuclear como misiles
juego en secuencia –100 para URSS
contra URSS
Si la uRSS ataca, la mejor Lanza B
opción para Estados los misiles
unidos es no responder contra E.U. E.U.
(punto E). Si la uRSS no
E –50 para E.U.
ataca, la mejor opción para No ataca
Estados unidos también 100 para URSS
es no atacar (punto G). A
Como la uRSS obtiene una URSS
mayor recompensa en E
que en G, atacará. Si se cree Lanza los F –50 para E.U.
que Estados unidos es un misiles
maximizador de utilidades, contra URSS –50 para URSS
su amenaza de tomar C
represalias por el primer No ataca
ataque no será creíble. E.U.
G 0 para E.U.
No ataca
0 para URSS

¿Por qué una compañía hace una inversión que sabe que no usará nunca?
En la actualidad la Torre Sears de Chicago es el edificio más alto de Estados Unidos. Este hecho le
confiere una forma especial de prestigio, gracias al cual sus propietarios cobran rentas más caras
que en otros edificios parecidos. Ahora bien, suponga que la compañía X pondera si construye un
EL NATURALISTA edificio todavía más alto. Suponga que sabe que toda compañía que tiene la propiedad permanente
ECONÓMICO del edificio más alto obtiene mayores utilidades económicas. Como es natural, lo que le preocupa es
13.2 que Sears (o cualquier otra empresa) construye un edificio todavía más alto, lo que disminuiría sus
ganancias de manera sustancial.
Tanto Sears como la compañía X se dan cuenta de que son participantes de un juego en secuen-
cia, del tipo que se esquematiza en la figura 13.2. El juego empieza en el punto A, donde X debe
decidir si participa con un edificio más alto. Si no lo hace, Sears tiene una recompensa de 100 y X una
de 0. En cambio, si X participa, el juego pasa al punto B, donde Sears tiene que decidir si construye un
edificio más alto o si no hace nada. Suponga que si Sears construye más alto, su recompensa será de
30, mientras que X obtiene una recompensa de –50; y si Sears no construye más alto, su recompensa
será de 40, mientras que la de X será de 60. Como es lógico, Sears no quiere que X entre. Incluso
podría anunciar su intención de construir un edificio más alto en caso de que X entre. Pero siempre
que X sepa las recompensas de Sears, puede concluir que la mejor opción si entra es que Sears no
haga nada. El equilibro de Nash de este juego en secuencia es el punto E, donde X entra y Sears no
hace nada.
Ahora suponga que antes de que Sears construyera su torre, tenía la opción de construir una
plataforma en la parte superior del edificio sobre la cual podría erigir una adición que acrecentara
el tamaño del edificio. Construir esta plataforma cuesta 10 unidades, pero reduce en 20 unidades el
costo de construir un edificio más alto. Si Sears tuviera instalada la plataforma, el juego en secuencia
entre la compañía X sería como se indica en el árbol del juego en la figura 13.3. En el punto D, la
recompensa de Sears es de 40 (ahorra 20 del costo del edificio menos las 10 unidades que cuesta
construir la plataforma). Su recompensa en C y E son 10 unidades menos que en la figura 13.2 (para
reflejar el costo de la plataforma). Pese a la escasa magnitud de estos cambios en la recompensa, la
presencia de la plataforma altera de forma notable el resultado del juego. Esta vez X puede pronos-

13_CHAPTER 13.indd 424 3/6/09 8:05:25 PM


INtRoDuCCIóN a la tEoRía DE JuEGoS 425

FIGURA 13.2
Decisión de construir el
D 30 para Sears edificio más alto
Construir
–50 para X Si la compañía X construye
un rascacielos más alto
B que la torre Sears, Sears
Entrar
Sears tiene que decidir si
construye más (punto D)
No E 40 para Sears
construir
o si renuncia a su estatus
60 para X del edificio más alto (punto
E). Como Sears obtiene
A una recompensa mayor en
Compañía X
el punto E que en D, no
construirá otro edificio
No C 100 para Sears más alto. además, como X
entrar 0 para X lo sabe, va a entrar en el
mercado, pese a la amenaza
de Sears de construir un
edificio mayor.

FIGURA 13.3
Disuasión estratégica
D 40 para Sears para evitar la entrada
Construir Si Sears hubiera construido
–50 para X
una plataforma en la parte
B alta de su edificio, a un
Entrar costo de 10 unidades,
Sears habría reducido 20
E 30 para Sears unidades al costo de
No
construir
construir un edificio
60 para X
más elevado. Entonces
A la compañía X habría
Compañía X calculado que no valdría
la pena construir un
C 90 para Sears edificio más alto, porque a
No Sears le habría convenido
entrar 0 para X responder con una adición.
El equilibrio de Nash del
juego modificado ocurre en
el punto C.

ticar que si entra con un edificio más alto, lo mejor para Sears será acrecentar su edificio, lo cual
significa que X recibirá una recompensa de –50. Entonces, a X no le parecerá provechoso entrar en
este mercado y el juego termina en el punto C. La recompensa para Sears en C es 90 (los 100 origina-
les menos el costo de 10 unidades de construir la plataforma). Así, su inversión de 10 unidades en la
plataforma aumenta en 50 su recompensa neta (la diferencia entre la recompensa de 90 unidades que
recibe con la plataforma y la recompensa de 40 unidades que habría recibido sin la misma).10
La inversión en la plataforma que se acaba de analizar es un ejemplo de lo que los economistas
llaman disuasión estratégica para evitar la entrada. Estas inversiones resultan eficaces porque cambian
las expectativas de los rivales sobre cómo responderá la empresa si se amenaza su posición en el
mercado.

10
Los ejemplos más recientes de juegos de disuasión estratégica para evitar la entrada son Internet Explorer de Microsoft contra
Netscape y Barnes and Noble contra Borders.

13_CHAPTER 13.indd 425 3/6/09 8:05:27 PM


426 Capítulo 13 CoMpEtENCIa IMpERFECta: ENFoQuE DE la tEoRía DE JuEGoS

¿Para qué una empresa construye una fábrica


con más capacidad de la que podría necesitar?
Los centros de producción grandes tienen más
costos fijos y menos costos marginales que los de
EL NATURALISTA menores dimensiones. Entonces, la pregunta pue-
ECONÓMICO de volver a plantearse de la siguiente manera: ¿en
qué le beneficia a una empresa tener una fábrica
13.3 con un costo marginal muy bajo, aun si el resulta-
do fuera hacer que su costo total fuera mayor que
el de una instalación más pequeña?
Una respuesta posible es que la fábrica gran-
de constituye otro ejemplo de disuasión estraté-
gica para evitar la entrada. Si los posibles nuevos
participantes supieran que una empresa de la in-
dustria tiene costos marginales extremadamente
bajos, pronosticarían que le conviene quedarse en
el negocio incluso con un nivel de precios dema-
siado bajo para que los nuevos entrantes ganen
utilidades normales. En ese caso, no sería racional
¿Para qué quiere una empresa construir una fá-
que entraran en el mercado. Entonces, su ausencia
brica más grande que lo que necesita?
permitiría a la empresa cobrar un precio bastante
alto para pagar los costos del centro más grande.

MODELOS ESPECÍFICOS DE OLIGOPOLIOS


Si una empresa piensa en cambiar su volumen de producción o su precio de venta, podría formular
muchos supuestos sobre la reacción de sus rivales. Por ejemplo, podría pronosticar que sus rivales
seguirían generando su volumen actual de producción. Como alternativa, podría suponer que con-
tinuarán cobrando los precios actuales. O anticipar que reaccionarán de maneras específicas a sus
cambios de precio y producción. En las secciones que siguen se van a explorar las consecuencias de
cada supuesto.

MODELO DE COURNOT
modelo de Cournot modelo Para comenzar se verá el caso más simple, el llamado modelo de Cournot, en el que las empresas
de oligopolio en el que las suponen que sus rivales seguirán generando su volumen actual de producción. Debe su nombre al
empresas suponen que los economista francés Auguste Cournot, que en 1838 lo introdujo para describir las acciones de dos
rivales seguirán generando su empresas que vendían agua embotellada de manantial. Un oligopolio de dos empresas se llama duo-
volumen actual de producción. polio, así que el modelo de Cournot a veces se denomina modelo del duopolio de Cournot, aunque sus
conclusiones se generalizan fácilmente para más de dos empresas.
La premisa básica del modelo de Cournot es que en un duopolio, cada participante trata el vo-
lumen del otro como una cifra fija, que no responde a sus propias decisiones de producción. Es una
forma débil de interdependencia, cierto, pero se verá que de cualquier manera lleva a un resultado en
el que las acciones de cada empresa afectan sustancialmente a la otra.
Suponga que la curva de demanda total del mercado de agua mineral está dada por:

P = a – b (Q1 + Q2) (13.1)

donde a y b son números positivos y Q1 y Q2 son la producción de las empresas 1 y 2, respectivamen-


te. Cournot supuso que el agua podría embotellarse sin costo marginal, pero este postulado es por
pura conveniencia. Se llegaría a conclusiones básicamente semejantes si cada empresa tuviera un
costo marginal positivo constante.

13_CHAPTER 13.indd 426 3/6/09 8:05:30 PM


MoDEloS ESpECíFICoS DE olIGopolIoS 427

Primero se ve el problema de maximización de utilidades que enfrenta la empresa 1. Dado el


supuesto de que la producción de la empresa 2 está fija en Q2, la curva de demanda de agua para la
empresa 1 está dada por

P1 = (a – b Q2) – bQ1, (13.2)

que está rescrita para hacer hincapié en que la empresa 1 toma Q2 como fijo.
Según muestra la ecuación 13.2, para obtener la curva de demanda de la empresa 1 se resta bQ2
de la ordenada en el origen de la curva de demanda del mercado. La idea es que la empresa 2 retiró
las primeras Q2 unidades de la curva de demanda del mercado y le dejó el resto a la empresa 1.
Si Q2 fuera igual a cero, la empresa 1 se quedaría con toda la curva de demanda del mercado,
según lo indica D en la figura 13.4. Si Q2 es positiva, se obtiene la curva de demanda de la empresa
1 desplazando a la derecha el eje de ordenadas del diagrama de demanda en Q2 unidades. La curva
de demanda de la empresa 1 es la parte de la curva de la demanda original que está a la derecha del
nuevo eje vertical, y por este hecho se llama curva de demanda residual. La curva de ingreso marginal
asociada se marca como IM1. La regla de maximización de utilidades de la empresa 1 es la misma que
la de cualquier empresa que tenga una curva de demanda con pendiente negativa, es decir, igualar
el ingreso y el costo marginales. En este ejemplo se supone que el costo marginal es cero, así que el
volumen de producción de las mayores utilidades para la empresa 1 es el nivel en el cual la curva de
ingreso marginal adopta un valor nulo.
La producción de equilibrio para un duopolio de Cournot puede deducirse a partir del diagra-
ma de demanda residual. Dado que la empresa 2 produce Q2, la empresa 1 maximiza sus utilidades
produciendo en el nivel en el cual el ingreso marginal es igual al costo marginal. El ingreso marginal
de la empresa 1 se obtiene mediante IM1 = (a – bQ2) – 2bQ1. El ingreso marginal tiene el doble de
pendiente que la demanda, así que interseca el costo marginal en la mitad de la distancia desde el
eje Q1 = 0 hasta la abscisa en el origen de la curva de la demanda. Por simetría (las dos empresas son
idénticas, así que deben actuar igual), Q2 = Q1, lo cual significa que cada uno de los tres segmentos
que se muestran en el eje horizontal de la figura 13.4 tienen la misma longitud. Esto implica que cada
empresa tiene una producción igual a un tercio de la distancia desde el origen hasta la abscisa en el
origen de la curva de demanda. La curva de demanda P = a – bQ tiene una intersección horizontal de

FIGURA 13.4
Duopolio de Cournot
de maximización de
utilidades
$/Q
la curva de demanda
a Q1 = 0 de un participante en el
D duopolio de Cournot se
obtiene desplazando el eje
vertical a la derecha en un
a – bQ2 valor igual a la cantidad
producida por el otro
miembro del duopolio
(en el diagrama, Q2). la
P*1 = (a – bQ2)/2 parte de la curva original
de demanda del mercado
IM 1
que está a la derecha de
Q2
este nuevo eje vertical es
la curva de demanda de
0 Q1 la empresa 1. la empresa
Q*1 = (a – bQ2)/2b 1 maximiza sus utilidades
igualando el ingreso
marginal con el costo
marginal, el segundo de los
cuales es cero.

13_CHAPTER 13.indd 427 3/6/09 8:05:31 PM


428 Capítulo 13 CoMpEtENCIa IMpERFECta: ENFoQuE DE la tEoRía DE JuEGoS

Q = a/b; por lo tanto, Q1 = Q2 = a/(3b). Un enfoque más general consiste en fijar el ingreso marginal
igual al costo marginal y despejar la producción de la empresa 1 en términos de la producción de la
empresa 2.11
a − bQ2
Q *2 = . (13.3)
2b

función de reacción Los economistas llaman a la ecuación 13.3 la función de reacción de la empresa 1 y la repre-
curva que indica el nivel de sentan como Q*1 = R1(Q2). Esta notación es sugerente porque la función de reacción señala cómo
producción que maximiza las responderá la cantidad de la empresa 1 al nivel de cantidad que ofrezca la empresa 2.
utilidades de un miembro del Como el problema del duopolio de Cournot es por completo simétrico, la función de reacción
oligopolio, para cada cantidad de la empresa 2 tiene exactamente la misma estructura:
suministrada por el otro.
a − bQ1
Q *2 = R2 (Q1 ) = . (13.4)
2b
En la figura 13.5 se representan gráficamente las dos funciones de reacción. Para ilustrar el fun-
cionamiento del concepto de función de reacción, suponga que la empresa 1 produjo al principio
una cantidad de Q01. Entonces, la empresa 2 generaría el nivel de producción que correspondiera a Q01
en su función de reacción. La empresa 1 respondería a ese volumen de producción tomando el punto
correspondiente de su propia función de reacción, etc. El resultado final del proceso es un equilibrio
estable en la intersección de las dos funciones de reacción. Cuando las dos empresas generan a/3b
unidades de producción, ninguna quiere cambiar.12 Estos volúmenes de producción constituyen el
equilibrio de Nash para los dos miembros del duopolio de Cournot.
¿Es redituable para ambas empresas el duopolio de Cournot? Como la producción combinada
de las dos empresas es 2a/3b, el precio de mercado será P = a – b(2a/3b) = a/3. En ese precio cada
una tendrá un ingreso total igual a (a/3)(a/3b) = a2/9b. Como ninguna de las dos incurre en costos
de producción, los ingresos totales y las utilidades económicas son lo mismo.

FIGURA 13.5
Funciones de reacción Q1
del duopolio de
Cournot a/b
la función de reacción del Función de reacción
duopolio está dada por su de la empresa 2 = R2(Q1)
nivel de producción que Q01
maximiza las utilidades,
como una función del nivel
de producción de la otra a/2b
empresa. los miembros
del duopolio están en Qe1 = a/3b Función de reacción
equilibrio estable en el de la empresa 1 = R1(Q2)
punto de intersección de
sus funciones de reacción.
Q2
a/2b a/b
Qe2 = a/3b

11
Por ejemplo, se necesita este enfoque más general si las empresas son asimétricas (diferentes) y no generan el mismo nivel de
producción.
12
Para resolver algebraicamente el volumen de producción de equilibrio de la empresa 2, se sustituye Q*1 = Q*2 en su función de
reacción y se resuelve:

a − bQ *2 a − bQ *1
R1 (Q 2*) = = = Q 1*
2b 2b

lo que produce Q*1 = a/3b.

13_CHAPTER 13.indd 428 3/6/09 8:05:34 PM


MoDEloS ESpECíFICoS DE olIGopolIoS 429

Un duopolio de Cournot se enfrenta a una curva de demanda del mercado dada por P = 56 EJEMPLO 13.1
–2Q, en la que Q es la demanda total del mercado. Cada miembro puede generar producción
a un costo marginal constante de 20/unidad. Grafique sus funciones de reacción y encuentre
el precio y la cantidad de equilibrio.

La figura 13.6a muestra la curva de la demanda residual de la empresa 1 cuando la empresa 2 produce
Q unidades. La curva de ingreso marginal de la empresa 1 tiene la misma ordenada en el origen que
su curva de la demanda y el doble de pendiente. Así, la ecuación de la curva de ingreso marginal de
la empresa 1 es IM1 = 56 – 2Q2 – 4Q1. Si se iguala IM1 con el costo marginal (20), y se resuelve para la
función de reacción de la empresa 1, Q*1 = R1 = 9 – (Q2/2). Por simetría, la función de reacción de la
empresa 2 es R2 = 9 – (Q1/2). Las dos funciones se muestran en la figura 13.6b, en la que se intersecan
en Q1 = Q2 = 6. La producción total del mercado será Q1 + Q2 = 12. Si examina la curva de demanda
del mercado, verá que el precio del mercado será P = 56 – 2(12) = 32.

FIGURA 13.6
$/Q Q1 Obtención de las
funciones de reacción
56 18 de un duopolio
En la parte a) se muestra el
IM1 = 56 – 2Q2 – 4Q1 volumen de producción de
56 – 2Q2 R2 mayores utilidades para la
empresa 1 (Q*1 ) cuando la
CM = 20 empresa 2 produce Q2. Esta
20 9 expresión más la expresión
paralela de la empresa 2
6 constituyen las funciones
R1 de reacción que se grafican
Q2 en la parte b).
Q Q2
0 Q1 6 9 18
Q*1 = 9 – (Q2/2)

a) b)

EJERCICIO 13.2
Repita el ejemplo 13.1 con dos empresas que tienen una curva de demanda del mercado de
P = 44 – Q.

Tal vez se haya preguntado por qué el duopolio de Cournot supone que su rival ignorará sus
decisiones de producción. En ese caso, se habrá hecho una pregunta clave, la misma que planteó
un crítico de Cournot, el economista francés Joseph Bertrand. A continuación se va a considerar su
solución al problema del duopolio.

MODELO DE BERTRAND
Bertrand se percató de que desde el punto de vista del comprador, lo que cuenta es comparar el
precio que cobran las dos empresas. Como los miembros del duopolio venden agua mineral idéntica,
todo consumidor quiere comprar al que venda más barato. El modelo de Bertrand postula que cada modelo de Bertrand modelo
empresa escoge su precio suponiendo que el precio de su rival es fijo. A primera vista esta suposición de oligopolio en el que cada
no parece más verosímil que la de Cournot, y como precios y cantidades se corresponden sólo en las empresa supone que sus
curvas de demanda del mercado, parecería natural preguntarse si la suposición de Bertrand conduce rivales siguen cobrando el
realmente a otro resultado. Al investigarlo, las conclusiones son, de hecho, muy diferentes. precio actual.

13_CHAPTER 13.indd 429 3/6/09 8:05:35 PM


430 Capítulo 13 CoMpEtENCIa IMpERFECta: ENFoQuE DE la tEoRía DE JuEGoS

Para ejemplificarlo, suponga que la demanda del mercado y las condiciones de costo son las
mismas que en el modelo de Cournot. Considere también que la empresa 1 cobra un precio inicial
de P10. Entonces, la empresa 2 tiene tres opciones: 1) puede cobrar más que la empresa 1, con lo que
no va a vender nada, 2) puede cobrar lo mismo que la empresa 1, en cuyo caso las dos se dividen
la demanda del mercado a ese precio, y 3) puede vender a un precio marginalmente menor que la
empresa 1 y entonces captará toda la demanda del mercado a ese precio. La tercera opción siempre
será, por mucho, la más rentable.13
Como en el modelo de Cournot, en el de Bertrand las situaciones de los miembros del duopolio
son por completo simétricas, lo cual significa que la opción de vender a un precio marginalmente
menor que la competencia será la estrategia preferida por las dos empresas. No hace falta decir que
no puede haber un equilibrio estable en el que las dos empresas vendan más barato que la otra. El
proceso de avance y retroceso de reducción de precios continúa hasta que alcanza su límite econó-
mico natural, a saber, el costo marginal, que en el caso del agua de manantial es nulo. (Si, en cambio,
se considerara un ejemplo en el que las dos empresas tuvieran el mismo costo marginal positivo, el
precio habría bajado a ese valor.) Cuando las dos empresas rebajan su precio al costo marginal, ya no
hay incentivos para bajarlo más. Cuando las dos empresas venden al costo marginal, comparten el
mercado de manera equitativa.

EJEMPLO 13.2 El duopolio de Bertrand tiene una curva de demanda del mercado dada por P = 56 – 2Q. Las
dos pueden generar producción a un costo marginal constante de 20/unidad. Encuentre el
precio y la cantidad de equilibrio.

La solución es que las dos empresas fijen un precio en el costo marginal de P = CM = 20. La produc-
ción de la industria está determinada por la demanda del mercado: 20 = 56 – 2Q implica que Q =
18. Las empresas dividen el mercado equitativamente, de modo que cada una genera la mitad de la
producción de la industria, Q1 = Q2 = Q/2 = 9.

EJERCICIO 13.3
Si la curva de demanda de mercado del duopolio de Bertrand se obtiene mediante P = 10 – Q
y las dos tienen un costo marginal de 2, ¿cuál será el precio y la cantidad de equilibrio de cada
una?

Así, se ve que un cambio menor en las premisas iniciales sobre el comportamiento de una em-
presa (que cada miembro da por sentado el precio de su rival y no la cantidad) lleva a un equilibrio
marcadamente diferente. Ahora considere otro cambio menor en los supuestos iniciales sobre el
comportamiento de las empresas que lleva incluso a otro equilibrio.

MODELO DE STACKELBERG
En 1934 el economista alemán Heinrich von Stackelberg se hizo una pregunta simple pero provoca-
tiva: “¿qué haría una empresa si supiera que su única competidora es una duopolista de Cournot in-
genua?” La respuesta es que querría determinar su nivel de producción tomando en cuenta el efecto
que esa decisión tendría en el volumen de producción de su rival.
Para volver al modelo de Cournot, suponga que la empresa 1 sabe que la empresa 2 dará por
seguro el nivel de producción de la empresa 1. ¿Cómo puede aprovechar de manera estratégica
ese conocimiento? Para responder, recuerde que la función de reacción de la empresa 2 está dada

En el caso de un precio infinitesimalmente menor que el de la empresa 1, las utilidades de la empresa 2 serán casi el doble en la
13

opción 3 que en la opción 2.

13_CHAPTER 13.indd 430 3/6/09 8:05:35 PM


MoDEloS ESpECíFICoS DE olIGopolIoS 431

por Q*2 = R2(Q1) = (a – bQ1)/2b. Si se sabe que la producción de la empresa 2 depende de Q1 de esta
manera, la empresa 1 puede sustituir R2(Q1) para Q2 en la ecuación de la curva de demanda del mer-
cado, lo cual da lugar a la siguiente expresión para su propia curva de demanda:
 a − bQ1  a − bQ1
P = a − b[Q1 + R2 (Q1 )] = a − bQ1 + = . (13.5)
 2b  2

Esta curva de demanda y su curva de ingreso marginal asociada se muestran como D1 e IM1 en la
figura 13.7. Como se supone que el costo marginal es cero en el ejemplo del agua de manantial, el
nivel de producción que maximiza las ganancias de la empresa 1 será aquel en el cual IM1 es cero, a
saber, Q*1 = a/2b. El precio de mercado será a/4.

FIGURA 13.7
$/Q Curvas de demanda e
ingreso marginal del
líder de Stackelberg
a/2 Cuando la empresa 1
sabe que la empresa
2 es un duopolista de
Cournot, puede tomar
P*1 = a/4 en cuenta el efecto de su
D1 comportamiento sobre la
elección de la cantidad de
IM 1 la empresa 2. El resultado
es que sabe con exactitud
Q1 cuál será su curva de
Q*1 = a/2b a/b demanda.

Un líder y un seguidor de Stackelberg tienen una curva de demanda dada por P = 56 – 2Q. Los EJEMPLO 13.3
dos pueden generar una producción a un costo marginal constante de 20/unidad. Encuentre
el precio y la cantidad de equilibrio.
Para encontrar la solución, se sustituye la función de reacción de la empresa 2, Q2 = 9 – Q1/2 en la
demanda de la empresa 1, P = (56 – 2Q2) – 2Q1, para encontrar P = 38 – Q1, con un ingreso margi-
nal correspondiente IM1 = 38 – 2Q1. Establecer el ingreso marginal igual al costo marginal deja la
producción de la empresa 1 en Q1 = 9. Si se sustituye la producción de la empresa 1 en la función de
reacción de la empresa 2 da la producción de la empresa 2, Q2 = 92– . La producción total de la industria
es Q = Q1 + Q2 = 27
–2 , con precio P = 56 – 2Q = 56 – 27 = 29.

EJERCICIO 13.4
La curva de demanda del mercado de un líder y un seguidor de Stackelberg está dada por P =
10 – Q. Si los dos tienen un costo marginal de 2, ¿cuál será el precio y la cantidad de equilibrio
de cada uno?

Por razones obvias, la empresa 1 se denomina líder de Stackelberg. El término seguidor de Sta-
ckelberg designa a la empresa 2. Para poner en una perspectiva más clara el comportamiento del
líder de Stackelberg, vuelva a considerar la gráfica de las funciones de reacción de las dos empresas
de Cournot, que aquí se representan en la figura 13.8. Como vio en la figura 13.7, a/2b es la mejor
producción para la empresa 1 en cuanto tiene presente que la empresa 2 responderá a su decisión de
acuerdo con la función de reacción R2(Q1). Una vez que la empresa 1 produce a/2b, la empresa 2 exa-

13_CHAPTER 13.indd 431 3/6/09 8:05:37 PM


432 Capítulo 13 CoMpEtENCIa IMpERFECta: ENFoQuE DE la tEoRía DE JuEGoS

minará R2 y su respuesta será producir a/4b. Ahora aquí está el paso crucial. Si la empresa 1 piensa
que la empresa 2 se quedará en a/4b en cualquier circunstancia, su mejor apuesta sería examinar su
propia función de reacción y producir la cantidad correspondiente, a saber, 3a/8b. Al hacerlo ganaría
más que produciendo a/2b. El problema es que la empresa 1 se da cuenta de que si reduce a 3a/8b,
incitará otra reacción de la empresa 2 que culminará en una espiral descendente que terminará en el
punto de intersección de las dos funciones de reacción. La empresa 1 haría bien en moverse a 3a/8b
si de algún modo pudiera inducir a la empresa 2 a quedarse en a/4b; pero no puede. La mejor opción
que tiene la empresa 1 es apretar los dientes y quedarse en a/2b.

FIGURA 13.8
El equilibrio de
Stackelberg Q1
En el modelo de
Stackelberg la empresa 1 a/b
ignora su propia función
de reacción del modelo de R2(Q1)
Cournot. Escoge su propia
cantidad para maximizar
utilidades tomando en
cuenta el efecto que tendrá a/2b
en la cantidad que ofrezca
la empresa 2. 3a/8b
R1(Q2)

Q2
a/4b a/2b a/b

COMPARACIÓN DE RESULTADOS
Ahora que se han considerado tres tipos de comportamiento de los duopolios, se van a comparar los
resultados de los modelos. Un monopolio que enfrenta las mismas condiciones de demanda y costos
que un duopolio de Cournot generaría a/2b unidades de producción a un precio de a/2 y obtendría
utilidades económicas de a2/4b (vea la figura 13.9). La interdependencia de los miembros del duopo-
lio de Cournot causa que el precio sea un tercio más bajo y que la cantidad total sea un tercio más
alta que los valores correspondientes en el caso del monopolio.14 Mientras el precio y la cantidad de
equilibrio en el modelo de Cournot varían sólo por un factor de un tercio con respecto a los del caso
del monopolio, en el modelo de Bertrand son idénticos que en el caso monopolista.15
¿Cuáles son los resultados para el duopolio de Stackelberg? Como es natural, al líder le va mucho
mejor, porque es el único que manipula de manera estratégica el comportamiento del seguidor. Si
consulta la figura 13.7, se ve que la utilidad de la empresa 1 es a2/8b, el doble que la de la empresa 2.
Resulta que es exactamente lo que la empresa 1 habría ganado si se hubiera aliado con la empre-
sa 2 para cobrar el precio del monopolio, a/2 y se dividieran el mercado en forma equitativa (véase
la figura 13.9). La producción conjunta de las dos empresas en el caso de Stackelberg es 3a/4b, que

14
Como una fracción de la producción para una industria perfectamente competitiva, la producción de la industria en un duopolio
de Cournot es N/(N + 1) con N empresas. A medida que el número de empresas N aumenta, la producción de la industria de
Cournot (y, por consiguiente, el precio y la utilidad) se aproxima a la de una industria perfectamente competitiva. En este sentido, el
duopolio de Cournot está de verdad entre el monopolio y la competencia perfecta.
15
Si las empresas escogen la capacidad y luego el precio, el resultado concuerda con el equilibrio de Cournot. Vea David Kreps y
José Scheinkman, “Quantity Precommitment and Bertrand Competition Yield Cournot Outcomes”, en Bell Journal of Economics, 14,
1983, pp. 326-337.

13_CHAPTER 13.indd 432 3/6/09 8:05:39 PM


MoDEloS ESpECíFICoS DE olIGopolIoS 433

FIGURA 13.9
Comparación del
precio y la cantidad de
equilibrio
$/Q El monopolista maximiza
las utilidades cuando el
a ingreso marginal es igual
a cero, puesto que no
hay costos marginales de
producción. El precio de
equilibrio será mayor y
Monopolio la cantidad de equilibrio
P*m = a/2
Cournot menor que en el caso de
Stackelberg Cournot.
MR D
Bertrand/
Competencia perfecta
Q
* = a/2b
Qm a/b

TABLA 13.5
Comparación de modelos de oligopolio
los cuatro modelos
Producción de Precio del Utilidad de la suponen una curva
Modelo la industria, Q mercado, P industria, Π de demanda del
mercado de
Monopolio compartido Qm = a/(2b) Pm = a/(2) Πm = a2/(4b) P = a – bQ y un
costo marginal igual a
Cournot (4/3)Qm (2/3)Pm (8/9)Πm cero (desde luego, si
Stackelberg (3/2)Qm (1/2)Pm el costo marginal no
(3/4)Πm
es cero, las entradas
Bertrand 2Qm 0 0 serán diferentes de
Competencia perfecta 2Qm 0 0 las mostradas).

es un poco mayor que en el caso de Cournot, con el resultado de que el precio del mercado a/4 es
ligeramente inferior que en el caso de Cournot (a/3). En la tabla 13.5 se resumen los resultados de las
cuatro posibilidades que se han considerado hasta este momento.
El modelo de Stackelberg representa una mejora evidente de los modelos de Cournot y Ber-
trand en cuanto a que permite que por lo menos una de las empresas tenga un comportamiento
estratégico. Pero, ¿por qué sólo una empresa se debería conducir de esta manera? Si la empresa 1
puede hacer un uso estratégico de la función de reacción de su rival, ¿por qué la empresa 2 no puede
hacer lo mismo? De hecho, suponga que las dos empresas tratan de ser líderes de Stackelberg. En-
tonces, cada una ignorará su propia función de reacción y producirá a/2b, con el resultado de que
la producción total y el precio de la industria serán a/b y 0, respectivamente, lo mismo que en el
modelo de Bertrand. Desde el punto de vista de los consumidores éste es un resultado muy deseable,
desde luego; pero para los propietarios de las empresas, un comportamiento estratégico universal
trae el peor resultado posible.

13_CHAPTER 13.indd 433 3/6/09 8:05:40 PM


434 Capítulo 13 CoMpEtENCIa IMpERFECta: ENFoQuE DE la tEoRía DE JuEGoS

COMPETENCIA CUANDO HAY RENDIMIENTOS


CRECIENTES DE ESCALA
Considere el ejemplo de un duopolio en una industria con rendimientos crecientes de escala. ¿Cómo
sobrevivirían dos empresas en tal industria si sus condiciones de costo las convierten en un monopo-
lio natural? Es muy fácil imaginar dos empresas que comienzan en una etapa inicial del desarrollo de
un producto nuevo y que cada una atiende a un segmento distinto del mercado. Pero ahora suponga
que la industria maduró y no hay más que un único mercado nacional para el producto. ¿Espera que
una empresa saque a la otra del mercado y que se apodere del papel de monopolista natural? En tal
caso, ¿qué precio cobrará?

FIGURA 13.10
Mercado compartido
con rendimientos
crecientes de escala $/Q
Con dos empresas en el
mercado, los costos son
mayores que con una, pero
quizá no haya ninguna
tendencia para que una
saque a la otra del negocio.
AC’

AC0 CT
CM
Q
Q0/2 Q0

Para hacer concreta la exposición, suponga que la tecnología tiene un costo marginal constante
y un costo total promedio decreciente, como se muestra en la figura 13.10. Por simplicidad, suponga
que el tamaño del mercado total está fijo en Q0. Con dos empresas en la industria, cada una a cargo
de la mitad de la producción, el costo promedio es AC´. Si nada más hubiera una empresa, su costo
promedio sería sólo de AC0. Pero, ¿con qué proceso cree que una empresa vaya a eliminar a la otra?
La estrategia obvia para las dos empresas es la fusión. La dificultad estriba en que las normas
antimonopolio del Tribunal de Defensa de la Competencia estadounidense no permiten fusiones
entre empresas cuya participación del mercado combinada supere una pequeña fracción del total de
la producción de dicho mercado. En este caso la participación combinada sería de 100%, lo que haría
imposible la aprobación.
La segunda alternativa es que una de las empresas anuncie una rebaja precautoria de precios,
con la esperanza de sacar a la otra del negocio. Por ejemplo, suponga que cobra AC0, que sería su
costo promedio si, de hecho, su estrategia funcionara. ¿Cómo respondería su rival? Puede igualar
el recorte del precio y volver a dividir el mercado o puede negarse y no vender nada. Sin embargo,
con ese precio las dos empresas pierden dinero; y si el precio se sostiene, es sólo cuestión de tiempo
para que una de las dos salga del negocio. Pero podría tardar mucho tiempo, y entre tanto la empresa
superviviente experimentaría pérdidas cuantiosas.
Más importante aún, desde el punto de vista de la empresa que pondera si hace la rebaja, no hay
seguridad de que vaya a ser la que sobreviva. Entonces, si se considera la decisión desde el punto de
vista de la empresa, iniciar una guerra de precios parece una proposición muy arriesgada. Sin una

13_CHAPTER 13.indd 434 3/6/09 8:05:41 PM


CoMpEtENCIa MoNopolíStICa 435

amenaza de entrada, es fácil ver por qué una estrategia de tolerancia mutua sería irresistiblemente
atractiva.
Pero suponga para fines de esta exposición que una empresa se las arregla para captar todo el
mercado y también que los posibles nuevos participantes se enfrentan a costos de inversión sustancia-
les para entrar en él. ¿La empresa superviviente quedará libre para cobrar el precio de monopolio?
Por los mismos motivos que un duopolista estaría renuente a iniciar una guerra de precios, un
intruso tendría la precaución de no entrar en una industria para encarar la posibilidad de una batalla
ruinosa con la empresa superviviente. Pero podría ser posible que una empresa externa firmara con-
tratos con compradores. Si ofreciera un precio menor que la empresa dominante aseguraría todo el
mercado. Pero la empresa externa se da cuenta de que a la dominante le interesaría igualar o superar
cualquier precio que pudiera ofrecer. Después de todo, el entrante potencial tiene que cobrar lo ne-
cesario para saldar sus costos de inversión si su entrada en el mercado ha de ser redituable, mientras
la dominante sólo tiene que pagar sus costos variables. Desde luego, esta última estaría encantada
de pagar todos sus costos, pero más que salirse del mercado, lo lógico es que acepte un precio que
apenas salde sus costos variables.
En virtud de esta asimetría, nunca rinde frutos que una empresa nueva tome la iniciativa de
entrar en el mercado de un monopolista natural (a menos que la nueva empresa, por alguna razón,
tenga una ventaja de costo sustancial). Y todo el dinero que gastara en encuestas de mercado, nego-
ciaciones contractuales, etc., se perdería en cuanto la dominante hiciera una mejor oferta, lo cual
siempre le convendría.
Hay otro camino a lo largo del cual la amenaza de entrar podría servir para disciplinar a un mo-
nopolista natural dominante. Aun cuando nunca sería rentable para un aspirante potencial incurrir
en gastos con la intención de penetrar en este tipo de mercado, podría muy bien pagar compradores en
el mercado para aguantar el gasto de introducir a un entrante potencial. Si los compradores absorben
este gasto, pueden esperar de nuevo que la dominante acepte ofrecer un precio competitivo del pro-
ducto. (Si no lo hace, simplemente puede firmar un convenio para comprarle a la empresa externa.)
De modo que si para los compradores es práctico actuar en forma colectiva bajo su propio costo para
negociar con los entrantes potenciales, hasta un monopolista natural dominante se puede ver obliga-
do a vender a un precio cercano al competitivo. Un ejemplo son la mayoría de los gobiernos locales
que actúan como agentes de compra para sus comunidades al negociar contratos con potenciales
proveedores monopolistas de servicios para la comunidad.
En los mercados cuyos productos se venden en privado, muchas veces los compradores son de-
masiados para organizarse y actuar colectivamente de este modo. Pocas son las personas que están dis-
puestas a ir a una junta municipal todas las noches para discutir las negociaciones con los proveedores
potenciales de innumerables productos. Si para los compradores no es práctico organizar una acción
colectiva directa, los agentes privados pueden alcanzar casi el mismo objetivo en nombre propio.
Por ejemplo, ciertas tiendas departamentales podrían ser consideradas como las que representan
esta función. La cadena Sears Roebuck obtuvo su éxito inicial a través de su lema “Calidad a un precio
justo”. Su función era actuar como un agente de compras de la comunidad y negociar contratos con
proveedores de una fuente. Se ganó una reputación al impulsar tratos de compraventa fuertes con
estos proveedores y trasladar el ahorro a sus clientes. ¿Por qué Sears y otras tiendas departamentales
deben trasladar dichos ahorros a través de precios más bajos? Los comerciantes compiten entre sí no
sólo por la venta de los productos manufacturados por monopolistas naturales, sino también por
una enorme lista de productos competitivos. Tener la reputación de ofrecer mercancía de calidad a
precios razonables es un elemento esencial de las estrategias de marketing de muchos de ellos, y los
últimos beneficiados son los consumidores.

COMPETENCIA MONOPOLÍSTICA
La competencia monopolística, una estructura de mercado cercana a la competencia perfecta, ocu-
rre cuando muchas empresas sirven a un mercado con entrada y salida libres, pero en el cual los pro-
ductos de una empresa no son sustitutos perfectos de los de otras compañías. El grado de sustitución
entre productos determina, por lo tanto, cuánto se asemeja la industria a la competencia perfecta.

13_CHAPTER 13.indd 435 3/6/09 8:05:41 PM


436 Capítulo 13 CoMpEtENCIa IMpERFECta: ENFoQuE DE la tEoRía DE JuEGoS

EL MODELO DE CHAMBERLIN
Edward Chamberlin y Joan Robinson desarrollaron de manera independiente, en la década de 1930,
el modelo tradicional de competencia monopolística. Empieza con la suposición de un “grupo de
industrias” bien definido, que consta de una serie de fabricantes de productos que se sustituyen de ma-
nera cercana, aunque imperfecta, entre sí. El mercado de camisas para hombre es un ejemplo conve-
niente. Las camisas hechas por Gant sirven en esencia para el mismo propósito que las fabricadas por
Van Heusen, Ralph Lauren, Arrow o Tommy Hilfiger; sin embargo, para muchos consumidores es
apenas indiferente qué marca comprar.
Hay dos consecuencias importantes que se derivan de estos supuestos acerca de la estructura de
la industria. La primera es que, como los productos se consideran sustitutos cercanos, cada empresa
confronta un esquema de demanda con pendiente negativa. Quien tenga una preferencia particular
por las camisas Gant estará dispuesto a pagar más por ella que por una camisa de otro fabricante.
Pero dejemos que Gant aumente lo suficiente sus precios y a la larga hasta estos compradores se
cambiarán a otra marca. La segunda consecuencia, que se deduce del supuesto de un gran número
de empresas independientes, es que cada empresa actuará como si sus decisiones de precio y canti-
dad no influyeran en el comportamiento de otras empresas de la industria. Y como los productos son
sustitutos cercanos, esto a su vez significa que cada empresa percibe su esquema de demanda como
sumamente elástico.
Una característica fundamental del modelo Chamberlin es la simetría perfecta de la posición de
todas las empresas en la industria. En términos metafóricos, se puede considerar una empresa tipo
Chamberlin como uno de muchos barcos de pesca, cada uno de las cuales tiene una cantidad de cañas
de pesca en el agua. Si un barco colocara en sus anzuelos una carnada más tentadora y los demás
siguieran con la misma, el efecto sería que la innovación aumentaría su participación de la pesca
total por un margen sustancial. Después de todo, sus cañas son más productivas no sólo en términos
absolutos, sino también en relación con las cañas de otros barcos. Sin embargo, como la situación es
perfectamente simétrica, si para un bote es más lógico usar una carnada atractiva, también lo es para
otros. Sin embargo, cuando todos usan una mejor carnada, las cañas del innovador ya no son más
productivas que antes en términos relativos. En consecuencia, la adición a su pesca total será mucho
más reducida que la que tendría si los demás se hubieran mantenido en su comportamiento original.
La analogía entre el ejemplo de la pesca y el comportamiento de asignación de precios por parte
de la empresa monopolísticamente competitiva de Chamberlin es completa. Al contemplar la de-
manda de su propio producto, la empresa supone que sus competidores no responden de manera
alguna a sus decisiones de precios y cantidad. Al igual que los pescadores de los barcos, la empresa
tiene razón en suponer que un cambio de su comportamiento no provocará que los demás cambien.
Sin embargo, la simetría entre las empresas garantiza que si tiene sentido que una empresa altere su
precio, será lógico que las demás hagan lo mismo.
El resultado es que la empresa en realidad se enfrenta a dos curvas de demanda diferentes: una
que describe lo que ocurrirá cuando cambie el precio sola y una segunda que refleja lo que sucederá
cuando todos los precios cambian al unísono. Así, por ejemplo, la curva dd de la figura 13.11 repre-
senta la curva de demanda de la empresa de Chamberlin si sólo ella varía su precio; la curva DD es la
curva de la demanda cuando todas las empresas cambian precios al mismo tiempo. En una situación
inicial en la cual todas las empresas cobran P', cada una venderá Q'. Si sólo una empresa baja su pre-
cio a P″′, venderá Q″′. Pero si otras igualan su recorte de precio, cada una venderá sólo Q″.
Es importante destacar que las empresas individuales necesitan no fallar para darse cuenta de
que los precios de empresas con una situación similar tienden a moverse juntos. Al contrario, cada
empresa puede ser perfectamente consciente de ello. Pero también se da cuenta de que sus movi-
mientos en los precios no son la causa de que otras empresas cambien su comportamiento. Por lo
tanto, cuando piensa en las consecuencias de un movimiento de los precios, se ve obligada a pensar
en términos de movimientos a lo largo de dd y no a lo largo de la curva de la demanda que describe lo
que sucede cuando todos los precios cambian al unísono (DD).16

16
En este caso el problema es el mismo al que se enfrentan los participantes en el dilema del prisionero. Es probable que cada perso-
na sepa que lo racional para el otro individuo es huir y que, por lo tanto, espere que eso haga. Pero cada persona sabe también que
su comportamiento no influirá en lo que haga el otro.

13_CHAPTER 13.indd 436 3/6/09 8:05:42 PM


CoMpEtENCIa MoNopolíStICa 437

P FIGURA 13.11
D Las dos curvas
de demanda
del competidor
d monopolístico
la demanda a la que
se enfrenta cualquier
empresa será más elástica
P’
si los demás mantienen los
precios constantes (dd)
que si todas las empresas
P’’ varían sus precios al mismo
tiempo (DD).
d
D
Q
Q’ Q’’ Q’’’

EQUILIBRIO DE CHAMBERLIN A CORTO PLAZO


Para efectos del ejemplo considere la empresa monopolísticamente competitiva cuyas curvas de
demanda (la curva dd), ingresos marginales, costo total promedio y costo marginal a corto plazo se
presentan en la figura 13.12. Si se sigue al pie de la letra el argumento que se empleó en el caso del
monopolio puro, es posible demostrar con facilidad que la cantidad de maximización de utilidades a
corto plazo es Q*, en cuyo valor la curva de ingresos marginales interseca la curva de costo marginal
a corto plazo. El precio de maximización de utilidades es P*, el valor que corresponde a Q* en la
curva de demanda dd.

FIGURA 13.12
$/Q
Equilibrio a corto plazo
D
para la empresa de
Chamberlin
d la empresa
monopolísticamente
competitiva de Chamberlin
maximiza la utilidad
P* económica a corto plazo al
CMC
CTP igualar el ingreso marginal
y el costo marginal a
corto plazo. la utilidad
económica es Π, el área del
CTP Q* rectángulo sombreado.
D d
IM
Q
Q*

Observe también en la figura 13.12 que la curva de demanda DD interseca la curva de demanda
dd en el precio que maximiza las utilidades, P*. Se trata de otra consecuencia de la simetría funda-
mental que existe en las empresas de Chamberlin. Recuerde que la curva DD es el lugar a lo largo del
cual la cantidad de cada empresa se moverá con el precio si varían al unísono los precios de todas las
empresas. Por el contrario, la curva dd es el lugar geométrico a lo largo del cual la cantidad de una
empresa se mueve con el precio cuando los precios de las demás empresas están fijos. Como la situa-
ción a la que se enfrenta cada empresa es la misma, si P* es el precio de maximización de utilidades de
una, también lo debe ser para las demás. Por lo tanto, el nivel de precio al cual están fijos los precios
de otras empresas a lo largo de dd es P*, lo cual implica que en P* sobre dd el precio de cada empresa
será P*. Por esa razón dd interseca a DD en P*.

13_CHAPTER 13.indd 437 3/6/09 8:05:44 PM


438 Capítulo 13 CoMpEtENCIa IMpERFECta: ENFoQuE DE la tEoRía DE JuEGoS

EQUILIBRIO DE CHAMBERLIN A LARGO PLAZO


Como en el caso de competencia perfecta, el hecho de que haya utilidades económicas a corto pla-
zo tendrá el efecto de atraer a más empresas hacia la industria monopolísticamente competitiva.
¿Cuál es el efecto de la entrada de estas empresas? En el caso competitivo se vio que era desplazar
hacia la derecha la curva de oferta de la industria, lo cual provocaba una disminución en el precio de
equilibrio a corto plazo. Visto de otro modo, el efecto de entrada sobre el modelo de competencia
perfecta es provocar que la curva de demanda horizontal de cada empresa se mueva hacia abajo. En
el modelo de Chamberlin el efecto análogo es desplazar la curva de demanda de cada empresa hacia
la izquierda. Para ser más precisos, si se supone que cada empresa compite en igualdad de circuns-
tancias por una participación en la demanda total de la industria, el efecto de entrada es provocar
una disminución proporcional equitativa en la cantidad que cada empresa puede vender a un precio
determinado. Cada empresa en el mercado reclama en esencia una participación equitativa de la
demanda de la industria y, con más empresas en ésta, necesariamente baja la participación.

EJERCICIO 13.5
Cada una de las 20 empresas en una industria monopolísticamente competitiva de Chamberlin
enfrenta la curva dd dada por P = 10 – 0.001Q. ¿Cuál será la curva dd de cada empresa después
de la entrada de cinco empresas nuevas?

Después del cambio a la izquierda en la demanda provocada por la entrada de empresas, cada
una tiene la oportunidad de reajustar el tamaño de su capital accionario y elegir su nuevo nivel de
producción que maximice las utilidades. Si se mantiene una utilidad superior a lo normal, continuará
la entrada.
La posición de equilibrio a largo plazo es aquella en la cual la curva de demanda dd se desplaza
hacia la izquierda al punto en que es tangente a la curva de costo promedio a largo plazo (y también
a la curva de costo promedio a corto plazo asociada). Observe en la figura 13.13 que Q*, el nivel de
producción que maximiza las utilidades según el criterio IM = CM, es exactamente igual al nivel
de producción para el cual la curva dd es tangente a las curvas de costo promedio a corto y largo
plazos. No es una simple coincidencia. Es posible argumentar al margen de la condición IM = CM
que el punto de tangencia debe ser igual al punto de utilidad máxima. En éste, la empresa tiene una
utilidad económica de cero, lo cual significa que la utilidad económica sería negativa.
Observe de nuevo en la figura 13.13 que la curva de demanda DD interseca a dd en el precio de
equilibrio P* por los motivos antes analizados. Si cada empresa aumentara el precio a partir de P*,

FIGURA 13.13
Equilibrio a corto $/Q D
plazo en el modelo de
Chamberlin
Cuando se presenta la d
entrada de empresas,
desplaza dd a la izquierda CMC
hasta ser tangente a la P*
curva Cpl. la empresa ATC
produce Q*, vende a P*
y obtiene una utilidad CPL
económica nula.

d
D IM
Q
Q*

13_CHAPTER 13.indd 438 3/6/09 8:05:46 PM


CoMpEtENCIa MoNopolíStICa 439

se moverían en forma ascendente a lo largo de DD y obtendrían una utilidad económica. Pero en


ausencia de un acuerdo de colusión obligatorio, a ninguna empresa le interesaría mantener su precio
por encima de P*, porque en este caso su ingreso marginal (a lo largo de la curva IM asociada con dd)
superaría su costo marginal. En cualquier precio superior a P*, podría obtener mayores utilidades si
recorta el precio y vende más producción. El único resultado estable es el punto de tangencia que se
muestra en la figura 13.13.

COMPETENCIA PERFECTA VERSUS COMPETENCIA


MONOPOLÍSTICA DE CHAMBERLIN
Hay varios puntos de comparación obvios entre las posiciones de equilibrio a largo plazo de la com-
petencia perfecta y la competencia monopolística de Chamberlin. Primero, la competencia cumple
con la prueba de eficiencia de asignación, la competencia monopolística no. Con la competencia el
precio es exactamente igual al costo marginal a largo plazo, lo cual significa que no hay posibilidades
sin explotar para las ganancias mutuas a través de la negociación. Por el contrario, con la competen-
cia monopolística, el precio supera el costo marginal, incluso a largo plazo. Lo anterior significa que
en la sociedad hay gente que valora mucho más una unidad adicional de producción que el valor de
los recursos requeridos para producirla. Si los competidores monopolísticos pudieran encontrar una
forma de reducir el precio para dichos compradores sin recortar el precio de las ventas existentes, lo
harían con gusto, ayudando a todos en el proceso. Como se vio en el caso del monopolista, a veces
se puede lograr el recorte selectivo de precios. Pero es imperfecto por naturaleza y, a este respecto,
la competencia monopolística está destinada a estar por debajo de la competencia perfecta en el
estrecho estándar de eficiencia.
Algunos economistas también argumentan que la competencia monopolística es menos eficien-
te que la competencia perfecta porque en el caso anterior las empresas no producen en los puntos
mínimos de sus curvas de costo promedio a largo plazo (CPL). Sin embargo, no es una comparación
que diga mucho porque los demás aspectos de los dos casos son muy diferentes. La pregunta relevan-
te es si la gente que hoy compra a los competidores monopolísticos estaría más contenta si los pro-
ductos fueran exactamente iguales y tuvieran un precio un poco menor. No es una pregunta sencilla
y, a falta de una respuesta, no es posible concluir que la competencia monopolística sea ineficiente
sólo porque las empresas no producen en los puntos mínimos de sus curvas CPL.
Existe al menos un sentido, relacionado con el análisis del párrafo anterior, en el que el mo-
delo de Chamberlin es mucho más realista que el de competencia. Recuerde que, en el caso de la
competencia perfecta, el precio y el costo marginal son los mismos en equilibrio. Esto implica que
la empresa debe reaccionar con indiferencia ante una oportunidad para surtir un nuevo pedido al
precio actual de mercado. Por el contrario, en el caso de la competencia monopolística, el precio su-
pera al costo marginal, lo cual implica que la empresa debe recibir con entusiasmo un nuevo pedido
al precio actual de mercado. Aparte de los periodos de escasez, se conocen pocos casos de la reacción
anterior. Casi todo empresario disfruta de aceptar nuevos pedidos al precio actual de mercado.
Por último, en términos de rentabilidad a largo plazo, las posiciones de equilibrio del competi-
dor perfecto y del competidor monopolístico de Chamberlin son exactamente iguales. La libertad de
entrada de cada caso mantiene la utilidad económica a largo plazo en cero. En este mismo sentido, la
libertad de salida asegura que no habrá pérdidas a largo plazo en ningún caso.

CRÍTICAS AL MODELO DE CHAMBERLIN


George Stigler, ganador del premio Nobel, y otros han criticado el modelo de Chamberlin por varios
motivos. Ante todo, es difícil definir a qué se refería el concepto amorfo de “grupo de industria”. El
concepto de Chamberlin abarca un grupo de productos que son diferentes entre sí de cierta forma no
especificada, pero que tienen la misma probabilidad de ser atractivos para un comprador determina-
do. Stigler se quejó de que, en la práctica, es imposible trazar de esta manera límites operacionales
entre grupos de productos. Visto desde una perspectiva, un producto como Coca-Cola es único en
sí mismo. (Por lo tanto, los compradores se enojaron cuando Coca cambió ligeramente su fórmula
tradicional, lo cual orilló a la compañía a reintroducir la “Coca clásica”.) Sin embargo, visto desde un

13_CHAPTER 13.indd 439 3/6/09 8:05:46 PM


440 Capítulo 13 CoMpEtENCIa IMpERFECta: ENFoQuE DE la tEoRía DE JuEGoS

ángulo diferente, Coca es un sustituto de Pepsi, que sustituye a la leche, la cual a su vez sustituye al
helado. Pero el helado sustituye al pastel de chocolate, que en definitiva no parece un sustituto de la
Coca-Cola. Con esta lógica en extremo, Stigler argumentaba que el grupo de productos de Cham-
berlin se expande e incluye con rapidez a casi cada producto de consumo en la economía.
Desde un punto de vista metodológico, Stigler apoyó el argumento de Milton Friedman de
que no se debe juzgar una teoría por la precisión descriptiva de sus supuestos, sino por su capacidad
para pronosticar las respuestas a cambios en el ambiente económico. (Vea el capítulo 1.) Stigler
consideraba que la teoría de Chamberlin complica de manera significativa la teoría de la competen-
cia perfecta, sin alterar mucho sus proyecciones más importantes. Con respecto a muchos temas
específicos, esta imputación tiene un mérito evidente. Por ejemplo, ambas teorías pronostican que
la utilidad económica atrae la entrada de empresas, la cual reduce los precios y elimina las utilidades
a largo plazo.
Pero la crítica más reveladora del modelo de Chamberlin no es que se parezca mucho al modelo
de competencia, sino en que al menos en un aspecto muy importante no se desvía lo suficiente de
él. El problema radica en el supuesto crítico de que cada empresa tiene la misma oportunidad de
atraer a cualquier comprador de una industria. En algunos casos esta descripción es bastante precisa,
pero falla en otros. Por ejemplo, en la industria del cereal para el desayuno, la persona que compra
Fruit-‘n-Fiber podría pensar en cambiar a Grape-Nuts o Shredded Wheat, pero jamás se le ocurriría
cambiar a Captain Crunch o Fruit Loops.
En años recientes la investigación sobre la teoría de la competencia monopolística se ha enfocado
en modelos que incorporan características específicas de un producto que hace que los compradores
los prefieran por encima de otros. Opuestamente al modelo Chamberlin, estos modelos producen
conclusiones que con frecuencia difieren en gran medida de las del modelo de competencia perfecta.
Ahora se analizará esta otra manera de pensar acerca de la competencia monopolística.

INTERPRETACIÓN ESPACIAL
DE LA COMPETENCIA MONOPOLÍSTICA
Como se observó antes, el grado en que el producto de una empresa de competencia monopolística
es un sustituto eficaz de otro determina cuánto se parece la industria a la competencia perfecta. Una
idea concreta sobre la falta de sustitución completa es la distancia. La gasolina que se vende fuera de
la ciudad no es el sustituto perfecto de la gasolina que se vende a la vuelta de la esquina, sobre todo
cuando el tanque está casi vacío.
Imagine que es residente de un pequeño país en una isla con un lago grande en el centro. Por
naturaleza, la actividad empresarial se ve limitada por la forma de dona que constituye la periferia
de la isla. Hay una especialización considerable de mano de obra en su isla. La gente se dedica a sus
respectivas obligaciones durante el día y cena en restaurantes. Los pobladores carecen de la acostum-
brada preferencia por la diversidad culinaria; en cambio, sus vecinos y usted prefieren cenar siempre
papas al horno y carne asada. Los alimentos de cualquier restaurante se producen con rendimientos
crecientes de escala: cuantos más alimentos se producen, menor es el costo promedio por alimento.
¿Cuántos restaurantes debería haber en este país isleño? Estaríamos tentados a decir que sólo
uno para mantener así el costo por alimento al mínimo. Si la circunferencia de la isla fuera, por decir,
de 300 yardas, casi seguro sería la respuesta correcta. Pero en una isla más grande, es probable que
el costo directo de los alimentos no sea la única preocupación para usted y los demás residentes.
También le preocupará el costo de ida y vuelta hacia el restaurante más cercano. Si, por ejemplo, la
isla fuera de 300 millas a la redonda, el ahorro en el costo de tener un solo restaurante difícilmente
compensaría el costo del viaje en que incurrirían quienes viven al otro lado de la isla.
El mercado de las cenas en esta isla en cierto modo es igual al de los mercados que se han con-
siderado en capítulos anteriores: en todos los restaurantes se sirve un solo alimento estandarizado.
Pero el tipo de alimento que se sirve no es la única característica importante. A los compradores
también les interesa dónde se sirve la comida. Cuando los productos difieren en una o más dimensio-

13_CHAPTER 13.indd 440 3/6/09 8:05:47 PM


INtERpREtaCIóN ESpaCIal DE la CoMpEtENCIa MoNopoIlíStICa 441

nes importantes, como ubicación, tamaño, sabor, calidad, etc., de inmediato se plantea la pregunta
general de cuánta diversidad de productos debe haber. ¿Una economía debe tener 5, 10 o 15 marcas
de autos diferentes? ¿Cuántos tipos de raquetas para tenis debe haber?
Para fijar las ideas, suponga que al inicio hubiera cuatro restaurantes a la misma distancia al-
rededor de la periferia de la isla, como se presenta en los cuadros de la figura 13.14. Imagine que la
circunferencia de la isla es de una milla. Entonces la distancia entre los restaurantes sería de –14 de milla
y nadie vivirá a más de –18 de milla del restaurante más cercano, el viaje de ida que tendría que hacer
alguien que viviera exactamente a medio camino entre dos restaurantes.

FIGURA 13.14
Industria en la que
la ubicación es
un diferenciador
importante
los restaurantes (cuadros)
son iguales excepto en
su ubicación geográfica.
Cada persona cena en el
restaurante más cercano a
su casa. Si la circunferencia
1
del anillo es de una milla,
8 significa que la distancia
entre restaurantes es de
1
–4 de milla, lo que da un
viaje sencillo máximo de –18
1 de milla.
4

Para completar la estructura del lugar, suponga que hay L consumidores esparcidos de mane-
ra uniforme alrededor del círculo y que el costo del viaje es de t dólares por milla. Entonces, por
ejemplo, si t es igual a 24 dls./milla, el costo de transporte en el que incurre una persona que vive a
–1 de milla del restaurante más cercano sería el producto de la distancia del viaje redondo (2d)
d = 16
1– milla) = 3 dólares.
por el costo unitario del viaje (t): 2td = 2(24 dls./milla)( 16
Además, suponga que cada consumidor come exactamente un alimento al día en el restaurante,
por el cual el precio total (el que se cobra por el alimento más los costos del transporte) es el más
bajo. Por último, suponga que cada restaurante tiene una curva de costo total de

CT = F + MQ. (13.6)

En los primeros capítulos se vio que la curva de costo total de la ecuación 13.6 tiene un costo
fijo F y un costo marginal constante M. En este caso F podría considerarse como la suma del alqui-
ler de equipo, costos de oportunidad del capital invertido y otros costos fijos asociados con la actividad
de un restaurante; y M es la suma de la mano de obra, materia prima y demás costos variables en que
se incurre al producir un alimento adicional.
Además recuerde que el costo total promedio (CTP) es simplemente el costo total dividido en-
tre la producción. Con una función de costo total dada por CT = F + MQ, el costo total promedio es
igual a F/Q + M. Lo anterior significa que cuantos más clientes sean atendidos en un lugar determi-
nado, más bajo será el costo total promedio.
Por ejemplo, suponga que cada uno de los cuatro restaurantes tiene una curva de costo total
medida en dólares por día, definida por CT = 50 + 5Q, donde Q es el número de alimentos que se sir-
ven al día. Si la población, L, es igual a 100 personas, cada restaurante servirá (100/4) = 25 alimentos
al día y su costo total se obtiene con CT = 50 + 5(25) = 175 dólares/día. El costo total promedio de
cada restaurante es CT/25 = (175 dls./día)/(25 alimentos/día) = 7 dls./alimento. En comparación,

13_CHAPTER 13.indd 441 3/6/09 8:05:48 PM


442 Capítulo 13 CoMpEtENCIa IMpERFECta: ENFoQuE DE la tEoRía DE JuEGoS

si sólo hubiera dos restaurantes, cada uno serviría 50 alimentos al día y tendría un costo total prome-
dio de sólo 6 dls./alimento.
¿Cuál sería el costo promedio del transporte con cuatro restaurantes? Depende de los costos
unitarios de transporte (t) y de la distancia entre los restaurantes. Recuerde que la distancia entre
los cuatro restaurantes es de –14 de milla. Algunas personas viven al lado del restaurante, así que su
costo de transporte es cero. Con cuatro restaurantes, lo más lejano que vive alguien de un restau-
rante es –18 de milla, que es la distancia de ida para una persona que vive a medio camino entre dos
restaurantes. Para ella el viaje de ida y vuelta implica recorrer un cuarto de milla; y si t de nuevo es
igual a 24 dls./milla, el costo del viaje para esta persona es de (24 dls./milla)( –14 milla) = 6 dólares. Co-
mo las personas están dispersas en forma equitativa alrededor del anillo, el viaje redondo promedio
es la mitad entre dos extremos; por lo tanto, abarca una distancia de –18 de milla y el costo es de 3
dólares.
El costo total promedio por alimento es la suma del costo total promedio (7 dls./alimento en el
ejemplo de cuatro restaurantes) y el costo de transporte promedio (en este caso, 3 dólares por ali-
mento) da como resultado 10 dls./alimento.

NÚMERO ÓPTIMO DE UBICACIONES


Si el costo de transportación unitario (t) fuera cero, entonces es claro que lo óptimo sería tener un solo
restaurante, porque minimizaría el costo promedio general por alimento. Si el costo de transporte
fuera lo suficientemente alto, no sería óptimo tener sólo uno porque el consumidor promedio tendría
que viajar una distancia muy grande. El número óptimo de restaurantes es, por lo tanto, el resulta-
do de una compensación entre el establecimiento y otros costos fijos (F) necesarios para la apertura
de ubicaciones nuevas por un lado y el ahorro de costos de transporte más bajos por el otro.
¿Cuál es el mejor número de locales? La estrategia para responder a esta pregunta sería pregun-
tar si disminuiría el costo promedio general por alimento servido (costo total promedio más costo
de transporte promedio) si se tuviera un restaurante más de los que existen ahora. De ser así, se
debe agregar otro restaurante y repetir la pregunta. En cuanto deje de disminuir el costo promedio
general se habrá alcanzado el número óptimo de restaurantes.
Para ilustrarlo, suponga que se aumenta el número de restaurantes del ejemplo anterior de cua-
tro a cinco. ¿Cómo afectará esto el costo promedio general? Suponga de nuevo que los restaurantes
están distribuidos en forma equitativa en el anillo, y ahora cada uno atraerá a una quinta parte de
los 100 habitantes de la isla, lo cual significa que cada uno servirá 20 alimentos al día. Por lo tanto, el
CTP de cada restaurante es [50 + 5(20)]/20 = 7.50 dls./alimento, 0.50 dl./alimento más que el valor
anterior. (Recuerde que el CTP con cuatro restaurantes era de 7 dls./alimento.) Con cinco restau-
rantes, la distancia entre dos adyacentes es de –13 de milla. Esto significa que el viaje de ida promedio
con cinco restaurantes es de 20 –1 de milla, lo cual a su vez significa que la distancia promedio de un
1
– de milla. El costo de transporte promedio es, por lo tanto, ( 10
viaje de ida y vuelta es de 10 –1 milla)(24
dls./milla) = 2.40 dólares. Observe que sólo representa 60 centavos de dólar menos que el costo de
transporte promedio anterior de 3 dólares, esto refleja la disminución de la longitud del viaje prome-
dio. Al sumar el costo total promedio y el costo de transportación promedio, se tiene que el costo
total promedio con cinco restaurantes es de 7.50 dls. + 2.40 dls. = 9.90 dls./alimento.

EJERCICIO 13.6
En el ejemplo anterior, ¿cuál es el costo promedio total por alimento si se agrega un sexto
restaurante al anillo?

El cálculo del ejercicio 13.6 demuestra que el costo total promedio por alimento aumenta
cuando se incrementa la cantidad de restaurantes de cinco a seis. Y como el costo total promedio
bajó cuando se pasó de cuatro a cinco, significa que el número óptimo de restaurantes para la isla
es de cinco.
Es posible hacer más general el análisis anterior si se supone que hay N locales alrededor del ani-
llo, como lo muestran los cuadros de la figura 13.15. Ahora la distancia entre los locales adyacentes

13_CHAPTER 13.indd 442 3/6/09 8:05:49 PM


INtERpREtaCIóN ESpaCIal DE la CoMpEtENCIa MoNopolíStICa 443

es de 1/N y la distancia máxima del viaje de ida es de la mitad, es decir 1/2N. Si se supone de nuevo
que la gente está distribuida en forma equitativa alrededor del anillo, la distancia promedio de ida
al punto de venta más cercano es 1/4N (que es la mitad del camino entre 0, la distancia que recorre
la persona más cercana a un punto de venta determinado, y 1/2N, la distancia para la persona más
alejada del mismo). El viaje redondo promedio es el doble de la distancia de ida promedio y, por lo
tanto, igual a 1/2N.

FIGURA 13.15
Distancias con N locales
Con N locales, la distancia
entre locales adyacentes
es 1/N. lo más lejos que
puede vivir una persona
de un local es 1/2N.
Y la distancia de ida
promedio que debe viajar
una persona al local más
1 cercano es de 1/4N. la
distancia del viaje redondo
4N promedio es 1/2N.
1
1
2N
N

Como la distancia entre restaurantes disminuye conforme aumenta el número de los mismos, el
costo de transporte total, Ctrans, es una función decreciente del número de locales. Como el costo de
transporte es t dólares por persona por milla recorrida, el costo de transporte total será el producto
—1 ):
del costo por milla (t), población (L) y la distancia del viaje redondo promedio ( 2N

1
C trans = tL . (13.7)
2N

El costo total de los alimentos servidos, Calim, también depende de la población y el número de loca-
les, se obtiene mediante

Calim = LM + NF, (13.8)

donde el primer término a la derecha refleja el hecho de que cada persona L come un alimento cuyo
costo marginal es M, y el segundo término es el costo fijo total de N locales. El objeto es elegir N para
minimizar la suma de los dos tipos de costos, Ctrans + Calim.
Las dos funciones de costo y su suma se presentan gráficamente en la figura 13.16, donde N*
representa el número de locales que minimiza los costos.17
La pendiente de la curva Calim es igual a F, que representa el costo de un local más. La pendiente
de la curva Ctrans es igual a –tL/2N2 y representa los ahorros en el costo de transporte derivados de

17
Estas funciones se grafican como si N fuera una variable continua, no un entero. Para las industrias que involucran números
grandes de empresas, la aproximación de continuidad sólo introducirá un error mínimo.

13_CHAPTER 13.indd 443 3/6/09 8:05:51 PM


444 Capítulo 13 CoMpEtENCIa IMpERFECta: ENFoQuE DE la tEoRía DE JuEGoS

FIGURA 13.16
Número óptimo de dls./día
locales
El costo de transporte
total (Ctrans) disminuye con
el número de locales (N),
en tanto el costo total
Calim + Ctrans
de los alimentos servidos
(Calim) aumenta con N. El Calim
número óptimo de locales
(N*) es el que minimiza la
suma de estos costos.

Ctrans

Número de locales

añadir otro local.18 Si la pendiente de Calim es menor que el valor absoluto de la pendiente de Ctrans,
la disminución en el costo de transporte por agregar otro local compensaría con creces el costo fijo
adicional de incluir dicho local. El número óptimo de locales, N*, es aquel para el cual la pendiente
de la curva de Calim es igual al valor absoluto de la pendiente de la curva Ctrans. Por lo tanto, N* debe
satisfacer
tL
= F, (13.9)
2N * 2
que produce
tL
N* = . (13.10)
2F

Esta expresión del número óptimo de locales tiene una interpretación económica directa. Obser-
ve primero que si aumenta el costo de transporte, también aumentará N*. Lo anterior tiene sentido
porque el objetivo de incluir más locales es ahorrar en costos de transporte. Cabe notar también que
N* aumenta con la densidad de población, L. Cuantas más personas vivan en el segmento del anillo,
más serán las beneficiadas si se acorta la distancia promedio al local más cercano. Por último, observe
que N* disminuye con F, el costo de abrir un local adicional, que también es como se esperaba.
Al aplicar la ecuación 13.10 al ejemplo de los restaurantes en el que L = 100, t = 24 y F = 50, se
obtiene N* = √(2 400/100) = 4.9. Está por demás decir que es imposible que haya 4.9 restaurantes,
así que se elige el entero más cercano a 4.9, es decir, 5. Y, de hecho, como se vio en los cálculos

18
Para calcular esta pendiente, se obtiene la derivada

d (C trans ) −tL
= ,
dN 2N 2
tratando siempre a N como si fuera una variable continua. Los estudiantes que han estudiado cálculo se pueden convencer de que
esta expresión es correcta al dejar que ∆N sea, por ejemplo, 0.001 y después calcular el cambio resultante en Ctrans,

tL tL −0.001tL
∆C trans = − = .
2( N + 0.001) 2N 2N ( N + 0.001)
Por lo tanto, la razón ∆Ctrans/∆N es
∆C trans −tL −tL
= ≈ .
0.001 2N ( N + 0.001) 2N 2

13_CHAPTER 13.indd 444 3/6/09 8:05:55 PM


INtERpREtaCIóN ESpaCIal DE la CoMpEtENCIa MoNopolíStICa 445

anteriores, tener cinco restaurantes da como resultado un costo promedio general menor que el de
cuatro o seis restaurantes.

EJERCICIO 13.7
¿Cómo cambiaría N* en el ejemplo anterior si hubiera 400 personas en la isla en vez de 100?

¿Las acciones independientes de empresas privadas que buscan utilidades dan como resultado
un número óptimo de locales alrededor del anillo? Esta pregunta suena muy simple, pero es muy
difícil de responder. Ahora sabe que en ciertas condiciones habrá una tendencia a un número mayor
al óptimo, mientras que en otras el número será menor.19 Pero por el momento sólo se verá que el
número de locales que surge de las acciones independientes de las empresas que buscan utilidades
en general se relaciona con el número óptimo de la siguiente forma: cualquier cambio ambiental
que conduce a un cambio en el número óptimo de locales (en este caso, cualquier variación en la
densidad demográfica, el costo de transporte o el costo fijo) generará un cambio en la misma direc-
ción en el número de equilibrio de locales. Por ejemplo, una caída del costo de transporte tenderá a
disminuir el número óptimo de locales y el número de locales que de hecho se ven en la práctica.

¿Por qué en la mayoría de las ciudades ahora hay menos tiendas de abarrotes que en 1930? EL NATURALISTA
¿Por qué los barrios residenciales de Nueva York tienen más tiendas de abarrotes que los
barrios residenciales de Los Ángeles?
ECONÓMICO
La venta de abarrotes al menudeo, igual que otras formas minoristas, se caracteriza por economías 13.4
de escala sólidas. Por lo tanto, contempla la compensación usual entre, por un lado, el costo directo
de producción y, por otro, el costo de transporte. En el siglo
xx, los patrones cambiantes en el parque automovilístico
afectaron el tamaño y la ubicación de las tiendas de abarro-
tes en Estados Unidos. En 1920 casi ninguna familia tenía
auto y debían hacer sus compras a pie. En términos de la
expresión que se dedujo para el número óptimo de locales
(ecuación 13.10), lo anterior significaba un valor alto de t,
costo de transporte unitario. Desde luego que hoy en día
casi todas las familias tienen coche, lo cual ha provocado
que la gente aproveche los precios más bajos de tiendas más
grandes. Una excepción a este esquema es Manhattan. In-
cluso ahora casi ningún residente de Manhattan tiene auto.
Además, la densidad demográfica es sumamente alta, lo
que representa un valor alto de L en la ecuación 13.10. El
efecto combinado de valores altos de L y t es que muy pocos
residentes de Manhattan tienen que caminar más de dos
cuadras para llegar a la tienda de abarrotes más cercana. La
población total de Los Ángeles también es muy alta, pero
está distribuida en un área mucho más grande y casi todas
las familias tienen al menos un automóvil. El resultado es
que las tiendas de abarrotes de Los Ángeles son más gran-
des y están más lejos que sus contrapartes de la ciudad de
Nueva York.

19
Se encuentra una detallada exposición técnica de algunas cuestiones rele-
vantes en Avinash Dixit y Joseph Stiglitz, “Monopolistic Competition and
Optimal Product Diversity”, en American Economic Review, 1977, pp. 297-308;
¿Por qué las tiendas de abarrotes están más cerca en la ciudad de Nueva
y A. Michael Spence, “Product Selection, Fixed Costs, and Monopolistic Com-
petition”, en Review of Economic Studies, 1976, pp. 217-235. York que en Los Ángeles?

13_CHAPTER 13.indd 445 3/6/09 8:05:56 PM


446 Capítulo 13 CoMpEtENCIa IMpERFECta: ENFoQuE DE la tEoRía DE JuEGoS

LA ANALOGÍA DE LAS CARACTERÍSTICAS DE LOS PRODUCTOS


El poder de la interpretación espacial de la competencia monopolística es que no sólo se puede apli-
car a la ubicación geográfica, sino también a una variedad de otras características de los productos.
Por ejemplo, tome en cuenta los diferentes vuelos aéreos entre dos ciudades en un día determinado.
La gente tiene preferencia por viajar a distintas horas del día, así como diversos gustos para comer o
hacer compras. La figura 13.17 muestra un mercado de viajes aéreos (por ejemplo de Kansas City a
Minneapolis) con cuatro vuelos al día programados a medianoche, 6 a.m., mediodía y 6 p.m. Con la
elección del vuelo aéreo, así como del lugar donde cenar, la gente suele preferir la alternativa que se
acerca más a su opción más preferida. Por lo tanto, una persona que prefiriera viajar a las 7 p.m., quizá
tome el vuelo de las 6 p.m. En términos del modelo espacial, tener que esperar un vuelo es análogo a
tener que desplazarse cierta distancia para llegar a una tienda.

FIGURA 13.17
Interpretación espacial
de la programación de 12 medianoche
vuelos
En un mercado con cuatro
vuelos al día, no hay viajero
para el que no haya un
vuelo que difiera en más
de 3 horas de su hora de 6 p.m . 6 a .m .
salida preferida.

12 mediodía

¿Por qué no sale un avión cada cinco minutos para que nadie se vea obligado a viajar a una
hora inconveniente? La respuesta, de nuevo, se relaciona con la compensación entre el costo y la
conveniencia. Cuanto más grande es el avión, más bajo es el costo promedio por asiento. Si la gente
quisiera vuelos frecuentes, las líneas se verían obligadas a usar aviones más pequeños y cobrar tarifas
más altas. Por el contrario, si a la gente no le importara la hora en que tiene que viajar, la línea aérea
podría usar el avión más grande posible (en la flota de la actualidad sería en Airbus A380 con 853
asientos) y volar en el intervalo que fuera necesario para acumular suficientes pasajeros y llenar el
avión (en el mercado Paducah, Kentucky-Klamath Falls, Oregon sería ¡un vuelo cada ocho años!)
Pero la mayoría de los pasajeros tienen horarios que cumplir y están dispuestos a pagar un poco
más por un vuelo con un horario más conveniente. El resultado se basa en el mismo tipo de razona-
miento que se vio en los ejemplos del restaurante y de la tienda de abarrotes.
Casi cada producto de consumo se puede interpretar de manera fructífera dentro del contexto
del modelo espacial. Por ejemplo, en el mercado automotriz, la disponibilidad de permutas entre tur-
bo o sin turbo, automático o estándar, coupé o convertible, sedán o camioneta, dos o cuatro puertas,
asientos individuales o unidos, con o sin aire acondicionado, índigo metálico o verde bosque, etc., ha
llevado a una cantidad extraordinariamente grande de posibilidades. Desde luego que sería mucho
más barato si sólo se tuviera un solo modelo estándar. Pero la gente está dispuesta a pagar un po-
co más por la variedad, así como por una tienda con una ubicación más conveniente. En el lenguaje
del modelo espacial se dice que los fabricantes de automóviles “localizan” sus modelos en un “espa-
cio del producto”. Su objetivo es que pocos compradores se queden sin una opción que se “acerque”
al auto que más les convenga. Se aplican interpretaciones semejantes a cámaras, estéreos, vacacio-
nes, vehículos, relojes de pulsera, anillos matrimoniales y casi todos los productos para los cuales la
gente tiene un gusto por la variedad.

13_CHAPTER 13.indd 446 3/6/09 8:05:57 PM


INtERpREtaCIóN ESpaCIal DE la CoMpEtENCIa MoNopolíStICa 447

SE PAGA POR LA VARIEDAD


Como se ha visto, la variedad cuesta. Muchos críticos afirman que las economías de mercado des-
perdician elevados niveles de variedad de productos. Estos críticos preguntan si el mundo no sería
un mejor lugar si se tuviera una variedad de productos más sencilla de dónde elegir que costara
menos que los modelos sumamente específicos de la actualidad. Los costos extra de una variedad de
productos de hecho puede parecer una carga innecesaria impuesta a los consumidores que no desean
o simplemente no pueden comprar. Sin embargo, en un modelo de competencia espacial un poco
más detallado estos problemas parecen ser menos graves.
En el modelo simple que se ha expuesto aquí se supone que cada comprador se enfrentó al
mismo costo de transporte unitario. Esta suposición es irreal incluso en modelos en los que la única
dimensión de variedad es la ubicación geográfica. Por ejemplo, los compradores que posean auto-
móviles tendrán costos de transporte mucho más bajos que quienes no lo tienen; del mismo modo
que los compradores cuyo tiempo tiene poco valor, en términos del costo de oportunidad, tienen
menos gastos de transporte que aquéllos con tiempo más valioso. Ocurre lo mismo con la variedad
de productos. Los que se preocupan mucho por las características especiales del producto tienen
“costos de transporte” más altos que quienes no se preocupan, lo cual simplemente significa que la
gente del primer grupo está dispuesta a pagar más que los demás por un producto cuyas característi-
cas especiales se ajustan mejor a sus gustos particulares.
También es cierto que, como propuesta general, la demanda de variedad aumenta en gran
medida con el ingreso. En el lenguaje del capítulo 5, la variedad es un lujo, no una necesidad. La
asociación entre ingreso y demanda de variedad representa una función esencial en la forma que la
mayoría de los fabricantes comercializa sus productos. Considere por ejemplo la diversidad de auto-
móviles que ofrece General Motors, que va desde el subcompacto Aveo de Chevrolet hasta el STS
Cadillac de gran tamaño. Estos autos incorporan una variedad de características especializadas que
son resultado de una investigación y desarrollo muy caros. Los costos de investigación y desarrollo
son fijos, lo que da origen a una economía de escala sustancial en la producción de autos. Si GM (o
cualquier otra empresa) pudiera vender más coches en un año determinado, produciría cada uno a
un costo más bajo.
Cuando el costo marginal se encuentra por debajo del costo promedio, no se puede cobrar a
cada comprador un precio igual al costo marginal y tener de todas maneras una utilidad normal.
(Recuerde del capítulo 10 que el costo promedio de una empresa incluye una tasa de rendimiento
normal.) Como se vio en el capítulo 12, la empresa con economía de escala tiene un incentivo para
expandir su mercado estableciendo el precio cerca del costo marginal, si puede hacerlo sin alterar los
precios de los productos que tiene a la venta. Pero también se vio que si algunos compradores pagan
precios por debajo del costo promedio, otros deben pagar precios por arriba del mismo.

FIGURA 13.18
$/Q $/Q Distribución del costo
de la variedad
Chevrolets Cadillacs
los compradores que
se preocupan más por la
variedad por lo general
48 000 eligen el modelo con
características de alto
valor. al fijar los precios
de sus modelos de manera
14 000 24 500 CM Cad diferente, el vendedor
10 000 CM Chev recupera la mayoría de
los costos extra de la
IM Chev DChev IMCad DCad variedad cobrándosela a los
compradores responsables
QChev QCad
de que ocurra.
a) b)

13_CHAPTER 13.indd 447 3/6/09 8:05:58 PM


448 Capítulo 13 CoMpEtENCIa IMpERFECta: ENFoQuE DE la tEoRía DE JuEGoS

La respuesta de los fabricantes de automóviles a esta situación es establecer los precios de sus
mejores modelos por arriba del costo promedio, mientras fijan el precio de los modelos inferiores
por debajo del mismo. Las secciones a) y b) de la figura 13.18 muestran, por ejemplo, el costo y las
condiciones de demanda de los Chevrolet y Cadillac, respectivamente. El costo marginal de produc-
ción de un Cadillac sólo es un poco superior al de fabricar un Chevrolet. La compañía cuenta con
las innovaciones de diseño básicas de ambos coches y las partes producidas para el Cadillac son un
poco más caras que las que se usan en el Chevrolet. Pero como los compradores a los que les interesa
la variedad están dispuestos a pagar mucho más por un Cadillac que por un Chevrolet, la compañía
puede fijar precios marcadamente diferentes para los dos modelos. En un año promedio el excedente
del ingreso total sobre el costo variable de los modelos (que es la suma de los dos rectángulos som-
breados de la figura 13.18) sólo es lo suficiente para pagar los costos de investigación y desarrollo de
la compañía y otros costos fijos.
Como dicen los críticos, la variedad cuesta mucho. Pero el costo de la variedad no se distribuye
de manera uniforme entre los compradores. En el ejemplo anterior lo pagaron los compradores de
los Cadillac y no aquellos que adquieren un Chevrolet. Del mismo modo, las personas que compran
el BMW 328i disfrutan casi todas las ventajas importantes del extenso programa de investigación de
BMW por 6 000 dólares menos que quienes compran el BMW 330i, cuya diferencia con el 328i es un
motor un poco más potente. Incluso con el argumento que usaron los críticos del sistema de merca-
do, parece un mejor resultado que el que pedían con insistencia; es decir, que todos los compradores
tengan un “auto popular” estandarizado. Según los acuerdos comerciales actuales, la gente que no
valora mucho la variedad disfruta a expensas de quienes se preocupan más por tenerla. La alternativa
sería negar la variedad a la gente que le interesa mucho sin generar un ahorro real en el costo para
los compradores que estarían satisfechos con un producto estandarizado. Incluso Volkswagen, que
alguna vez se jactó de las virtudes de un modelo único estandarizado, ahora ofrece varias docenas de
versiones de sus líneas Golf, Jetta y Passat.
Estrategias de precio semejantes afectan la recuperación de los costos de variedad de casi todas
las industrias. Volviendo a la industria restaurantera. En una ciudad en la que la mayoría de sus habi-
tantes tienen auto, es decir, en casi todas, la forma más económica de ofrecer alimentos preparados
sería tener un restaurante único con un solo platillo en el menú. Un ejército de cocineros trabajaría
en calderos gigantescos de chícharos y puré de papa, en tanto otros se encargarían de enormes hor-
nos con pollos rostizados. Pero la gente no quiere comer lo mismo todas las noches, así como tampo-
co quiere el mismo coche. Incluso en pequeñas ciudades como Ithaca, Nueva York, hay restaurantes
de comida india, mexicana, tailandesa, china, japonesa, coreana, griega, italiana, francesa, vietnamita
y española, además de las cadenas de comida rápida y nacional.
¿Cómo se distribuyen los costos extra en esta variedad? La mayoría de los restaurantes fijan los
precios de los diferentes platillos de su menú en diversos múltiplos del costo marginal. Las bebidas
alcohólicas, los postres y el café son los que casi siempre tienen un precio varias veces por encima del
costo marginal, en tanto el margen de ganancia de la mayoría de los platos principales es mucho más
pequeño. Asimismo, casi todos los restaurantes ofrecen especiales del día, cuyo precio es muy cerca-
no al costo marginal. El resultado es que el comensal que quiere ahorrar en el costo de cenar fuera,
ya sea porque su ingreso es bajo o por cualquier otro motivo, sólo pide la comida básica y toma las
bebidas antes de la cena, y el postre y el café en su casa (o no pide esos extras). Dichos comensales
pagan un precio inferior al costo marginal de ser atendido. Otros comensales están dispuestos o
pueden comprar la opción más cara para tener el paquete completo en el restaurante. Es mucho
más probable que los comensales que buscan la última estrategia sean quienes prefieran la variedad
(porque, como ya se ha dicho, la demanda de la variedad está muy relacionada con el ingreso). Según
los acuerdos comerciales actuales, son quienes pagan la mayor parte del costo. A quienes no les pre-
ocupa tanto la variedad disfrutan aun cenar un día cordero vindaloo y pollo Szechuan al siguiente; y
si no piden bebidas y postre, pagan un poco más que en una cocina económica.
Como último ejemplo de cómo se distribuyen los costos de variedad en mercados de competen-
cia monopolística, tomemos en cuenta el caso de los precios de las líneas aéreas. Antes ya se discutió
que la importancia de la dimensión de variedad es el horario de los vuelos. No todos los viajeros
tienen las mismas demandas urgentes de vuelos con horarios frecuentes. Algunos están dispuestos
a pagar mucho más por un horario de salida un poco más temprano, mientras otros esperarían una

13_CHAPTER 13.indd 448 3/6/09 8:05:59 PM


Nota hIStóRICa: loS vENDEDoRES DE hot DoGS DE hotEllING 449

semana por no pagar 5 dólares más. El grado en el que las líneas aéreas usen aviones más pequeños
y costosos (por asiento) para ofrecer vuelos más frecuentes, depende de su interés en satisfacer las
necesidades del primer grupo.
¿Quién paga el costo adicional de los vuelos más cortos? Casi todas las líneas aéreas utilizan el
modelo de obstáculo para la discriminación de precios que se describió en el capítulo 12. La variante
particular que usan las líneas aéreas ofrece descuentos de aproximadamente 50% a los pasajeros que
cumplen con dos restricciones: 1) deben comprar los boletos con anticipación, por lo general al me-
nos siete días antes de la fecha del vuelo; y 2) su viaje debe incluir un sábado en la noche. El efecto
de estas restricciones es eliminar a los pasajeros que exigen una flexibilidad máxima para viajar. En
particular, los viajeros de negocios cuyos horarios suelen ser mucho más estrictos que los de los
vacacionistas, casi siempre pagan la tarifa normal en clase turista según el sistema comercial actual.
Por el contrario, muy pocos vacacionistas no cumplen los requisitos de al menos una forma de des-
cuento.
Si las demandas de horario de los viajeros de negocios dictan en gran medida el uso de aviones más
pequeños y caros, esta distribución de costos no parece sólo justa, sino también eficiente. Los boletos
con descuento permiten a las líneas aéreas atraer a pasajeros que de otro modo no viajarían y éstos per-
miten el uso de aviones más grandes y menos costosos. Los viajeros de negocios terminan con el servi-
cio frecuente que quieren y los viajeros de placer no están obligados a pagar los costos adicionales.20

NOTA HISTÓRICA: LOS VENDEDORES


DE HOT DOGS DE HOTELLING
En el documento original de Harold Hotelling sobre el modelo espacial de la competencia mono-
polística,21 se plantea el problema de dos vendedores de hot dogs que tienen la libertad de colocarse
donde quieran a lo largo de una playa. Suponga que ésta tiene una longitud de una milla y que en
cada extremo está limitada por un obstáculo natural, y también que los vendedores cobran el mismo
precio, que los clientes se dispersan a lo largo de la playa y que cada cliente compra un hot dog al
vendedor más cercano. Si la meta es vender la mayor cantidad de hot dogs posible, ¿dónde se deben
ubicar?
Suponga que, como se muestra en la figura 13.19, el vendedor 1 se coloca en el punto A y el
vendedor 2 escoge el punto B, estando ambos a –14 de milla del punto intermedio de la playa, que es el
punto C. En esta configuración, todos los clientes a la izquierda de C están más cercanos al vendedor
1 a quien le compran; en tanto los que se encuentran a la derecha de C le compran al vendedor 2. Por
lo tanto, cada vendedor obtiene la mitad del mercado. La distancia de ida más grande que cualquier
cliente tiene que desplazarse es –14 de milla y la distancia de ida promedio entre los clientes y su ven-
dedor más cercano es la mitad de eso, o sea –18 de milla.

FIGURA 13.19
0 .25 .50 .75 1.00
El problema de la
ubicación de los
A C B vendedores de hot dogs
para cada vendedor de hot
dogs lo mejor es colocarse
en el centro de la playa,
aun cuando esa ubicación
no minimiza la distancia
promedio que deben
desplazarse sus clientes.

20
Para una exposición más detallada del problema, véase R. Frank, “When Are Price Differentials Discriminatory?”, en Journal of
Policy Analysis and Management, 2, invierno de 1983, pp. 238-255.
21
Harold Hotelling, “Stability in Competition”, en The Economic Journal, 39, 1929: pp. 41-57.

13_CHAPTER 13.indd 449 3/6/09 8:06:00 PM


450 Capítulo 13 CoMpEtENCIa IMpERFECta: ENFoQuE DE la tEoRía DE JuEGoS

Los lectores con inclinación matemática pueden verificar que A y B de hecho son las ubicacio-
nes que minimizan la distancia de traslado promedio de todos los consumidores. No obstante, estos
lugares no son las mejores desde el punto de vista de cada vendedor. Para ver por qué, suponga que
el vendedor 1 se mueve 10 pasos hacia B. Los clientes a la izquierda de C lo siguen considerando el
vendedor más cercano. Pero los clientes que están a menos de 5 pasos a la derecha de C, los que es-
taban más cercanos al vendedor 2, de pronto cambian al vendedor 1. El desplazamiento a la derecha
aumentará las ventas del vendedor 1. El vendedor 1 maximizará sus ventas al colocarse tan cerca
como pueda del vendedor 2 en el lado de éste que se halle más próximo al centro de la playa.
Por supuesto que el vendedor 2 puede razonar igual, por lo que su estrategia será perfectamente
simétrica: intentará aproximarse lo más que pueda al vendedor 1 en el lado de éste que esté más cer-
cano al centro. Y cuando ambos vendedores se comportan de esta manera, el único resultado estable
es que cada uno se ubique en C, el centro de la playa. En C cada uno obtiene la mitad del mercado,
como ocurrió originalmente. Pero la distancia promedio que deben trasladarse los clientes ahora es
de –14 de milla, el doble que cuando los vendedores estaban en A y B.
Desde el punto de vista de la ventaja de los clientes, no es óptimo que ambos vendedores estén
en el centro de la playa y, sin embargo, ningún vendedor estaría mejor si se moviera unilateralmente.
Por lo tanto, el problema de ubicación de los vendedores de hot dogs no es uno de esos casos en los
que la mano invisible de Adam Smith guía los recursos para producir el mayor bien para todos.

PREFERENCIAS DE LOS CONSUMIDORES


Y PUBLICIDAD
En mercados de competencia perfecta, a un productor nunca le conviene anunciar su producto. Al
ser sólo uno de los muchos fabricantes de productos idénticos, la empresa que anuncia sólo atrae
una participación insignificante de cualquier aumento resultante en la demanda. En los mercados
de competencia monopolística y oligopolio los incentivos son diferentes. Como los productos se
diferencian, muchas veces los productores pueden desplazar de manera considerable sus curvas de
demanda hacia afuera mediante la publicidad.
¿De qué manera influye la publicidad en la eficiencia con la que los mercados asignan recursos?
En la descripción del mundo que ofrece la teoría de la elección racional, los productores son en esen-
cia representantes de consumidores, los cuales votan con sus dólares de compra y los productores
son rápidos para adoptar su oferta. Esta descripción se denomina secuencia tradicional, y el finado
economista de Harvard, John Kenneth Galbraith, era uno de sus críticos más destacados. En su lugar,
Galbraith propuso una secuencia revisada en la que el productor, no el consumidor, ocupa el asiento
del conductor. En el esquema de Galbraith la empresa decide cuáles son los productos más baratos
y convenientes de producir y después usa la publicidad y otros instrumentos promocionales a fin de
crear la demanda para ellos.
La secuencia revisada de Galbraith remodela la mano invisible de Adam Smith en una nueva luz
brillante. Recuerde que la historia de Smith era que los productores motivados sólo por el interés
propio ofrecerían los productos que mejor satisficieran los deseos de los consumidores. Quienes no
lo hacían, no atraían clientes y salían del negocio. Pero si Galbraith tiene razón, entonces la historia
está al revés: es como decir que la mano demasiado visible de Madison Avenue guía a los clientes para
que sirvan a los intereses de las grandes corporaciones.
La secuencia revisada de Galbraith tiene un atractivo intuitivo. Por ejemplo, es lógico que mu-
chas personas sean escépticas cuando un economista dice que el objetivo de la publicidad es mante-
ner mejor informados a los consumidores. Después de todo, el hecho es que muchas veces esta no es
la intención. Por ejemplo, el afamado bigote de leche seguramente no forma parte de un proceso en
el que nos hacemos conscientes de los méritos alimenticios del consumo de leche.
Pero a pesar del evidente atractivo de los mensajes publicitarios, el punto de vista de Galbraith
sobre el proceso pasa por alto algo fundamental: es mucho más fácil vender un producto bueno que
uno malo. Lo único razonable que puede esperar el anuncio es inducir al consumidor para que pruebe
el producto. Si lo prueba y le gusta, es probable que lo compre en repetidas ocasiones. También es
posible que lo recomiende a sus amistades. Pero si no le gusta, normalmente el proceso termina ahí.

13_CHAPTER 13.indd 450 3/6/09 8:06:00 PM


pREFERENCIaS DE loS CoNSuMIDoRES Y puBlICIDaD 451

Aun si una empresa lograra que todos lo probaran una vez, de todas maneras no podría mantener una
empresa redituable y continua.
Imagínese dos productos alternativos, uno que cumple con las necesidades humanas reales pero
cuya producción es costosa y otro que no satisface necesidades reales, pero que es un poco más bara-
to. ¿Cuál de los dos publicitaría mejor un productor ávido de utilidades? Dada la importancia del ne-
gocio repetido y la afirmación de boca en boca, por lo general el primero es más atractivo y muchas
veces por un margen contundente. El hecho de que su producción sea más costosa no detiene a los
consumidores, a menos que los beneficios no justifiquen los costos extra.
Los productos nuevos atraviesan por extensas pruebas de mercado antes de llegar alguna vez
a los anaqueles. Se gastan millones de dólares en el análisis de las reacciones de los individuos que
los prueban. Al final, la mayoría de los productos que entran en este proceso no logran ver la luz
del día. Sólo cuando una empresa tiene evidencia concreta de que es probable que un producto sea
bien recibido, se atreve a comprometer los millones de dólares que se requieren para una campaña
publicitaria intensa a nivel nacional.
Las empresas que no logran adoptar esta postura, con frecuencia lo pagan caro. Por ejemplo,
la compañía de programas de cómputo Lotus, gastó más 10 millones de dólares en anunciar Jazz, su
programa de hoja de cálculo para Apple Macintosh, aun cuando era evidente que carecía de carac-
terísticas específicas que muchos usuarios consideran vitales. Los anuncios de Lotus eran increíble-
mente rebuscados y sin duda lograron la venta de un buen número de programas. No obstante, el
principal rival de Jazz, Excel de Microsoft, un producto mucho mejor, captó con rapidez el mercado
con apenas una pequeña fracción de los gastos en publicidad de Lotus.
La publicidad y otros esfuerzos para convencer a los consumidores se consideran más bien parte
de un proceso como el de purga de una bomba. Dada la enorme cantidad de costos, por lo común
conviene promover nada más los productos que tal vez quieran comprar los consumidores una y
otra vez o de los cuales hablen bien a los amigos. De hecho, es obvio que la mayoría de las empresas
siguen esta estrategia. Los productores de cenas congeladas anuncian sus platos fuertes exóticos y
extravagantes y no los pasteles de pollo. Los editores anuncian libros que posiblemente sean un éxito
de venta y no sus títulos con atractivo más limitado. Los estudios cinematográficos hacen alarde de
las películas que esperan sean un éxito de taquilla, no de aquéllas de bajo presupuesto.
Como los productores tienen el incentivo de anunciar sólo los productos que es probable que los
consumidores consideren satisfactorios, la llamada secuencia tradicional es más plausible de lo que
reconocen Galbraith y otros críticos. Lo cierto es que, cuando las diferencias en calidad entre los pro-
ductos en competencia son menores, la publicidad puede ser una influencia importante en la marca
Ilustración de Lorenz. © 1976 The New Yorker Magazine, Inc.

“Parafraseando al gran Vince Lombardi, empacar no es todo, es lo único.”

13_CHAPTER 13.indd 451 3/6/09 8:06:01 PM


452 Capítulo 13 CoMpEtENCIa IMpERFECta: ENFoQuE DE la tEoRía DE JuEGoS

que elija el consumidor. Pero como una primera aproximación, sigue teniendo sentido suponer que
los consumidores tienen conocimientos razonablemente definidos de lo que quieren y los producto-
res se esfuerzan mucho en procurar atender dichos conocimientos.
Sin embargo, no con esto se quiere decir que los incentivos del mercado conducen a la cantidad
de publicidad que es mejor desde el punto de vista de la sociedad. Como se vio en el capítulo, la
competencia estratégica entre rivales provoca que las empresas gasten demasiado en publicidad.

RESUMEN
• La característica distintiva de los mercados oligopólicos es la poco inferior y una cantidad ligeramente mayor que se darían si
interdependencia entre las empresas. La interdependencia entre las empresas se coludieran para alcanzar como resultado el mo-
las empresas oligopólicas se suele analizar con éxito con la teoría nopolio. En cambio, el modelo de Bertrand lleva básicamente al
matemática de juegos. Los tres elementos básicos de cualquier mismo resultado que se vio en la competencia perfecta.
juego son los participantes, el conjunto de estrategias posibles • En el modelo de Stackelberg se postula una forma más elabora-
y la matriz de ganancias. Hay un equilibrio de Nash cuando la da de interdependencia entre empresas: una cumple un papel de
estrategia de cada participante es la mejor, dada la elección de liderazgo y sus rivales la siguen. La estructura de este modelo
estrategia de los otros participantes. Una estrategia es dominan- es parecida al modelo de Cournot, salvo que si las empresas de
te si es óptima, cualquiera que sea la estrategia que elija el otro Cournot dan por sentadas las cantidades de unas y otras, el líder
participante. de las de Stackelberg manipula estratégicamente las decisiones
• Los incentivos para las empresas que tratan de coludirse son sobre cantidades de sus rivales.
semejantes a los que tienen quienes se enfrentan al dilema del • La competencia monopolística se define mediante dos con-
prisionero. La dificultad de tener unidos los cárteles estriba en diciones simples: 1) la existencia de numerosas empresas que
que la estrategia dominante de cada miembro consiste en trai- manufacturan un producto que es sustituto cercano, aunque
cionar el acuerdo. Las interacciones repetidas entre muy pocas imperfecto, de los productos de otras, y 2) la libre entrada y
empresas pueden sostener un comportamiento de colusión en salida de empresas. En el modelo espacial de la competencia
circunstancias en las que son eficaces estrategias del tipo ley del monopolística los clientes tienen una preferencia mayor por
talión. ubicaciones particulares o características de productos. El resul-
• A veces las empresas dominantes actúan en forma estratégica tado es que las empresas compiten con más intensidad por los
para disuadir a posibles rivales de entrar en su mercado. A me- consumidores de los productos que más se parecen a los suyos.
nudo consiste en incurrir en costos mayores que los necesarios. • Una característica central del modelo espacial de competencia
• No siempre hay una tendencia marcada para que surja un mo- monopolística es la disyuntiva entre el deseo, por un lado, de
nopolio natural de la interacción de pocas empresas, de las cua- aminorar los costos y, por el otro, de tener mayor variedad o ubi-
les cada una produce con rendimientos a escala creciente. Aquí caciones cómodas. El grado óptimo de la diversidad de producto
resulta importante la función de los costos de inversión. En depende de varios factores. Se espera más diversidad con mayor
ocasiones es necesario que los compradores emprendan ciertas densidad de población y mayores costos de transporte (en el caso
iniciativas para tener disponible la menos cara de las opciones. general, los “costos de transporte” miden la disposición a pagar
• En el modelo de Cournot cada empresa da por seguras las can- por características que se desean en los productos). La diversi-
tidades producidas por su competencia. Por el contrario, en el dad óptima de los productos guarda una relación inversa con los
modelo de Bertrand las empresas dan por seguros los precios de costos iniciales de agregar nuevas características a los productos
las rivales. La orientación con que funcionan las empresas es o nuevas ubicaciones. El mercado imparte una justicia aproxi-
muy parecida en los dos casos, pero los resultados sorprenden mada en la que los costos de la variedad adicional tienden a ser
por lo diferentes. El modelo de Cournot arroja un precio un aplicados a quienes le confieren mayor importancia.

pREgUNtaS dE REpaSo
1. ¿Cuál es la diferencia fundamental entre los modelos de oligo- 4. ¿El equilibrio del modelo de Cournot satisface la definición de
polio de Cournot, Bertrand y Stackelberg? un equilibrio de Nash?
2. ¿Cómo se asemeja el problema de colusión de oligopolios a la 5. Describa la disyuntiva entre costo y variedad.
estructura del dilema del prisionero? 6. ¿De qué manera se relaciona el grado óptimo de variedad de
3. ¿Cuál es la dificultad de la estrategia de la ley del talión como producto con la densidad demográfica? ¿Con el costo de trans-
una probable solución al problema de colusión de oligopolios? portación? ¿Con el costo fijo de ofrecer productos nuevos?

13_CHAPTER 13.indd 452 3/6/09 8:06:02 PM


pRoBlEMaS 453

pRoblEMaS
1. La curva de demanda del mercado del agua mineral está definida por P = 15 – Q. Si hay dos empre-
sas que producen agua mineral, cada una con un costo marginal constante de 3 por unidad, llene las
entradas de los cuatro modelos de duopolio que se indican en la tabla. (En el modelo de Stackelberg
suponga que la empresa 1 es la líder.)

Modelo Q1 Q2 Q1 + Q2 P ∏1 ∏2 ∏1 + ∏2
Monopolio compartido
Cournot
Bertrand
Stackelberg

2. La curva de demanda del mercado para un par de duopolistas de Cournot se obtiene mediante
P = 36 – 3Q, donde Q = Q1 + Q2. La constante por costo marginal unitario es de 18 para cada duopo-
lista. Encuentre el precio de equilibrio, la cantidad y las utilidades de Cournot.
3. Resuelva el problema anterior para duopolistas de Bertrand.
4. La curva de demanda del mercado para un par de duopolistas se obtiene con P = 36 – 3Q, donde
Q = Q1 + Q2. El costo marginal constante por unidad es 18 para cada duopolista. Encuentre el precio
de equilibrio, la cantidad y la utilidad de cada empresa, suponiendo que las empresas actúan como
líder y seguidor de Stackelberg, con la empresa 1 como líder.
5. Debido a su experiencia única con los explosivos, los hermanos Zambino han disfrutado mucho
tiempo de un monopolio del mercado estadounidense de fuegos artificiales para presentaciones
ante públicos de más de un cuarto de millón de personas. La demanda anual de estas presentacio-
nes de fuegos artificiales es P = 140 – Q. El costo marginal de preparar una presentación de fuegos
artificiales es de 20. Una disputa familiar dividió a la empresa en dos. Ahora, Alfredo Zambino dirige
una empresa y Luigi Zambino otra. Todavía tienen los mismos costos marginales pero ahora son
duopolistas de Cournot. ¿Cuántas utilidades ha perdido la familia?
6. Mientras calificaba un examen final, un profesor descubre que dos estudiantes tienen respuestas casi
idénticas. Habla con cada uno por separado y les dice que está seguro de que se copiaron, pero no
sabe quién a quién. Ofrece a cada estudiante un trato: si ambos firman una declaración admitiendo
que copiaron, cada uno tiene calificación reprobatoria durante el curso. Si sólo uno firma la decla-
ración, se le permitirá abandonar el curso y el que no firme será expulsado de la universidad. Por
último, si ninguno firma la declaración, ambos tendrán una calificación de 6 durante el curso porque
el profesor no tiene evidencia suficiente para comprobar que hicieron trampa. Si se supone que no
se permite la comunicación entre los estudiantes, prepare la matriz de ganancias resultante. ¿Cada
estudiante tiene una estrategia dominante?
7. Suponga que A y B saben que van a interactuar en el dilema del prisionero cuatro veces exactas. Ex-
plique por qué la estrategia tipo ley del talión no es una forma eficaz de garantizar la cooperación.
8. La empresa 1 y la empresa 2 son productoras de automóviles. Cada una tiene la opción de producir
un auto grande o uno pequeño. Las ganancias para cada una de las cuatro combinaciones posibles
de opciones se presentan en la siguiente matriz de ganancias. Cada empresa debe hacer su elección
sin saber qué prefirió la otra.

Empresa
Auto grande Auto pequeño
Auto grande Π1 = 400 Π1 = 800
Π2 = 400 Π2 = 1 000
Empresa 2
Auto pequeño Π1 = 1 000 Π1 = 500
Π2 = 800 Π2 = 500

13_CHAPTER 13.indd 453 3/6/09 8:06:03 PM


454 Capítulo 13 CoMpEtENCIa IMpERFECta: ENFoQuE DE la tEoRía DE JuEGoS

a) ¿Alguna de las empresas tiene una estrategia dominante?


b) Este juego tiene dos equilibrios de Nash. Identifíquelos.
9. Suponga que tiene la misma matriz de ganancias que en el problema 8, salvo que ahora la empresa
1 tiene la primera opción de movimiento y sabe que la empresa 2 verá los resultados de la elección
antes de decidir el tipo de auto que va a fabricar.
a) Trace el árbol de juego para este juego de secuencia.
b) ¿Cuál es el equilibrio de Nash de este juego?
10. El Estado anunció su plan de dar licencia a dos empresas para atender un mercado cuya curva de
demanda está definida por P = 100 – Q. La tecnología es tal que cada una puede producir cualquier
nivel determinado de producción a costo cero, pero una vez que se ha elegido la producción de cada
empresa, no se puede alterar.
a) ¿Cuál es el máximo que estaría dispuesto a pagar por una de estas licencias si supiera que podría
elegir primero su nivel de producción (suponga que la empresa rival puede observar su elec-
ción)?
b) ¿Cuánto estaría dispuesto a pagar su rival por el derecho de elegir en segundo lugar?
*11. La empresa 1 y la empresa 2 compiten por una franquicia de televisión por cable. El valor actual de
los ingresos netos generados por la franquicia es igual a R. La probabilidad de cada empresa de ganar
la franquicia se obtiene mediante la proporción del total que han gastado las dos empresas durante
la negociación con el comité gubernamental local que otorga la franquicia. Es decir, si I1 e I2 repre-
sentan los gastos de negociación de las empresas 1 y 2, respectivamente, entonces la probabilidad de
que la empresa 1 gane se obtiene mediante I1 /(I1 + I2), mientras que la probabilidad de que gane
la empresa 2 está dada por I2 /(I1 + I2). Si cada empresa supone que el gasto de la otra empresa es
independiente del suyo, ¿cuál es el nivel de equilibrio de gasto para cada empresa?
12. Establezca si es verdadero o falso y explique brevemente por qué: si el propietario de un negocio
acepta con agrado pedidos adicionales al precio actual, no puede tratarse de un productor perfecta-
mente competitivo maximizador de utilidades.
13. Una comisión encargada de carreteras de cuota planea ubicar garajes para grúas a lo largo de una
carretera circular de 100 millas. Cada garaje tiene un costo fijo de 5 000 dólares por día. Los trabajos
de remolque son igualmente probables a lo largo de cualquier punto de la carretera y el costo por
milla remolcada es de 50 dólares. Si hubiera 5 000 trabajos de remolque al día, ¿qué cantidad de
garajes minimizaría la suma de los costos fijos y de remolque?
14. Los 1 000 residentes de Great Donut Island son pescadores. Todas las mañanas van al puerto más
cercano para botar sus barcos de pesca y regresan en la tarde con lo que hayan capturado. Los resi-
dentes están distribuidos de manera uniforme a lo largo de un perímetro de 10 millas de la isla. Cada
puerto tiene un costo fijo de 1 000 dólares al día. Si el número óptimo de puertos es 2, ¿cuál debe ser
el costo de viaje por milla?

RESpUEStaS a loS EjERcicioS dEl capítUlo


13.1. Independientemente de la estrategia de la empresa 1, a la empresa 2 le va mejor con un presupues-
to de investigación grande. La elección de un presupuesto de investigación grande es, por lo tanto,
una estrategia dominante de la empresa 2. La empresa 1 no tiene una estrategia dominante. Si la
empresa 2 elige un presupuesto escaso para investigación, entonces a la empresa 1 le va mejor al
elegir un presupuesto cuantioso para investigación. Como la empresa 1 puede pronosticar que la
empresa 2 elegirá un presupuesto cuantioso, su mejor estrategia es escoger lo mismo. La combina-
ción de “alto presupuesto para investigación-alto presupuesto para investigación” es un equilibrio
de Nash.

*Este problema requiere que se apliquen las técnicas de maximización del cálculo.

13_CHAPTER 13.indd 454 3/6/09 8:06:03 PM


RESpuEStaS a loS EJERCICIoS DEl Capítulo 455

13.2 $/Q Q1

44 24

44 – Q2 IM1 = 44 – Q2 – 2Q1 R2

20 12

8
R1
Q2
Q Q2
0 Q1 8 12 24
Q*1 = 12 – (Q2/2)

13.3 El precio se establece en el costo marginal y, por lo tanto, P = 2. La demanda de mercado corres-
pondiente, Q = 8, es compartida de manera equitativa por las dos empresas: Q1 = Q2 = 4.
13.4 La curva de ingreso marginal de la empresa 2 está definida por 10 – Q1 – 2Q2. Al establecer
IM = CM = 2, se tiene la función de reacción de la empresa 2, R2(Q1) = Q*2 = 4 – (Q1/2). Al sus-
tituir en la función de demanda de la empresa 1 se obtiene P1 = 10 – Q1 – 4 + (Q1/2) = 6 – (Q1/2)
y la curva de revisión marginal correspondiente IM1 = 6 – Q1. IM1 = CM = 2 se resuelve para
Q*1 = 4. Esto significa que Q2 será 2 unidades, para una producción total del mercado de 6 unidades.
El precio de mercado será 10 – 6 = 4.
1 1
20 = 5% de la demanda total, pero ahora sólo obtendrá — 25 = 4%.
13.5 Al inicio cada empresa obtuvo —
Lo anterior significa que en cada precio, la cantidad demandada será 20% más baja que antes (véase
gráfica de abajo). La nueva curva dd es P = 10 – 0.00125Q.

10

Q
8 000 10 000

1
13.6 Con seis restaurantes, la distancia del viaje redondo es de —
12 de milla, que produce un costo de
transportación promedio de 2 dólares. En promedio, cada restaurante atraerá 100/6 personas al
día, que produce un CTP de [50 + 5(100/6)]/(100/6) = 8 dls./día. El costo promedio total con seis
restaurantes es entonces 10 dls./día.
13.7 N* será ahora √[(24)(400)/100] ≈ 9.8, así que ahora habrá 10 restaurantes.

13_CHAPTER 13.indd 455 3/6/09 8:06:05 PM


13_CHAPTER 13.indd 456 3/6/09 8:06:05 PM
Pa rt e

L A MERCADOS
4
O F E RTA Y DEL AFACTORES
D E M A N DA

en los dos capítulos siguientes se examina el funcionamiento de los mercados


de insumos para la producción. en el capítulo 14 se verá que aunque el merca-
do laboral funciona en muchos aspectos como el mercado de bienes y servi-
cios ordinarios, es muy distinto en otros sentidos importantes. en el capítulo
15 se estudian los mercados de capital real y financiero. Una característica que
distingue al capital de otros insumos, como se verá, es que los otros insumos se
contratan por periodos, mientras que el equipamiento de capital es propiedad
completa de la empresa.

457

14_CHAPTER 14.indd 457 3/7/09 12:56:16 PM


14_CHAPTER 14.indd 458 3/7/09 12:56:17 PM
CaPítUlo

14
MANO DE OBRA

n la temporada de 1931 el jugador de los yanquis Babe Ruth era el beisbolista mejor pa-
E gado, con un salario anual de 85 000 dólares. Cuando le preguntaron qué sentía por ganar
más que el presidente Herbert Hoover, respondió, con su acostumbrada bravuconería, que
se lo merecía, y explicó: “Tuve un año mucho mejor que el presidente Hoover.”
Pero no siempre las diferencias de productividad bastan para explicar las discrepancias de pago
de los trabajadores. Por ejemplo, hay un amplio movimiento de empleados entre los sectores públi-
co y privado, y por el estudio de lo que ocurre con esos movimientos se sabe que las personas muy
productivas casi siempre ganan mucho menos en el sector público. Así, el ex presidente del Consejo
de la Reserva Federal, Alan Greenspan, que ocupaba el que quizás era el puesto más importante del
gobierno de Estados Unidos, ganaba menos de un décimo de lo que cobraba en Wall Street.
¿Por qué la gente acepta puestos en el gobierno con tal sacrificio de ingresos? A decir de to-
dos, el atractivo de los altos puestos en la burocracia es el poder y la atención pública que reciben.
Greenspan, que ahora dirige una empresa de asesoría, vuelve a ganar muchas veces lo que ganó en el
gobierno. Pero sus decisiones diarias ya no afectan la vida de millones. El público ansioso de oír sus
ideas y opiniones se evaporó de la noche a la mañana. Greenspan asumió la presidencia de la Reserva
y la grave reducción de sueldo que la acompaña, porque el puesto acarreaba beneficios que ningún
otro empresario podía ofrecer. Tomado en conjunto, la oferta le pareció atractiva.
Los trabajos que confieren mucha visibilidad pública no siempre entrañan reducciones salaria-
les. El tercera base de los Yanquis de Nueva York, Alex Rodríguez, aparece en las noticias todos los
días durante por lo menos seis meses del año. Sin duda, hay un público ansioso de oír sus opiniones;
pero con un salario de más de 25 millones de dólares al año, no parece que haya hecho ningún
sacrificio económico. Hay muchas personas —entre las que me cuento— que estarían dispuestas a
hacer el trabajo de Rodríguez por menos dinero. De hecho, seguramente el propio Rodríguez estaría

459

14_CHAPTER 14.indd 459 3/7/09 12:56:18 PM


460 CaPítUlo 14 mano de obra

dispuesto a trabajar por un salario de subsistencia antes que desempeñar toda la vida un oscuro
trabajo en algún puesto anónimo del sector privado.
Su elevado salario es el resultado de dos importantes factores: 1) puede hacer cosas valiosas por
los demás, y 2) hay más de un empresario que puede ofrecerle un lugar en las candilejas. Observe
que sólo el primer factor se aplica al caso de Alan Greenspan. Si usted quiere ser presidente del Con-
sejo de Gobernadores del Sistema de la Reserva Federal de Estados Unidos, el gobierno es el único
empresario con el que puede trabajar. En cambio, si quiere jugar en las ligas mayores de beisbol,
30 equipos pueden solicitar sus servicios. En el caso de Rodríguez, si los Yanquis no le pagaran ese
salario tan alto, otro equipo estaría encantado de hacerlo. Miles de aficionados van al estadio con
el objetivo de ver a quien es quizás el mejor jugador de todos. La dirección de los Yanquis sabe que
podría contratarme por mucho menos de lo que le paga a Rodríguez, pero como son listos, saben
también que yo sería un mal negocio, incluso si no cobrara.

VISIÓN PRELIMINAR DEL CAPÍTULO


La meta en el capítulo es examinar las fuerzas económicas que rigen los salarios y otras condiciones
del empleo. Modelos relativamente simples del mercado laboral arrojan luces sobre varias preguntas
interesantes: ¿cuánto ganará un trabajador con determinadas habilidades? ¿Por qué varían las condi-
ciones laborales de un lugar a otro? ¿Qué hacen los sindicatos?, etcétera.
Para empezar, se deducirá la curva de demanda de mano de obra, tanto a corto como a largo
plazo. Luego se abordará la oferta del mercado laboral desde el punto de vista del trabajador indivi-
dual que trata de decidir cuánto trabajar por una tasa salarial determinada.
El siguiente paso consistirá examinar el problema de compensar las diferencias salariales, que
son variaciones de los sueldos que reflejan discrepancias en los puestos de trabajo. El resultado gene-
ral es que el atractivo del paquete total de pago (sueldos y factores ambientales en conjunto) tiende a
equipararse entre los puestos ocupados por trabajadores con capacidades de cierto nivel. Para mayor
ilustración del concepto de compensar diferencias, se examinará la cuestión de los niveles de seguri-
dad en el trabajo.
También se aplicará la teoría económica de los mercados laborales a temas como las leyes de
discriminación y salario mínimo. Se concluye con un examen de por qué las diferencias salariales
unas veces parecen exagerar los contrastes de productividad, mientras que en otras parecen menos-
preciarlos.

DEMANDA DE MANO DE OBRA A CORTO


PLAZO DE LA EMPRESA PERFECTAMENTE
COMPETITIVA
Considere una empresa que genera su producción a partir de dos factores: capital (K) y mano de
obra (L). Suponga que a corto plazo las existencias de capital son fijas. Si la empresa vende toda su
producción en un mercado perfectamente competitivo al precio actual y si puede contratar todos
los trabajadores que quiera a una tasa salarial de 12 dls./hora, ¿cuántas unidades de mano de obra
contratará?
Si el gerente de la empresa piensa como economista, razonará como sigue: “El beneficio de con-
tratar una unidad adicional de mano de obra será el precio al que pueda vender el producto adicional
que obtenga. El costo será la tasa salarial. Así, debo contratar una unidad de mano de obra adicional
mientras el valor de la producción supere el costo. En el caso contrario, debo reducir la mano de
obra que contrate.”
Este razonamiento se traduce fácilmente en una simple regla de contratación gráfica. En la fi-
gura 14.1a se muestra la curva de producto marginal del insumo mano de obra cuando el capital
es fijo (vea el capítulo 9). Recuerde que la curva de producto marginal indica qué producción extra

14_CHAPTER 14.indd 460 3/7/09 12:56:19 PM


demanda de mano de obra a Corto Plazo 461

FIGURA 14.1
Producto marginal de la mano
de obra (unidades de producto/ Demanda de mano de
Valor del producto marginal
unidades de mano de obra) (dls./unidad de trabajo) obra a corto plazo de la
empresa competitiva
Cuando la tasa salarial es
8 16 Cantidad óptima de mano de 12 dólares por unidad
de obra cuando w = 12 (sección b), la empresa
perfectamente competitiva
6 w = 12
contrata 80 unidades de
mano de obra, la cantidad
4 8 para la cual VPmL y la tasa
salarial son iguales.
PML
VPML = P × PML

L (persona- L (persona-
40 80 120 hora/día) 40 80 120 hora/día)
a) b)

tendrá la empresa cuando contrate una unidad adicional de mano de obra. Por ejemplo, si se em-
plean 40 unidades de mano de obra, contratar una unidad adicional rinde ocho unidades de produc-
ción. La pendiente negativa de la curva de producto marginal expresa la ley de los rendimientos
decrecientes.
En la figura 14.1b se multiplica la curva de producto marginal por el precio de la producción, en
este caso P = 2 dólares. El resultado del precio del producto multiplicado por el producto marginal
P × PML, se llama valor del producto marginal de la mano de obra, denotado VPML (en el que L valor del producto marginal
alude a labor, o mano de obra), que es el ingreso extra que obtendrá la empresa por la producción (VPM) valor, al precio actual
adicional generada por la unidad extra de mano de obra. La regla de contratación de la empresa consiste del mercado, de la producción
en escoger la cantidad de mano de obra para la cual la tasa salarial es igual al VPML. En la figura 14.1b, la extra generada por una unidad
regla indica que la empresa debe contratar 80 unidades de mano de obra cuando la tasa salarial es de de insumo adicional.
12 dólares.
Para dar un ejemplo de la lógica de la regla, suponga que la empresa contrató sólo 40 unidades
de mano de obra. En ese nivel de empleo, el valor de la producción extra de un trabajador adicional
(16 dólares) es mayor que el costo de contratar al trabajador (12 dólares), así que la empresa puede
aumentar sus utilidades si contrata más trabajadores. Como alternativa, suponga que la empresa
contrató 120 unidades de mano de obra. El VPML en L = 120 es de apenas 8 dólares, que es menor
que la tasa salarial de 12 dólares, así que la empresa puede aumentar sus utilidades despidiendo traba-
jadores. Sólo en L = 80 la empresa puede dar otros pasos para incrementar las utilidades.1 En la figura
14.1a se traza PM L = 10 − ( 201 ) L . Cuando P = 2, el valor del producto marginal de la mano de obra,
representado en la figura 14.1b, es
VMPL = P ( MPL ) = 2(10 − 201 L ) = 20 − 101 L.
Si el sueldo es w = 12, la cantidad de mano de obra demandada por la empresa será
w = VMPL ⇒ 12 = 20 − 101 L ⇒ 8 = 101 L ⇒ L * = 80.

EJERCICIO 14.1
Con una tasa salarial de 12 dls./unidad de mano de obra, ¿cuántas unidades de mano de obra
contrataría la empresa representada en la figura 14.1 si su producción se vendiera no a 2 dls./
unidad, sino a 3 dls./unidad?

1
Hay una limitación importante a la aplicación de la regla w = VMPL. Suponga que la tasa salarial fuera mayor que el valor del pro-
ducto promedio de la mano de obra —que es el resultado de multiplicar el precio por el producto promedio de la mano de obra—,
denotado VPPL. Si la empresa paga un salario mayor que VPPL, gastará más que el valor total de lo que producen los trabajadores,
lo cual significa que sufrirá una pérdida por cada trabajador que contrate. Así, para valores de w superiores a VPPL, la empresa
perfectamente competitiva no demandará mano de obra.

14_CHAPTER 14.indd 461 3/7/09 12:56:23 PM


462 CaPítUlo 14 mano de obra

LA DEMANDA DE MANO DE OBRA A LARGO PLAZO


DE LA EMPRESA PERFECTAMENTE COMPETITIVA
En el corto plazo, la única manera que tiene una empresa para responder a una reducción de la tasa
salarial es contratar más mano de obra. Pero a la larga, todos los insumos son variables. Como se vio
en el capítulo 9, una reducción del precio de la mano de obra hará que la empresa sustituya capital
con trabajo, con lo cual reduce todavía más su costo marginal. Esta reducción adicional del costo ge-
nera una expansión aún mayor de la producción. Se deduce que la respuesta de contratación a largo
plazo de la empresa ante un cambio en la tasa salarial será mayor que su respuesta a corto plazo. La
relación entre las dos curvas de demanda de mano de obra se representa en la figura 14.2.

FIGURA 14.2 Salario (dls./día)


Curvas de demanda de
mano de obra a corto y
largo plazos
la demanda de mano de
obra es más elástica a largo
plazo porque la empresa
tiene la posibilidad de
Demanda de mano de
sustituirla con capital. en
obra a largo plazo
el corto plazo su única
posible respuesta es Demanda de mano de
aumentar la producción. obra a corto plazo
Mano de obra (persona-hora/día)

La demanda de mano de obra de la empresa también es más elástica cuando la demanda de su


producto es más elástica. Si una reducción del precio estimula un incremento grande en la cantidad
demandada de un producto, también estimulará un aumento notable en la demanda de mano de
obra necesaria para elaborarlo. Por último, la demanda de mano de obra por parte de la empresa será
más elástica cuanto más sea capaz de reemplazar los servicios de los trabajadores con otros insumos.
En igualdad de circunstancias, la empresa con isocuantas en forma de L tendrá la curva de demanda
de mano de obra menos elástica.

CURVA DE DEMANDA DEL MERCADO


PARA LA MANO DE OBRA
Recuerde del capítulo 5 que la técnica para obtener una curva de demanda del mercado para un pro-
ducto consiste en sumar horizontalmente las curvas de demanda de los consumidores individuales.
La técnica para generar la curva de demanda del mercado para la mano de obra es parecida, salvo por
una importante diferencia. En la figura 14.3, la curva marcada ΣVPML, P = P1 es la suma horizontal
de las curvas VPML individuales cuando el precio de la producción es igual a P1. En ese valor del
precio de la producción, las empresas tomadas en conjunto demandan L1 unidades de mano de obra
por periodo cuando la tasa salarial es igual a w1. Ahora, suponga que la tasa salarial baja a w2. Todas
las empresas contratarán más trabajadores y, entre tanto, bajarán por su propia curva de demanda de
mano de obra. Como cada empresa responde de la misma manera, ofrece más de su producto para
la venta en el mercado. Esta acción de parte de cualquier empresa en un mercado competitivo no
cambiaría el precio de la producción; pero el efecto de que todas las empresas actúen en conjunto es
impulsar un movimiento descendente por la curva de demanda del producto de la industria.
Este aumento de la producción implica por fuerza una reducción del precio, que a su vez hace
que la curva VPML de cada empresa se desplace hacia abajo. Si el precio del producto baja de P1 a P2,
la demanda agregada de mano de obra queda dada por el punto que corresponde a w2 sobre la curva
marcada ΣVPML, P = P2. En virtud de este razonamiento, se ve que la curva de demanda del mercado
laboral (la curva marcada DD) tendrá una pendiente más pronunciada que la suma horizontal de las
curvas VPML.

14_CHAPTER 14.indd 462 3/7/09 12:56:25 PM


demanda de mano de obra de Un ComPetidor imPerfeCto 463

FIGURA 14.3
$/L Curva de demanda del
mercado para la mano
D de obra
Cuando la tasa salarial baja
de w1 a w2, cada empresa
w1
contrata más trabajadores
y genera más producción. el
aumento de la producción
w2 hace que su precio baje,
ΣVPML, P = P1
lo cual reduce el valor del
ΣVPML, P = P2 producto marginal de la
mano de obra. Por lo tanto,
la curva de demanda del
D mercado para la mano de
L obra tiene una pendiente
L1 L2 más inclinada que la suma
horizontal de las curvas de
demanda individuales.

En la exposición anterior se supuso de manera implícita que sólo hay una clase de mano de obra
y que toda se ocupa en una sola industria competitiva. Desde luego, en la realidad las cosas son más
complicadas. Hay incontables categorías de trabajadores: carpinteros, electricistas, médicos, aboga-
dos, maestros de preparatoria, etc., y los de cualquier clase encuentran colocación en muchas indus-
trias. Así, los electricistas trabajan en la construcción de casas, en la industria de los automóviles, en
edificios de oficinas, acerías, empresas de cómputo y pesqueras, por citar una muestra muy pequeña.
Por lo tanto, la curva de demanda de electricistas está compuesta por la demanda individual de las
empresas no de una sola industria, sino de muchas.
Suponga que los pagos que hacen empresas de diversas industrias a los electricistas constituyen
una pequeña fracción (por ejemplo, 0.1%) de sus costos totales respectivos. Un cambio pequeño
(como de 10%) en el salario de los electricistas generaría un cambio casi imperceptible (de 0.01%) en
los costos totales de cada industria y, por consiguiente, no tendría un efecto apreciable en el precio
correspondiente del producto. En estas circunstancias la demanda de electricistas se aproximaría me-
diante la suma horizontal de las curvas de demanda de las empresas individuales y podría ignorarse
la complicación expuesta en relación con la figura 14.3.

DEMANDA DE MANO DE OBRA DE UN


COMPETIDOR IMPERFECTO
En el presente estudio de la demanda de mano de obra se ha supuesto que la empresa tiene una de-
manda perfectamente elástica de su producto. Cualquier producción adicional manufacturada por
trabajadores adicionales se vendería al mismo precio que la producción anterior. Desde luego, en el
caso de un competidor imperfecto, esto no sería así. Tal empresa tiene una curva de demanda con pen-
diente negativa y si contrata más trabajadores debe reducir sus precios para vender más producción.
Se vio que con un competidor perfecto, el valor de la producción adicional obtenida por con-
tratar más trabajadores es el resultado de multiplicar el precio por el producto marginal de la mano
de obra. En cambio, en el caso del competidor imperfecto es el resultado de multiplicar el ingreso
marginal por el producto marginal. Este producto se llama producto del ingreso marginal de la producto del ingreso
mano de obra y se denota PIML (por sus siglas en inglés). En términos de las definiciones del ingreso marginal PIM cantidad por
marginal y el producto marginal, PIML está dado por la que aumenta el ingreso total
por el empleo de una unidad
∆Q ∆IT adicional de insumo.
PIM L = , (14.1)
∆L ∆Q

14_CHAPTER 14.indd 463 3/7/09 12:56:27 PM


464 CaPítUlo 14 mano de obra

que se reduce a
∆IT
PIM L = . (14.2)
∆L
VPML y PIML se parecen en que las dos representan la adición al ingreso total que resulta de su-
mar una unidad de mano de obra. La diferencia entre ellos es que PIML toma en cuenta que la venta
de un producto adicional requiere que el competidor imperfecto rebaje el precio; por su parte, VPML
valora el producto adicional al precio del producto anterior, que no se ve afectado por las variaciones
en la producción del competidor perfecto. PIML valora la producción adicional en su ingreso margi-
nal, el cual es menor que su precio.
¿Cuántos trabajadores contratará una empresa si su producción tiene una curva de demanda
con pendiente negativa? La respuesta es que contratará la cantidad en la que la tasa salarial y PIML
sean iguales. El argumento en que se basa esta aseveración es semejante en lo esencial al que se
ofreció para la condición w = VPML del competidor perfecto.
En el caso de una empresa perfectamente competitiva, la demanda de mano de obra a corto
plazo tiene una pendiente negativa por la ley de los rendimientos decrecientes. Entre más trabaja-
dores contrate la empresa, menor será el PML y, por ende, más bajo será el VPML. También para el
monopolista la ley de rendimientos decrecientes hace que la curva de demanda de mano de obra a
corto plazo tenga una pendiente negativa. Pero hay otra causa en el caso del monopolista, y es que su
curva de ingreso marginal también tiene una pendiente negativa.
Por los mismos motivos que se vieron en el caso de la competencia perfecta, la demanda de
mano de obra a largo plazo del monopolista será más elástica que la demanda a corto plazo. Pero no
es necesario hacer más ajustes a la curva PIML cuando se pasa de la curva de demanda de la empresa
a la curva de demanda de la industria, lo mismo en el corto que en el largo plazos. La demanda de
mano de obra del monopolista es la demanda de mano de obra de la industria; ya tiene en cuenta el
hecho de que producción adicional significa un precio menor del producto.

OFERTA DE MANO DE OBRA


Por simplicidad, imagine un nuevo supuesto en el que sólo hay una categoría de mano de obra y que
la elección que debe hacer cada trabajador es cuántas horas trabajar al día. La alternativa del trabajo
es pasar el tiempo en “actividades recreativas”, que en este caso incluyen jugar, dormir, comer y
muchas otras actividades aparte del trabajo asalariado en el mercado laboral. Si se paga a los trabaja-
dores una tasa constante de 10 por hora trabajada, ¿cuántas horas debe trabajar?
Si se piensa, se trata simplemente de un problema de elección del consumidor, como el que se
analizó en el capítulo 3. En este contexto la elección se hace entre dos bienes que se pueden denomi-
nar “ingreso” y “tiempo libre”. Como en el problema estándar de elección del consumidor, se supone
que el individuo tiene preferencias por los dos bienes que pueden resumirse en la forma de un mapa
de indiferencia. En la figura 14.4 se representan tres curvas indicadas con I1, I2 e I3 de un trabajador
hipotético.
La recta marcada B en el mismo diagrama representa la restricción de presupuesto del individuo.
Si dedica todo su día al ocio, no gana ningún ingreso, lo cual significa que el punto (24, 0) es la inter-
sección con el eje horizontal de B. Por el contrario, si trabajara 24 horas diarias a una tasa salarial de
w0 = 10 dls./hora, su ingreso diario sería de 24w0 = 240 dólares, lo cual indica que el punto (0, 240)
debe ser la intersección con el eje vertical de B. El resto de B es la recta que une estos dos puntos.
Su ecuación es M = w(24 – h) = 10(24 – h) = 240 – 10h, donde M es el ingreso diario en dólares. La
pendiente de B es la tasa salarial por hora con signo negativo, –w0 = –10.
Dadas sus preferencias y sus restricciones de presupuesto, lo mejor que puede hacer este hipoté-
tico trabajador es el punto A de la figura 14.4, la tangente entre B y la curva de indiferencia I2. En este
caso, el paquete óptimo corresponde al gasto h* = 15 hr/día de tiempo de ocio y el resto, 24 – h* = 9
horas de trabajo pagado. El ingreso diario en dólares será (24 – h*)w0 = 90. En A, la tasa marginal de
sustitución entre ocio e ingreso es exactamente w0, la tasa salarial por hora. Esto significa que en el
paquete óptimo el valor marginal de una hora adicional de tiempo libre es exactamente igual al costo

14_CHAPTER 14.indd 464 3/7/09 12:56:29 PM


oferta de mano de obra 465

FIGURA 14.4
Ingreso (dls./día) Elección óptima de
I3
tiempo libre e ingreso
I2 la cantidad óptima de
I1
tiempo libre es h* = 15
h/día, que corresponde al
punto de tangencia entre la
24w0 = 240 restricción de presupuesto
(B) y la curva de
B indiferencia I2. la cantidad
correspondiente de mano
A de obra pagada es 24 – h*
(24 – h*)w0 = 90 = 9 h/día, que arroja un
ingreso salarial diario de
(24 – h*)w0 = 90 dls./día.
Tiempo libre (h/día)
h* = 15 24
Pendiente = –w0

de oportunidad de adquirirla, a saber, los 10 dólares que el consumidor habría ganado si hubiera
trabajado esa hora adicional.

EJERCICIO 14.2
Suponga que el salario es w = 20 dls./h. Encuentre la ecuación de la restricción de presupuesto
ingreso-tiempo libre y trace su gráfica. Suponga que con este salario un individuo escoge h =
14 horas de tiempo libre. Encuentre el ingreso diario M del trabajador para esta cantidad de
tiempo libre.

Para generar la curva de oferta de mano de obra de un trabajador, se plantea la pregunta de


cómo varía la cantidad óptima de mano de obra a medida que cambia la tasa salarial. En la figura
14.5 se observan las opciones óptimas de tiempo libre para tres tasas salariales por hora, w = 4 dls.,
w = 10 dls. y w = 14 dls. La oferta de mano de obra correspondiente a w = 4 dls. es 24 – h*1 = 6 h;
para w = 10 dls., 24 – h*2 = 9 h y para w = 14 dls., 24 – h*3 = 7 h.

FIGURA 14.5
Ingreso (dls./día)
Elección óptima de
tiempo libre con
24(14) = 336 diversas tasas salariales
Cuando el salario por hora
sube de 4 a 10 dólares, la
cantidad óptima de tiempo
w = 14 libre baja de 18 a 15 horas/
24(10) = 240 día. Pero si el salario sube
I3 más, hasta 14, la cantidad
I2
óptima de tiempo libre
w = 10 pasa a 17 horas por día.
24(4) = 96 I1

w=4
Tiempo libre (h/día)
h*2 = 15 h*1 = 18 24
h*3 = 17

14_CHAPTER 14.indd 465 3/7/09 12:56:32 PM


466 CaPítUlo 14 mano de obra

FIGURA 14.6
Curva de oferta de Salario (dls./día)
mano de obra para el Si
i-ésimo trabajador
Para este trabajador, 14
un aumento de salario
estimula una mayor oferta
10
de trabajo cuando el pago
es menor que 10 dólares
por hora, pero una oferta
de trabajo menor si el pago
es mayor que 10 dólares 4
por hora.
Oferta de mano
6 7 9 de obra de i (h/día)

En la figura 14.6 se grafica la relación entre la tasa salarial y las horas de trabajo suministradas
por el trabajador hipotético cuyo mapa de indiferencia se representa en la figura 14.5. Si se dice
que este trabajador es el i-ésimo de muchos, se ve que su curva de oferta es la línea denotada Si. Al
compararla con las otras curvas de oferta que se han visto, la característica más destacada de Si es
que en ningún momento su pendiente es positiva.2 En particular, es una curva “que se dobla hacia
atrás” para valores de w mayores que 10 dls./hora, lo cual es otra manera de decir que, en esa región,
salarios más elevados generan menos horas de trabajo suministrado.
Los colonialistas que empleaban trabajadores no calificados en países menos desarrollados pen-
saban que era síntoma de atraso que sus empleados trabajaran menos horas cuando subía su salario.
Pero como lo deja en claro el ejemplo siguiente, este comportamiento es congruente con la búsque-
da racional de un objetivo perfectamente coherente.

EJEMPLO 14.1 Smith quiere ganar 200 dólares al día porque con esa suma puede vivir cómodamente y
cumplir con todas sus obligaciones económicas. Grafique la curva de oferta de mano de obra
de Smith.
Si LS representa las horas por día que Smith escoge trabajar, debe satisfacer wLS = 200, donde w es la
tasa salarial por hora de Smith en dólares. Así, la curva de oferta de Smith está dada por LS = 200/w,
como se muestra en la figura 14.7.

FIGURA 14.7
Curva de oferta de w
mano de obra de un
trabajador que busca
cierto nivel de ingresos 40
Cuanto mayor sea su tasa
salarial horaria, menos
horas tiene que trabajar
Smith para ganar su meta
diaria de 200 dólares.
20

LS
0 5 10 20

2
Recuerde que en el capítulo 5 vio una curva de oferta parecida en el caso de los ahorros.

14_CHAPTER 14.indd 466 3/7/09 12:56:34 PM


oferta de mano de obra 467

Como es obvio, tratar de ganar un ingreso determinado no es la única meta que pueda interesar
a una persona racional. Pero tampoco tiene nada de retrógrado. Una persona que tenga esta meta
siempre trabajará menos cuando la tasa salarial suba.

EJERCICIO 14.3
Trace la curva de oferta de mano de obra de una persona con un objetivo de ingreso diario de
120 dólares.

¿Por qué es tan difícil encontrar un taxi cuando llue- EL NATURALISTA


ve? ECONÓMICO
En Nueva York, Chicago y otras grandes ciudades se puede
parar un taxi en cuestión de segundos cuando hace buen 14.1
tiempo. Pero en los días lluviosos es muy difícil. ¿A qué se
debe la diferencia?
Quizá la explicación más obvia sea que muchas per-
sonas que caminan distancias cortas con buen tiempo, pre-
fieren tomar un taxi cuando llueve. Pero hay otro factor, a
saber, que los taxistas trabajan menos cuando el tiempo es
malo. Se debe, según un estudio reciente, a que muchos
¿Por qué muchos taxis desaparecen
trabajan sólo lo necesario para juntar el nivel de ingreso cuando llueve?
establecido para cada día.3 En los días soleados de poca
demanda tienen que dedicar buena parte de su día a buscar pasajeros, así que tardan más en alcanzar
su objetivo de ingresos. El mismo objetivo se consigue mucho antes en los días lluviosos de mucha
demanda, cuando los taxis pasan llenos casi todo el tiempo.

EJERCICIO 14.4
Suponga que la meta de un taxista fuera ganar ciertas ganancias no por día, sino por semana.
También preferiría no manejar más horas de las necesarias para alcanzar su objetivo y en su
ciudad siempre llueve dos días por semana. Compare las horas que debe manejar en días lluvio-
sos con las que tiene que hacerlo en días soleados.

No todos los individuos exhiben curvas de oferta que se doblan hacia atrás. Un aumento del sa-
lario tiene tanto un efecto de ingreso como uno de sustitución en la cantidad de tiempo libre deman-
dada. Al hacer más caro el ocio, un aumento salarial lleva a la gente a consumir menos tiempo libre
y, por consiguiente, a trabajar más: un efecto de sustitución. Pero un incremento del salario también
da a la gente más poder de compra real y, dada la suposición verosímil de que el tiempo libre es un
bien normal, hace que se demande más de él (efecto de ingreso). Si el efecto de ingreso domina sobre
el de sustitución en un intervalo extenso de tasas salariales se tiene una curva de oferta de mano de
obra que se dobla hacia atrás. De otra manera, el resto de la curva tiene una pendiente positiva.

Encuentre la demanda óptima de tiempo libre para un salario w = 20 dls./hora de alguien EJEMPLO 14.2
que ve el ingreso y el ocio como complementos perfectos en una proporción de 10 a 1 (que
requiere una hora de tiempo libre por cada 10 dólares de ingreso).
La restricción de presupuesto ingreso-ocio es
M = w ( 24 − h ) = 20 ( 24 − h ) = 480 − 20h.

3
Babcock, C. Camerer, G. Loewenstein y R. Thaler, “Labor Supply of New York City Cab Drivers: One Day at a Time”, en Quarterly
Journal of Economics, 111, 1997, pp. 408-441.

14_CHAPTER 14.indd 467 3/7/09 12:56:36 PM


468 CaPítUlo 14 mano de obra

FIGURA 14.8
Cuando tiempo Ingreso (dls./día)
libre e ingreso son 480
complementos
perfectos
Si el ingreso y el tiempo
libre son complementos
perfectos en una M = 480 – 20h
proporción de 10 a 1, un M = 10h
individuo consume tiempo
libre en el punto de la
160
restricción de presupuesto
que satisface M = 10h.

0 Tiempo libre (h/día)


16 24

Como el individuo requiere una hora de ocio por cada 10 dólares de ingreso, el punto de consu-
mo debe estar sobre la línea M = 10h. La intersección de la restricción de presupuesto y esta línea de
consumo (vea la figura 14.8) da la demanda de tiempo libre:
480 − 20h = 10h ⇒ 480 = 30h ⇒ h = 16 h/día.

EJERCICIO 14.5
Encuentre la demanda óptima de tiempo libre para un salario w = 20 dls./hora de una persona
que ve el ingreso y el tiempo libre como sustitutos perfectos en una proporción de 10 a 1 (que
está dispuesta a sacrificar una hora de tiempo libre por 10 dólares de ingreso). Sugerencia: las
curvas de indiferencia de esta persona son rectas con la ecuación M = a – 10h para varios
valores de a.

Para muchas personas la tasa salarial varía con el número de horas trabajadas, como en el caso
del salario más elevado por tiempo extra. La gente que tiene la oportunidad de ganar una prima
por trabajar horas extras tiene una restricción presupuestaria quebrada, como se ejemplifica en el
ejercicio 14.6.
Muchas veces, es posible juzgar si un cambio en las posibilidades del ingreso deja a la gente
mejor o peor sin información detallada sobre sus preferencias. Bastaría conocer sus dos restricciones
presupuestarias (antes y después) y su elección inicial de tiempo libre. Si esta última se halla sobre la
nueva restricción de presupuesto pero en ese punto el salario cambió, el individuo debe estar mejor.
El individuo no estaría peor puesto que puede darse el lujo de tener el mismo tiempo libre y el mis-
mo ingreso. Ahora bien, el trabajador puede ajustar su elección de tiempo libre (más tiempo libre si
baja su ingreso, menos tiempo libre si sube su ingreso) para llegar a una curva de indiferencia mayor.
Este razonamiento le servirá para resolver el ejercicio 14.6.

EJERCICIO 14.6
Maynard puede trabajar tantas horas como desee. En su empleo actual gana 5 dls./hora por
las primeras 8 horas y 20 dólares por cada hora adicional. Conforme a este esquema de pago,
Maynard decide trabajar 12 horas al día. Si se le ofrece un nuevo empleo en el que le pagan 10
dólares por hora, ¿cambiará de trabajo? Explique.

En el mercado laboral estadounidense se ha dado una tendencia constante para que la sema-
na laboral promedio se reduzca con el paso del tiempo a la vez que aumentan los salarios reales.
Por ejemplo, en 1980 los trabajadores de manufactura laboraban 20% menos horas que los de 1914,
aunque su salario real era más de cuatro veces mayor en 1980 que en 1914. Naturalmente, esta

14_CHAPTER 14.indd 468 3/7/09 12:56:38 PM


reaCCión del qUe no eS eConomiSta ante el modelo de la oferta de mano de obra 469

correlación negativa entre tasas salariales y horas promedio no establece que el aumento de los sa-
larios sea la única causa por la cual las semanas laborales se reduzcan, pero dada la teoría de oferta
de trabajo individual que se ha expuesto aquí, parece viable suponer que es parte de la explicación.
Esta interpretación es reforzada por la observación de que una reducción modesta de los salarios de
manufactura desde 1985 ha estado acompañada por un incremento pequeño de las horas promedio
por semana.
La teoría de la oferta de mano de obra desempeña un papel crucial en la lógica de la reforma de
la asistencia social, cuya meta es dar un ingreso adicional a los pobres. Sin embargo, preocupa que
la ayuda debilite los incentivos para trabajar. A este respecto es importante la forma específica que
adopte la asistencia social. Por ejemplo, como se muestra en el siguiente ejercicio, es más probable
que las transferencias de cantidades globales reduzcan la oferta de mano de obra que los subsidios
salariales porque las transferencias generan un mayor efecto en los ingresos relacionado con el efecto
de sustitución en la oferta de mano de obra.

EJERCICIO 14.7
Considere los dos siguientes programas para combatir la pobreza: un pago de 24 dólares dia-
rios o uno de 40% del ingreso salarial. Si se supone que los pobres tienen la opción de trabajar
por 5 dls./hora, muestre el efecto de cada programa sobre la restricción de presupuesto de un
trabajador pobre representativo. ¿Qué programa es más probable que reduzca el número de
horas trabajadas?

¿ES EL TIEMPO LIBRE UN BIEN GIFFEN?


En el problema estándar de elección del consumidor que se estudió en el capítulo 3 se vio que la
curva de demanda individual para un producto tiene pendiente negativa, salvo en el caso anómalo
de un bien Giffen. En el presente capítulo se ha visto que la curva de oferta de mano de obra puede
doblarse hacia atrás, lo cual es otra manera de decir que la curva de demanda de tiempo libre puede
tener una pendiente positiva. ¿Significa esto que el tiempo libre es un bien Giffen?
La respuesta es no. Recuerde que un bien Giffen es de calidad muy inferior y tiene sustitutos
atractivos pero más caros. En el caso de este bien, si se mantiene constante el ingreso de dinero y
el precio aumenta, la cantidad demandada aumenta, porque el efecto ingreso supera el efecto susti-
tución. En cambio, en el caso del tiempo libre, un aumento de la tasa salarial constituye no sólo un
incremento del precio (o del costo de oportunidad) del tiempo libre, sino también un incremento en
el ingreso monetario (para cualquier número dado de horas trabajadas). Como con cualquier otro
bien, el efecto sustitución de un precio alto del tiempo libre es reducir su demanda. Pero si el tiempo
libre es un bien normal, el ingreso agregado aumenta su demanda. La curva de oferta de mano de
obra se dobla hacia atrás sólo cuando el efecto ingreso del aumento de salario sobrepasa el efecto
sustitución. Entonces, si el tiempo libre es un bien inferior, la curva de oferta de mano de obra nunca
puede doblarse hacia atrás. Y como sólo los bienes inferiores pueden ser bienes Giffen, el tiempo
libre no lo es.

REACCIÓN DEL QUE NO ES ECONOMISTA ANTE EL


MODELO DE LA OFERTA DE MANO DE OBRA
Al ver por primera vez el modelo económico de la oferta de mano de obra, quienes no saben de
economía consideran que es la descripción menos realista de la manera en que la gente distribuye
su tiempo entre trabajo y ocio. La mayoría de los trabajos dan pocas opciones en cuanto a las horas
diarias que alguien quiera trabajar. Desde luego, se puede escoger entre trabajo de tiempo completo
y medio tiempo, pero la segunda opción es tan poco atractiva que muchos trabajadores la ven como
una opción que ni siquiera vale la pena considerar.

14_CHAPTER 14.indd 469 3/7/09 12:56:38 PM


470 CaPítUlo 14 mano de obra

En parte, esta crítica al modelo de mano de obra se basa en una mala comprensión, pues no dice
que la gente, en sentido literal, escoge el número de horas que trabaja a diario. Los críticos aciertan en
que no es una opción que tengan la mayoría de los empleados. Pero al cabo de meses o años, la gente
tiene mucho más control sobre el tiempo que trabaja. Por ejemplo, los egresados de las escuelas de
derecho pueden trabajar en bufetes de procesos acelerados, en los que es normal laborar 14 horas
diarias los siete días de la semana, o pueden optar por empresas en las que todos se van a las cinco de
la tarde. La gente puede escoger trabajos, como en la docencia, que dejan libres los veranos. Pueden
pasar las noches en vela. Pueden cambiar de empleo a menudo y tomarse un tiempo entre tanto.
Pero aun con todas estas formas posibles, todavía puede decirse que las opciones de la mayoría
de las personas son limitadas. Si las empresas pudieran ofrecer flexibilidad sin perder la productivi-
dad, les convendría ofrecerla. Pero la mayor parte de las compañías contratan grupos de empleados
que deben interactuar y las cosas comienzan a dejar de funcionar si no están todos en las instalacio-
nes a las mismas horas del día. Esto aún deja abierta la posibilidad de que haya empresas con horarios
de distinta duración, lo cual, como ya se dijo, se ve en alguna medida. Pero incluso aquí hay límites.
Si los trabajadores de una empresa tiene que relacionarse con los de otras empresas, así sea sólo para
cambiar información por teléfono, hay un horario común en el que puede confiarse en que la gente
esté en su lugar.
Entonces, para muchas personas el tiempo que dedican al trabajo es más el resultado de las
restricciones impuestas por los empleadores que de elecciones deliberadas de su propia parte. La
necesidad de interacción de los trabajadores explica la existencia de una semana laboral común, pero
no explica por qué dura 40 horas y no 30. Tal es la pregunta que el modelo económico de la oferta
de mano de obra ayuda a responder. Éste establece que la semana laboral tiene 40 horas porque,
en promedio, es lo que los trabajadores quieren que dure. Si a la mayoría de las personas les parece
que una hora adicional de tiempo de ocio es mucho más valiosa que una hora adicional de salario,
los empleadores, que buscan el lucro, tendrían un incentivo inmediato para reducir la duración de
la semana laboral. Aquí se ve de nuevo el poder de una teoría simple para explicar lo que hacen las
personas, aun si perciben correctamente que las causas inmediatas de sus actos son fuerzas que están
lejos de su control.

CURVA DE OFERTA DEL MERCADO


La curva de oferta del mercado para cualquier categoría de mano de obra se obtiene sumando ho-
rizontalmente las curvas individuales de los posibles prestadores de mano de obra de esa categoría.
Aunque muchos individuos tengan curvas de oferta que se doblan hacia atrás (y aunque la nación en
conjunto tenga una curva de este tipo), de todos modos la curva de cualquier categoría particular
tendrá pendiente positiva casi de seguro. La explicación es que si aumenta el salario en una categoría
de mano de obra, no sólo cambia el número de horas que trabaja la gente que ya está en dicha
categoría, sino que también atrae a personas de otros ámbitos. Así como un aumento del precio de
la soya hace que muchos algodoneros empiecen a cultivarla, un aumento de los salarios que ganan
los peluqueros incita a oficinistas, vendedores de mostrador y otros a probar fortuna en los cortes de
pelo.

EJEMPLO 14.3 El aumento de las matrículas en los programas de maestría en administración de empresas
ha incrementado la demanda de catedráticos de economía. Si en la actualidad la mayoría de
los economistas enseñan en universidades generales, ¿qué efecto tiene este aumento de la
demanda de las escuelas de negocios en los salarios y el empleo de los economistas en los dos
entornos?

En la sección derecha de la figura 14.9 se muestra la curva de oferta del mercado de los economistas,
como la línea marcada S. Tiene una pendiente positiva por la suposición de que las tasas salariales
mayores para los economistas inducirán a algunos a escoger la economía antes que otras profesiones.
La curva de demanda de los economistas por parte de las universidades generales se muestra en la
sección de la izquierda. En la sección central, las curvas de demanda, original y nueva, de economis-

14_CHAPTER 14.indd 470 3/7/09 12:56:39 PM


CUrVa de oferta del merCado 471

FIGURA 14.9
Aumento de la
demanda de una
categoría de empresario
w w w DA + DB2 la demanda de economistas
DA + DB1
para que enseñen en las
escuelas de negocios
aumenta (sección central)
y hace que suba la curva
w2 w2 w2
de demanda total de
economistas en el mercado
w1 w1 w1 (sección de la derecha). el
empleo con el nuevo salario
S más alto se determina
DB2 consultando las curvas de
DA DB1
demanda respectivas del
LA LB L sector de las universidades
QA2 QA1 QB1 QB2 Q1 Q2 generales (sección de la
izquierda) y del sector de
las escuelas de negocios
(sección central).

tas por parte de las escuelas de negocios están marcadas como DB1 y DB2. Si se suman horizontalmen-
te las curvas de demanda de las escuelas de negocios (con el supuesto de que los salarios que pagan
a los economistas son una parte muy pequeña de los costos totales de la universidad para tener un
efecto importante en las colegiaturas) se tiene en la sección de la derecha las curvas de demanda total
original y nueva de los economistas, marcadas como DA + DB1 y DA + DB2, respectivamente.
Observe que el aumento de la demanda en las escuelas de negocios hace que la tasa salarial para
los economistas suba en los dos entornos de w1 a w2. Para ver el efecto sobre el empleo en los dos am-
bientes, se traza w2 a la izquierda de las curvas de demanda correspondientes. El aumento del trabajo
para economistas en las escuelas de negocios es QB2 – QB1, mientras que la reducción del empleo en
las universidades generales es de QA1 – QA2. La ganancia de empleo en las escuelas de negocios será
igual a la suma de los movimientos de salida de los puestos en las universidades generales (QA1 – QA2)
y el movimiento general hacia la profesión económica desde fuera (Q2 – Q1).
Con este ejemplo se ilustran dos puntos de interés particular. En primer lugar, la tendencia a que
los salarios de los trabajadores de determinada ocupación se igualen entre los sectores de la econo-
mía en que se emplea esa profesión. Si la demanda de carpinteros sube por un auge en los proyectos
de construcción comerciales, quien quiera construir una sala de juegos en su sótano también acaba
por pagar más. La idea es simple: si los salarios de los carpinteros que trabajan en la construcción
doméstica no se elevan, muchos dejarían el sector y se irían al de la construcción comercial. Siempre
que el trabajo en los dos entornos sea igualmente deseable en otros aspectos, el único resultado
estable es que el salario sea el mismo en ambos.
El segundo punto que se desprende del ejemplo es que un sector ocupacional pequeño puede
experimentar un aumento proporcional grande de la demanda sin que se incrementen de manera
apreciable los salarios en toda la profesión. Como, para empezar, las escuelas de negocios emplean
sólo a una pequeña fracción del número total de economistas académicos, pueden incrementar sus-
tancialmente el empleo sin tener que pagar salarios mucho más altos. La regla general es que la elas-
ticidad efectiva de la oferta en un sector ocupacional pequeño es mucho mayor que en el mercado
ocupacional tomado en conjunto.
Como cuestión estrictamente empírica, los economistas que enseñan en escuelas de negocios
ganan salarios 20% mayores que quienes dan cátedra en universidades generales. Es una diferencia
bastante grande para indicar que algo falta en nuestra teoría que reclama salarios iguales en cada
sector. No basta con observar que las escuelas de negocios son “más ricas” y pueden darse el lujo
de pagar más. La pregunta es por qué deben pagar más si la única alternativa de los economistas es
trabajar con un salario de universidad general.

14_CHAPTER 14.indd 471 3/7/09 12:56:40 PM


472 CaPítUlo 14 mano de obra

La suposición implícita en el modelo que tiene más probabilidad de ser inválida es que los eco-
nomistas consideran igualmente atractivos los dos entornos laborales. Por motivos que se explora-
rán más adelante, parece que es necesario dar a los economistas una prima para inducirlos a dejar la
universidad y pasar a la escuela de negocios.

MONOPSONIO
La ilustración clásica de un mercado laboral de un solo empresario o patrón es el llamado pueblo em-
presa. Los trabajadores no pueden o no quieren dejar la zona y no pueden entrar empresas nuevas.
Una empresa en esta posición se llama monopsonista (“comprador único”) en su mercado laboral. ¿Se
deduce que un monopsonio explotará a sus trabajadores pagándoles muy poco o dándoles muy poca
seguridad?
Se verá primero la cuestión de los salarios. Una empresa que contrata mano de obra en un
mercado laboral perfectamente competitivo tiene una curva de oferta que es una línea horizontal
costo factor promedio en el salario del mercado. Sus decisiones de contratación no tienen efecto en el salario del mercado.
(CFP) otra manera de En cambio, la curva de oferta de mano de obra del monopsonista es la curva de oferta del mercado.
llamar a la curva de oferta de Suponga, para los fines de la exposición, que tiene pendiente positiva, como la curva marcada S en
un insumo. la figura 14.10. S también se llama curva del costo factor promedio (CFP), porque indica el pago
promedio por trabajador necesario para alcanzar un nivel dado de empleo. El costo total de un nivel
costo factor total (CFT) dado de empleo (llamado costo factor total, o CFT) es el resultado de multiplicar ese nivel de em-
resultado de multiplicar el nivel pleo por el valor correspondiente del CFP. Así, en la figura 14.10, el costo factor total de un nivel de
de empleo de un insumo por
empleo de 100 trabajadores-hora es igual a 100 × 4 dls. = 400 dls./hora.
su costo factor promedio.
Ahora suponga que la empresa ya tiene 100 trabajadores y está considerando el costo del centé-
costo factor marginal (CFM) simo primero. Para aumentar su empleo en una unidad, debe elevar el salario 0.04 dl./hora no sólo
monto en que cambia el costo de la unidad adicional que quiere contratar, sino también de las 100 unidades actuales. El costo factor
factor total con el empleo total de 101 trabajadores es 4.04 dls. × 101 = 408.04 dólares. El costo factor marginal (CFM) del tra-
de una unidad adicional de bajador centésimo primero es el monto en que cambia el costo total de los factores como resultado
insumo. de contratar al trabajador:
∆CFT
CFM = . (14.3)
∆L

FIGURA 14.10
Costos factor promedio
y marginal $/L
Cuando la curva de oferta CFM = ∆CFT/∆L
(S) de un monopsonista
tiene pendiente positiva, 8.04
el costo de contratar otra
unidad de mano de obra S = CFP
(Cfm) ya no es nada más
el salario que hay que pagar.
a ese salario se le debe
sumar el pago adicional
4.04
que debe hacerse a los
4.00
trabajadores anteriores
(rectángulo sombreado).

L
100
101

14_CHAPTER 14.indd 472 3/7/09 12:56:43 PM


monoPSonio 473

Para el ejemplo de la figura 14.10 se tiene entonces que CFM = 408.04 dls. – 400 dls. = 8.04
dls./hora. El CFM del trabajador centésimo primero es la suma de los 4.04 dls./hora que se le pagan
directamente y los 4 dólares adicionales que hay que repartir entre los 100 trabajadores anteriores.
Como contratar un trabajador adicional siempre significa pagar más a los trabajadores anteriores, la
curva CFM siempre estará arriba de la curva CFP correspondiente. Si esta última es una recta con la
fórmula CFP = a + bL, la curva CFM correspondiente será una recta con la misma intersección y el
doble de pendiente que la curva CFP: CFM = a + 2bL (vea la nota 4 a pie de página).
En la figura 14.11 se describe el salario de equilibrio y los niveles de empleo de un monopsonis-
ta. La curva de demanda de mano de obra de un monopsonista se construye de la misma manera
que la de cualquier empresa. Si se trata de un competidor perfecto en el mercado de su producto,
su demanda de mano de obra será VPML. Si la curva de la demanda de su producto tiene pendiente
negativa, su demanda de mano de obra será PIML. Dada esta curva de demanda, su nivel óptimo de
empleo donde CFM y la demanda de mano de obra se intersecan, L* en la figura 14.11. En ese nivel
de empleo, debe pagar el salario dado por el valor de su curva de oferta, a saber, w*.

FIGURA 14.11
$/L Salario y niveles de
CFM = ∆CFT/∆L empleo que maximizan
las utilidades de un
monopsonista
en L*, el costo de expandir
S = CFP o contraer el empleo
es exactamente igual de
benéfico. los dos exceden
w* el salario que maximiza las
utilidades w*.

D = VPML o PIML

L
L*

El argumento de que L* es el nivel de empleo que maximiza las utilidades adopta una forma
muy parecida a la que se vio con las otras estructuras del mercado laboral. Recuerde que la curva de
demanda de mano de obra representa el incremento del ingreso total de la empresa que resulta de
contratar una unidad adicional de mano de obra, mientras que la curva CFM representa la adición
correspondiente a sus costos totales. A la izquierda de L*, la primera supera a la segunda, así que
las utilidades de la empresa aumentan si se expande el empleo. A la derecha, la segunda excede a la
primera, así que haría bien en contraerse.

EJERCICIO 14.8
La demanda de mano de obra de un monopsonista está dada por w = 12 – L. Si su curva CFP
está dada por w = 2 + 2L, con un CFM correspondiente = 2 + 4L, ¿qué tasa salarial ofrecerá y
cuánta mano de obra contratará?

4
En términos del cálculo, CFM se define como:
d ( CFT )
CFM = .
dL
Por lo tanto, si CFP = a + bL, entonces CFT = CFP × L = aL + bL2, lo que da CFM = a + 2bL.

14_CHAPTER 14.indd 473 3/7/09 12:56:46 PM


474 CaPítUlo 14 mano de obra

¿Cómo son el salario y los niveles de empleo en un monopsonio comparados con los que existen
en los mercados competitivos de mano de obra? Si la misma demanda total fuera el resultado de las
contrataciones hechas no por una, sino por muchas empresas, el nivel de empleo subiría a L**, el
punto en el cual la demanda interseca la curva de oferta de la figura 14.12. La tasa salarial también
aumentaría, de w* a w**.

FIGURA 14.12
Comparación del
$/L
monopsonio y
competencia en el CFM = ∆CFT/∆L
mercado laboral
Como el monopsonista
toma en cuenta el efecto
de las expansiones del CFM* S = CFP
empleo en los salarios
pagados a los trabajadores, w**
w*
empleará y pagará
menos que las cifras
correspondientes en la
competencia.
D = VPML o PIML

L
L* L**

En comparación con la norma competitiva, el equilibrio del monopsonio es ineficiente en el


mismo sentido que lo es el equilibrio del monopolista en el mercado de productos: no agota todas
las ganancias potenciales del comercio. Observe en la figura 14.12 que cuando el nivel de empleo es
L*, los trabajadores estarían dispuestos a suministrar una hora adicional de mano de obra por un
pago de únicamente w*, mientras el ingreso adicional que sería producido por esa unidad extra es
CFM*. Si la empresa pudiera incrementar de alguna manera el empleo total en una unidad sin pagar
más a los trabajadores, tanto éstos como el trabajador adicional estarían mejor. En la medida en que
estos intercambios están vedados por el cálculo de la maximización de utilidades, la estructura del
monopsonio es menos eficiente que el ideal competitivo.
Entonces, para la empresa monopsonista los salarios serán menores que en la competencia, lo
que presta fuerza a la acusación de explotación que hacen los críticos. ¿Qué sucede con otros elemen-
tos de la compensación, como el equipo de seguridad? En este caso también hay una tendencia a que
el monopsonista ofrezca menos, por los mismos motivos paralelos. Sin embargo, no se deduce que
los empleados de un monopsonista estén mejor con una regulación que exija equipo de seguridad adi-
cional. Los incentivos del monopsonista lo llevan a ofrecer un paquete de compensación (compuesto
de salarios, equipo de seguridad y otras prestaciones) que vale menos que el correspondiente en un
esquema de competencia. Pero el monopsonista tiene incentivos para repartir el monto total que
gasta en compensación dentro del paquete exactamente en la forma que quieren los trabajadores.
Por ejemplo, suponga que un dispositivo de seguridad vale 10 dls./semanales por trabajador y su ins-
talación y operación cuesta sólo 9 dls./semana. Este dispositivo pasaría la prueba de costo-beneficio
estándar y el monopsonista ganaría mayores utilidades si lo instala. Los trabajadores aceptarían una
reducción de 10 dólares por semana en su pago antes que estar sin el dispositivo. Como alternativa,
suponga que el dispositivo cuesta 11 dls./semana. Entonces, tanto la empresa como los trabajadores
estarían mejor sin instalarlo.
Los incentivos de la empresa en lo que respecta a la distribución de la compensación total son
exactamente los mismos que los de los trabajadores. Una ley que exigiera a la empresa instalar el
dispositivo reduciría aún más los salarios. Y si los salarios del monopsonista ya son muy bajos, no por
fuerza la verán los trabajadores como una solución atractiva.

14_CHAPTER 14.indd 474 3/7/09 12:56:47 PM


monoPSonio 475

¿Es muy importante el problema del monopsonio? Recuerde que el requisito para la competen-
cia perfecta en el mercado laboral es que los trabajadores se muevan con libertad. Muchos trabajado-
res, en especial los de más edad, tienen compromisos (redes de amigos, pagos de hipoteca, hijos en
la escuela, etc.) que les dificultan los traslados. Por otro lado, no está tan claro que esto confiera un
poder de explotación. En el nivel de entrada al mercado laboral, casi todos los trabajadores son libres
para moverse y muchos estudian con cuidado los puestos que acaban por aceptar. El difunto Stephen
Marston calculó que nada más entre 1970 y 1978, la migración entre ciudades fue de más de 25%
de la población urbana.5 Ninguna empresa puede sobrevivir mucho sin afluencia de trabajadores
nuevos; y sin un paquete de compensación atractivo, es difícil que atraigan trabajadores de nuevo
ingreso.
Los críticos responden a esta observación diciendo que las empresas ofrecen términos compe-
titivos a los trabajadores de nuevo ingreso pero luego les reducen salarios y prestaciones (o no los
aumentan lo suficiente) cuando echan raíces. Pero las empresas se hacen de una reputación en el
mercado laboral tanto como en el mercado de productos. En igualdad de circunstancias, una em-
presa con una reputación de pagar salarios competitivos a todos sus trabajadores podrá atraer a los
mejores empleados de nuevo ingreso en lugar de las empresas con reputación de explotar a los em-
pleados de más edad.
Pero incluso si ningún trabajador quiere ni puede mudarse, a la larga las empresas no están en
posición de explotar a sus empleados. Si las empresas de un campo del mercado laboral pagan mucho
menos que el valor de lo que ellos producen, es posible que lleguen empresas nuevas que compitan
por los servicios de esos trabajadores. Así, muchas empresas de ingeniería se mudaron a la región de
Seattle en la década de 1970, al inicio de la recesión de la industria aeroespacial que dejó sin trabajo a
miles de empleados técnicos.
El argumento más convincente contra la acusación de explotación indiscriminada es que las ta-
sas de utilidades que se observan en la práctica son demasiado bajas para ser compatibles con un gra-
do importante de poder de un monopsonio. Si se supone que el monopsonista mantiene los salarios
10% abajo del nivel que se paga en mercados competitivos, se deduce que la tasa de rendimiento para
dicho monopsonista será aproximadamente 50% mayor que la tasa de rendimiento competitiva.6 Sin
embargo, pocas empresas ganan constantemente tasas tan altas.
Más importante, las industrias que tienen tasas de rendimiento mayores al promedio siempre
resultan ser las de mayores salarios. En estudios empíricos se ha encontrado una y otra vez que las
tasas de utilidades tienen una influencia positiva, no negativa, sobre las tasas salariales.7 Sin duda, el

5
Véase Stephen T. Marston, “Two Views of the Geographic Distribution of Unemployment”, en Quarterly Journal of Economics,
1985.
6
Para ilustrar esta afirmación considere el caso de una empresa que contrata mano de obra en un mercado competitivo a una tasa
salarial anual de w y que puede tomar prestado a una tasa anual de interés r. Si L denota el nivel de empleo de esta empresa y K es
el tamaño de sus existencias de capital (que son sus únicos factores de producción), entonces sus costos totales, CT, están dados por:

CT = wL + rK.
Suponga que r = 0.10 y que los costos de la mano de obra son 70% de los costos totales. Entonces,

CT = 0.7 CT + 0.1K .
de lo cual se concluye que K = 3CT (en palabras, que el valor de las existencias de capital de la empresa son tres veces sus costos
anuales totales de producción).
Ahora considere una empresa idéntica a la anterior, salvo que su poder de monopsonio en el mercado laboral le permite
pagar a sus trabajadores 90% del nivel salarial competitivo. Sea Π el excedente de utilidades que recibe como resultado de su poder
de monopsonio. Para calcular Π, observe primero que el ingreso total del monopsonista (que será igual que los costos totales del
competidor, CT) será igual a los costos totales más sus utilidades mayores que las normales. Así, de la siguiente ecuación
( 0.9 )( 0.7 )CT + ( 0.1) 3CT + Π = IT = CT
se despeja Π, lo que da Π = 0.07CT. Suponga que la mitad de las existencias de capital de la empresa (1.5CT) está en manos de los ac-
cionistas; la otra mitad se financia mediante préstamos. Entonces, su tasa de rendimiento será la suma del rendimiento competitivo
del capital (0.10 × 1.5CT = 0.15CT) más el excedente del rendimiento (0.07CT) dividido entre el capital que posee (1.5CT), lo que da
0.22CT/1.5CT ≈ 0.147, un excedente de 47% sobre la tasa competitiva de rendimiento.

14_CHAPTER 14.indd 475 3/7/09 12:56:51 PM


476 CaPítUlo 14 mano de obra

minero de Virginia Occidental en un pueblo de una sola mina gana un salario bajo. Pero es probable
que la mina que le da trabajo exista por su margen de rentabilidad económica. Decir que la explo-
tación es la causa de los salarios bajos parece extraño, porque la consecuencia de pagar salarios más
elevados sería obligar a la mina a quebrar.
El argumento de la rentabilidad no dice que las empresas nunca tienen curvas de oferta de mano
de obra con pendiente positiva, pero postula que estas condiciones del mercado laboral no son un
mecanismo importante por el cual los propietarios de capital saquen una ventaja injusta de sus tra-
bajadores. Es evidente que no da una justificación convincente para regular los procedimientos de
seguridad en el centro de trabajo. (En el capítulo 17 se considera otra justificación de las normas de
seguridad, la cual no toma en cuenta el poder de mercado de los empleadores.)

LEGISLACIÓN DEL SALARIO MÍNIMO


En 1938 el Congreso estadounidense aprobó la Ley de Estándares de Trabajo Justo. Una de sus dis-
posiciones establece un salario mínimo para todos los trabajadores amparados. Al principio se limitó
a los trabajadores de empresas grandes que practicaban el comercio interestatal, pero ahora es casi
universal. El objetivo de la legislación era elevar los salarios de los trabajadores no especializados lo
suficiente para sacarlos de la pobreza. Sin embargo, desde hace mucho los economistas se sienten
escépticos en cuanto al poder del gobierno para legislar el precio de cualquier cosa. Y, en efecto, las
leyes del salario mínimo han tenido diversas consecuencias imprevistas e indeseables.
En la figura 14.13 se muestran las curvas de oferta y demanda de mano de obra no calificada, que
se intersecan en un salario real de equilibrio de w0, en el que el empleo es L0. Si el salario mínimo obli-
gatorio se fija en wm, el efecto es reducir el empleo a Dm, mientras que aumenta la cantidad de mano
de obra suministrada a Sm. La diferencia Sm – Dm es el desempleo que resulta del salario mínimo.
De acuerdo con el modelo simple de la figura 14.13 hay ganadores y perdedores por la imposi-
ción del salario mínimo. Los trabajadores no calificados que conservan su trabajo ganan más, pero,
como es obvio, quienes lo pierden ganan menos. Que el efecto neto sea aumentar el ingreso ganado
por trabajadores no especializados depende de la elasticidad de la demanda para esa categoría de
mano de obra. Si excede de 1, las ganancias bajan; si es menos que 1, aumentan.

FIGURA 14.13
Salario mínimo w
obligatorio S
el efecto del salario mínimo Desempleo
es reducir el empleo de los
trabajadores no calificados
de L0 a Dm mientras wm
incrementa la oferta de L0 a
Sm. la diferencia resultante, w0
Sm – Dm, es el desempleo
imputable al salario mínimo.

D
Lno especializado
Dm L0 Sm

7
Véase, por ejemplo, George J. Stigler, “The Economics of Minimum Wage Legislation”, en American Economic Review, 36, 1946, pp.
358-365; Laurence Siedman, “The Return of the Profit Rate to the Wage Equation”, en Review of Economics and Statistics, 61, 1979,
pp. 139-142; y Alan Kreuger y Lawrence Summers, “Reflections on the Interindustry Wage Structure”, en Econometrica, 1987.

14_CHAPTER 14.indd 476 3/7/09 12:56:52 PM


legiSlaCión del Salario mínimo 477

Los defensores del salario mínimo suponen de manera implícita que la curva de demanda para
trabajadores no calificados es casi vertical. En cambio, sus opositores la describen como muy elástica.
Estimaciones empíricas dan por resultado que es muy variable, pero en su mayor parte está debajo
de 1, lo cual indica que el efecto neto es aumentar los pagos de salarios a trabajadores no especia-
lizados.8 En algunos estudios se indica que las leyes del salario mínimo no elevan en absoluto el
desempleo general.9
Como quiera que sea, hay un gran consenso en que la legislación del salario mínimo ha redu-
cido el empleo de los adolescentes. La magnitud de la reducción del empleo de cualquier grupo
depende no sólo de la elasticidad de la demanda, sino también de la medida en que el salario mínimo
excede el nivel de saldo del mercado. Los adolescentes como grupo son mucho menos productivos
que los adultos, acaso porque tienen menos educación y experiencia; entonces, el mínimo obligato-
rio crea una brecha de desempleo más grande para ellos que para otros grupos. Recientemente se ha
propuesto en el Congreso estadounidense que de plano se elimine a los adolescentes de la legislación
del salario mínimo o bien que se les aplique un salario “submínimo” mucho menor. Quienes se opo-
nen a estas propuestas temen que algunas empresas sustituyan a los adultos con adolescentes, pero
dichas propuestas han obtenido apoyo sustancial.
Hay una excepción interesante al enunciado general de que los salarios mínimos implican una
reducción del empleo. En la figura 14.14 se muestra el caso de un monopsonio que, sin salario mí-
nimo, contrataría L* trabajadores a un salario de w*. Dado un salario mínimo de wm, su curva de
costo factor marginal se vuelve de pronto horizontal sobre la región de 0 a L1. No importa cuántos
trabajadores contrate en esa región, el costo marginal de un trabajador adicional es constante en wm.
Si quiere expandir el empleo a más de L1, debe ofrecer un salario mayor que wm, según se indica en
la curva de oferta original. Con el salario mínimo en vigor, la curva de demanda de mano de obra
del monopsonista interseca la nueva curva CPM en Lm. Así, el efecto de la legislación es aumentar el
salario y el nivel de empleo para el monopsonista.

FIGURA 14.14
Ley del salario
$/L mínimo en el caso del
CFM
monopsonio
el efecto de un salario
mínimo en wm es hacer
que la curva Cfm del
monopsonista sea
CFM*
horizontal en la región de
S 0 a L1, lo cual aumenta el
wm empleo de L* a Lm.
w*

L
0 L* Lm L1

8
Véase, por ejemplo, Daniel Hamermesh, “Economic Studies of Labor Demand and Their Applications to Public Policy”, en Journal
of Human Resources, otoño de 1976, pp. 507-525; Edward M. Gramlich, “Impact of Minimum Wages on Other Wages, Employment,
and Family Incomes”, Brookings Papers on Economic Activity, 2, 1976; Jacob Mincer, “Unemployment Effects of Minimum Wages”, en
Journal of Political Economy, agosto de 1976; Sar Levitan y Richard Belous, More Than Subsistence: Minimum Wages for the Working Poor,
Baltimore: Johns Hopkins University Press, 1979; y Finis Welch, Minimum Wages: Issues and Evidence, Washington, DC: American
Enterprise Institute, 1978. Para una revisión, véase el capítulo 4 de Ronald Ehrenberg y Robert S. Smith, Modern Labor Economics,
Glenview, IL, Scott, Foresman, 1982.
9
Vea David Card, “Using Regional Variations in Wages to Measure the Effects of the Federal Minimum Wage”, en Industrial and
Labor Relations Review, octubre de 1992, pp. 22-37.

14_CHAPTER 14.indd 477 3/7/09 12:56:53 PM


478 CaPítUlo 14 mano de obra

Los salarios mínimos no siempre aumentan el empleo en los mercados de trabajo de monopso-
nios. Por ejemplo, si el salario mínimo se fijara por arriba de CFM*, el efecto sería reducir el empleo.
Y sin importar dónde se fije el salario mínimo en la región por encima de w*, el efecto será reducir la
tasa general de rendimiento sobre la inversión del monopsonista. Si desde el principio las utilidades
del monopsonista estaban cercanas a lo normal, el efecto a largo plazo sería inducirlo a salir del
mercado. No hace falta decir que esto significaría una reducción del empleo para los trabajadores no
especializados.

EJERCICIO 14.9
La curva de demanda de mano de obra de un monopsonista está dada por w = 12 – L. Si pri-
mero tenía una curva CFP definida por w = 2 + 2L, con un correspondiente CFM = 2 + 4L, ¿qué
efecto tendrá sobre su oferta de salario y empleo la aprobación de una ley que requiera w ≥ 8?
¿Y en el caso de una ley que requiera w ≥ 10?

La legislación del salario mínimo fue alguna vez un tema mucho más debatido que en la ac-
tualidad, porque la inflación ha reducido a tal grado el valor real del salario mínimo que está por
debajo del nivel de equilibrio salarial en muchos mercados de mano de obra no especializada. Por
ejemplo, en la región de Boston los empleados de reciente ingreso en los restaurantes de comida
rápida ganan aproximadamente el doble del salario mínimo. Hasta que el Congreso estadounidense
no promulgue un incremento sustancial del salario mínimo, es probable que el interés en la materia
se siga esfumando.

SINDICATOS
Aproximadamente uno de cada seis trabajadores del sector no agrícola de la economía estadouniden-
se pertenece a un sindicato. La principal diferencia entre empleados sindicalizados y no sindicalizados
es simple. Los primeros negocian colectivamente los términos y condiciones del empleo. En el caso
de los segundos la empresa anuncia su oferta y ellos la toman o la dejan, quedándose o abandonando
la compañía. Los sindicatos también facilitan la comunicación entre trabajadores y patrones.
Durante buena parte del siglo pasado los economistas se centraron exclusivamente en el aspecto
de negociación colectiva de las actividades sindicales. El consenso en la profesión es que los sindicatos
eran en el mercado de mano de obra el análogo de los cárteles en el mercado de productos, puesto
que velaban únicamente por fomentar los intereses de sus integrantes, a expensas del bienestar eco-
nómico general. El argumento básico con que se justificaba esta afirmación es sencillo.
Para ilustrarlo, considere una economía simple con dos sectores, uno sindicalizado y el otro no.
Suponga que la oferta total de mano de obra de los dos sectores está fija en S0 y que las curvas de

FIGURA 14.15
Efectos distributivos wU wN
de las negociaciones Sector sindicalizado Sector no sindicalizado
colectivas
Sin negociación colectiva, el
mismo salario, w0, prevalece
en todos los sectores. Con
el salario sindical fijo en wU, wU
el empleo cae en el sector
w0 w0
sindical. los trabajadores
desplazados buscan wN
colocarse en el sector no
sindicalizado, lo que baja los
salarios. el resultado es una DU DN
reducción de la producción LU LN
nacional. LU1 LU0 LN0 LN1

14_CHAPTER 14.indd 478 3/7/09 12:56:55 PM


SindiCatoS 479

demanda de mano de obra sindicalizada y no sindicalizada se muestran en DU y DN en las secciones


izquierda y derecha de la figura 14.15, respectivamente. Sin negociación sindical, el mismo salario
w0, prevalecería en cada sector y los niveles de empleo en los dos sectores serían LU0 y LN0, respectiva-
mente, donde LU0 + LN0 = S0.
La negociación colectiva fija el salario del sector sindical en wU > w0. La demanda de mano de
obra tiene una pendiente negativa y esto hace que las empresas del sector sindicalizado reduzcan el
empleo de LU0 a LU1. Los trabajadores desplazados del sector sindicalizado se ven obligados a buscar
trabajo en el no sindicalizado, lo que rebaja el salario a wN en este último sector.
A primera vista, el proceso asemeja un juego de suma cero, en el que las ganancias de los traba-
jadores sindicalizados quedan compensadas exactamente por las pérdidas de los que no pertenecen
a un sindicato. Pero con un examen más minucioso se ve que, en realidad, se reduce el valor de la
producción total. Recuerde que en el capítulo 9 se vio que la condición de maximización de la pro-
ducción mediante dos procesos es que el valor del producto marginal del recurso sea igual en ambos.
Con el salario fijo inicialmente en w0 para los dos sectores, esa condición quedaba satisfecha. Pero
con la divergencia salarial causada por las negociaciones colectivas, el valor total de la producción
ya no está en su máximo. Observe que si un trabajador queda fuera del sector no sindicalizado, la
reducción en el valor de la producción será de sólo wN, que es menor que wU, la ganancia en el valor
de la producción cuando el mismo trabajador se incorpora al sector sindicalizado.
La distorsión económica que implica el análisis de la figura 14.15 es exagerada. Si se obliga a la
empresa sindicalizada a pagar salarios más altos, atraerá un exceso de oferta de mano de obra. En
la práctica, las habilidades de los trabajadores varían mucho y la respuesta natural de la empresa
sindicalizada es elegir a los solicitantes de empleo más capaces. En el otro lado de la misma moneda,
las empresas no sindicalizadas tienen que contratar trabajadores que son menos productivos que el
promedio. Estudios empíricos han demostrado que la prima sindical que no es justificada por las
diferencias en la calidad de los trabajadores es de, apenas, alrededor de 10%. Esto significa que la
ganancia de mover trabajadores del sector no sindicalizado al sindicalizado será menor de lo que
parecía al principio.
Pero aun si la prima salarial sindical es de sólo 10%, hay algo que no se entiende acerca de la
capacidad de las empresas sindicalizadas para competir de manera provechosa con sus contrapartes
no sindicalizadas. Desde luego, a veces estas últimas se las arreglan para sacar del negocio a aquéllas,
como ocurrió cuando la industria de los textiles se mudó al sur para escapar a la carga de los salarios
más altos de los sindicatos en Nueva Inglaterra. Pero la mayor parte del tiempo las empresas sindica-
lizadas y no sindicalizadas compiten frente a frente durante largos periodos. Si los costos de éstas son
significativamente mayores, ¿cómo se las arreglan para sobrevivir?
Los investigadores han descubierto varias maneras en las cuales los sindicatos fomentan la pro-
ductividad.10 En la opinión revisionista se subraya su papel como transmisores de las preferencias
de los trabajadores a la administración. Cuando la comunicación entre trabajadores y patrones no
fluye sin estorbos, la única opción abierta para los trabajadores insatisfechos es irse de la empresa
en busca de una mejor situación. En la explicación revisionista, la función del sindicato es dar a los
trabajadores una voz como alternativa para que no renuncien. La organización de procedimientos
formales de queja, junto con una remuneración monetaria más alta, eleva la moral de los trabaja-
dores sindicalizados, lo cual lleva a aumentos de la productividad. Por ejemplo, las tasas de renuncia
de las empresas sindicalizadas son mucho menores que en las empresas no sindicalizadas, lo que
les permite economizar costos de contratación y capacitación. En trabajos empíricos recientes se
indica que la productividad de los sindicatos puede ser lo bastante alta para compensar los salarios
privilegiados. Es decir, aunque los salarios monetarios sean mayores en las empresas sindicalizadas,
los costos de mano de obra por unidad de producción pueden no serlo. Si esta conclusión es correc-
ta, resuelve la paradoja de cómo las empresas sindicalizadas sobreviven en la competencia con sus
rivales no sindicalizadas.
Pero así se suscita una pregunta todavía más inquietante: si los sindicatos elevan la moral y
aumentan los salarios sin elevar los costos por unidad de mano de obra, ¿por qué no se sindicalizan

10
Véase, en particular, Albert O. Hirschman, Exit, Voice, and Loyalty, Cambridge, MA, Harvard University Press, 1973; y Richard B.
Freeman y James Medoff, What Do Unions Do?, Nueva York, Basic Books, 1985.

14_CHAPTER 14.indd 479 3/7/09 12:56:56 PM


480 CaPítUlo 14 mano de obra

todas las empresas? La tendencia de la sindicalización en Estados Unidos se hizo más lenta durante la
posguerra, precisamente lo contrario de lo que la teoría revisionista haría pensar. Con base en lo que
se sabe, es tentador concluir que los sindicatos fomentan la productividad al grado de compensar los
aumentos salariales en unas industrias pero no en otras. Sin embargo, hay que emprender muchas
más investigaciones para tener una idea clara de lo que hacen los sindicatos.

DISCRIMINACIÓN EN EL MERCADO
LABORAL
Uno de los problemas económicos de mayor volatilidad emocional es el fenómeno de la discrimi-
nación en el mercado de mano de obra. Las discusiones empiezan casi siempre señalando las
grandes disparidades de ganancias que hay entre los grupos de la población económicamente acti-
va. Por ejemplo, las ganancias promedio de los hombres negros son alrededor de 30% menores
que las de los blancos. La proporción correspondiente entre mujeres y hombres es más o menos la
misma.
Todos aceptan que por lo menos algunos componentes de estas diferencias no tienen nada que
ver con la discriminación de los empleadores. Por ejemplo, parte de la distinción entre blancos y
negros refleja el hecho de que el promedio de edad de estos últimos es casi una década menor que
la de aquéllos. Como las ganancias aumentan con la experiencia, los blancos ganan más, en parte,
sólo porque son mayores. En el mismo tenor, parte de la diferencia entre hombres y mujeres refleja
la pauta histórica de que la participación de las mujeres en las actividades económicas siempre fue
mucho más intermitente que la de los hombres. Los salarios aumentan más cuando un trabajador
sigue el avance profesional ordenado que es característico de los esquemas de empleo para hombres.
La situación tradicional de las mujeres era dejar de trabajar varias veces como consecuencia de los
partos, lo que muchas veces significaba volver a empezar cada vez desde el primer peldaño del esca-
lafón corporativo.
Sin duda, cada efecto es el resultado de cierta discriminación contra los grupos afectados. Por
ejemplo, el promedio de edad de los negros es menor, en parte, porque crecen en medio de la pobre-
za, sin los recursos educativos o médicos para tener la misma esperanza de vida que los blancos. Na-
die niega que estas condiciones estén arraigadas en una historia de segregación social de los negros.
Tampoco hay muchas personas que nieguen que la distribución asimétrica de las responsabilidades
en la crianza de los hijos entre los sexos sea resultado, en alguna medida, de las actitudes sociales dis-
criminatorias sobre los papeles sexuales. Pero para los fines actuales es importante recalcar que desde
el punto de vista de cada empresario, estos efectos son ejemplos de discriminación que no es propia
del mercado, es decir, aminoran la productividad antes de que los solicitantes siquiera se pongan
en contacto con las empresas. Las diferencias salariales de las que es causa la discriminación ajena
al mercado no pueden atribuirse por lógica a los usos actuales de contratación de los empleadores.
Un patrón que no discriminara en absoluto tendría que pagar diferencias salariales semejantes de
acuerdo con estos efectos o sería orillado a la quiebra por sus competidores que sí discriminan.
Aquí lo que interesa es la parte de las diferencias salariales que no puede imputarse a la discri-
minación ajena al mercado. En particular, importa el caso en que una empresa paga menos a un
negro, una mujer, un latinoamericano o cualquier otro miembro de un grupo minoritario de lo que
le pagaría a un blanco igualmente productivo (o, en el caso más extremo, que se niega a contratar
miembros de tales grupos). Se han postulado numerosas teorías para explicar por qué las empresas
operan de esa manera.
Una teoría es que los clientes de una empresa no quieren tratar con empleados de minorías.
La llamada discriminación de los clientes es más fuerte en los vívidos ejemplos de segregación en los
comedores del sur de Estados Unidos en la primera mitad de la década de 1960. Cualquier dueño
de un comedor sureño que tomaba la iniciativa unilateral de fracturar la barrera de color corría el
riesgo de que el grueso de su negocio se fuera a los competidores que sostenían la tradición de un
personal completamente blanco. La Ley de Derechos Civiles de 1964 proscribió esta discrimina-
ción, para que ninguna empresa pudiera tener una ventaja sobre sus rivales si se negaba a contratar
negros.

14_CHAPTER 14.indd 480 3/7/09 12:56:56 PM


diSCriminaCión en el merCado laboral 481

Cuando la discriminación laboral es el resultado de actitudes de los clientes de la empresa, la


acción colectiva a través de la legislación es una forma lógica, quizás incluso la única práctica, de
terminar con el estancamiento. El motivo es que, sin legislación, la discriminación puede ser la única
estrategia abierta para las empresas que es congruente con la maximización de utilidades y, por ende,
con la supervivencia. Es probable que el personal de raza negra sea tan productivo como el de raza
blanca para preparar y servir alimentos. Pero desde el punto de vista de un comedor, las personas de
raza negra son menos productivas debido a la inclinación racial de los clientes.
Se pueden aplicar consideraciones semejantes a la decisión de un bufete de abogados con respec-
to a si debe contratar licenciadas. Si es menos probable que los clientes, incluso los jueces, acepten
con seriedad la recomendación de una abogada, la posición del bufete de abogados es totalmente
análoga a la del propietario sureño del comedor. El bufete de abogados podría estar convencido de
que los clientes y jueces cambiarían de parecer con respecto a las licenciadas si tuvieran la expe-
riencia suficiente en el trato con ellas. Pero si contrata abogadas y los competidores no lo hacen,
en el corto plazo su negocio sufrirá las consecuencias. Una vez más, puede ser que en este caso la
legislación que requiera igualdad de trato en la contratación sea la única forma eficaz de terminar
con el estancamiento.
La discriminación de los clientes es una explicación poderosa de la discriminación laboral en
casos como los anteriores. Pero no puede explicar las diferencias de salarios en el caso de trabajado-
res, como obreros de producción de manufactura, que nunca tienen contacto con los clientes. Las
diferencias de salario en dichos casos a veces se explican como el resultado de la discriminación de los
compañeros de trabajo. Por ejemplo, es posible que los empleados de raza blanca que se sientan incó-
modos por trabajar con personas de raza negra prefieran laborar en empresas donde sólo contraten
trabajadores de raza blanca. O los egos frágiles de algunos hombres que no puedan tolerar la idea de
recibir órdenes de una supervisora.
Dichas preferencias implican segregación laboral, mas no una pauta de diferencias de sala-
rios para trabajadores igualmente productivos. Las personas de raza negra pueden trabajar jun-
tas en algunas empresas o plantas, mientras los trabajadores de raza blanca lo hacen en otras. O
quizá los hombres tiendan a trabajar en ambientes separados de las mujeres. A veces se observa
este tipo de segregación y esta opción dificulta que la teoría de discriminación de compañeros de
trabajo explique cualquier aspecto significativo de las diferencias de salarios observados. Un
empresario de un establecimiento que tuviera exclusivamente personas de raza negra o muje-
res y que pagara salarios más bajos que los recibidos por hombres de raza blanca de la misma
productividad tendría costos más bajos y, por lo tanto, utilidades más altas que los empleadores
de hombres de raza blanca nada más. Esto proporcionaría un incentivo para que una empresa
nueva hiciera una oferta por los trabajadores de ese patrón, un incentivo que debe persistir
hasta eliminar todas las diferencias de salarios.
La discriminación del empresario es el término que se suele usar para describir las diferencias sala-
riales que surgen de una preferencia arbitraria por parte del patrón de un grupo de trabajadores en
vez de otro. Como es el tipo que visualizan los reportes populares de discriminación en el mercado
laboral, se va a estudiar con detalle. Para describir de manera formal el proceso, suponga que hay
dos grupos de fuerza de mano de obra, los M y los F y que no hay diferencias de productividad entre
ellos. En concreto suponga que los valores de sus productos marginales respectivos son iguales:

VPMF = VPM¿M = V0, (14.4)

y que los patrones discriminantes pagan un salario de V0 a M, pero sólo V0 – d a F.


Los costos de mano de obra para un patrón discriminante serán un promedio ponderado de V0 y
V0 – d, en cuyo caso los pesos son las respectivas participaciones de los dos grupos en la fuerza laboral
del patrón. Por lo tanto, entre más M contrate el patrón, mayores serán sus costos.
Además de los casos aislados en los que la discriminación del cliente podría ser un factor perti-
nente, un consumidor no estará dispuesto a pagar más por un producto hecho por un F. Si el precio
del producto no se ve afectado por la composición de la fuerza laboral que lo genera, la utilidad de
la empresa será menor mientras más M emplee. La empresa más redituable será la que sólo em-
plee F.

14_CHAPTER 14.indd 481 3/7/09 12:56:57 PM


482 CaPítUlo 14 mano de obra

Dada la suposición inicial de que a los M se les paga el valor de su producto marginal, las empre-
sas que sólo emplean M obtendrán una utilidad normal, mientras las que contraten una combina-
ción recibirán una utilidad económica positiva. La diferencia salarial inicial ofrece a los patrones una
oportunidad de contratar principalmente F para crecer a expensas de sus rivales. De hecho, como
dichas empresas obtienen una utilidad fuera de lo normal por la venta de cada unidad de producto,
su incentivo es expandirse lo más rápido posible. Y para hacerlo, es obvio que querrán continuar con
la contratación exclusiva de F.
Pero conforme las empresas que buscan utilidades continúan en la búsqueda de esta estrategia,
la oferta de F en la tasa salarial V0 – d no será adecuada para una mayor expansión. La solución en el
corto plazo es ofrecer a F un salario ligeramente más alto. Pero esta estrategia sólo sirve si otras em-
presas no la ponen en práctica. Una vez que ellos también ofrezcan un salario más alto, los F estarán
escasos de nuevo. Desde luego que, al final, la única solución será que la oferta salarial de F suba hasta
V0, eliminando así mayores oportunidades de una expansión redituable al contratar más F.
Cualquier patrón que quiere externar su preferencia por contratar M ahora lo debe hacer pagan-
do a M un salario mayor que V0. Los patrones pueden discriminar a los F si quieren, pero sólo si están
dispuestos a pagar primas salariales a los M de sus utilidades. Antes se vio que si un monopolista paga
a sus trabajadores 10% menos que el salario actual (o VPML), ganaría escasamente 50% más que la
tasa de rendimiento competitiva sobre la inversión. Un cálculo paralelo revela que una empresa que
paga a sus trabajadores 10% más que el VPML ganaría apenas 50% menos que la tasa de rendimiento
competitiva. Pocas empresas podrían continuar atrayendo capital durante mucho tiempo a tasas de
utilidad muy inferiores a las normales.
El modelo de mercado de mano de obra competitivo postula que la persistencia de discrimina-
ción significativa del patrón requiere que los propietarios de la empresa suministren capital a una
tasa de rendimiento sustancialmente menor que la que ganarían invirtiendo el dinero en otra parte.
La teoría de los mercados de mano de obra competitivos dice que a menos que se tenga una razón
verosímil para suponer que es posible hacerlo, se debe concentrar la búsqueda de las fuentes de las
diferencias de salarios en factores distintos a la discriminación patronal. O se deben buscar otras for-
mas en las cuales la teoría de los mercados de mano de obra competitiva no ofrezca una descripción
completa de la realidad.

DISCRIMINACIÓN ESTADÍSTICA
En el capítulo 6 se vio cómo las compañías de seguros utilizaron los datos de reclamaciones pro-
medio de varios grupos para encontrar tasas diferenciales para los asegurados cuyos registros de
reclamación individual fueran idénticos. La teoría de los mercados de mano de obra competitiva dice
que se pagará a los empleados los valores de sus respectivos productos marginales. Pero el producto
marginal de un empleado no es como un número tatuado en la frente para que todos lo vean. Por el
contrario, como la gente muchas veces trabaja en actividades de equipo complicadas, con frecuencia
es sumamente difícil, incluso después de muchos años de trabajo, calcular la contribución de cada
uno. El problema de estimar la productividad de los solicitantes de empleo, con quienes el patrón no
ha tenido trato directo, es obviamente mucho más difícil.
Aun así, la tarea no es caso perdido. Así como las compañías de seguros saben por su vasta ex-
periencia que es más probable que los adolescentes de sexo masculino presenten reclamos de acci-
dentes que otros conductores, los empleadores también saben que los solicitantes de ciertos grupos
tienen mayor probabilidad de ser más productivos que otros. Por ejemplo, el graduado universitario
promedio será más productivo que el graduado de preparatoria promedio, a pesar de que muchos
preparatorianos son mucho más productivos que sus contrapartes universitarios.
En el caso de los seguros se vio que aun cuando dos personas tuvieran registros de manejo idén-
ticos, las presiones competitivas condujeron a diferentes tasas si pertenecían a grupos con diferentes
registros de accidentes. En el mercado de la mano de obra se ven resultados muy parecidos. Aun
cuando la información de patrón indica que dos individuos tienen exactamente la misma producti-
vidad, habrá presión competitiva por pagar un salario más alto a la persona que pertenezca al grupo
con productividad promedio más alta. El problema es que, a menos que la información del patrón
sobre la productividad individual sea perfecta, la pertenencia a un grupo transmite información re-

14_CHAPTER 14.indd 482 3/7/09 12:56:58 PM


diSCriminaCión eStadíStiCa 483

levante sobre las probables diferencias de productividad, información que una empresa sólo puede
ignorar bajo su propio riesgo.
Para demostrar cómo influye ser miembro de un grupo en el cálculo de la productividad indivi-
dual, considere un grupo de mercado de mano de obra, llámele grupo A, cuyos VPM de sus miem-
bros se distribuyeron uniformemente entre 10 dls/h y 30 dls/h, como se muestra en la figura 14.16.
Esto significa que si se tuviera que elegir al azar a una persona del grupo A, el valor de su producto
marginal sería igualmente probable que fuera cualquier número entre 10 dls/h y 30 dls/h. Si no se
supiera nada de esta persona aparte de que pertenece al grupo A, el valor esperado de su productivi-
dad simplemente sería el promedio de los miembros de ese grupo, que es de 20 dls/h.

FIGURA 14.16
Distribución hipotética
uniforme de la
productividad
los valores de
productividad para los
Grupo A miembros de este grupo se
distribuyeron de manera
Valor del producto uniforme entre 10 dls/h y
0 10 20 30 40 marginal (dls./h) 30 dls/h. esto significa que
el VPm de una persona
elegida al azar del grupo
es igualmente probable
que sea cualquier número
entre 10 dls/h y 30 dls/h.
el VPm promedio para los
miembros de este grupo es
de 20 dls./h.

Si no hay una forma práctica de aprender algo sobre la productividad específica de una persona
y si se conociera la distribución de productividad de su grupo, las presiones competitivas requerirían
que los miembros del grupo A recibieran 20 dls/h (véase la nota 11). Suponga que un patrón ofrece
menos, por ejemplo 15 dls./h, quizá por temor a no tener suerte y contratar al miembro menos pro-
ductivo del grupo A. Este patrón no podría retener a sus trabajadores porque una empresa competiti-
va podría ofrecer 16 dls/h y atraerlos. Y como los trabajadores del grupo A valen un promedio de 20
dls./h cada uno, esta empresa de competencia aumentaría sus ganancias esperadas 4 dls/h por cada
trabajador contratado. Pero por el mismo motivo, con el tiempo también perdería a los trabajadores
ante otra empresa en competencia.
De manera alternativa considere una empresa que pagó 25 dls./h a los trabajadores del grupo
A, quizá porque le incomodó pagar un salario inferior a los miembros más productivos de ese grupo.
Esta empresa perdería un promedio de 5 dls/h por cada trabajador del grupo A que contratara, a
menos que tuviera alguna fuente de utilidades aparte de las normales, tarde o temprano se vería
obligada a cerrar.
Ahora suponga que los patrones tienen una prueba de productividad. Ésta no es perfecta, pero
ofrece información acerca de los valores de productividad individuales. Para simplificar el análisis,
suponga que la prueba es 100% precisa la mitad de las veces, pero en la otra mitad no tiene valor (es
decir, produce un número al azar obtenido de la distribución de productividad del grupo); y que los
patrones no tienen manera de saber en qué momento es precisa.

11
En este análisis se ignora la complicación de compensar las diferencias salariales de una clasificación interna, que se analiza en la
siguiente sección.

14_CHAPTER 14.indd 483 3/7/09 12:56:59 PM


484 CaPítUlo 14 mano de obra

Además, suponga que la prueba se aplica a un trabajador del grupo A y que produce un valor
de 24 dls/h. ¿Cuál es la mejor estimación de la productividad real de este trabajador? La prueba es
100% precisa la mitad de las veces y si se supiera que esta ocasión es una de ésas, desde luego que la
respuesta sería 24 dls/h. Al contrario, si se supiera que ésta es una de las veces en que la prueba no
vale, el mejor cálculo sería el valor esperado de un número al azar obtenido de la distribución unifor-
me entre 10 dls/h y 30 dls/h, a saber, 20 dls/h, el valor de productividad promedio del grupo A. El
problema es que no se sabe en qué modo particular queda la prueba. Así que lo mejor que se puede
hacer es tomar un promedio ponderado de los dos resultados (en el que los pesos son las probabili-
dades respectivas de que ocurran). El mejor cálculo del VPM de un trabajador del grupo A con una
calificación de prueba es 24 dls/h, denotada como VPM(24), se obtiene, por lo tanto, con

VPM(24) = ( –12 )(24 dls/h) + ( –12 )(20 dls/h) = 22 dls./h (vea la nota 12 a pie de página). (14.5)

Esto significa que si se toma un número de personas grande del grupo A cuya calificación en la prue-
ba fue 24 dls./h, su valor de productividad promedio entonces resultaría ser 22 dls./h.
Suponga que, en cambio, se observa que el resultado de la prueba de un miembro del grupo A
es 16 dls./h. El mejor cálculo de su VPM entonces sería

VPM(16) = ( –12 )($16/h) + ( –12 )(20 dls/h) = 18 dls/h. (14.6)

Observe en esta ocasión que el efecto de la incertidumbre en la prueba provoca que se revise hacia
arriba la estimación de la productividad individual. En general, la regla es que cuando una prueba es infe-
rior a la precisión absoluta, la mejor estimación de la productividad de un trabajador radicará entre su califica-
ción real de la prueba y la productividad promedio del grupo al que pertenece. Y, de nuevo, la predicción de la
teoría del mercado laboral competitivo es que cualquier empresa que no pague a sus trabajadores de
acuerdo con las mejores estimaciones disponibles de sus VPM respectivos, tarde o temprano saldrá
del mercado debido a las fuerzas de competencia.

EJERCICIO 14.10
En el ejemplo anterior, ¿cuál es el mejor cálculo del VPM de una persona con una calificación
de 12?

Ahora suponga que un empresario tiene solicitantes de empleo no sólo del grupo A, sino tam-
bién del grupo B. Y suponga que la distribución de VPM del grupo B es uniforme entre 20 dls./h y 40
dls./h, como se muestra en la figura 14.17. Por último, suponga que en la mañana llegan dos solici-
tantes, uno del grupo A y uno del grupo B y que cada uno recibe una calificación de 28 en la prueba
(la misma que antes). ¿Cuáles son las mejores estimaciones del patrón de sus respectivos valores de
productividad?
En ambos casos la imperfección de la prueba requiere un ajuste hacia el promedio del grupo
relevante. En concreto, la mejor estimación del VPM para el trabajador del grupo A, denotada como
VPMA(28), es

VPMA(28) = ( –12 )(28 dls/h) + ( –12 )(20 dls/h) = 24 dls/h, (14.7)

12
Observe la similitud de este procedimiento de ajuste con el que se analizó en el capítulo 8, por medio del cual se estimó la probabi-
lidad de que una persona tímida determinada fuera bibliotecaria.

14_CHAPTER 14.indd 484 3/7/09 12:57:00 PM


eStrUCtUra Salarial interna 485

FIGURA 14.17
Distribuciones de la
productividad para dos
grupos
los valores de VPm de
los miembros del grupo A
se distribuyen de manera
Grupo B uniforme entre 10 dls/h
y 30 dls/h, mientras los
Grupo A miembros del grupo B se
Valor del producto distribuyen entre 20 dls/h y
marginal (dls/h) 40 dls/h. Si sólo se conocen
0 10 20 30 40 los grupos a los que
pertenecen las personas,
la mejor estimación de un
VPm individual sería el VPm
promedio de su grupo, 20
dls/h para el grupo A y 30
dls/h para el grupo B.

mientras la estimación correspondiente del trabajador del grupo B es

VPMB(28) = ( –12 )(28 dls/h) + ( –12 )(30 dls/h) = 29 dls/h. (14.8)

Por consiguiente, aun cuando los dos trabajadores obtengan exactamente la misma calificación
de la prueba, el empresario ajusta hacia abajo en un caso y hacia arriba en el otro. De nuevo observe
que si la empresa no paga a sus trabajadores de acuerdo con las mejores estimaciones disponibles de
sus VPM, está en peligro de extinción. Es un eufemismo cruel decir que tales exigencias competitivas
han provocado gran dolor a los empleados y patrones. Los miembros más talentosos y productivos
del grupo A no pueden evitar sentirse humillados cuando la identidad de su grupo provoca que sean
tratados de manera diferente de los miembros de otros grupos. Y seguramente no puede haber mu-
chos empleadores que se sientan cómodos ofreciendo diferentes salarios a personas cuyos registros
parecen iguales.
Observe con atención que la discriminación estadística es el resultado y no la causa de las diferencias de
productividad promedio entre grupos. Su único efecto es reducir la variación de los salarios dentro de cada
grupo. Si los empleadores de repente cambiaran a una política para establecer salarios con base estric-
tamente en la información específica del individuo, la diferencia de salarios promedio entre grupos
se mantendría igual que antes.

ESTRUCTURA SALARIAL INTERNA


En el primer examen la estructura salarial de muchas empresas privadas parece mucho más equitativa
de lo que garantizaría la teoría de productividad marginal de salarios. Por ejemplo, muchas empresas
siguen fórmulas salariales estrictas que se basan en la experiencia, educación y duración del ejercicio
del cargo dentro de la empresa, aun cuando haya grandes diferencias visibles en la productividad de
los trabajadores pagan lo mismo de acuerdo con estas fórmulas. De hecho, las pautas de pago del
tipo que se proyectan en la teoría de productividad marginal casi nunca se observan en la práctica.
Una simple modificación a la teoría que se ha expuesto aquí ayuda a cuadrarla con las distribu-
ciones salariales que se observan en la práctica.13 La modificación se basa en dos supuestos posibles:
1) la mayoría de la gente prefiere puestos de alto rango entre sus compañeros de trabajo; y 2) nadie
puede ser obligado a permanecer en una empresa en contra de su voluntad.

13
Para un desarrollo más completo del argumento, vea R. Frank, Choosing the Right Pond, Nueva York, Oxford University Press,
1985.

14_CHAPTER 14.indd 485 3/7/09 12:57:02 PM


486 CaPítUlo 14 mano de obra

De acuerdo con las leyes de la aritmética simple no todos pueden satisfacer su preferencia de
un puesto alto en la distribución salarial de la empresa. Después de todo, sólo 50% de los miembros
de cualquier grupo pueden estar en la mitad superior. Pero si la gente tiene la libertad de asociarse
con quien quiera, ¿por qué los miembros de grupos con puestos inferiores permanecen con gusto?
¿Por qué no se van y forman grupos nuevos propios en los que ya no estén cerca del fondo? Sin
duda muchos trabajadores lo hacen. Sin embargo, también se observan muchos grupos estables y
heterogéneos. No todos los contadores de General Motors son igual de talentosos; y en cada bufete
de abogados, algunos socios atraen más clientes que otros. Si todos prefieren estar cerca de la parte
superior de su grupo de compañeros de trabajo, ¿qué mantiene unidos a estos grupos?
La respuesta aparente es que los miembros de categoría baja reciben una compensación extra. Si
se van, lo que ganarían es no tener una condición inferior. Sin embargo, del mismo modo, los miem-
bros de categoría alta perderían; ya no disfrutarían su condición superior. Si sus ganancias por tener
una categoría alta son más grandes que los costos que pagan los miembros con una calificación baja,
no tiene sentido que se deshaga el grupo. A todos les puede ir mejor si los trabajadores con categoría
alta inducen a sus colegas de categoría baja a permanecer si comparten con ellos parte de su pago.
Desde luego que no todos asignan el mismo valor a tener una categoría alta. A quienes no les
preocupa les irá mejor si se integran en empresas donde la mayoría de los trabajadores son más
productivos que ellos. Como miembros con una calificación más baja, reciben una compensación
extra. Por el contrario, a las personas que les preocupa mucho la calificación, querrán incorporarse
a empresas donde la mayoría de los trabajadores sean menos productivos que ellas. Por el privilegio
de ocupar puestos de categoría alta en esas empresas tendrán que trabajar por un valor inferior al que
tiene lo que producen.
Así, los trabajadores se pueden clasificar entre una jerarquía de empresas de acuerdo con sus
demandas de posición dentro de la empresa. En la figura 14.18 se describe el menú de opciones a las
que se enfrentan los trabajadores cuya productividad ocupa un valor determinado, M0. Las líneas
representan los programas salariales ofrecidos en tres empresas. Indican el salario que cada empresa
pagaría a un trabajador con una productividad determinada. El nivel de productividad promedio
es más alto en la empresa 3, el siguiente más alto está en la empresa 2 y el más bajo en la empresa
1. El problema que tienen las personas con nivel de productividad M0 es elegir en cuál de estas tres
empresas trabajar.

FIGURA 14.18
Estructura salarial Salario
cuando importa la
posición local
a mayor nivel de Empresa 3
productividad promedio C
de una empresa, menor
la clasificación de un
trabajador determinado. Empresa 2
M0
los trabajadores que eligen B
posiciones con categoría
alta (A) deben compartir Empresa 1
parte de su salario con los A
compañeros de trabajo
con calificación más baja.
quienes eligen puestos
de calificación más baja
(C) reciben pagos de 45°
Productividad (VPML)
compensación de sus 0 M0
compañeros de trabajo.

A los trabajadores que les preocupa más el estatus querrán “comprar” puestos de categoría alta
como el que se identifica con A en la empresa 1. En dichos puestos, trabajan por menos que el valor
de lo que producen. Por el contrario, a quienes no les preocupa tanto la posición elegirán recibir

14_CHAPTER 14.indd 486 3/7/09 12:57:03 PM


eStrUCtUra Salarial interna 487

primas salariales trabajando en puestos de categoría baja como el identificado con C en la empresa 3.
Los trabajadores con una preocupación moderada acerca de la calificación local serán atraídos hacia
los puestos intermedios como el identificado con B en la empresa 2, por el que no pagan ni reciben
una compensación por una categoría local.
Observe también en la figura 14.18 que aun cuando no a todo trabajador de cada empresa se
le paga el valor de lo que produce, los trabajadores considerados como grupo sí reciben el valor de
lo que producen. La compensación adicional que reciben los trabajadores de categoría baja de cada
empresa se compensa exactamente con el recorte en el pago de sus trabajadores de categoría alta.
¿Qué tan grandes son las diferencias salariales compensatorias de categoría dentro de la empre-
sa? La respuesta será diferente según las ocupaciones; en las que los compañeros de trabajo no se
relacionan mucho entre sí, la gente no está dispuesta a pagar mucho por un puesto de categoría alta.
Después de todo, las comparaciones que más importan son entre personas con mayor interacción. El
precio pagado por la categoría alta (y recibido por la categoría baja) será más alto en ocupaciones en
las que los compañeros de trabajo laboran estrechamente durante periodos prolongados.
El modelo de productividad marginal extendido pronostica que el salario aumentará menos
de un dólar por cada dólar extra de valor producido y que la diferencia entre la productividad y el
pago aumentará con la intensidad de la interacción entre compañeros de trabajo. Los pronósticos del
modelo original se comparan con los del modelo extendido en la figura 14.19.

FIGURA 14.19
Salario Modelo VPML original Esquemas salariales
y la intensidad de la
Modelo ampliado con interacción
interacción menos intensa mientras más intensa
sea la interacción de los
compañeros de trabajo,
mayor será la diferencia
salarial compensatoria
Modelo ampliado con por la categoría en la
interacción más intensa
distribución salarial interna.

45°
VPML

En la tabla 14.1 se presentan las estimaciones de las tasas a las cuales aumentan los ingresos con
la productividad para tres ocupaciones. Éstas se presentan en orden creciente de cercanía de interac-
ción. Los vendedores de bienes raíces, que tienen el contacto menos intenso, pagan las cantidades
más bajas por los puestos de categoría alta. En el otro extremo del espectro, los químicos de inves-
tigación, que trabajan en grupos muy unidos durante periodos prolongados, de hecho pagan canti-
dades muy grandes. En la muestra de estudio los químicos más productivos aportaron alrededor de
200 000 dólares en ingresos al año más que sus colegas menos productivos y sin embargo sólo reci-
bieron salarios ligeramente más altos.14 Los vendedores de autos no se asocian con tanta intensidad
como los químicos, pero a diferencia de los vendedores de bienes raíces, pasan sus horas de trabajo
en el mismo sitio. Como se pronosticó, el precio de los puestos de categoría alta para los vendedores
de autos cae entre los de las otras dos ocupaciones.
Las entradas en la tabla 14.1 postulan que, al menos para algunas ocupaciones, las diferencias
salariales compensatorias por la categoría interna son sustanciales. Si se toman en cuenta estas dife-
rencias, la estructura salarial igualitaria dentro de las empresas privadas parece perfectamente con-

14
Ibíd., cap. 4.

14_CHAPTER 14.indd 487 3/7/09 12:57:04 PM


488 CaPítUlo 14 mano de obra

TABLA 14.1
Pago versus productividad para tres ocupaciones

Ingreso extra por dólar adicional de producción las diferencias salariales


Ocupación Pronosticado por el compensatorias por la
Real modelo original categoría interna son más
grandes en ocupaciones
Ventas de bienes raíces $0.70 $1
en las que los compañeros
Ventas de automóviles $0.24 $1 de trabajo interactúan con
químicos investigadores <$0.09 $1 mayor intensidad.

gruente con la teoría de productividad marginal de los salarios. Aquí de nuevo parece que los críticos
consideraron de manera prematura que el modelo económico estaba cancelado.
El tema de la preocupación acerca de la categoría en la distribución salarial interna ayuda a acla-
rar el ejemplo que se consideró antes en el que los economistas que trabajan en escuelas comerciales
con frecuencia ganan 20% más que sus contrapartes que trabajan en colegios no especializados. Las
escuelas comerciales tienen que pagar a los demás miembros del cuerpo docente —contadores, mer-
cadólogos, analistas financieros y otros— salarios altos debido a las oportunidades de ingresos altos
de que disponen en estos campos dentro del sector privado. (Se dice que un decano dijo a un profesor
de letras que se quejó de los salarios altos de la facultad de derecho: “Si no te parece, ¿por qué no vas
y abres una empresa consultora de letras clásicas?”) Los economistas que aceptan trabajar en una
escuela de negocios se encuentran al final de la escala de pagos entre sus compañeros de trabajo. Las
primas salariales que reciben en comparación con los economistas de las universidades generales se
puede interpretar en parte como una diferencia compensatoria por ocupar un puesto de categoría
baja.

MERCADOS EN QUE EL GANADOR


SE QUEDA CON TODO
En esta sección se verá que las desigualdades en la categoría a veces causan que pequeñas diferencias
en la capacidad se traducen en grandes discrepancias en los valores de los productos marginales.15
La esencia de la idea se captura en el siguiente ejemplo simple. Imagine que Ford demandó a su
compañía, General Motors, por 1000 millones de dólares por violación de patente. En los alegatos
el caso está tan cerrado que lo más seguro es que se falle a favor de quien contrate al mejor abogado.
Suponga que Dershowitz y Jamail son los dos principales abogados del país y que, aunque los dos son
igual de talentosos en todo aspecto, Jamail es percibido como el mejor.
Como es natural, GM y Ford quieren contratar a Jamail, por lo que empiezan a hacer fuertes
pujas por sus servicios. ¿Cuánto tendrá que pagar el que gane? En un momento de reflexión, debe
quedar en claro que la respuesta debe ser 1 000 millones de dólares. Si Ford sólo ofrece 999 millones,
a GM le interesaría hacer una propuesta más alta porque la alternativa sería perder la demanda.
Pero Ford responderá aumentando su propia propuesta, ya que su alternativa también es perder
la demanda. A menos que Ford y GM puedan coludirse con éxito, el único resultado estable es que
Jamail reciba 1 000 millones de dólares. El valor de Dershowitz, aun cuando es ligeramente menos
talentoso, es exactamente cero, pues se supone que quien lo contrate perderá la demanda.
El ejemplo es una caricatura, aunque captura el sabor de lo que ocurre en una variedad de con-
textos del mercado laboral. Por ejemplo, considere la estructura de pagos del tenis profesional. Dada
la cantidad limitada de tiempo que la mayoría de la gente está dispuesta a pasar viendo los partidos

15
La exposición de esta sección se basa en R. Frank, “The Economics of Buying the Best”, Cornell University Department of Econo-
mics Working Paper, 1978; Sherwin Rosen, “The Economics of Superstars”, en American Economic Review, septiembre de 1981; y R.
Frank y P. Cook, The Winner-Take-All Society, Nueva York, The Free Press, 1995.

14_CHAPTER 14.indd 488 3/7/09 12:57:05 PM


reSUmen 489

de tenis en televisión, lo práctico es ver nada más a unos cuantos jugadores en acción durante un
año determinado. Y de tener la opción, la mayoría de los aficionados estarían dispuestos a pagar un
poco más para ver jugar a los tenistas mejor clasificados. El resultado es que la demanda de tenistas
clasificados entre los 10 primeros es cientos de veces más grande que la de los jugadores clasificados
alrededor del lugar 100. Sin embargo, la diferencia en la capacidad de juego entre las dos categorías
con frecuencia es muy pequeña. Si el jugador sembrado número 101 se enfrenta al 102, los aficiona-
dos verán un partido tan emocionante como el que protagonizarían los jugadores clasificados con
los números 1 y 2. Desde la perspectiva de la pareja de categoría inferior, el problema es que la
mayoría de los aficionados sólo tienen tiempo para ver un partido y, obviamente, preferirían ver a la
pareja mejor clasificada. El resultado es que los jugadores mejor sembrados ganan millones al año,
mientras los del segundo nivel apenas obtienen lo suficiente para pagar sus gastos del torneo.
Los efectos de superestrella se observan en casi todos los deportes profesionales, el mundo del
entretenimiento e incluso en los trabajos ordinarios empresariales. Tres tenores ganan el mayor vo-
lumen de regalías derivadas de los discos compactos que compran los amantes de la ópera. Las com-
pañías reguladas gastan sumas importantes haciendo licitaciones por los servicios de unos cuantos
testigos expertos. Un pequeño número de actores y actrices elige entre los mejores papeles.
Para que ocurra el efecto de superestrella, debe haber un efecto de “el ganador se queda con
todo” en alguna parte del proceso de producción. En el tenis, los mejores jugadores capturan a casi
todo el público que ve un partido. En el ejemplo de la demanda, el mejor abogado la gana. Para que
el efecto de superestrella sea importante, las apuestas en la competencia deben ser altas, como en
cada uno de estos ejemplos.
Se ha criticado la teoría de productividad marginal para la determinación de pagos con base
en que los trabajadores con capacidades casi idénticas muchas veces reciben como pago cantidades
totalmente diferentes. A primera vista, dichas observaciones en efecto parecen contradecir la teoría.
Pero si se presta atención, se ve que los críticos se apresuraron a condenar el modelo. Como ya se ha
visto, lo difícil es que pequeñas diferencias en la capacidad a veces se traducen en discrepancias muy
grandes en los valores de los productos marginales.
En el apéndice de este capítulo se estudian las diferencias salariales compensatorias para se-
guridad y el efecto de las preocupaciones acerca de los ingresos relacionados con esas opciones de
seguridad.

RESUMEN
• La meta del capítulo fue examinar las fuerzas económicas que que un aumento de la producción requiere que la vendan a
gobiernan los salarios y otras condiciones del empleo. La regla menor precio.
de contratación de la empresa perfectamente competitiva es, • El análisis de la oferta en el mercado laboral empezó consi-
en el corto plazo, seguir contratando hasta que el valor de lo derando la decisión de un trabajador de cuánto laborar por
que produce el último trabajador (el VPML) sea exactamente determinada tasa laboral. Cuanto más trabaja, más gana, pero
igual a la tasa salarial. A largo plazo, la curva de demanda de tiene menos tiempo para otras actividades. El resultado es un
mano de obra de la empresa es más elástica que a corto plazo problema estándar de decisión de consumo, como el que se es-
porque tiene la posibilidad adicional de sustituir capital con tudió en el capítulo 3. En el caso del consumidor, un aumento
mano de obra. en el precio de un producto viene acompañado de una reduc-
• Agrupar las curvas de demanda de las empresas en una curva ción de la cantidad demandada (salvo en el caso anómalo del
de demanda de mano de obra de la industria comprende más bien de Giffen). Por el contrario, en el contexto de la oferta de
que la simple suma horizontal de las curvas individuales. Hay trabajo, es frecuente que las personas trabajen menos horas
que hacer un ajuste porque al aumentar la producción de la cuando las tasas salariales aumentan. Para generar la curva
industria baja el precio del producto. de oferta del mercado se suman horizontalmente las curvas.
• La curva de demanda de mano de obra de un monopolista en Las curvas de oferta y demanda del mercado se intersecan y
el mercado del producto se traza comparando el salario no determinan el nivel salarial de la industria y el volumen total
con el valor de la producción que generan los trabajadores, de empleo de la industria.
sino con la cantidad en que la producción del trabajador mo- • La imagen convencional de los sindicatos es que acrecientan el
difica el ingreso total: PIML. A diferencia de la empresa perfec- poder de negociación de los trabajadores ante los patrones, lo
tamente competitiva, los monopolistas deben tener en cuenta que aumenta su parte del pastel económico fijo. Sin embargo,

14_CHAPTER 14.indd 489 3/7/09 12:57:06 PM


490 CaPítUlo 14 mano de obra

en investigaciones recientes se indica que los sindicatos pue- caría que las empresas dejan pasar oportunidades de mejorar
den realmente mejorar la productividad de los trabajadores, sus utilidades. Se vieron varios motivos, incluida la discrimina-
lo que además de aumentar su tajada del pastel económico, ción de instituciones distintas a las empresas, por los cuales los
incrementa también la parte de la gerencia. grupos afectados ganan salarios más bajos.
• Los defensores de las leyes de salario mínimo dicen que son • Una anomalía patente es el hecho de que gente con capaci-
necesarias para proteger a los trabajadores de la explotación dades que difieren muy ligeramente a veces gana salarios
por parte de patrones con demasiado poder en el mercado. Sin muy distintos. En este caso, la clave para resolver las contra-
embargo, si la legislación alcanza esta meta es una pregunta dicciones es observar que en muchos contextos el valor de lo
empírica difícil. Aunque en el caso de los adolescentes puede que alguien produce depende no sólo del nivel absoluto de
argumentarse a favor de eximirlos de esas leyes. sus habilidades, sino del resultado de compararlas con las de
• Los críticos dicen que muchas empresas pagan a los integran- otros. En la lucha libre, ser un poco más fuerte que el opo-
tes de ciertos grupos (en particular, negros y mujeres) menos nente equivale a ganar casi siempre. También en el mercado
de lo que les pagan a los hombres blancos por la misma pro- laboral, ser un poco mejor que la competencia significa ganar
ductividad. Esta acusación plantea una objeción fundamental mucho más.
al mismo núcleo de la teoría microeconómica, porque signifi-

pREgUNTAS DE REpASO
1. ¿Cuál es la diferencia entre la curva VPML del competidor per- 3. ¿Por qué podrían los patrones locales pagar a los trabajadores
fecto y la curva PIML del competidor imperfecto? el valor de lo que producen incluso si éstos no pueden o no
2. Si un monopolista comprara todas las empresas de una indus- quieren mudarse a otra área para aceptar un mejor trabajo?
tria anteriormente competitiva y adquiriera el derecho legal 4. ¿Por qué la teoría económica lleva a poner más atención en la
de excluir la entrada, ¿cómo influirá en la cantidad de mano de discriminación por parte de personas e instituciones en lugar de
obra empleada? la que ejercen los empleadores como una causa de las diferen-
cias salariales que rebasan las discrepancias de productividad?

pRObLEMAS
1. Con la información de la siguiente tabla complete el valor del producto marginal de mano de obra
para el precio P = 4. Encuentre la demanda óptima de mano de obra de la empresa perfectamente
competitiva para un salario de w = 4 dls/h.

L PM VPM
0 4
10 3
20 2
30 1
40 0

2. Con la información de la siguiente tabla grafique la restricción de presupuesto (descrita para w = 6


dls/h, donde h es horas de ocio al día y M es ingreso diario). Encuentre y grafique el nuevo límite
presupuestal (ingreso diario = M') para w = 12 dls./h. ¿Cómo son las pendientes de los dos límites
presupuestales comparadas entre sí y por qué?

h M M'
0 144
6 108
12 72
18 36
24 0

14_CHAPTER 14.indd 490 3/7/09 12:57:06 PM


ProblemaS 491

3. Con la información de la siguiente tabla encuentre la demanda de mano de obra óptima del mono-
psonista y el salario pagado.

L CFP CFT CFM VPM


0 0 0 0 16
10 2 20 4 12
20 4 80 8 8
30 6 180 12 4

4. Una empresa perfectamente competitiva tiene MPL = 22 – L. Encuentre y grafique su valor del pro-
ducto marginal de la mano de obra a un precio de producto de P = 5. Encuentre la cantidad óptima
demandada de mano de obra a un salario de w = 10 dls/h.
5. En su trabajo actual, Smith puede trabajar cada día las horas que desee, y se le pagará 1 dl./h durante
las primeras 8 horas que trabaje, 2.50 dls/h por cada hora adicional después de las primeras ocho.
Ante este esquema de pagos, Smith elige trabajar 12 h/día. Si a Smith le ofrecen un trabajo que paga
1.50 dls/h por las horas que elija trabajar, ¿lo aceptará? Explique.
6. Considere los dos programas siguientes contra la pobreza: 1) este año se pagarán 10 dls/día a cada
persona que el año anterior haya sido clasificada como pobre; y 2) cada persona clasificada como
pobre recibirá un beneficio igual a 20% del ingreso salarial que gane al día este año.
a) Suponga que esas personas pobres tienen la opción de trabajar a 4 dls/h, muestre cómo cada pro-
grama afectaría la restricción de presupuesto diaria de un trabajador pobre representativo durante
este año.
b) ¿Qué programa es más probable que reduzca el número de horas trabajadas?
7. La curva de la demanda de mano de obra de un monopsonista se obtiene con w = 12 – 2L, donde w es
la tasa salarial por hora y L es el número de personas contratadas por hora.
a) Si la curva de oferta del monopsonista (CFP) se obtiene con w = 2L, que genera una curva de
costo factor marginal de MFC = 4L, ¿cuántas unidades de mano de obra empleará y qué salario
pagará?
b) ¿En qué serían diferentes sus respuestas al inciso a) si el monopsonista recibiera un proyecto de ley
de salario mínimo que le requiriera pagar por lo menos 7 dls/h?
c) ¿En qué serían diferentes sus respuestas a los incisos a) y b) si el patrón en cuestión no fuera un
monopsonista sino un competidor perfecto en el mercado laboral?
8. Acme es la única proveedora de sistemas de seguridad en el mercado de esos productos y el único
empleador de cerrajeros en el mercado laboral. La curva de demanda de los sistemas de seguridad
está definida por P = 100 – Q, donde Q es el número de sistemas instalados por semana. La función de
producción a corto plazo para los sistemas de seguridad se obtiene con Q = 4L, donde L es el número
de cerrajeros de tiempo completo empleados a la semana. La curva de oferta de cerrajeros se obtiene
con W = 40 + 2L, donde W es el salario semanal de cada cerrajero. ¿Cuántos cerrajeros contratará
Acme y cuál será el salario?
9. La curva de la demanda de mano de obra para un monopsonista se obtiene con W = 35 – 6L; la curva
de oferta (CFP) para este monopsonista es W = 3 + L, con la ecuación correspondiente CFM = 3 +
2L, donde W representa la tasa salarial por hora y L es el número de personas contratadas por hora.
a) Encuentre la cantidad óptima de mano de obra y la tasa salarial para este monopsonista maximi-
zador de utilidades.
b) Suponga que una ley de salarios mínimos impuso un salario mínimo de 17 dls/h. ¿Cómo se ve
afectada la cantidad de mano de obra demandada por esta empresa?
10. Un monopolista puede contratar cualquier cantidad de mano de obra por 10 dls/h. Si su producto
marginal de mano de obra actualmente es de 2 y su precio de producto actual es de 5 dls/unidad,
¿debe aumentar o disminuir la cantidad de mano de obra que contrata?
11. Ajax Coal Company es el único patrón en su área. Su único insumo variable es la mano de obra, que
tiene un producto marginal constante igual a 5. Como es el único patrón en el área, la empresa tiene
una curva de oferta de mano de obra definida por W = 10 + L, donde W es la tasa salarial y L es el
número de personas-hora empleadas. Esta curva de oferta produce la curva de costo factorial

14_CHAPTER 14.indd 491 3/7/09 12:57:07 PM


492 CaPítUlo 14 mano de obra

marginal CFM = 10 + 2L. Suponga que la empresa puede vender lo que desee a un precio cons-
tante de 8.
a) ¿Cuánta mano de obra emplea la empresa, cuánto produce y cuál es el salario?
b) Suponga que ahora la empresa vende un tipo especial de carbón de modo que ahora tiene una
curva de demanda con pendiente negativa para su producción. En particular, suponga que Ajax
tiene una curva de demanda definida por P = 102 – 1.96Q. ¿Cuánta mano de obra emplea la em-
presa, cuánto produce, qué precio fija para la producción y cuál es el salario?
c) Suponga que Ajax continúa con una curva de demanda de P = 102 – 1.96Q, pero ahora suponga
además que Ajax tiene cinco trabajadores contratados para producir carbón con un salario de
15. Si Ajax tiene la opción de contratar más trabajadores con un salario más alto sin aumentar el
sueldo de los cinco trabajadores contratados, ¿aumentará su fuerza laboral? Explique.
12. Suponga que llegan las fechas de vacaciones con intervalos de una semana y que la disponibilidad
total para pagar el tiempo total de vacaciones por parte de trabajadores jóvenes y mayores en una
industria competitiva se presenta en la tabla siguiente:

Disponibilidad total para pagar


Tiempo total de Trabajadores Trabajadores
vacaciones, semanas jóvenes mayores
1 300 500
2 475 800
3 600 1050
4 700 1250
5 750 1400

Suponga un VPM = 150/sem para trabajadores jóvenes, 175/sem para trabajadores mayores y que
las empresas existentes dan a todos sus trabajadores, jóvenes y mayores, 5 semanas al año de vacacio-
nes. ¿Estas empresas pueden maximizar sus ganancias? De ser así, explique. De lo contrario, indique
los cambios que debe hacer y cuál será la utilidad extra resultante.
13. Los miembros de dos grupos, los azules y los verdes, tienen valores de productividad que varían de 5
a 15 dls/h. La productividad promedio de los azules es de 6 dls/h y el promedio correspondiente de
los verdes es de 12 dls/h. Se sabe que una prueba de productividad sin costo tiene la capacidad de dar
el valor de productividad correcto con una probabilidad de 1/3 y un valor de productividad aleatorio
obtenido de la distribución del grupo relevante con una probabilidad de 2/3.
a) Si los mercados de mano de obra son competitivos, ¿qué pago recibirá un azul con un valor de
prueba de 9?
b) ¿Qué pago recibirá un verde con el mismo valor de prueba?
c) ¿Está bien decir que la discriminación estadística es la responsable de que los verdes, como grupo,
reciban un pago superior al de los azules?
14. Una empresa tiene que llevar a cabo una tarea que tiene que ver con la oportunidad de flojear con po-
cas probabilidades de ser descubierto. Si puede contratar a un empleado que no pierda el tiempo para
esta responsabilidad, ganará mucho dinero. Su estrategia para encontrar a una persona que no flojee
es pagar primero un salario muy bajo, luego aumentarlo gradualmente cada año para que, cuando el
trabajador lleve 10 años en la empresa, gane más que en cualquier otra parte. El valor presente de las
primas salariales en los últimos años es mayor que el valor presente del déficit en los primeros años.
a) Explique cómo ayuda esta estrategia para atraer a un empleado que no flojee. ¿Funcionaría la
misma estrategia si la probabilidad de descubrir a quien flojea fuera cero?
b) Explique por qué la capacidad de la empresa para poner en práctica esta estrategia podría depen-
der de cierto modo de su propia reputación en el mercado laboral.
15. Considere una economía de dos sectores que emplea un total de 80 unidades de un solo insumo, la
mano de obra. N1 de estas unidades se asignan al sector 1, donde el salario es de 100 para los cinco
principales trabajadores en ese sector y cero para los demás. (El salario de los principales trabajadores

14_CHAPTER 14.indd 492 3/7/09 12:57:07 PM


reSPUeStaS a loS ejerCiCioS del CaPítUlo 493

y el número que recibe ese salario no varían ante los cambios de N1.) Las unidades N2 = 80 – N1
restantes de mano de obra están en el sector 2, donde cada trabajador recibe un salario de 10. Todos
los trabajadores del sector 1 tienen la misma probabilidad de estar entre los cinco principales trabaja-
dores, 5/N1, y todos los trabajadores son indiferentes al riesgo.
a) ¿Cuántos trabajadores estarán en el sector 1?
b) ¿Cuál será el valor del PIB para la economía?
c) ¿En qué serían diferentes sus respuestas si hubiera un impuesto de 50% sobre las ganancias de los
trabajadores en el sector 1?
*16. Una empresa produce de acuerdo con la función de producción Q = K1/2L1/2. Si vende su producción
en un mercado de competencia perfecta a un precio de 10 y si K está fijo en 4 unidades, ¿cuál es la
curva de demanda de mano de obra a corto plazo de la empresa?
*17. ¿En qué sería diferente su respuesta al problema anterior si el empleado en cuestión vendiera su
producto de acuerdo con el esquema de demanda P = 20 – Q?

* Esta pregunta requiere la definición del cálculo del producto marginal de la mano de obra: PML = ∂Q/∂L.

RESpUESTAS A LOS EjERCiCiOS DEL CApíTULO


14.1. Cuando el precio del producto aumenta a 3 dólares, la curva VPML es como se muestra en la sección
b) del siguiente diagrama. La nueva cantidad de mano de obra demandada a w = 12 dólares es 120
unidades.

Producto marginal de la mano


de obra (unidades de producto/ Valor del producto marginal
unidad de mano de obra) ($/unidad de mano de obra)

8 24 Cantidad óptima de mano


de obra cuando w = 12
6 18

4 w = 12

PML
VPML = P × PML

L (persona-h/día) L (persona-h/día)
40 80 120 40 80 120
a) b)

El valor del producto marginal es


VPM L = P ( PM L ) = 3(10 − 201 L ) = 30 − 203 L.
La cantidad óptima de mano de obra se encuentra al solucionar
w = VPM L ⇒ 12 = 30 − 203 L ⇒ 18 = 203 L ⇒ L * = 120.
14.2. Con w = 20 dls/h, la restricción de presupuesto ingreso-ocio es

M = w(24 – h) = 20(24 – h) = 480 – 20h.

14_CHAPTER 14.indd 493 3/7/09 12:57:10 PM


494 CaPítUlo 14 mano de obra

Con un ocio de h = 14 h/día, el ingreso diario es


M = 20(24 – h) = 20(24 – 14) = 20(10) = 200 dls.
Ingreso (dls/día)
480

200
M = 480 – 20h

Ocio (h/día)
0 14 24

14.3. w

40

20

LS
0 3 6

14.4. El conductor puede llegar a un objetivo de ingreso semanal en menos horas totales por semana si
trabaja más horas durante los días lluviosos y menos horas en días que no llueve.
14.5. La restricción de presupuesto permanece en M = 480 – 20h. Como el individuo está dispuesto a
sacrificar un día de ocio por 10 dólares de ingreso, las curvas de indiferencia son líneas rectas de
la forma general M = a – 10h proporcionadas en la sugerencia. La curva de indiferencia más alta
que comparte un punto con la restricción ingreso-ocio es M = 480 – 10h, con una demanda de ocio
óptima h = 0. Esta forma de preferencias muestra los efectos de sustitución extrema (no se consume
ocio en ningún w > 10 y se consume todo el ocio en cualquier w < 10).
Ingreso (dls/día)
480

M = 480 – 10h

M = 480 – 20h

0 Ocio (h/día)
24

14.6. En su trabajo actual el ingreso máximo de Maynard por trabajar 24 horas es la suma de 8 horas a 5
dls/h y las 16 horas restantes a 20 dls/h.
8(5) + 16(20) = 40 + 320 = 360 dólares/día.

14_CHAPTER 14.indd 494 3/7/09 12:57:13 PM


reSPUeStaS a loS ejerCiCioS del CaPítUlo 495

Cada hora de ocio requiere un sacrificio de 20 dólares hasta h = 16 h, pero sólo de 5 dólares más allá
de las 16 hrs. El consumo de h = 16 h de ocio y trabajar 24 – h = 24 – 16 = 8 h resulta en 8(5) = 40 de
ingreso (en el quiebre de la restricción de presupuesto). Por lo tanto, la restricción de presupuesto
original de Maynard es
 360 − 20h; 0 ≤ h ≤ 16
M1 = 
 40 − 5h; 16 ≤ h ≤ 24
La restricción de presupuesto tiene dos partes que reflejan los dos salarios (normal y tiempo extra).
Si Maynard trabaja 12 horas, entonces disfruta h1 = 12 h de ocio al día y, por lo tanto, con la restric-
ción de presupuesto original gana un ingreso de
M1 = 360 – 20h = 360 – 20(12) = 360 – 240 = 120 dls/día.
En su potencial nuevo trabajo el ingreso máximo de Maynard por trabajar 24 horas es 24(10) =
240 dólares. Una hora de ocio requiere sacrificar un ingreso de 10 dólares hasta 24 horas. La nueva
restricción de presupuesto de Maynard es
M2 = 240 – 10h.
En este caso la restricción de presupuesto es una línea recta simple. La elección de la oferta de mano
de obra óptima original de Maynard podría ser posible con la nueva restricción de presupuesto: es
decir, podría ganar el mismo ingreso con la misma cantidad de tiempo de ocio.
M2 = 240 – 10h = 240 – 10(12) = 240 – 120 = 120 dólares/día.
Así, Maynard no puede estar peor con la nueva restricción de presupuesto. Sin embargo, tendrá un
costo de oportunidad de tiempo de ocio de w = 10 con la nueva restricción de presupuesto en lugar
de w = 20 con la restricción de presupuesto anterior. Por lo tanto, debe ajustar de manera óptima su
oferta de mano de obra más hacia el ocio. Maynard estará más contento con su nueva elección de
oferta de mano de obra óptima: llega a una curva de indiferencia más alta (I2) entre ingreso y ocio,
así que aceptará el nuevo empleo.

Ingreso (dls/día)
360 I1 I
2
B1

240 B2

120

40
0 Ocio (h/día)
12 16 24
14.7. La restricción de presupuesto original es
M0 = w(24 – h) = 5(24 – h) = 120 – 5h.
El primer programa produce una restricción de presupuesto
M1 = S + w(24 – h) = 24 + 5(24 – h) = 144 – 5h.
El segundo programa produce una restricción de presupuesto
M2 = (1 + s)w(24 – h) = (1 + 0.4) 5(24 – h)
= 7(24 – h) = 168 – 7h.
Es más probable que el primer programa reduzca las horas trabajadas porque aumenta el ingreso
aunque deja el costo de oportunidad del ocio sin cambios: si se supone que el ocio es un bien normal,

14_CHAPTER 14.indd 495 3/7/09 12:57:16 PM


496 CaPítUlo 14 mano de obra

el ingreso más alto conduce a un mayor consumo de ocio. Por el contrario, el segundo programa in-
crementa el costo de oportunidad del ocio: para niveles de ingresos bajos, un aumento en el salario
por lo general incrementa la oferta de mano de obra en tanto el efecto de sustitución sobrepase al
efecto de ingreso. Por eso es probable que el pobre trabaje menos con el primer programa y más con
el segundo.
Ingreso (dls/día)
168
B2
144
120

84

B1
B0

0 Ocio (h/día)
12 24
14.8. Salario, MFC (dls/unidad de mano de obra)

MFC = 2 + 4L
12
CFP = 2 + 2L
CFM = 10

w=6
Demanda de
mano de obra

L
2 12

14.9. Cuando el salario mínimo es 8, la curva CFM del monopsonista es el lugar geométrico con la dis-
continuidad en L = 3. La curva de demanda de mano de obra pasa por esta discontinuidad, lo cual
significa que el monopsonista contrata 3 unidades de mano de obra a un salario de 8. Cuando el
salario mínimo es 10, la empresa paga w = 10 y contrata 2 unidades de mano de obra, la misma
cantidad que habría contratado sin tener un salario mínimo.

$/L
CFM

12 CFP = 2 + 2L
10

2 Demanda de mano de obra

L
2 3 12

14.10. VPM(12) = ( 12 )(12dls/h ) + ( 12 )( 20 dls/h ) = 16 dls/h.

14_CHAPTER 14.indd 496 3/7/09 12:57:21 PM


A p é N D i C E

14
lA EcONOMíA DE lA
sEguRiDAD lABORAl

DIFERENCIAS SALARIALES COMPENSATORIAS:


EL CASO DE LA SEGURIDAD
Quienes no saben de economía en el Congreso y otros lados hablan como si todas las situaciones
tuvieran que ser tan seguras como fuera posible sin importar los costos. A estas personas les gusta
decir: “La seguridad de los seres humanos no es cuestión de economía.” Pero la noción de que los
costos no tienen que ver en las decisiones de seguridad no resiste ni el más somero escrutinio. Para
ver qué concepto tan curioso es, basta examinar algunas de las conclusiones que se derivan de él.
Por ejemplo, considere la cuestión de la seguridad en los autos. En todo momento hay una
pequeña probabilidad de que fallen los frenos de un auto cualquiera. Cuando ocurre así, las conse-
cuencias son terribles. La probabilidad de matarse en un accidente de tránsito se reduciría si la gente
hiciera que un mecánico calificado revisara exhaustivamente los frenos todos los días. Pero nadie
sueña con que le prueben los frenos tan seguido, porque los costos serían demasiado elevados en
relación con los probables beneficios. Casi todos llevan a revisar su auto una o dos veces por año y
nadie pretendería seriamente que haya una línea de acción más razonable.
La gente valora la seguridad. Para tenerla, deben gastar recursos reales. Como todas las pregun-
tas que tienen que ver con recursos, la mejor manera de zanjar la cuestión de cuánta seguridad debe
haber es comparar los costos y los beneficios correspondientes.
Cada persona verá de distinta manera los costos de la seguridad y los beneficios. Por ejemplo,
quien se sienta aterrorizado ante la perspectiva de ser alcanzado por un rayo se sentirá más tranquilo
497

14_CHAPTER 14.indd 497 3/7/09 12:57:21 PM


498 CaPítUlo 14 aPéndiCe 14: la eConomía de la SegUridad laboral

si instala un pararrayos que quien piensa que las tormentas son emocionantes. El grado óptimo de
esta forma concreta de seguridad será mayor para el primer individuo que para el segundo.
Los temas son fundamentalmente los mismos cuando se trata de tomar las decisiones relativas
a la seguridad en el trabajo. Muchas actividades de producción entrañan riesgos de salud y seguridad
que se reducen con aparatos y procedimientos. Las modificaciones implican costos adicionales y, en
la mayor parte de los casos, sería imposible eliminar por completo los riesgos. Que deban hacerse
estas modificaciones y en qué medida depende del resultado de comparar su valor con su costo.
Para hacer más concreta la exposición, tome el caso de una mina de carbón en la que se pondera
si se instalan filtros para eliminar del aire los polvos del mineral. El costo de instalar y operar estos
filtros es de 50 dólares/semana por minero. Ya instalados los filtros, la esperanza de vida de los mine-
ros es igual que la de la gente que ocupa puestos de oficina, pero sin los filtros, es de 10 años menos.
La pregunta sobre si deben instalarse los filtros se reduce a la pregunta sobre si los mineros valoran
la mayor esperanza de vida más que los 50 dólares semanales. En tal caso, hay que instalar los filtros;
en caso contrario, no hay que instalarlos. Por ejemplo, suponga que los mineros valoran los 10 años
adicionales en sólo 40 dólares por semana. Un concesionario que instale los filtros tendría que pagar
a sus trabajadores 50 dólares menos para compensar el aumento de sus costos (si se sospecha que
los compradores de carbón no estarán dispuestos a pagar más por el carbón extraído de minas con
filtros). Pero suponga que los mineros prefieren tener esos 50 dólares que la protección que brindan
los filtros.
Desde luego, y como cuestión práctica, algunos mineros opinan que los filtros son una pro-
posición atractiva, mientras que otros piensan que no lo es. Por ejemplo, suponga que 30% de los
mineros valoran los filtros en 60 dls./semana, mientras que el 70% restante los valora en 40. Así, la
tendencia del mercado será que 30% de las minas instalen los filtros y 70% no. Las que los instalen
pagarán un salario que es 50 dólares semanales menor que en las otras. Los mineros que valúan los
filtros en 60 dólares por semana escogerán trabajar en minas con filtros, mientras que el resto de ellos
gravitará hacia las minas sin filtros.
Más en general, la elección involucra más que dos alternativas. La línea marcada B en la figura
A14.1 muestra un continuo de apareamientos técnicamente viables entre salario y seguridad. El eje
horizontal mide la seguridad como la probabilidad anual de sobrevivir en el trabajo (un empleo per-
fectamente seguro sería aquel en que la probabilidad es de 1.0). El eje vertical mide la tasa salarial
por hora. B tiene una pendiente negativa porque hay que gastar recursos para mejorar la seguridad.
La curvatura descendente de B es resultado del hecho de que se instalan primero los dispositivos de
seguridad eficaces más baratos y hasta después se pasa a los más caros. La pendiente casi vertical de
B a medida que la probabilidad de sobrevivir se aproxima a 1.0 indica que no importa cuánto se gaste
en seguridad, no es posible garantizar absolutamente la supervivencia.

FIGURA A14.1
La combinación óptima
de salario y seguridad Salario
quienes corren riesgos
(personas con tasas
marginales de sustitución Arriesgado
bajas de salarios por B
probabilidad de sobrevivir) A
wA
prefieren trabajos más Precavido
arriesgados con salarios
más altos (punto A). los C
wC
precavidos prefieren
trabajos más seguros con
menos salario (punto C).

Probabilidad anual de supervivencia


rA rC 1.0

14_CHAPTER 14.indd 498 3/7/09 12:57:22 PM


diferenCiaS SalarialeS ComPenSatoriaS: el CaSo de la SegUridad 499

Dado este conjunto de empleos técnicamente viables, el puesto adecuado para un trabajador
depende de lo que piense sobre el equilibrio entre riesgo y bienes materiales. Alguien que siente un
fuerte desagrado por el riesgo tendrá curvas de indiferencia inclinadas, que reflejen su disposición a
aceptar una rebaja salarial grande a cambio de fortalecer sus probabilidades de sobrevivir. La elección
óptima de esta persona está representada por la tangencia en C en la figura A.14.1.
Por contraste, quien esté menos preocupado por los riesgos tendrá curvas de indiferencia con
pendiente menos pronunciada, como reflejo de su menor disposición a renunciar al ingreso a cam-
bio de seguridad. La elección óptima del puesto para esta persona está representada por la tangencia
en A.
La teoría de las diferencias salariales compensatorias pronostica que en igualdad de circunstan-
cias, cuanto más peligroso sea un trabajo, mayor tendrá que ser la tasa salarial. Robert S. Smith,
experto en economía del trabajo de Cornell, revisó ocho estudios empíricos que examinaron la rela-
ción entre tasas salariales y riesgo en los trabajos.1 En todos los estudios, que se basaron en conjuntos
de datos diferentes, se encontró una asociación positiva entre las tasas salariales y la probabilidad de
matarse en el trabajo.
Las estimaciones del tamaño de las diferencias salariales compensatorias variaron de 20 dls./año
a 300 dls./año por cada aumento de 1 en 10 000 en la probabilidad anual de morir en el trabajo. Para
poner esta probabilidad en perspectiva, 1 de cada 10 000 acereros muere en el trabajo cada año, en
comparación con 10 de cada 10 000 leñadores. Para fines del ejemplo, suponga que los trabajadores
valoran una reducción de 1 en 10 000 en la probabilidad de morir en 100 dólares por año en prome-
dio. Esto implica que la diferencia salarial compensatoria de la seguridad debe ser 900 dólares al año
mayor para los leñadores que para los acereros, porque la probabilidad de morir de los primeros es
9/10 000 más alta. Desde luego, la diferencia salarial general entre dos ocupaciones cualesquiera es
el resultado de muchas diferencias además del riesgo de muerte o lesión, así que la diferencia salarial
entre acereros y leñadores puede ser mayor o menor que 900 dls./año. Según la teoría, si la silvicul-
tura se hiciera tan segura como el trabajo en una fundidora, el salario pagado a los leñadores bajaría
por 900 dls./año.
Una vez fui a una conferencia en la que un economista abordó el concepto de las diferencias
salariales compensatorias en materia de seguridad. Un sociólogo del público reaccionó con enojo a la
sola mención de la idea. Anunció triunfante que “la teoría es completamente inválida porque, como
todos saben, los trabajos más peligrosos y desagradables son desempeñados siempre por los trabaja-
dores con menos paga”. En efecto, los trabajos más peligrosos los hacen trabajadores de bajos ingre-
sos, pero esta observación no sirve de nada para invalidar la teoría. La teoría establece, de nuevo, que
en igualdad de circunstancias los salarios serán mayores en los trabajos menos agradables. Si se toma
una muestra grande de trabajadores, hay otros factores que de ninguna manera son iguales entre ellos.
En particular, la escolaridad, inteligencia, experiencia y energía varían enormemente, y con éstas la
capacidad de ganar más por la venta de mano de obra en el mercado. Con la premisa verosímil de
que la seguridad es un bien normal, se concluye que los trabajadores con mayor poder de compra
escogen trabajos en los que las tasas salariales y la seguridad son mayores. En el mismo tenor, los
trabajadores con la menor productividad escogerán trabajos con los menores niveles de seguridad,
con salarios que también son bajos. Pero al optar por esos trabajos, sus salarios son mayores de lo que
habrían sido si hubieran escogido trabajos más seguros.
El argumento anterior se traduce fácilmente en el marco gráfico para representar las opciones
óptimas de salario y seguridad. La línea marcada B en la figura A.14.2 representa el conjunto de
combinaciones técnicamente viables entre salario y seguridad correspondientes a un trabajador muy
productivo. La línea inferior, B2, es el conjunto correspondiente para un trabajador con mucha menor
productividad. B1 y B2 cumplen el papel de restricciones de presupuesto en el análisis y el trabajador
altamente productivo está muy lejos del origen. Si los dos trabajadores tiene curvas de indiferencia
idénticas y la seguridad es un bien normal, se deduce que el trabajo óptimo para el trabajador más

1
Robert S. Smith, “Compensating Wage Differentials and Public Policy. A Review”, en Industrial and Labor Relations Review, 32, abril
de 1979, pp. 339-352.

14_CHAPTER 14.indd 499 3/7/09 12:57:23 PM


500 CaPítUlo 14 aPéndiCe 14: la eConomía de la SegUridad laboral

FIGURA A14.2
Efecto de la
productividad en la
elección de seguridad
óptima Salario
la restricción de
presupuesto de los
trabajadores muy
productivos (B1) presenta B1
un conjunto de opciones
más favorable que la de A
los trabajadores menos wA
productivos (B2). Si la B2 C
seguridad es un bien wC
normal, más trabajadores
productivos escogerán
trabajos con sueldo mayor
y más seguridad (A) que
los puestos elegidos por Probabilidad anual de supervivencia
rC rA 1.0
los menos productivos
(C). Pero para cualquier
nivel de productividad,
de todos modos hay
una compensación entre
salarios y seguridad.

productivo (A) tendrá un salario y una probabilidad de sobrevivir más altos que el trabajo óptimo del
trabajador menos productivo (C).
Muchos economistas argumentan que la existencia de diferencias salariales compensatorias
hacen innecesarias las normas de seguridad en los mercados laborales competitivos. Para ilustrar
el argumento, revise el ejemplo de la silvicultura y la fundición de acero. Suponga que hay pasos
(comprar equipo más seguro, aplicar reglas de seguridad más estrictas, etc.) que podrían darse para
reducir la probabilidad anual de muerte en la silvicultura a sólo 1 en 10 000, la misma tasa que en
las fundidoras. Si se supone que los trabajadores están dispuestos a pagar 100 dólares al año por
cada reducción de 1 en 10 000 en la probabilidad de muerte, esto significa que los leñadores estarían
dispuestos a renunciar hasta a 900 dólares al año en salarios para que se dieran estos pasos adiciona-
les en seguridad. Las compañías aserradoras, que maximizan utilidades, tendrían un incentivo para
implantar dichos pasos si su costo fuera menor que 900 dls./año por trabajador. Pero si el costo de
las acciones fuera de más de 900 dólares anuales por trabajador, ni la compañía ni los trabajadores se
dejarían convencer. Cierto, las acciones aumentarían la seguridad de la aserradora, pero a un costo
que los propios leñadores considerarían demasiado alto para que valiera la pena.
La Ley de Seguridad y Salud Ocupacional (Occupational Safety and Health Act, OSHA), apro-
bada por el Congreso estadounidense en 1970, fija normas estrictas de seguridad en el trabajo. Su
propósito declarado es “garantizar el mayor grado de protección de la salud y la seguridad para el
empleado”. Muchos economistas critican la OSHA sobre la base de que obliga a muchos trabajado-
res a comprar más seguridad que la que adquirirían por su cuenta. Por ejemplo, considere al traba-
jador que habría escogido el puesto de A en la figura A.14.3 si hubiera estado atenido a sus propios
medios. Suponga ahora que la OSHA exige que todos los trabajos tengan una probabilidad anual de
supervivencia de por lo menos rc. El efecto de la exigencia será obligar al trabajador a abandonar el
trabajo de A y tomar uno de C. Pero el trabajo de C, aunque es más seguro, pone al trabajador en
una curva de indiferencia más baja. En C, la cantidad que está dispuesto a gastar por un incremento
de la seguridad es menor que el costo de proporcionar ese incremento. Si se acepta la valuación
de las alternativas por parte del propio trabajador, los requisitos de seguridad lo dejaron en peor
situación.

14_CHAPTER 14.indd 500 3/7/09 12:57:25 PM


deCiSioneS de SegUridad e ingreSo relatiVo 501

FIGURA A14.3
Requisito de seguridad
que reduce la utilidad
Salario dejado a sus propios
medios, el trabajador
del diagrama escogería
el trabajo en A. Sin
B embargo, cuando hay una
probabilidad mínima de
A
wA supervivencia de rC, debe
escoger el trabajo en C, que
C lo pone en una curva de
wC indiferencia más baja que el
trabajo en A.
Probabilidad anual de supervivencia
rA rC 1.0

Los defensores de las normas de seguridad y salud responden que la crítica que hacen los eco-
nomistas a la OSHA estaría justificada en un mercado laboral muy competitivo y perfectamente
informado. Pero estas condiciones nunca o casi nunca se cumplen en la práctica. Con respecto a los
temas de salud en general y en particular de los compuestos tóxicos, muchas veces los trabajadores
están completamente carentes de información y no pueden tomar decisiones inteligentes sobre los
paquetes de compensación. Desde este punto de vista, los empleados facultan a las autoridades gu-
bernamentales para que actúen como consejeros expertos en su nombre, para que los protejan de los
riesgos que les parezcan demasiado complejos para que ellos mismos los evalúen inteligentemente.
Esta respuesta de los defensores de las normas de seguridad y salud parece suficiente para justifi-
car la intervención en muchos casos; pero no explica muchos otros. Reconsidere al minero que exca-
va carbón y que puede optar entre una mina polvosa y una limpia. Prácticamente todos los mineros
están conscientes de que la probable consecuencia de trabajar en una mina polvosa es que al final van
a contraer neumoconiosis (la enfermedad de los pulmones negros), una dolencia grave, debilitante y
a veces mortal. Como es probable que todos los mineros tengan algún familiar cercano afectado con
la enfermedad, la ignorancia de las consecuencias de trabajar en una mina polvosa parece una razón
fundamental no plausible para pedir filtros.
Si la crítica que hacen los economistas de las normas de salud y seguridad se dirige a los mineros
de las carboneras, es de esperar que los trabajadores se opongan a dichas normas. Pero estas medidas
convocan un apoyo extendido y entusiasta de los trabajadores afectados por ellas, lo que plantea la
pregunta desconcertante sobre por qué los dueños de las minas, que buscan maximizar las utilidades,
no instalan por su cuenta los filtros para atraer a los mineros que insisten en que las minas los tengan
(y que están dispuestos a aceptar las reducciones salariales suficientes para pagarlos).
Otra razón fundamental para regular la seguridad y otros términos específicos del contrato la-
boral es que los mercados, de hecho, no son competitivos en la práctica. Los defensores de este punto
de vista afirman que los trabajadores carecen de la libertad y el movimiento necesarios para tener
una gama de opciones y por eso son vulnerables a la explotación de parte de sus patrones.

DECISIONES DE SEGURIDAD E INGRESO RELATIVO


Otra posible justificación de las normas de seguridad es que las decisiones que toman los trabajado-
res como individuos no son las mismas que toman colectivamente. En particular, suponga que a los
mineros no sólo les interesa la seguridad y el monto absoluto del ingreso que ganan, sino también
cómo son sus ingresos comparados con los de otros miembros de su comunidad. Quizá tengan la
meta de mandar a sus hijos a una buena escuela y se dan cuenta de que comprar una casa en uno de
los mejores distritos requiere tener mayor poder de compra que la mayoría de las demás personas.

14_CHAPTER 14.indd 501 3/7/09 12:57:27 PM


502 CaPítUlo 14 aPéndiCe 14: la eConomía de la SegUridad laboral

Por simplicidad, considere una comunidad consistente en dos trabajadores idénticos, Jones y
Smith; cada uno de ellos tiene que escoger entre una mina limpia que paga 200 dls./semana y una
polvosa que paga 250 dls./semana. La diferencia de 50 dólares refleja el costo por trabajador de filtrar
el polvo. Como el ingreso relativo tiene importancia, el atractivo de cada elección depende de lo que
escoja cada persona. Suponga que las cuatro combinaciones posibles se ordenan en la tabla A.14.1.

TABLA A14.1
Decisiones de seguridad laboral cuando el salario importa

Smith Sin importar lo que hace


la otra persona, cada una
Mina insegura 250
está mejor si escoge la mina
Segura 200 dls/sem. dls/sem.
insegura. Cuando los dos
Mina segura Segunda mejor para Peor para jones mejor escogen la mina insegura,
200 dls/sem. los dos para Smith tienen un peor resultado
Jones
Mina insegura mejor para jones peor tercera mejor para que si los dos escogen la más
250 dls/sem. para Smith los dos segura.

En primer lugar, observe que los dos trabajos son equivalentes cuando los trabajadores escogen
el mismo tipo, es decir, cuando la combinación de opciones da por resultado el mismo nivel de ingre-
so para los dos trabajadores. La celda superior izquierda de la tabla corresponde a que los dos escojan
la mina segura y esta alternativa tiene una calificación elevada (la segunda mejor para los dos) que la
celda inferior de la derecha, que corresponde a escoger la mina insegura (tercera mejor para los dos).
Esto indica que, dejando a un lado las preocupaciones por el ingreso relativo, la seguridad adicional
vale más que los 50 dólares semanales de cada trabajador.
Pero suponga que Smith escoge un trabajo en la mina segura. Entonces, el mejor resultado para
Jones será trabajar en la mina insegura. Así, renuncia a su seguridad que vale más de 50 dólares por
semana a cambio de una recompensa de apenas 50 dólares semanales. Entre tanto, gana más salario
que Smith. De acuerdo con la jerarquía de las preferencias de Jones, esto compensa con creces su
pérdida de seguridad, porque ahora puede mandar a sus hijos a una escuela mejor.
Como alternativa, suponga que Smith hubiera escogido la mina insegura. De nuevo a Jones
le va mejor si escoge la mina insegura y se queda con la tercera mejor alternativa, mientras que la
mina segura lo habría dejado en la peor combinación. En síntesis, haga lo que haga Smith, el mejor
resultado para Jones es escoger la mina insegura.
Los incentivos de Smith son exactamente los mismos. También hace lo mejor si escoge la mina
insegura independientemente de lo que haga Jones. El resultado es que las dos personas escogen la
mina insegura, que es peor que elegir la segura. La elección que tienen que hacer estos trabajadores
es un dilema del prisionero, como la que se vio en los capítulos 7 y 13. La dificultad, como en todas
estas disyuntivas, es que no le conviene a ningún participante, actuando por su cuenta, escoger la
alternativa que sea mejor para los dos. En esta situación se ve con facilidad por qué Jones y Smith
desearían establecer un acuerdo obligatorio para trabajar en la mina segura.
También se observa fácilmente por qué los analistas que ignoran las preocupaciones por el in-
greso relativo concluyen que las normas de seguridad en las minas ponen en peor situación a las
personas. Antes de las normas —afirman— cada trabajador escogía libremente trabajar en la mina
insegura. De esa decisión se concluiría que la seguridad adicional vale menos que 50 dólares por
semana a los ojos de ambos trabajadores. Por último, la conclusión aparente es que las normas de
seguridad perjudican a los trabajadores porque los hacen comprar seguridad que vale menos para
ellos de lo que cuesta.
Este argumento toca una fi bra sensible en el corazón de muchos amantes de la libertad. Sin
embargo, se equivoca al suponer que las elecciones de los individuos siempre revelan preferencias

14_CHAPTER 14.indd 502 3/7/09 12:57:27 PM


ProblemaS 503

subyacentes. Cuando importa el ingreso relativo, la estructura del problema es tal que las decisiones
individuales no indican cómo se sienten las personas en cuanto a los resultados acumulados de lo que
escogieron.

pRObLEMAS
1. A y B tienen que escoger entre trabajar en una mina segura por 200 dls./semana o en una mina
insegura por 300 dls./semana. La diferencia salarial entre las dos minas refleja los costos del equipo
de seguridad que se usa en la mina segura. La única consecuencia adversa de trabajar en la mina
insegura es que la esperanza de vida se acorta 10 años. A y B tienen funciones de utilidad de la forma
Ui[XiSiR(Xi)] = Xi + Si + R(Xi) para i = A, B, donde

Xi = ingreso de i por semana, en dólares


Si = 200 si la mina de i es segura, 0 si es insegura
R(Xi) = 200 si Xi > Xj, 0 si Xi = Xj, –250 si Xi < Xj (i, j = A,B; j ≠ i).

a) Si los dos escogen de manera independiente, ¿en qué mina van a trabajar? Explique. (Sugerencia:
use la función de utilidad para elaborar una matriz de rendimientos como la que se describe en el
texto.)
b) Si puede negociar entre ellos un acuerdo obligatorio que no cueste, ¿escogerán lo mismo que en
el inciso a? Explique.

14_CHAPTER 14.indd 503 3/7/09 12:57:28 PM


14_CHAPTER 14.indd 504 3/7/09 12:57:28 PM
Capítulo

15
CAPITAL

U
na vez la revista Fortune realizó una encuesta en la que pidió a directores de empresas que
señalaran a las “empresas mejor administradas de Estados Unidos”. A los entrevistados
se les pidió que no nombraran a su propia compañía y hubo numerosas coincidencias
en las listas que se entregaron. Algunas empresas, como Procter & Gamble, aparecieron en casi to-
das las listas.
Los entrevistados eran los observadores más informados a los que hubiera podido hacerse esa
pregunta y no hay ningún motivo para no creer que las compañías que citaron son, en efecto, de
las mejor administradas. Sin embargo, en un estudio de seguimiento se descubrió que las personas
que compraron acciones de estas compañías después de la publicación de la encuesta, en realidad
ganaron con sus inversiones algo menos del rendimiento promedio del mercado bursátil tomado en
conjunto.
Se ve un esquema semejante con respecto a los consejos de inversión publicados en los boletines
especializados. Éstos los compilan los principales analistas financieros del país y llevan a vender cien-
tos de dólares por año. En la lista de suscriptores se cuentan algunos de los más avezados miembros
de la comunidad inversora. Sin embargo, las acciones recomendadas en los boletines no tienen un
mejor comportamiento, en promedio, que las acciones escogidas por un mono que lanzara dardos a
la página de finanzas.
Por anómalos que parezcan estos esquemas a primera vista, descubrirá que son precisamente lo
que se habría pronosticado haciendo un análisis atento del mercado de capitales. En cuanto a los bo-
letines para inversionistas, la verdadera anomalía no es que las acciones que recomiendan no rindan
más que otras, sino que la gente siga pagando tanto por esos consejos.

505

15_CHAPTER 15.indd 505 3/6/09 8:11:47 PM


506 Capítulo 15 Capital

VISIÓN PRELIMINAR DEL CAPÍTULO


En este capítulo se examina el mercado de servicios de insumos de capital. En muchos aspectos se
van a recuperar intactos los resultados del estudio que se ha hecho ya de los insumos de trabajo.
Una característica que distingue al capital de otros insumos es que mientras éstos se contratan por
periodo, el equipo de capital es propiedad de la empresa. En el primer ejercicio se verán los factores
que rigen la decisión de una empresa de comprar una pieza de maquinaria de capital.
Luego se estudia la diferencia entre tasas de interés reales y nominales, una distinción que ayuda
a aclarar por qué las tasas que cobran los bancos y otros prestamistas suben con la inflación general.
Después se verá cómo se determinan las tasas de interés en el mercado de fondos destinados a
préstamos. Un tema que captará en particular su atención será el mercado de acciones y bonos y las
anomalías aparentes que se mencionan en la introducción. Otros temas de este capítulo son la renta
económica y los precios de consumo de carga máxima.

CAPITAL FINANCIERO Y CAPITAL REAL


Cuando las personas usan el término “capital”, en general se refieren a dos cosas muy diferentes:
capital financiero dinero u piensan en capital financiero, que básicamente significa dinero o alguna otra forma de activos en
otro activo en papel que hace papel que funciona como dinero; o en capital real (o capital físico), que es el equipo de producción,
las veces de dinero. como un torno o una prensa, que genera un flujo de servicios productivos en sucesión. Cuando nos
referimos al capital como factor de producción, casi siempre hablamos de capital real.1 Cuando se
capital real equipo habla de “mercado de capital”, se entiende el mercado de capital financiero, como préstamos ban-
productivo que genera un flujo
carios, acciones corporativas y bonos. En este capítulo nuestro interés inmediato es el capital real,
de servicios; también se llama
capital físico.
pero como las empresas necesitan capital financiero para comprar capital real, también se deben
considerar los mercados de capital financiero.

DEMANDA DE CAPITAL REAL


La teoría de demanda de mano de obra en la empresa individual que se desarrolló en el capítulo 14 se
aplica sin modificaciones a la demanda de otros insumos. A corto plazo, si la empresa puede adquirir
los servicios de todo el capital que desee a una renta de tasa constante anual de r, debe emplear
capital hasta el punto en que el producto del ingreso marginal (PIMK) sea exactamente igual a la tasa
de la renta:

PMK = IM × PMK = r, (15.1)

donde IM es el ingreso marginal de la empresa y PMK es el producto marginal de capital.


Si resulta que la empresa es un competidor perfecto en el mercado de su producto, de modo que
su ingreso marginal es igual que el precio de su producto, la ecuación 15.1 se reduce a la forma más
simple:

VPMK = P × PMK = r, (15.2)

donde VPMK denota el valor del producto marginal de capital y P es el precio de la producción de la
empresa.
Para las empresas que están en una industria perfectamente competitiva, la acumulación de las
curvas de demanda en una curva de demanda de capital de toda la industria tiene las mismas com-
plicaciones que se vieron en el caso de la mano de obra. Se debe tener en cuenta que una expansión
de la producción de la industria implica una rebaja del precio del producto, con las consiguientes

1
La excepción es lo que se llama dinero de “capital de trabajo” que la empresa mantiene a su disposición para pagar oportunamente
las deudas cuando sus ingresos no alcanzan. Como favorece el funcionamiento más eficiente de la empresa, este capital de trabajo es
un factor de producción como la mano de obra o la maquinaria.

15_CHAPTER 15.indd 506 3/6/09 8:11:47 PM


relaCión entre tasa de la renta y tasa de interés 507

reducciones de la cantidad de capital demandada. De nuevo, como antes, este efecto ya se tomó en
cuenta en la curva de demanda de capital del monopolista.
Una diferencia sobresaliente entre los mercados de capital y de mano de obra es que mientras
los trabajadores se especializan en ciertas actividades, las nuevas fuentes de capital (capital finan-
ciero) son casi completamente fungibles. Así, una suma dada financia lo mismo la construcción
de una máquina de helados o una prensa o la producción de una película de dibujos animados.
Sin embargo, cuando el capital financiero se usa para comprar capital real, la empresa pierde fle-
xibilidad. Si, con ciertos costos, es posible capacitar a los trabajadores para que realicen tareas
nuevas cuando cambian las condiciones, es mucho más difícil transformar una perforadora en una
máquina de coser.

RELACIÓN ENTRE TASA DE LA RENTA


Y TASA DE INTERÉS
¿Qué relación guarda el precio de renta de una unidad de equipo de capital real con la tasa de interés a
la que podría prestarse el dinero? Para dar la respuesta, póngase en el lugar de una empresa que tiene
el negocio de rentar máquinas. Suponga que el precio de compra de una máquina es de 1 000 dólares
y que la tasa de interés es de 5% anual. Para pagar nada más que el costo de oportunidad de los 1 000
dólares que están invertidos en la máquina, tendría que cobrar 50 dls./año. Pero, en general, también
hay otros costos. Suponga que una máquina requiere 100 dls./año para mantenimiento. Así, la renta
de alquiler de equilibrio sube a 150 dls./año. Por último, hay que considerar los cambios en el precio
futuro de la máquina.
Por simplicidad, suponga que el nivel total de los precios en la economía es estable (más ade-
lante se profundizará en lo que pasa cuando se suaviza esta condición). Hasta una máquina bien
mantenida pierde parte de su valor cada año. De hecho, si cada año se diseñan máquinas nuevas
más eficientes, una máquina vieja puede perder su valor de la noche a la mañana aunque siga fun-
cionando como nueva. Este fenómeno se llama obsolescencia tecnológica. Si el resultado neto obsolescencia tecnológica
de estos factores (uso y desgaste físico y obsolescencia tecnológica) es que el precio de la máquina proceso por el cual un
de este ejemplo baje 100 dls./año, el costo total de aprovisionarla será de 250 dls./año: 50 que se bien pierde valor no por
van en intereses, 100 en mantenimiento y 100 en valor de mercado perdido. Cualesquiera costos depreciación física, sino por
adicionales en los que incurra como negocio de renta (como los salarios del personal) tendrán que mejoras de la tecnología que
vuelven más atractivos los
sumarse a esta cifra.
productos sustitutos.
Sea m los gastos anuales de mantenimiento, expresados como una fracción del precio del bien de
capital, y sea ∂ la depreciación física y tecnológica, expresada de la misma manera. Si i denota la tasa
de interés del mercado, expresada en forma decimal, la tasa anual de renta de capital, k, será la suma
de i, m y ∂:

k = i + m + ∂. (15.3)

A veces una máquina aumenta su valor, en lugar de depreciarse con el tiempo. Esto pasa, por
ejemplo, cuando se encarece un insumo clave de la máquina. En tales casos, el término ∂ en la
ecuación 15.3 sería negativo. Por ejemplo, si la empresa de rentas de la que se habló antes esperara
que el precio de la máquina subiera 100 dólares el año entrante, alcanzaría su punto de equilibrio a
una renta de apenas 50 dólares, 200 dólares menos que si el precio de la máquina bajara 100 dólares.
Las expectativas de que vaya a subir el precio de un activo explican por qué cuando los precios
de las viviendas suben rápidamente, las rentas son menores que los pagos correspondientes de las
hipotecas.

EJERCICIO 15.1
Suponga que el precio de compra de una máquina vendedora de refrescos de cola es de 5 000
dólares. Si la tasa de interés anual es de 0.08, la tasa de mantenimiento es de 0.02 y la tasa de
depreciación física y tecnológica es de 0.10, ¿cuál será la renta anual de la máquina?

15_CHAPTER 15.indd 507 3/6/09 8:11:48 PM


508 Capítulo 15 Capital

CRITERIO PARA COMPRAR UN BIEN DE CAPITAL


Otro factor que distingue al capital de la mano de obra es que las empresas tienen la opción de
comprar el equipo de capital. A veces, los deportistas profesionales se compran y venden como si
fueran iguales que las máquinas, pero ni siquiera a ellos los pueden obligar a jugar indefinidamente
en un equipo en contra de sus deseos. En general, los contratos laborales permiten a los trabajadores
irse cuando las condiciones del empleo ya no los atraen, por la razón principal de que no es práctico
hacer que trabaje para el bien de la empresa un empleado que se siente a disgusto. Desde luego, la
máquina no goza de tales libertades. Se va con el mejor postor.
¿Qué factores rigen la decisión de una empresa respecto a si compra equipo de capital? Como
siempre, se ponderan las bondades de tener la máquina en relación con los costos. En el lado de las
ventajas, la máquina fomenta el ritmo de producción, no sólo en el periodo actual sino también en
el futuro. Suponga que la producción adicional que se genera con la máquina mejora los ingresos to-
tales de la empresa por un factor R durante cada año de los siguientes N años. Dígase también que el
mantenimiento de la máquina cuesta M por año y que al final de N años tiene un valor como chatarra
de S dólares. Ya por último suponga que la empresa recibe la máquina y espera que opere N años,
luego de los cuales la venderá como chatarra. ¿Cuánto aumentará el valor presente de la corriente de
utilidades de la empresa?
Para responder, es necesario traducir los nuevos ingresos que recibirá la empresa en el futuro en
un valor presente equivalente. Como lo vio en el capítulo 6, el valor presente neto de un dólar recibi-
do el año 1 contado a partir de ahora es 1 dl./(1 + i), donde i denota la tasa de interés del mercado. El
valor presente de un dólar recibido el año 2 a partir de hoy es 1 dl./(1 + i)2. (Para ver esto, pregúntese
cuánto dinero tiene que poner en el banco hoy a una tasa de interés anual de i para que su cuenta
valga 1 dólar dentro de dos años.) El valor presente neto de la corriente de rendimientos producida
por la máquina, incluyendo las ganancias de la venta como chatarra, está dada por:

R−M R−M R−M S


VP = + + + + . (15.4)
1+ i (1 + i ) 2 (1 + i ) N (1 + i ) N

El costo de la máquina es su precio de compra, PK. La empresa debe comprar la máquina si y


sólo si su valor presente es mayor o igual que PK. En la ecuación 15.4 se ve que el valor presente está
relacionado inversamente con la tasa de interés del mercado. Así, como en el caso de la empresa que
renta equipo, la empresa que es dueña de su propio capital quiere emplear más cuanto más baja sea
la tasa de interés del mercado.

EJERCICIO 15.2
Suponga que una máquina genera 121 dólares de ingresos al final de cada uno de los próximos
dos años, después de lo cual puede venderse como chatarra en 242 dólares. Si la tasa de interés
anual es de 0.10, ¿cuál es el importe máximo que pagaría una empresa por esta máquina?

DETERMINACIÓN DE LA TASA DE INTERÉS


Para recapitular, la demanda de equipo de capital de una empresa depende de la tasa de interés, el
precio de compra de capital y la tasa de depreciación tecnológica y física. Las tasas de interés están
determinadas por la intersección de las curvas de oferta y demanda de fondos para préstamo. Como
el capital financiero es perfectamente fungible, el mercado de fondos para préstamo es una encarna-
ción casi a la letra del ideal de un producto estandarizado y homogéneo. El resultado es un mercado
nacional (o, mejor dicho, internacional) de fondos para préstamo en el que la tasa de interés que se
cobra a un tipo de prestatario es prácticamente la misma en todas partes.
¿Cómo se relaciona la demanda de fondos para préstamo con la demanda de capital? La de-
manda de capital de una empresa dice cuánto capital quisiera emplear a una tasa de renta de capital

15_CHAPTER 15.indd 508 3/6/09 8:11:49 PM


determinaCión de la tasa de interés 509

determinada, k. Si es una empresa que tiene tiempo en operación, es de creer que ya haya adquirido
gran parte del capital que necesita.
Por simplicidad suponga que en el presente año la empresa quiere cerrar la brecha entre el capi-
tal que tiene y el que quisiera tener. Esta brecha constituye su demanda de fondos para préstamo. De
manera similar, en el nivel de la industria la demanda de dichos fondos es la diferencia entre el monto
de capital que las empresas juntas quisieran tener y el capital que ya tienen. A continuación, el precio
que se usa para racionar el dinero en el mercado de fondos para préstamo es la tasa de interés.
Las empresas no son las únicas prestatarias del mercado de fondos para préstamo. Los consumi-
dores toman préstamos para financiar la compra de casas y otros bienes. Los gobiernos toman prés-
tamos para tender caminos, levantar escuelas y financiar el déficit del presupuesto general. La curva
de demanda de fondos para préstamo es la suma horizontal de la demanda de todas estas fuentes.
En el lado de la oferta hay también numerosas fuentes de fondos para préstamo. Los ahorros
de los consumidores aumentan los fondos que ponen a disposición las empresas sacándolos de sus
utilidades. En los últimos años ha aumentado la importancia de la participación activa de presta-
mistas extranjeros en el mercado estadounidense de estos fondos. La teoría del comportamiento
del consumidor establece que un incremento en la tasa de interés aumenta o disminuye los ahorros
del consumidor. El efecto total es el resultado neto de los efectos compensatorios del ingreso y la
sustitución, y la teoría sola no indica cuál predomina. En los estudios empíricos se señala el hecho de
que la elasticidad de los ahorros de los consumidores con respecto a las tasas de interés unas veces es
positiva y otras negativa, pero en todos los casos es muy pequeña.
En cuanto a los ahorros de las empresas privadas no hay una analogía con el efecto ingreso en el
caso de los consumidores, así que la cantidad de fondos para préstamo suministrados por las empre-
sas responderá en forma positiva a las tasas de interés. La mayor parte de los prestamistas extranjeros
se contentan con suministrar fondos a los prestatarios estadounidenses cuando la tasa de interés
iguala o supera a la que ganan en su país. Si se suman horizontalmente todas las fuentes de oferta se
obtiene la curva de oferta agregada de fondos para préstamo. El volumen cuantioso de los préstamos
provenientes del extranjero en los últimos años indica que a esta fuente se debe la mayor parte de la
elasticidad que se ve en la curva de oferta de fondos para préstamo. La intersección de esta curva con
la de demanda agregada de fondos para préstamo, que se muestra en la figura 15.1, determina la tasa
de interés del mercado, i*, y el volumen total de los fondos intercambiados, FP*.

FIGURA 15.1
Equilibrio en el
mercado de fondos
Tasa de interés para préstamo
la cantidad de fondos para
S préstamo demandada a
cualquier tasa de interés
(D) es la diferencia entre
las existencias deseadas de
capital a la tasa de interés
i* y la cantidad de existencias
de capital que ya se tiene.
la oferta de fondos para
préstamo (S) viene de los
consumidores, empresas y
D prestamistas extranjeros.
la creciente importancia
Fondos para préstamo (dls./año) de los prestamistas
FP*
extranjeros garantiza que la
curva de oferta de fondos
para financiamiento tenga
pendiente positiva.

15_CHAPTER 15.indd 509 3/6/09 8:11:50 PM


510 Capítulo 15 Capital

TASAS DE INTERÉS NOMINALES Y REALES


Suponga que toma un préstamo de 1 000 dólares de un banco y acepta pagarlos dentro de un año a
una tasa de interés de 5%. Suponga también que una vez que pasa el año el nivel general de precios
de la economía subió 10% (como, por ejemplo, pasaría si todos los precios aumentaran en ese por-
centaje). ¿Cuál fue el costo real del préstamo?
Para responder esta pregunta imagine que cuando pidió el dinero lo usó para comprar 100 on-
zas de plata a 10 dls./onza. Como se supone que el precio de la plata, como cualquier otro precio,
aumenta 10% al año, esto significa que puede vender su plata en 11 dls./onza cuando el préstamo se
venza. Las ganancias de la venta serán de 1 100 dólares, o 50 más que los 1 050 que tiene que pagarle
al banco. El costo real del préstamo, medido en dólares a la fecha de vencimiento, es menos 50. No
sólo no le costó nada pedir el dinero, sino que incluso salió con una ventaja de 50 dólares. En el lado
recíproco de la transacción, el banco que le prestó el dinero perdió 50 dólares.
No hace falta decir que un banco no tiene esperanzas de quedarse en el negocio si sigue prestan-
do en términos tan desfavorables. Cuando los bancos esperan que el nivel general de precios aumen-
te, cobran una prima de interés para contrarrestar la erosión del poder de compra real de los pagos
futuros de los préstamos. La cifra real que aparece en el contrato de préstamo del banco se llama tasa
de interés nominal: en el ejemplo que se planteó antes, de 5%. Si n denota la tasa nominal de interés
anual, expresada como fracción, y q denota la tasa anual de inflación, también expresada como frac-
ción, la tasa de interés real queda dada por

n−q
i= . (15.5)
1+ q

Con los valores del ejemplo hipotético que se viene analizando, se tiene i = (0.05 – 0.10)/1.10
= –0.0455 o sea –4.55%. En la ecuación 15.5 se ve que cuando la tasa de inflación es baja, la tasa de
interés real es aproximadamente igual a la diferencia entre la tasa nominal de interés y la tasa de infla-
ción, n – q. En todos los ejemplos anteriores se supuso en forma implícita que la tasa de interés era la
tasa real. En sus decisiones de inversión, la empresa quiere comparar los costos reales del capital con
los beneficios reales y proceder sólo si los segundos superan a los primeros.

MERCADO DE ACCIONES Y BONOS


Un método común mediante el cual las empresas recaudan dinero para nuevas inversiones es la emi-
sión de bonos corporativos. Un bono es en esencia un pagaré que expide la empresa. Un inversionista
le da dinero a la empresa —por ejemplo, 10 000 dólares— y a cambio la empresa le devuelve un
bonito certificado impreso en el que se compromete a pagarle una tasa fija de interés —por ejemplo,
de 10%— durante determinado tiempo. El valor nominal del bono es la cantidad a la cual se vendió al
inversionista que lo compró a la empresa. La vida de los bonos corporativos varía considerablemen-
te. Los bonos de corto plazo prometen devolver su valor nominal en 90 días. Muchos bonos de largo plazo
se vencen en 30 años y algunos tienen plazos todavía más largos.
Una vez comprado, un bono suele ser comerciado en el mercado abierto. Si es de corto plazo,
su precio estará siempre cercano a su valor nominal. Sin embargo, en el caso de bonos a largo pla-
zo, el precio que alcanza en el mercado abierto puede diferir de manera sustancial del precio no-
minal.
Para ver por qué, suponga que la tasa de interés del mercado es de 10% cuando un inversionista
compra un bono de 10 000 dólares a una corporación; el bono promete pagarle 1 000 dólares de
interés anual durante los siguientes 30 años y luego devolverle los 10 000 dólares íntegros. Siem-
pre que la tasa de interés se mantenga en 10%, el bono seguirá “valiendo” 10 000 dólares, en el sen-
tido de que su pago de interés anual de 1 000 dólares compensa por completo al inversionista por el
costo de oportunidad de no tener su dinero. Pero suponga que de pronto la tasa de interés baja 5%.
Entonces, el costo de oportunidad de no tener 10 000 dólares baja de 1 000 dólares anuales a sólo 500
dls./año. El inversionista que tiene un bono que promete darle 1 000 dls./año no estará dispuesto

15_CHAPTER 15.indd 510 3/6/09 8:11:51 PM


merCado de aCCiones y bonos 511

a venderlo por únicamente 10 000 dólares, porque con la tasa de interés en 5%, necesitaría 20 000
dólares para ganar los 1 000 dólares anuales de interés que tendría si conservara el bono.
Ahora bien, el precio del bono de este ejemplo no aumentará hasta 20 000 dólares, porque el
nuevo comprador sabe que el bono sólo valdrá 10 000 cuando llegue a su vencimiento. Si el venci-
miento es inminente, el precio estará cerca de 10 000 dólares cualquiera que sea la tasa de interés.
Pero cuanto más alejada esté la fecha de vencimiento, menos afectará su valor nominal al precio
actual de mercado. De hecho, hay un tipo particular de bono, llamado bono perpetuo o consol, para bono perpetuo bono que
el cual el valor nominal no importa en absoluto. Un bono perpetuo es una promesa de pagar a su entrega un pago fijo cada año a
tenedor una suma fija cada año para siempre. Como aproximación, el precio actual en el mercado perpetuidad; también se llama
de un consol es la cantidad de dinero que se necesitaría, a la tasa de interés actual, para generar el consol.
mismo monto de interés que paga el consol. Así, por ejemplo, un consol que promete pagar 1 000
dls./año valdrá 10 000 dólares cuando la tasa de interés sea de 10% y 20 000 cuando la tasa sea de
5%. Más en general, si I representa el pago anual del consol y si i es la tasa de interés del mercado, el
precio del consol, PC, estará dado por:

I
PC = . (15.6)
i

EJERCICIO 15.3
Considere un bono perpetuo que paga 120 dls./año a su propietario. ¿Cuánto aumentaría el
precio de dicho bono si la tasa de interés bajara de 10 a 5%?

Las corporaciones no son las únicas instituciones que emiten bonos. Los gobiernos federal,
estatal y local también lo hacen. En los ejemplos que se vieron antes se supuso que hay una tasa
de interés del mercado única y uniforme en todo momento, pero de hecho hay muchas tasas de
interés. La regla general es que cuanto mayor sea el riesgo de que un prestatario no reembolse el
préstamo, mayor es la tasa de interés que tiene que pagar. Los bonos del gobierno de Estados Unidos
representan el menor riesgo de incumplimiento de pago en el mercado de bonos, así que el gobierno
paga tasas de interés más bajas que los emisores de otros bonos. Por ejemplo, suponga que los bonos
de 10 000 dólares a 30 años de General Motors Corporation pagan una tasa de interés anual de 8%,
mientras que el mismo bono emitido por el gobierno federal sólo paga 5%. La diferencia en la tasa
de interés de 3% es la prima de riesgo, que compensa al inversionista por el hecho de que General prima de riesgo diferencial
Motors tiene una probabilidad mayor de no pagar su préstamo que el gobierno federal sí paga. de pago necesario para
Quien posea un bono corporativo no tiene una participación en la propiedad de la corporación. compensar al proveedor de un
La posición financiera del tenedor de un bono es semejante a la de un banco que emitió un préstamo bien o servicio por tener que
a esa corporación. Los accionistas de la corporación son de hecho los propietarios. Una empresa correr un riesgo.
que quiere reunir dinero para invertir en equipo de capital puede contratar a un corredor para que
disponga una nueva emisión de certificados de acciones. Entonces, el corredor prepara una descrip-
ción de la propuesta de inversión de la empresa y con la cooperación de una red de otros corredores
ofrece la nueva acción en venta pública.
Si una empresa vende un millón de acciones, cada una constituye el derecho a un millonésimo
de las utilidades actuales y futuras de la empresa. Las utilidades se reparten directamente a los ac-
cionistas en la forma de dividendos o se reinvierten en la compañía, lo que aumenta el valor de las
utilidades futuras de ésta.
¿Qué precio debe pedirse por cada acción? Suponga que se sabe con certeza que el valor presen-
te de las utilidades actuales y futuras de la hipotética compañía del millón de acciones será de 500
millones de dólares. Entonces, las acciones deben venderse exactamente a 500 dólares cada una. A un
precio menor los inversionistas aumentarían su riqueza de inmediato comprándolas, y a cualquier
precio mayor que 500 dólares nadie tendría ningún incentivo económico para adquirirlas.

15_CHAPTER 15.indd 511 3/6/09 8:11:51 PM


512 Capítulo 15 Capital

Nunca o casi nunca se sabe con certeza el flujo futuro de utilidades de una compañía. El precio
que la gente está dispuesta a pagar por las acciones depende de sus mejores estimaciones sobre las
perspectivas de una empresa. Para las empresas nuevas o las que pasan a un territorio desconocido,
el riesgo de tener pocas ganancias es muy grande. Si una compañía de ingeniería genética encuentra
una manera de clonar una proteína inofensiva que destruya el virus del Sida, sus utilidades serán
ilimitadas. Pero muchas compañías luchan por ser la primera de la carrera y el fracaso es el destino
de la mayoría.
En otros ámbitos de la economía, las perspectivas económicas de una compañía son más fáciles
de pronosticar. Hertz tiene muchas décadas en el negocio de la renta de autos y no se ven grandes
sorpresas en el horizonte. Hertz casi no tiene probabilidades de dar un golpe fuerte y, por lo mismo,
sus probabilidades de sobrevivir son grandes.
Considere dos empresas con el mismo valor esperado de utilidades actuales y futuras. El valor
presente del flujo de utilidades de la empresa 1 es, con seguridad, de 100 millones de dólares. En
cambio, el valor presente de la empresa 2 tiene la mitad de probabilidades de tener 200 millones
de dólares o nada. Si el precio de las acciones de las dos empresas fuera el mismo, ¿cuál preferiría
comprar? La mayoría de los inversionistas sienten aversión por los riesgos (vea el capítulo 6) y, por lo
tanto, prefieren la empresa 1, la más segura de las dos.
Como la mayoría de las personas tienen esta preferencia, las acciones de las empresas con ganan-
cias futuras más riesgosas se venden a precios más bajos, así como los bonos más arriesgados deben
pagar tasas de interés más elevadas. El inversionista que enfrenta el mercado de acciones tiene una
restricción de presupuesto como la curva BB de la figura 15.2. A lo largo de BB, cuanto más segura es
la inversión, menor es el rendimiento esperado. Los inversionistas con tasas de sustitución marginal
relativamente bajas entre rendimiento y seguridad escogerán inversiones más arriesgadas, como A,
que ofrece rendimientos esperados relativamente altos. Las que tienen tasas de sustitución margi-
nal mayores entre rendimiento y seguridad escogerán una inversión como C. Prácticamente todos
quisieran tener acciones con rendimientos esperados mayores y también más seguridad. Pero los
términos disponibles en el mercado obligan a las personas a escoger entre uno de los dos atributos.

FIGURA 15.2
Compensación entre
seguridad y rendimiento
esperado
Como la mayor parte de los Rendimiento esperado Curva de indiferencia de
inversionistas sienten aversión un inversionista menos cauto
por el riesgo, no comprarán A
una acción riesgosa a menos
B
que su rendimiento esperado Curva de indiferencia
sea mayor que el de las de un inversionista cauto
acciones con menos riesgo.
Qué tipos de acciones adquirir
depende de las preferencias del C
comprador. los inversionistas
cautos prefieren acciones más B
seguras, como C. los menos Seguridad
cautos renunciarán a parte
de la seguridad a cambio del
mayor rendimiento esperado
sobre inversiones como A.

HIPÓTESIS DE LOS MERCADOS EFICIENTES


La mayoría de los economistas creen que el mercado accionario es eficiente. Con esto se quiere
decir que el precio de una acción incorpora toda la información disponible de sus perspectivas de

15_CHAPTER 15.indd 512 3/6/09 8:11:52 PM


merCado de aCCiones y bonos 513

ganancias actuales y futuras. Para ejemplificarlo, considere un caso hipotético sobre Genetech, una
exitosa compañía de ingeniería genética. Suponga que de acuerdo con las mejores perspectivas de
ganancias, el valor actual de la acción de Genetech es de 100 dólares. Ahora suponga que uno de sus
investigadores encuentra una cura milagrosa para el cáncer. El descubrimiento es simple y fácil de
patentar. Es seguro que la compañía conseguirá la aprobación oficial de su descubrimiento y enton-
ces sus ganancias aumentarán en forma considerable. Pero por el papeleo burocrático los trámites
de aprobación nunca tardan menos de tres años. Usted leyó sobre el descubrimiento de Genetech en
NewsWeek y decidió comprar acciones de la compañía. ¿Es una decisión inteligente?
La respuesta es que casi seguramente no, pero no porque la compañía no tenga el futuro color
de rosa que se le pronostica. La dificultad, de acuerdo con la hipótesis de los mercados eficientes,
es que el valor del nuevo descubrimiento se insertará en el precio de mercado de la acción casi al
instante. Para cuando alguien se entere, hace mucho que ocurrió el aumento en el precio que incitó
el descubrimiento.
Los críticos de la hipótesis de los mercados eficientes objetan que se refiere a un mundo ideal
sin fricción. Aseveran que en la realidad pasa mucho tiempo para que la información se difunda,
de modo que el efecto en los precios de las acciones es gradual y demorado. Concluyen que si las
noticias del descubrimiento de Genetech tuvieran apenas unas semanas de conocidas, aún quedaría
mucho margen para que los precios de las acciones aumentaran por este efecto.
Casi sin duda esta opinión está equivocada. La dificultad estriba en una confusión que surge
porque la nueva información no siempre viene de forma tan segura como se supuso en el ejemplo,
sino en una versión muy incierta. En la práctica sería más común que el mercado se enterara pri-
mero de que un investigador de Genetech tiene una pista prometedora para una cura del cáncer.
Esta información más limitada justificaría un aumento mucho más moderado en el precio de las
acciones de la compañía, que sería seguido por nuevos aumentos si el desarrollo continuara siendo
promisorio. Pero estaría seguido por un derrumbe de los precios si el avance se esfumara. En cu-
alquier caso, todo el valor de la información se reflejaría en el precio de las acciones del momento.
Como la información sobre nuevas oportunidades de hacer ganancias surge gradualmente, muchos
observadores cometen el error de concluir que la respuesta del mercado a la nueva información
también es paulatina.
A diferencia de las condiciones del ejemplo hipotético planteado aquí, en el mundo real es difícil
cuantificar exactamente qué información queda disponible en momentos específicos. Además, casi
siempre hay margen para que surjan diferencias de interpretación de cualquier informe. Por lo ante-
rior, es muy difícil verificar en forma empírica la hipótesis de los mercados eficientes. Sin embargo,
muchos economistas creen que es correcta. Si es imposible verificar empíricamente la hipótesis, ¿qué
explica la firme aceptación de ella por parte de los economistas?
La respuesta es que la hipótesis alternativa (que los precios de las acciones no incorporan toda
la información disponible) lleva a conclusiones que son difíciles de aceptar. Para ejemplificarlo, con-
sidere de nuevo el ejemplo de la cura del cáncer y suponga que el mercado no encareció de inmediato
el precio de las acciones como respuesta a las mayores utilidades futuras que se deducen del nuevo
descubrimiento. Entonces usted o alguien más tomaría el teléfono y giraría instrucciones a sus co-
rredores para que compraran todas las acciones de Genetech que pudiera pagar. A continuación se
sentaría a esperar que el mercado pujara por sus acciones hasta darles todo su valor en el mercado
para cosechar ganancias sustanciales.
La creencia que abrigan los economistas más profundamente que cualquier otra es que la única
manera de cosechar tales ganancias es mediante una combinación de talento, esfuerzo y suerte. Pero
si se negara la hipótesis de los mercados eficientes habría dinero puesto sobre la mesa para obtener
ganancias como éstas. No se necesitaría talento, esfuerzo en el trabajo y, como la información es
segura, tampoco buena suerte. Sólo llamaría a su corredor y esperaría a que el dinero cayera. Sobra
la gente que estaría encantada de ganarse la vida con este medio tan indoloro. Que por norma gen-
eral parezca imposible hacerlo es toda la confirmación de la hipótesis de los mercados eficientes que
necesitan los economistas.

15_CHAPTER 15.indd 513 3/6/09 8:11:53 PM


514 Capítulo 15 Capital

¿Por qué tener una acción de un monopolio no es mejor que tener una acción de una empre-
sa perfectamente competitiva?
Muchas personas creen que es mejor comprar acciones de una compañía muy rentable que de una
con rendimientos promedio. Una consecuencia importante de la hipótesis de los mercados eficientes
EL NATURALISTA es que esta idea está equivocada. Para entenderlo, considere dos empresas idénticas en todo, excepto
ECONÓMICO en que una es un monopolio y gana el doble de uti-
lidades que la otra. Si los precios de las dos acciones
15.1 fueran iguales, naturalmente que todos querrían
tener acciones del monopolio; pero por esa misma
causa, los precios de las dos acciones no pueden ser
iguales. El exceso de utilidades del monopolio dará
por resultado que sus acciones se vendan al doble
de precio que las de la otra empresa. Por lo tanto,
desde el punto de vista de la persona que compra
las acciones, la tasa de rendimiento será exacta-
mente la misma para las dos empresas. Es cierto, el
monopolio es dos veces más rentable, pero cuesta
el doble comprar sus acciones.
En El naturalista económico 15.1 se explica
una de las aparentes anomalías que se mencionó
¿Son buena oportunidad las acciones de las al comienzo del capítulo. Recuerde que las ac-
empresas monopolistas? ciones de las compañías mejor administradas no
rindieron más que la bolsa de valores tomada en
conjunto. De hecho, hasta tuvieron un rendimiento ligeramente peor. Ahora está en posición de
constatar que esto no significa que las compañías mejor administradas no sean más redituables que
las otras (aunque bien puede ser así). Si tienen más utilidades porque están mejor administradas y si
los inversionistas lo saben, el precio de sus acciones habría sido más alto desde el principio. No hay
ningún motivo para esperar que aumenten más deprisa que los precios de otras acciones.

LA ANOMALÍA DEL BOLETÍN DE INVERSIONES


Siempre me divierte la frecuencia con que me piden que dé o que reciba un consejo experto sobre la
bolsa de valores. Por ejemplo, cuando en las fiestas alguien se entera de que soy economista, siempre
me pregunta qué acciones debe comprar. Le digo que si supiera la respuesta, no tendría que ganarme
la vida trabajando.
Los corredores bursátiles que no saben que soy economista, sacan mi nombre de listas de dis-
tribución y sus asistentes me llaman para preguntarme si quisiera recibir consejos de sus jefes sobre
cómo invertir. También en este caso me veo obligado a declinar y, desde luego, no porque sepa a qué
acciones les irá bien.
En virtud de la fe que tienen en la hipótesis de los mercados eficientes, muchos economistas
creen que es infructuoso seguir los consejos de inversión. La única excepción importante a esta regla
general ocurre cuando el consejo se basa en información que no tienen otros inversionistas. Por
ejemplo, suponga que su hermano es el investigador de Genetech que se topó con la cura milagrosa
para el cáncer. Usted sabe que ha estado trabajando en el problema en el despacho que tiene en la
casa que comparten y sabe, por los brincos que da, que acaba de resolverlo. Y como se enteró de esto
antes que los demás, puede estar seguro de que va a hacer mucho dinero si compra acciones de Ge-
netech. Pero observe que ni siquiera este ejemplo infringe la regla de que las ganancias sustanciales
requieren talento, mucho trabajo o suerte. En este caso fue por su buena fortuna que se enteró del
descubrimiento antes de que los demás pudieran reaccionar.
Pero, en general, la información que se obtiene sobre las nuevas oportunidades de utilidades lle-
ga con días, semanas o hasta meses de retraso. Es muy difícil ver cómo información tan añeja puede
tener algún valor económico residual. Sin embargo, es patente que los inversionistas acuciosos se
comportan como si valiera la pena guiarse por noticias viejas.

15_CHAPTER 15.indd 514 3/6/09 8:11:55 PM


la anomalía del boletín de inversiones 515

Uno de los ejemplos más desconcertantes de este comportamiento es el boletín de inversiones.


Las principales corredurías emplean analistas para estar al día sobre los acontecimientos de la in-
dustria. Los resultados de los análisis se incorporan periódicamente en los boletines de inversiones,
que a continuación se remiten por correo a los suscriptores. Habitualmente las suscripciones a esos
boletines (que salen sólo una vez al mes) ascienden a varios cientos de dólares al año. La pregunta
incómoda para los economistas es: ¿por qué alguien pensaría que los consejos que vienen en los
boletines valen la pena de ser seguidos?
Para ilustrar el problema, piense en un analista que el 1 de junio descubre en sus investigacio-
nes que por procedimientos contables equivocados se subestimaron las utilidades de una compañía.
Como los inversionistas pensaban que ésta era menos redituable, sus acciones se vendieron a un
precio muy bajo. El analista comenta este resultado con sus colegas y superiores, quienes indagan un
poco más y llegan a la misma conclusión. El 15 de junio el investigador escribe sobre sus resultados
en un artículo para el boletín de la compañía, éste pasa a formación de páginas el 22 de junio y regre-
sa para lectura de pruebas el 6 de julio. Se corrigen los errores y la imprenta manda los ejemplares
terminados el 20 de julio. El personal prepara el boletín para enviarlo por correo y el 1 de agosto está
en las manos de los suscriptores.
Durante los casi dos meses que transcurrieron desde el descubrimiento inicial de la informa-
ción y el momento en que llega a los suscriptores, muchas personas han estado en posición de
aprovecharla. Por ejemplo, todo el personal de la correduría tuvo casi 60 días para comprar las
acciones subvaluadas. Con los extensos fondos de recursos que pueden aprovechar las corredurías,
60 minutos habrían sido suficientes para capitalizar por completo el descubrimiento. Desde luego,
muchos boletines salen con más frecuencia que una vez al mes, pero incluso con un boletín diario el
problema es esencialmente el mismo. Aun con el carácter instantáneo de Internet, es poco probable
que alguien sea el primero en consultar una actualización: alguien tuvo que escribirla en la página
electrónica.
¿Por qué alguien pensaría que es posible hacer dinero con la información de un boletín? Y, por
consiguiente, ¿por qué alguien gastaría cientos de dólares al año en comprar una suscripción con esta
información? Quizá muchos inversionistas adquieren los boletines no por los consejos de inversión,
sino para estar informados sobre los acontecimientos de la industria. La compraventa de activos en
los mercados de capital involucra transacciones entre personas reales. En las reuniones sociales y de
negocios que tienen lugar en el sector inversionista, como es natural, a los invitados parece ventajoso
aparentar que están bien informados y los boletines les sirven para cumplir este objetivo. Pero es
difícil ver cómo alguien podría ganar dinero siguiendo sus consejos de inversión.

CONSEJO SENSATO PARA LOS INVERSIONISTAS


De la exposición anterior podría parecer que los consejeros de inversión no sirven para nada. Al
contrario: hay una función evidente para el consejo profesional sobre inversiones, sólo que de un
género distinto al que muchos inversionistas esperan. La hipótesis de los mercados eficientes esta-
blece que los consejeros en inversiones no están en posición de enseñar cómo seleccionar acciones
para obtener más que el mercado en su conjunto. Pero pueden decirle cómo seleccionar las clases de
acciones que se presten mejor para sus objetivos como inversionista. En concreto, ayudan a decidir
de manera inteligente qué combinación de riesgo y rendimiento esperado conviene para los fines de
cada quien. Si usted es un joven que quiere ahorrar para su jubilación, lo lógico es que adquiera una
cartera de acciones más arriesgadas con rendimientos mayores que el promedio. Quizá no les vaya
bien en determinados periodos, pero si su verdadera preocupación es tener el mayor crecimiento a
largo plazo, es la mejor combinación para usted.
En contraste, si usted es una persona cercana a la jubilación, un asesor competente le dirá que
escoja acciones que sean seguras, aunque tengan menores rendimientos, porque en esta situación su
principal preocupación no es el crecimiento a largo plazo, sino asegurarse de que sus ahorros estén
protegidos de una pérdida grande de su valor.
Al salir al mundo, recibirá llamadas de corredores que le dirán que pueden ayudarlo a ganar en
el mercado. Rechace cortésmente sus servicios y busque un consejero con un sentido más realista de
lo que puede lograrse.

15_CHAPTER 15.indd 515 3/6/09 8:11:55 PM


516 Capítulo 15 Capital

POLÍTICA FISCAL Y MERCADO DE CAPITAL


La política fiscal del gobierno con respecto a los ingresos derivados de inversiones tiene un fuerte
efecto en cómo la gente asigna sus recursos. Un ejemplo es la exención del impuesto federal sobre
la renta que se aplica a los intereses ganados por los tenedores de bonos municipales. En muchos
aspectos un bono municipal es como uno corporativo, salvo que lo expide un gobierno local, no
una corporación. Un inversionista da al gobierno algún dinero (por ejemplo, 10 000 dólares) y el
ayuntamiento paga al inversionista una tasa fija de interés (más o menos del 5%) por un periodo
especificado, a menudo de 10 años, y luego devuelve los 10 000 dólares.
Para facilitar que los gobiernos locales consigan dinero, el Congreso eximió al interés ganado
por bonos municipales del impuesto federal sobre la renta. El tenedor del bono del ejemplo recibiría
500 dólares de intereses libres de impuestos al año. En cambio, el interés obtenido por los fondos del
gobierno federal está sujeto al gravamen completo, al igual que los intereses de los bonos emitidos
por las corporaciones.
La primera vez que alguien conoce esta política gubernamental se pregunta por qué hay quien
compra otros bonos, aparte de los municipales. ¿Por qué comprar bonos estatales o federales y pagar
impuestos por los intereses? En efecto, la gente sólo compraría bonos municipales si las condiciones
que ofrecieran fueran en todo lo demás iguales a las de los bonos federales o corporativos. Pero,
como es natural, sus condiciones no son las mismas. En particular, los gobiernos locales descubrie-
ron pronto que no tienen que pagar una tasa de interés tan alta como los emisores de otros bonos.
Así, un certificado de la Tesorería estadounidense a 10 años (un bono federal común) pagaría 6% de
interés, mientras un bono de la ciudad de Iowa paga sólo 5%.
Qué bono haya que comprar depende de la tasa marginal a la que se grave el ingreso del inver-
sionista. Suponga que usted tiene un gravamen de 33% sobre cualesquiera ingresos adicionales que
gane. Un bono federal que paga un interés de 600 dls./año lo dejará con sólo 400 dólares de interés
después de descontar los impuestos. Lo mejor que puede hacer es comprar el bono municipal con su
menor interés, de 500 dls./año, pero sin impuestos. En cambio, suponga que paga impuestos a una
tasa marginal de sólo 10%. Sus ganancias por el bono federal, después de descontar impuestos (540
dólares), serían mayores que las del bono municipal (500 dólares).
La política fiscal también afecta la decisión de una empresa de comprar o arrendar su equipo de
capital. Según la legislación fiscal estadounidense se concede a las empresas un margen de desgra-
vación de todo el capital que poseen. Los detalles de esto último son complicados, pero un examen
simple servirá para mostrar el punto clave. Si una empresa posee una máquina con, por ejemplo,
una vida de 10 años, se le permite deducir de sus utilidades corporativas 10% del precio de compra
de la máquina cada año, en correspondencia con la depreciación del valor de la máquina. Con esto,
se evita tener que pagar impuestos sobre esa parte de las utilidades. Es lógico desde el punto de vista
económico, porque el desgaste del equipo es un gasto operativo legítimo de la empresa. Ya no tiene
que pagar impuestos sobre ese gasto, como tampoco por los que eroga en mano de obra, papel o
cualquier otro insumo.
La desgravación por depreciación reduce el impuesto sobre la renta de la empresa sólo si tiene
que pagar dicho impuesto. Si en lugar de utilidades la empresa sufre pérdidas o si es una organización
sin fines de lucro, no le debe impuestos al gobierno federal y no se ejerce el derecho a la desgrava-
ción. Este hecho abre una oportunidad a los emprendedores que inician empresas para que arrienden
equipo de capital a empresas que deben poco o nada de impuesto sobre la renta. Como la compañía
arrendadora puede reclamar el valor completo de la desgravación, suministra a sus clientes capital
más barato que el que éstos pueden conseguir por sí mismos. Sin embargo, desde el punto de vista
de toda la sociedad, no hay ahorros de recursos. Lo que se ahorran las compañías con este sistema, lo
pierde el gobierno en impuesto sobre la renta. De hecho, casi con seguridad, el efecto neto es reducir
el valor total de la producción, puesto que se gastan recursos adicionales para organizar las empresas
arrendadoras.
Las corporaciones gastan recursos reales de muchas otras maneras para reducir su impuesto
sobre la renta. Este gasto puede eliminarse si se prescinde del impuesto sobre las utilidades. El efecto
sería transferir más ingresos a los accionistas de las corporaciones, los cuales podrían ser gravados
con la tasa que el gobierno considerara conveniente.

15_CHAPTER 15.indd 516 3/6/09 8:11:56 PM


renta eConómiCa 517

RENTA ECONÓMICA
En el uso cotidiano, el término “renta” o “alquiler” se refiere al pago que recibe un casero, una
empresa de alquiler de autos u otro propietario a cambio del uso de un activo económico real. Sin
embargo, en el análisis económico, el término ha adoptado una definición ligeramente diferente. La
renta económica es la diferencia entre el pago que recibe el propietario de un factor de producción y renta económica diferencia
su precio de reserva (la cantidad mínima necesaria para inducirlo a que lo emplee en su uso actual). entre lo que paga un factor
Por ejemplo, si un terrateniente prefiere que su tierra quede en barbecho antes que dejar que alguien de producción y la cantidad
la trabaje si el pago es de menos de 100 dls./mes, entonces sólo 150 dólares de un pago mensual de mínima necesaria para
250 que recibe actualmente es renta económica. inducirlo a dejarlo en su uso
Si la oferta de un insumo es perfectamente inelástica (es decir, si el propietario lo entrega por actual.
bajo que sea el precio), todo el pago al propietario es renta económica. Esta situación se muestra en
la figura 15.3a. Sin embargo, suponga que el propietario de un insumo tiene una curva de oferta con
pendiente positiva que interseca la curva de demanda del insumo a un precio de r*1, como se indica
en la figura 15.3b. Si los compradores del insumo se coludieran para hacer al propietario una oferta
de “tómalo o déjalo” por K*1 unidades del insumo, la cantidad menor que el propietario aceptaría es
igual al área bajo la curva de oferta hasta K*1 (la sección inferior sombreada en la parte b de la figura).
Pero si los compradores no se coluden, el propietario recibe un precio de K*1 por cada unidad vendida
y, así, obtiene más que este monto mínimo. Su renta económica es el área sombreada por arriba de
la curva de oferta.

FIGURA 15.3
Renta económica
a) Cuando la oferta de un
insumo es perfectamente
r r
inelástica, todo el pago
S0 que se recibe es renta
S1 económica. b) la renta
económica recibida por
un insumo con una curva
r*0 r*1 de oferta con pendiente
positiva es el área
sombreada por encima de
dicha curva.
D D
K K
K*0 K*1

a) b)

La renta económica es el análogo en el mercado de factores del excedente del productor en el


mercado de bienes. Recuerde que el excedente del productor es el ingreso que supera al mínimo
requerido para obtener determinada oferta de producción en el mercado de bienes. Como en el caso
del excedente del productor, si el resto de los factores permanece sin alteración, la renta económica
será mayor cuanto más inelástica sea la curva de oferta del producto.
Que el pago a un factor de producción constituya una renta económica depende del punto de
vista desde el cual se contemple una transacción. Por ejemplo, considere el terreno sobre el cual está
la empresa matriz de McGraw-Hill. Desde la perspectiva del dueño, ninguna parte del pago mensual
de McGraw-Hill es renta económica, porque si la empresa pagara menos por el sitio, el dueño podría
alquilarlo por la misma cantidad a otra empresa. En este sentido, McGraw-Hill no paga un centavo
más de lo necesario. Pero si se ve la misma transacción desde el ángulo de la economía en conjunto,

15_CHAPTER 15.indd 517 3/6/09 8:11:57 PM


518 Capítulo 15 Capital

prácticamente todo el pago es renta económica, porque el propietario del terreno lo ofrecería a al-
guien por mucho que baje el precio.
Aunque en la mente de la opinión pública la renta se asocia con pagos a los propietarios de insu-
mos de capital, las rentas económicas son de mayor importancia en el mercado de mano de obra. Re-
cuerde, en particular, la exposición de la economía en el capítulo 14 de que el ganador se queda con
todo. Las personas con talentos o capacidades especiales ganan salarios excepcionalmente elevados,
aunque en muchos casos, estarían dispuestas a realizar esos servicios por casi nada. Por ejemplo, los
salarios multimillonarios de los más populares miembros de la farándula y del deporte profesional
son casi todos renta económica, no pago por la inconveniencia de sacrificar su tiempo libre.

PRECIOS DE CONSUMO DE CARGA MÁXIMA


La demanda de capital de una empresa depende no sólo de la tasa de renta de capital, sino también de
cómo prorratea los costos de su equipo de capital entre los compradores del producto. Para ilustrar
la naturaleza de esta relación, y al mismo tiempo aclarar un tema de política muy importante, consi-
dere el caso de una generadora de energía eléctrica cuya demanda cambia notablemente a diferentes
horas del día. La pauta histórica ha sido que el Estado regule estas empresas, a las cuales se les ha
ordenado que cobren un precio único y uniforme por la electricidad que venden a diversas horas del
día o del año. Este precio está fijo en un nivel suficiente para que la compañía pague sus costos de
mano de obra, maquinaria, equipo y otros, más un rendimiento normal sobre la inversión.
Pero hace poco comisiones reguladoras han comenzado a modificar esta política y han impues-
to tarifas en las que los precios se relacionan directamente con la intensidad del uso general en el
momento de consumo. Por ejemplo, se le pide a una compañía cuyas cargas son mayores en las
horas hábiles que cobre más por la electricidad que se consume entre las 8 de la mañana y las 8 de la
precio de consumo de carga noche. Esta estructura de tarifas se llama fijación de precio de consumo de carga máxima.
máxima práctica por la que se Para ejemplificar los efectos de los precios de carga máxima, considere una central eléctrica que
cobran precios más elevados usa sólo dos insumos: generadores y combustible. Suponga que la demanda de electricidad a corto
por bienes y servicios en los plazo cambia con la hora del día, de acuerdo con la pauta que se muestra en la figura 15.4. La curva
periodos en que se consumen de demanda durante las horas hábiles se llama “demanda pico”.
más.

FIGURA 15.4
Efecto del precio de
carga máxima
al cobrar más durante
P(centavos/kWh)
las horas pico (P = 12) y
menos en las otras (P = 5),
las centrales eléctricas dan
a sus clientes un incentivo
para cambiar el consumo a
las horas de baja demanda. Demanda
Demanda pico
la reducción que se baja
produce en el consumo de
los periodos pico permite a Costo máximo = 12
las compañías atender a sus
Costo promedio = 10
clientes con mucho menos
existencias de equipo de
generación. Costo bajo = 5

Q(MWh/mes)
100 140 205 250

15_CHAPTER 15.indd 518 3/6/09 8:11:58 PM


reCursos no renovables Como insumos de la produCCión 519

La curva de demanda durante el resto del día se considera “demanda baja”. Suponga que una
compañía vende primero toda su energía a la misma tasa, ¢10/kWh, y que sus ingresos a esta tarifa
saldan exactamente todos sus costos. Observe en el diagrama que cuando la electricidad se vende a
¢10/kWh, la demanda es de 250 MWh/mes.
Si el costo promedio de producción es de ¢10/kWh, se sabe que el costo marginal de atender
a los usuarios en las horas bajas debe ser de menos de ¢10/kWh, mientras el costo marginal de
atender a los consumidores en las horas pico será mayor. Esto se deduce porque es posible atender
a un usuario adicional en horas bajas sin tener que agregar equipo de generación, mientras que es
necesario sumar generadores adicionales para atender a los usuarios en las horas pico. El único costo
de atender a un usuario en horas bajas será el combustible adicional que se requiera para accionar
la capacidad de generación que en otro caso estaría ociosa en ese periodo. Suponga que este costo
marginal de horas bajas es de ¢5/kWh. El costo extra durante las horas pico abarca no sólo el precio
del combustible, sino también el del capital extra que se requiere. Para fines de ejemplo, suponga que
estos costos de horas pico suman ¢12/kWh.
Por último, suponga que la central cobra ¢12/kWh a los usuarios de horas pico y sólo ¢5/kWh a
los otros. Observe en la figura 15.4 que el efecto es reducir el consumo en las horas pico en 45 MWh/
mes por mes, que en su mayor parte se traslada a las horas bajas. El cambio se produce de varias
maneras. Por ejemplo, la gente puede comprar cronómetros que programan calentadores, sistemas
de aire acondicionado y calefacción sólo en las horas bajas. De la misma manera, no usan lavaplatos,
lavadoras ni secadoras durante las horas pico. El resultado neto del cambio de consumo es que una
central puede atender a sus clientes con mucha menor capacidad de generación. Los ahorros que se
consiguen representan un incremento real del nivel de vida de los clientes.
La fijación de precios de carga máxima no se limita a la industria eléctrica. Las aerolíneas tam-
bién fijan precios por pico de consumo, de modo que cancelan una parte o el total de sus asientos
de descuento en los periodos de tránsito intenso. Muchas zonas de esquí aumentan los precios del
teleférico en los días festivos. Las diferencias estacionales de precios son una práctica común entre
los hoteles. El Metro de Washington cobra más por los boletos en las horas pico. Y como se dijo en
el capítulo 12, muchos cines cobran menos a la hora de la comida. La experiencia con estas prácticas
indica que cuando los costos de capital se asignan a los usuarios responsables de que se incurra en
ellos, el nivel total de capital requerido se reduce de manera significativa.

RECURSOS NO RENOVABLES COMO INSUMOS


DE LA PRODUCCIÓN
Además de las máquinas y otros equipos hechos por el hombre, los recursos naturales también son
insumos importantes en muchos procesos de producción. El análisis microeconómico arroja datos
de particular interés en el caso de los recursos no renovables. Un recurso no renovable es el que no
es posible restituir. Son ejemplos petróleo, oro, titanio y aluminio. Cuando las reservas en la Tierra
de estos recursos se agoten, tendremos que arreglárnoslas sin ellos. ¿Cómo asigna los recursos no
renovables un mercado competitivo?
El propietario de un recurso no renovable tiene dos opciones: 1) conservar el recurso por ahora o
2) venderlo. El costo de oportunidad implícito en la primera opción es el interés que se hubiera ganado
si el recurso se hubiera vendido y las ganancias se hubieran depositado en un banco (o si se hubieran
destinado a comprar una acción o un bono). La única razón económica que tendría el dueño de un
recurso no renovable para conservarlo sería la esperanza de que aumentara su precio en relación con
los precios de otros bienes y servicios. Suponga que usted es propietario de varios millones de barriles
de petróleo que vende a un precio actual de 60 dls./barril. Si la tasa de interés real es de 5%, ¿cuánto
tiene que aumentar el precio el petróleo el año próximo para que usted quiera conservar por lo menos
parte de su petróleo? Suponga que el precio sube a 66 dls./barril. Si usted vende todo su petróleo aho-
ra y deposita las ganancias en una cuenta al 5%, su riqueza total aumentará en este mismo porcentaje
el año próximo. Pero si se queda con su petróleo su riqueza total aumentará 10%. Como es evidente
que la segunda opción es más atractiva, lo más probable es que no venda su petróleo. En cambio, si se
espera que el precio del petróleo aumente sólo a 61.50 dls./barril el año entrante, su mejor opción se-

15_CHAPTER 15.indd 519 3/6/09 8:11:58 PM


520 Capítulo 15 Capital

ría deshacerse de todo su petróleo ahora e invertir las ganancias a 5%. Un aumento de 1.50 en el precio
de 60 dólares por barril significa que el valor del petróleo que conserve se acrecentará sólo 2.5%.
A partir de este ejemplo, debe quedar claro que para que el mercado de un recurso esté en
equilibrio, su precio debe subir exactamente a la tasa de interés real. Un aumento del precio menor
que esta cifra daría por resultado que todos los propietarios trataran de vender. Un aumento mayor
significaría el cierre total del comercio. Suponga que P0 representa el precio actual de un recurso no
renovable; por ejemplo, petróleo. Si este precio aumenta a la tasa de i por año, entonces la expresión
matemática del precio después de t años está dada por:2

Pt = P0(1 + i)t, (15.7)

cuya gráfica se muestra en la figura 15.5.

FIGURA 15.5
Trayectoria del precio
de equilibrio de un
recurso no renovable Precio
Cuando el mercado de un
recurso no renovable está
en equilibrio, su precio
aumenta a la tasa de
interés real.

P0

Tiempo (años)
0 t1

Se deducen dos conclusiones importantes del hecho de que los precios de los recursos no re-
novables aumenten a la tasa real de interés. En primer lugar, como las curvas de demanda de los
recursos no renovables tienen una pendiente negativa como cualquier otra, el aumento gradual del
precio genera una reducción paulatina en la cantidad demandada del recurso. Esto a su vez significa
que las reservas iniciales del recurso se gastan poco a poco, no de golpe. A medida que queda cada
vez menos del recurso, el aumento de los precios hace más lento el ritmo al cual se agota.
El segundo efecto importante del aumento de precios es que estimula la producción de susti-
tutos para los recursos no renovables. Tarde o temprano el mundo se quedará sin petróleo. Algún
día las actividades para las cuales se consume petróleo se tendrán que hacer con otro medio o dejar
de hacerlas. A medida que el petróleo se encarezca, los emprendedores tendrán más incentivos para
descubrir otras maneras de realizar las actividades que requieren petróleo.
A pesar de la incertidumbre sobre el volumen de las reservas petroleras actuales y sobre los cos-
tos futuros de las tecnologías alternativas, los mercados de petróleo y otros recursos no renovables

2
Si el aumento del precio es continuo, la relación exacta será:

Pt = P0eit,

donde e representa la constante 2.7183. La ecuación 15.7 proporciona una aproximación cercana a esta relación.

15_CHAPTER 15.indd 520 3/6/09 8:11:59 PM


resumen 521

funcionan notablemente bien la mayor parte del tiempo. Los trastornos graves son resultado de re-
ducciones de la oferta ocasionadas por razones políticas. Aparte de las interrupciones de este género,
quizá la mayor amenaza para el funcionamiento de estos mercados sean las políticas que pretenden
restringir el incremento natural de los precios de los recursos no renovables.
En un intento por tener suministros de energía al alcance de los pobres el gobierno del presi-
dente estadounidense Carter estableció un complejo sistema de controles de los precios de los com-
bustibles a finales de la década de 1970. Al mantener los precios de los energéticos muy por debajo
de sus valores de equilibrio, se pone en peligro la transición gradual entre fuentes de energía que es
característica de la asignación del mercado. Es mucho mejor dejar que las fuerzas del mercado rijan
los precios de la energía y encontrar otra manera de aliviar las cargas de los pobres (para saber más
sobre el tema, consulte el capítulo 18).
En el apéndice de este capítulo se comenta con más detalle el consumo de recursos no renova-
bles.

RESUMEN
• El propósito de este capítulo fue examinar el mercado para los • El mercado de acciones y bonos es una de las principales fuentes
servicios de insumos de capital. Muchos de los resultados del de fondos para financiar nuevo equipo de capital. Un bono cor-
estudio de los insumos de mano de obra se aplican también al porativo es en esencia un préstamo del comprador del bono a la
capital. Así, la demanda de una empresa de los servicios de un corporación. A medida que el bono se acerca al vencimiento, su
insumo de capital es el producto del ingreso marginal de dicho precio debe coincidir con su valor nominal. En cambio, los bo-
insumo, que en el caso de la empresa de competencia perfecta nos a los que les falta mucho para vencerse presentan una nota-
es igual que el valor del producto marginal del capital. ble relación inversamente proporcional entre las tasas de interés
• Una característica que distingue al capital de otros insumos es que corren y su precio. El precio de una acción cualquiera es el
que mientras que éstos se contratan por periodos, el equipo de valor presente (descontado convenientemente por el riesgo) de
capital es propiedad de la empresa. Cuando considera si compra las utilidades presentes y futuras a las que tenga derecho.
una máquina, la empresa tiene que preguntarse cuánto aumen- • En la “hipótesis de los mercados eficientes” se establece que, si
tará su producción no sólo en el periodo actual, sino también en se mantiene el riesgo constante, toda la información disponible
periodos futuros. La regla de decisión de la empresa es adquirir la sobre ganancias actuales y futuras de una empresa se incorpora
máquina si, y sólo si, el valor presente y futuro de los aumentos de inmediato al precio de sus acciones. La consecuencia es que
de ingresos posibilita que la máquina supere su precio de compra. un inversionista debe obtener resultados igualmente buenos sin
La regla ilustra los factores que determinan la tasa de la renta de importar qué acciones compre. Por lo tanto, la hipótesis de los
capital, que incluye la tasa de interés (o el costo de oportunidad) mercados eficientes explica por qué las sugerencias de inversión
de los fondos tomados en préstamo, la tasa de depreciación física de los “expertos” tienen poco o ningún valor.
y tecnológica y los movimientos futuros esperados del precio del • La política fiscal tiene numerosos efectos en las decisiones del
capital. mercado de capital. El hecho de que los bonos municipales es-
• La tasa real de interés mide el interés en términos de cantidades tén libres de impuestos explica por qué sus tasas de interés sue-
alternativas de bienes o servicios reales. Por ejemplo, si un ban- len ser menores que las de los bonos con intereses gravables. A
co presta 100 onzas de oro y pide un pago de 105 onzas al cabo veces la política fiscal también induce a las empresas a arrendar,
de un año, la tasa de interés real sería de 5%. Cuando la tasa de antes que comprar, el equipo de capital.
inflación es baja, la tasa nominal de interés es aproximadamente • El sentido que los economistas dan al término “renta” es dife-
igual a la tasa real de interés más la tasa inflacionaria. Esta rela- rente del uso familiar cotidiano. Es el pago a un factor de pro-
ción aclara por qué las tasas de interés que cobran los bancos y ducción que excede el valor mínimo requerido para mantener
otros prestamistas suben con la tasa inflacionaria general. el uso presente de dicho factor. De acuerdo con esta definición,
• La demanda de dinero prestado de una empresa depende de la una participación significativa de los pagos recibidos por los
comparación entre la cantidad de equipo de capital que quiere dueños del capital constituye renta económica. Las rentas tam-
tener y el que posee ahora. La oferta de fondos de financiamien- bién constituyen una parte cuantiosa del ingreso generado en el
to es muy sensible a las tasas de interés por la importancia de los mercado de mano de obra.
prestamistas extranjeros en el mercado. La tasa de interés del • En los esquemas de precio de carga máxima, empresas y auto-
mercado y el nivel de equilibrio de los préstamos están determi- ridades tienen que decidir cuánto cobran por el uso del equipo
nados por la intersección de las curvas de oferta y demanda de de capital cuando la intensidad de la demanda varía de manera
los fondos de financiamiento. considerable. Como siempre, la regla de asignación eficiente de

15_CHAPTER 15.indd 521 3/6/09 8:11:59 PM


522 Capítulo 15 Capital

recursos es fijar los precios con base en el costo marginal. Al rés real. Esto no sólo reduce la tasa a la cual se usan los recursos
establecer precios de carga máxima las empresas atienden su no renovables, sino que también estimula la tasa a la cual se
mercado al tiempo que usan cantidades significativamente me- desarrollan nuevos sustitutos. Además asegura una transición
nores de equipo de capital. sin sobresaltos entre el uso de un recurso no renovable y su sus-
• En los mercados competitivos de recursos no renovables, como tituto potencial.
petróleo o titanio, los precios tienden a elevarse a la tasa de inte-

pREgUNtaS dE REpaSo
1. ¿Cuál es la diferencia entre capital real y capital financiero? 5. ¿Por qué los precios de los bonos y las tasas de interés mantie-
¿Por qué la preocupación por uno comprende invariablemen- nen una relación inversa?
te la preocupación por el otro? 6. ¿Por qué es tan poco probable que los consejos de inversión
2. Explique por qué la depreciación es un costo económico que se publican tengan algún valor?
como cualquiera otro. 7. Dé tres ejemplos de fijación de precios de carga máxima que
3. ¿Por qué tasas de interés elevadas restan importancia econó- se usen en su comunidad.
mica a los acontecimientos futuros? 8. ¿Quiénes tienen mayores rendimientos esperados por sus
4. ¿Por qué las tasas nominales de interés aumentan aproxima- acciones: los inversionistas jóvenes o los de mayor edad que
damente a la par que la inflación? tienen una necesidad más inmediata de ingresos para el retiro?

pRoblEMaS
1. Está en trance de decidir cuál de dos computadoras adquiere. La tasa de interés es de 0.09 y la tasa
de mantenimiento de las dos máquinas es de 0.01. La primera computadora tiene un precio de 4 000
dólares y una tasa de depreciación física y tecnológica de 0.10. La segunda, a punto de quedar obso-
leta, tiene una tasa de depreciación física y tecnológica de 0.30. Si el pago de la renta anual de las dos
computadoras es igual, ¿cuál debe ser el precio de la segunda computadora para que usted quiera
comprarla?
2. Una máquina cuyo costo asciende a 100 dólares dará un rendimiento de 30 dólares al final de cada
uno de los siguientes tres años. Al término, se venderá como chatarra en 30 dólares. Si la tasa de
interés para la empresa es de 10%, ¿debe comprar la máquina?
3. Suponga que un bono a perpetuidad paga 3 000 dls./año a su propietario. ¿Cuál es el precio del bono
a una tasa de 5%? ¿Cuál al 6%?
4. Si la tasa fiscal marginal de todos es de 50% y si la tasa de interés sobre los bonos gravables del go-
bierno es de 8%, ¿cuál será la tasa de interés de los bonos gubernamentales exentos de impuestos?
5. Si la tasa de interés de los consoles gravables del gobierno es de 10%, ¿qué pasará con el precio de un
consol gubernamental exento de impuestos si el gobierno reduce la tasa marginal de todos de 50 a
30%?
6. Suponga que a la mayoría de los inversionistas no les importa si las compañías con las que hacen
negocios están en China. ¿Difiere (y por cuánto) la tasa de rendimiento de las acciones de empresas
que negocian en China de la tasa de las empresas que no lo hacen?
7. En la peluquería de Tony hay cuatro butacas y cuatro peluqueros. Casi todo el tiempo por lo menos
uno de ellos está ocioso, salvo los sábados por la mañana, cuando los cuatro están ocupados. Expli-
que, en términos que entienda alguien que no sabe de economía, por qué el precio de cortarse el
pelo es mayor la mañana de los sábados que en otros días.
8. Alrededor de 1890 los viñedos de Burdeos se infectaron de filoxera, un bicho que ataca las raíces de
las vides. Para conservar las variedades de uva originales, las viñas bordelesas fueron injertadas con

15_CHAPTER 15.indd 522 3/6/09 8:12:00 PM


respuestas a los ejerCiCios del Capítulo 523

raíces tomadas de plantas americanas. Desde 1890 todos los vinos de Burdeos están hechos con vides
injertadas con raíces americanas. Dichos vinos no tienen el mismo sabor que antes de la infección de
filoxera. Una botella de Burdeos elaborada antes de la infección se vende en miles de dólares. Suponga
que usted tiene una botella de Burdeos de antes de la filoxera que puede vender hoy en 2 000 dólares
y a usted sólo le interesa su valor como inversión. Para que le convenga conservar la inversión, ¿cuál
debe ser el precio esperado de la botella en n años si la tasa de interés del mercado es i?

RESpUEStaS a loS EjERcicioS dEl capítUlo


15.1. Como k = i + m + ∂, se tiene k = 0.08 + 0.02 + 0.10 = 0.20. Por lo tanto, el pago de la renta anual
será de k(5 000 dls.) = 1 000 dólares.
15.2. PV = (121/1.1) + (121/1.12) + (242/1.12) = 110 + 100 + 200 = 410 dls.
15.3. El precio a 10% es de 120 dls./0.10 = 1 200 dólares. Al 5% el precio es de 120 dls./0.05 = 2 400 dls.,
es decir que el precio aumenta en 1 200 dólares.

15_CHAPTER 15.indd 523 3/6/09 8:12:00 PM


15_CHAPTER 15.indd 524 3/6/09 8:12:00 PM
a p é N d i c E

15
EXAMEN MÁS DETALLADO DE
LA ASIGNACIÓN DE RECURSOS
NO RENOVABLES

RECURSOS NATURALES COMO INSUMOS


EN LA PRODUCCIÓN
Además de las máquinas y otro equipamiento hecho por el hombre, los recursos naturales también
son insumos importantes en muchos procesos de producción. Para fines de análisis, es común divi-
dirlos en dos categorías: 1) recursos renovables, como los árboles, y 2) recursos no renovables, que
existen en cantidades finitas que no se pueden recuperar cuando se agotan. Un ejemplo común de un
recurso no renovable es el petróleo. A continuación se estudian estas categorías.

RECURSOS RENOVABLES
Para ejemplificar los temas económicos que surgen en relación con los recursos renovables como
insumos, tome el caso de una maderera que se dedica a producir tablones a partir de árboles que
cultiva en sus propios terrenos. Su objetivo es plantar, cultivar y talar los árboles en forma tal que
maximice el valor presente de sus utilidades actuales y futuras.
Un agrónomo asesora a la empresa sobre la distancia a la que se plantan los árboles, qué fer-
tilizantes aplicar, etc. La competencia del economista es responder la pregunta referente a cuándo
talar los árboles. Cada año la empresa tiene que decidir si corta un árbol o si lo deja crecer otro año.
525

15_CHAPTER 15.indd 525 3/6/09 8:12:01 PM


526 Capítulo 15 apéndiCe 15: examen más detallado de la asignaCión de reCursos no renovables

El beneficio de cortar un árbol ahora es obtener la ganancia derivada de venderlo en seguida. Si la


empresa espera otro año, pierde el uso de ese ingreso por el momento, pero, mientras tanto, el árbol
sigue creciendo.
Suponga que los árboles crecen de acuerdo con la curva de desarrollo marcada B en la figura
A.15.1. Suponga también que el precio de la madera se mantiene constante con el paso del tiempo
y que la tasa real de interés del mercado, expresada como fracción, es constante: i/año. ¿A qué edad
debe talarse un árbol para aprovechar su madera?

FIGURA A.15.1
Curva de crecimiento Volumen de madera
de un árbol (pies de madera)
la curva marcada B indica
el volumen de madera de B
un árbol en función de su ∆B
edad en años. la pendiente ∆t
de la curva en un punto
está dada por la razón
ΔB/Δt.

Tiempo (años)
0

El ingreso por la venta del árbol es proporcional al volumen de la madera que contiene. La
pendiente de la curva de crecimiento, ΔB/Δt, indica cuánta madera más tendrá la empresa si espera
Δt unidades. La tasa a la cual crecerá el ingreso si no se corta el árbol es la madera adicional dividida
entre el tamaño del árbol (ΔB/Δt)/B. Así, si Δt es un año, y si ΔB = 0.10B, entonces la tasa a la que
crece el árbol maderable (y, por consiguiente, la tasa a la que aumenta el ingreso total) será de 0.10/
año. Como a la larga la pendiente de la curva B también se hará descendente (vea la figura A.15.1), se
concluye que el valor de (ΔB/Δt)/B también disminuirá con el tiempo, como se indica en la figura
A.15.2.

FIGURA A.15.2
Mejor momento para Tasa de crecimiento
cosechar
(∆B/∆t)/B
el momento óptimo de la
tala, t*, es cuando la tasa de
crecimiento del árbol, (ΔB/Δt)/ i
B, es exactamente igual a la
tasa de interés real, i. en ese
punto los ingresos adicionales
por dejarlo un Δt adicional es
exactamente igual al interés
que se ganaría por talar el Tiempo
árbol e invertir las ganancias a t*
la tasa i.

Si en lugar de dejar que el árbol siga creciendo la empresa lo tira ahora e invierte las ganancias
a la tasa de interés del mercado, su ingreso crecerá a una tasa de i/año. Se deduce que el árbol debe
talarse en cuanto
∆B / ∆t
= i, (A15.1)
B

15_CHAPTER 15.indd 526 3/6/09 8:12:03 PM


reCursos naturales Como insumos en la produCCión 527

lo que sucederá para t = t* en la figura A.15.2. Para valores de t a la izquierda de t*, la tasa de cre-
cimiento del árbol excede la tasa de crecimiento del dinero depositado a la tasa i, lo que significa
que la empresa debe esperar un poco más. En cambio, para valores de t mayores que t*, el dinero
depositado a la tasa i aumentará más de prisa que la madera del árbol, así que la empresa debe talarlo
un poco antes.

EJERCICIO A15.1
El volumen de madera en un árbol está dado por B = 80√t; la pendiente de su curva en cual-
quier punto está dada por ΔB/Δt = 40/√t. Si la tasa anual de interés es de 0.02, ¿a qué edad
debe cortarse el árbol?

Observe que el momento óptimo para cortar el árbol no es cuando alcanza su máxima altura.
Todo lo contrario, en el momento óptimo de talarlo todavía crece a un ritmo i > 0. El objetivo de la
empresa no es sacar de un árbol la mayor cantidad posible de madera, sino maximizar los ingresos
netos que resultan de un proceso continuo de silvicultura. Esto significa, necesariamente, retirar los
árboles maduros lentos para dejar el terreno a los jóvenes de crecimiento rápido.
Muchos observadores se quejan de que las madereras explotan los árboles en forma muy dis-
pendiosa, incluso para el estándar de la ecuación A.15.1. Cuando esto ha ocurrido así, la razón es
que, en general, los árboles crecían en un terreno que no era del silvicultor. Por ejemplo, cuando
el árbol crece en terrenos comunes y el primero que llegue es libre de derribarlo, siempre es talado
antes de alcanzar su madurez económica. Toda empresa en esta situación preferiría que los árboles
crecieran un poco más. Pero todas saben que un árbol que no tala una empresa hoy, lo talará otra.
Así, de mala gana, una empresa corta los árboles en cuanto tienen tamaño suficiente para justificar el
costo de procesarlos. Pero si un árbol crece en un terreno propio o controlado por la empresa que lo
va a cosechar, tiene todos los incentivos financieros para no talar demasiado pronto, porque hacerlo
reduciría el valor presente de las ganancias futuras de la empresa.

MÁS SOBRE RECURSOS NO RENOVABLES


Todas las fuentes de energía no renovables acabarán por agotarse, y entonces nos veremos obligados
a depender de recursos energéticos renovables. En esta sección el objetivo es investigar con mayor
detalle el proceso de transición de las fuentes no renovables a las renovables.
De nuevo, por simplicidad, suponga que el petróleo es la única fuente de energía no renovable
y que cuando se acabe se empleará la energía solar. Recuerde que en el capítulo se dijo que el equili-
brio del mercado petrolero requiere que el precio aumente a la tasa de interés. En la figura A.15.3, en
la que D es la curva de demanda del petróleo, se resume el efecto de elevar los precios del petróleo a
la tasa a la cual se consume.

FIGURA A15.3
P Efecto de aumentar
los precios sobre el
uso de un recurso no
P2 renovable
la curva de demanda de
un recurso no renovable,
P1 como cualquier otro,
tiene pendiente negativa.
P0 por consiguiente, elevar
gradualmente los precios
D genera reducciones
Q paulatinas en las cantidades
0 Q2 Q1 Q0 a las cuales se agotan las
reservas cada año.

15_CHAPTER 15.indd 527 3/6/09 8:12:04 PM


528 Capítulo 15 apéndiCe 15: examen más detallado de la asignaCión de reCursos no renovables

Si se sabe cómo aumentará el precio del petróleo a lo largo del tiempo y si se conoce su curva de
demanda es simple calcular el volumen de petróleo que queda en cualquier momento. Suponga que
las reservas actuales de petróleo son de S0 y su precio actual es P0. Si consulta la curva de demanda
del petróleo (figura A.15.3), verá que los consumidores usarán Q0 unidades de petróleo este año, con
lo que quedarán S0 – Q0 unidades al comienzo del año entrante. Al precio P1 del año entrante consu-
mirán Q1 unidades y quedarán S0 – Q0 – Q1 a principios del año siguiente, etc. Si se grafica la cantidad
de petróleo que queda en cada momento, se tiene la trayectoria de agotamiento de las reservas que se
muestra en la figura A.15.4.

FIGURA A.15.4
Trayectoria de
agotamiento de las Petróleo
reservas
S0
en el momento actual
(t = 0), las reservas
de petróleo son de S0
unidades. dado su precio
actual de P0 se consulta la
curva de demanda (figura S0 – Q0
a.15.3) para tener el
consumo de este año (Q0).
S0 – Q0 – Q1
las reservas que quedan a Trayectoria de agotamiento de las reservas
comienzos del año entrante S0 – Q0 – Q1 – Q2
(t = 1) será S0 – Q0. al
precio del año siguiente de Tiempo
P0(1 + i) = P1, el consumo 0 1 2 3
será Q1, lo que significa que
las reservas que queden
al inicio del año siguiente
serán S0 – Q0 – Q1, etcétera.

Suponga que el precio de la energía solar es P* por unidad y se espera que se mantenga constan-
te al paso del tiempo. Suponga también que las reservas de petróleo actuales representan S0 unidades
de energía. Ya se vio que la teoría de los mercados de recursos no renovables establece que el precio
de la energía en la forma de petróleo aumenta a la tasa de interés real. Pero la teoría también predice
que la última gota de petróleo se consumirá en el preciso instante en que su precio (medido en dóla-
res por unidad de energía) llegue a P*, el nivel en el que se vuelve económico usar energía solar.
Para ver por qué esta segunda predicción debe ser válida, suponga que los propietarios del pe-
tróleo no la esperan. En primer lugar suponga que esperan que cuando el precio llegue a P* les que-
dará todavía petróleo. El lugar geométrico marcado SEP0 en la sección superior de la figura A.15.5
representa la trayectoria de agotamiento de las reservas que corresponden a un nivel inicial de S0,
con precio inicial de P0. A partir de P0 los precios suben a la tasa real de interés e intersecan a P* en el
tiempo = t1 (sección inferior de la figura A.15.5). Observe en la sección superior que en t1 quedan S1
unidades de petróleo, pero sus propietarios saben que cuando el precio llegue a P*, ya no subirá más.
¿Por qué nadie va a querer pagar más que P* por el petróleo si puede tener energía solar a ese precio?
Si a los propietarios les queda petróleo cuando el precio llegue a P*, sólo podrán venderlo a la tasa a
la cual la gente demande energía a ese precio, y esto significa que al final tendrán que quedarse con
un activo (es decir, lo que quede del petróleo) que no aumentará de precio, y ningún inversionista
quiere eso.
Los propietarios en lo individual pueden escapar a este resultado si venden su petróleo ahora.
Como todos los propietarios tienen el mismo incentivo para vender, se deduce que el precio actual
bajará de P0 a P'0 en la sección inferior de la figura A.15.5. Esta caída del precio provoca dos cosas:

15_CHAPTER 15.indd 528 3/6/09 8:12:05 PM


reCursos naturales Como insumos en la produCCión 529

FIGURA A.15.5
Reservas petroleras Ajustes cuando
los inversionistas
esperan quedarse con
S0 excedentes petroleros
Trayectoria original de agotamiento de las reservas (SEP0 )
si los inversionistas creen
que todavía tendrán algo
Trayectoria nueva de agotamiento de las reservas (SEP1) de petróleo cuando el
precio llegue a P* (el
S1 precio de la energía solar),
Tiempo tratarán de reducir sus
reservas. esto provoca
que el precio actual baje
Precio (sección inferior), lo que
P* acrecienta el uso actual y
futuro. el aumento de la
tasa de consumo se refleja
en el desplazamiento hacia
P0 abajo de la trayectoria
de agotamiento de las
reservas (sección superior).
P0’

Tiempo
0 t1 t2

FIGURA A.15.6
Reservas petroleras Ajustes cuando los
inversionistas esperan
S0
Trayectoria original de agotamiento de las reservas (SEP0) que el petróleo se
acabe muy pronto
Trayectoria nueva de agotamiento de las reservas (SEP1) si se espera que el
petróleo se agote antes
de llegar al precio P*, los
propietarios concluyen
Tiempo
que pueden ganar más
que la tasa real de interés
Precio conservando su propiedad.
esto hace que el precio
P* del petróleo suba (sección
inferior). el ajuste al
alza del precio continúa
mientras los inversionistas
P0’ esperan que llegue a P*
Pt1 en el momento en que se
P0 agoten las reservas.

Tiempo
0 t1 t2

1) cuando el precio vuelve a subir a la tasa real de interés, pasará más tiempo antes de llegar a P*, y 2)
los niveles de consumo de petróleo serán mayores ahora y en el futuro (porque la curva de demanda
del petróleo tiene pendiente negativa). En la figura A.15.5 el primer efecto se refleja en el hecho de
que t2 > t1; el segundo, en que la nueva trayectoria de agotamiento de los recursos (SEP1) está debajo
de la original. Los dos efectos tenderán a aminorar la cantidad de petróleo que quede cuando el
precio llegue a P*. Como está dibujada la figura A.15.5, la última gota de petróleo se consume en

15_CHAPTER 15.indd 529 3/6/09 8:12:07 PM


530 Capítulo 15 apéndiCe 15: examen más detallado de la asignaCión de reCursos no renovables

el momento en que la nueva trayectoria de precio llegue a P*. Si los propietarios todavía esperaran
que quede algo en t2, el precio actual bajaría todavía más.
Por otro lado, suponga que los propietarios piensan que se van a quedar sin petróleo antes de
que el precio llegue a P*. Es decir, suponga, como en la figura A.15.6, que comienza con unas reser-
vas iniciales de S0 y precio de P0 y que se agotará el petróleo en t1, antes de que el precio llegue a P*.
Cuando el petróleo se agote, habrá que pagar P* por unidad de energía, puesto que la energía solar
será la única disponible. Así, los propietarios anticipan que en t1 no van a cobrar Pt1, sino P* por su
petróleo. Esto significa que al conservar su petróleo hasta t1 pueden ganar más que la tasa real de
interés. Así, si los propietarios del petróleo esperan que se agote antes de que el precio llegue a P*
tendrán un incentivo inmediato para dejar de venderlo ahora. Esto genera un incremento del precio
actual de P0 a P'0 en la sección inferior de la figura A.15.6, lo que a su vez causa un desplazamiento
hacia arriba de la trayectoria de agotamiento de las reservas. El aumento del precio seguirá mientras
los propietarios esperen que la nueva trayectoria del precio llegue a P* en el momento exacto en que
la trayectoria correspondiente de agotamiento de las reservas llegue a cero.
La teoría de los mercados de recursos no renovables que se citó antes establece de manera im-
plícita que los inversionistas saben cuánto petróleo queda en el subsuelo en todo momento. Pero
en la práctica nadie lo sabe con seguridad. Del mismo modo no se sabe exactamente cuánto costará
la energía solar cuando el petróleo se agote, porque eso depende de adelantos tecnológicos que es
difícil anticipar. En lugar de cifras conocidas de reservas petroleras y de precios de otras fuentes de
energía, el mercado depende de estimaciones muy imprecisas y siempre sujetas a revisiones a medi-
da que se tiene nueva información. Si alguien descubre un enorme yacimiento nuevo o un método
notablemente más barato de aprovechar la energía solar, los precios de los energéticos cambiarían
mucho, como se ilustra en el ejemplo siguiente.

Suponga que por los adelantos en la tecnología de los superconductores el precio de la ener-
EJEMPLO 1.11
gía solar se reduce a la mitad, de P* a P*/2. Demuestre el efecto que tiene en la trayectoria de
los precios del petróleo y en el momento en que se pasa de hidrocarburos a energía solar.
Suponga que el ajuste original de las reservas y la trayectoria de los precios están dados en la figura
A.15.7. Con un precio de la energía solar de P*, la última gota se consumirá cuando t = t1, el momen-
to en que el precio del petróleo llega a P*. Si el precio del petróleo continúa en su trayectoria original
cuando baje el precio de la energía solar, llegaría a P*/2 en t = t', cuando quedarían S't unidades de
petróleo. Por las razones que se explicaron antes, esto haría que bajara el precio del petróleo. El des-

FIGURA A.15.7
Respuesta a la caída Reservas petroleras
del precio de la
energía solar S0
Trayectoria original de agotamiento de las reservas (SEP0)

Trayectoria nueva de agotamiento de las reservas (SEP1)

S’t
Tiempo

Precio
P*

P*/2 Trayectoria original del precio


P0
P’0 Nueva trayectoria del precio
Tiempo
0 t’ t2 t1

15_CHAPTER 15.indd 530 3/6/09 8:12:08 PM


respuesta al ejerCiCio del apéndiCe 531

censo seguiría hasta llegar a un nivel (P0 en la figura A.15.7) en el cual la nueva trayectoria del precio
llegaría a P*/2 en el mismo momento en que la trayectoria de ajuste de las reservas correspondiente
llegara a cero. Como se muestra en el diagrama, el efecto de la reducción del precio de la energía
solar desplaza hacia abajo las trayectorias de precio y de reservas del petróleo. La era de la energía solar
iba a empezar en t = t1, pero ahora empezará mucho antes, en t = t2.

RESUMEN
• En mercados competitivos de recursos no renovables, como pe- generan nuevos sustitutos. También se asegura una transición
tróleo o titanio, los precios tienden a aumentar a la tasa real de uniforme del uso de un recurso no renovable a su reemplazo
interés. Esto no sólo reduce la tasa a la cual se usan los recursos final.
consumibles, sino que también estimula la velocidad a la que se

pREgUNta dE REpaSo
1. ¿Por qué el equilibrio necesita que el precio de los recursos no renovables aumente a la tasa real de
interés?

pRoblEMaS
1. Suponga que es posible generar energía solar a un costo de 2 dólares por unidad de energía. Supon-
ga que el precio actual del petróleo es de 1.80 dólares por unidad de energía y que hay suficiente
petróleo para 100 años más, a los niveles actuales de consumo. Si la tasa real de interés es de 0.05,
¿qué espera que ocurra con el precio actual del petróleo?
2. Quedan dos fuentes de energía no renovable, el petróleo extraído de yacimientos terrestres y el de
yacimientos marinos, cuyos costos de extracción son de 2 y 6 dls./barril, respectivamente. También
hay energía solar con precio de 12 dólares por energía equivalente a un barril de petróleo. Todos
piensan que actualmente quedan reservas de S1 barriles terrestres y S2 barriles marinos. ¿Qué efec-
to tendría el descubrimiento de A barriles adicionales de petróleo de extracción terrestre sobre el
tiempo y el precio bruto a los cuales el petróleo marino debe extraerse? ¿Qué efecto tendrá el descu-
brimiento en la duración de la “edad del petróleo marino”, es decir, en el tiempo durante el cual se
aproveche el petróleo marino?
3. Suponga que cierta especie de árbol crece de acuerdo con la función B = 20√t, donde B es el volu-
men de madera en el árbol, medido en pies de madera, y t es la edad del árbol en años. Si la tasa de
interés es de 0.05, ¿a qué edad deben cortarse los árboles si la meta es maximizar las utilidades a largo
plazo?
4. Suponga que quedan dos clases de petróleo, de subsuelo terrestre y aceite de esquisto bituminoso.
El costo de extraer una unidad de petróleo de subsuelo es de 2 dls./barril y de aceite de esquisto es
de 10 dls./barril. Una vez extraídos, los dos tipos de petróleo son idénticos. Explique, en términos
que entienda alguien que no sabe de economía, por qué no es lógico empezar a consumir el aceite
de esquisto bituminoso hasta que se haya agotado el petróleo del subsuelo.

RESpUESta al EjERcicio dEl apéNdicE


A.15.1 (ΔB/Δt)/B = (40/√t)/80√t = 1/2t = 0.02, que al resolverse da t = 25 años.

15_CHAPTER 15.indd 531 3/6/09 8:12:08 PM


15_CHAPTER 15.indd 532 3/6/09 8:12:08 PM
Pa rt e

ExTERNALiDADEs,
L A O F E RTA Y LbiENEs
5
A D E MpúbLicOs
A N DA
Y biENEsTAR sOciAL
esta parte del texto examina con detalle las condiciones en las que los merca-
dos no regulados producen resultados eficaces. en el capítulo 16 se examina la
función de un sistema de derechos de propiedad bien definido en el funciona-
miento de los mercados y las consecuencias de las externalidades, positivas y
negativas. en el capítulo 17 se examina lo que indica la teoría microeconómica
sobre el papel del gobierno. en el capítulo 18W (disponible en la red en www.
mhhe.com/frank7e) se aplica la teoría del comportamiento del consumidor y
la empresa para explorar la teoría formal subyacente en la mano invisible de
adam Smith.

533

16_CHAPTER 16.indd 533 3/7/09 10:45:25 AM


16_CHAPTER 16.indd 534 3/7/09 10:45:26 AM
CaPítulo

16
EXTERNALIDADES, DERECHOS
DE PROPIEDAD Y EL
TEOREMA DE COASE

n la esquina de las calles 22 y M noroeste, en Washington, DC, se encuentra un restaurante


E venerable llamado Blackie’s House of Beef. Durante el auge inmobiliario de la década
de 1970 este lugar se convirtió en centro principal para la construcción de edificios co-
merciales de lujo. Con el paso de los meses el costo de oportunidad de seguir operando
el restaurante de un piso crecía como la espuma. Pero los propietarios de Blackie’s no pensaban en
abandonar su lugar. El restaurante había sido negocio familiar durante muchos años y la familia
estaba determinada a ver que persistiera.
A la larga encontraron una solución creativa. Negociaron un acuerdo multimillonario mediante
el cual, sin mover ni un ladrillo del restaurante, se levantaría una elevada edificación sobre pilotes
por encima del restaurante. Blackie todavía opera: un hostal pintoresco, a la vieja usanza campirana,
anidado bajo el alto hotel Marriott. En una transacción multimillonaria semejante, un urbanizador
compró los derechos para construir un nuevo rascacielos a horcajadas sobre el Museo de Arte Mo-
derno en el centro de Manhattan.
En la mayor parte de las jurisdicciones, tener una propiedad confiere el derecho de evitar que
alguien levante un edificio en el espacio aéreo por encima de ella. Pero no se extiende el mismo
derecho a otras formas de actividad en tal espacio. Por ejemplo, cientos de miles de viviendas estado-
unidenses se localizan bajo las rutas de la aviación comercial entre ciudades importantes y cada día
miles de aeronaves usan el espacio sobre esas casas sin pagar un centavo. Este esquema de derechos

535

16_CHAPTER 16.indd 535 3/7/09 10:45:27 AM


536 CaPítulo 16 externalidadeS, dereChoS de ProPiedad y el teorema de CoaSe

no es un episodio histórico. Surgió, según se verá, como un medio para hacer el uso más eficiente de
la propiedad cuando es difícil negociar acuerdos caso por caso.

VISIÓN PRELIMINAR DEL CAPÍTULO


En este capítulo, los temas son las externalidades y los derechos de propiedad. Se comienza con una
serie de ejemplos que ilustran lo que ocurre cuando una acción de una parte daña a otra y ambas
están dispuestas a negociar sin que esto implique un costo. Luego se considera un conjunto relacio-
nado de ejemplos en los que la negociación es costosa. A continuación se aplicarán los principios
que se desprenden de estos ejemplos a varias preguntas concernientes al diseño de los derechos de
propiedad. ¿Debe concederse al dueño de un muelle el derecho de impedir a un barquero atracar
durante una tormenta? ¿Cuándo debe permitirse a una persona que impida a otras cruzar por sus
tierras o que obstaculicen su vista? ¿Los pastizales deben ser propiedad privada o común? ¿Debe
aceptarse que un urbanizador construya un edificio de oficinas sobre la propiedad de otra persona
sin su consentimiento? ¿Debe tolerarse que los aviones vuelen sobre las casas? Como se verá, las
respuestas a estas preguntas dependen de los arreglos a los que llegarían las personas si fueran libres
para negociar de forma gratuita.
A continuación se aplica la teoría de los derechos de propiedad y las externalidades del tema
de la competencia por las posiciones relativas. Se concluye el capítulo con un examen del gravamen
fiscal como posible solución al problema de las externalidades negativas.

NATURALEZA RECÍPROCA
DE LAS EXTERNALIDADES
En la primera edición de este texto (1991), comenzaba esta sección con la siguiente reflexión:

Una de las grandes injusticias de la vida académica es que no le hayan dado el Premio
Nobel de economía a Ronald H. Coase.1

Por eso me sentí encantado cuando supe que, por fin, le concedieron el premio a Coase en 1992.
En la actualidad es profesor emérito de la Escuela de Derecho de la Universidad de Chicago y autor
del artículo de economía más citado de la posguerra: “The Problem of Social Cost” (“El problema
del costo social”).2 Este trabajo cambió profundamente la manera en que economistas expertos en
derecho, filósofos de la política y otros piensan acerca de las externalidades y las instituciones legales
y sociales que evolucionaron para hacerles frente.
Coase partió de un ejemplo relativo a un médico cuya habilidad para examinar pacientes era
trastornada por el ruido de la maquinaria de una confitería que operaba en el edificio contiguo.
Históricamente, la opinión económica y legal sobre la situación era simple: el ruido del repostero
perjudicaba al médico y había que acallarlo. La idea de Coase fue que esta opinión ignora por com-
pleto la naturaleza recíproca del problema. Cierto, el ruido del repostero perjudica al médico, pero
si se impide el ruido se perjudica al repostero. El repostero no hace ruido para perjudicar al médico
sino para ganarse la vida. En tales situaciones alguien resulta perjudicado pase lo que pase. Que el perjuicio
que el ruido causa al médico sea mayor que el perjuicio que se causaría al repostero si se le prohibiera
hacerlo es, en rigor, una cuestión empírica. Según entendía Coase, el interés común de las partes es
evitar el mayor de estos dos resultados desagradables.
La opinión unilateral anterior sobre las externalidades fundó una tradición legal en la que el
repostero era responsable de los daños que su ruido le causaba al médico. Sin embargo, Coase señaló

1
Coase se pronuncia aproximadamente como “cous”.
2
Journal of Law and Economics, 3, 1960, pp. 144-171.

16_CHAPTER 16.indd 536 3/7/09 10:45:27 AM


naturaleza reCíProCa de laS externalidadeS 537

que si el médico y el repostero pudieran negociar sin costo entre ellos, el mejor resultado ocurriría
independientemente de si el repostero tenía la responsabilidad. La argumentación simple y elegante
con la que justificó sus afirmaciones se ilustra en la siguiente serie de ejemplos.

Suponga que el beneficio para el repostero de seguir haciendo ruido es de 40, mientras el EJEMPLO 16.1
costo para el médico es de 60 (véase nota 3 a pie de página). Si la única alternativa para el
repostero es no producir ruido, ¿qué pasa si se le imputa la responsabilidad por el daño del so-
nido? (Tener la responsabilidad por el daño significa quedar obligado a indemnizar al médico
por cualquier perjuicio causado por el ruido.)
El repostero examinará sus dos opciones (cerrar o compensar al médico) y escogerá aquella con la
que salga mejor librado. Si se queda en el negocio, ganará 40, pero tendrá que pagar 60 al médico, lo
cual representa una pérdida neta de 20. Si cierra, su ganancia neta es de cero y, puesto que es mejor
que perder 20, suspenderá su operación.
Por otro lado, suponga que no se imputa al repostero ninguna responsabilidad por los daños del
ruido. Es decir, que la ley le concede el derecho de seguir operando sin compensación para el médi-
co. Coase argumenta que, en este caso, el médico pagará al repostero para que cierre. Si el repostero
no cierra, sólo ganará 40, en tanto que el médico perderá 60. Pero el médico puede compensar al
repostero por la pérdida de cerrar y todavía le quedará suficiente para estar mejor que si el repostero
no cierra. Por ejemplo, suponga que el médico paga al repostero 50 por cerrar. Ahora, la ganancia
neta del repostero será de 10 más que si permaneciera en el negocio. La ganancia neta de 10 del
médico sería 10 más que si el ruido continuara.
Si P denota el pago que el médico hace al repostero para compensarlo por cerrar, se sabe que P
debe ser por lo menos de 40 (lo que obtendría el repostero si quedara abierto) y no más de 60 (lo que
el médico ganaría si no hubiera ruido). Los resultados de los dos regímenes legales (si el repostero
tiene o no tiene una responsabilidad civil) se resumen en la tabla 16.1.

TABLA 16.1
Resumen de los resultados y ganancias del ejemplo 16.1

Beneficio neto la ganancia para el


repostero por operar
Régimen legal Resultado Médico Repostero Total
es de 40. la pérdida del
responsabilidad el repostero cierra para no 60 0 60 médico por el ruido es de
indemnizar 60. el resultado eficaz es
Sin el médico paga P al 60 – P P 60 que el repostero cierre
responsabilidad repostero para que cierre y así ocurre con los dos
40 ≤ P ≤ 60 regímenes legales.

Observe que como la ganancia del repostero por operar su maquinaria (40) es menor que el
daño por el ruido que causa al médico (60), el resultado más eficiente es que el repostero cierre. En
el ejemplo 16.1 se aclara que si el médico y el repostero son racionales y pueden negociar de manera
gratuita, esto es lo que ocurrirá independientemente de si el repostero tiene una responsabilidad
civil por el daño ocasionado por el ruido. En términos de eficiencia, el régimen legal es una cuestión

3
Las cifras de costo y beneficio que se usan en este ejemplo y los siguientes representan los valores presentes de todos los costos y
beneficios actuales y futuros para las partes.

16_CHAPTER 16.indd 537 3/7/09 10:45:28 AM


538 CaPítulo 16 externalidadeS, dereChoS de ProPiedad y el teorema de CoaSe

de absoluta indiferencia en este caso. Sin embargo, en términos de distribución, las partes no son
neutrales en cuanto a la responsabilidad civil. Si el repostero no es responsable, su ganancia es P ≥ 40,
mientras que si es responsable, se verá obligado a cerrar sin ganar nada. La ganancia neta del médico
será de 60 si el repostero es responsable, pero de 60 – P si no es responsable.

EJEMPLO 16.2 Igual que el ejemplo 16.1, salvo que el beneficio para el repostero por operar es de 60, la
ganancia para el médico de un ambiente limpio es de sólo 40. Suponga que el médico debe
cerrar si el ruido continúa.
Esta vez el resultado eficiente es que el repostero siga operando, puesto que su ganancia excede
el costo que impone al médico. Si no enfrenta responsabilidades civiles por los daños del ruido, el
repostero seguirá en el negocio y la mejor opción del médico es cerrar. Por otro lado, si el repostero
tiene una responsabilidad civil por los daños del ruido, seguirá operando y pagará al médico 40 para
compensarlo por sus pérdidas. Los resultados netos de este ejemplo se resumen en la tabla 16.2.
Observe que, como en el ejemplo 16.1, los dos regímenes legales llevan al resultado más eficiente,
pero tienen consecuencias distributivas muy diferentes.

TABLA 16.2
Resumen de los resultados y ganancias del ejemplo 16.2

Beneficio neto la ganancia para el


repostero por operar
Régimen legal Resultado Médico Repostero Total
es de 60. la pérdida del
responsabilidad el repostero no cierra y 40 20 60 médico por el ruido es de
paga al médico 40 40. el resultado eficiente
Sin el repostero no cierra; el 0 60 60 es que el repostero siga
responsabilidad médico cierra operando y así ocurre con
los dos regímenes legales.

En los ejemplos anteriores se supuso que las únicas alternativas abiertas para las dos partes eran
seguir operando de la forma actual o cerrar en definitiva. Pero en la práctica, una o las dos partes
tienen una gama más extensa de alternativas. Como se muestra en los ejemplos siguientes, aquí
también la capacidad de negociar en forma gratuita lleva a resultados eficientes.

EJEMPLO 16.3 Igual que en el ejemplo 16.1, salvo que ahora el repostero tiene acceso a un aparato aislante
que eliminará por completo el ruido de sus máquinas. El costo del aparato es de 20, lo cual
significa que si lo instala, su ganancia neta de operación bajará de 40 a 20. Como en el ejem-
plo 16.1, el médico gana 60 si no hay ruido, cero si hay ruido.

Si el repostero se enfrenta a alguna responsabilidad civil por el daño del ruido, su mejor opción es
instalar el aislante. Sus alternativas son cerrar o pagar al médico 60 por el daño del ruido y con
cualquiera de estas opciones está peor. Si el repostero no enfrenta una responsabilidad civil, lo mejor
para el médico será pagarle al repostero para que instale el aislante. En efecto, su alternativa es cerrar
o soportar los daños del ruido. El pago mínimo que sería aceptable para que el repostero instalara
su aislante es 20, lo que cuesta. Lo más que el médico estaría dispuesto a pagarle por la instalación es
60, la cantidad que perdería si no lo coloca. Sea P el pago del médico al repostero. En la tabla 16.3 se
resumen los resultados y ganancias de los dos regímenes legales.

16_CHAPTER 16.indd 538 3/7/09 10:45:28 AM


naturaleza reCíProCa de laS externalidadeS 539

TABLA 16.3
Resumen de los resultados y ganancias del ejemplo 16.3

Beneficio neto la ganancia para el


repostero por operar
Régimen legal Resultado Médico Repostero Total
sin aislante es de 40. el
responsabilidad el repostero paga por 60 0 80 aislante cuesta 20. la
instalar un aislante pérdida del médico por
Sin el médico paga P al 60 – P 20 + P 80 el ruido del repostero
responsabilidad repostero por instalar el es de 60. el resultado
aislante 20 ≤ P ≤ 60 eficiente es que el
repostero instale el
aislante y siga operando,
y así ocurre con los dos
regímenes legales.

Considere ahora lo que ocurre cuando el médico también tiene que hacer ajustes para evitar los
daños causados por el ruido del repostero.

Igual que el ejemplo 16.3, salvo que el médico puede evitar los daños del ruido si cambia la EJEMPLO 16.4
sala de auscultación al otro lado de su consultorio. La habitación ruidosa en la que ahora
examina a sus pacientes se puede usar como almacén. Este arreglo le cuesta 18 al médico.

Con esta nueva opción el médico está en posición de eliminar los daños del ruido al menor costo
posible. Si el repostero tiene una responsabilidad civil por los daños del ruido, ofrecerá al médico un
pago de P para compensarlo por remodelar su oficina. El pago debe ser por lo menos de 18, pues de
otro modo el médico no haría el reacomodo (recuerde que si el repostero tiene una responsabilidad
civil, el médico tiene la opción de recibir una indemnización completa por daños). El pago no puede
exceder de 20 porque en ese caso el repostero instalaría el aislante y resolvería el problema por su
cuenta. Si el repostero no tiene una responsabilidad civil, el médico correría con los gastos de remo-
delar su oficina. Los resultados y ganancias de este ejemplo se resumen en la tabla 16.4. Observe que
se vuelve a tener el resultado eficiente cualquiera que sea el régimen legal que se escoja. Observe
también que la elección de un régimen legal afecta de nuevo la distribución de costos y beneficios,
sólo que esta vez por un margen mucho menor que en el ejemplo 16.3. La diferencia es que cada par-
te tiene ahora un método más barato para resolver en forma unilateral el problema del ruido. En el
ejemplo 16.3 el médico no tenía alternativa, lo que le daba al repostero un gran poder de negociación
cuando no enfrentaba responsabilidades civiles por los perjuicios del ruido. Por el contrario, en este
ejemplo el repostero no puede sacar un pago grande del médico a cambio de no hacer ruido porque
el médico puede resolver el problema por su cuenta.

Los esquemas que se revelan de los ejemplos anteriores pueden establecerse formalmente así:

Teorema de Coase: cuando las partes afectadas por las externalidades pueden negociar
de manera gratuita una con otra, se alcanza un resultado eficiente, sin importar cómo se
asignen legalmente las responsabilidades por los daños.

Cuando se publicó el trabajo clásico de Coase se convirtió en tema de grandes polémicas. Mu-
chos lo interpretaron en el sentido de que el gobierno no tiene ninguna función verdadera para

16_CHAPTER 16.indd 539 3/7/09 10:45:29 AM


540 CaPítulo 16 externalidadeS, dereChoS de ProPiedad y el teorema de CoaSe

TABLA 16.4
Resumen de los resultados y ganancias del ejemplo 16.4

Beneficio neto la ganancia para el


repostero por operar sin
Régimen legal Resultado Médico Repostero Total
aislante es de 40. el aislante
responsabilidad el repostero paga P al 42 + P 40 – P 82 cuesta 20. la pérdida del
médico por remodelar su médico por el ruido es
oficina 18 ≤ P ≤ 20 de 60. el médico puede
Sin el médico paga la 42 40 82 remodelar su oficina para
responsabilidad remodelación de su oficina eliminar el problema del
ruido a un costo de 18.
el resultado eficaz es que
el médico remodele su
oficina y así ocurre con los
dos regímenes legales.

resolver problemas relacionados con la contaminación, el ruido y otras externalidades. Con esta in-
terpretación, el mensaje de Coase parecería ser que si el gobierno no interviene, la gente siempre en-
cuentra soluciones eficientes. Pero Coase dijo claramente que esta conclusión es válida sólo para un
mundo en el que las partes pueden negociar unas con otras a un costo relativamente bajo. Entendía
que con muchas externalidades importantes esta suposición no se satisfacía. En el nivel más simple
se requieren tiempo y energía para negociar, y cuando los beneficios posibles son magros, quizá no
valga la pena. Por otro lado, hay situaciones en las que un único contaminador perjudica a muchas
personas. Negociar con grupos grandes es de por sí difícil y costoso, y los miembros de un grupo
tienen muchos motivos para evitar tales costos. Otra barrera grave para la negociación es el proble-
ma de cómo dividir los excedentes. Recuerde que en el ejemplo 16.3 el resultado eficiente era que el
médico pagara al repostero por instalar el aislante. El pago mínimo aceptable para el repostero sería
20, el costo del aislante. Lo más que el repostero puede esperar obtener del médico es 60, el valor
para el médico de eliminar el ruido. Naturalmente, el médico quisiera pagar sólo 20 y al repostero le
gustaría tener 60. Si los dos adoptan una postura inflexible en las discusiones, brotarán animosidades
y se abre la posibilidad de que se rompa por completo el trato. Por estas razones y muchas otras, las
negociaciones son caras. En tal caso, es muy importante qué régimen legal se elija, como se ilustra en
los ejemplos siguientes.

EJEMPLO 16.5 Como en el ejemplo 16.2, suponga que la ganancia para el médico en un ambiente sin ruido
es de 40, mientras la ganancia para el repostero por operar sin cortapisas es de 60. Suponga
también que el repostero tiene acceso a un aparato aislante que elimina todos los daños por
ruido a un costo de 20. Por último, dígase que al médico y al repostero les cuesta 25 negociar
un acuerdo privado entre ellos. Para que la negociación sea una alternativa que valga la pena,
deben estar en posición de compartir este costo de manera que estén mejor que si no nego-
ciaran.

Si el repostero tuviera una responsabilidad civil por los perjuicios del ruido, instalará el aislante, por-
que su mejor alternativa es pagar al médico una indemnización de 40 por los daños4 y la instalación
del aislante le cuesta nada más 20. Como la responsabilidad civil es un motivo para que el repostero

4
Para que el repostero opere y pague los daños del ruido al médico, no es necesario que incurran en el costo de negociar un acuerdo
privado.

16_CHAPTER 16.indd 540 3/7/09 10:45:29 AM


naturaleza reCíProCa de laS externalidadeS 541

instale el aislante por su cuenta, no es necesario que negocie un acuerdo con el médico y, así, no tiene
que incurrir en los costos de la negociación.
Pero ahora suponga que el repostero no enfrenta ninguna responsabilidad civil por los daños del
ruido. Si la negociación no implicara ningún costo, el médico pagaría P al repostero, dado que 20 ≤
P ≤ 40, para instalar el aislante. Pero si negociar cuesta 25, ya no es posible que el médico compense
al repostero. Con el aislante el médico gana 40, que es insuficiente para pagar su precio (20) más el
costo de negociar el acuerdo (25), lo cual da en total 45. Cuando es caro negociar, ya no se llega a
un resultado eficiente independiente de qué régimen legal se escoja. En este ejemplo, cuyos datos
importantes se resumen en la tabla 16.5, se obtiene el resultado más eficiente sólo si el repostero
tiene una responsabilidad civil.

TABLA 16.5
Resumen de los resultados y ganancias del ejemplo 16.5

Beneficio neto la ganancia para el


Régimen legal Resultado Médico Repostero Total repostero por operar
sin aislante es de 60. el
responsabilidad el repostero paga la 40 40 80 aislante cuesta 20. la
instalación del aislante
pérdida del médico por el
Sin el repostero no instala el 0 60 60 ruido del repostero es de
responsabilidad aislante; el médico cierra 40. el costo de negociar
un acuerdo privado es de
25. el resultado eficaz es
que el repostero instale el
aislante, pero esto ocurre
sólo cuando tiene una
responsabilidad civil por
los daños del ruido.

En el ejemplo 16.5, la ganancia total para el conjunto de la sociedad es de 80 si el repostero tiene


una responsabilidad civil y de 60 si no la tiene. Pero como se ilustra en el ejemplo siguiente, la exis-
tencia de barreras para la negociación no garantiza que haya siempre un resultado eficaz por hacer a
las partes responsables de los daños causados por los efectos externos.

Igual que en el ejemplo 16.5, salvo que el repostero ya no tiene la opción del aislante; en EJEMPLO 16.6
cambio el médico tiene la alternativa de evitar el ruido remodelando su consultorio, lo que
le cuesta 18.
Si el repostero no tiene ninguna responsabilidad civil por los daños causados por el ruido, el médico
hace la remodelación. Pero si el repostero es responsable, el costo de la negociación estorba el pago
al médico por remodelar su oficina. La suma de los costos de negociar (25) y los costos de remodelar
(18) da 43, que es tres más de los 40 que se ahorrarían por evitar el ruido. Entonces, si el repostero
tiene una responsabilidad civil, su mejor opción es seguir operando y pagar al médico 40 por los
daños del ruido.5 En este caso, a diferencia del ejemplo 16.5, se tiene el resultado eficiente cuando
el repostero no tiene ninguna responsabilidad civil. En la tabla 16.6 se resumen los datos del ejem-
plo 16.6.

5
De nuevo, pagar una indemnización no requiere que las partes incurran en los costos de la negociación.

16_CHAPTER 16.indd 541 3/7/09 10:45:30 AM


542 CaPítulo 16 externalidadeS, dereChoS de ProPiedad y el teorema de CoaSe

TABLA 16.6
Resumen de los resultados y ganancias del ejemplo 16.6

Beneficio neto la ganancia para el


repostero por operar
Régimen legal Resultado Médico Repostero Total
es de 60. la pérdida del
responsabilidad el repostero opera y paga 40 20 60 médico por el ruido del
al médico 40 por los daños repostero es de 40. el
del ruido médico puede librarse
Sin el médico paga la 22 60 82 del ruido si remodela su
responsabilidad remodelación de su consultorio a un costo de
consultorio 18. el costo de negociar un
acuerdo privado es de 25.
el resultado eficiente es
que el médico remodele
su consultorio, pero
esto ocurre sólo cuando
el repostero no tiene
responsabilidad civil por
los daños del ruido.

EJERCICIO 16.1
¿Cómo se verían afectadas las entradas en la tabla 16.6 si el costo de la negociación fuera 25 en
lugar de 20?

APLICACIÓN: EFECTOS EXTERNOS DE LAS


PLANTAS DE ENERGÍA NUCLEAR
Aunque desde 1978 Austria tiene una ley que prohíbe las plantas de energía nuclear, está rodeada por
países en los que operan 41 plantas. Dos de ellas, situadas apenas a 35 millas (56 km) de la frontera
con Eslovaquia, comparten características de diseño similares a la planta de Chernobyl que en 1986
sufrió el peor accidente nuclear de la historia. Así, es comprensible que los austriacos se sientan pre-
ocupados por su vulnerabilidad ante un percance semejante.
En una aplicación notablemente audaz de la línea de razonamiento propuesta por Coase, en
enero de 1991 las autoridades austriacas ofrecieron a Eslovaquia (que entonces era parte de Checos-
lovaquia) energía eléctrica gratuita como incentivo para cerrar los dos reactores de diseño soviético.6
El ministro de economía de Austria, Wolfgang Scheussel, calculó que el costo de sustituir la energía
sería de alrededor de 350 millones de dólares por año.
La primera ministra checa Marian Calfa manifestó su interés en la oferta austriaca y se compro-
metió a que un grupo de trabajo la estudiara, pero nunca se llegó a un acuerdo. Como lo ejemplifica
esta experiencia, a veces los costos de una negociación obstaculizan incluso para llegar a acuerdos
que beneficiarían sustancialmente a las dos partes.
La observación de Coase de que las personas llegarán a resultados eficaces si pueden negociar
sin costo tiene múltiples aplicaciones. En efecto, en muchos casos los costos de negociación son me-
nores en relación con los beneficios de llegar a acuerdos sobre las externalidades. Pero las implica-
ciones de mayor alcance de la obra de Coase estriban en el esquema que se muestra en los ejemplos
16.5 y 16.6, en los que se ven las semillas de una poderosa teoría de las instituciones legales y sociales.
Reducida a su esencia, es posible postular la teoría con la regla siguiente:

6
Véase Michael Z. Wise, “Prague Offered Payoff to Shut Nuclear Plant”, en The Washington Post, 30 de enero de 1991.

16_CHAPTER 16.indd 542 3/7/09 10:45:30 AM


dereChoS de ProPiedad 543

Las instituciones legales y sociales eficaces son las que adjudican la carga del ajuste a las
externalidades sobre quienes pueden realizarla al menor costo.

Una de las consecuencias inmediatas de la regla es que no es posible identificar las mejores leyes
con respecto a los efectos nocivos si no se sabe algo sobre cuánto les cuesta a las partes evitarlos. Si,
como en el ejemplo 16.5, quien genera el ruido tiene costos menores, se tiene un resultado eficaz si
se le hace responsable de los daños. Pero si a la persona afectada por el ruido le cuesta menos evitarlo,
como en el ejemplo 16.6, lo mejor es que quien hace el ruido no sea responsable.
La regla de la eficiencia tiene aplicación en una gran variedad de situaciones. Se estudiarán va-
rias en las secciones que siguen.

DERECHOS DE PROPIEDAD
LEYES DE PROPIEDAD PRIVADA Y SUS EXCEPCIONES
Ninguna economía de mercado libre puede operar bien sin leyes que rijan el uso de la propiedad
privada. Entre otras cosas, estas leyes describen cómo puede la gente adquirir de manera legítima
diversas propiedades: por herencia, compra o regalo, pero no mediante robo ni otros medios que
entrañen el uso de la fuerza. En la mayor parte de los casos las leyes confieren a los propietarios el de-
recho de impedir que otros usen su propiedad sin permiso. Sin embargo, un cúmulo de excepciones
detalladas limitan notablemente los derechos de exclusión. Como se aprecia en los siguientes ejem-
plos de El naturalista económico, estas excepciones no son aleatorias, sino que siguen un esquema
sistemático que las ideas de Coase ayudan a entender.

¿Por qué la ley permite que las líneas aéreas vuelen sobre propiedad privada sin autorización?
Piense en el análisis con el que empezó el capítulo sobre los derechos de usar el espacio aéreo de
parcelas de terreno. Para que un arquitecto construya un hotel en el espacio aéreo de mis tierras,
primero debe tener mi autorización, que le daré sólo si, a cambio, me da un pago sustancial. Pero EL NATURALISTA
incluso después de los ataques terroristas del 11 de septiembre de 2001, la ley permite a las aerolíneas ECONÓMICO
comerciales volar sobre mis tierras cuanto quieran sin hacer ningún pago. ¿A qué obedece esta dis-
tinción? 16.1
Para empezar, observe que cada caso se relaciona con una
externalidad: en el primero, la mala vista y el inconveniente de
tener un hotel encima de la cabeza; en el segundo, el ruido y el
posible peligro de los aviones. El costo para mí de la primera ex-
ternalidad es mucho mayor que el segundo, pero eso por sí solo
no explica por qué se tratan los dos casos de manera distinta, ya
que los beneficios que saca el promotor inmobiliario por levantar
un hotel sobre mi terreno también es posible que sean cuantiosos.
La distinción crucial es que las negociaciones individuales son
mucho más prácticas en el caso del promotor inmobiliario que en
el caso de las aerolíneas. En el primer caso sólo hay dos partes y es
factible que los beneficios de un resultado eficaz sean lo bastante
grandes para justificar los costos de negociar. Entonces, en este
caso, es posible sentirse confiados en alcanzar un resultado eficaz
la mayor parte de las veces si se definen los derechos de propiedad
de modo que se impida a los promotores inmobiliarios construir ¿Debe permitirse a la aviación
en el espacio que está sobre la casa de alguien. En cambio, en el comercial volar sobre tierras
caso de las aerolíneas, los beneficios de volar sobre cualquier casa privadas sin autorización del
son pequeños y, de todos modos, el costo de negociar con todas dueño?
las partes potencialmente afectadas sería prohibitivo. Como los
beneficios totales de los vuelos son grandes en relación con los costos totales impuestos a los pro-
pietarios, en este caso se alcanza un resultado eficiente si los derechos de propiedad impiden que los
propietarios prohíban a los aviones volar sobre su cabeza.

16_CHAPTER 16.indd 543 3/7/09 10:45:32 AM


544 CaPítulo 16 externalidadeS, dereChoS de ProPiedad y el teorema de CoaSe

Sin embargo, hay excepciones a este principio general, las cuales también dan un ejemplo ins-
tructivo de la regla de eficiencia de Coase. La excepción más notable concierne a las rutas de aterri-
zaje y despegue de aeropuertos cercanos a zonas metropolitanas importantes. Los aviones vuelan
cerca del suelo cuando acaban de despegar y cuando están por aterrizar, y el ruido que se percibe en
las propiedades que están abajo es ensordecedor. En estas situaciones las normas locales prohíben
aterrizajes y despegues durante las horas en que a los propietarios les cuesta más adaptarse al ruido,
a saber, las horas en que casi todos están dormidos. Tampoco en este caso es práctico negociar y lo
mejor que se puede hacer es definir derechos para alcanzar la solución de menor costo.

¿Por qué la ley de allanamiento no se aplica a la propiedad costera?


En muchas culturas del mundo la gente considera que si un desconocido pasa por sus tierras, es un
intruso. El allanador —en la jerga del economista— confiere una exterioridad negativa al propieta-
rio. Estas externalidades se enfrentan de varias maneras. Por ejemplo, la mayoría de mis vecinos han
EL NATURALISTA cercado su patio para que la gente no tome atajos cruzando por su propiedad. Algunos hasta ponen
ECONÓMICO letreros advirtiendo que se tenga cuidado con perros violentos.
16.2 En muchas jurisdicciones es del todo legal tomar esas medidas
para excluir a los demás del uso de una propiedad. Pero las leyes
de mi comunidad no conceden los mismos derechos a quienes
tienen una casita en la orilla del cercano lago Cayuga. Por el con-
trario, permiten explícitamente que cualquier ciudadano camine
por cualquier parcela de tierra situada en la orilla del lago.
Esta distinción no se debe a que los propietarios de terrenos
en la orilla del lago estimen menos su intimidad que los otros.
Más bien, es porque el costo de no poder cruzar por las tierras de
alguien situadas en la orilla del lago es mucho mayor que en otras
partes. Para ilustrarlo, suponga que A, B y C en la figura 16.1 son
tres propiedades a la orilla del lago y que alguien que está en A
quiere visitar a otra persona que está en C. El acceso a las pro-
piedades por tierra significa pasar de la autopista a un sendero
¿debe permitirse el paso sin largo, inclinado y a veces traicionero. Si no tuviera la posibilidad
autorización por propiedad de cruzar la propiedad B, la persona de A tendría que subir por su
privada situada en una ribera? camino hasta la carretera principal, viajar hasta el sendero de C y
recorrerlo. Como los costos de esta ruta más larga son mucho
mayores que la ruta directa por el lago, la ley contra la invasión de propiedad hace una excepción
con estas propiedades. Los dueños consideran que una molestia ocasional inesperada es un precio
pequeño por la conveniencia de moverse libremente por la orilla del lago.

FIGURA 16.1
Propiedad a la orilla
del lago y ley de
allanamiento
el costo de ir de A a C
sin cruzar la propiedad B C
es mucho mayor que la B
ruta directa por la orilla.
Por este motivo la ley no A
permite a quienes tienen
propiedades a la orilla del
lago que impidan a otras
personas cruzar sus tierras.

16_CHAPTER 16.indd 544 3/7/09 10:45:34 AM


dereChoS de ProPiedad 545

En cambio, el derecho de pasar por la propiedad de alguien en mi vecindario valdría más bien
poco. Las calles están cercanas, así que siempre es fácil ir adonde uno quiera sin tener que tomar
atajos. Con respecto a la posibilidad de allanamientos en la orilla de los lagos y en otras propiedades,
las negociaciones son prohibitivamente caras una por una, así que la ley de propiedad define los
derechos de acceso de manera que, en promedio, lleva a un resultado eficiente. Da a la mayoría de
los propietarios el derecho de exclusión, pero lo retira a los propietarios de fincas a la orilla del lago.

¿Por qué las leyes de propiedad se suspenden durante las tormentas?


El 13 de noviembre de 1804 la familia Ploof salió a navegar en el lago Champlain de Vermont. Es-
talló de pronto una violenta tormenta que les impidió volver a su
EL NATURALISTA
puerto de partida. Desesperados buscaron refugio y atracaron en
un muelle de una isla del lago. El muelle era propiedad de un tal ECONÓMICO
Putnam, que envió a un sirviente para que arrojara a los Ploof de su
propiedad. Éstos soltaron amarras en la tormenta y en poco tiempo
16.3
su balandro se destruyó y varios miembros de la familia resultaron
heridos. Después los Ploof ganaron una demanda contra Putnam.
El tribunal falló que aunque de ordinario Putnam tenía derecho
a impedir que usaran su muelle, las circunstancias de la tormenta
crearon una excepción. Observe que al dictaminar de esta manera,
el tribunal de Vermont imitaba un resultado al que llegarían los due-
ños de muelles y los dueños de barcos si pudieran negociar sin costo ¿el dueño de una propiedad
y libremente durante una tormenta. El valor del muelle para un siempre tiene derecho a
barco en problemas es mucho mayor que el valor para el propietario evitar que los desconocidos
de poder vedarlo, y el tribunal de Vermont interpretó las leyes de usen su muelle?
propiedad del Estado con esta observación en mente.

¿Por qué los límites de altura para las edificaciones varían según la ciudad?
Considere la situación que se retrata en la figura 16.2. El residente A tiene una casa en una colina
que mira al mar y concede mucho valor a poder mirar el ocaso por la ventana de su sala. Entonces,
B compra la propiedad de abajo de A y pondera cuál de los dos casas construir. La primera es de
EL NATURALISTA
una planta y dejaría intacta la vista de A. La segunda es un proyecto de dos pisos que obstaculizaría ECONÓMICO
por completo la vista de A. Suponga que la ganancia de A por tener una vista despejada es 100, la
ganancia de B por tener una casa de un piso es 200 y la ganancia de B por tener una casa de dos pisos
16.4
es 280. Si las leyes de propiedad permiten que la gente construya a cualquier altura que desee y si la
negociación entre los propietarios no costara, ¿cuál de las dos casas construiría B?

FIGURA 16.2
Valor de una vista
despejada
la solución eficiente de B
es construir una casa de
A dos pisos si, y sólo si, el
valor adicional para B de
la casa más alta es mayor
que el valor para A de
B conservar su vista.

16_CHAPTER 16.indd 545 3/7/09 10:45:37 AM


546 CaPítulo 16 externalidadeS, dereChoS de ProPiedad y el teorema de CoaSe

Para responder la pregunta, observe primero que el aumento


de la ganancia de B por tener una casa más alta es de 80, que es 20
menos que el costo de A por la pérdida de la vista. Así, el resul-
tado eficiente es que B construya la casa de una planta, y eso es
exactamente lo que pasaría si las dos partes negociaran sin costo.
En lugar de esperar a que B construya la casa más alta, A podría
compensar a B por escoger el proyecto más bajo. Así, tendría que
dar a B por lo menos 80, porque a eso renuncia B por no tener la
casa de dos pisos. Lo máximo que A estaría dispuesto a dar a B es
100, puesto que es todo lo que vale la vista para él. Para cualquier
pago P, en el que 80 ≤ P ≤ 100, A conservará su vista.
Pero suponga que no fuera práctico que negociaran las dos
¿Por qué las leyes de partes. Entonces, B insistiría en construir la casa de dos plantas,
urbanización son más estrictas puesto que es el proyecto que más valora. Comparado con el
en San Francisco que en la proyecto de una planta, B ganaría 80, pero A perdería 100. La
mayor parte de las ciudades del estructura óptima de los derechos de propiedad en este ejemplo
medio oeste estadounidense? en particular sería prohibir que cualquier construcción bloquee la
vista de algún vecino.
Por supuesto, se llegaría a una conclusión distinta si las valoraciones asignadas por las partes
fueran diferentes. Si, por ejemplo, B valorara la casa de dos pisos en 300 y la vista de A fuera estimada
otra vez en 100, la estructura óptima de derechos de propiedad sería permitir que la gente construya
a la altura que desee. En cualquier caso, la estructura óptima de derechos de propiedad es la que
pone la carga del ajuste (la pérdida de la vista o la pérdida del proyecto de edificación preferido) de
parte de quien la pueda llevar al costo más bajo.
Como cuestión práctica, las leyes de propiedad en muchas jurisdicciones incorporan precisa-
mente este principio. En ciudades como San Francisco, donde las vistas al mar y la bahía son asom-
brosamente bellas, estrictas leyes de urbanización regulan las construcciones que estorban la línea
de visión de las edificaciones. Las leyes de urbanización en ciudades donde hay menos que ver son
mucho más liberales en cuanto a la clase de edificaciones que permiten. Pero incluso en ciudades que
no tienen ningún paisaje especial que proteger, las leyes de urbanización limitan la fracción del lote
que puede ser ocupado por estructuras artificiales. La mayoría de las personas aprecian por lo menos
algo de luz diurna y estas ordenanzas hacen posible que la tengan.

La mayoría de las personas que han crecido en economías de mercado como Estados Unidos
dan por sentadas las leyes de propiedad privada. Pero como lo han hecho notar los ejemplos anterio-
res, los detalles de las diversas leyes de propiedad tienen un alto grado de estructura económica. Ellas
incorporan complicados cálculos, con frecuencia implícitos, acerca de cómo alcanzar las soluciones
más eficientes para problemas prácticos que involucran externalidades. Sin duda, como lo ejemplifi-
ca la siguiente sección, la mera existencia de propiedad privada se puede rastrear hasta los primeros
intentos de enfrentar las externalidades.

LA TRAGEDIA DE LOS BIENES COMUNES


Para explorar los orígenes de la institución de la propiedad privada es ilustrativo considerar, como
en el ejemplo siguiente, lo que ocurriría en una sociedad en la que no hubiera una institución bien
establecida de derechos de propiedad.

EJEMPLO 16.7 Una aldea tiene seis pobladores y cada uno posee una riqueza de 100. Cada habitante puede
invertir su dinero en un bono del gobierno que paga 12% anual o destinarlo a comprar un
novillo de un año que pacerá en los pastizales comunes de la aldea (donde no hay tierras de
pastoreo de propiedad individual). Los novillos de un año y los bonos del gobierno se venden

16_CHAPTER 16.indd 546 3/7/09 10:45:39 AM


dereChoS de ProPiedad 547

en exactamente 100. No cuesta ningún esfuerzo atender a los novillos y se venden a un precio
que depende del peso que ganen durante el año. A su vez, la ganancia de peso anual depende
del número de novillos que pasten en los terrenos comunes. En la tabla 16.7 se dan los precios
de novillos de dos años en función del número total de animales. Si los residentes de la aldea
toman sus decisiones de inversión de manera independiente, ¿cuántos novillos pastarán en
los prados comunes?

TABLA 16.7
Precios de los novillos en función de la densidad de pastoreo

Número de novillos Precio por novillo de dos años a medida que pacen más
1 120 novillos en los pastos
2 118 comunes, cada uno
engorda menos, lo que da
3 114
por resultado que baje el
4 111 precio de cada uno.
5 108
6 105

Como los aldeanos no pueden controlar individualmente el acceso a los pastos comunes del ganado
de los demás, la estrategia que maximice el ingreso consistirá en llevar un novillo más a los pastizales
si, y sólo si, su precio el año entrante es de por lo menos 112 (a ese precio, la ganancia de tener un
novillo es igual a la ganancia de comprar un bono). Con estos cálculos se llevarán tres novillos a los
pastos comunes y el resto del dinero de los aldeanos se invertirá en bonos del gobierno. Con este
esquema el ingreso de la aldea por la inversión será de 14 por cada novillo y de 12 por los tres bonos,
para un total de 78.
Sin embargo, observe que no es el ingreso más grande que los aldeanos podrían haber ganado.
Desde el punto de vista de la aldea como un todo, la regla de inversión de los novillos debe ser: llevar
a pastar un novillo más si, y sólo si, su aportación marginal al valor del rebaño total después de un
año es mayor o igual que 112. Enviar el tercer novillo a los pastos comunes dio por resultado un
rebaño con valor de 3 × 114 = 342, que es apenas 106 más que el valor de un rebaño con sólo dos
novillos (2 × 118 = 236). El ingreso total de la aldea se maximiza comprando cuatro bonos y llevando
dos novillos a los pastos comunes. Este esquema da por resultado un ingreso de 48 por los bonos y 36
por los novillos, para un total de 84.

La razón por la cual la mano invisible no produce el mejor resultado social en este caso es que los
aldeanos ignoraron una externalidad importante. Su criterio para decidir si debían llevar otro novillo
a pastar fue considerar únicamente el aumento de precio de ese novillo en particular. No tomaron
en cuenta el hecho de que un novillo más haría que el ganado engordara menos. Los pastizales son
un recurso escaso en este ejemplo y los aldeanos no lo asignaron de manera eficiente porque podían
usarlo sin costo.
El problema se habría resuelto si los aldeanos hubieran podido tener pastizales propios de los
que hubieran podido excluir a los demás. Por ejemplo, suponga que el gobierno de la aldea decidiera
vender en subasta pública los pastizales. ¿Qué precio alcanzarían? Quien compre los pastizales tiene
derecho a restringir el número de novillos a dos, que es el monto de máximo ingreso. Ya se vio que
aprovechada de esta manera, la tierra generaría un ingreso anual de 36 a partir de una inversión
anual de 200 (el precio de dos novillos). Si mejor se hubieran destinado los 200 a la compra de bonos

16_CHAPTER 16.indd 547 3/7/09 10:45:39 AM


548 CaPítulo 16 externalidadeS, dereChoS de ProPiedad y el teorema de CoaSe

del gobierno, sólo se hubieran ganado 24. Así, tener el control sobre los pastos comunes rinde un
excedente de 12 por año sobre el ingreso al alcance de una persona que sólo pudiera comprar bonos
del gobierno. Se deduce que el precio de los pastizales en la subasta será de 100 (el precio de un bono
que paga un ingreso de 12 por año). Si el precio de los pastizales fuera menor que 100, todos los
inversionistas preferirían comprarlos en lugar de los bonos del gobierno. Si se venden en más de 100,
todos harían mejor en comprar los bonos. El gobierno de la aldea tomaría los 100 recaudados en la
subasta de los pastizales y los distribuiría entre los seis pobladores, que tendrían un pago promedio
de 100
6.

EJERCICIO 16.2
¿Qué cuota de pastoreo resolvería el problema de las tierras comunes que se estudió en el
ejemplo 16.7?

En las sociedades antiguas era práctica general que recursos importantes como pastizales y pes-
querías fueran de propiedad común. La dificultad de estos esquemas de propiedad estribaba en que
llevaban a la sobreexplotación del recurso. En la figura 16.3 se ilustra el problema de los aldeanos
que tienen la opción de trabajar en una fábrica a un salario de W/día o quedarse con toda la pes-
ca que puedan capturar en el lago de la aldea. La curva marcada AP muestra que la captura prome-
dio varía con el número de pescadores, mientras que MP muestra el cambio en la captura total como
función del número de pescadores. Si los pescadores se quedan con todo lo que capturan, la regla
de decisión será pescar hasta X', el punto donde AP = W. En X', el valor de la captura total es igual
exactamente al ingreso total que habrían ganado los aldeanos que pescaron si hubieran trabajado en
la fábrica.
La asignación óptima para la sociedad es pescar sólo hasta X* en la figura 16.3, el punto donde
W = MP, y que todos los demás aldeanos trabajen en la fábrica. Con esta asignación los aldeanos que
pescan ganarán un total de S* (el área sombreada) más de lo que hubieran ganado trabajando en la
fábrica.

FIGURA 16.3
La tragedia de los
bienes comunes
Cuando un recurso, como
una pesquería o pastizales,
es propiedad común,
cada usuario conserva el Producción por unidad de X
producto promedio de los
insumos de producción
propios que aplica al
Excedente perdido
recurso. los insumos
de propiedad privada se
aplicarán al recurso hasta AP*
S*
X', el punto donde el W
AP
producto promedio es igual Insumos de
a su costo de oportunidad, propiedad
W, lo que da por resultado privada
un excedente económico aplicados
de cero. la asignación X
0 X* X’ a un recurso
óptima para la sociedad de propiedad
es X*, el nivel de insumos común
MP
en el que W es igual al
producto marginal de los
insumos de propiedad
privada y da por resultado
una excedente económico
de S*.

16_CHAPTER 16.indd 548 3/7/09 10:45:40 AM


dereChoS de ProPiedad 549

Si los aldeanos tienen acceso libre a la pesquería del lago, la asignación que envía X* a pescar no
será estable. Como cada pescador ganará más que los aldeanos que trabajan en la fábrica, los obreros
tendrán un incentivo para dedicarse a pescar. El cambio se detendrá cuando X' se hayan ido de pesca
y las ganancias de las dos alternativas sean iguales. Como en el ejemplo anterior de los pastizales, los
pescadores adicionales ignoran la externalidad que les imponen a los pescadores anteriores. Cada
uno mira en exclusiva al tamaño de su captura y omite el hecho de que su presencia reduce la captu-
ra de todos los demás.
Para sostener una distribución eficiente, hay que hacer algo para limitar el acceso al lago. El
método más simple es cobrar a la gente por el derecho a ir de pesca. Si la cuota de pesca se fija en
AP* – W (véase la figura 16.3), la asignación óptima resultaría automáticamente de las decisiones de
maximizar el ingreso tomadas por los aldeanos en lo individual. En este caso, como en el ejemplo
de los pastizales, el problema es que los individuos explotan en demasía el recurso productivo si se
les permite aprovecharlo libremente. El mecanismo de la mano invisible funciona bien sólo cuando
todos los recursos se venden a precios que reflejan su verdadero valor económico.
Una de las causas constantes de ineficiencia en las economías modernas involucra la asignación
de recursos que las leyes de propiedad de ninguna nación sola pueden regir. Por ejemplo, varias es-
pecies de ballenas se han capturado en tal cantidad que casi se extinguen, porque no hay leyes inter-
nacionales de propiedad que restrinjan los incentivos que tienen los particulares para cazar ballenas.
Desde hace mucho el Mediterráneo tiene problemas graves de contaminación porque ninguna de
las muchas naciones que lo bordean tiene un incentivo económico para considerar los efectos de las
descargas de otros países. A medida que la población mundial aumenta, la falta de un sistema eficaz
de derechos de propiedad internacional será un problema económico de importancia creciente.
Un caso importante es la tendencia al calentamiento de la atmósfera. Los científicos estiman que
si el dióxido de carbono y otros gases de invernadero se siguen acumulando en la atmósfera al ritmo
actual, la temperatura promedio de la Tierra aumentará hasta 8°F (13°C) en este siglo, suficiente
para derretir las capas de hielo polar e inundar miles de millas cuadradas de tierras costeras. Si una
única entidad tuviera el poder de promulgar leyes ambientales a las que se sujetara todo el mundo,

©the new yorker Collection 1999 mick Stevens from cartoonbank.com. all rights reserved.

“Caballeros, es hora de pensar seriamente


en los efectos del calentamiento global.”

16_CHAPTER 16.indd 549 3/7/09 10:45:41 AM


550 CaPítulo 16 externalidadeS, dereChoS de ProPiedad y el teorema de CoaSe

sería una cuestión sencilla, aunque cara, reducir la acumulación de gases de invernadero. Pero en
nuestro mundo de naciones soberanas no existe ese poder.

EXTERNALIDADES, EFICIENCIA
Y LIBERTAD DE EXPRESIÓN
Como se ilustra en la exposición que sigue, los principios de eficiencia de Coase se aplican no sólo a
la determinación de derechos de propiedad, sino también a la redacción de constituciones. En par-
ticular, arrojan luces sobre la medida en que la sociedad tiene interés en proteger el derecho a la
libertad de expresión.
La Primera Enmienda de la Constitución estadounidense protege la mayoría de las formas de
expresión, incluso las que tienen efectos muy dolorosos en los demás. Una vez un hombre escribió al
columnista de un periódico para confesar un acto cruel que había cometido décadas atrás, en su últi-
mo año de preparatoria. Él y sus amigos habían hojeado el anuario de la escuela y habían tomado la
foto de la muchacha que les pareció la más fea de su generación. Entonces, el autor de la carta llamó
a la muchacha para felicitarla por su elección. Durante los años que siguieron nunca pudo olvidar la
amarga queja que ella le devolvió como respuesta. El hombre lo daría todo —dijo— por retrasar el
tiempo y retractarse de esa llamada de teléfono.
Dada la elección entre recibir una llamada así o recibir un golpe fuerte con un palo en el brazo,
muchas personas escogen de inmediato lo segundo. Si los muchachos le hubieran dado un palazo a la
chica, habrían ido a la cárcel. Pero hacer la llamada telefónica estaba perfectamente al amparo de los
derechos que les confería la Primera Enmienda estadounidense.
¿Por qué la constitución prohíbe una forma de daño pero no la otra? En el marco teórico de
Coase, lo primero que hay que entender es que es poco práctico negociar las soluciones caso por
caso con respecto a cualquiera de las formas de daño. Imposible imaginar que los muchachos y la
chica regatearan cuánto estaría dispuesta a pagar ella por evitar oír un comentario doloroso o, en
el caso de ellos, por evitar recibir un palazo. Por consiguiente, la estructura de la ley debe guiarse
por un juicio sobre qué estructura de derechos generará el mejor resultado cuando es poco práctico
negociar caso por caso.
Sin duda, la mayoría de las personas estaría de acuerdo en que el mundo sería mejor si se pro-
hibieran libertades de expresión como la broma telefónica. La pregunta práctica es si es posible es-
tructurar una ley que evite estas expresiones sin prevenir otras que se valoran mucho. Tristemente, la
respuesta parece ser no. Cualquier ley que prohibiera a las personas hacer comentarios crueles sobre
los demás posiblemente también obstaculizaría muchas formas de expresión de gran valor. El miedo
a la crítica mantiene a raya a personas que de otra manera tendrían un comportamiento caprichoso
y eso beneficia a la sociedad. Si fuera práctico redactar una ley que permitiera las críticas justificadas
pero prohibiera las injustificadas o simplemente las descorteses, todos se sentirían muy tentados a
aplicarla. Pero hasta ahora, a nadie se le ha ocurrido tal ley.
Con todo, la protección de la libertad de expresión que concede la Primera Enmienda en Estados
Unidos está lejos de ser absoluta. Por ejemplo, no protege el derecho de una persona a gritar “¡fuego”!
en un cine atestado. Tampoco permite proferir groserías en las esquinas de las calles ni está permitido
propugnar el derrocamiento del gobierno por medios violentos. En estos casos, se acepta decir que
los beneficios de la libertad de expresión son demasiado pocos para justificar sus costos externos.

REGLAS PARA FUMAR, PÚBLICAS Y PRIVADAS


En estudios de investigación se muestra que la exposición al humo del cigarro exhalado por otros
puede ser nociva para la salud. Estos resultados han prestado considerable apoyo a la tendencia re-
ciente a promulgar leyes que prohíban fumar en lugares públicos. Sobre las premisas verosímiles de
que 1) en general no es práctico negociar con desconocidos en lugares públicos, y 2) el daño a los no
fumadores por la exposición indeseada al humo es más importante que el daño a los fumadores por
no poder expeler humo en lugares públicos, esta ley tiene sentido según el marco teórico de Coase.
Sin embargo, hasta ahora no se ha propuesto ninguna ley que prohíba fumar en viviendas par-
ticulares. El resultado es que a veces las personas están expuestas al humo de sus compañeros de

16_CHAPTER 16.indd 550 3/7/09 10:45:42 AM


reglaS Para Fumar, PúbliCaS y PriVadaS 551

habitación. Con la premisa verosímil de que los costos de negociación con posibles compañeros son
bajos, el ejemplo siguiente ilustra que es poco probable que la falta de tales leyes lleve a un resultado
indeseable.

Smith y Jones intentan decidir si comparten un departamento de dos recámaras o viven por EJEMPLO 16.8
separado en departamentos de una recámara. Las rentas son de 300 dls/mes por el depar-
tamento de una recámara y de 420 dls./mes, 210 dólares cada uno, por un departamento de
dos recámaras. Smith fuma y está dispuesto a ceder 250 dls/mes con tal de poder fumar en
su casa. Por otro lado, Jones no fuma y prefiere sacrificar 150 dls./mes antes que vivir con un
fumador. Además de los aspectos de tabaquismo y la renta, a los dos les da lo mismo vivir
solos. Ninguno tiene la alternativa de elegir otro compañero de cuarto. ¿Vivirán juntos o
separados?

Si viven por separado, cada uno puede hacer lo que quiera en cuanto al cigarro. La desventaja es
que vivir solos es más costoso. Si viven juntos, ahorrarán en la renta, pero uno de ellos tendrá que
comprometerse. Es decir, o Smith deja de fumar o Jones tolera el humo de Smith. Si alguno llega a
hacer un compromiso, sería Jones, porque está dispuesto a pagar menos que Smith. Si comparten
un departamento, cada uno se ahorra 90 dls/mes de renta. Si no se toma en cuenta la posibilidad de
negociación no vivirán juntos porque este ahorro es menor que el costo para Jones de tener que vivir
con un fumador.
Pero suponga que pueden negociar sin que les cueste. La pregunta práctica entonces es si el
ahorro total en la renta justifica el costo del compromiso para Jones. El ahorro total de la renta es
180 dls/mes, que es la diferencia entre el total de 600 dls/mes que pagarían si vivieran solos y los 420
dólares que desembolsarían por vivir juntos. Y como el ahorro supera por 30 dls/mes el costo para
Jones, deberían poder llegar a un acuerdo en el que los dos prefieran vivir juntos. Smith tendrá que
dar parte de su ahorro de 90 dls/mes a Jones.
Sea X la cantidad que Smith le da a Jones. Como el costo para Jones por vivir con un fumador es
de 150 dls/mes y su ahorro en la renta es de sólo 90 dls/mes, X debe ser por lo menos de 60 dls/mes.
Como Smith sigue fumando en el acuerdo para compartir el departamento, su ahorro de 90 dls/mes
en la renta es ganancia pura, lo cual significa que 90 dls/mes es el valor más alto posible para X. En la
tabla 16.8 se presenta el resumen de los detalles relevantes de este ejemplo.

TABLA 16.8
Resumen del pago del ejemplo 16.8

Pago neto de la renta Para Smith, el costo de


(dls./mes) Ganancia neta (dls./mes) no fumar es de 250 dls/
mes. Para Jones, el costo de
Jones Smith Jones Smith Total
vivir con un fumador
Vivir por separado 300 300 ––– ––– ––– es de 150 dls/mes. el
ahorro total de la renta
Vivir juntos; Smith le 210 – X 210 + X X – 60 90 – X 30 por vivir juntos es de 600
paga a Jones X para dls/mes – 420 dls/mes
compensarlo por = 180 dls/mes, que son
fumar. 60 ≤ X ≤ 90 30 dls/mes más que el
compromiso menos
costoso requerido
por compartir un
departamento, que
representa los 150 dls/mes
que le cuesta a Jones vivir
con un fumador.

16_CHAPTER 16.indd 551 3/7/09 10:45:42 AM


552 CaPítulo 16 externalidadeS, dereChoS de ProPiedad y el teorema de CoaSe

El ejemplo 16.8 destaca el punto de que los efectos externos son totalmente recíprocos. El ta-
baquismo de Smith daña a Jones, como enfatizan los análisis tradicionales del problema. Pero negar
a Smith la oportunidad de fumar lo daña, al menos así lo percibe él. En lo que respecta a la cuestión
de compartir un alojamiento, el problema del cigarro es, en esencia, compartido. Como las personas
tienen la libertad de llegar a cualquier acuerdo para vivir juntas, a Jones no se le puede obligar a so-
portar el humo del cigarro contra su voluntad. De igual modo, a Smith no se le puede obligar a dejar
de fumar. Si lo que buscan con el hecho de compartir un departamento es ahorrar, una de las partes
se debe comprometer en cuanto al problema del cigarro y la otra debe hacerlo en el aspecto econó-
mico. A menos que los términos de su acuerdo representen una clara mejoría para ambas partes con
respecto a la alternativa de vivir solos, simplemente no llegarán a un acuerdo.

EJERCICIO 16.3
¿De qué manera serían diferentes las entradas de la tabla 16.8 si hubiera un sistema de escape
que eliminara por completo el daño del humo del cigarro a un costo de 60 dls./mes?

EXTERNALIDADES POSITIVAS
El teorema de Coase no sólo se aplica a las externalidades negativas, sino también a las positivas.
Cuando un apicultor y un cultivador de manzanas operan en propiedades adyacentes, las actividades
de cada uno confieren externalidades positivas al otro. Si el apicultor incluye otro panal, se poliniza-
rán más manzanos de la hortaliza adyacente, garantizando una cosecha más grande. Si el dueño de la
hortaliza siembra más árboles, aumentará la producción de miel del apicultor. Pero si la negociación
entre ellos no representa un costo, el apicultor puede ofrecer al dueño de la hortaliza un subsidio para
sembrar más árboles. Del mismo modo, el dueño de la hortaliza puede ofrecer pagos para inducir
al apicultor a agrandar su apiario. Con las externalidades positivas o negativas ocurren ineficiencias
sólo si es muy costoso o impráctico negociar acuerdos acerca de cómo corregirlos.

EXTERNALIDADES POR POSICIÓN


En muchas actividades las recompensas se determinan no por un desempeño absoluto sino por la
manera en que se actúa con respecto a los demás. Por ejemplo, para que usted sea campeón de
natación lo que cuenta no es qué tan rápido nade en términos absolutos, sino cómo son sus tiempos
comparados con los de otros. El nadador Mark Spitz ganó siete medallas de oro en los juegos olímpi-
cos de 1972, pero incluso sus tiempos ganadores no le hubieran permitido calificar para ser parte del
equipo masculino de natación estadounidense en 2004.
En situaciones en las cuales las recompensas se determinan mediante un desempeño relativo
se suelen denominar competencias. En casi todas ellas los participantes emprenden una variedad de
acciones en un intento por mejorar su posibilidad de ganar. De hecho, la esencia de lo que significa
concursar es llevar a cabo dichas acciones. Algunas de las acciones sólo implican gastos mínimos. Por
ejemplo, los nadadores a veces se rasuran la cabeza y el cuerpo para deslizarse con mayor suavidad
en el agua.
Pero en las competencias donde está en riesgo algo importante, los competidores casi siempre
toman decisiones mucho más costosas para ganar. En la carrera por los puestos políticos nacionales
los competidores gastan cientos de millones de dólares en publicidad. En la carrera por la supremacía
militar los países invierten miles de millones de dólares en el desarrollo y la fabricación de armas
nuevas.
Como las recompensas de las competencias se distribuyen conforme a la posición relativa, las
leyes de la aritmética simple dicen que cualquier acción que aumente la probabilidad de ganar de
un competidor, necesariamente debe reducir las oportunidades de otros. Si se toma en cuenta esta
observación, es instructivo pensar que las acciones que mejoran el desempeño dan origen a externa-

16_CHAPTER 16.indd 552 3/7/09 10:45:43 AM


externalidadeS Por PoSiCión 553

lidades por posición. Si A y B compiten por un premio que sólo puede obtener uno de ellos, lo que le
sirva a A necesariamente perjudica a B.
El resultado es que cuando las probabilidades son altas, las competencias no reguladas casi siem-
pre conducen a costosas carreras armadas por la posición. Por ejemplo, a falta de regulaciones eficaces
sobre los medicamentos, parece que muchos jugadores de la Liga Nacional de Futbol Americano
ahora se sienten obligados a mejorar su tamaño y fuerza mediante el uso de esteroides anabólicos. Es
fácil ver por qué. En un ámbito en el que el volumen físico representa un papel central, es obvio que
no usar estas hormonas peligrosas pondría en riesgo la posición de un jugador en su equipo.
Sin embargo, al igual que muchas otras carreras armadas, la competencia por ser más grande y
fuerte genera pocos beneficios reales para el grupo de competidores en conjunto. Después de todo,
la competencia en la línea de golpeo sólo puede tener un ganador, ya sea que los jugadores del equi-
po pesen en promedio 300 o 240 libras. Al mismo tiempo, la carrera impone costos sustanciales. Los
esteroides anabólicos se relacionan con el cáncer de hígado y otros problemas de salud graves.
En el hockey el procedimiento normal de los defensivos es lanzarse al hielo para evitar que el
tiro de un contrario entre en la portería. En las competencias de hockey de la NCAA esta práctica
rara vez provoca lesiones graves porque ahora los jugadores usan casco con rejilla gruesa sobre la
cara. Pero antes de que aparecieran estos cascos, sin duda era una práctica riesgosa. Cuando un disco
de hockey que viaja a más de 100 millas/h pega en el rostro, los resultados son desagradables. Desde
un punto de vista objetivo, parece bastante absurdo que un jugador se arriesgue a quedar mutilado
lanzándose de cara en la trayectoria del disco; sin embargo, pocos jugadores dudaron en hacerlo
cuando se les presentaba la oportunidad. La urgencia de ganar, de hacerlo bien en términos relati-
vos, es una fuerza poderosa de la naturaleza humana. En situaciones en las que las compensaciones
materiales por ganar son muy grandes esto no es sorprendente. Pero aun cuando las probabilidades
son ostensiblemente bajas, como el ejemplo de un partido de hockey de una preparatoria al norte
de Nueva York que no pertenece a la liga, las personas se arriesgan en gran medida por mejorar sus
posibilidades.
Con lo que está en riesgo, la limitación voluntaria rara vez es una solución eficaz para las ca-
rreras armadas por la posición. Y por lo mismo las autoridades que rigen muchos deportes ahora
requieren a los atletas en competencias pruebas farmacológicas más estrictas. De igual modo, la
regla del casco de la NCAA ha sido una solución para cuidarse la cara, tanto en sentido literal como
figurado; sin ella pocos jugadores se hubieran atrevido a usar una rejilla en la cara. (En la Liga Nacio-
nal de Hockey, que no tiene un requisito así, los cascos con rejilla en el rostro son más raros que los
dientes frontales de la defensiva.)
Como ejemplo de la limitación colectiva de las externalidades de posición, considere la anti-
gua práctica del duelo. Alguna vez un caballero se sintió obligado a defender su honor retando a la
parte ofensora a un duelo con pistolas al amanecer. Los duelistas, y gente que los estimaba, pronto
reconocieron al duelo sin trabas como una práctica inaceptable y costosa. Con el tiempo las reglas
evolucionaron y se redujo el índice de mortalidad. Por ejemplo, la distancia a la que se disparaban
cada vez fue más larga y se prohibieron las pistolas con cañones ranurados en espiral. Con estas
restricciones sólo 1 de cada 16 duelistas recibía el balazo y sólo 1 de cada 14 fallecía. Desde luego que
fue un precio muy alto el que se pagó y, con el tiempo, condujo a una prohibición inmediata de los
duelos. Con sanciones legales firmes, ahora es posible defender el honor de muchas formas menos
perjudiciales.
Una de las competencias más importantes a las que se enfrenta la gente en la vida es la tarea de
garantizar que sus hijos entren en el mercado laboral con una buena educación. Esta misión es una
competencia porque la “buena” educación, al igual que un jugador de futbol americano “eficaz”,
es un concepto ineludiblemente relativo. Si un jugador eficaz es más grande, fuerte y rápido que
la mayoría, una buena educación es aquella que es superior a la que recibe la mayoría. Este aspecto
relativista de nuestro objetivo es lo que nos hace vulnerables a una carrera armada por la posición del
tipo que se ve en otras partes.
En el contexto educativo ¿qué forma adopta esta carrera armada? Como las escuelas públicas
son financiadas principalmente con impuestos locales a la propiedad, la calidad educativa y la cali-
dad de la comunidad están estrechamente relacionadas en los sistemas de escuela pública estado-
unidenses. Por consiguiente, la competencia por la posición con frecuencia implica cambiarse a la

16_CHAPTER 16.indd 553 3/7/09 10:45:43 AM


554 CaPítulo 16 externalidadeS, dereChoS de ProPiedad y el teorema de CoaSe

mejor colonia posible. Es común que las familias soporten muchas dificultades, como trabajar mu-
chas horas, aceptar trabajos riesgosos, no tener vacaciones, escatimar en gastos y así sucesivamente,
para poder juntar el dinero necesario para cambiarse a un mejor distrito escolar. Sin embargo, una
vez más, las leyes de la aritmética simple hacen recordar que no es posible que toda la sociedad
avance en los mismos términos relativos. Sólo 10% de los alumnos pueden ocupar la décima parte
de los lugares de mejor nivel en las escuelas, independientemente de cuánto se esfuerce un pa-
dre de familia.
Incluso en competencias con apuestas muy bajas se ha visto que las personas con frecuencia
aceptan sacrificio y riesgo considerables por mejorar su probabilidad de ganar. La competencia para
colocar bien a los hijos en la vida es una en la que las apuestas son altas. El esfuerzo de cualquier fa-
milia por avanzar en términos relativos impone una externalidad negativa a los demás. En una varie-
dad de contextos se ha visto que las instituciones sociales evolucionan a fin de promover soluciones
eficientes a las externalidades. Con esta visión se pueden tener perspectivas semejantes en varias
instituciones sociales que limitan la carrera armada por la posición entre familias. Se verá que con
frecuencia las explicaciones tradicionales de muchas de estas instituciones generan más preguntas
que las que responden.

LIMITACIÓN DE LA SEMANA LABORAL


La Ley de Normas de Mano de Obra Justa estadounidense requiere, entre otras cosas, que los patro-
nes paguen una prima salarial de 50% cuando la gente trabaje más de 8 horas al día o 40 horas a la
semana. Esta regulación desalienta sobremanera el tiempo extra en el trabajo y se le ha defendido
con base en que, sin ella, los patrones que son monopsonistas requerirían jornadas de trabajo inacep-
tablemente largas.
Los críticos de las leyes sobre el tiempo extra responden que si a los trabajadores no les agrada-
ran los horarios laborales largos, la competencia resultaría en una prima por el tiempo extra incluso
sin regulación. De manera alternativa, si los trabajadores quisieran horarios de trabajo largos, ¿por
qué apoyarían una ley que desalienta a los patrones para que lo hagan? Para sus detractores la ley del
tiempo extra es irrelevante o perjudicial.
Las externalidades de posición sugieren una justificación para la regulación de las horas labora-
les. Si alguien se queda unas cuantas horas extra, aumentarán sus ganancias en términos absolutos y
relativos. Un resultado es que podrá tener una casa en un distrito escolar mejor. Pero el problema es,
una vez más, que el progreso de una familia en términos relativos significa un retroceso para otros.
En lugar de ver que sus familias se quedan rezagadas, otros sentirán la presión de trabajar horarios
largos. Al final, estos esfuerzos se compensan en gran medida. Como vimos antes, sólo 10% de los
niños pueden ocupar un lugar en los principales distritos escolares.
Al trabajar diario hasta las 8 p.m., es posible producir más y disfrutar de ingresos más altos que si
sólo se trabajara hasta las 5 p.m. Pero en el proceso se tiene menos tiempo para estar con la familia y
los amigos. Es fácil ver por qué la gente prefiere vivir en comunidades donde todos salen de trabajar a
las 5 p.m. Es muy fácil ver también por qué podría haber pocas comunidades así a falta de leyes sobre
el tiempo extra.

AHORROS
Muchos observadores se quejan, con toda razón, de que el sistema del seguro social no les permite
decidir cuándo y cuánto ahorrar para el retiro. Desde el punto de vista tradicional, que postula si es
mejor tener más opciones que menos, parecería mejor que la participación en el seguro social fuera
totalmente voluntaria. Sin embargo, la mayoría de las sociedades tienen programas obligatorios para
complementar los ingresos para el retiro. Una vez más, las externalidades de posición pueden ayudar
a entender por qué.
El argumento es en esencia el mismo que en el caso de la regulación de los horarios. Un padre
tiene la opción de ahorrar parte de sus ingresos actuales para el retiro o gastar ese ingreso ahora

16_CHAPTER 16.indd 554 3/7/09 10:45:44 AM


el graVamen de laS externalidadeS 555

en una casa en un mejor distrito escolar. Como ya se vio, muchos padres consideran obligatoria la
segunda opción.
Sin embargo, los efectos acumulados de dichas opciones no son suficientes para lo que preten-
den los padres. Cuando alguien gasta más en una casa en un mejor distrito escolar, el resultado es
que aumentan los precios de las casas. En el proceso nadie avanza en la jerarquía de la educación y
así los padres terminan con muy pocos ahorros para el retiro. Sin embargo, si actúan como indivi-
duos, su única alternativa real es enviar a sus hijos a escuelas menos deseables.
El sistema de seguridad social aminora este dilema al retener una porción del ingreso de cada
persona para que no la gaste. También ayuda a solucionar una serie de externalidades de posición
relacionadas. Por ejemplo, lo recomendable es que una persona que busca empleo se vea lo mejor
posible para sus entrevistas de trabajo. Sin embargo, verse bien no sólo es usar ropa limpia y pre-
sentable. Al igual que una buena educación, una apariencia con buen gusto es un concepto relativo.
Verse bien significa tener un aspecto mejor que otros y la forma más práctica de hacerlo es gastar
más que los demás en ropa. El problema es que este mismo cálculo funciona para todos los demás.
Al final se obtiene un escalamiento infructuoso de la cantidad que una persona tiene que gastar en
ropa simplemente para no estar mal vestida. Visto desde la perspectiva de la población como un
todo, lo lógico sería ahorrar más y gastar menos en ropa. Pero a nadie que actúe solo le redituaría
esta medida. El sistema de seguridad social, al guardar una parte de los ingresos, limita la cantidad
que una persona gasta en ésta y una variedad de situaciones análogas.

SEGURIDAD EN EL TRABAJO
Como ejemplo final de las externalidades de posición, considere el caso de las regulaciones de segu-
ridad en el trabajo. Al igual que en el caso de las reglamentaciones de los horarios, quienes proponen
las regulaciones de seguridad con frecuencia se defienden diciendo que los monopsonistas de otro
modo obligarían a sus trabajadores a laborar en condiciones riesgosas inaceptables. Pero como se
vio en el capítulo 14 este argumento subestima seriamente las presiones de competencia en el mer-
cado laboral. Con dichas presiones en mente, los críticos de la regulación de seguridad dicen que
priva a los trabajadores del derecho a decidir personalmente qué tanta seguridad quieren comprar
en el trabajo.
Sin embargo, en cuanto se toman en cuenta las externalidades de posición, instituir una regu-
lación de seguridad no parece tan confuso. La elección por parte de un trabajador de un empleo de
mayor riesgo dificulta que otros liciten con tanta eficacia como antes por adquirir una casa en los
mejores distritos escolares. Como sienten esta presión, lo más probable es que opten por trabajos de
mayor riesgo. Desde luego que, en términos de posición, estos movimientos se compensan entre sí
en gran medida. Es probable que la gente prefiera neutralizar esta carrera armada adoptando leyes
que establezcan estándares mínimos de seguridad en el trabajo.7

EL GRAVAMEN DE LAS EXTERNALIDADES


Antes del artículo de Coase de 1960 la economía estaba unida a la perspectiva, de la que es pionero
el economista británico A. C. Pigou, de que la mejor solución a las externalidades negativas es gra-
varlas. La idea es simple. Si A realiza una actividad que impone un costo a B, entonces gravar a A por
el monto de ese costo le proporcionará el incentivo adecuado para considerar la externalidad en sus
decisiones de producción. Sin embargo, como se verá en el siguiente ejemplo, dichos impuestos a
veces empeoran las cosas más que si no se hiciera nada.

7
Si estos estándares logran su meta establecida de que haya más seguridad en el trabajo es una cuestión estrictamente empírica.
Algunos autores han argumentado que las ineficiencias burocráticas de la regulación de seguridad de hecho han llevado a menores ni-
veles de seguridad. Véase Albert Nichols y Richard Zeckhauser, “Government Comes to the Workplace: An Assessment of OSHA”,
en The Public Interest, 49, 1977, pp. 39-69.

16_CHAPTER 16.indd 555 3/7/09 10:45:44 AM


556 CaPítulo 16 externalidadeS, dereChoS de ProPiedad y el teorema de CoaSe

EJEMPLO 16.9 Considere de nuevo al médico y al repostero de los ejemplos 16.1 a 16.6. Suponga que el mé-
dico gana 60 por operar en un ambiente sin ruido y que el repostero gana 40 por operar su
equipo ruidoso. Suponga también que el médico puede eliminar el problema del ruido reaco-
modando su oficina a un costo de 18. Y, por último, suponga que la negociación entre el médico
y el repostero es prohibitivamente costosa. El enfoque del gravamen recomienda aplicar un
impuesto al repostero igual al daño que provocaría su actividad, el cual, a falta de respuesta por
parte del médico, significa un costo de 60. ¿Cómo se compara el resultado con este impuesto
con lo que hubiera ocurrido si no existiera?

Si hubiera negociaciones sin costo, el repostero podría pagar al médico por el reacomodo de su con-
sultorio y después operaría sin pagar el impuesto porque ya no causaría daños con su ruido. Pero
como la negociación no es práctica, el médico no tiene motivo para incurrir en este costo. Sabe que,
sin hacer algo, el repostero tendrá que pagar un impuesto de 60 si opera, lo cual a su vez significa
que la mejor opción del repostero es cerrar. A fin de cuentas, para empezar, su operación genera una
ganancia de 40 únicamente. Si el repostero ya no está en operación, el médico tendrá una ganancia
de 60 y el repostero de 0.
Pero, sin el impuesto, el repostero seguiría en operaciones por una ganancia de 40. La mejor
respuesta del médico habría sido reacomodar su consultorio a un costo de 18, dejándolo con una
ganancia neta de 42. Por lo tanto, sin el impuesto se obtiene el mejor resultado eficiente, mientras
que la ganancia total con el impuesto es considerablemente más baja. Los datos importantes de este
ejemplo se presentan en el resumen de la tabla 16.9.

TABLA 16.9
Resumen del resultado y el pago del ejemplo 16.9

Beneficio neto Por su operación el


repostero gana 40. la
Esquema legal Resultado Médico Repostero Total
pérdida del médico por
impuesto de 60 el repostero cierra 60 0 60 el ruido del repostero
para el repostero es 60. el médico
Sin impuesto ni el médico reacomoda su 42 40 82 puede reacomodar su
responsabilidad consultorio y paga los gastos consultorio para eliminar
el problema del ruido a un
costo de 18. el resultado
eficiente es que el médico
reacomode su oficina y
esto sólo ocurre si no se
grava al repostero.

Como lo demuestra ampliamente el ejercicio 16.9, un impuesto a la contaminación da lugar a


una posición peor que la que existe sin el impuesto. Esto no sorprende si se reconoce que un impues-
to sobre la contaminación tiene, en esencia, el mismo efecto que responsabilizar a quien contamina
por los daños que produce. Pero este mismo reconocimiento implica que el gravamen no siempre es
ineficiente. En el ejemplo 16.9 resultó inoperante porque el médico era la parte que estaba en mejor
posición de manejar el problema del ruido y el impuesto eliminaba todo incentivo para que lo hicie-
ra. Por el contrario, suponga que el médico no cuenta con medios no costosos para escapar del daño
por el ruido. Aun así, el impuesto hubiera provocado que el repostero cerrara, lo cual hubiera sido el
mejor resultado eficiente. (Véase ejemplo 16.1.)

16_CHAPTER 16.indd 556 3/7/09 10:45:45 AM


el graVamen de laS externalidadeS 557

Como alternativa suponga que el repostero hubiera tenido medios para eliminar el problema
del ruido sin costo alguno. Por ejemplo, suponga que podría haber colocado material a prueba de
ruido por un costo de 10. En este caso, también, el impuesto hubiera provocado el resultado más
eficiente. El repostero hubiera instalado el material a prueba de ruido para evitar el impuesto y el
médico hubiera operado sin que lo molestara.
El que sea eficiente gravar la contaminación depende, pues, de las circunstancias particulares
presentes. Si la negociación no genera costos, el gravamen siempre conducirá a un resultado eficien-
te. (Pero, para ese efecto, también el no gravar.) Si la negociación es impráctica, gravar la contami-
nación de todas maneras causará un resultado eficiente si el contaminador posee la manera menos
costosa de reducir los daños por la contaminación. Sólo si la negociación no es práctica y la víctima
tiene el medio menos costoso de evitar los daños, gravar la contaminación conducirá a un resultado
ineficiente. En esencia, gravar o no gravar producirá el mismo resultado si los costos de limitar el
daño por la contaminación son casi los mismos para el contaminador y la víctima.
Suponga que la sociedad considera que los productores de contaminantes son quienes, de he-
cho, pueden atenuar los daños que provocan al costo más bajo. Entonces, la sociedad debe elegir
una política que ofrezca un incentivo para que quien contamina actúe. Una opción es establecer
límites directos sobre la cantidad de contaminantes emitidos. Como alternativa se podría adoptar un
impuesto a la contaminación, lo cual significa cobrar una comisión a quienes contaminan por cada
unidad de contaminantes que liberan. El siguiente ejemplo demuestra que la opción del gravamen
ofrece una ventaja irresistible sobre la opción de la regulación directa.

Dos empresas, X y Y, tienen acceso a cinco procesos de producción, cada uno tiene un costo EJEMPLO 16.10
diferente y genera una cantidad distinta de contaminación. Los costos diarios de los procesos
y el número correspondiente de toneladas de humo se presentan en la tabla 16.10. Si la con-
taminación no está regulada y la negociación entre las empresas y sus víctimas es imposible,
cada empresa usará A, el menos costoso de los cinco procesos, y cada una emitirá 4 toneladas
de contaminantes al día, para un total de 8 ton/día. El municipio quiere reducir a la mitad las
emisiones de humo. Para conseguirlo consideran dos opciones. La primera es requerir que
cada empresa reduzca a la mitad sus emisiones. La alternativa es establecer un impuesto de
T a cada tonelada de humo emitido al día. ¿Qué tan grande tendría que ser T para reducir a
la mitad las emisiones? ¿Y cómo serían en comparación los costos para la sociedad de estas
dos alternativas?

TABLA 16.10
Costo y emisiones de cinco procesos de producción

Proceso A B C D E Cada empresa tiene acceso


(humo) (4 ton/día) (3 ton/día) (2 ton/día) (1 ton/día) (0 ton/día) a cinco alternativas de
procesos de producción,
Costo para la 100 190 600 1 200 2 000
A-E, que varían en
empresa X
costo y en la cantidad
Costo para la 50 80 140 230 325 de contaminación que
empresa Y producen.

Si se le pide a cada empresa que reduzca la contaminación a la mitad, cada una debe cambiar del
proceso A al C. El resultado será 2 ton/día de contaminación por cada empresa. El costo del cambio
para la empresa X será de 600/día – 100/día = 500/día. El costo para Y será de 140/día – 50 /día =
90/día, que representa un costo total para las dos empresas de 590/día.
¿Cómo responderá cada empresa a un impuesto de T por tonelada de contaminantes? Primero
tendrá que preguntarse si el cambio del proceso A al B aumentará sus costos en más o menos que

16_CHAPTER 16.indd 557 3/7/09 10:45:46 AM


558 CaPítulo 16 externalidadeS, dereChoS de ProPiedad y el teorema de CoaSe

T/día. Si es menos, pagará por el cambio, porque el proceso B, que produce 1 tonelada menos de
humo, ahorrará a la empresa T/día en impuestos. Sin embargo, si los costos del proceso B superan a
los de A por más que T, la empresa no cambiará. Le será más económico quedarse con A y pagar el
T extra en impuestos. Si el cambio de B a C paga, entonces la empresa hará la misma pregunta acerca
del cambio de B a C. Seguirá cambiando hasta que los costos extras del siguiente proceso ya no sean
menores que T.
Para ilustrar, suponga que se recaudó un impuesto de 50/ton. La empresa X se quedaría con el
proceso A porque cuesta 90/día menos que el proceso B y sólo produce 1 ton/día de humo adicional
y, por lo tanto, 50/día en impuestos adicionales. Por el contrario, la empresa Y cambiará al proceso B
porque sólo cuesta 30/día más y ahorrará 50/día en impuestos. Pero la empresa Y no cambiará a C
porque cuesta 60/día más que B y sólo ahorrará 50/día adicionales en impuestos. Si la empresa X se
queda en A y la empresa Y cambia a B se obtiene una reducción total de contaminación de 1 ton/día.
Por consiguiente, un impuesto de 50/ton no produce la reducción deseada de 50% de contaminación.
La solución es que siga aumentando el impuesto hasta que se obtenga el resultado deseado.
Considere qué sucede con un impuesto de 91/ton. Éste conducirá a que la empresa X adopte el
proceso B y que la empresa Y adopte el proceso D. El total de emisiones serán las 4 ton/día deseadas.
El costo para la empresa X será de 190/día – 90/día, y el costo para la empresa Y será de 230/día
– 50/día = 180/día. Por lo tanto, el costo total para ambas empresas sólo es de 270/día o 320/día me-
nos que el costo de conseguir que cada empresa reduzca la contaminación a la mitad. Observe que
los impuestos que paga la empresa no se incluyen en el cálculo de los costos sociales de la alternativa
del gravamen, porque ante la sociedad no se ha perdido este dinero. Se puede usar para reducir los
impuestos que de otro modo tendrían que cobrarse a los ciudadanos.
La ventaja del enfoque fiscal es que concentra la reducción de contaminantes en manos de las
empresas que pueden lograrlo en la forma menos costosa. El enfoque regulador directo de requerir
que cada empresa reduzca las emisiones a la mitad no toma en cuenta el hecho de que la empresa Y
puede reducir la contaminación de forma mucho más económica que la empresa X. De acuerdo con
este enfoque fiscal, observe que el costo de la última tonelada de humo eliminado es el mismo para
cada empresa.

En términos más generales suponga que hay dos productores, la empresa X y la empresa Y,
cuyos costos marginales para eliminar el humo se representan con las curvas marcadas con CMX y
CMY, respectivamente, en la figura 16.4. Si la meta es reducir el total de emisiones de humo en Q =
Q*X + Q*Y tons/día, un impuesto de T* logrará que se alcance ese objetivo de la forma menos costosa.
La característica de esta solución es que el costo marginal de la reducción de contaminación sería
exactamente el mismo para todas las empresas. Si no fuera ése el caso, siempre podría reasignarse la
reducción de contaminación de forma que se redujeran los costos totales.

FIGURA 16.4
El enfoque del
gravamen para reducir
la contaminación dls./toneladas CMX dls./toneladas
CmX y CmY representan el
costo marginal de reducir el
humo para las empresas X y
Y, respectivamente. Cuando
la contaminación se grava a CMY
una tasa fija, cada empresa
reduce sus emisiones hasta
T*
el punto donde el costo
marginal de una mayor
reducción es exactamente
igual al impuesto. el QX (tons/día) QY (tons/día)
resultado es la forma Q*X Q*Y
menos costosa de lograr la
reducción de contaminación
acumulada correspondiente.

16_CHAPTER 16.indd 558 3/7/09 10:45:47 AM


el graVamen de laS externalidadeS 559

El enfoque regulatorio directo (que indica a cada empresa cuánto debe reducir la contamina-
ción) también podría lograr alguna disminución total de la contaminación a un costo mínimo, si los
reguladores conocieran la curva de reducción del costo marginal de cada empresa. De este modo
podrían asignar cuotas de reducción de manera que uniformaran los costos marginales para todas las
empresas. El problema es que, por lo general, las autoridades reguladoras no tienen ni la más mínima
idea de cómo son estas curvas. La mayor ventaja del enfoque con gravamen es que logra la eficiencia
sin necesidad de ningún conocimiento por parte de los reguladores.
Cuando el gobierno asigna un impuesto a un producto cuya producción no genera externalida-
des, éste se venderá a un precio superior a su costo marginal (vea el capítulo 2). Y cuando el precio
rebasa el costo marginal, la producción no alcanza el nivel que maximizaría el excedente económico.
En contraste, cuando no hay ningún impuesto sobre un producto cuya fabricación genera externa-
lidades negativas, su producción supera el nivel que maximizaría el excedente económico. Por lo
tanto, otra ventaja de gravar las externalidades negativas es que proporciona un medio para elevar
los ingresos del gobierno que no ocasiona pérdidas en la eficiencia. Por el contrario, se ha visto que
gravar con impuestos las externalidades negativas puede incrementar la eficiencia. Saber si el grava-
men sobre las externalidades negativas producirá ingresos suficientes para que el gobierno realice
todas sus actividades es una pregunta empírica. Si lo hiciera, desaparecerían las preocupaciones por
las ineficiencias derivadas de los impuestos.

EL GRAVAMEN DE LAS EXTERNALIDADES DE POSICIÓN


Existe evidencia considerable de que la utilidad que la gente obtiene del consumo depende no sólo
de los niveles de consumo absolutos, sino también de los niveles relativos. En el siglo xix nadie se
sentía en desventaja por no tener un automóvil o un televisor; sin embargo, en la actualidad, la gente
que no tiene esos artículos se siente muy insatisfecha. Y la insatisfacción no es sólo una cuestión de
envidiar las posesiones de los vecinos. Si nadie tuviera un auto no sería necesario que poseyéramos
uno para satisfacer las demandas mínimas de la existencia social. Pero como en la actualidad casi
todos tienen auto, es muy difícil vivir sin uno.
Si el consumo relativo es importante, se deduce por lógica que el consumo de cada persona im-
pone externalidades negativas en los demás. Cuando alguna persona aumenta su consumo, eleva, tal
vez en forma imperceptible, el estándar de consumo para los demás. Como el economista británico
Richard Layard dijo alguna vez: “En una sociedad pobre, un hombre prueba a su esposa que la ama
dándole una rosa, pero en una sociedad rica debe darle una docena de rosas.”
El hecho de que muchas formas de consumo generen externalidades negativas tiene consecuen-
cias importantes en la política fiscal. Para ejemplificarlo considere la decisión de un hombre joven
sobre el tamaño del diamante que le regalará a su prometida. Como la función de este regalo es
servir como un símbolo de compromiso, la gema que compre deberá tener un precio suficiente. Su
joyero le dirá que la costumbre en su país es pagar el salario de dos meses por la piedra. Si gana 60 000
dls./año, tendrá que gastar 10 000 dólares, pues de lo contrario sentirá que es un avaro.
Desde el punto de vista de la economía en general, el resultado sería mejor si existiera un im-
puesto de 400% sobre la joyería. El precio después de impuesto del que ahora es un diamante de
2 000 dólares se elevaría a 10 000. Al comprar este diamante más pequeño el hombre incurriría en el
mismo problema económico que antes. Y como ésta es la esencia de la función del regalo, su objetivo
no se vería comprometido por el impuesto. Tampoco su prometida sufriría ninguna pérdida real.
Como ahora todos comprarían diamantes más pequeños, la piedra de menor tamaño daría práctica-
mente la misma satisfacción que la grande. Desde el punto de vista positivo, el gobierno recibe 8 000
dólares adicionales para financiar sus gastos o reducir otros impuestos. El único perdedor es el cártel
de diamantes deBeers de Sudáfrica, que ganará 8 000 dólares menos que antes del impuesto.
Los estándares que definen las escuelas, casas, guardarropas, autos, vacaciones aceptables, así
como muchos otros elementos importantes del presupuesto están relacionados con las cantidades de
dinero que otras personas gastan en ellos. Como los consumidores individuales casi siempre ignoran
las externalidades de posición cuando eligen, el resultado es que estos productos son mucho más
atractivos para los individuos que para la sociedad en general. Por las mismas razones por las que a

16_CHAPTER 16.indd 559 3/7/09 10:45:47 AM


560 CaPítulo 16 externalidadeS, dereChoS de ProPiedad y el teorema de CoaSe

menudo resulta eficiente gravar la contaminación, sería eficiente gravar muchas de estas formas de
consumo. En cuanto a la eficiencia, estos impuestos serían un sustituto atractivo para los graváme-
nes que ya existen y que interfieren con la distribución eficiente de los recursos.

REsUMEN
• Cuando un acto de una parte daña a otra y ambas negocian legales resultan ser las que más se asemejan a las que las per-
sin incurrir en costos, las externalidades negativas se resuelven sonas habrían negociado por su cuenta si hubiera sido práctico
con eficiencia independientemente de que la ley responsabili- que lo hicieran.
ce a las personas por los efectos perjudiciales de sus actos. Este • Conclusiones semejantes se aplican a situaciones que invo-
resultado se conoce como teorema de Coase. lucran externalidades positivas. Si una negociación no tiene
• Cuando la negociación es costosa, sí importa cómo se asignan costos, las personas llegan a acuerdos que producen resultados
las responsabilidades. En general, el mejor resultado ocurre eficientes, aun en casos en que los actos de una parte crean
cuando la ley adjudica la carga de evitar los efectos perjudicia- beneficios indirectos para la otra. Y cuando la negociación
les en la parte que puede hacerlo al menor costo. cuesta, surgen instituciones que alientan las actividades con
• Este principio general aclara diversas preguntas que concier- efectos externos positivos.
nen al diseño de los derechos de propiedad. Explica por qué el • En las competencias por las posiciones, como en todas las
propietario de un muelle no tiene derecho a impedir que un contiendas, los esfuerzos de un competidor confieren una
barquero atraque durante una tormenta, cuándo una persona externalidad negativa a los demás participantes: todo lo que
puede prohibir que otras crucen por sus tierras o que obsta- aumenta las probabilidades de que una parte gane reduce ne-
culicen su vista, por qué un pastizal es más productivo si es de cesariamente las probabilidades de las otras. El efecto es casi
propiedad privada que si es común, por qué los aviones pue- siempre inducir una carrera armada en la cual los esfuerzos
den volar sobre la propiedad de alguien pero los urbanistas no de cada parte se cancelan en buena medida unos con otros. La
pueden construir ahí sin autorización. En cada caso las leyes teoría de las externalidades y los derechos de propiedad arroja
de propiedad tienen el objetivo de imitar lo mejor posible los luces sobre las leyes con las que los ciudadanos de las socieda-
arreglos a los que llegan las personas por su cuenta si negocia- des modernas restringen estas carreras.
ran entre sí sin incurrir en costos. • Aplicar impuestos es una solución al problema de las externa-
• Principios similares se aplican a otras restricciones guberna- lidades negativas. Aunque no es siempre una respuesta ideal,
mentales sobre la conducta. En el caso de la libertad de expre- ofrece varias ventajas importantes sobre la regulación directa
sión y otros derechos constitucionales, las mejores soluciones en muchas situaciones.

pREgUNTAs DE REpAsO
1. Cuando los costos de negociación son insignificantes, ¿por qué 4. ¿Por qué se permite que los aviones y no los desarrolladores
la asignación de la responsabilidad por las externalidades no es de bienes raíces usen el espacio aéreo sobre las casas sin un
relevante para la eficiencia? consentimiento previo?
2. ¿La asignación de la responsabilidad es importante por razo- 5. ¿Por qué la mayor parte de las leyes de propiedad limitan la
nes de distribución? propiedad privada costera a la línea del agua cuando la marea
3. Suponga que usted es la parte que puede evitar un efecto ex- está alta?
terno al costo más bajo. ¿Por qué estaría usted a favor de una 6. Mencione tres ejemplos de la tragedia de los bienes comunes
regla general que asigne la responsabilidad a cualquiera de las en su campus.
partes que pueda evitar el daño al costo más bajo?

pRObLEMAs
1. En noviembre de cada año, Smith y Jones tienen que decidir entre quemar las hojas de su jardín o
meterlas en bolsas de basura. Quemarlas es mucho más fácil pero produce humo nocivo. Los valores
de utilidad para cada persona se muestran en la tabla para cada una de las cuatro combinaciones de
acciones posibles:

16_CHAPTER 16.indd 560 3/7/09 10:45:48 AM


ProblemaS 561

Smith
Quemar Embolsar
Jones: 4 Jones: 8
Quemar
Smith: 4 Smith: 2
Jones
Jones: 2 Jones: 6
Embolsar
Smith: 8 Smith: 6

a) Si Smith y Jones son maximizadores de utilidades y toman decisiones en forma individual, ¿qué
harán?
b) ¿De qué manera cambiaría su respuesta al inciso a), si es que lo hace, en el caso de que Smith y
Jones puedan llegar a un acuerdo entre ellos?
Ahora suponga que la matriz de beneficios es la siguiente:

Smith
Quemar Embolsar
Jones: 6 Jones: 8
Quemar
Smith: 6 Smith: 2
Jones
Jones: 2 Jones: 4
Embolsar
Smith: 8 Smith: 4

c) ¿Qué harán esta vez si pueden llegar a un acuerdo?


2. Smith puede producir con o sin un filtro en su chimenea. La producción sin el filtro da como re-
sultado mayor daño a Jones a causa del humo. Las ganancias y pérdidas relevantes para ambos se
muestran en la tabla siguiente:

Con filtro Sin filtro


Ganancias para Smith 200 dls/sem 245 dls/sem
Daño para Jones 35 dls/sem 85 dls/sem

a) Si Smith no es responsable del daño causado por el humo y no hay costos de negociación, ¿instala-
rá un filtro? Explique su respuesta en forma detallada.
b) ¿En qué sería diferente el resultado en el caso de que Smith sea responsable del daño causado
por el humo y el costo del filtro sea 10 dls./sem más alto que el indicado en la tabla? Explique su
respuesta en forma detallada.
3. Smith puede operar su aserradero con o sin protección contra el ruido. La operación sin protección
causará daños por el ruido a su vecino Jones. Las ganancias y pérdidas relevantes para Smith y Jones
se muestran en la tabla:

Con filtro Sin filtro


Ganancias para Smith 150 dls/sem 34 dls/sem
Daño para Jones 125 dls/sem 6 dls/sem

16_CHAPTER 16.indd 561 3/7/09 10:45:49 AM


562 CaPítulo 16 externalidadeS, dereChoS de ProPiedad y el teorema de CoaSe

a) Si Smith no es responsable del daño causado por el ruido y no hay costos de negociación, ¿instala-
rá una protección contra el ruido? Explique su respuesta.
b) ¿Cómo cambiaría su respuesta si los costos de la negociación de un acuerdo fueran de 4 dls./sem?
Explique su respuesta.
c) Ahora suponga que Jones puede evitar el daño causado por el ruido mudándose a otro lugar, lo
que le costaría 120 dls./sem. Suponga otra vez que los costos de la negociación son cero, ¿en qué
sería diferente su respuesta al inciso a)? Explique su respuesta.
4. Smith y Jones tratan de decidir si van a compartir un departamento. Si vivieran separados cada uno
tendría que pagar una renta de 300 dls/mes. La renta de un departamento grande que pueden com-
partir es de 450 dls/mes. Independientemente de los costos, les es indiferente vivir juntos o separa-
dos, excepto por los siguientes dos problemas: a Smith le gusta escuchar música hasta muy tarde por
la noche, lo que interrumpe el sueño de Jones; y a Jones le gusta cantar en la regadera a las 6 a.m.,
lo que despierta a Smith. Jones sacrificaría hasta 80 dls/mes por no dejar de cantar en la regadera y
Smith sacrificaría hasta 155 dls/mes por no dejar de escuchar música en las noches. Smith toleraría
el canto de Jones y Jones la música de Smith a cambio de pagos de compensación no menores que 75
dls/mes y 80 dls/mes, respectivamente.
a) ¿Deben vivir juntos? Si la respuesta es sí, indique cómo pueden dividir la renta de manera que a
cada uno le convenga más que vivir solo. En caso contrario, explique por qué no es factible un
arreglo.
b) Ahora suponga que Smith gana un par de audífonos para su estéreo. Si los usa por las noches no
interrumpirá el sueño de Jones. A Smith le gustan los audífonos, pero estaría dispuesto a pagar
40/mes por seguir escuchando por las noches su música proveniente de las bocinas de su estéreo.
¿De qué manera la existencia de esta nueva opción afecta su respuesta al inciso a)? Explique su
respuesta en forma detallada.
5. A y B pueden vivir separados pagando una renta de 400 dls/mes cada uno o juntos con una renta de
600 dls. Cada uno estaría dispuesto a pagar 30 dls/mes para no tener que renunciar a su intimidad.
Además de la pérdida de esta última, el hecho de vivir juntos da lugar a otros dos conflictos, pues
cada uno tiene un comportamiento que el otro considera ofensivo: B toca la trompeta y A fuma. B
estaría dispuesto a pagar 60 dls/mes para no tolerar que alguien fume en su casa y 120 dls/mes para
seguir tocando la trompeta. Por su parte, A pagaría hasta 100 dls/mes por seguir fumando y hasta
90 dls/mes por no tener que escuchar la música de la trompeta. ¿Vivirán juntos? Explique en forma
detallada su respuesta. ¿Su respuesta sería diferente si a A no le importara renunciar a su intimidad?
6. A y B viven en terrenos adyacentes. Cada uno tiene dos usos potenciales para su terreno, cuyo valor
presente depende del uso que adopte el otro, como lo indica el resumen de la tabla. Todos los valores
de la tabla son conocidos por ambas partes.
A
Cultivo de manzanas Granja de cerdos
A: $200 A: $450
Alquiler de viviendas
B: $700 B: $400
B
A: $400 A: $450
Apicultura
B: $650 B: $500

a) Si no hay costos de negociación, ¿qué actividades realizarán los dos en sus terrenos?
b) Si los costos de negociación son de 150 dólares, ¿qué actividades realizarán ambos en sus terrenos?
c) ¿Cuál es el ingreso neto máximo que A puede ganar en los incisos a) y b)?
7. Una villa tiene seis residentes, cada uno de los cuales tiene 1 000 dólares. Cada residente puede in-
vertir su dinero en un bono del gobierno, que paga 11%/año, o utilizarlo para comprar un toro de
un año de edad que apacentará en las áreas comunes de la villa. Los toros de un año y los bonos
del gobierno se venden en 1 000 dólares cada uno. Los toros no requieren de ningún esfuerzo y se
pueden vender a un precio que depende de la cantidad de peso que hayan ganado durante el año. A
su vez, la ganancia en el peso depende del número de toros que apacientan en las áreas comunes. Los
precios de los toros de dos años aparecen en la tabla como una función del número total de toros.

16_CHAPTER 16.indd 562 3/7/09 10:45:49 AM


ProblemaS 563

Número de toros Precio por toro de dos años


1 $1 200
2 1 175
3 1 150
4 1 125
5 1 100
6 1 075

a) Si los residentes de la villa toman sus decisiones de inversión en forma independiente, ¿cuántos
toros apacentarán en las áreas comunes?
b) ¿Cuántos toros apacentarán en las áreas comunes si las decisiones de inversión se toman en forma
colectiva?
c) ¿Qué cuota por toro daría como resultado el número óptimo de animales?
8. Una industria pesquera competitiva consiste en cinco barcos pesqueros con dueños y operadores
independientes que zarpan del puerto de Ithaca. Suponga que nadie más pesca en el lago Cayuga y
que el costo marginal de operación de un bote durante un día equivale a 70 libras de pescado. (Un
buque inactivo no genera costos.) La pesca total por línea costera, en libras, se muestra en la tabla
siguiente como una función del número de barcos que pescan en las costas este y oeste del lago:

Pesca total
Número de barcos por costa Costa este Costa oeste
1 100 85
2 180 150
3 255 210
4 320 260
5 350 300

a) Si el dueño de cada barco decide en forma independiente en qué lado del lago va a pescar y cada
lancha puede ver a los demás, ¿cuántas lanchas esperaría encontrar pescando en cada orilla en un
día determinado? ¿Cuál es la pesca neta (es decir, la pesca total en ambas orillas, menos los costos
operativos)?
b) ¿Esta distribución de la pesca es la óptima desde el punto de vista social? De ser así, explique por
qué. De lo contrario, ¿cuál es la distribución socialmente óptima y la pesca neta correspondiente?
9. Dos empresas, X y Y, tienen acceso a cinco procesos de producción diferentes, cada uno de los cuales
emite una cantidad distinta de contaminantes. Los costos diarios de los procesos y el número de
toneladas de humo correspondiente se muestran en la tabla:

Proceso A B C D E
(humo) (4 ton/día) (3 ton/día) (2 ton/día) (1 ton/día) (0 ton/día)
Costo para la 100 120 140 170 220
empresa X
Costo para la 60 100 150 255 375
empresa Y

a) Si la contaminación no está regulada, ¿qué proceso va a utilizar cada empresa y cuál será el total
de emisiones de humo diarias?
b) El municipio quiere reducir a la mitad las emisiones de humo. Para lograrlo, exige un permiso
municipal por cada tonelada de humo emitida y limita el número de permisos al nivel de emisio-

16_CHAPTER 16.indd 563 3/7/09 10:45:50 AM


564 CaPítulo 16 externalidadeS, dereChoS de ProPiedad y el teorema de CoaSe

nes deseado. Los permisos se subastan a los mejores postores. Si X y Y son las únicas empresas que
contaminan ¿cuánto costará cada permiso? ¿Cuántos permisos comprará X? ¿Cuántos permisos
adquirirá Y?
c) Compare el costo total para la sociedad de esta subasta de permisos con el costo total que implica
que cada empresa reduzca las emisiones a la mitad.
10. Suponga que el gobierno trata de limitar la contaminación exigiendo una cantidad máxima de conta-
minantes que cada empresa puede emitir. En general, esto dará como resultado un costo del control
de la contaminación más alto que el necesario. Explique por qué.
*11. Una pequeña villa tiene seis personas, cada una de las cuales puede pescar en una laguna o trabajar en
una fábrica. Los sueldos en la fábrica son de 4 dls/día. El pescado se vende en los mercados competi-
tivos en 1 dólar la pieza. Si L personas pescan en la laguna, el número total de pescados está definido
por F = 8L – 2L2. La gente prefiere pescar, a menos que espere ganar más trabajando en la fábrica.
a) Si la gente decide de manera individual si pesca o trabaja en la fábrica, ¿cuántas personas van a
pescar? ¿Cuáles serán las ganancias totales de la villa?
b) ¿Cuál es el número de pescadores socialmente óptimo? Con ese número ¿cuáles serán las ganan-
cias totales de la villa?
c) ¿Por qué hay una diferencia entre el número de pescadores de equilibrio y socialmente óptimo?
12. Una vez a la semana Smith compra un paquete de seis latas de refresco de cola y las guarda en su re-
frigerador para que sus dos hijos las beban más tarde. Invariablemente, Smith descubre que los niños
se tomaron las seis latas el primer día. Jones también compra un paquete de seis latas de refresco de
cola una vez por semana para sus dos hijos pero, a diferencia de Smith, les dice que cada día pueden
beber no más de tres latas. Explique por qué el refresco de cola dura mucho más en la casa de Jones
que en la de Smith.
13. Suponga que Smith trabaja y es dueño de una pastelería ubicada junto a un café al aire libre propiedad
de Jones. A los clientes del café les agrada el olor que emana de la pastelería. Cuando Smith deja sus
ventanas abiertas, el café tiene una curva de demanda de PC = 30 – 0.2QC, y cuando las ventanas están
cerradas la demanda es PC = 25 – 0.2QC. Sin embargo, a Smith no le gusta el ruido proveniente de la
calle que se escucha cuando sus ventanas están abiertas y, en particular, la pérdida de utilidad que
experimenta tiene un valor monetario de 5. Suponga que el café tiene un costo marginal constante
de 10 y que la integración (fusión) no es una posibilidad porque cada propietario disfruta al poseer y
manejar su propio establecimiento.
a) En ausencia de un contrato entre las partes, ¿las empresas se comportan de manera eficiente?
Si no lo hacen, describa la gama de contratos que podrían surgir en respuesta al problema de
externalidad presente en el ambiente. Al responder esta pregunta suponga que Smith entiende de
qué manera el aroma de la pastelería afecta la demanda del café y Jones sabe cuánto le desagrada a
Smith el ruido de la calle.
b) Ahora suponga que todo es igual que en el inciso a), excepto que dada la distribución actual de
las sillas en el establecimiento, el café no tiene una demanda más alta cuando las ventanas de la
pastelería están abiertas. Para aprovechar la demanda más alta, Jones necesita hacer una inversión
no recuperable de 50, con lo que podría acercar las sillas a la pastelería. ¿Es conveniente para Jones
realizar esta inversión antes de firmar un contrato con Smith? Explique su respuesta.
c) Regrese a la situación inicial, pero ahora suponga que la pérdida de utilidad por el ruido de la ca-
lle para Smith es de 50 en lugar de 5. Además, suponga que antes de que las partes lleguen a un
acuerdo sobre un contrato, Jones se convierte en alcalde y se otorga a sí mismo los derechos de
propiedad para decidir si las ventanas de la pastelería deben permanecer abiertas o cerradas. ¿Lo
anterior afecta la posibilidad de que las partes logren un resultado eficiente? Explique su respuesta.
14. Smith y Jones enfrentan la decisión de manejar su auto hasta su trabajo temprano o tarde. Si ambos
van en auto a su trabajo a la misma hora, cada uno estorba al otro en su camino, de modo que el
camino diario toma más tiempo y es más molesto. Los beneficios monetarios para cada persona
aparecen en la tabla siguiente en cada una de las cuatro combinaciones de acciones posibles:
a) Si Smith y Jones son maximizadores de beneficios y toman las decisiones en forma individual,
¿qué harán?
* Este problema se resuelve con más facilidad si se aplica la definición del cálculo del producto marginal que se da en el apéndice del
capítulo 9.

16_CHAPTER 16.indd 564 3/7/09 10:45:50 AM


ProblemaS 565

b) Si Smith y Jones pueden llegar a un acuerdo entre ellos, ¿qué harán?

Smith
Temprano Tarde
Jones: 30 Jones: 50
Temprano
Smith: 30 Smith: 20
Jones
Jones: 20 Jones: 10
Tarde
Smith: 50 Smith: 10

15. La misma situación que en el problema 14, sólo que ahora los valores de los beneficios para cada
persona son

Smith
Temprano Tarde
Jones: 30 Jones: 50
Temprano
Smith: 30 Smith: 20
Jones
Jones: 20 Jones: 10
Tarde
Smith: 60 Smith: 10

a) Si Smith y Jones son maximizadores de los beneficios y toman las decisiones de manera individual,
¿qué harán?
b) Si Smith y Jones pueden llegar a un acuerdo entre ellos, ¿qué harán?
c) ¿En qué difieren sus respuestas de las que dio en el problema 14 y por qué?
16. A Smith le gustan mucho los perros y tiene una pareja de terriers de West Highland. Jones les teme
a los perros y no soporta ni verlos. Smith y Jones deciden si van a vivir en Arlington o en Bexley. Si
terminan viviendo en la misma parte de la ciudad, Jones se encontrará con Smith mientras pasea a
sus perros y se va a asustar. Por lo tanto, Jones prefiere estar físicamente separado de Smith. La tabla
siguiente muestra los beneficios para cada persona que ofrece cada una de las cuatro combinaciones
de acciones posibles:

Smith
Arlington Bexley
Jones: 0 Jones: 500
Arlington
Smith: 800 Smith: 900
Jones
Jones: 800 Jones: 0
Bexley
Smith: 800 Smith: 900

a) Si Smith y Jones son maximizadores de beneficios y toman sus decisiones en forma individual,
¿qué van a hacer?
b) Si Smith y Jones pueden llegar a un acuerdo entre ellos, ¿qué van a hacer?

16_CHAPTER 16.indd 565 3/7/09 10:45:51 AM


566 CaPítulo 16 externalidadeS, dereChoS de ProPiedad y el teorema de CoaSe

Smith
Arlington Bexley
Jones: 0 Jones: 500
Arlington
Smith: 800 Smith: 1 000
Jones
Jones: 600 Jones: 1 000
Bexley
Smith: 800 Smith: 900

17. La misma situación que en el problema 16, sólo que ahora los valores de los beneficios para cada
persona son
a) Si Smith y Jones son maximizadores de beneficios y toman sus decisiones de manera individual,
¿qué harán?
b) Si Smith y Jones pueden llegar a un acuerdo entre ellos, ¿qué harán?
c) ¿En qué son diferentes sus respuestas de las que dio en el problema 16 y por qué?

REspUEsTAs A LOs EjERciciOs DEL cApíTULO


16.1. Con un costo de negociación de sólo 20, ahora resulta práctico para el repostero, cuando la respon-
sabilidad recae sobre él, pagarle al médico para que redistribuya su oficina. Pero observe en la tabla
siguiente que sigue siendo más eficiente para el repostero no ser el responsable:

Beneficio neto
Régimen legal Resultado Médico Repostero Total
Responsable el repostero opera y paga al 22 + P 40 – P 62
médico 18 ≤ P ≤ 20 para que
redistribuya la oficina
No responsable el médico paga para redistribuir 22 60 82
su oficina

16.2. Recuerde que el número óptimo de toros es dos. La cuota por apacentar debe ser más de 2 para evi-
tar el envío de un tercer toro. La cuota no puede ser más de 6 sin evitar la eliminación del segundo
toro.
16.3. Ahora el costo de adaptarse al problema del humo es 60, que una vez más es menor que los ahorros
conjuntos en renta. Suponga que X representa la contribución de Jones al costo del sistema de esca-
pe, lo cual significa que la contribución de Smith es 60 – X. X no puede ser mayor que 90, pues de lo
contrario Jones viviría solo; y X no puede ser menor que –30, pues de ser así Smith viviría solo. La
ganancia total es 180 – 60 = 120.

Pago neto de renta


(dls./mes) Ganancia neta (dls./mes)
Jones Smith Jones Smith Total
Viviendo separados 300 300 ––– ––– –––
Viviendo juntos e 210 + X 270 – X 90 – X 30 + X 120
instalando el sistema
de escape para el
humo, –30 ≤ X ≤ 90

16_CHAPTER 16.indd 566 3/7/09 10:45:52 AM


Capítulo

17
GOBIERNO

as compañías de telefonía local son monopolios regulados, por lo que los reguladores del
L gobierno deben supervisar las tarifas que le cobran al público. Históricamente las normas
reguladoras impidieron los cargos por llamadas al servicio de información en la creencia
de que “disminuirían el valor de una red de comunicaciones públicas vitales”. Está por
demás decir que para la mayoría de los economistas esta conclusión parecía desesperadamente vaga.
Las llamadas al servicio de información cuestan mucho dinero a la compañía telefónica (y, por con-
siguiente, a la sociedad) y el temor inmediato del economista es que su uso sea poco rentable para la
gente, así como cualquier otro recurso por el cual no tiene que pagar.
Hace unos años la Comisión de Servicios Públicos del Estado de Nueva York (que regula a las
compañías telefónicas de ese estado) propuso que las compañías cobraran 10 centavos por llamada
realizada al servicio de información. Esta sensible propuesta para dar a la gente un incentivo para
buscar por sí misma los números en la sección amarilla animó a los economistas, pero generó dife-
rentes respuestas en los defensores del consumidor. Estos grupos contrataron a sociólogos y otros
testigos expertos que testificaron que si se sancionaba a las personas por intentar ponerse en contac-
to con otras, se deterioraría en gran medida la estructura social. Otros testigos se quejaron de que la
propuesta impondría una carga inaceptable a la gente pobre.
Cuando la comisión propuso una enmienda brillante con la que se conservarían las ganancias de
eficiencia al tiempo que se eliminarían los efectos adversos para los pobres, la propuesta pareció des-
tinada al fracaso. La enmienda fue que cada suscriptor telefónico recibiría un crédito de 30 centavos
en su recibo mensual que reflejara el costo ahorrado por las llamadas al servicio de información. Por
ejemplo, a la persona que hace una llamada al servicio de información en un mes se le cobrarían 10

567

17_CHAPTER 17.indd 567 3/6/09 8:14:14 PM


568 Capítulo 17 gobierno

centavos que, si se combinan con el crédito, tendría un recibo mensual con 20 centavos menos que
antes. La persona que hace tres llamadas al servicio de información acabaría a la par, el que hace cua-
tro llamadas, pagaría 10 centavos más que antes, y así sucesivamente. En la plausible suposición de
que sería más probable que un cargo de 10 centavos indujera a una persona de bajos ingresos a hacer
menos llamadas al servicio de información que a una de altos ingresos, el efecto neto de la propuesta
modificada de hecho aumentó el poder de compra real de los pobres.

VISIÓN PRELIMINAR DEL CAPÍTULO


El episodio del servicio de información ilustra dos puntos muy importantes acerca de la política eco-
nómica gubernamental: 1) que los asuntos de distribución permean el análisis de incluso las políticas
aparentemente más triviales; y 2) que la solución más eficiente para un problema de política pública
es la que permite que ricos y pobres mejoren por igual. En este capítulo la tarea es explorar dos fun-
ciones gubernamentales importantes: la provisión de los bienes públicos y la redistribución directa
de los ingresos. Como se verá, los problemas de equidad y eficiencia tienen una relación indisoluble
en ambas áreas.
Asimismo, se verá que el simple hecho de que un bien tenga las características de bien público no
significa que necesariamente lo proporcione el gobierno. Se estudiarán una variedad de esquemas in-
geniosos que van desde la televisión comercial gratuita hasta contratos legales colectivos sumamente
estructurados en los que se ofrecen bienes públicos con una mínima participación del gobierno.
Observará que problemas semejantes a los que surgen en relación con los bienes públicos se
encuentran cuando hay indivisibilidades o economías de escala significativas en la producción de
bienes de consumo privados.
Después se continúa con la pregunta de cómo deciden las sociedades entre proyectos públicos
que compiten entre sí, con un enfoque particular en el análisis costo-beneficio como una alternativa
para los esquemas de votación mayoritaria.
Se abordará también un problema que aqueja a los mecanismos de toma de decisiones públicas,
donde las partes tienen un incentivo para influir en que los resultados estén a su favor. El problema
se llama “búsqueda de rentas” y se ha convertido en una amenaza grave para el sistema de bienestar
social.
De los problemas de elección pública, que tienen connotaciones de distribución importantes, se
pasa al estudio del tema de los programas de transferencia directa de ingreso. En este caso el enfoque
se centra en cómo lograr dichas transferencias sin afectar los incentivos para trabajar y arriesgarse.

BIENES PÚBLICOS
Son aquellos bienes o servicios que poseen, en diversos grados, las propiedades de no disminución y no
exclusión. La propiedad de no disminución dice que el consumo de un bien público por cualquier per-
sona no influye en la cantidad que está disponible para otros. La propiedad de no exclusión significa
que, en cuanto a costos, es imposible o prohibitivo excluir a los no pagadores del consumo del bien.
bien público puro bien que Los bienes con elevados grados de ambas propiedades a menudo se denominan bienes públicos
tiene un elevado grado de no puros, un ejemplo clásico de ellos es la defensa nacional. Los bienes que sólo tienen la propiedad
disminución y no exclusión. de no disminución a veces se conocen como bienes colectivos. Éstos son proporcionados por el
gobierno pero también por compañías privadas. El gobierno provee la mayoría de los bienes públicos
bien colectivo  bien que es puros, aunque incluso en este caso hay casos excepcionales en los que las compañías que buscan
prescindible y tiene un elevado
utilidades diseñan esquemas para ofrecerlos.
grado de no disminución.
El análisis comienza con el caso de un gobierno que intenta decidir la cantidad de un bien pú-
blico puro que va a proveer, por ejemplo, la programación de la televisión pública. Por simplicidad,
suponga que sólo hay dos ciudadanos A y B y que cada uno asigna un valor diferente a cualquier can-
tidad del bien público. En la figura 17.1 el eje horizontal mide la cantidad de programación. La curva
marcada como AA' representa la cantidad que estaría dispuesto a pagar A por una unidad adicional
de programación y BB' representa la curva correspondiente de B. Por lo tanto, en el nivel de 4 unidades
de programación, A estaría dispuesto a pagar 9 dls./sem. por una unidad adicional, mientras B estaría

17_CHAPTER 17.indd 568 3/6/09 8:14:14 PM


bienes públiCos 569

FIGURA 17.1
Curva agregada de
$/Q disposición a pagar por
un bien público
22 D AA' y BB' representan las
cantidades respectivas
que A y B están dispuestos
Curva agregada de disposición a pagar
a pagar por una unidad
adicional del bien público.
15 la curva agregada de
A disposición a pagar es la
12
suma vertical de las curvas
10 B individuales, que es la curva
9
marcada como DD'A'.
6 D’

B’ A’
Q
4 10 16

dispuesto a pagar únicamente 6 dls./sem. El hecho de que las dos curvas de disposición a pagar ten-
gan pendientes negativas refleja el hecho de que entre más programación haya, menos valor tendrá
la unidad adicional.
El hecho central de proveer un bien público puro es que cada persona debe consumir la misma
cantidad. Por el contrario, en los mercados con bienes privados cada persona puede consumir la
cantidad que elija al precio prevaleciente. Para obtener la curva de demanda del mercado de un
bien privado se suman horizontalmente las curvas de demanda individuales. En el caso de los bienes
públicos el análogo a la curva de demanda del mercado es la curva agregada de la disposición a pagar.
Se obtiene con la suma vertical, no horizontal, de las curvas individuales de disposición a pagar. Por
ejemplo, en Q = 4 unidades de programación de la figura 17.1, A y B están dispuestos a pagar un total
de 9 + 6 = 15 dls./sem. por una unidad de programación adicional. La curva marcada como DD'A'
representa la suma vertical de las dos curvas individuales de disposición a pagar.

EJERCICIO 17.1
Diez consumidores homogéneos tienen curvas individuales de disposición a pagar P = 12 – 1– Q
5
por un bien público, por ejemplo un concierto en un parque abierto (en el que P se mide en
dólares y Q en minutos). Construya y grafique la curva agregada de disposición a pagar. ¿Cuán-
to es lo máximo que estaría dispuesto a pagar cada individuo por un concierto de 30 minutos?

ANALOGÍA DE LA PRODUCCIÓN CONJUNTA


Vea la sorprendente semejanza entre el procedimiento de generar la curva agregada de disposición a
pagar por un bien público y el procedimiento mediante el cual se genera la curva de demanda de un
producto como el pollo a partir de las curvas de demanda de sus diversas partes. Por simplicidad, su-
ponga que los pollos constan sólo de dos partes: alas y muslos, cuyas curvas de demanda aparecen en
la figura 17.2 como WW' y DD'. El eje horizontal de la figura 17.2 mide tres cosas simultáneamente:
pares totales de muslos, pares totales de alas y la cantidad total de pollos, porque un número determi-
nado de pollos dará la misma cantidad de pares de alas y muslos. Dada la suposición simplificada de
que las únicas dos piezas de pollo son las alas y los muslos, se tiene la curva de demanda del mercado

17_CHAPTER 17.indd 569 3/6/09 8:14:15 PM


570 Capítulo 17 gobierno

FIGURA 17.2
Equilibrio en un
mercado para $/Q
productos fabricados de C
manera conjunta
DD' es la curva de demanda S’
para los pares de muslos. D
WW' es la curva de la
demanda para los pares
de alas. su suma vertical,
CC'D', es la curva de P*C
demanda del mercado P*D
de pollos. el precio de W
equilibrio y la cantidad de C’
pollos se determina con la
intersección de esta curva PW*
S W’ D’
de demanda y la curva de Q
oferta del mercado. Q*

de pollos mediante la suma vertical de las curvas de demanda de alas y muslos. La curva marcada
como CC'D' que se presenta en la figura 17.2 representa esta sumatoria vertical.
La curva marcada como SS' en la figura 17.2 es la de la oferta de pollos. Si se supone que la
industria del pollo es competitiva, es la suma horizontal de las curvas de costo marginal de los pro-
ductores de pollo individuales. Al igual que en cualquier otro mercado competitivo, el equilibrio en
el mercado de pollos ocurre en la intersección de las curvas de oferta y de demanda. La cantidad de
equilibrio de pollos será Q* y esa cantidad dará origen a Q* pares de muslos y Q* pares de alas. Los
precios de compensación del mercado para los muslos y las alas son P*D y P*W, respectivamente. Estos
dos precios se suman al precio de equilibrio de pollos, P*C .
Hay varios puntos importantes que se deben observar acerca del equilibrio en el mercado de
bienes producidos de manera conjunta. En primer lugar, las cantidades de equilibrio de alas, muslos
y pollos son eficientes según el principio de Pareto. En Q*, el costo para la sociedad de producir otro
pollo es P*C , y éste es precisamente el valor total que los consumidores asignan a sus componentes.
Cualquier otra cantidad de pollos hubiera abierto la posibilidad de ganancias mutuas derivadas del
intercambio. Observe también que no se puede determinar el precio de cada pieza de pollo nada más
con base en la información del costo, aunque se supiera el costo marginal exacto de obtener otro
pollo. Simplemente no hay una base científica para repartir el costo de toda el ave entre cada una de
las partes que la constituyen. Los muslos y alas se venden a sus respectivos precios porque son los
necesarios para compensar los mercados de cada uno. En una manera precisamente análoga no hay
correspondencia entre la cantidad que cualquier individuo está dispuesto a pagar por un bien público
y su costo marginal de producción.

CANTIDAD ÓPTIMA DE UN BIEN PÚBLICO


De regreso en el ejemplo de la programación de televisión pública. Dada la curva agregada de la dis-
posición a pagar, ¿cuál es la cantidad óptima de programación? La respuesta se determina de forma
muy parecida a la del mercado de pollos. En la figura 17.3 la curva marcada como DD'A' representa
de nuevo la curva agregada de la disposición a pagar por la programación de televisión abierta. La
curva marcada como MC representa el costo marginal de la programación de televisión como una
función de su cantidad. La intersección de estas dos curvas establece que Q* = 4 es el nivel óptimo de
programación de televisión abierta. En Q* = 4, las cantidades que estarían dispuestos a pagar A y B
por otra unidad de programación iguala exactamente el costo (15 dls./sem.) de producir otra unidad.

17_CHAPTER 17.indd 570 3/6/09 8:14:16 PM


bienes públiCos 571

FIGURA 17.3
$/Q Suministro óptimo de
CM un bien público
D
el nivel óptimo de un bien
Curva agregada de la disposición a pagar público es Q* = 4, el nivel
en el cual la suma marginal
de la disposición a pagar
por el bien es exactamente
A* + B* = 15 igual a su costo marginal.
A
B
A* = 9

B* = 6 D’

B’ A’
Q
Q* = 4

Si no se mantiene esta igualdad, fácilmente se podría demostrar que la sociedad estaría mejor sin que
se expandiera o contrajera la cantidad de programación.

EJERCICIO 17.2
Considere el escenario que se describe en el ejercicio 17.1 y suponga que el costo marginal de
dar el concierto es CM = 2Q. Determine la duración óptima del concierto.

PAGO POR Q*
Se debe hacer una pequeña salvedad con respecto al enunciado de que Q* es el nivel óptimo del bien
público de la figura 17.3. El enunciado es verdadero sujeto a la previsión de que el costo total de Q*
no exceda la cantidad total que el público estaría dispuesto a pagar por él. La disposición total a pagar
por Q* es el área bajo la curva agregada de la disposición a pagar hasta Q*. El costo total es el área
bajo la curva del costo marginal hasta Q* más cualquier costo fijo. Siempre que el costo total sea me-
nor que la disposición total a pagar, Q* es el nivel óptimo del bien público. Esta salvedad es semejante
al requisito de que una empresa que maximiza sus utilidades produzca cuando IM = CM, sujeto a la
disposición de que los ingresos totales paguen los costos totales (los costos variables totales a corto
plazo y todos los costos a largo plazo).
Si el gobierno va a producir Q* unidades de un bien público, de algún modo debe recaudar
suficientes impuestos para saldar los costos totales de dicha cantidad. Para simplificar la exposición,
suponga que la estructura fiscal del gobierno exige una recaudación de impuestos igual para todos
los ciudadanos. En el ejemplo de la figura 17.3 la disposición de B a pagar por el bien público es
menor que la de A. Se deduce que B votará por el suministro de Q* sólo si su costo total es menor que
el doble del área bajo BB' hasta Q*. Por ejemplo, si el costo total del bien es 100 y cada parte tienen
el mismo gravamen, B votará por dicho bien sólo si su disposición total a pagar excede de 50. Si el
monto que B está dispuesto a pagar por el bien público es sólo, por ejemplo, 40, no queda satisfecha
la condición y el proyecto no gana la aprobación.
Pero se sabe que los beneficios de este bien público para todos los ciudadanos suman más que
sus costos. En comparación con la alternativa en la que no se provee el bien público, A y B están
mejor si se da una cantidad Q* del bien y luego se grava más a A que a B al pagarlo. Se concluye que

17_CHAPTER 17.indd 571 3/6/09 8:14:17 PM


572 Capítulo 17 gobierno

una estructura fiscal que recauda el mismo impuesto a todos los ciudadanos no puede ser eficiente
según el principio de Pareto.
La situación es análoga al caso en que los ingresos de una pareja son muy diferentes. Suponga
que Julie gana 200 000 dólares al año mientras su esposo, Bruce, gana sólo 30 000. Dado su ingreso,
Julie como individuo quisiera gastar mucho más que Bruce en vivienda, viajes, entretenimiento y
muchos otras cosas que consumen juntos. Pero suponga que la pareja adopta la regla de que tienen
que aportar una parte igual para la adquisición de esos bienes. El resultado sería restringir a la pareja
a vivir en una casa pequeña, tomar vacaciones baratas, saltarse los entretenimientos y las salidas a co-
mer, etc. Es fácil imaginar que a Julie le parecería atractivo pagar considerablemente más que 50% de
los bienes consumidos juntos, para que los dos gasten en la medida en que sus ingresos combinados
lo permiten.
Como en el caso de los bienes privados, la disposición a pagar por bienes públicos es una función
creciente del ingreso. En promedio, los ricos asignan mayor valor a los bienes públicos que los po-
bres, no porque tengan gustos distintos, sino porque tienen más dinero. Un sistema fiscal que grave
a los pobres tanto como a los ricos daría por resultado que los ricos tuvieran menos bienes públicos.
Para que no pase eso, los ricos aceptarían de buena gana un sistema impositivo que les asignara una
mayor proporción de la carga fiscal. Sería errado criticar el sistema diciendo que es injusto porque
permite al pobre disfrutar los servicios de los bienes públicos a un menor precio. Sin duda que tiene
esta prerrogativa, pero desde el punto de vista de los ricos, sus términos todavía son atractivos por-
que los pagos fiscales de los pobres, aunque pequeños, significan que los ricos paguen menos que si
tuvieran que financiar los bienes públicos por sí mismos.

SUMINISTRO PRIVADO DE BIENES PÚBLICOS


Los gobiernos no son los únicos que proporcionan bienes públicos en una sociedad. Cantidades sus-
tanciales de éstos se prestan de manera rutinaria a través de varios canales privados. Si no es práctico
impedir que las personas consuman un bien público, la pregunta es ¿cómo se paga un bien, si no es
mediante los impuestos obligatorios?

Financiamiento mediante donaciones


Un método para financiar los bienes públicos son las donaciones voluntarias. Las personas donan
obras maestras a los museos; hacen aportaciones a estaciones de radio patrocinadas por los escuchas,
a refugios de animales, a centros de investigación en enfermedades debilitantes, etc. Las motivacio-
nes para estas donaciones son tan variadas como los proyectos que apoyan. Algunos piensan que las
donaciones de beneficencia son una manera de ganar respeto y admiración en la comunidad.1 Otros
sienten la presión de dar para evitar el ostracismo social. En realidad, estas motivaciones son dos
lados de la misma moneda: la recompensa social en el primer caso y el castigo social en el segundo.
Cuando estas fuerzas sociales operan de manera efectiva, son una manera práctica de excluir a los
que no pagan del goce pleno del bien público.
Por otro lado, las personas donan porque el incremento del bien público que financia su aporta-
ción vale para ellas esa misma suma de dinero. Es más probable que esta motivación sea importante
en situaciones en las que los actos de una persona afectan de manera significativa la escala del bien
público. Por ejemplo, alguien que vive al final de un camino de terracería en una zona rural conside-
rará valioso pavimentar todo el camino a sus expensas. Naturalmente, estaría más contento si todos
los que viven junto al camino cooperaran, pero antes que quedarse sin el camino, considera valioso
pavimentarlo por su cuenta. Del mismo modo, una persona que planta un jardín de flores frente a
su casa ofrece un bien público que disfrutan sus vecinos. Si su disfrute del jardín supera su costo, el
interés personal es motivo suficiente para cultivarlo.
Pero los incentivos del interés personal no son suficientes para explicar por qué las personas
hacen donativos anónimos que no tendrán ningún efecto apreciable en los beneficios que reciban

1
Para materializar el beneficio social de una acción de beneficencia, la donación debe conocerse públicamente. Casi todas las organi-
zaciones de beneficencia hacen pública la lista de sus donantes.

17_CHAPTER 17.indd 572 3/6/09 8:14:17 PM


bienes públiCos 573

ellas mismas. En el caso de las estaciones patrocinadas por los escuchas, la aportación de una persona
cualquiera no marca una diferencia perceptible en la naturaleza o calidad de la programación. La
estación seguirá operando en la misma forma que hasta ahora, mejorará o empeorará independien-
temente de lo que haga una persona.
En estas situaciones la lógica del interés personal recomendaría el disfrute-libre, abstenerse con disfrute-libre escoger la no
la esperanza de que otros aporten. Sin embargo, millones de personas donan a estas organizaciones donación a una causa pero
todos los años. Para muchas, la satisfacción de dar (de haber contribuido al bien común) es un fin aun así beneficiarse de las
en sí mismo. Como se vio en el capítulo 7, ser una persona de ese talante acaso tenga importantes donaciones de otros.
ventajas materiales.
El hecho de que a veces los bienes públicos se mantengan mediante aportaciones voluntarias no
significa por fuerza que el apoyo alcance niveles socialmente óptimos. Los residentes pueden estar
del todo dispuestos a pagar suficientes impuestos para construir el camino socialmente óptimo; pero,
sin impuestos, el camino que al final se tienda quizá sea considerablemente más pequeño. Del mismo
modo, muchas personas quieren con firmeza que la sociedad invierta más en la programación de la
televisión abierta. Pero estas mismas personas se muestran reacias a dar de manera voluntaria lo que
estarían dispuestas a pagar en impuestos.

Venta de derivados
A veces los problemas del disfrute-libre se resuelven concibiendo medios novedosos para financiar el
bien público. Una manera es la venta de un importante derivado del bien público. Por ejemplo, en
el caso de la televisión comercial el financiamiento viene de los patrocinadores, en general corpora-
ciones privadas, que pagan por el derecho a difundir mensajes publicitarios a la audiencia atraída por
la difusora. Los televidentes cautivos son un derivado de la transmisión y los patrocinadores están
dispuestos a pagar mucho por tener acceso a ellos. Pero como se aclara en el ejemplo siguiente, el
sistema no siempre asegura una asignación óptima de los recursos de la transmisión.

En un horario dado, una cadena de televisión tiene la alternativa de difundir el Jerry Springer EJEMPLO 17.1
Show o Masterpiece Theater. Si escoge Springer, ganará 20% de la audiencia, pero sólo 18% si
escoge Masterpiece Theater. Suponga que quienes escogerían Jerry Springer estarían dispues-
tos a pagar, entre todos, 10 millones de dólares por el derecho a ver el programa, mientras
que los que escogerían Masterpiece Theater pagarían 30 millones. Por último, suponga que ese
horario lo financia una compañía de detergentes. ¿Qué programa escogerá la cadena? ¿Qué
programa es el óptimo para la sociedad?
Lo que al patrocinador le interesa principalmente es el número de personas que verán sus
anuncios y, entonces, escogerá el programa que atraiga más televidentes; en este caso, el Jerry
Springer Show. El hecho de que quienes prefieren Masterpiece Theater estén dispuestos a pagar mu-
cho más por verlo le importa poco al patrocinador. Pero esta diferencia en la disposición a pagar
es crítica cuando se trata de determinar el resultado óptimo desde el punto de vista de la sociedad.
Como las personas que prefieren Masterpiece Theater podrían indemnizar a los espectadores de
Springer por renunciar a su horario, Masterpiece Theater es un resultado de Pareto superior. Pero si
los interesados no compran más jabón, en total, que los espectadores de Springer, este último pre-
valecerá. La dificultad de depender de la publicidad y otros mecanismos indirectos para financiar
los bienes públicos estriba en que no se tiene la seguridad de que reflejen los beneficios pertinentes
para la sociedad.

Desarrollo de nuevos métodos para excluir a los que no pagan


Otra manera de financiar en forma privada los bienes públicos es diseñar nuevos métodos baratos
para excluir a las personas que no pagan por tales bienes. En la transmisión de televisión era imposi-
ble evitar que alguien sintonizara un programa una vez que había sido difundido. Pero con la llegada
de la televisión por cable, ahora es fácil hacer exclusiones. Con la capacidad para cobrar por determi-
nados programas, ya no es necesario tomar decisiones de programación con base en qué programa
cosecha la mayor audiencia. En el ejemplo de Jerry Springer comparado con Masterpiece Theater, una
difusora que pudiera excluir a los que no pagan tendría todos los motivos para transmitir Masterpiece

17_CHAPTER 17.indd 573 3/6/09 8:14:18 PM


574 Capítulo 17 gobierno

Theater porque sus defensores tienen un medio práctico de convertir su mayor disposición a pagar en
utilidades para el productor.
Pero observe que mientras el resultado de la televisión de paga es más eficiente en el sentido de
elegir los programas que más aprecia el público, es menos eficiente en otro aspecto importante. Al
cobrar a los hogares una tarifa por ver, desanima a algunos a prender la televisión. Y como el costo
social marginal de que un hogar adicional vea un programa es de exactamente cero, es ineficiente
limitar al público de esta manera. Una pregunta empírica es cuál de las dos ineficiencias es más im-
portante: la de la televisión abierta para escoger programas o la de la televisión de paga para excluir
a los beneficiarios potenciales.

Contratos privados
Los contratos legales entre particulares ofrecen otro medio de superar dificultades relativas al pro-
blema del disfrute-libre. Por ejemplo, considere el bien público que consiste en mantenimiento y
remozamiento. Como los vecindarios se organizan con base en la costumbre, no es práctico excluir a
sus vecinos de los beneficios que cosecharían si usted tiene su casa bien pintada y el pasto recortado.
Tampoco sería eficiente excluirlos porque su consumo de estos beneficios no mengua de ninguna
manera el valor que tienen para usted y otros. A este respecto, el mantenimiento y remozamiento de
los hogares satisface la definición de un bien público puro y, por este motivo, en general los particula-
res lo suministran en forma insuficiente.
En el capítulo 16 se vio que si las transacciones fueran gratuitas, sus vecinos podrían subvencio-
nar su inversión en el mantenimiento y remozamiento de su hogar y usted podría hacer lo mismo
por ellos. Establecidos en los niveles apropiados, los subsidios de este tipo darían por resultado niveles
óptimos de inversión para todos los propietarios. Pero, en general, es caro negociar estos subsidios
caso por caso, así que, al final, el nivel de inversión en mantenimiento doméstico termina por estar
debajo del nivel óptimo.
Los organizadores de condominios, cooperativas y otras formas de asociaciones residenciales
legales encontraron una solución eficaz al problema. En el contrato de condómino se exige a cada
propietario que aporte una suma específica cada mes para mantenimiento y remozamiento. Este
pago funciona en buena medida como impuesto, en el sentido de que es obligatorio para todas las
partes del contrato. La ventaja radica en que es menos coercitivo que un impuesto en un aspecto
importante: las personas que quieren gastar menos en mantenimiento doméstico son libres de vivir
en otra parte.
Ocurre una selección similar entre vecindarios que se encuentran en distritos escolares diferen-
tes. Un distrito puede decidirse a gastar más en las escuelas (y a que se cobren más impuestos para
financiarlas), en comparación con otro distrito contiguo. Los hogares se seleccionan por sí mismos:
las familias con hijos residen en la zona de más impuestos y los solteros y jubilados prefieren vivir en
el otro distrito.2

La economía de los clubes


Un bien público puro tiene la propiedad de que el consumo de una persona adicional no limita la can-
tidad de que disponen los otros. Dicho de otra manera, el costo marginal del consumo adicional de
un bien público es exactamente cero. Muchos bienes de producción privada tienen la propiedad de
que el costo marginal, aunque no es cero, baja notablemente con el número de usuarios atendidos.
Las albercas son un buen ejemplo. El número de nadadores que caben en una alberca se incrementa
en proporción con su superficie, pero el costo aumenta mucho más lentamente. La diferencia entre
estos bienes y los que satisfacen a la perfección el criterio de no disminución es de grado más que de
especie.
Cuando el costo marginal de extender la capacidad de un bien privado es poco en comparación
con su costo promedio, los consumidores tienen el incentivo económico de compartir la compra y el
uso del bien. En el ejemplo de la alberca, el costo para cada una de las 20 familias de una alberca de
tamaño suficiente para que la compartan todas será mucho menor que el costo de una cuyo tamaño

2
Vea también la exposición siguiente de los bienes públicos locales.

17_CHAPTER 17.indd 574 3/6/09 8:14:18 PM


bienes públiCos 575

sea apropiado para atender las necesidades de una sola familia. Sin duda, la misma afirmación es
verdadera para prácticamente cualquier bien que no mantenga en uso continuo una sola persona.
Por ejemplo, la mayoría de los propietarios de casas usan la escalera telescópica una o dos veces por
año, lo que posibilita que varias familias la compartan y reduzcan los costos.
La desventaja de compartir, aparte del hecho de que requiere que alguien tome la iniciativa de
organizar un acuerdo, es que limita la intimidad y la flexibilidad del acceso al bien. Así, el propietario
de una vivienda que quiera usar la escalera un sábado por la tarde, podría encontrarse con que la
están usando otros copropietarios. Algunas veces estos inconvenientes son triviales en comparación
con los ahorros, pero otras, no lo son tanto.
Por consiguiente, las oportunidades de compartir la propiedad plantean al consumidor una va-
riante del problema de decisión. Para ilustrarlo, considere de nuevo la elección entre una alberca
de propiedad privada y una compartida. Si se mide la intimidad y flexibilidad para usar una alberca
con una escala de 0 a 1.0, una alberca privada tomaría el valor de 1.0 para representar la máxima
intimidad y flexibilidad. El caso límite en el otro extremo es una gran alberca compartida por muchas
otras personas; el índice de flexibilidad de esta alberca tiene un valor de 0, que representa ninguna
privacidad ni flexibilidad.
El eje vertical en la figura 17.4 mide la cantidad que gasta el consumidor en todos los otros bie-
nes aparte de las albercas. Si compra su propia alberca privada, a un costo de Y' – Y0, tendrá un índice
de intimidad y flexibilidad de 1.0. El otro extremo representa una alberca atestada a un costo de 0 y
un índice de flexibilidad de 0. Las albercas de concurrencia y tamaño intermedio representan puntos
medios de la restricción de presupuesto BB' de la figura 17.4. La mejor opción del consumidor es
(F*, Y*) el punto donde la restricción de presupuesto es tangente a una curva de indiferencia (CI*).

FIGURA 17.4
Compensación entre
privacidad y costo
Todos los demás costos Cuando el costo marginal
de acomodar otro
usuario de un bien de
consumo es menor que
B el costo promedio, los
Y’
consumidores pueden
ahorrar dinero si forman
Y* clubes para compartir la
propiedad de los bienes. el
Y0 club óptimo para miembros
CI* con los mismos gustos
es uno en el que la tasa
marginal de sustitución
B’ entre todos los demás
Intimidad y flexibilidad
0 F* 1.0 bienes y la intimidad es
exactamente igual al costo
de la intimidad adicional.

Dada la suposición verosímil de que la demanda de intimidad aumenta con el ingreso, se podría
predecir que los consumidores con mayor ingreso tendrían más probabilidad de comparar su alberca
que los consumidores de menor ingreso. Pero incluso consumidores con ingreso muy elevado consi-
derarán atractivo concertar acuerdos de participación de bienes de consumo extremadamente caros.
Por ejemplo, en lugar de mantener los derechos exclusivos para operar un avión que esté ocioso en la
pista la mayor parte de la semana, incluso los pilotos aficionados acaudalados se hacen miembros de
clubes de vuelo en los que se comparte el uso de las naves.

17_CHAPTER 17.indd 575 3/6/09 8:14:19 PM


576 Capítulo 17 gobierno

En cambio, en el caso de bienes muy baratos se esperaría que la demanda de privacidad tu-
viera precedencia sobre el atractivo de ahorrar costos, aun entre los consumidores de medios más
modestos. Un cepillo de dientes de propiedad personal, como un avión o una piscina privados, está
destinado a permanecer ocioso la mayor parte de las horas del día. Su costo por propietario bajaría
sustancialmente si lo compartieran los miembros de un club de cepillos. Pero el ahorro de esa tran-
sacción sería demasiado pequeño para justificar el sacrificio de la intimidad. Prácticamente todos,
incluso los ciudadanos más pobres, aprecian el acceso exclusivo a su propio cepillo de dientes.

ELECCIÓN PÚBLICA
Si los bienes públicos son suministrados por gobiernos, organizaciones de beneficencia o clubes pri-
vados, hay que tomar decisiones sobre los tipos y cantidades por ofrecer. La restricción presupuesta-
ria que enfrenta el grupo suele ser clara. El aspecto más difícil del problema es concebir algún medio
para traducir las preferencias de los miembros del grupo en una sola voz.

VOTO DE LA MAYORÍA
Un método para discernir las preferencias de un grupo es el voto de la mayoría. Según este esque-
ma, los proyectos favorecidos por la mayoría (en un referendo directo o por voto de representantes
electos) se adoptan y los demás se abandonan. En los últimos años se ha prestado mucha atención al
hecho de que la mayoría de los votos llevan a intransitividades en la clasificación de las alternativas.
Para ejemplificarlo, considere un grupo con tres miembros (McCain, Biden y Schumer), cada uno
de los cuales tiene una jerarquía ordenada de proyectos alternativos: un nuevo misil, un proyecto
de investigación médica y ayuda a los más pobres. McCain tiene en primer lugar a los misiles, la
investigación médica en segundo lugar y la ayuda a los pobres en último lugar. Biden prefiere la
investigación médica, después la ayuda a los pobres y por último los misiles. Por último, Schumer
prefiere la ayuda a los pobres, luego un nuevo misil y por último la investigación médica. En la tabla
17.1 se resumen estas jerarquías.

TABLA 17.1
Preferencias que producen elecciones intransitivas en el voto de la mayoría en el voto de la
mayoría, el misil le
gana al programa
McCain Biden Schumer de investigación y
éste a la ayuda a
Preferida Misil investigación ayuda los pobres. pero la
ayuda a los pobres
Segunda investigación ayuda Misil le gana al misil.
los esquemas de
Tercera ayuda Misil investigación votación mayoritaria
pueden llevar a
intransitividades
aunque el orden de
las preferencias de
los individuos sea
transitivo.

Dados estos ordenamientos, observe lo que pasa cuando se someten a voto los tres pares de al-
ternativas. Al decidir por qué par de alternativas votar, cada votante escogerá, como es natural, la que

17_CHAPTER 17.indd 576 3/6/09 8:14:19 PM


eleCCión públiCa 577

prefiera y no la otra. Así, en un voto entre un nuevo misil e investigación médica, el misil obtiene dos
votos (McCain y Schumer); la investigación sólo uno (Biden). En un voto entre investigación y ayuda
a los pobres, la investigación obtiene dos votos (McCain y Biden); la ayuda, un voto (Schumer). Por
último, en un voto entre ayuda y el misil, la ayuda recibe dos votos (Schumer y Biden); el misil, uno
(McCain). Así, el misil gana al programa de investigación y el programa de investigación le gana a la
ayuda a los pobres; pero la ayuda a los pobres le gana al misil. Se suponía que estas intransitividades
no sucedían en el ordenamiento de las preferencias personales, pero es fácil que ocurran si la elección
social se produce por mayoría sucesiva de votos entre pares de alternativas.

Manipulación de los procesos electorales


La posibilidad de intransitividades en el voto de la mayoría vuelve de importancia crítica el orden
en que el electorado considera las alternativas. Por ejemplo, suponga que McCain está a cargo de
fijar el procedimiento de votación. Su prioridad será evitar una confrontación directa entre el misil
(el proyecto que más apoya) y el proyecto de ayuda (que sabe le ganará al misil por mayoría de
votos). Puede garantizar el triunfo del misil si realiza una votación entre el proyecto de ayuda y el
de investigación, seguida por una votación entre el ganador de la elección y el misil. El proyecto de
investigación gana la primera votación y luego pierde ante el misil en la segunda. Si tuvieran el poder
de fijar el procedimiento, Biden o Schumer tomarían medidas semejantes para asegurar la victoria,
del proyecto de investigación o del proyecto de ayuda.

Teorema del votante medio


No siempre se producen ordenamientos intransitivos cuando se consideran pares de alternativas en
un sistema de voto mayoritario. Por ejemplo, no se tienen intransitividades cuando las alternativas
representan diferentes cantidades de un bien público y cada votante las ordena de acuerdo con cuán-
to se aproximen a lo que le parezca que es la cantidad óptima del bien. Para ilustrarlo, suponga que
los tres votantes ponderan qué porcentaje del PIB debe dedicarse a la defensa nacional. Suponga tam-
bién, como se muestra en la figura 17.5, que los porcentajes ideales para McCain, Biden y Schumer
son 50, 6 y 10, respectivamente. Por último, considere que los porcentajes propuestos para adoptarse
son 5, 8, 11, 20, 40 y 60.

FIGURA 17.5
El poder del votante
medio
Biden McCain las opciones preferidas de
biden, schumer y McCain
6 50
son 6, 10 y 50%, de modo
que schumer es el votante
0 10 100
medio. no importa qué par
Schumer PIB dedicado de porcentajes propuestos
a la defensa,% para defensa se someta a
votación, gana el que más
se acerque a la opción
preferida por schumer.

¿El poder de fijar el orden en que se consideran los pares de alternativas confiere el poder de
elegir el resultado final? En esta ocasión la respuesta es no. En cualquier par de alternativas sometido
a voto, la ganadora siempre será la que prefiera Schumer. Por ejemplo, suponga que se someten a vo-
tación cinco y ocho. McCain y Schumer votarán por ocho y Biden por cinco, de modo que la elección
de Schumer gana. Si las alternativas son 20 y 60, Biden y Schumer votarán por 20 y McCain por 60 y

17_CHAPTER 17.indd 577 3/6/09 8:14:20 PM


578 Capítulo 17 gobierno

Schumer vuelve a ganar. Como el resultado preferido por Schumer está entre las que más prefieren
votante medio votante cuyo los otros dos, es el llamado votante medio y en esta situación su voto prevalece siempre. El teorema
resultado ideal queda arriba del votante medio establece que cuando las alternativas se ordenan de acuerdo con su proximidad al
de los resultados ideales de la resultado ideal de cada votante, el voto de la mayoría siempre escogerá la alternativa que prefiera el
mitad de los votantes. votante medio.

EJERCICIO 17.3
Dado que los porcentajes del PIB considerados para gastarse en defensa nacional son de nuevo
5, 8, 11, 20, 40 y 60, ¿qué resultado se escogerá si los porcentajes preferidos de Biden, Schumer y
McCain son 11, 25 y 40, respectivamente?

La característica técnica de las preferencias que elimina las intransitividades en el ejemplo del
preferencias exclusivas gasto de defensa se llama preferencias exclusivas. Tener preferencias exclusivas con respecto a la
preferencias que exhiben un proporción del PIB que se gaste en defensa significa tener un único resultado preferido y ordenar los
único resultado preferido. demás resultados según su distancia respecto de éste. Tales preferencias excluyen la preferencia por
los demás resultados se el 10% y luego dan mejor lugar a 30 que a 20%.
rezagan conforme aumente En contextos como el ejemplo de la defensa parece viable suponer que las preferencias son real-
su distancia con respecto al mente exclusivas. Pero en otros contextos, como en el ejemplo del misil, la ayuda y la investigación,
resultado preferido.
las preferencias no tienen que tener esta propiedad. En la práctica se presentan numerosos ejemplos
importantes en los que el voto de la mayoría produce ordenamientos intransitivos, por lo que el
poder de fijar el programa equivale a tener el poder de escoger el resultado final.

ANÁLISIS COSTO-BENEFICIO
La dificultad de la elección pública mediante el voto de la mayoría no es sólo que en ocasiones lleva
a intransitividades. Un problema más grave es que oculta casi por completo importantes diferencias
en la intensidad con la cual los votantes sostienen sus preferencias. Por ejemplo, suponga que se vota
entre dos alternativas: 1) tolerar que se fume en lugares públicos, 2) prohibir que se fume en dichos
sitios. Si 51% de los votantes prefieren la primera alternativa y 49% la segunda, el resultado será
tolerar que se fume en lugares públicos. Pero suponga que el 49% que está a favor de la prohibición
tienen convicciones muy firmes al respecto y en conjunto estarían dispuestos a pagar 100 millones
de dólares al año con tal de tener la prohibición. Suponga también que quienes se oponen a la prohi-
bición sólo sienten un rechazo moderado; saben que en el corto plazo les representa inconvenientes,
pero de cualquier manera casi todos quieren fumar menos o dejar de hacerlo y se dan cuenta de que
la ordenanza les ayudaría a lograr este fin. En conjunto, lo más que estarían dispuestos a pagar por
seguir fumando en instalaciones públicas es 1 millón de dólares por año. En estas circunstancias hay
un simple pago de transferencia que hace que el resultado elegido por la mayoría, en términos del
principio de Pareto, sea inferior a la prohibición de fumar en instalaciones públicas. Si los prohibicio-
nistas dieran a los fumadores 10 millones de dólares anuales a cambio de que acepten el bando, los
dos grupos estarían mejor que si continuara el tabaquismo: los fumadores por 9 millones de dólares
anuales; los no fumadores, por 90 millones al año.
El análisis costo-beneficio es una alternativa al voto de la mayoría con el que se pretende hacer
un recuento explícito de la firmeza de las opiniones de las personas acerca de las alternativas que se
ponderan. El método para medir la intensidad de las preferencias consiste en calcular cuánto estarían
dispuestas a pagar las personas para tener las diversas alternativas. En el ejemplo del tabaquismo,
inmediatamente se decidiría a favor de la prohibición, porque los beneficios para sus defensores (me-
didos por lo que estén dispuestos a pagar por ella) superan por mucho los costos de sus oponentes
(medidos por lo que estarían dispuestos a pagar por evitarla).
Otra ventaja del análisis costo-beneficio es que también se evitarían las intransitividades que
surgen con la votación de la mayoría. Para ilustrarlo, considere cómo se manejaría le decisión sobre
el misil, la ayuda y la investigación que se vio antes. En la tabla 17.2 se anotan valoraciones hipoté-
ticas asignadas por McCain, Biden y Schumer a cada una de las tres alternativas. Las entradas posi-

17_CHAPTER 17.indd 578 3/6/09 8:14:21 PM


eleCCión públiCa 579

tivas de la tabla representan las cantidades que pagaría cada uno por tener el programa que quiere.
Las entradas negativas representan lo que pagarían por evitar los programas que no les gustan. Por
ejemplo, las entradas de la primera columna indican que McCain pagaría 100 por tener el programa
del misil, 35 por el programa de investigación médica y 20 por evitar el programa de ayuda para los
pobres. Observe que el ordenamiento de las alternativas de cada persona es el mismo en la tabla
17.2 que en la 17.1.

TABLA 17.2
Disposición a pagar por los tres proyectos
el análisis costo-beneficio
señala el proyecto con
McCain Biden Schumer Total el mayor excedente de
beneficios sobre los costos.
Misil 100 –25 45 120 si cada proyecto cuesta 100,
el excedente será 20 para
investigación 35 90 40 165 el misil, 65 para el programa
de investigación y 35 para el
ayuda –20 60 95 135 programa de ayuda. por lo
tanto, la prueba de costo-
beneficio selecciona el
programa de investigación.

Para mantener simple la exposición, suponga que el costo de cada programa es 100, pero que
por escasez presupuestal sólo puede emprenderse uno de los tres programas. ¿Cómo se escogería
uno mediante al análisis costo-beneficio? Se elegiría aquel en que el excedente del beneficio total
sobre el costo fuera mayor. También por simplicidad suponga que las cantidades que los tres vo-
tantes pagarían por cada programa capta bien todos los beneficios pertinentes. El beneficio total
de cada programa será, pues, la suma de lo que cada votante habría pagado por tenerlo o evitarlo.
Estos totales se anotan en la última columna de la tabla 17.2, en la que se indica que el programa de
investigación es el ganador.
Observe también que el programa de investigación no habría ganado si McCain hubiera fija-
do el procedimiento electoral en una sesión de voto de la mayoría. Primero hubiera enfrentado el
proyecto de investigación y el de ayuda, al que habría derrotado dos a uno. Además, el proyecto del
misil, favorito de McCain, derrotaría al proyecto de investigación por el mismo margen. Schumer, a
través de una manipulación equivalente del procedimiento, organizaría que su proyecto favorito, el
de ayuda, terminara como ganador en una secuencia de voto de la mayoría.
Por último, observe que si no se adopta el proyecto de investigación, de todos modos es posi-
ble elaborar un intercambio de Pareto con él. Por ejemplo, supongamos que McCain establece el
programa de la secuencia de votación de la mayoría y el resultado es que se escoge el proyecto del
misil. Esto da una pérdida de 25 para Biden. Por el contrario, si se hubiera escogido el proyecto de
investigación, Biden hubiera tenido una ganancia de 90, una mejora neta de 115 para él, que es bas-
tante grande para compensar a McCain y Schumer por las pérdidas que habrían sufrido por cambiar
el proyecto del misil por el de investigación. Por ejemplo, suponga que Biden da 70 a McCain y 10
a Schumer para que se cambien. Entonces, los beneficios netos para McCain y Schumer serían de
105 y 50, respectivamente, una ganancia de cinco sobre sus posiciones con el proyecto del misil. Es
posible realizar una mejora de Pareto semejante si se hubiera comenzado con el proyecto de ayuda.
De hecho, la prueba de costo-beneficio lleva a un resultado eficaz de Pareto.
Si el análisis costo-beneficio satisface el criterio de Pareto, pero no el voto de la mayoría (o, por lo
menos, no siempre), ¿por qué acudimos tan a menudo a la votación para tomar decisiones colectivas?
Una objeción al análisis costo-beneficio es que como mide beneficios según lo que la gente estaría
dispuesta a pagar, da un peso insuficiente al interés de las personas que tienen poco dinero. Según

17_CHAPTER 17.indd 579 3/6/09 8:14:21 PM


580 Capítulo 17 gobierno

esta opinión, los pobres podrían tener una opinión muy firme sobre un tema, pero sus impresiones
no contarían mucho en un análisis costo-beneficio porque no se traducen en cifras grandes de dispo-
sición a pagar. A primera vista parece una objeción grave, pero como se demuestra en el ejemplo que
sigue, no resiste un escrutinio más detenido.

EJEMPLO 17.2 Suponga que sólo hay dos personas, R (que es rico) y P (que es pobre). Suponga también que
R está a favor de un proyecto público al que P se opone. En términos puramente psicológicos,
la intensidad de sus sentimientos es la misma. Pero como R tiene mucho más dinero, estaría
dispuesto a pagar 100 por el proyecto, mientras que P pagaría 10 por evitarlo. Si pudieran
escoger un método para decidir sobre los proyectos públicos, ¿cuál favorecería cada uno, el
análisis costo-beneficio o el dictado de la mayoría?
A primera vista el dictado de la mayoría suena atractivo porque le daría a P un poder de veto sobre
cualquier proyecto que no le favoreciera. Pero el primer paso que tomaría P si tuviera poder de veto
sería renunciar a cambio de una indemnización. Si R valora el proyecto en 100 y P pagaría sólo 10
para evitarlo, el resultado más eficiente es seguir con el proyecto. Si R da a P una indemnización de
X, de manera que 10 ≤ X ≤ 100, cada parte estaría mejor que en el caso que P insistiera en ejercer su
veto. Según los cálculos de P, la inconveniencia del proyecto es menor que el valor que tiene para él
la indemnización. El hecho de que la prueba costo-beneficio conduzca siempre al mayor excedente
económico significa que a R y P invariablemente les convendrá aplicarla.

Los críticos del análisis costo-beneficio conceden que llevaría a resultados óptimos de Pareto en
todos los casos si fuera práctico pagar las indemnizaciones necesarias. Pero luego argumentan que
éstas no son prácticas tratadas caso por caso. Además, concluyen, es injusto tomar decisiones con
base en dicho análisis.
Este argumento tampoco resiste un examen detallado. En primer lugar, observe que en la ma-
yor parte de las sociedades todos los años se toman literalmente miles de decisiones con respecto
a proyectos y bienes públicos. Los que se adoptan sirven a unos y perjudican a otros. Casi siempre,
la magnitud de las pérdidas y ganancias de una decisión cualquiera es extremadamente pequeña,
mucho menos de 1% de las ganancias anuales incluso de una persona pobre. Si los proyectos se de-
ciden de acuerdo con el criterio costo-beneficio, el monto que reciben los ganadores por cualquier
proyecto adoptado supera necesariamente el monto que pierden los derrotados. Entonces, cuando
se trata de proyectos pequeños, la prueba de costo-beneficio es como arrojar una moneda que está
trucada a nuestro favor. En cada lanzamiento se puede perder o ganar, pero la probabilidad de ga-
nar excede a la de perder. Si ganancias y pérdidas son pequeñas y siguen una distribución aleatoria
entre los individuos, y si la moneda se lanza miles de veces, es una apuesta muy atractiva. La ley de
los números grandes (véase el capítulo 6) establece que es casi seguro que todos resulten ganadores
al final.
Pero suponga que las ganancias y pérdidas de cada resultado no son aleatorias; que, por lo contra-
rio, a los pobres les sale el lado perdedor de la prueba costo-beneficio por su incapacidad para respaldar
sus proyectos favoritos con valores mayores de disposición a pagar. Aun si es imposible compensar a
los pobres caso por caso, todavía es posible alcanzar un mejor resultado para todos apoyándose en el
criterio costo-beneficio. La explicación es que se puede compensar constantemente a los pobres me-
diante el sistema fiscal. Si la alternativa es confiar en el voto de la mayoría, con el cual los pobres podrían
bloquear los proyectos cuyos beneficios superan los costos, el criterio costo-beneficio junto con una
indemnización a través del sistema fiscal daría los mejores resultados para todas las partes.
El único argumento a favor de la votación mayoritaria es su simplicidad. Es mucho más fácil
hacer una votación mayoritaria que reunir información detallada acerca de lo que cada individuo es-
taría dispuesto a pagar por sus alternativas preferidas. En los últimos años se ha avanzado mucho en
el diseño de mecanismos que inducen a las personas a revelar sus verdaderas valoraciones. Pero éstos
siguen siendo complicados y es mucho más fácil dejar que la gente revele sus preferencias por medio
del voto. Y, desde luego, en muchas situaciones la votación mayoritaria y el análisis costo-beneficio
dan el mismo resultado.

17_CHAPTER 17.indd 580 3/6/09 8:14:21 PM


eleCCión públiCa 581

BIENES PÚBLICOS LOCALES Y EL MODELO DE TIEBOUT


Aun con un mecanismo perfecto para elegir entre bienes públicos alternativos, es difícil evitar la
necesidad de una dolorosa solución de compromiso. Un grupo creerá sinceramente que es deber
de la sociedad proporcionar cuidado de la salud a todos los ciudadanos; otro pensará con la misma
sinceridad que es responsabilidad de cada individuo proporcionarse estos servicios. En vista de esta
clase de diferencias, a menudo el resultado es una forma de solución de compromiso (apoyo público
parcial para el cuidado de la salud) que complace a ambos grupos de votantes.
En cuanto a los bienes públicos proporcionados en el nivel local, el profesor Charles Tiebout
sugiere que es posible evitar por lo menos algunos de estas soluciones de compromiso si la gente es
libre de formar comunidades con otras personas que tienen gustos similares.3 Aquellos que favore-
cen altos niveles de bienes públicos se pueden agrupar en comunidades en las que están dispuestos a
aceptar las elevadas tasas fiscales necesarias para financiarlos. Y quienes favorecen una variedad más
limitada de bienes y servicios públicos pueden formar grupos propios y tener tasas fiscales más bajas.
Como una cuestión empírica, los gobiernos locales difieren en gran medida en cuanto al nivel
de bienes públicos que proporcionan. Aun así existen dificultades prácticas respecto a la idea de tra-
tar de adaptar un ambiente local a las preferencias de cada persona. Considere, por ejemplo, el asun-
to del apoyo público para los pobres. La gente tiene diferencias legítimas sobre el nivel de ayuda
que debe proporcionar. Pero quienes favorecen los niveles de apoyo alto a menudo gastan más de
lo que negociaron cuando fueron promulgadas políticas de bienestar generosas en el ámbito local. La
dificultad es que dichas políticas atraen nuevos beneficiarios de bajos ingresos de otras jurisdicciones
donde los beneficios son más bajos. Esto, a su vez, hace que sea necesario elevar las tasas fiscales, lo
que provoca la salida de algunos contribuyentes de altos ingresos, aumentando aún más el desequili-
brio fiscal. La capacidad de formar comunidades locales de votantes con formas de pensar similares
reduce la necesidad de compromiso en algunas áreas, pero de ninguna manera la elimina.

VIVIR DE LAS RENTAS


En forma práctica, las ganancias derivadas de las elecciones públicas a menudo crecen y se concen-
tran en manos de unos cuantos, mientras los costos, aunque también son altos, se distribuyen entre
muchos. La dificultad que estas situaciones crean para el público es evidente. Los beneficiarios po-
tenciales de un programa público tienen incentivos poderosos para apoyar al gobierno, mientras
que cada uno de los posibles perjudicados tiene muy poco que perder. Con mucha frecuencia, el
resultado es que los proyectos se aprueban aun cuando sus beneficios no excedan sus costos.
Una dificultad relacionada surge en el caso de proyectos similares cuyos beneficios exceden
los costos. Como hay ganancias muy altas por parte del proyecto, los participantes privados están
dispuestos a invertir grandes sumas con el fin de mejorar sus probabilidades de ser elegidos como
beneficiarios. La búsqueda de estas ganancias recibe el nombre de búsqueda de concesiones, una de
cuyas consecuencias es que las ganancias esperadas de los proyectos del gobierno a menudo se ven
reducidas por la competencia entre los posibles beneficiarios.
Considere, por ejemplo, un gobierno local que se enfrenta a la tarea de otorgar la franqui-
cia de televisión por cable local. A menos que el gobierno esté preparado para comprometerse en
regulaciones estrictas del índice de utilidades, lo que no sucede con la mayor parte de los gobiernos
locales, el franquiciatario puede esperar obtener utilidades de monopolio muy elevadas. La probabi-
lidad de que cualquier solicitante consiga la franquicia es una función cada vez mayor de la cantidad
de dinero que invierte en apoyar a los legisladores locales. Por lo tanto, el atractivo de las ganancias
esperadas de la franquicia hace que los solicitantes participen en una guerra por obtenerla. Y, como
ilustra el ejemplo siguiente, estas guerras suelen disipar gran parte de las ganancias que el proyecto
hace posible.

3
Charles Tiebout, “The Pure Theory of Local Expenditure”, en Journal of Political Economy, octubre de 1956, pp. 416-424.

17_CHAPTER 17.indd 581 3/6/09 8:14:22 PM


582 Capítulo 17 gobierno

EJEMPLO 17.3 Tres empresas cumplieron con la fecha límite para solicitar la franquicia para operar el
sistema de televisión por cable de Cedar Rapids durante el próximo año. El costo anual de
operación del sistema es 25, y la curva de demanda de sus servicios se da mediante P = 50
– Q, donde P es el precio anual por suscriptor y Q es el número de suscriptores. La franquicia
dura exactamente un año y permite al franquiciatario cobrar el precio que elija. El concejo
o ayuntamiento de la ciudad elegirá al solicitante que invierta más dinero en la negociación
con sus miembros. Si los solicitantes no se pueden coludir, ¿cuánto va a gastar cada uno en
ese apoyo?
El ganador establecerá el precio de monopolio para el servicio, que corresponde a la cantidad en la
que el ingreso marginal es igual al costo marginal. El ingreso marginal para el sistema de cable se
da mediante IM = 50 – 2Q y el costo marginal es cero por suposición. La cantidad que maximiza
la utilidad por lo tanto es 25, lo que da lugar a un precio de 25. La ganancia total será 25(25) = 625,
que representa una utilidad de 625 – 25 = 600. Si cualquiera de los solicitantes gasta más en apoyar
a la ciudad que los otros dos, ganará la franquicia. Si los tres gastan lo mismo, cada uno tendrá una
probabilidad de 1 en 3 de obtener una utilidad de 600, lo que significa una utilidad esperada de 200.
Si los negociadores se pudieran coludir, cada uno acordaría invertir la misma cantidad. Pero a falta
de un acuerdo, cada uno se sentirá tentado a tratar de gastar más que los demás. Si el gasto de cada
empresa llega a 200, cada uno tendrá utilidades esperadas de cero (una posibilidad de un tercio de ga-
nar 600 menos 200 gastados en la negociación o cabildeo). En este punto, parecería absurdo ofrecer
más porque los niveles de gasto más alto significarían una pérdida esperada. No obstante, si alguno
de los tres gasta 201, mientras los otros dos se quedan en 200, seguramente obtendrá la franquicia
y ganará una utilidad neta de 399. Cada uno de los perdedores tendrá pérdidas de 200. En lugar de
enfrentar una pérdida segura de 200, a los perdedores les resultará muy atractivo ofrecer 201. Nadie
sabe dónde parará este proceso.4 Lo único que parece seguro es que disipará parte o todas las ganan-
cias que se podrían haber obtenido del proyecto. Desde el punto de vista de una empresa individual
es perfectamente razonable cabildear de esta manera para obtener los beneficios del gobierno. Sin
embargo, desde la perspectiva de la sociedad en general esta actividad es prácticamente un desperdi-
cio. El gobierno eficiente es aquel que realiza cualquier acción factible para desalentar la búsqueda de
concesiones; por ejemplo, al seleccionar a los contratistas con base en el precio que prometen cobrar
y no en la cantidad con la que lo apoyan.

DISTRIBUCIÓN DEL INGRESO


En las economías de mercado el principal medio para obtener ingresos es vendiendo factores de pro-
ducción. Algunas personas, que con mucho constituyen una minoría, ganan una parte importante
de sus ingresos de la propiedad de acciones, bonos y otros instrumentos financieros. La mayor par-
te de la gente depende sobre todo de las ganancias de la venta de su propio trabajo.
Este sistema de distribución de los ingresos dista mucho de ser perfecto, pero ofrece varias
propiedades atractivas. En primer lugar, lleva a un resultado determinado: la teoría de los merca-
dos competitivos de factores indica que cada factor se va a pagar al valor de su producto marginal,
y que en un equilibrio competitivo a largo plazo, estos pagos sumarán exactamente el producto
total disponible para la distribución.5 Dada la posibilidad obvia de que las demandas excedan la
producción disponible en cualquier sistema, el hecho de que el esquema de productividad mar-

4
El siguiente experimento ofrece algunas evidencias relevantes. Se subasta un billete de un dólar con base en las reglas que se expli-
can a continuación. El billete será para el mejor postor, quien deberá pagar al subastador la cantidad que ofrezca. El segundo mejor
postor no recibe nada, pero también deberá pagar al subastador la cantidad de su licitación. En un ensayo típico de esta subasta,
las licitaciones se acercan lentamente a los 50 centavos, punto en el que se hace una pausa. Luego, el segundo licitador ofrece más
de 50 centavos y las licitaciones aumentan con rapidez hasta llegar a 1 dólar. Hay otra pausa en esta cantidad, después de la cual el
segundo licitador ofrece más de 1 dólar y las licitaciones aumentan otra vez en poco tiempo. No es raro que la licitación ganadora
sea de varios dólares.

17_CHAPTER 17.indd 582 3/6/09 8:14:22 PM


distribuCión del ingreso 583

ginal identifique con claridad un pago factible de cada parte no es una ventaja despreciable. Una
segunda característica atractiva del sistema de productividad marginal es que recompensa la ini-
ciativa, el esfuerzo y la aceptación de riesgos. Mientras más duro, largo y efectivo sea el trabajo
de una persona, más alto será su pago. Y si arriesga su capital en una empresa que tiene éxito,
cosechará altos dividendos.

CRÍTICA DE RAWLS AL SISTEMA


DE PRODUCTIVIDAD MARGINAL
Sin embargo, el sistema de productividad marginal tiene algunas fallas. La crítica más común es
que a menudo genera un alto grado de desigualdad. Aquellos que tienen un buen desempeño en el
mercado terminan con mucho más dinero del que pueden gastar, mientras que quienes fracasan a
menudo no pueden satisfacer sus necesidades básicas. Esta desigualdad sería más fácil de aceptar si
se debiera estrictamente a diferencias en el esfuerzo. Pero no es así. El talento desempeña un papel
importante en casi todas las tareas, y aunque es posible nutrirlo y desarrollarlo cuando se tiene, el
hecho de tenerlo es, en esencia, una cuestión de suerte.
Incluso el hecho de tener mucho talento no es una garantía de éxito. También es necesario tener
el talento correcto. El hecho de lanzar una bola de beisbol a una gran distancia en forma consistente
puede representar una ganancia de millones de dólares al año, mientras que ser el mejor maestro
de cuatro grado del país le dará una ganancia muy baja; y ser el mejor jugador de balonmano en el
mundo no le dará ninguna ganancia. La estrella de beisbol gana mucho más no sólo porque trabaja
más o tiene más talento, sino porque tiene la suerte de ser bueno en algo por lo que la gente está
dispuesta a pagar mucho dinero.
John Rawls, filósofo moral de Harvard, elaboró una crítica ética del sistema de productividad
marginal, basada sobre todo en la teoría microeconómica de la elección. La pregunta que formuló
es: “¿Qué constituye una distribución justa del ingreso?” Para responderla, propuso el siguiente
experimento de pensamiento. Imagine que usted y otros ciudadanos de algún país se encuentran
en una reunión para elegir las reglas de distribución del ingreso. Esta reunión tiene lugar detrás de
un “velo de ignorancia”, que oculta a cada persona todos los talentos y habilidades que tanto ella
como los demás tienen. Ninguno de los individuos sabe si es inteligente o no, fuerte o débil, rápido
o lento, etc., lo que significa que nadie sabe qué reglas de la distribución en particular podrían
funcionar para su beneficio. Rawls argumenta que las reglas que la gente elegiría en este estado de
ignorancia necesariamente serían justas; y si las reglas son justas, la distribución a la que den lugar
también lo será.
¿Qué reglas elegiría la gente desde atrás de un velo de ignorancia? Si el ingreso nacional por
distribuir fuera una cantidad fija todos los años, es probable que la mayoría decidiera dar a todos una
parte equitativa. Es muy probable, según Rawls, porque la mayoría de las personas siente aversión
por el riesgo. Como una distribución desigual comprendería no sólo una oportunidad de hacerlo
bien, sino también de hacerlo mal, la mayoría de las personas preferiría eliminar el riesgo seleccio-
nando una distribución equitativa.
Sin embargo, la dificultad radica en que la cantidad total de ingreso disponible para su distribu-
ción no es fija todos los años. En vez de ello, depende del trabajo de la gente, las iniciativas y riesgos
que enfrenten, etc. Si a todos se les garantizara una parte equitativa del ingreso nacional desde un
principio, ¿para qué trabajarían más o correrían riesgos? Sin una recompensa por el trabajo arduo y
los riesgos enfrentados, el ingreso nacional sería mucho menor que con la existencia de esas recom-
pensas. Desde luego, las recompensas materiales por el esfuerzo y el riesgo llevan necesariamente a
la desigualdad. Pero Rawls argumenta que la gente estaría dispuesta a aceptar cierto grado de des-

5
Recuerde que el equilibrio competitivo a largo plazo ocurre en el punto más bajo de la curva de costo promedio a largo plazo
de cada empresa; en ese punto existen ganancias constantes en la producción. Una propiedad de las funciones de producción con
ganancias constantes es que F(K, L) = K∂F/∂K + L∂F/∂L, que indica que pagar a cada factor su producto marginal permitirá que se
agote el total del producto disponible.

17_CHAPTER 17.indd 583 3/6/09 8:14:22 PM


584 Capítulo 17 gobierno

igualdad siempre y cuando las recompensas produjeran un incremento suficiente en la cantidad total
de producción disponible para su distribución.
¿Cuánta desigualdad? Mucho menos que la producida por los mercados de factores pura-
mente competitivos, argumenta Rawls. La idea es que cada persona detrás del velo de ignorancia
sentiría un temor racional a estar en una posición de desventaja, de modo que cada uno elegiría
las reglas de distribución que maximizaran el ingreso de los ciudadanos más pobres. Es decir, una
desigualdad adicional se podría considerar justificada siempre y cuando tuviera el efecto de au-
mentar el ingreso de todos y cada uno de los ciudadanos. Las propias críticas a la teoría de Rawls
señalan que esta propuesta es demasiado conservadora; que la mayor parte de la gente permitiría
una desigualdad adicional si el efecto fuera, por ejemplo, aumentar casi todos los ingresos. Pero el
punto básico de Rawls es que la gente detrás del velo de ignorancia elegiría las reglas que produ-
jeran una distribución del ingreso más equitativa que con el sistema de productividad marginal. Y
como estas elecciones definen lo que constituye una distribución justa del ingreso, argumenta, la
justicia requiere por lo menos de un intento para reducir la desigualdad producida por el sistema
de productividad marginal.

RAZONES PRÁCTICAS PARA LA REDISTRIBUCIÓN


El argumento moral de Rawls tiene una fuerza obvia. Pero también existen razones prácticas muy
atractivas para limitar la desigualdad. Ya se vio, por ejemplo, que un impuesto igual cobrado a cada
ciudadano dará como resultado un nivel deficiente de bienes públicos. En la medida en que la dis-
posición a pagar por los bienes públicos aumenta con el ingreso, los ciudadanos con un ingreso alto
apoyarán una estructura fiscal en la que reciban una mayor participación de la carga fiscal que el
pobre. Y en la medida en la que los bienes públicos financiados con este sistema fiscal estén dispo-
nibles de manera equitativa para personas con distintos niveles de ingreso, el efecto será reducir la
desigualdad.
Fuerzas análogas a las que dan forma a la distribución de los pagos en las empresas sugieren otra
razón práctica para la redistribución del ingreso en la sociedad. Recuerde que en el capítulo 14 se vio
que dentro de cualquier empresa la tendencia es que a los empleados más productivos se les pague
menos de los valores de sus productos marginales, y a los empleados menos productivos se les
pague más. La diferencia entre el sueldo de un trabajador y el valor de su producto marginal se puede
interpretar como una diferencia de compensación que refleja su rango en la empresa. Los grupos he-
terogéneos de trabajadores permanecerán unidos en una empresa sólo si aquellos que ocupan puestos
de nivel inferior reciben una compensación adecuada por parte de quienes tienen un rango elevado.
Estas fuerzas en la empresa se reflejan también en la sociedad. Es obvio que existe una ventaja
por ocupar un puesto en la parte superior de la distribución del ingreso en la sociedad. Sin embargo,
estas posiciones existen sólo si hay otras personas dispuestas a ocupar puestos en la parte inferior de
la distribución del ingreso. La sociedad tiene un interés muy claro en establecer términos con los
cuales todos sus miembros consideren que conviene a sus intereses seguir siendo parte de la misma.
Si la experiencia sirve de guía, la cohesión social simplemente no será posible sin un intento por
compensar a la gente por la carga implícita de ocupar posiciones inferiores en la distribución general
del ingreso.

EQUIDAD Y EFICIENCIA
Ya se vio que los esfuerzos por reducir la desigualdad se pueden justificar con base en argumentos
morales y prácticos. Al parecer, se considera atractiva cierta combinación de argumentos, ya que
ninguna economía moderna deja la distribución del ingreso totalmente en manos del mercado. Este
compromiso subyacente con las normas de igualdad es fuerte y desempeña un papel primordial en
casi todos los debates sobre la política pública.
La ventaja natural del economista radica en responder las preguntas relacionadas con la eficien-
cia. Por esta razón, muchos economistas se niegan siquiera a hablar de los asuntos relacionados con

17_CHAPTER 17.indd 584 3/6/09 8:14:23 PM


distribuCión del ingreso 585

la igualdad. Sin embargo, prácticamente todos los cambios en la política afectan no sólo a la eficien-
cia, sino también a la distribución del ingreso. Y se sabe que la mayor parte de las sociedades parecen
preparadas para rechazar las distribuciones eficientes si no toman en cuenta la justicia. El resultado es
que, a menos que los economistas estén preparados para trabajar dentro de las limitaciones sociales
sobre la desigualdad, habrá muy poca o ninguna audiencia para sus recomendaciones políticas.
Por ejemplo, durante una interrupción de la oferta de algún producto importante, los econo-
mistas casi siempre responden con rapidez recomendando que se permita un aumento en el precio.
Después de todo se sabe que esta política da lugar a una distribución eficiente del producto que es
escaso. Sin embargo, la queja social es que un aumento excesivo de los precios impone una carga
inaceptable a los pobres. De modo que, al enfrentar una escasez, el gobierno rechaza a menudo el
camino del mercado libre en favor del racionamiento, las filas y otros métodos de distribución más
complicados.
La ironía desafortunada de esta respuesta es que las soluciones deficientes hacen que el pastel de
la economía sea más pequeño para todos, ricos y pobres por igual. Si se adoptan soluciones eficien-
tes, seguramente todos podrán recibir una rebanada más grande. Pero el simple hecho de adoptar
una política eficiente no garantiza que todos estén mejor. Por lo regular también es necesario utilizar
ciertos beneficios para compensar a quienes se vean afectados por el cambio en la política. De modo
que cuando los economistas recomiendan una política, deberán estar preparados para explicar de qué
manera sus consecuencias de distribución se pueden alterar para satisfacer las limitaciones sociales.
Un ejemplo es el episodio con el que inicia este capítulo. Recuerde que el asunto es si permitir
que las compañías telefónicas locales de Nueva York cobren las llamadas hechas al servicio de infor-
mación. La propuesta de la Comisión de Servicios Públicos de cobrar dichas llamadas fue recibida
con quejas de que la política impondría tarifas inaceptables para los pobres. Los funcionarios de la
comisión la salvaron con una enmienda que requería que cada suscriptor al servicio telefónico obtu-
viera un crédito de 30 centavos en su cuenta de teléfono como un reflejo de los costos ahorrados por
tener menos llamadas al servicio de información.
A continuación se examina cómo funciona esta propuesta enmendada. En la figura 17.6 el eje
horizontal mide las llamadas por mes al servicio de información y el eje vertical mide los gastos en
todos los demás bienes. La línea horizontal con la etiqueta B2 representa la limitación de presupuesto
para un cliente con un ingreso mensual de Y0 en caso de que no se cobren las llamadas al servicio

FIGURA 17.6
Cobro por servicio de
información
Todos los otros bienes Cuando las llamadas al
servicio de información
son gratis, el consumidor
Y0 + 30 centavos hace C3 cada mes. el hecho
I3 de cobrar 10 centavos por
I2 cada una reduce el volumen
de estas llamadas en gran
Y0 B2
medida, lo suficiente para
I1 financiar un crédito de 30
centavos al mes para cada
cliente. el nuevo sistema
B3 es más eficiente que el
antiguo y da más poder de
compra al usuario típico.
B1
Llamadas al servicio
C1 C2 C3 10Y 0 de información por mes

17_CHAPTER 17.indd 585 3/6/09 8:14:24 PM


586 Capítulo 17 gobierno

de información. B1 representa el presupuesto de la misma persona cuando hay un cargo de 10 cen-


tavos por esas llamadas. Y B3 es la restricción de presupuesto cuando hay un crédito mensual de 30
centavos además del cargo de 10 centavos por llamada. I1, I2 e I3 son curvas de indiferencia. Tienen la
forma convencional, sólo que más allá de cierto número de llamadas al mes se dirigen hacia arriba,
reflejando el hecho de que la mayoría de las personas no desean hacer un número ilimitado de lla-
madas al servicio de información aun cuando sean gratis. En el caso del consumidor que se muestra,
el hecho de no cobrar estas llamadas da como resultado C3 llamadas/mes. Una tarifa de 10 centavos
da como resultado C1 llamadas/mes, un número mucho más bajo. El cobro de 10 centavos con un
crédito de 30 centavos/mes resulta en C2 llamadas/mes. Con base en la suposición plausible de que
30 centavos/mes es una cantidad de ingreso trivial incluso para una persona pobre, C2 y C1 serían casi
iguales. El crédito de 30 centavos es financiado por el ahorro en costos que resulta cuando el número
de llamadas baja de C3 a C2. Esta reducción en las llamadas permite que la compañía telefónica mane-
je sus operaciones con menos conmutadores y operadoras; recursos que se liberan para invertirlos
en algo más útil.
Un científico social de otro planeta encontraría difícil de creer que la carga trivial de pagar las
llamadas al servicio de información haya disuadido a una comisión reguladora de aprobar el cobro.
Sin embargo, de esa magnitud es la fuerza de las preocupaciones por la distribución en el debate de la
política pública. Al compartir los ahorros en costos con los contribuyentes en forma de un crédito de
30 centavos al mes, se salvó una política que estaba destinada al fracaso.
En muchos otros mercados, como los de la gasolina y el gas natural, las consecuencias de dis-
tribución del cobro de precios con base en el costo son mucho más evidentes que en el caso de las
llamadas al servicio de información. Desde luego, en estas situaciones el asunto de la distribución es
el más importante. Las ganancias en eficiencia al cobrar precios que reflejan los costos son mucho
mayores en estos casos, y con suficiente atención a la distribución de estas ganancias, debe ser posible
llegar a un acuerdo político sobre cómo lograrlas.

MÉTODOS DE REDISTRIBUCIÓN
Los métodos por los cuales la sociedad redistribuye el ingreso están sujetos al mismo tipo de análisis
que los economistas manejan en otros programas e instituciones. Nuestra principal preocupación
es que los programas de redistribución mal diseñados pueden afectar con facilidad las ganancias en
eficiencia para las que fueron creados.

Programas actuales de asistencia social en Estados Unidos


Abba Lerner, uno de mis ex profesores en el último año de universidad, señaló en una ocasión que el
principal problema que enfrentan los pobres es que tienen un ingreso muy bajo. La solución, desde
su punto de vista, era muy sencilla. Debería dárseles más dinero. Sin embargo, los programas de asis-
tencia social tradicionales son mucho más complicados. Tenemos bonos para comida, bonos para
renta, bonos para electricidad, subsidios para guarderías, ayuda para las familias con hijos dependien-
tes y otros numerosos programas, cada uno con una burocracia administrativa propia. El resultado
final es que se necesitan alrededor de 7 dólares de impuestos para obtener un dólar adicional de
ingreso que llegue a manos de un pobre.
Estos costos, por muy altos que sean, no constituyen el problema más importante desde el pun-
to de vista de la eficiencia. Todavía más preocupante es el efecto que los programas actuales tienen
sobre los incentivos laborales. Para ilustrar la dificultad, primero es necesario describir algunos de
los detalles administrativos de los programas. Cada uno tiene un nivel de prestación mínima al cual
tienen derecho todas las personas que no alcanzan cierto nivel de ingresos. Una vez que un beneficia-
rio empieza a ganar un salario más alto sus beneficios se reducen una fracción determinada por cada
dólar adicional que recibe. Esta fracción se llama tasa de reducción marginal de la prestación. La figura
17.7 muestra la variación de la prestación con el ingreso para un programa con un nivel de beneficio
completo de 1 000 dls./año, un nivel de ingreso máximo de 4 000 dls./año y una tasa de reducción
marginal de la prestación de 50%.
El verdadero problema se presenta cuando una persona participa en varios programas de asis-
tencia social a la vez, como es común en el sistema actual estadounidense. Considere, por ejemplo,

17_CHAPTER 17.indd 586 3/6/09 8:14:25 PM


distribuCión del ingreso 587

FIGURA 17.7
Beneficios o prestación
contra ingreso para un
programa característico
Prestaciones (dls./año) de asistencia social
las personas que ganan
menos de 4 000 dls./
año reciben el nivel de
1 000 beneficio completo de
1 000 dls./año. por cada
dólar ganado por encima
de los 4 000, los beneficios
bajan 50 centavos. una
vez que el ingreso de
Ingreso (dls./año) una persona llega a los
4 000 6 000 6 000 dls/año, todos los
beneficios cesan.

una persona suscrita en cuatro programas como el que muestra la figura 17.7. Una vez que su in-
greso llega a 4 000 dls./año, pierde 2 dólares en beneficios (50 centavos de cada uno de los cuatro
programas) por cada dólar adicional que gane. Sobra decir que estos términos conducen a un gasto
de esfuerzo. Los efectos adversos sobre las decisiones de oferta laboral constituyen uno de los costos
más graves de este sistema de asistencia social actual.

Ingreso sobre la renta negativo


Milton Friedman calculó que por el costo de los programas actuales cada hombre, mujer y niño clasi-
ficado como pobre en Estados Unidos podría recibir un pago de más de 8 000 dls./año. Este cálculo,
aunado a su preocupación por los efectos adversos sobre los incentivos laborales, llevó a Friedman a
proponer una reforma radical en la que todo el conjunto de programas actuales sería reemplazado
por un solo programa al que llamó impuesto negativo sobre la renta (INR).
La versión del INR de Friedman empieza por dar a cada hombre, mujer y niño (pobre o rico) un
crédito para el impuesto sobre la renta suficiente para sostener un mínimo nivel de vida adecuado.
Una persona que no tenga ingreso recibiría este crédito en efectivo. La gente que recibe un ingreso
pagaría un impuesto sobre éste con una tasa de menos de 100%. El crédito inicial y la tasa fiscal
se combinan para determinar un nivel de ingresos de equilibrio en el que la responsabilidad fiscal
de cada persona compense con precisión su crédito fiscal inicial. La gente que devengue un sueldo
inferior a ese nivel recibirá un pago de beneficio neto por parte del gobierno, aunque esa ganancia
adicional constituiría un pago fiscal neto.
La figura 17.8 muestra cómo funcionaría el programa con un crédito fiscal de 4 000 dls./año y
una tasa fiscal de 50%. El nivel de ingreso de equilibrio para los valores de este programa es de 8 000
dls./año. Una persona que gana 4 000 dls./año recibirá un pago de beneficio neto de 2 000 dls./año,
mientras que otra que gana 12 000 dls./año hará un pago fiscal de 2 000 dls./año.

EJERCICIO 17.4
Calcule el nivel de ingreso de equilibrio para un programa cuyo crédito fiscal es de 6 000 dóla-
res y su tasa fiscal es de 40%. ¿Cuál es el beneficio neto recibido por una persona que gana 4 000
dls./año de sueldo en un empleo?

El INR se manejaría en forma muy similar al impuesto sobre la renta actual. Una ventaja impor-
tante del INR es, por lo tanto, su promesa de eliminar las costosas burocracias superpuestas de los
programas actuales. Pero el principal atractivo del INR para los economistas es que tiene un efecto

17_CHAPTER 17.indd 587 3/6/09 8:14:25 PM


588 Capítulo 17 gobierno

FIGURA 17.8
Programa de ingreso Ingreso después
negativo sobre la renta de descontar impuestos
este inr hace que cada (dls/año)
persona empiece con un
crédito fiscal de 4 000
dls./año. la gente que no Ingreso después
gana un ingreso recibe 10 000 de descontar impuestos
esa cantidad en efectivo.
luego, todos los ingresos
8 000
recibidos son gravados a
una tasa de 50%, lo que da
como resultado un nivel 6 000
de ingreso uniforme de
8 000 dls./año. la gente
que gana menos que esa 4 000
cantidad recibe un pago de
beneficio neto por parte
del gobierno; la gente que Ingreso antes
gana más hace un pago 45° de descontar
fiscal neto. impuestos (dls/año)
0 4 000 8 000 12 000

mucho menos negativo sobre los incentivos laborales que los programas actuales. Como la tasa fiscal
marginal que enfrentan los pobres nunca excedería de 100% con el INR, la gente tendría la seguridad
de tener un mayor ingreso después de impuestos trabajando más horas.
Aunque el problema de los incentivos es menos grave con el INR que con los programas de
asistencia social actuales, sigue siendo una dificultad muy seria. Si el INR fuera el único medio de
proteger a la gente contra la pobreza, el pago para la gente que no tiene un ingreso debe ser por lo
menos igual al salario mínimo. Si el pago es suficiente para vivir, inevitablemente hará que muchas
personas dejen de trabajar. La importancia de este problema fue confirmada por experimentos fe-
derales con el INR durante la década de 1970. Aunque los retiros de la fuerza laboral observados en
estos experimentos fueron menores a los que proyectaban las críticas más agudas en contra del INR,
fueron sustanciales.
Pero aun cuando el INR haga que un grupo de personas viva a expensas de los contribuyentes
fiscales, los críticos encontrarían a estas personas y habría una gran audiencia para los informes sobre
su vida en los noticiarios nocturnos. Tanto liberales como conservadores por igual se sentirían mo-
lestos al ver a los beneficiarios del INR tocando la guitarra y jugando volibol los lunes por la mañana.
Frente a estas imágenes, un INR con una donación suficientemente alta para apoyar a las personas
que pueden ser productivas y que prefieren no trabajar sería políticamente insostenible.

Empleo público para los pobres


Al igual que el INR, las propuestas de empleos públicos para los que están en desventaja ( JOBS) re-
cibieron mucha atención durante los primeros días de la guerra contra la pobreza. Estas propuestas
tenían el atractivo obvio de no tomar en cuenta a las personas que podían mantenerse solas. En el
lenguaje de los defensores del programa, el gobierno serviría como “el patrón de último recurso”,
el garantizador de “un empleo decente con un salario decente” para todos aquellos que no pudieran
encontrarlo en el sector privado.
Como el único mecanismo para sacar al pobre de su situación, la idea de los empleos públicos
fue víctima de muchas críticas. Quizá la más importante fue que el empleo público garantizado po-
dría hacer que la gente desertara del sector privado en forma masiva. Esta afirmación se basaba en
la evidencia de que los trabajadores no calificados consideraban los empleos públicos mucho más
atractivos que los privados con sueldos similares. Miles de solicitantes hacen filas cuando se abren
nuevos empleos en el gobierno; al mismo tiempo, los anuncios que solicitan ayudantes de cocina en
el sector privado no reciben ninguna respuesta. Con la potencial migración laboral a gran escala en

17_CHAPTER 17.indd 588 3/6/09 8:14:26 PM


distribuCión del ingreso 589

mente, los políticos llegaron a la conclusión de que los recursos no permitirían una oferta abierta de
empleos en el gobierno con salarios comparables a los del sector privado.
Una segunda crítica fue que los puestos públicos para las personas en desventaja inevitablemen-
te serían tareas tan inútiles como la solicitud de Keynes de que la gente excave hoyos y los vuelva a
llenar. Esta crítica encontró un eco importante en Estados Unidos, donde la predisposición a menudo
ha sido ver todos los empleos en el gobierno como trabajos inútiles. Esta actitud por sí misma habría
terminado con las propuestas de JOBS, aun cuando pudieran haber sido económicamente factibles.

Combinación de INR y JOBS


Sin embargo, con unos cuantos cambios sencillos, los programas INR y JOBS se pueden combinar
de modo que eliminen muchas de las dificultades que se encuentran al considerar cada uno por
separado como un arma única contra la pobreza.
Por ejemplo, con JOBS el gobierno podría solicitar licitaciones por parte de contratistas privados
para contratar a trabajadores no calificados con sueldos inferiores al mínimo para que realicen gran
variedad de tareas específicas. (Se dirá más sobre estas tareas en un momento.) Con un sueldo mu-
cho más bajo que en los empleos privados no habría ninguna razón para temer un éxodo masivo del
sector privado.
Tener empleos públicos para los pobres administrados por contratistas privados elegidos me-
diante licitaciones competitivas ayudará en gran medida a eliminar la administración deficiente que
se presenta con frecuencia en las operaciones gubernamentales. Como se indica en el capítulo 12,
algunas ciudades han descubierto que sus costos bajan a más de la mitad, sin reducciones en la cali-
dad, cuando prestan servicios como protección contra incendios y recolección de basura por medio
de contratistas privados. Si la administración de JOBS estuviera sujeta a las presiones de reducción de
precios de los mercados privados, habría muchas razones para esperar un desempeño eficiente.
Existe el acuerdo general de que, con la administración competente, la gente que no tiene mu-
cha experiencia ni capacitación puede llevar a cabo muchas tareas útiles. ¿A qué ciudad no le agra-
daría tener un mantenimiento adicional en sus parques? Con la supervisión apropiada las personas
poco hábiles pueden realizar este tipo de tareas. Y pueden transportar a los ancianos y discapacitados
en camionetas especialmente equipadas, rellenar baches en las calles de las ciudades, cambiar los
focos fundidos del alumbrado público, trasplantar semilleros en proyectos de control de la erosión,
borrar el graffiti de los lugares públicos, pintar edificios del gobierno, y reciclar periódico, aluminio
y envases de vidrio.
Éstas y muchas otras tareas socialmente útiles deben realizarse, y podrían hacerlas personas
que no tienen las habilidades necesarias para encontrar un empleo en el sector privado. Incluso los
david Young-Wolft/photoedit, inc.

incluso las personas con pocas habilidades pueden prestar servicios útiles.

17_CHAPTER 17.indd 589 3/6/09 8:14:27 PM


590 Capítulo 17 gobierno

padres solteros con niños pequeños podrían participar ayudando en las guarderías a las que asisten
sus hijos.
Un cambio sencillo en el diseño eliminaría también las principales dificultades relacionadas con
el INR tradicional. Este cambio consistiría en limitar el pago máximo del INR a una cifra que, al igual
que el salario para los JOBS, estuviera muy por debajo del equivalente a las ganancias anuales de un
trabajo de tiempo completo con el salario mínimo. Con el pago debajo de este nivel no será posible
que la gente abandone el mercado laboral para vivir a expensas de los contribuyentes. Sin embargo,
al combinarlo con las ganancias del programa JOBS o, mejor aun, con las ganancias de un empleo
privado, el INR sacaría a una persona de la pobreza (vea la figura 17.9). Ningún programa por sí
mismo puede lograr esta meta sin crear efectos secundarios inaceptables. Pero los dos programas
juntos sí pueden.

FIGURA 17.9
Fuentes de ingreso en
el programa INR-JOBS
un inr con una donación
en efectivo muy baja para INR +
vivir no puede motivar a trabajo
la gente a que abandone privado
la fuerza laboral. tampoco INR +
un empleo patrocinado trabajo
por el gobierno con público
un salario mínimo
atraería lo suficiente a Límite de la pobreza
los trabajadores con un Trabajo
empleo productivo para público
abandonar su trabajo en
el sector privado. pero
el ingreso combinado de
ambos programas sería INR
suficiente para sacar a
la gente de la pobreza; y
debido a la paga tan baja
en los empleos públicos,
los participantes tendrían
fuertes incentivos para
seguir buscando un trabajo
en el sector privado.

Beneficios indirectos del programa combinado


La combinación JOBS-INR no sería barata. Pero tampoco lo es el sistema actual. Además de sus
costos directos y de los incentivos laborales adversos, los programas actuales imponen innumerables
costos indirectos. Muchos de éstos adoptan la forma de regulaciones diseñadas para ayudar a los
pobres. Como los programas actuales no pueden transferir un ingreso adicional directamente a los
pobres sin socavar más los incentivos laborales, los políticos enfrentan la presión constante de interfe-
rir con los mercados privados para proteger a los pobres de los incrementos de precios. Por ejemplo,
como se estudió en el capítulo 2, los burócratas diseñaron un sistema bizantino de regulaciones con
el fin de evitar incrementos en los precios de la gasolina durante la interrupción del suministro de
petróleo en 1979. Era común ver filas en las gasolineras que ocupaban varias calles, y muchos con-
ductores fueron asesinados o heridos en disputas por el lugar en esas filas.
Con una combinación del INR y el JOBS en lugar de los programas actuales de asistencia social,
los políticos podrían haber transferido el ingreso adicional directamente a los pobres en el inicio
de la crisis petrolera en 1979. Y esto a su vez habría permitido distribuir el petróleo de manera más
sensible y eficiente mediante el mecanismo de precios.
Muchas ciudades han adoptado controles sobre el ingreso similares en nombre de su preocu-
pación por los pobres. Sin embargo, como cualquier estudiante de microeconomía puede explicar,

17_CHAPTER 17.indd 590 3/6/09 8:14:28 PM


resuMen 591

estos controles cuestan mucho dinero por cada dólar que dan a los pobres. Las viviendas de bajos
ingresos se deterioran, los abogados prosperan gracias a conversiones de condominios que de otra
manera no tienen sentido, las parejas que ya no tienen a sus hijos en casa siguen viviendo en departa-
mentos de ocho habitaciones y situaciones similares. Con una combinación de los programas JOBS
e INR, los ingresos de la gente podrían aumentar directamente, evitando así la necesidad de medidas
inútiles como el control de las rentas.

Una objeción a JOBS


Muchas personas que se consideran amigas de los pobres pensarán que no es justo pedir a la gente
que preste algún servicio a cambio de la asistencia pública. Algunos se opondrán a esta política de
servidumbre forzada y afirmarán que priva a la gente pobre de su dignidad.
Estas objeciones se derivan de la negación a reconocer la verdadera gama de opciones políticas. En
un mundo ideal, liberales y conservadores por igual acordarían proporcionar una asistencia generosa
incondicional a quienes no se puedan valer por sí mismos y ninguna ayuda para los demás. Sin embar-
go, en el mundo en el que vivimos no existe un esquema confiable para diferenciar a los miembros de
ambos grupos. Al carecer de un esquema así, difícilmente se está en posición de ofrecer beneficios a un
grupo al tiempo que se le niegan al otro. Y nadie pretende que sea posible ofrecer asistencia incondicio-
nal generosa a todos. Las alternativas prácticas, a la vista de todos, son 1) ofrecer asistencia incondicional
muy limitada a todos (prácticamente, el sistema actual), o bien 2) ofrecer una asistencia más generosa
condicionada a la realización de tareas útiles.
Como estudiante universitario, no es probable que usted sea muy pobre. Pero por un momento
trate de imaginar que no sólo es pobre, sino que además tiene mala suerte. Existen empleos privados
que pagan un salario decente, pero tiene tan mala suerte que nunca ha encontrado uno. Los políticos
en Washington tratan de decidir qué hacer con usted. Tanto usted como ellos saben que las opciones
están limitadas a las dos alternativas mencionadas. ¿Cuál preferiría usted?
Suponga que prefiere la segunda, realizar una tarea útil a cambio de un salario para vivir. Y ra-
zona que, de esta manera, podrá vivir en un departamento en el que sus hijos no coman hojuelas de
pintura con plomo que se desprendan de las paredes y los marcos de las puertas. También sabe que
trabajar en un puesto del gobierno le ayudará a adquirir habilidades que le permitirán obtener un
empleo con un salario más alto que realmente le agrade. Por último, a pesar de que ha escuchado lo
contrario, no siente que realizar una tarea socialmente útil lo rebaje en ninguna forma. ¿Qué sentiría
entonces que sus “amigos” de un nivel más alto lo desanimaran en cuanto a la alternativa elegida
diciéndole que “le quitará su dignidad”?
Liberales y conservadores tienen el interés compartido, tanto moral como práctico, de redistri-
buir el ingreso de manera que no socaven la eficiencia. Los programas de redistribución actuales son
en su mayor parte costosos e ineficientes. El análisis microeconómico tiene mucho qué enseñarnos
en lo referente a la reforma de estos programas así como sobre muchos otros aspectos políticos
importantes que se han estudiado a lo largo de este libro.

RESUMEN
• Los bienes públicos son como otros bienes en el sentido de que más alas de pollo es necesario producir más muslos de pollo. Así
su valor puede medirse por lo que la gente está dispuesta a pagar como la cantidad de un bien público debe ser igual para todos los
por tener más. Pero mientras la curva de demanda agregada de consumidores, también la cantidad de alas de pollo consumidas
un bien privado se forma sumando horizontalmente las curvas debe ser igual a la cantidad de muslos consumidos. Y así como el
de demanda individuales, la curva agregada de la disposición a precio que una persona está dispuesta a pagar por una cantidad
pagar por un bien público es la suma vertical de las curvas indi- dada de un bien público difiere del de otra persona, en general el
viduales correspondientes. Esta diferencia obedece a que la can- precio de los muslos de pollo es diferente del precio de las alas.
tidad de un bien público debe ser la misma para todos los consu- • Igual que con los bienes privados, la curva de oferta de un bien
midores. Por el contrario, en el caso privado, el precio es igual público es el costo marginal de producirlo. La cantidad óptima
para los compradores, que luego escogen cantidades diversas. de un bien público es el nivel para el cual la curva agregada de la
• Hay una clara analogía entre la demanda de bienes públicos y la disposición a pagar interseca la curva de oferta. Para pagar por
de bienes privados producidos de forma conjunta. Para producir las cantidades óptimas de bienes públicos, en general es necesario

17_CHAPTER 17.indd 591 3/6/09 8:14:28 PM


592 Capítulo 17 gobierno

que los pagos de impuestos de los particulares varíen directamen- otros. Si se fuerza a votantes heterogéneos a coexistir en una
te con los importes que están dispuestos a pagar por los bienes jurisdicción, el resultado frecuente es una dolorosa solución
públicos. En la medida en que las personas con mayores ingresos de compromiso que no satisface a nadie. Pero la necesidad de
demandan más cantidades de bienes públicos, el resultado es que estas soluciones de compromiso se reduce enormemente si los
tanto ricos como pobres estén a favor de un sistema impositivo votantes pueden agruparse en comunidades con gustos más o
que asigne una parte mayor de la carga fiscal a los ricos. menos homogéneos.
• El simple hecho de que un bien tenga las características de un • Un problema que aqueja a todos los mecanismos de toma de
bien público no significa que deba proveerlo forzosamente el decisiones públicas es que partidos con intereses particulares
gobierno. Hay muchos esquemas ingeniosos, que van de anun- tienen motivos para influir en los resultados que los favorezcan.
cios gratuitos a contratos colectivos muy estructurados, me- El problema recibe el nombre de “búsqueda de las rentas”, y se
diante los cuales se abastecen los bienes públicos prácticamente ha convertido en una amenaza grave para nuestro sistema de
sin intervención del Estado. asistencia social.
• Cada vez que hay indivisibilidades o economías de escala im- • El principal mecanismo para distribuir el ingreso en las econo-
portantes en la producción de bienes de consumo privados se mías de mercado es el mercado de factores. Las personas venden
encuentran problemas parecidos a los que surgen en relación con su trabajo a cambio de un pago igual al valor de su producto
los bienes públicos. En estas situaciones, como se vio, es común marginal. Además, invierten sus ahorros a tasas de interés que
que se formen clubes en los que los miembros comparten los también están vinculadas a la productividad marginal del capi-
costos de los bienes de consumo importantes. La compensación tal. Este método de distribución del ingreso tiene varias pro-
que tiene que establecer un miembro potencial de tales clubes es piedades deseables en el ámbito de la eficiencia; en particular,
entre el ahorro de costos y la menor intimidad en el uso del bien. recompensa los esfuerzos y la disposición a correr riesgos. Pero
• A veces el voto de la mayoría genera clasificaciones intransitivas los críticos, especialmente John Rawls, objetan que las personas
entre proyectos. Cuando ocurre así, el poder de elegir el orden nunca escogerían de buena gana vivir amparadas en un proceso
en que se evalúan pares de alternativas se considera equivalente que arroja resultados tan desiguales como los que se vieron en
al poder de determinar el resultado final. Hay una clase especial mercados de factores no atenuados.
de temas en los que el voto de la mayoría no es vulnerable a la • Además del argumento moral que esgrime Rawls, hay por lo
manipulación de los programas. Con respecto a los problemas menos dos razones prácticas para redistribuir el ingreso. En
únicos en los que cada votante clasifica las alternativas confor- primer lugar, los ricos estarían a favor de pagar más que una
me a qué tan alejadas están de su elección ideal, al resultado parte equitativa de la carga fiscal total porque, en caso contra-
final será el que prefiera el votante medio, independientemente rio, terminarían con una provisión ineficientemente pequeña de
del orden en que se reciban los votos. Este resultado se conoce bienes públicos. En segundo, la redistribución es necesaria para
como teorema del votante medio. mantener un sentido voluntario de cohesión social, algo que
• El análisis costo-beneficio es una alternativa simple pero muy interesa tanto a los ricos como a los pobres.
poderosa al voto de la mayoría. Aplicado de manera correcta • El esquema actual de los programas de asistencia social es caro,
a un número suficientemente grande de pequeñas decisiones, no sólo por la duplicación burocrática de funciones, sino tam-
casi siempre satisface el criterio de Pareto. bién por sus efectos indirectos en los incentivos laborales y en
• Incluso con un mecanismo perfecto para revelar los sentimien- las políticas públicas con respecto a los mercados privados. Una
tos de la gente sobre lo deseable de diferentes bienes públicos, combinación de un pequeño impuesto sobre la renta negativo,
de todos modos hay que tomar decisiones difíciles sobre qué completada con trabajos públicos con pago inferior al mínimo
bienes producir. El problema es que los bienes públicos que más transferiría ingresos a los pobres sin muchos de los efectos se-
prefieren unos grupos se enfrentan a la fuerte oposición de cundarios indeseables de los programas actuales.

pREgUNtaS dE REpaSo
1. ¿Por qué las curvas individuales de disposición a pagar se la compensación entre flexibilidad y costo que confrontan los
suman vertical y no horizontalmente para obtener la curva usuarios de estos bienes.
conjunta de disposición a pagar por un bien público? 5. ¿De qué manera la votación de la mayoría lleva a clasificacio-
2. ¿En qué sentido los bienes privados producidos en forma con- nes sociales intransitivas?
junta son análogos a los bienes públicos? 6. Describa dos formas de ineficiencia relacionadas con la bús-
3. ¿Por qué es probable que los ciudadanos ricos se opongan a queda de ingresos.
que ricos y pobres paguen los mismos impuestos? 7. ¿Por qué un impuesto negativo sobre la renta negativo por
4. ¿En qué se parece un bien privado producido en condiciones sí mismo no puede solucionar el problema de la redistribu-
de economía a escala creciente a un bien público? Describa ción?

17_CHAPTER 17.indd 592 3/6/09 8:14:29 PM


probleMas 593

pRoblEMaS
1. Un gobierno trata de decidir la cantidad de un bien público que debe ofrecer. Las curvas de dispo-
sición a pagar para cada uno de sus dos ciudadanos se muestran en el diagrama. La curva de costo
marginal para el bien público se define mediante CM = Q/2, donde Q es la cantidad del bien. Hay
también un costo fijo de 10 relacionado con la producción del bien.

2
Disposición a pagar de B
1
Disposición a pagar de A

0 1 2 3 4 5 6 7 8 9 10 11 12 13 14 15 16

a) ¿Cuál es la cantidad óptima del bien público?


b) Si ambos ciudadanos deben pagar el mismo impuesto por tener el bien, ¿recibirá el gobierno un
voto mayoritario?
2. Si se supone que el bien público descrito en el problema 1 se ofrece en un nivel óptimo, ¿cuánto debe
cobrar el Estado a cada ciudadano cada vez que utiliza el bien público?
3. Diez consumidores idénticos tienen curvas de disposición a pagar individuales de P = 5 – — 1
20 Q por
un bien público; por ejemplo, parques locales (donde P se mide en cientos de dólares y Q se mide en
acres). Elabore y grafique la curva agregada de disposición a pagar. Para 50 acres de parques ¿cuál es
el máximo que cada individuo estaría dispuesto a pagar?
4. Considere la situación descrita en el problema 3, pero que ahora el costo marginal de proporcionar
parques medido en cientos de dólares es CM = –12 Q. Determine el tamaño óptimo de los parques
locales.
5. Las alas y los muslos de pollo son bienes privados de producción conjunta. La introducción de las
alas con salsa Búfalo (la sensación de la comida rápida) ha dado lugar a un importante aumento en la
demanda de alas de pollo. Demuestre cómo esta situación afecta el precio de equilibrio y la cantidad
de muslos.
6. La madera y las lijas son productos conjuntos cuyas funciones de demanda son DL y DS, como muestra
el diagrama. El eje de la cantidad mide el número de árboles. Los puntos en los programas de la de-
manda indican las demandas de madera y lijas que equivalen a una cantidad de árboles determinada.
P

Oferta de árboles

DL
DS
Q
0

17_CHAPTER 17.indd 593 3/6/09 8:14:31 PM


594 Capítulo 17 gobierno

a) Proporcione una interpretación económica del hecho de que el programa de demanda de lijas se
extiende por debajo del eje horizontal.
b) Si el programa de la oferta de árboles se da en el diagrama, muestre los precios y las cantidades de
equilibrio de lijas y de madera en una gráfica.
7. A menudo, las cadenas de televisión que tienen el apoyo de los televidentes dan a sus contribuyentes
“regalos” por hacer contribuciones en distintos niveles. (“Dos bonitos tarros para café de News Hours
por un donativo de 120 dólares.”) Con base en lo que sabe acerca de la psicología de la asignación
de valor a las decisiones (revise el capítulo 8), explique por qué esta práctica ayuda a las estaciones a
ganar más dinero.
8. Es común que los colegios y universidades pongan a sus edificios y facultades de administración y
medicina los nombres de sus donantes más importantes (por ejemplo, la Facultad de Administración
Max Fisher del Estado de Ohio y el Schottenstein Center/Value City Arena). Mientras tanto, pocos
donantes tienen el cuidado de destinar sus donaciones al mantenimiento rutinario de los edificios
de las universidades. ¿De qué manera los beneficios sociales de las obras de beneficencia podrían
explicar estas observaciones?
9. Cierto o falso: como los intereses presentes en las elecciones presidenciales nacionales son mucho
más importantes que los de la elección del alcalde de una pequeña ciudad, la teoría de las elecciones
racionales indica que en la primera habrá una proporción de votantes mucho mayor que en la segun-
da. Explique su respuesta.
10. Cierto o falso: el hecho de que la rotación de votantes sea mucho mayor en las elecciones presidencia-
les que en las locales apoya la propuesta de que los votantes son racionales. Explique su respuesta.
11. Una fraternidad que consiste en 20 estudiantes de primer año, 20 de segundo y 20 de último año está
a punto de elegir a su nuevo presidente. Arnold, Bo y Chuck son los tres candidatos. Los miembros
de cada clase tienen los esquemas de calificación que se dan en la tabla siguiente:

Clase Mejor Segundo mejor Último


seniors arnold bo Chuck
Juniors bo Chuck arnold
sophs Chuck arnold bo

La tradición en la fraternidad consiste en enfrentar dos candidatos y luego al ganador de ese concur-
so en contra del tercero. Si usted fuera estudiante de primer año, ¿a quiénes enfrentaría en la ronda
preliminar? ¿Y si fuera estudiante de último año?
12. Smith es una persona muy dura de corazón que está a favor de dar al pobre sólo lo suficiente para
ayudarle a no sentir hambre. Si se supone que todos los demás miembros de la comunidad de Smith
sienten lo mismo, ¿por qué éste se opondría a una propuesta para permitir que cada comunidad
establezca su propio nivel de asistencia social?
13. A. Smith, quien actualmente está desempleado, participa en cuatro programas de asistencia que
ofrecen beneficios diarios de 10 dólares a cada persona que no tiene un ingreso. Después, cada pro-
grama reduce sus beneficios 50 centavos por cada dólar de ingreso recibido. El gemelo idéntico de A.
Smith, B. Smith, se inscribe en un programa experimental de impuesto negativo sobre la renta que le
da 40 dls./día en beneficio y luego le cobra un impuesto de 50% por cada dólar de ingreso recibido.
Ahora suponga que a A. Smith le ofrecen un trabajo que paga 4 dls/h, el mismo salario que recibe su
hermano.
a) Elabore la restricción de presupuesto para cada uno de los gemelos.
b) ¿Cuántas horas tendrían que trabajar antes de que A. Smith termine con más dinero en efectivo
neto y más beneficios que su hermano?

17_CHAPTER 17.indd 594 3/6/09 8:14:31 PM


respuestas a los eJerCiCios del Capítulo 595

RESpUEStaS a loS EjERcicioS dEl capítUlo


17.1. La curva agregada de la disposición a pagar sería P = 120 – 2Q, que es la suma vertical de las curvas
de disposición a pagar individuales (véase la gráfica). Para Q = 30 minutos, cada individuo estaría
dispuesto a pagar hasta P = 12 – –15 Q = 12 – –15 (30) = 6 dólares, para un total de 60 dólares de 10
clientes.
17.2. Para encontrar la duración óptima del concierto, iguale la disposición a pagar agregada P = 120
– 2Q y el costo marginal CM = 2Q para encontrar Q = 30 minutos.
Disposición a pagar (dls.)
120

CM

60

12
0 Concierto (min)
30 60

17.3. Una vez más, Schumer es el votante medio. La alternativa más cercana a su porcentaje ideal es 20 y
ésta obtendrá una mayoría de votos sobre cualquier par de alternativas.
17.4. Suponga que Y* indica el nivel de ingreso de equilibrio. Para calcular Y*, se despeja de 6 000 +
(1 – 0.4)Y* = Y*, que produce Y* = 15 000 dls./año. Una persona que gana 4 000 dls./año pagaría
0.4(4 000 dls.) = 1600 dls./año en impuestos y por lo tanto recibiría un beneficio anual neto de
6 000 dls. – 1 600 dls. = 4 400 dólares.

17_CHAPTER 17.indd 595 3/6/09 8:14:32 PM


17_CHAPTER 17.indd 596 3/6/09 8:14:32 PM
ÍNDICE TEmáTICo 597

Í N D I C E T E m áT I C o

Los números de página seguidos de una n indican que el material se encuentra en las notas.

A Autopista de cuota de massachusetts, 375


Abuso de las drogas, 245 Autoridad portuaria de Nueva York, 376
Acuerdo, 581 Autoridades aeroportuarias, 376
Adams, Walter, 115n Auto-seguro, 190, 192
Adaptación al consumo, 243-245 Avance tecnológico
Adquisiciones de bienes de capital, 508 como determinante de la oferta, 39
Adversarios, información compartida con, 170-171 en la producción de alimentos, 268-269
Aeropuerto de Schiphol, 316 impacto de los monopolios sobre, 406-407
AFL-CIo, 315 Averch, Harvey, 400
Ahorro de combustible, 19 Axelrod, Robert, 420,421
Ahorros Azúcares, 214-215
dispositivos de compromiso para , 225
externalidades posicionales en, 554-555
perfiles de consumo y, 244-245 B
Airbus, 446 Babashoff, Shirley, 250
Ajustes de primas, 190, 191 Babcock, L., 467n
Akerlof, George, 175-176 Barnes y Noble, 425n
Albercas, 574-575 Base tarifaria, 401
Alchian, Armen, 336n Bazerman, max, 203
Alcohol, 154-155 Bebedores empedernidos, 154-155
Alimentos en las aerolíneas, 18 Becker, Gary, 228
Allais, m., 207 Belous, Richard, 447n
Allen, Woody, 263, 264 Beneficio económico
Alternativas irrelevantes, 252-254 beneficio contable versus, 334-335
Altruismo, 213-214 con equilibrio competitivo a corto plazo, 345-346
Amoco, 277-278 excedente del productor versus, 347
Análisis de la oferta y la demanda Beneficios, 8, 12-14; véase también Cálculos costo-beneficio
determinantes principales en, 37-40 Beneficios marginales, 12-15
forma algebraica, 41-42 Beneficios promedio, 12-14
impactos de los impuestos, 47-50 Bentham, Jeremy, 212
uso de gráficas, 26-29 Bertrand, Joseph, 429
Analogía de la producción conjunta, 569-570 Bien compuesto, 60, 61, 73
Anclaje y ajuste, 250-251 Bienes colectivos, 568
Apuestas, 179-187 Bienes Giffen, 469
Apuestas justas, 181-182, 189 Bienes inferiores
Arbitraje, 392-393 como bienes Giffen, 103-104
Arrendamientos petroleros, 203 curva de demanda compensada por el ingreso
Arrow, 436 para, 136, 137
Asador para barbacoa, 123-124 definición, 37, 100
Asadores de propano, 123-124 efectos ingreso y sustitución, 100-101, 102, 103
Asbestosis, 501 elasticidad ingreso de la demanda para, 122
Asesoría sobre inversiones, 505, 514-515 Bienes normales
Ashworth, Tony, 421 curva de la demanda compensada por el ingreso
Asignación de recursos, 272-275 para, 136,137
Asistencia sobre la guía telefónica, 567-568, 585-586 definición, 37, 100
Asistencia social efectos ingreso sobre, 101
efectos de niveles elevados, 581 Bienes posicionales, 178
eficiencia, 586-587 Bienes públicos
pagos en efectivo versus estampillas para analogía con producción conjunta, 569-570
alimentos, 73-75 cantidades óptimas, 570-572
Autocontrol peligroso, 255-256 locales, 581 597

18_NAME INDEX.indd 597 3/7/09 12:40:59 PM


598 ÍNDICE TEmáTICo

método de selección para, 576-582 Cantidad y precio de equilibrio


principios básicos, 568-569 ajuste a la, 30-31
suministro privado de, 402-403, 568, 572-576, 589 características del bienestar, 31-32
Bienes puramente públicos, 568-569, 574 impactos de los impuestos, 47-50
Blackie’s House of Beef, 535 predicción de cambios, 40-41
BmW, 176, 177, 448 visión general, 29-30
Böhm-Bawerk, Eugen von, 162 Cantidades absolutas, versus proporciones, 11-12
Boletín informativo, inversión, 505, 514-515 Cantidades, equilibrio, 29-30
Boletos de entrada para el Super Bowl, 242-243 Capital
Bolsa de valores de Nueva York, 26 costo del, 400-402
Bono escolar, 142-144 demanda real de capital, 506-507, 509
Bonos a corto plazo, 510 financiero versus real, 506
Bonos a largo plazo, 510 política fiscal y, 516-517
Bonos corporativos, 511 precios comparativos del, 314
Bonos gubernamentales, 511 renta económic;a y, 517-518
Bonos municipales, 516 tasas de interés versus tasas de renta, 507
Bonos perpetuos, 511 Capital circulante, 506n
Borders Books, 425n Capital financiero, 506, 508-509; véase también Capital
Borges, Jorge Luis, 212 Capital físico, 506
Bray, J. W., 115n Capital real
Breton, Albert, 400n capital financiero versus, 506
Brue, Stanley, 304 criterios de compra, 508
Brumberg, Richard, 160 demanda de, 506-507, 509
Burger King, 199, 375 impuesto sobre, 516-517
Burocracias, 400 Capitán Crunch, 440
Búsqueda de rentas, 581-582 Card, David, 477n
Búsqueda del riesgo, 183, 184 Carreras armamentistas, 553-554
Búsquedas de empleo, 199-203 Carter, Jimmy, 136, 140, 142
Búsquedas de trabajo, 199-203 Caza de ballenas, 549
Celofán, 372-373
Chamberlin, Edward, 435
C Chammah, A., 420n
Cabildeo para franquicias públicas, 582 Chevrolet, 447, 448
Cadillac, 447, 448 Civil Aeronautics Board (Consejo civil de aeronáutica),
Cálculos costo-beneficio 32-33
de bienes públicos, 578-581 Clapton, Eric, 55
elementos del, 4-5 Clasificación de las aseguradoras, 191
errores en la aplicación, 6-14 Clima, 39
uso del producto marginal en, 270 Clubes de aviación, 576
Calentamiento global, 549-550 Coase, Ronald H., 536-537, 539-540, 542
Calfa, marian, 542 Coca-Cola, 72, 375, 439-440
Calidad de los productos Cohesión social, distribución del ingreso y, 584
omitida en IPC, 152-153 Colusión, 414-416, 420
revelar, 171-172, 174 Comerciantes de camellos, 395
Cambio de marca, 417 Compañeros de bridge, 170
Cambios de precio Comparación general del bienestar, 147-153
efecto sobre la demanda individual, 96-97, 98 Compartimiento de riesgos, 188-189
efecto sobre restricciones presupuestales, 58-59 Compasión, 16, 17
efectos ingreso y sustitución, 100-105 Compensación de diferencias salariales, 487, 497-501
predicción, 40-41 Competencia monopolística
reacción del consumidor frente a, 106-108 factores espaciales en, 440-445
Cambios en el ingreso incentivos de propaganda con, 450-452
efecto sobre la demanda individual, 98-99, 100 modelo de Chamberlin, 435-440
efecto sobre las restricciones presupuestales, publicación de Hotelling, 449-450
59-60 variedad bajo, 446-449
Cambios en la demanda, 40 Competencia, teoría de la mano invisible, 15-16, 353-354
Camerer, C., 467n Complementos
Camioneros, 315, 365-366 como determinantes de la demanda, 38
Canastas de bienes; véase también Restricciones efectos ingreso y sustitución, 104-105
de presupuesto elasticidad precio cruzado de la demanda para, 125
Canastas de bienes más asequibles, 69-73 perfectos, 104-105, 277-278, 468
Cantidad demandada, 40-41; véase también Curva de efectos ingreso y sustitución, 104-105
demanda ingreso y ocio como, 468
Cantidad ofrecida, 40; véase también Curvas de oferta mapas de isocuantas para, 277-278
Cantidad óptima de puntos de venta, 442-445 Comportamiento agresivo, 215-218

18_NAME INDEX.indd 598 3/7/09 12:40:59 PM


ÍNDICE TEmáTICo 599

Computadoras, 184-185, 375, 376 Costos de búsqueda, 202-203


Computadoras Apple, 375, 451 Costos de oportunidad
Computadoras personales; véase Computadoras costos directos versus, 242-243
Computadoras usadas, 184-185 definición, 7
Comunicaciones; véase Información en la educación superior, 8-9
Conciencia, 232 en los cálculos de ganancias económicas, 334
Concurso, 552-554 ignorar, 7-9
Condición de cierre, 341-343, 386-387 Costos de transporte, 440-445, 447
Condiciones para maximización de la ganancia a Costos del propio bolsillo, 242-243
corto plazo, 339-343 Costos del trabajo calificado, 315-316
Conducta de votacion, 228-229 Costos del trabajo no calificado, 315-316
Conducta, predicción, 6 Costos externos y beneficios, 16-17
Confianza, 17 Costos fijos
Conjunto accesible, 58 de producción de la información, 376
Conjunto asequible, 58 definición, 298-299
Construcción de relaciones, 176-177, 244 gráfica de, 301, 304-306
Consumo promedio, 303, 306
adaptación al, 243-245 Costos fijos promedio, 303, 304-306; véase también
como flujo, 57 Costos fijos
conspicuo, 177-179 Costos implícitos, 7-9, 334
efecto básico de los precios sobre, 96-97, 98 Costos inevitables, 299
efecto básico del ingreso sobre, 98-99, 100 Costos marginales
externalidades negativas del, 559-560 costos promedio versus, 12-14
futuro, 155-162 de largo plazo versus los de corto plazo, 321-322, 330
Contratación (de personal), discriminación en la, 480-485 decisiones de asignación basadas en, 308-309
Contratación exclusiva, 402-403 definición, 303
Contratación privada para bienes públicos, 402-403, en relación con el producto marginal, 310-311
574, 589 graficar, 14-15, 306-308
Contrato colectivo de trabajo, 478-480 gráficas a largo plazo, 317-319
Contratos de condominio, 574 maximización de la ganancia a corto plazo y, 340-341
Control de las rentas, 34-36, 37, 590-591 Costos no recuperables
Control de las rentas de departamentos, 34-36 aseguramiento de la calidad y, 172
Control de precios error de ignorarlos, 9-11
para combustible, 34, 521 necesidad de ignorarlos, 238, 242
para productos agrícolas, 36, 41 Costos promedio
para rentas, 34-36, 37, 590-591 curva de la industria a largo plazo, 356-359
Control exclusivo de los insumos, 373 en relación con las estructuras industriales, 319-321,
Controles a los precios de la gasolina, 34, 590 374
Convexidad del ordenamiento de las preferencia, 64-65 gráficas a largo plazo, 318-319
Cook, Philip, J., 154-155, 488n largo plazo versus corto plazo, 321-322, 329-330
Cooperadores y traidores, 220-226 marginal versus, 12-14
Corto plazo, 266, 266-275 Costos totales promedio
Costo de la vigilancia, 223-225 definición, 303
Costo de oportunidad de la educación superior, 8-9, gráfica, 304, 305, 306-307
véase también Educación Costos totales
Costo del capital, 400-402 a largo plazo versus a corto plazo, 321-322, 327-330
Costo del factor marginal, 472-473 definición, 299
Costo promedio de un factor, 472, 473 gráficas a corto plazo, 301, 304-305, 306-307
Costo psíquico, 7 gráficas a largo plazo, 316-319
Costo total de los factores, 472 maximización de las ganancias a corto plazo y, 339-343
Costos promedio, 303
implícitos, 7-9 Costos variables
no recuperables, 9-11, 172, 238, 242 definición, 299
promedio versus marginales, 12-14 gráfica, 300-302, 304-305
reciprocidad con los beneficios, 8 promedio, 303, 306
Costos a corto plazo Costos variables promedio
a largo plazo versus, 321-322, 327-330 condiciones de cierre para, 341-342, 386-387
conceptos básicos, 298-299, 303 definición, 303
graficar, 300-302, 304-308 en relación con el producto promedio, 310-311
producto marginal y promedio versus, 310-311 gráfica, 304, 305, 306
Costos a largo plazo maximización de la ganancia a corto plazo y, 340-341
de corto plazo versus, 321-322, 327-330 Costumbres, 9
gráfica, 316-319 Cournot, Auguste, 426
optimización, 311-316 Crecimiento de la población, 268-269
relación con las estructuras industriales, 319-321, 374 Credenciales de la educación de élite, 173

18_NAME INDEX.indd 599 3/7/09 12:41:00 PM


600 ÍNDICE TEmáTICo

Cuentas por cobrar, ventas de, 421 Curvas de ingreso total


Cuestiones normativas, 19 maximización de la ganancia a corto plazo y, 339-343
Cuestiones positivas, 19 para monopolios, 377-379
Cuotas Curvas de oferta
con controles de precio, 35 ajustes a largo plazo, 351-353
escala móvil, 392 ausente en monopolios, 388
sistemas de dos partes, 146-147 características básicas, 28-29
Curva de demanda compensada por ingreso, 135-138 condiciones competitivas a corto plazo, 343-344
Curva de demanda con elasticidad constante, 133-134 desplazamiento versus movimiento a lo largo de la, 40
Curva de demanda con gasto constante, 134 efectos del costo a largo plazo, 356-359
Curva de demanda residual, 427 elasticidad del precio y, 360-361
Curva de disponibilidad a pagar agregada, 569 factores que la afectan, 39
Curva ingreso consumo, 98-99, 141 forma algebraica, 42
Curvas CPL en forma de U, 356-357, 364-365 impacto de los impuestos, 47-50
Curvas de costo promedio a largo plazo (CPL) Curvas de oferta de la industria
creación de, 318 ajustes a largo plazo, 351-353
en mercados competitivos, 320, 321, 351-352, 356-359, condiciones competitivas a corto plazo, 343-344
364-365 efectos de los costos a largo plazo, 356-359
en mercados de monopolio, 319-320, 374 elasticidad del precio y, 360-361
en relación con las curvas a corto plazo, 321, 322, Curvas de oferta de mercado para el trabajo, 470-472
327-330 Curvas de oferta de trabajo con pendiente negativa,
impacto sobre las estructuras de mercado, 319-321 466, 467, 469
para competidores monopolistas, 438, 439 Curvas de producto total, 269-272
que favorecen mercados altamente concentrados, 320 Curvas precio-consumo, 96-97
Curvas de demanda; véase también Curvas de demanda
del mercado
agregada, 109-111 D
características básicas, 27 Darwin, Charles, 225, 336
compensadas por el ingreso, 135-138 Davies, N., 169n
con competencia monopolística, 436, 437 Dawkins, Richard, 169n, 173n
de consumidores individuales, 97, 98 Decisiones difíciles, 252-254
desplazamiento versus movimiento a lo largo de, 40 Declaración de los derechos civiles de 1964, 480
elasticidad del precio y, 112-114 Deducibles, 192
factores que la afectan, 38 Déficits en el presupuesto federal, 153
forma algebraica, 42 Demanda; véase también Elasticidad precio de la demanda
impacto de los impuestos, 47-50 cambios en, 40
ingreso marginal sobre, 381-382, 383, 384, 394 de capital real, 506-507, 509
medición del excedente del consumidor con, 144-146 determinantes principales, 37-38, 96-100
parte inelástica, 385-386 efectos ingreso y sustitución sobre, 100-108, 120-125
residual, 427 elasticidad constante, 133-134
Curvas de demanda del mercado; véase también Curvas elasticidad del precio, 111-120
de demanda elasticidades precio cruzados, 125-126
bajo competencia perfecta, 344-345 individual versus del mercado, 109-111
con competencia monopolística, 436, 437 Demanda agregada; véase Demanda del mercado; Curva de
con duopolios de Cournot, 426, 427-428, 429 demanda del mercado
con monopolios, 373, 388 Demanda de café, 414
en el modelo de Bertrand, 429-430 Demanda de elasticidad unitaria, 112
en el modelo de Stackelberg, 431 Demanda de trabajo
para trabajo, 462-463 a corto plazo, 460-461
Curvas de demanda individual, 97, 98, 109-111; véase a largo plazo, 462
también Curvas de demanda bajo competencia imperfecta, 463-464
Curvas de demanda perfectamente elásticas, 113-114 de monopsonios, 472-474
Curvas de demanda perfectamente inelásticas, 114 en mercados competitivos, 460-463
Curvas de Engel, 99, 100, 120-123 Demanda del mercado
Curvas de indiferencia determinación, 109-111
a partir de las funciones de utilidad, 84-89 efectos del ingreso sobre, 120-124
canastas de bienes más asequibles sobre las, 69-73 Demanda elástica, 111, 112
con conducta altruista, 213-214 Demanda inelástica, 111, 112
formas algebraicas, 89-93 Demsetz, Harold, 402
intertemporal, 157-160 Departamento automotriz, 400
para la oferta de trabajo, 464-465 Depreciación, 175-176, 516
para la seguridad, 499 Derechos de propiedad, 542-550
preocupación por la equidad y, 230-231 Derechos internacionales a la propiedad, 549-550
principios básicos, 65-67 Derechos sobre el espacio aéreo, 535-536, 543-544
tasa marginal de sustitución en, 67-69 Desabastecimiento, 30

18_NAME INDEX.indd 600 3/7/09 12:41:00 PM


ÍNDICE TEmáTICo 601

Desabastecimiento de gasolina, 25 DuPont Corporation, 372-372


Descuento a estudiantes, 392-393 Duración de la semana de trabajo, 468-469, 470, 554
Descuentos, 140-142, 396
cantidad, 395 E
en el modelo de obstáculo, 396-397, 405-406 Economías controladas, 354
honorarios de escala móvil, 392 Economías de escala
Descuentos en los cines, 371,392-393 en el revelado de fotografías, 359, 360
Deseconomías pecuniarias 358-359 información como fuente de, 376-377
Desigualdad del ingreso; véase Distribución del ingreso orígenes del monopolio debido a las, 374, 376
Despegues y aterrizajes, 544 Economías de red, 375, 376
Destrucción mutuamente asegurada, 422-424 Economías pecuniarias, 359, 374
Deuel, Corey, 6 Economista, demanda para, 470-472
Diamantes, 373 Ecuaciones
Diamantes sintéticos, 373 costo del factor marginal, 472
Diener, E., 243n costo fijo promedio, 303
Diferencial salarial entre blancos y negros, 480 costo total, 299
Diferencial salarial entre hombres y mujeres, 480 costo total promedio, 303
DiFranco, Ani, 22 costo variable, 299
Dilema del prisionero costo variable promedio, 303
con varios jugadores, 420 curva de demanda con elasticidad constante, 133
elementos de, 219-220, 414-418 demanda del mercado, 110
Dilema del prisionero con múltiples jugadores, 420-422 elasticidad del ingreso de la demanda, 122, 123
Discriminación elasticidad del precio cruzado de la demanda, 125
empleo, 480-485 elasticidad del precio de la demanda, 112, 135, 382
estadística, 191, 482-485 elasticidad del precio de la oferta, 360
precio, 389-397, 403-406 función de producción, 265, 275
Discriminación de género, 480, 481 función de producción Cobb–Douglas, 291
Discriminación de los compañeros de trabajo, 481 función de producción Leontief, 292
Discriminación del cliente, 480, 481 función de reacción, 428
Discriminación del empleador, 481-482 función de utilidad, 87
Discriminación del empleo, 480-485 gasto total, 116
Discriminación del precio ingreso marginal, 379, 381
en mercados diferentes, 390-393 margen de maximización de la ganancia de los
en método del obstáculo, 395-397, 403-406 monopolios, 386
para cantidades desiguales, 395 método de los multiplicadores de Lagrange, 90-91
por monopolistas perfectamente discriminantes, oferta y demanda, 42
393-395 participación de compradores y vendedores en
visión general, 389 los impuestos, 48
Discriminación del precio de primer grado, 393 precio del bono perpetuo, 511
Discriminación del precio de segundo grado, 395 productividad marginal del trabajo, 463-464
Discriminación del precio de tercer grado, 391-392 producto marginal, 269
Discriminación estadística, 191, 482-485 producto promedio, 271
Discriminación no de mercado, 480 restricciones presupuestales, 58
Discriminación racial, 480, 481 tasa de interés real, 510
Discriminación sexual, 480, 481 tasa de renta anual del capital, 507
Disney, 147 tasa marginal de sustitución, 70
Disponibilidad heurística, 248 utilidad marginal, 85
Dispositivo de seguridad valor actual, 156, 508
compensación de diferenciales salariales para, 497-501 Edgeworth, Francis, 89
en el mercado de trabajo monopsónico, 474 Edificios más altos, 424-425
externalidades de, 555 Educación universitaria, 8-9; véase también Educación
ingreso relativo y, 501, 503 Educación
Distribución del ingreso aspectos relacionados, 553-554
crítica de Rawl, 583-584 bono escolar, 142-144
efectos sobre la demanda del mercado, 121-124 costos de oportunidad, 8-9
equidad y eficiencia en, 584-586 políticas de matriculación, 139
métodos de redistribución, 586-591 Efecto certeza, 207-208
principales factores en, 582-583 Efecto del refrigerador de agua chapeado en oro, 401
Disuasión nuclear, 422-424 Efecto del subsidio cruzado, 401-402
Disyuntiva de la privacidad, 575-576 Efecto halo, 254
Dixit, Avinash, 445n Efecto ingreso
Donaciones, 572-573 de cambios en la tasa salarial, 467-469
Duelo, 426 sensibilidad del, 106-107
Dulce, gusto por, 214-215 sobre la demanda del mercado, 120-125
Duopolios, 426 sobre los individuos, 100-105

18_NAME INDEX.indd 601 3/7/09 12:41:01 PM


602 ÍNDICE TEmáTICo

Efecto sustitución Estructuras internas del salario, 485-488


de cambios en la tasa salarial, 467-469 Estructuras prefabricadas, 279-280
sensibilidad de, 106-107 Estructuras prefabricadas para techos, 279-280
sobre los individuos, 100-105 Evolución, 214-215, 218, 336
Efecto total del aumento de los precios, 101 Exahustividad del ordenamiento de las preferencias, 64
Efectos de regresión, 249-250 Excedente agregado del productor, 348,349
Efectos superestrella, 489 Excedente del consumidor
Eficiencia bienestar general y, 147-153
de los impuestos a la contaminación, 556-557, 559 captura del monopolio, 393-394, 398
en la asignación, 346-347 con fijación de precio en dos partes, 146-147
en la distribución del ingreso, 584-586 excedente del productor más, 349-350
perdida con los monopolios, 397-398 medición, 144-146
Ehrenberg, Ronald, 477n Excedente del productor, 347-350, 517
Ejemplo del administrador eficiente, 355-356 Excedente económico, 548
Elasticidad ingreso de la demanda, 121-125 Excedentes; véase también Excedente del consumidor
Elasticidad precio cruzado de la demanda, 125-126, consumidor, 144
372-373 definición, 30
Elasticidad precio de la demanda, 125 económicos, 548
algunos ejemplos, 115, 153-155 productor, 347-350, 517
determinantes de, 118-120 productos agrícolas, 36
gasto total versus, 115-118 Excel, software, 451
gráfica de, 112-114 Exceso de demanda, 30, 32-33
independencia de las unidades, 114-115 Exceso de oferta, 30
método del cociente de los segmentos, 135 Expectativas, 38, 39
para monopolios, 372, 378, 382 Explotación, 475
visión general, 111-112 Expresiones faciales, 225-226
Elogio y culpa, 250 Externalidades
Elzinga, K., 115n de la protección a la libertad de expresión, 550
Emociones, 225-226 del fumar, 550-552
Empleados de confianza, 172-173 derechos de propiedad y, 542-550
Empleo para adolescentes, 477 gravar, 555-560
Empleo público, 588-589 posicionales, 552-555
Empresa refresquera Adirondack, 373 positivas, 552
Empresas conjuntas, 220-221 principios más importantes, 536-543
Energía solar, 527-531 Externalidades de posición, 552-555, 559-560
Entrar sin autorización en propiedad ajena, 544-545 Externalidades negativas; véase Externalidades
Entrevistas de empleo, 174-175, 555 Externalidades positivas, 552
Entrevistas de trabajo, 174-175, 555
Equidad
basada en motivos desinteresados, 227, 229-232 F
en la distribución del ingreso, 583-586 Factores de producción, 39
objeción a los monopolios basada en, 398, 405 Falacia mal-resultado-implica-mala-decisión, 192, 208
Equilibrio competitivo a corto plazo, 344-347 Famila Ploof, 545
Equilibrio competitivo a largo plazo, 350-354 Farrell, Joseph, 375n
Equilibrio de Chamberlin a corto plazo, 437 Farrelly, m. C., 115n
Equilibrio de Chamberlin a largo plazo, 438 Farris, Paul L., 115n
Equilibrio de Nash, 418-419 Fijación de precios de la hora pico, 518-519
Errores afectivos de pronóstico, 243-245 Fisher, Irving, 89
Escala mínima eficiente, 320 Flujos, 57
Escasez, 3-4 Fondos prestables, 509, 510
Espíritu empresarial, 264 Ford motor Company, 22, 488
Esquema tarifario por bloques descendientes, 395 Formas para mantener el compromiso, 220, 255
Estampillas para alimentos, 73-75 Formato beta, 375
Estigma del recién llegado, 176 Formato VHS, 375
Estimaciones insesgadas, 204 Fortune, revista periódica, 505
Estimaciones sesgadas, 204, 247-251 Frank, Robert, 245n, 449n, 485n, 488n
Estrategia de disuasión de entrada, 425, 426 Franquicias, 375-376, 582
Estrategia maximin, 419 Freeman, Richard B., 479n
Estrategia ojo por ojo, 420-422 Fricción, 338
Estrategias dominantes Friedman, milton, 6, 160, 161, 440, 587
definición, 415 Fruit loops, 440
en el dilema del prisionero, 415, 416, 417 Fruit-’n-Fiber, 440
en el método maximin, 419 Fumar, 550-552
equilibrio de Nash, 418-419 Función asimétrica de valor, 239-241, 247
Estructura de costo de la industria, 319-321 Función de asignación del precio, 37

18_NAME INDEX.indd 602 3/7/09 12:41:01 PM


ÍNDICE TEmáTICo 603

Función de producción Cobb-Douglas, 291-292, 294 Hiperplanos, 60


Función de producción de proporciones fijas, 292-293 Hipótesis de ingreso permanente, 160-161
Función de producción Leontief, 292-293 Hipótesis de los mercados eficientes, 512-514
Función racionadora del precio, 37 Hipótesis del ciclo de vida, 160-161
Funciones de producción Hirschman, Albert o., 479n
características a corto plazo, 266-275 Hockey, 553
características a largo plazo, 275-278 Homo economicus, 17
Cobb-Douglas, 291-292 Honda, 152
conceptos de costo básico, 298-299 Honorarios de escala móvil, 392
curvas de costo y, 300-302 Hoover, Herbert, 459
elementos de, 264-266 Horas extras, 554
Leontief, 292-293 Hotelling, Harold, 449
rendimientos a escala en, 278-281, 293-294 Houthakker, H. S., 115n, 120n, 125n
Funciones de reacción, 428 Hutchens, Robert, 245n
Funciones de utilidad
cardinales versus ordinales, 87-89
en el modelo de utilidad esperada, 180-187 I
formas algebraicas, 89-93 Impuestos
principios básicos, 83-87 a pagar por bienes públicos, 571-572, 581
Funciones de valor, 239-241, 247 aplicaciones de la elección racional, 140-142
Fungibilidad de la riqueza, 239 consumo, 560
Fusiones, 26-27, 403 curvas de demanda compensada con ingreso, 135-137
impacto sobre los mercados de capital, 516-517
impactos de la oferta y la demanda, 47-50
G negativos, 587-588, 590
Galbraith, John Kenneth, 354n, 450, 451 sobre bebida alcohólica, 154
Ganancia contable, 334-335 sobre externalidades negativas, 555-560
Ganancia material, 232-233 sobre externalidades posicionales, 559-560
Ganancia normal, 334,345 sobre las ganancias de un monopolio, 405-406
Ganancias, definición, 334 sobre negocios, 364-365
Gant, 436 Impuestos a la contaminación, 556-559
Garantía, 174 Impuestos a la gasolina, 140-142
Garantías, 174 Impuestos a las bebidas alcohólicas, 154
Garantías liberales, 174 Impuestos a las ventas, 50
Gasto total, 115-118 Impuestos negativos al ingreso (INI), 587-588, 590
Genentech, 513 Impuestos para las ganancias de las inversiones, 516-517
General motors, 447, 486, 488, 511 Incentivos de trabajo, 586-587, 588
Gerstein, Dean, 154n Incertidumbre
Gilovich, Thomas, 243, 250n elección racional bajo, 179-187
Grabowski, Henry, 374n respuestas muestrales a, 245-247
Gráficas seguro contra, 187-192
costo y beneficio marginal, 14-15 Índice de precios al consumidor (IPC), 150-153
elasticidad precio de la demanda, 112-114 Industria aérea
uso en el análisis de la oferta y la demanda, 26-29 costo de planeación (de horarios), 446
Gramlich, Edward m., 477n exceso de demanda, 32-33
Granjas corporativas, 362, 363 externalidades de, 535-536, 543-544
Granjas familiares, 362-364 precio de hora pico, 519
Grape-Nuts, 440 precio obstáculo en, 396,397, 448-449
Greenspan, Alan, 459, 460 rendimientos a escala en, 279
Grupos de industrias, 435-436 Industria automotriz
Guardarropas, 178-179, 555 cambios de calidad, 152-153
Guerra de trinchera, 420-421 precios de los automóviles usados versus precio de los
Guerra fría, 422-424 automóviles nuevos, 175-176
Gustos, 38, 228-233 variedad de productos, 446, 447-448
Guth, Werner, 229, 231 Industria textil, 479
Industrias con costos decrecientes, 359
Industrias de costo creciente, 359
H Industrias propiedad del estado, 399-400
Halvorsen, R., 125n Industrias propiedad del gobierno, 399-400
Hamermesh, Daniel, 477n Ineficiencia X, 399
Herrnstein Richard, 287, 289 Información; véase también Toma de decisiones; Teoría
Hertz, 9, 512 de la elección racional
Heurísticas, 247-251 difícil de falsificar, 171-173
Hicks, John, 89 efecto sobre los precios de las acciones, 513
Hinske, Erik, 250 en la construcción de una relación, 176-177

18_NAME INDEX.indd 603 3/7/09 12:41:01 PM


604 ÍNDICE TEmáTICo

falta de, en los trabajadores, 501 Leibenstein, Harvey, 399


incompleta, 238-239 Leontief, Wassily, 292
para reducir la incertidumbre, 185-186 Lerner, Abba, 586
percepciones de la, 251-252 Levitan, Sar, 477n
perfecta, 338 Ley Clayton, 403
principio de apertura total, 173-176 Ley de estándares de trabajo justos, 476, 554
relación de enemistad e, 170-171 Ley de la demanda, 28
Ingreso a lo largo de toda la vida, valor presente, 157 Ley de la oferta, 28
Ingreso marginal, 340, 379-384, 394 Ley de los grandes números, 188, 208
Ingreso relativo, 501-503 Ley de los rendimientos decrecientes
Ingresos como determinantes de la demanda, 37 para la demanda de trabajo, 464
Ingresos promedio, 341-342, 386-387 para la producción de alimentos, 268-269
Innovación, impacto de los monopolios sobre la, 406-407 rendimientos decrecientes a escala versus, 280
Innovaciones ahorradoras de costos, 365-366 Ley de un solo precio, 371
Innovaciones de la bombilla eléctrica, 406-407 Ley de Weber-Fechner, 251-252
Insumo variable, 266-275 Ley Sherman, 403
Insumos, control exclusivo de, 373 Leyes antimonopolio, 403
Insumos fijos, 266 Leyes antitabaco, 550
Intel, 376 Leyes de libertad de expresión, 550
Interés personal Leyes de urbanismo de San Francisco, 546
como estándar de racionalidad, 211, 212 Leyes de zonificación (urbanismo), 546
efecto de la mano invisible, 15-16, 353-354 Libre entrada y salida, 337-338
en el dilema del prisionero, 415 Licencias, 375-376
limitaciones de, 17, 219-226 Licencias gubernamentales, 375-376
Interpretación espacial de la competencia monopolística, Liga Nacional de Fútbol, 290, 553
440-449 Liga Nacional de Hockey, 553
Interpretación horizontal de la curva de la demanda, 27 Limitaciones cognitivas, 237
Interpretación horizontal de la curva de la oferta, 29 Límites de altura, 545-546
Interpretación vertical de la curva de demanda, 28 Límites para la altura para las construcciones, 545-546
Interpretación vertical de la curva de oferta, 29 Líneas de isocostos
Investigación química, 487, 488 con mapas de isocuantas, 312-313, 327-330
Isocuantas, 275 creación, 311-312
Loewenstein, George, 162, 243n, 467n
Lotus, 451
J Louisville Slugger, 264
Jazz, software, 451 Lujos, 122
Johnson, Leland, 400 Lutz, Wolfgang, 268n
Jolt, 72
Juego del ultimátum, 229-231
Juegos secuenciales, 422-426 M
Jugada de tenis, 287-289 macroeconomía, 19-20
Jugadas de fútbol, 290 maldición del atleta, 250
Jugadores de billar, 6 maldición del ganador, 203-208
Jugadores de tenis, 488-489 malthus, Thomas, 268
Juicios sobre el carácter, 226-227 manipulación de agenda 577, 579
Jureen, L., 125n manove, michael, 206
mantenimiento de los precios de productos agrícolas,
36, 41, 362-364
K mantenimiento de precios, 36, 41, 362-364
Kahneman, Daniel, 55n, 207, 208, 239-241, 243n, 245, 247- mantenimiento y embellecimiento del hogar, 574
251 mapas de indiferencia
Katz, m. L., 375n a partir de las funciones de utilidad, 84-89
Klein, Benjamin, 172n comparación de preferencias con, 69
Kmart, 129 definición, 66
Krebs, John, 169n, 173n formas algebraicas, 83-93
Kreps, David, 422n, 432n intertemporal, 157-160
Kreuger, Alan, 476n mapas de isocuantas, comparación, 276
Kurihara, K., 160n para la oferta de trabajo, 464-465
mapas de isocuantas
función de producción Cobb-Douglas, 292
L funciones básicas, 276
Lanzamientos de una moneda, 179-180 líneas de isocostos con, 312-313, 327-330
Largo plazo, definición, 266 para sustitutos y complementos perfectos, 277-278
Layard, Richard, 559 rendimiento a escala sobre, 280, 281
Leffler, Keith, 172n técnica geométrica, 290-291

18_NAME INDEX.indd 604 3/7/09 12:41:02 PM


ÍNDICE TEmáTICo 605

máquina del día del juicio final, 423 modelo de Bertrand, 429-430, 433
marriot, 535 modelo de Chamberlin, 435-440
marshall, Alfred, 60 modelo de Cournot, 426-429, 432, 433
marston, Stephen, 475 modelo de elección atemporal, 155
marx, Groucho, 176 modelo de elección intertemporal, 155-162
Masterpiece Theater, 573, 574 modelo de Stackelberg, 430-433
matriz de pagos, 415 modelo de Tiebout, 581
matthews, Dave 22 modelo de utilidad esperada de von Neumann-morgenstern,
maximización de la ganancia 180-187
ajustes a largo plazo, 350-353 modelo del obstáculo, 395-397, 403-406, 449
con equilibrio competitivo a corto plazo, 344-347 modigliani, Franco, 160
condición a corto plazo para, 339-343 monopolios
para monopolios, 377-388 ajustes a largo plazo, 388-389, 434-435
por monopsonios, 473, 474 ausencia de curva de oferta para, 388
supuestos básicos, 334-337 características básicas, 372-373
mazur, James, 287, 289 demanda de trabajo para, 464
mcConnell, Campbell, 304 discriminación de precios por, 389-397
mcDonald´s, 375 natural, 320, 374, 398-407
mcGinnis, Joe, 171 orígenes de, 373-377
mcGraw-Hill, 304, 517-518 pérdida de eficiencia con, 397-398
medias Rojas de Boston, 199 posesión de acciones en, 514
medicamentos de patente con receta, 374 monopolios naturales
medoff, James, 479n enfoques políticos a, 398-407
membresía a un club, 574-576 estrategias para tomar el mercado, 434-435
memoria, 248 orígenes de, 320, 374
mercado bursátil, 510-514 monopolistas discriminadores perfectos, 393-395
mercado de bonos, 510-514 monopsonios, 472-478
mercados altamente concentrados, 320 montaña de producción, 290-291
mercados de trabajo, 480-485, 488-489 moore, mark, 154n
mercados en los que el que gana se lleva todo, 488-489 morgenstern, oskar, 180, 415
mercados libres, pobreza y, 32-36 movilidad del trabajo, 475
mercados perfectamente competitivos movimiento estacional de los precios, 40, 41
ajustes a largo plazo, 350-353, 356-359, 360 multiplicadores de Lagrange, 90-91, 330-331
condiciones para, 337-338 museo de Arte moderno, 535
demanda de capital de la industria en, 506-507 Myasthenia gravis, 3
demanda de trabajo, 460-463
elasticidad de la demanda en, 360-361
equilibrio a corto plazo en, 344-347 N
excedentes del productor en, 347-350 Nash, John F., 418n
insumos extraordinarios en, 354-356 Naturalismo económico, 17-19
la mano invisible en, 15-16, 353-354 Necesidades, 122
limitaciones al aplicar el modelo de, 361-366 Negociación, 536-543
maximización de la ganancia a corto plazo en, 339-343 Nepal, 297-298, 314
oferta a corto plazo en, 343-344 Nestlé, 26-27
versus competencia de Chamberlin, 439 Netscape, 425n
mercados, definición de, 26-27 Neutralidad ante el riesgo, 183, 184, 187
método de los multiplicadores de Lagrange, 90-91 New York State Public Service Commision, 567, 585
método del cociente de los segmentos, 135 New York Times, The, 409
método punto pendiente, 112, 113 Nichols, Albert, 555n
métodos algebraicos para curvas de indiferencia, 89-93 Nisbet, Charles T., 115n
metropolitan Atlanta Rapid Transit Authority Niskanen, William, 400n
(metro de Atlanta), 140, 153-154 Nivel de ingreso meta, 466-467
microeconomía, 19-20 Niveles de habilidad, 177-179
microprocesador, 376 No pagadores, exclusión de, 573-574
microsoft, 375, 425n, 451 Nordhauser, Fred, 115n
milgrom, Paul, 422n Norma del objetivo presente, 211-212, 213, 214
minas de diamantes deBeers, 373, 559 Nub City, 190-191
mincer, Jacob, 477n Número de proveedores, 39
mineros, 501
minimización de costos
método gráfico, 311-313 O
método matemático, 330-331 oakland Scavenger Company, 402
mitchell, Bridger, 115n objeción de eficiencia a los monopolios naturales, 398,
modelo competitivo; véase mercados perfectamente 403-405
competitivos obsolencia tecnológica, 507

18_NAME INDEX.indd 605 3/7/09 12:41:02 PM


606 ÍNDICE TEmáTICo

ocio Porsche, 243-244


como bien Giffen, 469 Posesión de un automóvil, 445
efectos ingreso y sustitución sobre el, 467-468 Precio de aceptación, 202-203
trabajo versus, 464 Precio de los insumos, 340
oferta Precio de reserva
cambios en, 40-41 definición, 5
determinantes principales, 39 en la curva de demanda, 27-28
elasticidad del precio, 360-361 para seguro, 189-190
oferta a corto plazo, 343-344 Precio elasticidad de la oferta, 360-361
oferta de trabajo Precio en dos partes, 146-147
curva de oferta del mercado, 470-472 Precio máximo, 34; véase también Control de precios
efecto de las opciones limitadas, 469-470 Precio mínimo, 36; véase también Control de precios
principios básicos, 464-469 Precios
ofertas unilaterales, 227, 230-231 equilibrio, 29-30
oligopolios funciones de asignación y de racionamiento, 37
con rendimientos de escala creciente, 433-435 impactos de los impuestos,47-50
incentivos de publicidad con, 450-452 método de la hora pico, 518-519
modelos de, 426-433 sistemas de dos partes, 146-147
obstáculos a la colusión para, 414-416 Precios de los insumos, 316-319, 358-359
onassis, Aristóteles, 3 Precios del trabajo, 314
ordenamiento de las preferencias, 63-65 Precios reales, 27
oum, Tae, H., 115n Predicción de una conducta, 6
Preferencia negativa en el tiempo, 158, 162
Preferencia neutral en el tiempo, 158
P Preferencia positiva en el tiempo, 158, 159
Pago de la prima, 468 Preferencia única, 578
Pagos en efectivo versus transferencia en especie, 73-76 Preferencias; véase también Curvas de indiferencia;
Papas, 103 Teoría de la elección racional
Pareto, Vilfredo, 89 en la teoría de la elección racional, 56, 63-69
Parfit, Derek, 211n estratégicas, 214-218
Park, Rolla Edward, 115n temporales, 158, 161-162
Parques de diversiones, 147 únicas, 578
Parrilladas al aire libre, 123-124 Préstamos del exterior, 509
Participación del vendedor en el impuesto, 48, 50 Prima de riesgo, 511
Patentes, 374, 398 Primera Guerra mundial, 420-421
Patentes de medicamentos, 374 Primeras impresiones, 178
Patrocinio, 573 Principio de apertura total, 173-176
Patrones de consumo en un pueblo pequeño, 178 Principio de los “lemons”, 175-176
Pendiente versus elasticidad, 114-115 Principio del difícil-de-falsificar, 171-173
Pepsi, 375, 440 Probabilidad, valor esperado y, 179-180
Percepciones, 251-252 Problema de disuasión, 227
Pérdida irrecuperable de eficiencia, 397, 398 Problema de negociación, 227
Pérdidas importantes, seguro contra, 192 Problema del aprovechado, 573-574
Pérdidas menores, asegurarse contra, 192 “Problema del costo social, el”, 536
Perfiles de consumo creciente, 244-245 Problema del fraude, 220-226
Perrier Corporation, 373 Problemas de compromiso, 219-220
Personas adversas al riesgo, 181, 182 Problemas de maximización restringida, 90
Pigou, A. C., 555 Procter & Gamble, 505
Planeación central, 354 Producción
Planos multidimensionales, 60 asignación entre procesos, 308-309
Plantas de energía nuclear, 542-543 características a corto plazo, 266-275
Poblaciones creadas por una empresa, 472 características a largo plazo, 275-278
Pobreza definición, 263
descuentos profesionales para, 392 distinción entre marginal y promedio, 287-290
efectivo versus estampillas para alimentos para, 73-75 movilidad en mercados perfectamente competitivos,
impacto sobre las elecciones de bienes públicos, 580 337-338
mercados libres y, 32-36 rendimientos a escala en, 278-281, 293-294
métodos de redistribución del ingreso para, 586-591 Producción a corto plazo, 266-275
Polaroid, 373 Producción a largo plazo, 275-278
Política laissez-faire frente a los monopolios, 403-406 Producción de grava, 297-298, 314
Políticas de matriculación, 139 Productividad
Políticas públicas cálculo para el trabajo, 271
aplicaciones de la elección racional, 140-144 del trabajo, 271, 482-485
hacia los monopolios naturales, 398-407 impacto de los sindicatos sobre, 479-480
impacto sobre los mercados de capital, 516-517 pertenencia al grupo y, 482-485

18_NAME INDEX.indd 606 3/7/09 12:41:03 PM


ÍNDICE TEmáTICo 607

Producto del ingreso marginal del capital, 506 Read, Herbert, 421n
Producto del ingreso marginal del trabajo, 463-464 Reagan, Ronald, 51
Producto marginal Reclamaciones fraudulentas de seguros, 190-191
cálculo del, 269-270 Recursos colectivos, 546-550
del capital, 506 Recursos naturales
del trabajo en mercados competitivos, 460-461 dilema de los bienes colectivos, 546-550
en relación al producto total y al producto promedio, no renovables versus renovables, 519-521, 525-531
272, 287-290 Recursos no renovables
en relación con el costo marginal, 310-311 el dilema de los bienes colectivos, 546-550
en relación con la tasa marginal de sustitución técnica, fijación del precio, 519-521
277 transición de los, 527-531
usando asignación de recursos, 272-275 Recursos renovables, 525-527
Producto promedio Red Hot Chili Peppers, 63
asignación de recursos usando, 272-275 Redistribución del ingreso, 586-591
cálculo del, 271 Reforma a la asistencia social, 469
en relación con el producto total y el producto Regalos, 75-76
marginal, 272, 287-290 Regla de la oferta para monopolios, 388
en relación con los costos variables promedio, 310-311 Reglamentación de la tasa de rendimiento, 400-402
Productos agrícolas, 36, 41, 362-364 Reglamentación del servicio telefónico, 567-568, 585
Productos estandarizados, 337 Reglamentación estatal de los monopolios, 400-402
Productos intermedios, 266 Reglamento de seguridad minera, 501-503
Programa de tala de árboles, 525-527 Reglas de decisión, 4-5
Programas JoBS, 588-591 Regulación gubernamental
Pronóstico de la seguridad en el puesto de trabajo, 500, 501-502
errores afectivos en, 243-245 de los monopolios, 400-402
papel de la elasticidad del ingreso en, 124-125 de los servicios telefónicos, 567-568, 585
Propiedad compartida, 575-576 Relación duradera en los negocios, 421
Propiedad conjunta, 188 Rendimiento a escala
Propiedad de la elasticidad de la demanda de ser definición matemática, 293-294
independiente de las unidades, 114-115 en la curva de costos a largo plazo, 318-319
Propiedad de no exclusividad en el consumo, 568 monopolios naturales y, 374
Propiedad de no rivalidad en el consumo, 568 principios básicos, 278-281
Propiedad más-es-mejor, 64 Rendimientos a escala creciente
Propiedades a la orilla de un lago, 544-545 condiciones que favorecen, 278-279
Propiedades de equilibrio de la asistencia social, 31-32 curvas de costo a largo plazo con, 318-319
Propiedades en zona ribereña, 544-545 duopolios con, 433-435
Proporción del impuesto que pagan los compradores, Rendimientos constantes a escala, 279, 318
48, 49-50 Rendimientos decrecientes; véase también Ley de los
Proporciones, ver costos y beneficios como, 11-12 rendimientos decrecientes
Protección de la primera enmienda, 550 Rendimientos decrecientes a escala, 279, 280-281, 318, 319
Proyecto de irrigación, 333, 354-355 Renta económica, 517-518
Pruebas de mercado, 451 Representatividad heurística, 248-250
Publicidad Reservación de canchas de tenis, 146, 147, 237-238
como dilema del prisionero, 417-418 Restaurantes, 448
financiamiento de bienes públicos a través de, 573 Restricciones colectivas, 553
incentivos con competencia imperfecta, 450-452 Restricciones del presupuesto
sobre cigarros, 417-418 canastas de bienes más asequibles dentro de, 69-73
Punto de equilibrio, 345 compensación por descuentos, 141
Puntos de beneficio máximo global, 387 con bienes múltiples, 60
Puntos locales de ganancia máxima, 387 con cambios en el precio o en el ingreso, 58-60, 148
Puntos locales de ganancia mínima, 387 con y sin bonos escolares, 142-143
hacer inferencias a partir de, 62-63
intertemporal, 156-157
Q preferencias del trabajo individual y, 464-465
Químico, 487, 488 quebradas, 61-62, 468
visión general, 56-58
Restricciones presupuestales quebradas, 61-62, 468
R Revelado de fotografías, 359, 360
Racionalidad, 211-212, 238-239 Revelado de fotografías a color, 359, 360
Racionalidad limitada, 238-239 Revelado de fotografías en blanco y negro, 359, 360
Radiodifusión, 266 Revista Newsweek, 513
Ralph Lauren, 436 Riesgo compartido, 188-189
Ranqueo en la distribución del salario, 485-488 Riesgo moral, 190-191
Rapoport, Anatol, 420n Riesgo y recompensa, 583-584
Rawls, John, 583-584 Riqueza, 239

18_NAME INDEX.indd 607 3/7/09 12:41:03 PM


608 ÍNDICE TEmáTICo

Roberts, John, 422n Sindicatos de trabajadores; véase Sindicatos


Robinson, Joan, 435 Sindicatos
Rodriguez, Alex, 459-460 defensa del salario mínimo, 315-316
Ropa, 178-179, 555 impacto sobre el mercado laboral, 478-480
Rosen, Sherwin, 488n Sistema de productividad marginal de la distribución
Ruta de agotamiento de reservas, 528 del ingreso, 583-584
Rutas de expansión de los insumos, 316 Sistema de seguridad social, 554-555
Ruth, Babe, 459 Sistema operativo Windows, 375
Sistemas de mercado, beneficios de, 353-354
Situación óptima para un monopolista, 380
S Smith, Adam, 15-16, 278, 353, 450, 533
Sal, 106 Smith, Eric, 191n
Salario de aceptación, 200-201 Smith, Robert S., 477n, 499
Salarios Solow, Robert, 284
de monopsonios, 474, 475-476 Soluciones de esquina, 71-73
efectos de la igualación sobre, 471 Soluciones interiores, 73, 274
estructuras internas, 485-488 Spann, R. m., 400n
impacto sobre la oferta de trabajo, 465-467 Spence, A. michael, 445n
leyes del salario mínimo, 315-316, 476-478 Spitz, mark, 552
Salarios mínimos, 315-316, 476-478 Sport Illustrated, 250
Saloner, Garth, 375n Stackelberg, Heinrich von, 430
Samuelson, Paul, 284 Stein, Herbert, 268
Samuelson, William, 203 Stigler, George, 228, 439-440, 476n
Sanderson, Warren, 268n Stiglitz, Joseph, 445n
Sapos, 169, 173-174 Stouffer’s Corporation, 26-27
Satisfacción, 29 Subasta, 32-33, 203-208
Savas, E. S., 402n Subproducto de los bienes públicos, 573
Scheinkman, Jose, 432n Suma horizontal, 109
Schelling, Thomas, 219-220, 255 Suministro de petróleo, 519-521, 527-531
Scheussel, Wolfgang, 542 Summers, Lawrence, 476n
Schkade, David, 243n Sunbeam, 123
Schwartz, N., 243n Superficie aerodinámica, 365-366
Sclar, Elliott D., 400n, 402 Supuesto de la toma de una decisión racional, 6
Sears, 257, 435 Sustitutos
Secuencia corregida, 450 como determinantes de la demanda, 28,38
Secuencia tradicional, 450, 451 con competencia monopolística, 436
Segmentación del mercado, 389-397 elasticidad precio cruzado de la demanda de, 125,
Seguridad de la inversión, 512 372-373
Seguridad en el automóvil, 497 no considerados en el IPC, 151
Seguridad en el puesto de trabajo, 498-501, 555 perfectos, 72, 105, 277-278
Seguridad ocupacional y ley de salud, 500 Sustitutos cercanos, 436; véase también Sustitutos
Seguro, 187-192 Sustitutos perfectos
Seguro contra choques, 192 efectos ingreso y sustitución, 105
Seguro de automóvil mapas de isocuantas para, 277-278
deducibles en, 192 tasa marginal de sustitución para, 72
diferenciales de tarifas, 191
principios de riesgo compartido, 188-189
selección adversa en, 190 T
Seguro de responsabilidad, 188-189 Tala de árboles, 525-527
Selección adversa, 190 Talento y recompensa, 583
Selección natural, 214-218, 336 Tareas trabaja por trabajar, 589
Sentimientos morales, 16, 17 Tasa de reducción del beneficio marginal, 586-587
Señalización, 171, 177-179 Tasa de renta anual del capital, 507
Sero, 436 Tasa marginal de preferencia cronológica, 158
Servicios comerciales de citas, 177 Tasa marginal de sustitución
Servicios públicos, contratación privada para, 402-403 comparación con la TmST, 276-277
Shapiro, Carl, 375n con la canasta de bienes más asequible, 70-73
Sharapova, maria, 18 para sustitutos perfectos, 72
Sherbov, Sergei, 268n principios básicos, 67-69
Shin, D. C., 245n Tasa marginal de sustitución técnica, 276-278
Show de Jerry Springer, el, 573, 574 Tasas de alquiler, 507
Shredded Wheat, 440 Tasas de fracaso, 251
Siedman, Laurence, 476n Tasas de interés
Simon, Herbert, 238 factores determinantes de las, 508-509
Simonson, Itamar, 253 real versus nominal, 510

18_NAME INDEX.indd 608 3/7/09 12:41:03 PM


ÍNDICE TEmáTICo 609

restricciones presupuestales intertemporales y, 160 Tommy Hilfiger, 436


tasas de renta versus, 507 Tormenta, derechos de propiedad durante, 545
Tasas de utilidad Torre Sears, la, 424-425
a la baja, 395 Toyota, 22, 243-244
método del precio-hora pico, 518-519 Trabajador i-ésimo, 466
regulación estatal, 400-402 Trabajo como insumo variable de producción, 266-268
Tasas del servicio eléctrico Tractor de semirremolque, 365-366
disminución, 395 Tradición mormona, 172-173
método de la hora pico, 518-519 Tragedia de los bienes colectivos, 546-550
regulación estatal, 400-402 Traidores y cooperadores, 220-226
Tasas marginales decrecientes de sustitución, 68-69 Transferencias en especie, 73-76
Taxistas, 467 Transferencias fijas, 469
Taylor, Lester, 115n, 120n, 125n Transitividad del ordenamiento de las preferencias, 64
Tecnología para la producción de alimentos, 268-269 Trasmisiones, 19
Televisión de paga, 573-574 Transmisiones automotrices, 19
Televisión por cable, 573-574, 582 Transmisiones manuales, 19
Tendencias de la población, 38 Tucker, A. W., 219
Tenis profesional, 488-489 Tullock, Gordon, 400n
Teorema de Coase, 539, 552 Tversky, Amos, 55n, 207, 208, 239-241, 245, 247-251, 253
Teorema del votante mediano, 577-578
Teoría de juegos
dilema del prisionero, 414-418, 420 U
equilibrio de Nash, 418-419 Ubicación, 440-445
estrategia maximin, 419 United Airlines, 397
estrategias ojo por ojo, 420-422 United Auto Workers, 315
para juegos secuenciales, 422-426 Universidad de Cornell, 139, 237
Teoría de la búsqueda, 199-203 Uso de esteroides, 553
Teoría de la elección racional; véase también Toma Uso de esteroides anabólicos, 553
de decisiones Utilidad cardinal, 87-89
aplicación de políticas, 140-144 Utilidad esperada, 180-187
aplicaciones de la realidad, 73-76 en la teoría de la búsqueda (de empleo), 200-201
asequibles e inasequibles, 58 maldición del ganador y, 207-208
canasta de bienes más asequible en, 69-73 principios básicos, 180-187, 245-246
compensación del bienestar bajo, 147-153 Utilidad marginal, 85, 91
consumo intertemporal, 155-160 Utilidad marginal decreciente de la riqueza, 181, 183
definición, 56 Utilidad ordinal, 87-89
elección intertemporal en, 155-162
ignorar los costos no recuperables bajo la, 238, 242
más asequible, 69-73 V
método de la función de utilidad, 83-87 Vakil, Firouz, 115n
preferencias en, 56, 63-69 Vales, 142-144.
respuestas predichas a la incertidumbre, 179-187 Valor actual neto, 508
restricciones de presupuesto en, 56-63 Valor del producto marginal
visión general, 56 del capital, 506
Teoría de la mano invisible, 15-17, 353-354 del trabajo, 461, 484-485
Teorías, 5-6 Valor equivalente de certeza, 187, 189-190
Teorías económicas, 5-6 Valor esperado de una apuesta
Texaco, 277-278 en la teoría de la búsqueda (de empleo), 200-201
Thaler, Richard, 6, 10n, 76n, 242-243, 252, 467n maldición del ganador y, 207-208
Ticketmaster, 21 principios básicos, 179-180, 181
Tiebout, Charles, 581 Valor nominal de los bonos, 510
Tiempo, 3, 57 Valor presente, 156-157, 508
Tiendas de abarrotes, 445 Valores, 19
Timidez, 176-177 Valores monetarios, basar las decisiones en, 4-5
Toma de decisiones; véase también Teoría de la Valores morales, 232-233, 583-584
elección racional Van Bedaf, Jos, 316
con información incompleta, 238-239 Van Boven, Leaf, 243
dificultades en la, 252-254 Van Heusen, 436
errores afectivos en, 243-245 Variedad de productos, 446-449
errores comunes, 6-14 Vendedor de autos, 487, 488
experimentos con elección bajo incertidumbre, 245-247 Vendedores de bienes raíces, 487, 488
heurísticas y sesgos, 247-251 Vendedores de hot dogs, 449-450
método costo-beneficio, 4-5 Venta de artículos con desperfectos, 396
percepciones y, 251-252 Venta de autos usados, 175-176
Tomadores del precio, 337 Venta de cuetes, 349-350

18_NAME INDEX.indd 609 3/7/09 12:41:04 PM


610 ÍNDICE TEmáTICo

Ventas de automóviles nuevos, 175-176 Waters, W. G., 11, 115n


Verhagen, Stefan, 316n Welch, Finis, 477n
Vigilancia, costo de, 223-225 Wetzel, Bruce m., 115n
Viking, 123-124 Willig, Robert, 145n
Vivienda, 107, 149-150 Wilson, Robert, 422n
Volkswagen, 448 Wintrobe, Ronald, 400n
Volvo, 176 Wise, michael Z., 542n
von Neumann, John, 180, 415 Wold, H., 125n
Votación mayoritaria sobre bienes públicos, 576-578, Wright Randall, 191n
579-581
Votación para elecciones presidenciales, 228
Votación sobre bienes públicos, 576-578, 579-581 Y
Yong, Jong Say, 115n

W
Wal-mart, 11-12 Z
Warsh, David, 203n Zeckhauser, Richard, 555n

18_NAME INDEX.indd 610 3/7/09 12:41:04 PM


18_NAME INDEX.indd 611 3/7/09 12:41:04 PM
18_NAME INDEX.indd 612 3/7/09 12:41:04 PM

También podría gustarte